Download as docx, pdf, or txt
Download as docx, pdf, or txt
You are on page 1of 657

III.

SEARCH AND SEIZURE


A. Source – Fourth Amendment, U.S. Federal Constitution
B. Scope of protection – natural and juridical persons; citizens and non-citizens
C. Reasonableness: touchstone of the validity of search and seizure

CASE: Social Justice Society v. Dangerous Drugs Board, G.R. No. 157870,
November 3, 2008

Ruling: 

Paragraphs (c) and (d) are constitutional while paragraphs (f) and (g) are
unconstitutional.

Sec. 36 (c) of RA 9165 - constitutional 

In the cases of Vernonia and Board of Education from US jurisprudence, 

1. schools and their administrators stand in loco parentis with respect to their
students; 
2. minor students have contextually fewer rights than an adult, and are subject
to the custody and supervision of their parents, guardians, and schools; 
3. schools, acting in loco parentis, have a duty to safeguard the health and well
- being of their students and may adopt such measures as may reasonably be
necessary to discharge such duty; and 
4. schools have the right to impose conditions on applicants for admission that
are fair, just, and non-discriminatory.
 
Schools and their administrators stand in loco parentis with respect to their
students and schools, acting in loco parentis, have a duty to safeguard the health
and well-being of their students and may adopt such measures as may reasonably
be necessary to discharge such duty; and schools have the right to impose
conditions on applicants for admission that are fair, just, and non-discriminatory. 

It is in this view that the provisions of RA 9165 requiring mandatory, random, and
suspicionless drug testing of students are constitutional. Indeed, it is within the
prerogative of educational institutions to require, as a condition for admission,
compliance with reasonable school rules and regulations and policies. To be sure,
the right to enroll is not absolute; it is subject to fair, reasonable, and equitable
requirements. 

The Court can take judicial notice of the proliferation of prohibited drugs in the
country that threatens the well - being of the people, particularly the youth and
school children who usually end up as victims. Accordingly, and until a more
effective method is conceptualized and put in motion, a random drug testing of
students in secondary and tertiary schools is not only acceptable but may even be
necessary if the safety and interest of the student population, doubtless a legitimate
concern of the government, are to be promoted and protected.
 
Sec. 36 (d) of RA 9165 - constitutional 
 
The mandatory but random drug test prescribed by Sec. 36 (d) of RA 9165 for
officers and employees of public and private offices is justifiable albeit not exactly
for the same reason as the students. SJS has failed to show how the mandatory,
random, and suspicionless drug testing under Sec. 36(c) and (d) of RA 9165
violates the right to privacy and constitutes unlawful and/or unconsented search
under Art. III, Secs. 1 and 2 of the Constitution.
 
The essence of privacy is the right to be left alone. Authorities are agreed though
that the right to privacy yields to certain paramount rights of the public and defers
to the state’s exercise of police power. As the warrantless clause of Sec. 2, Art III
of the Constitution is couched and as has been held, “reasonableness” is the
touchstone of the validity of a government search or intrusion. While every officer
and employee in a private establishment is under the law deemed forewarned that
he or she may be a possible subject of a drug test, nobody is really singled out in
advance for drug testing. The goal is to discourage drug use by not telling in
advance anyone when and who is to be tested. It is to be noted the very reason RA
9165 was enacted is to safeguard the well-being of the citizens from the deleterious
effects of dangerous drugs.
 
Sec. 36 (f) of RA 9165 - unconstitutional 
 
The operative concepts in the mandatory drug testing are “randomness” and
“suspicionless.” In the case of persons charged with a crime before the
prosecutor’s office, a mandatory drug testing can never be random or
suspicionless. The ideas of randomness and being suspicionless are antithetical to
their being made defendants in a criminal complaint. They are not randomly
picked; neither are they beyond suspicion. When persons suspected of committing
a crime are charged, they are singled out and are impleaded against their will. To
impose mandatory drug testing on the accused is a blatant attempt to harness a
medical test as a tool for criminal prosecution, contrary to the stated objectives of
RA 9165. Drug testing in this case would violate a persons’ right to privacy
guaranteed under Sec. 2, Art. III of the Constitution. Worse still, the accused
persons are veritably forced to incriminate themselves.
 
—-------------------------------------------------------------------------------------------------
---------------------------
In Pimentel’s petition, the court held section (g) unconstitutional. 
 
Unconstitutionality of Sec. 36 (g) of RA 9165 COMELEC cannot validly impose
qualifications on candidates for public office in addition to what the Constitution
prescribes; and Sec. 36 (g) of RA 9165, as sought to be implemented by the
assailed COMELEC resolution, effectively enlarges the qualification requirements
enumerated in Sec. 3, Art. VI of the Constitution. Said section unmistakably
requires a candidate for senator to be certified illegal-drug clean as a pre-condition
to the validity of his certificate for candidacy and a condition sine qua non to be
voted upon and be proclaimed as an elected official. The right of a citizen in the
democratic process of election should not be defeated by unwarranted impositions
of requirement not otherwise specified in the Constitution—the basic law to which
all laws must conform.

D. Meaning of “search”

CASES:
Katz v. U.S., 389 U.S. 347 (1967)

CERTIORARI TO THE UNITED STATES COURT OF APPEALS

FOR THE NINTH CIRCUIT

Syllabus

Petitioner was convicted under an indictment charging him with transmitting


wagering information by telephone across state lines in violation of 18 U.S.C. §
1084. Evidence of petitioner's end of the conversations, overheard by FBI agents
who had attached an electronic listening and recording device to the outside of the
telephone booth from which the calls were made, was introduced at the trial. The
Court of Appeals affirmed the conviction, finding that there was no Fourth
Amendment violation, since there was "no physical entrance into the area occupied
by" petitioner.

Held:

1. The Government's eavesdropping activities violated the privacy upon which


petitioner justifiably relied while using the telephone booth, and thus constituted a
"search and seizure" within the meaning of the Fourth Amendment. Pp. 389 U. S.
350-353.

(a) The Fourth Amendment governs not only the seizure of tangible items, but
extends as well to the recording of oral statements. Silverman v. United States, 365
U. S. 505, 365 U. S. 511. P. 389 U. S. 353.
(b) Because the Fourth Amendment protects people, rather than places, its reach
cannot turn on the presence or absence of a physical intrusion into any given
enclosure. The "trespass" doctrine of Olmstead v. United States, 277 U. S. 438,
and Goldman v. United States, 316 U. S. 129, is no longer controlling. Pp. 389 U.
S. 351, 389 U. S. 353.

2. Although the surveillance in this case may have been so narrowly circumscribed
that it could constitutionally have been authorized in advance, it was not in fact
conducted pursuant to the warrant procedure which is a constitutional precondition
of such electronic surveillance. Pp. 389 U. S. 354-359.

369 F.2d 130, reversed.

Page 389 U. S. 348

MR. JUSTICE STEWART delivered the opinion of the Court.

The petitioner was convicted in the District Court for the Southern District of
California under an eight-count indictment charging him with transmitting
wagering information by telephone from Los Angeles to Miami and Boston, in
violation of a federal statute. [Footnote 1] At trial, the Government was permitted,
over the petitioner's objection, to introduce evidence of the petitioner's end of
telephone conversations, overheard by FBI agents who had attached an electronic
listening and recording device to the outside of the public telephone booth from
which he had placed his calls. In affirming his conviction, the Court of Appeals
rejected the contention that the recordings had been obtained in violation of the
Fourth Amendment, because "[t]here was no physical entrance into the area
occupied by [the petitioner]." [Footnote 2] We granted certiorari in order to
consider the constitutional questions thus presented. [Footnote 3]

The petitioner has phrased those questions as follows:

"A. Whether a public telephone booth is a constitutionally protected area so that


evidence obtained by attaching an electronic listening recording device to the top
of such a booth is obtained in violation of the right to privacy of the user of the
booth. "

"B. Whether physical penetration of a constitutionally protected area is


necessary before a search and seizure can be said to be violative of the Fourth
Amendment to the United States Constitution."

We decline to adopt this formulation of the issues. In the first place, the correct
solution of Fourth Amendment problems is not necessarily promoted by
incantation of the phrase "constitutionally protected area." Secondly, the Fourth
Amendment cannot be translated into a general constitutional "right to privacy."
That Amendment protects individual privacy against certain kinds of governmental
intrusion, but its protections go further, and often have nothing to do with privacy
at all. [Footnote 4] Other provisions of the Constitution protect personal privacy
from other forms of governmental invasion. [Footnote 5] But the protection of a
person's general right to privacy -- his right to be let alone by other people --
is, like the protection of his property and of his very life, left largely to the law
of the individual States. [Footnote 7]

Because of the misleading way the issues have been formulated, the parties have
attached great significance to the characterization of the telephone booth from
which the petitioner placed his calls. The petitioner has strenuously argued that the
booth was a "constitutionally protected area." The Government has maintained
with equal vigor that it was not. [Footnote 8] But this effort to decide whether or
not a given "area," viewed in the abstract, is "constitutionally protected" deflects
attention from the problem presented by this case. [Footnote 9] For the Fourth
Amendment protects people, not places. What a person knowingly exposes to the
public, even in his own home or office, is not a subject of Fourth Amendment
protection.  But what he seeks to preserve as private, even in an area accessible to
the public, may be constitutionally protected.

See Rios v. United States, 364 U. S. 253; Ex parte Jackson, 96 U. S. 727, 96 U. S.


733.

The Government stresses the fact that the telephone booth from which the
petitioner made his calls was constructed partly of glass, so that he was as visible
after he entered it as he would have been if he had remained outside. But what he
sought to exclude when he entered the booth was not the intruding eye -- it was the
uninvited ear. He did not shed his right to do so simply because he made his calls
from a place where he might be seen. No less than an individual in a business
office, [Footnote 10] in a friend's apartment, [Footnote 11] or in a taxicab,
[Footnote 12] a person in a telephone booth may rely upon the protection of the
Fourth Amendment. One who occupies it, shuts the door behind him, and pays the
toll that permits him to place a call is surely entitled to assume that the words he
utters into the mouthpiece will not be broadcast to the world. To read the
Constitution more narrowly is to ignore the vital role that the public telephone has
come to play in private communication.

The Government contends, however, that the activities of its agents in this case
should not be tested by Fourth Amendment requirements, for the surveillance
technique they employed involved no physical penetration of the telephone booth
from which the petitioner placed his calls. It is true that the absence of such
penetration was at one time thought to foreclose further Fourth Amendment
inquiry, Olmstead v. United States, 277 U. S. 438, 277 U. S. 457, 277 U. S.
464, 277 U. S. 466; Goldman v. United States, 316 U. S. 129, 316 U. S. 134-136,
for that Amendment was thought to limit only searches and seizures of tangible
property. [Footnote 13] But "[t]he premise that property interests control the
right of the Government to search and seize has been discredited." Warden v.
Hayden, 387 U. S. 294, 387 U. S. 304. Thus, although a closely divided Court
supposed in Olmstead that surveillance without any trespass and without the
seizure of any material object fell outside the ambit of the Constitution, we have
since departed from the narrow view on which that decision rested. Indeed, we
have expressly held that the Fourth Amendment governs not only the seizure
of tangible items, but extends as well to the recording of oral statements,
overheard without any "technical trespass under . . . local property
law." Silverman v. United States, 365 U. S. 505, 365 U. S. 511. Once this much is
acknowledged, and once it is recognized that the Fourth Amendment protects
people -- and not simply "areas" -- against unreasonable searches and seizures, it
becomes clear that the reach of that Amendment cannot turn upon the presence or
absence of a physical intrusion into any given enclosure.

We conclude that the underpinnings of Olmstead and Goldman have been so


eroded by our subsequent decisions that the "trespass" doctrine there enunciated
can no longer be regarded as controlling. The Government's activities in
electronically listening to and recording the petitioner's words violated the privacy
upon which he justifiably relied while using the telephone booth, and thus
constituted a "search and seizure" within the meaning of the Fourth Amendment.
The fact that the electronic device employed to achieve that end did not happen to
penetrate the wall of the booth can have no constitutional significance.

The question remaining for decision, then, is whether the search and seizure
conducted in this case complied with constitutional standards. In that regard, the
Government's position is that its agents acted in an entirely defensible manner:
they did not begin their electronic surveillance until investigation of the petitioner's
activities had established a strong probability that he was using the telephone in
question to transmit gambling information to persons in other States, in violation of
federal law. Moreover, the surveillance was limited, both in scope and in duration,
to the specific purpose of establishing the contents of the petitioner's unlawful
telephonic communications. The agents confined their surveillance to the brief
periods during which he used the telephone booth, [Footnote 14] and they took
great care to overhear only the conversations of the petitioner himself. [Footnote
15]

Accepting this account of the Government's actions as accurate, it is clear that this
surveillance was so narrowly circumscribed that a duly authorized magistrate,
properly notified of the need for such investigation, specifically informed of the
basis on which it was to proceed, and clearly apprised of the precise intrusion it
would entail, could constitutionally have authorized, with appropriate safeguards,
the very limited search and seizure that the Government asserts, in fact, took
place. Only last Term we sustained the validity of such an authorization,
holding that, under sufficiently "precise and discriminate circumstances," a
federal court may empower government agents to employ a concealed
electronic device "for the narrow and particularized purpose of ascertaining
the truth of the . . . allegations" of a "detailed factual affidavit alleging the
commission of a specific criminal offense." Osborn v. United States, 385 U. S.
323, 385 U. S. 329-330. Discussing that holding, the Court in Berger v. New
York, 388 U. S. 41, said that "the order authorizing the use of the electronic
device" in Osborn "afforded similar protections to those . . . of conventional
warrants authorizing the seizure of tangible evidence." Through those
protections, "no greater invasion of privacy was permitted than was necessary
under the circumstances." Id. at 388 U. S. 57. [Footnote 16] Here, too, a similar
judicial order could have accommodated "the legitimate needs of law enforcement"
[Footnote 17] by authorizing the carefully limited use of electronic surveillance.

The Government urges that, because its agents relied upon the decisions
in Olmstead and Goldman, and because they did no more here than they might
properly have done with prior judicial sanction, we should retroactively validate
their conduct. That we cannot do. It is apparent that the agents in this case acted
with restraint. Yet the inescapable fact is that this restraint was imposed by the
agents themselves, not by a judicial officer. They were not required, before
commencing the search, to present their estimate of probable cause for detached
scrutiny by a neutral magistrate. They were not compelled, during the conduct of
the search itself, to observe precise limits established in advance by a specific court
order. Nor were they directed, after the search had been completed, to notify the
authorizing magistrate in detail of all that had been seized. In the absence of such
safeguards, this Court has never sustained a search upon the sole ground that
officers reasonably expected to find evidence of a particular crime and
voluntarily confined their activities to the least intrusive means consistent
with that end. Searches conducted without warrants have been held unlawful
"notwithstanding facts unquestionably showing probable cause," Agnello v.
United States, 269 U. S. 20, 269 U. S. 33, for the Constitution requires "that the
deliberate, impartial judgment of a judicial officer . . . be interposed between the
citizen and the police. . . ." Wong Sun v. United States, 371 U. S. 471, 371 U. S.
481-482. "Over and again, this Court has emphasized that the mandate of the
[Fourth] Amendment requires adherence to judicial processes," United States v.
Jeffers, 342 U. S. 48, 342 U. S. 51, and that searches conducted outside the judicial
process, without prior approval by judge or magistrate, are per se unreasonable
under the Fourth Amendment [Footnote 18] -- subject only to a few specifically
established and well delineated exceptions.

It is difficult to imagine how any of those exceptions could ever apply to the sort of
search and seizure involved in this case. Even electronic surveillance substantially
contemporaneous with an individual's arrest could hardly be deemed an "incident"
of that arrest.
Nor could the use of electronic surveillance without prior authorization be justified
on grounds of "hot pursuit." And, of course, the very nature of electronic
surveillance precludes its use pursuant to the suspect's consent. [Footnote 22]

The Government does not question these basic principles. Rather, it urges the
creation of a new exception to cover this case. [Footnote 23] It argues that
surveillance of a telephone booth should be exempted from the usual requirement
of advance authorization by a magistrate upon a showing of probable cause. We
cannot agree. Omission of such authorization "bypasses the safeguards provided by
an objective predetermination of probable cause, and substitutes instead the far less
reliable procedure of an after-the-event justification for the . . . search, too likely to
be subtly influenced by the familiar shortcomings of hindsight judgment."

Beck v. Ohio, 379 U. S. 89, 379 U. S. 96. And bypassing a neutral


predetermination of the scope of a search leaves individuals secure from
Fourth Amendment violations "only in the discretion of the police." Id. at 379
U. S. 97.

These considerations do not vanish when the search in question is transferred from
the setting of a home, an office, or a hotel room to that of a telephone booth.
Wherever a man may be, he is entitled to know that he will remain free from
unreasonable searches and seizures. The government agents here ignored "the
procedure of antecedent justification . . . that is central to the Fourth Amendment,"
[Footnote 24] a procedure that we hold to be a constitutional precondition of the
kind of electronic surveillance involved in this case. Because the surveillance here
failed to meet that condition, and because it led to the petitioner's conviction, the
judgment must be reversed.

It is so ordered.

MR. JUSTICE MARSHALL took no part in the consideration or decision of this


case.

18 U.S.C. § 1084. That statute provides in pertinent part:

"(a) Whoever being engaged in the business of betting or wagering knowingly uses
a wire communication facility for the transmission in interstate or foreign
commerce of bets or wagers or information assisting in the placing of bets or
wagers on any sporting event or contest, or for the transmission of a wire
communication which entitles the recipient to receive money or credit as a result of
bets or wagers, or for information assisting in the placing of bets or wagers, shall
be fined not more than $10,000 or imprisoned not more than two years, or both."

"(b) Nothing in this section shall be construed to prevent the transmission in


interstate or foreign commerce of information for use in news reporting of sporting
events or contests, or for the transmission of information assisting in the placing of
bets or wagers on a sporting event or contest from a State where betting on that
sporting event or contest is legal into a State in which such betting is legal."

386 U. S. 954. The petition for certiorari also challenged the validity of a warrant
authorizing the search of the petitioner's premises. In light of our disposition of this
case, we do not reach that issue.

We find no merit in the petitioner's further suggestion that his indictment must be
dismissed. After his conviction was affirmed by the Court of Appeals, he testified
before a federal grand jury concerning the charges involved here. Because he was
compelled to testify pursuant to a grant of immunity, 48 Stat. 1096, as amended, 47
U.S.C. § 409(l), it is clear that the fruit of his testimony cannot be used against him
in any future trial. But the petitioner asks for more. He contends that. his
conviction must be vacated and the charges against him dismissed lest he be
"subjected to [a] penalty . . . on account of [a] . . . matter . . . concerning which he
[was] compelled . . . to testify. . . ." 47 U.S.C. § 409(l). Frank v. United States, 347
F.2d 486. We disagree. In relevant part, § 409(l) substantially repeats the language
of the Compulsory Testimony Act of 1893, 27 Stat. 443, 49 U.S.C. § 46, which
was Congress' response to this Court's statement that an immunity statute can
supplant the Fifth Amendment privilege against self-incrimination only if it affords
adequate protection from future prosecution or conviction. Counselman v.
Hitchcock, 142 U. S. 547, 142 U. S. 585-586. The statutory provision here
involved was designed to provide such protection, see Brown v. United States, 359
U. S. 41, 359 U. S. 45-46, not to confer immunity from punishment pursuant to
a prior prosecution and adjudication of guilt. Cf. Regina v. United States, 364 U. S.
507, 364 U. S. 513-514.

"The average man would very likely not have his feelings soothed any more by
having his property seized openly than by having it seized privately and by
stealth. . . . And a person can be just as much, if not more, irritated, annoyed and
injured by an unceremonious public arrest by a policeman as he is by a seizure in
the privacy of his office or home."

Griswold v. Connecticut, 381 U. S. 479, 381 U. S. 509 (dissenting opinion of MR.


JUSTICE BLACK).

The First Amendment, for example, imposes limitations upon governmental


abridgment of "freedom to associate and privacy in one's associations." NAACP v.
Alabama, 357 U. S. 449, 357 U. S. 462. The Third Amendment's prohibition
against the unconsented peacetime quartering of soldiers protects another
aspect of privacy from governmental intrusion. To some extent, the Fifth
Amendment too "reflects the Constitution's concern for . . . . . . the right of each
individual "to a private enclave where he may lead a private life."'" Tehan v.
Shott, 382 U. S. 406, 382 U. S. 416. Virtually every governmental action interferes
with personal privacy to some degree. The question in each case is whether that
interference violates a command of the United States Constitution.

In support of their respective claims, the parties have compiled competing lists of
"protected areas" for our consideration. It appears to be common ground that a
private home is such an area, Weeks v. United States, 232 U. S. 383, but that an
open field is not. Hester v. United States, 265 U. S. 57. Defending the inclusion of
a telephone booth in his list the petitioner cites United States v. Stone, 232 F. Supp.
396, and United States v. Madison, 32 L.W. 2243 (D.C. Ct.Gen.Sess.). Urging that
the telephone booth should be excluded, the Government finds support in United
States v. Borgese, 235 F. Supp. 286.

It is true that this Court has occasionally described its conclusions in terms of
"constitutionally protected areas," see, e.g., Silverman v. United States, 365 U. S.
505, 365 U. S. 510, 365 U. S. 512; Lopez v. United States, 373 U. S. 427, 373 U. S.
438-439; Berger v. New York, 388 U. S. 41, 388 U. S. 57, 388 U. S. 59, but we
have never suggested that this concept can serve as a talismanic solution to every
Fourth Amendment problem.

Based upon their previous visual observations of the petitioner, the agents correctly
predicted that he would use the telephone booth for several minutes at
approximately the same time each morning. The petitioner was subjected to
electronic surveillance only during this predetermined period. Six recordings,
averaging some three minutes each, were obtained and admitted in evidence. They
preserved the petitioners end of conversations concerning the placing of bets and
the receipt of wagering information.

On the single occasion when the statements of another person were inadvertently
intercepted, the agents refrained from listening to them.

Although the protections afforded the petitioner in Osborn were "similar . . . to


those . . . of conventional warrants," they were not identical. A conventional
warrant ordinarily serves to notify the suspect of an intended search. But if Osborn
had been told in advance that federal officers intended to record his conversations,
the point of making such recordings would obviously have been lost; the evidence
in question could not have been obtained. In omitting any requirement of advance
notice, the federal court that authorized electronic surveillance in Osborn simply
recognized, as has this Court, that officers need not announce their purpose before
conducting an otherwise authorized search if such an announcement would
provoke the escape of the suspect or the destruction of critical evidence. See Ker v.
California, 374 U. S. 23, 374 U. S. 37-41.

Although some have thought that this "exception to the notice requirement where
exigent circumstances are present," id. at 374 U. S. 39, should be deemed
inapplicable where police enter a home before its occupants are aware that officers
are present, id. at 374 U. S. 55-58 (opinion of MR. JUSTICE BRENNAN), the
reasons for such a limitation have no bearing here. However true it may be that
"[i]nnocent citizens should not suffer the shock, fright or embarrassment attendant
upon an unannounced police intrusion," id. at 374 U. S. 57, and that "the
requirement of awareness . . . serves to minimize the hazards of the officers'
dangerous calling," id. at 374 U. S. 57-58, these considerations are not relevant to
the problems presented by judicially authorized electronic surveillance.

Nor do the Federal Rules of Criminal Procedure impose an inflexible requirement


of prior notice.Rule 41(d) does require federal officers to serve upon the person
searched a copy of the warrant and a receipt describing the material obtained, but it
does not invariably require that this be done before the search takes place. Nordelli
v. United States, 24 F.2d 665, 666-667.

Thus, the fact that the petitioner in Osborn was unaware that his words were being
electronically transcribed did not prevent this Court from sustaining his conviction,
and did not prevent the Court in Berger from reaching the conclusion that the use
of the recording device sanctioned in Osborn was entirely lawful. 388 U. S.
41, 388 U. S. 57.

In Agnello v. United States, 269 U. S. 20, 269 U. S. 30, the Court stated:

"The right without a search warrant contemporaneously to search persons lawfully


arrested while committing crime and to search the place where the arrest is made in
order to find and seize things connected with the crime as its fruits or as the means
by which it was committed, as well as weapons and other things to effect an escape
from custody, is not to be doubted."

Whatever one's view of "the longstanding practice of searching for other proofs of
guilt within the control of the accused found upon arrest," United States v.
Rabinowitz, 339 U. S. 56, 339 U. S. 61; cf. id. at 339 U. S. 71-79 (dissenting
opinion of Mr. Justice Frankfurter), the concept of an "incidental" search cannot
readily be extended to include surreptitious surveillance of an individual either
immediately before, or immediately after, his arrest.

Although "[t]he Fourth Amendment does not require police officers to delay in the
course of an investigation if to do so would gravely endanger their lives or the lives
of others,"

Warden v. Hayden, 387 U. S. 294, 387 U. S. 298-299, there seems little likelihood


that electronic surveillance would be a realistic possibility in a situation so fraught
with urgency.

A search to which an individual consents meets Fourth Amendment


requirements, Zap v. United States, 328 U. S. 624, but, of course, "the usefulness
of electronic surveillance depends on lack of notice to the suspect." Lopez v.
United States, 373 U. S. 427, 373 U. S. 463 (dissenting opinion of MR. JUSTICE
BRENNAN).

Whether safeguards other than prior authorization by a magistrate would satisfy the
Fourth Amendment in a situation involving the national security is a question not
presented by this case.

See Osborn v. United States, 385 U. S. 323, 385 U. S. 330.

MR. JUSTICE DOUGLAS, with whom MR. JUSTICE BRENNAN joins,


concurring.

While I join the opinion of the Court, I feel compelled to reply to the separate
concurring opinion of my Brother WHITE, which I view as a wholly unwarranted
green light for the Executive Branch to resort to electronic eavesdropping without a
warrant in cases which the Executive Branch itself labels "national security"
matters.

Neither the President nor the Attorney General is a magistrate. In matters where
they believe national security may be involved, they are not detached,
disinterested, and neutral as a court or magistrate must be. Under the separation of
powers created by the Constitution, the Executive Branch is not supposed to be
neutral and disinterested. Rather it should vigorously investigate and prevent
breaches of national security and prosecute those who violate the pertinent federal
laws. The President and Attorney General are properly interested parties, cast in
the role of adversary, in national security cases. They may even be the intended
victims of subversive action. Since spies and saboteurs are as entitled to the
protection of the Fourth Amendment as suspected gamblers like petitioner, I
cannot agree that, where spies and saboteurs are involved adequate protection of
Fourth Amendment rights is assured when the President and Attorney General
assume both the position of "adversary and prosecutor" and disinterested, neutral
magistrate.

There is, so far as I understand constitutional history, no distinction under the


Fourth Amendment between types of crimes. Article III, § 3, gives "treason" a very
narrow definition, and puts restrictions on its proof. But the Fourth Amendment
draws no lines between various substantive offenses. The arrests in cases of "hot
pursuit" and the arrests on visible or other evidence of probable cause cut across
the board, and are not peculiar to any kind of crime.

I would respect the present lines of distinction, and not improvise because a
particular crime seems particularly heinous. When the Framers took that step, as
they did with treason, the worst crime of all, they made their purpose manifest.
MR. JUSTICE HARLAN, concurring.

I join the opinion of the Court, which I read to hold only (a) that an enclosed
telephone booth is an area where, like a home, Weeks v. United States, 232 U. S.
383, and unlike a field, Hester v. United States, 265 U. S. 57, a person has a
constitutionally protected reasonable expectation of privacy; (b) that electronic, as
well as physical, intrusion into a place that is in this sense private may constitute a
violation of the Fourth Amendment, and (c) that the invasion of a constitutionally
protected area by federal authorities is, as the Court has long held, presumptively
unreasonable in the absence of a search warrant.

As the Court's opinion states, "the Fourth Amendment protects people, not places."
The question, however, is what protection it affords to those people. Generally, as
here, the answer to that question requires reference to a "place." My understanding
of the rule that has emerged from prior decisions is that there is a twofold
requirement, first that a person have exhibited an actual (subjective) expectation of
privacy and, second, that the expectation be one that society is prepared to
recognize as "reasonable." Thus, a man's home is, for most purposes, a place where
he expects privacy, but objects, activities, or statements that he exposes to the
"plain view" of outsiders are not "protected," because no intention to keep them to
himself has been exhibited. On the other hand, conversations in the open would not
be protected against being overheard, for the expectation of privacy under the
circumstances would be unreasonable. Cf. Hester v. United States, supra.

The critical fact in this case is that "[o]ne who occupies it, [a telephone booth]
shuts the door behind him, and pays the toll that permits him to place a call is
surely entitled to assume" that his conversation is not being
intercepted. Ante at 389 U. S. 352. The point is not that the booth is "accessible to
the public" at other times, ante at 389 U. S. 351, but that it is a temporarily private
place whose momentary occupants' expectations of freedom from intrusion are
recognized as reasonable. Cf. Rios v. United States, 364 U. S. 253.

In Silverman v. United States, 365 U. S. 505, we held that eavesdropping


accomplished by means of an electronic device that penetrated the premises
occupied by petitioner was a violation of the Fourth Amendment.

That case established that interception of conversations reasonably intended to be


private could constitute a "search and seizure." and that the examination or taking
of physical property was not required. This view of the Fourth Amendment was
followed in Wong Sun v. United States, 371 U. S. 471, at 371 U. S. 485,
and Berger v. New York, 388 U. S. 41, at 51. Also compare Osborn v. United
States, 385 U. S. 323, at 385 U. S. 327. In Silverman, we found it unnecessary to
reexamine Goldman v. United States, 316 U. S. 129, which had held that electronic
surveillance accomplished without the physical penetration of petitioner's premises
by a tangible object did not violate the Fourth Amendment. This case requires us to
reconsider Goldman, and I agree that it should now be overruled. * Its limitation
on Fourth Amendment protection is, in the present day, bad physics as well as bad
law, for reasonable expectations of privacy may be defeated by electronic as well
as physical invasion.

Finally, I do not read the Court's opinion to declare that no interception of a


conversation one-half of which occurs in a public telephone booth can be
reasonable in the absence of a warrant. As elsewhere under the Fourth
Amendment, warrants are the general rule, to which the legitimate needs of law
enforcement may demand specific exceptions. It will be time enough to consider
any such exceptions when an appropriate occasion presents itself, and I agree with
the Court that this is not one.

* I also think that the course of development evinced by Silverman. supra, Wong
Sun., supra, Berger, supra, and today's decision must be recognized as
overruling Olmstead v. United States, 277 U. S. 438, which essentially rested on
the ground that conversations were not subject to the protection of the Fourth
Amendment.

MR. JUSTICE WHITE, concurring.

I agree that the official surveillance of petitioner's telephone conversations in a


public booth must be subjected to the test of reasonableness under the Fourth
Amendment and that, on the record now before us, the particular surveillance
undertaken was unreasonable absent a warrant properly authorizing it. This
application of the Fourth Amendment need not interfere with legitimate needs of
law enforcement.*

In joining the Court's opinion, I note the Court's acknowledgment that there are
circumstances in which it is reasonable to search without a warrant. In this
connection, in footnote 23 the Court points out that today's decision does not reach
national security cases Wiretapping to protect the security of the Nation has been
authorized by successive Presidents. The present Administration would apparently
save national security cases from restrictions against wiretapping. See Berger v.
New York, 388 U. S. 41, 388 U. S. 112-118 (1967) (WHITE, J.,dissenting). We
should not require the warrant procedure and the magistrate's judgment if the
President of the United States or his chief legal officer, the Attorney General, has
considered the requirements of national security and authorized electronic
surveillance as reasonable.

* In previous cases, which are undisturbed by today's decision, the Court has
upheld, as reasonable under the Fourth Amendment, admission at trial of evidence
obtained (1) by an undercover police agent to whom a defendant speaks without
knowledge that he is in the employ of the police, Hoffa v. United States, 385 U. S.
293 (1966); (2) by a recording device hidden on the person of such an
informant, Lopez v. United States, 373 U. S. 427 (1963); Osborn v. United
States, 385 U. S. 323 (1966), and (3) by a policeman listening to the secret
microwave transmissions of an agent conversing with the defendant in another
location, On Lee v. United States, 343 U. S. 747 (1952). When one man speaks to
another, he takes all the risks ordinarily inherent in so doing, including the risk that
the man to whom he speaks will make public what he has heard. The Fourth
Amendment does not protect against unreliable (or law-abiding) associates. Hoffa
v. United States, supra. It is but a logical and reasonable extension of this principle
that a man take the risk that his hearer, free to memorize what he hears for later
verbatim repetitions, is instead recording it or transmitting it to another. The
present case deals with an entirely different situation, for as the Court emphasizes
the petitioner "sought to exclude . . . the uninvited ear," and spoke under
circumstances in which a reasonable person would assume that uninvited ears were
not listening.

MR. JUSTICE BLACK, dissenting.

If I could agree with the Court that eavesdropping carried on by electronic means
(equivalent to wiretapping) constitutes a "search" or "seizure," I would be happy to
join the Court's opinion For on that premise, my Brother STEWART sets out
methods in accord with the Fourth Amendment to guide States in the enactment
and enforcement of laws passed to regulate wiretapping by government. In this
respect, today's opinion differs sharply from Berger v. New York, 388 U. S. 41,
decided last Term, which held void on its face a New York statute authorizing
wiretapping on warrants issued by magistrates on showings of probable cause.
The Berger case also set up what appeared to be insuperable obstacles to the valid
passage of such wiretapping laws by States. The Court's opinion in this case,
however, removes the doubts about state power in this field and abates to a large
extent the confusion and near-paralyzing effect of the Berger holding.
Notwithstanding these good efforts of the Court, I am still unable to agree with its
interpretation of the Fourth Amendment.

My basic objection is two-fold: (1) I do not believe that the words of the
Amendment will bear the meaning given them by today's decision, and (2) I do not
believe that it is the proper role of this Court to rewrite the Amendment in order "to
bring it into harmony with the times," and thus reach a result that many people
believe to be desirable.

While I realize that an argument based on the meaning of words lacks the scope,
and no doubt the appeal, of broad policy discussions and philosophical discourses
on such nebulous subjects as privacy, for me, the language of the Amendment is
the crucial place to look in construing a written document such as our Constitution.
The Fourth Amendment says that
"The right of the people to be secure in their persons, houses, papers, and effects,
against unreasonable searches and seizures, shall not be violated, and no Warrants
shall issue, but upon probable cause, supported by Oath or affirmation, and
particularly describing the place to be searched and the persons or things to be
seized."

The first clause protects "persons, houses, papers, and effects against unreasonable
searches and seizures. . . ." These words connote the idea of tangible things with
size, form, and weight, things capable of being searched, seized, or both. The
second clause of the Amendment still further establishes its Framers' purpose to
limit its protection to tangible things by providing that no warrants shall issue but
those "particularly describing the place to be searched, and the persons or things to
be seized." A conversation overheard by eavesdropping, whether by plain snooping
or wiretapping, is not tangible and, under the normally accepted meanings of the
words, can neither be searched nor seized. In addition the language of the second
clause indicates that the Amendment refers not only to something tangible so it can
be seized, but to something already in existence, so it can be described. Yet the
Court's interpretation would have the Amendment apply to overhearing future
conversations, which, by their very nature, are nonexistent until they take place.
How can one "describe" a future conversation, and, if one cannot, how can a
magistrate issue a warrant to eavesdrop one in the future? It is argued that
information showing what is expected to be said is sufficient to limit the
boundaries of what later can be admitted into evidence; but does such general
information really meet the specific language of the Amendment, which says
"particularly describing"? Rather than using language in a completely artificial
way, I must conclude that the Fourth Amendment simply does not apply to
eavesdropping.

Tapping telephone wires, of course, was an unknown possibility at the time the
Fourth Amendment was adopted. But eavesdropping (and wiretapping is nothing
more than eavesdropping by telephone) was, as even the majority opinion
in Berger, supra, recognized,

"an ancient practice which, at common law, was condemned as a nuisance. 4


Blackstone, Commentaries 168. In those days, the eavesdropper listened by naked
ear under the eaves of houses or their windows, or beyond their walls seeking out
private discourse."

388 U.S. at 388 U. S. 45. There can be no doubt that the Framers were aware of
this practice, and, if they had desired to outlaw or restrict the use of evidence
obtained by eavesdropping, I believe that they would have used the appropriate
language to do so in the Fourth Amendment. They certainly would not have left
such a task to the ingenuity of language-stretching judges. No one, it seems to me,
can read the debates on the Bill of Rights without reaching the conclusion that its
Framers and critics well knew the meaning of the words they used, what they
would be understood to mean by others, their scope and their limitations. Under
these circumstances, it strikes me as a charge against their scholarship, their
common sense and their candor to give to the Fourth Amendment's language the
eavesdropping meaning the Court imputes to it today.

I do not deny that common sense requires, and that this Court often has said, that
the Bill of Rights' safeguards should be given a liberal construction. This principle,
however, does not justify construing the search and seizure amendment as applying
to eavesdropping or the "seizure" of conversations. The Fourth Amendment was
aimed directly at the abhorred practice of breaking in, ransacking and searching
homes and other buildings and seizing people's personal belongings without
warrants issued by magistrates. The Amendment deserves, and this Court has given
it, a liberal construction in order to protect against warrantless searches of
buildings and seizures of tangible personal effects. But, until today, this Court has
refused to say that eavesdropping comes within the ambit of Fourth Amendment
restrictions. See, e.g., Olmstead v. United States, 277 U. S. 438 (1928),
and Goldman v. United States, 316 U. S. 129 (1942).

So far, I have attempted to state why I think the words of the Fourth Amendment
prevent its application to eavesdropping. It is important now to show that this has
been the traditional view of the Amendment's scope since its adoption, and that the
Court's decision in this case, along with its amorphous holding in Berger last Term,
marks the first real departure from that view.

The first case to reach this Court which actually involved a clear-cut test of the
Fourth Amendment's applicability to eavesdropping through a wiretap was, of
course, Olmstead, supra. In holding that the interception of private telephone
conversations by means of wiretapping was not a violation of the Fourth
Amendment, this Court, speaking through Mr. Chief Justice Taft, examined the
language of the Amendment and found, just as I do now, that the words could not
be stretched to encompass overheard conversations:

"The Amendment itself shows that the search is to be of material things -- the
person, the house, his papers or his effects. The description of the warrant
necessary to make the proceeding lawful, is that it must specify the place to be
searched and the person or things to be seized. . . ."

"* * * *"

"Justice Bradley in the Boyd case [Boyd v. United States, 116 U. S. 616], and


Justice Clark[e] in the Gouled case [Gouled v. United States, 255 U. S. 298], said
that the Fifth Amendment and the Fourth Amendment were to be liberally
construed to effect the purpose of the framers of the Constitution in the interest of
liberty. But that cannot justify enlargement of the language employed beyond the
possible practical meaning of houses, persons, papers, and effects, or so to apply
the words search and seizure as to forbid hearing or sight."

277 U.S. at 277 U. S. 464-465.

Goldman v. United States, 316 U. S. 129, is an even clearer example of this Court's


traditional refusal to consider eavesdropping as being covered by the Fourth
Amendment. There, federal agents used a detectaphone, which was placed on the
wall of an adjoining room, to listen to the conversation of a defendant carried on in
his private office and intended to be confined within the four walls of the room.
This Court, referring to Olmstead, found no Fourth Amendment violation.

It should be noted that the Court in Olmstead based its decision squarely on the
fact that wiretapping or eavesdropping does not violate the Fourth Amendment. As
shown supra in the cited quotation from the case, the Court went to great pains to
examine the actual language of the Amendment, and found that the words used
simply could not be stretched to cover eavesdropping. That there was no trespass
was not the determinative factor, and indeed the Court, in citing Hester v. United
States, 265 U. S. 57, indicated that, even where there was a trespass, the Fourth
Amendment does not automatically apply to evidence obtained by "hearing or
sight." The Olmstead majority characterized Hester as holding

"that the testimony of two officers of the law who trespassed on the defendant's
land, concealed themselves one hundred yards away from his house, and saw him
come out and hand a bottle of whiskey to another, was not inadmissible. While
there was a trespass, there was no search of person, house, papers or effects."

277 U.S. at 277 U. S. 465. Thus, the clear holding of


the Olmstead and Goldman cases, undiluted by any question of trespass, is that
eavesdropping, in both its original and modern forms, is not violative of the Fourth
Amendment.

While my reading of the Olmstead and Goldman cases convinces me that they


were decided on the basis of the inapplicability of the wording of the Fourth
Amendment to eavesdropping, and not on any trespass basis, this is not to say that
unauthorized intrusion has not played an important role in search and seizure cases.
This Court has adopted an exclusionary rule to bar evidence obtained by means of
such intrusions. As I made clear in my dissenting opinion in Berger v. New
York, 388 U. S. 41, 388 U. S. 76, I continue to believe that this exclusionary rule
formulated in Weeks v. United States, 232 U. S. 383, rests on the "supervisory
power" of this Court over other federal courts and is not rooted in the Fourth
Amendment. See Wolf v. Colorado, concurring opinion, 338 U. S. 338 U.S.
25, 338 U. S. 39, at 40. See also Mapp v. Ohio, concurring opinion, 367 U. S.
367 U.S. 643, 367 U. S. 661-666. This rule has caused the Court to refuse to accept
evidence where there has been such an intrusion regardless of whether there has
been a search or seizure in violation of the Fourth Amendment. As this Court said
in Lopez v. United States, 373 U. S. 427, 373 U. S. 438-439

"The Court has in the past sustained instances of 'electronic eavesdropping' against
constitutional challenge when devices have been used to enable government agents
to overhear conversations which would have been beyond the reach of the human
ear [citing

Olmstead and Goldman]. It has been insisted only that the electronic device not be
planted by an unlawful physical invasion of a constitutionally protected
area. Silverman v. United States."

To support its new interpretation of the Fourth Amendment, which, in effect,


amounts to a rewriting of the language, the Court's opinion concludes that "the
underpinnings of Olmstead and Goldman have been . . . eroded by our subsequent
decisions. . . ." But the only cases cited as accomplishing this "eroding"
are Silverman v. United States, 365 U. S. 505, and Warden v. Hayden, 387 U. S.
294. Neither of these cases "eroded" Olmstead or Goldman. Silverman is an
interesting choice, since there the Court expressly refused to reexamine the
rationale of Olmstead or Goldman although such a reexamination was strenuously
urged upon the Court by the petitioners' counsel. Also, it is significant that,
in Silverman, as the Court described it, "the eavesdropping was accomplished by
means of an unauthorized physical penetration into the premises occupied by the
petitioners," 365 U.S. at 365 U. S. 509, thus calling into play the supervisory
exclusionary rule of evidence. As I have pointed out above, where there is an
unauthorized intrusion, this Court has rejected admission of evidence obtained
regardless of whether there has been an unconstitutional search and seizure. The
majority's decision here relies heavily on the statement in the opinion that the
Court "need not pause to consider whether or not there was a technical trespass
under the local property law relating to party walls." (At 365 U. S. 511.) Yet this
statement should not becloud the fact that, time and again, the opinion emphasizes
that there has been an unauthorized intrusion:

"For a fair reading of the record in this case shows that the eavesdropping was
accomplished by means of an unauthorized physical penetration into the premises
occupied by the petitioners."

(At 365 U. S. 509, emphasis added.) "Eavesdropping

accomplished by means of such a physical intrusion is beyond the pale of even


those decisions. . . ." (At 365 U. S. 509, emphasis added.) "Here . . . the officers
overheard the petitioners' conversations only by usurping part of the petitioners'
house or office. . . ." (At 365 U. S. 511, emphasis added.) "[D]ecision here . . . is
based upon the reality of an actual intrusion. . . ." (At 365 U. S. 512, emphasis
added.) "We find no occasion to reexamine Goldman here, but we decline to go
beyond it, by even a fraction of an inch." (At 365 U. S. 512, emphasis added.) As if
this were not enough, Justices Clark and Whittaker concurred with the following
statement:

"In view of the determination by the majority that the unauthorized physical


penetration into petitioners' premises constituted sufficient trespass to remove this
case from the coverage of earlier decisions, we feel obliged to join in the Court's
opinion."

(At 365 U. S. 513, emphasis added.) As I made clear in my dissent in Berger, the


Court in Silverman held the evidence should be excluded by virtue of the
exclusionary rule, and "I would not have agreed with the Court's opinion in
Silverman . . . had I thought that the result depended on finding a violation of the
Fourth Amendment. . . ." 388 U.S. at 388 U. S. 79-80. In light of this and the fact
that the Court expressly refused to reexamine Olmstead and Goldman, I cannot
read Silverman as overturning the interpretation stated very plainly
in Olmstead and followed in Goldman that eavesdropping is not covered by the
Fourth Amendment.

The other "eroding" case cited in the Court's opinion is Warden v. Hayden, 387 U.
S. 294. It appears that this case is cited for the proposition that the Fourth
Amendment applies to "intangibles," such as conversation, and the following
ambiguous statement is quoted from the opinion: "The premise that property
interests control the right of the Government to search and seize has been
discredited." 387 U.S. at 387 U. S. 304. But far from being concerned with
eavesdropping, Warden v. Hayden upholds the seizure of clothes, certainly
tangibles by any definition. The discussion of property interests was involved only
with the common law rule that the right to seize property depended upon proof of a
superior property interest.

Thus, I think that, although the Court attempts to convey the impression that, for
some reason, today Olmstead and Goldman are no longer good law, it must face up
to the fact that these cases have never been overruled, or even "eroded." It is the
Court's opinions in this case and Berger which, for the first time since 1791, when
the Fourth Amendment was adopted, have declared that eavesdropping is subject
to Fourth Amendment restrictions and that conversations can be "seized."* I must
align myself with all those judges who up to this year have never been able to
impute such a meaning to the words of the Amendment.

Since I see no way in which the words of the Fourth Amendment can be construed
to apply to eavesdropping, that closes the matter for me. In interpreting the Bill of
Rights, I willingly go as far as a liberal construction of the language takes me, but I
simply cannot in good conscience give a meaning to words which they have never
before been thought to have and which they certainly do not have in common
ordinary usage. I will not distort the words of the Amendment in order to "keep the
Constitution up to date" or "to bring it into harmony with the times." It was never
meant that this Court have such power, which, in effect, would make us a
continuously functioning constitutional convention.

With this decision the Court has completed, I hope, its rewriting of the Fourth
Amendment, which started only recently when the Court began referring
incessantly to the Fourth Amendment not so much as a law
against unreasonable searches and seizures as one to protect an individual's
privacy. By clever word juggling, the Court finds it plausible to argue that
language aimed specifically at searches and seizures of things that can be searched
and seized may, to protect privacy, be applied to eavesdropped evidence of
conversations that can neither be searched nor seized. Few things happen to an
individual that do not affect his privacy in one way or another. Thus, by arbitrarily
substituting the Court's language, designed to protect privacy, for the Constitution's
language, designed to protect against unreasonable searches and seizures, the Court
has made the Fourth Amendment its vehicle for holding all laws violative of the
Constitution which offend the Court's broadest concept of privacy. As I said
in Griswold v. Connecticut, 381 U. S. 479,

"The Court talks about a constitutional 'right of privacy' as though there is some
constitutional provision or provisions forbidding any law ever to be passed which
might abridge the 'privacy' of individuals. But there is not."

(Dissenting opinion, at 381 U. S. 508.) I made clear in that dissent my fear of the
dangers involved when this Court uses the "broad, abstract and ambiguous
concept" of "privacy" as a "comprehensive substitute for the Fourth Amendment's
guarantee against unreasonable searches and seizures.'" (See generally dissenting
opinion at 381 U. S. 507-527.)

The Fourth Amendment protects privacy only to the extent that it prohibits
unreasonable searches and seizures of "persons, houses, papers, and effects." No
general right is created by the Amendment so as to give this Court the unlimited
power to hold unconstitutional everything which affects privacy. Certainly the
Framers, well acquainted as they were with the excesses of governmental power,
did not intend to grant this Court such omnipotent lawmaking authority as that.
The history of governments proves that it is dangerous to freedom to repose such
powers in courts.

For these reasons, I respectfully dissent.

* The first paragraph of my Brother HARLAN's concurring opinion is susceptible


of the interpretation, although probably not intended, that this Court "has long
held" eavesdropping to be a violation of the Fourth Amendment and therefore
"presumptively unreasonable in the absence of a search warrant." There is no
reference to any long line of cases, but simply a citation to Silverman, and several
cases following it, to establish this historical proposition. In the first place, as I
have indicated in this opinion, I do not read Silverman as holding any such thing,
and, in the second place, Silverman was decided in 1961. Thus, whatever it held, it
cannot be said it "has [been] long held." I think my Brother HARLAN recognizes
this later in his opinion when he admits that the Court must now
overrule Olmstead and Goldman. In having to overrule these cases in order to
establish the holding the Court adopts today, it becomes clear that the Court is
promulgating new doctrine instead of merely following what it "has long held."
This is emphasized by my Brother HARLAN's claim that it is "bad physics" to
adhere to Goldman. Such an assertion simply illustrates the propensity of some
members of the Court to rely on their limited understanding of modern scientific
subjects in order to fit the Constitution to the times and give its language a
meaning that it will not tolerate.

Pollo v. Constantino-David, G.R. No. 181881, October 18, 2011

This case involves a search of office computer assigned to a government employee


who was charged administratively and eventually dismissed from the service. The
employee’s personal files stored in the computer were used by the government
employer as evidence of misconduct.

Before us is a petition for review on certiorari under Rule 45 which seeks to


reverse and set aside the Decision1 dated October 11, 2007 and Resolution2 dated
February 29, 2008 of the Court of Appeals (CA). The CA dismissed the petition
for certiorari (CA-G.R. SP No. 98224) filed by petitioner Briccio "Ricky" A. Pollo
to nullify the proceedings conducted by the Civil Service Commission (CSC)
which found him guilty of dishonesty, grave misconduct, conduct prejudicial to the
best interest of the service, and violation of Republic Act (R.A.) No. 6713 and
penalized him with dismissal.

The factual antecedents:

Petitioner is a former Supervising Personnel Specialist of the CSC Regional Office


No. IV and also the Officer-in-Charge of the Public Assistance and Liaison
Division (PALD) under the "Mamamayan Muna Hindi Mamaya Na" program of
the CSC.

On January 3, 2007 at around 2:30 p.m., an unsigned letter-complaint addressed to


respondent CSC Chairperson Karina Constantino-David which was marked
"Confidential" and sent through a courier service (LBC) from a certain "Alan San
Pascual" of Bagong Silang, Caloocan City, was received by the Integrated Records
Management Office (IRMO) at the CSC Central Office. Following office practice
in which documents marked "Confidential" are left unopened and instead sent to
the addressee, the aforesaid letter was given directly to Chairperson David.
The letter-complaint reads:

The Chairwoman
Civil Service Commission
Batasan Hills, Quezon City

Dear Madam Chairwoman,

Belated Merry Christmas and Advance Happy New Year!

As a concerned citizen of my beloved country, I would like to ask from you


personally if it is just alright for an employee of your agency to be a lawyer of an
accused gov’t employee having a pending case in the csc. I honestly think this is a
violation of law and unfair to others and your office.

I have known that a person have been lawyered by one of your attorny in the
region 4 office. He is the chief of the Mamamayan muna hindi mamaya na
division. He have been helping many who have pending cases in the Csc. The
justice in our govt system will not be served if this will continue. Please investigate
this anomaly because our perception of your clean and good office is being tainted.

Concerned Govt employee3


Chairperson David immediately formed a team of four personnel with background
in information technology (IT), and issued a memo directing them to conduct an
investigation and specifically "to back up all the files in the computers found in the
Mamamayan Muna (PALD) and Legal divisions."4 After some briefing, the team
proceeded at once to the CSC-ROIV office at Panay Avenue, Quezon City. Upon
their arrival thereat around 5:30 p.m., the team informed the officials of the CSC-
ROIV, respondents Director IV Lydia Castillo (Director Castillo) and Director III
Engelbert Unite (Director Unite) of Chairperson David’s directive.

The backing-up of all files in the hard disk of computers at the PALD and Legal
Services Division (LSD) was witnessed by several employees, together with
Directors Castillo and Unite who closely monitored said activity. At around 6:00
p.m., Director Unite sent text messages to petitioner and the head of LSD, who
were both out of the office at the time, informing them of the ongoing copying of
computer files in their divisions upon orders of the CSC Chair. The text messages
received by petitioner read:

"Gud p.m. This is Atty. Unite FYI: Co people are going over the PCs of
PALD and LSD per instruction of the Chairman. If you can make it here
now it would be better."

"All PCs Of PALD and LSD are being backed up per memo of the chair."
"CO IT people arrived just now for this purpose. We were not also informed
about this.

"We can’t do anything about … it … it’s a directive from chair."

"Memo of the chair was referring to an anonymous complaint"; "ill send a


copy of the memo via mms"5

Petitioner replied also thru text message that he was leaving the matter to Director
Unite and that he will just get a lawyer. Another text message received by
petitioner from PALD staff also reported the presence of the team from CSC main
office: "Sir may mga taga C.O. daw sa kuarto natin." 6 At around 10:00 p.m. of the
same day, the investigating team finished their task. The next day, all the
computers in the PALD were sealed and secured for the purpose of preserving all
the files stored therein. Several diskettes containing the back-up files sourced from
the hard disk of PALD and LSD computers were turned over to Chairperson
David. The contents of the diskettes were examined by the CSC’s Office for Legal
Affairs (OLA). It was found that most of the files in the 17 diskettes containing
files copied from the computer assigned to and being used by the petitioner,
numbering about 40 to 42 documents, were draft pleadings or letters 7 in connection
with administrative cases in the CSC and other tribunals. On the basis of this
finding, Chairperson David issued the Show-Cause Order 8 dated January 11, 2007,
requiring the petitioner, who had gone on extended leave, to submit his explanation
or counter-affidavit within five days from notice.

Evaluating the subject documents obtained from petitioner’s personal files,


Chairperson David made the following observations:

Most of the foregoing files are drafts of legal pleadings or documents that are
related to or connected with administrative cases that may broadly be lumped as
pending either in the CSCRO No. IV, the CSC-NCR, the CSC-Central Office or
other tribunals. It is also of note that most of these draft pleadings are for and on
behalves of parties, who are facing charges as respondents in administrative cases.
This gives rise to the inference that the one who prepared them was knowingly,
deliberately and willfully aiding and advancing interests adverse and inimical to
the interest of the CSC as the central personnel agency of the government tasked to
discipline misfeasance and malfeasance in the government service. The number of
pleadings so prepared further demonstrates that such person is not merely engaged
in an isolated practice but pursues it with seeming regularity. It would also be the
height of naivete or credulity, and certainly against common human experience, to
believe that the person concerned had engaged in this customary practice without
any consideration, and in fact, one of the retrieved files (item 13 above) appears to
insinuate the collection of fees. That these draft pleadings were obtained from the
computer assigned to Pollo invariably raises the presumption that he was the one
responsible or had a hand in their drafting or preparation since the computer of
origin was within his direct control and disposition. 9

Petitioner filed his Comment, denying that he is the person referred to in the
anonymous letter-complaint which had no attachments to it, because he is not a
lawyer and neither is he "lawyering" for people with cases in the CSC. He accused
CSC officials of conducting a "fishing expedition" when they unlawfully copied
and printed personal files in his computer, and subsequently asking him to submit
his comment which violated his right against self-incrimination. He asserted that
he had protested the unlawful taking of his computer done while he was on leave,
citing the letter dated January 8, 2007 in which he informed Director Castillo that
the files in his computer were his personal files and those of his sister, relatives,
friends and some associates and that he is not authorizing their sealing, copying,
duplicating and printing as these would violate his constitutional right to privacy
and protection against self-incrimination and warrantless search and seizure. He
pointed out that though government property, the temporary use and ownership of
the computer issued under a Memorandum of Receipt (MR) is ceded to the
employee who may exercise all attributes of ownership, including its use for
personal purposes. As to the anonymous letter, petitioner argued that it is not
actionable as it failed to comply with the requirements of a formal complaint under
the Uniform Rules on Administrative Cases in the Civil Service (URACC). In view
of the illegal search, the files/documents copied from his computer without his
consent is thus inadmissible as evidence, being "fruits of a poisonous tree." 10

On February 26, 2007, the CSC issued Resolution No. 07038211 finding prima
facie case against the petitioner and charging him with Dishonesty, Grave
Misconduct, Conduct Prejudicial to the Best Interest of the Service and Violation
of R.A. No. 6713 (Code of Conduct and Ethical Standards for Public Officials and
Employees). Petitioner was directed to submit his answer under oath within five
days from notice and indicate whether he elects a formal investigation. Since the
charges fall under Section 19 of the URACC, petitioner was likewise placed under
90 days preventive suspension effective immediately upon receipt of the
resolution. Petitioner received a copy of Resolution No. 070382 on March 1, 2007.

Petitioner filed an Omnibus Motion (For Reconsideration, to Dismiss and/or to


Defer) assailing the formal charge as without basis having proceeded from an
illegal search which is beyond the authority of the CSC Chairman, such power
pertaining solely to the court. Petitioner reiterated that he never aided any people
with pending cases at the CSC and alleged that those files found in his computer
were prepared not by him but by certain persons whom he permitted, at one time or
another, to make use of his computer out of close association or friendship.
Attached to the motion were the affidavit of Atty. Ponciano R. Solosa who
entrusted his own files to be kept at petitioner’s CPU and Atty. Eric N. Estrellado,
the latter being Atty. Solosa’s client who attested that petitioner had nothing to do
with the pleadings or bill for legal fees because in truth he owed legal fees to Atty.
Solosa and not to petitioner. Petitioner contended that the case should be deferred
in view of the prejudicial question raised in the criminal complaint he filed before
the Ombudsman against Director Buensalida, whom petitioner believes had
instigated this administrative case. He also prayed for the lifting of the preventive
suspension imposed on him. In its Resolution No. 070519 12 dated March 19, 2007,
the CSC denied the omnibus motion. The CSC resolved to treat the said motion as
petitioner’s answer.

On March 14, 2007, petitioner filed an Urgent Petition 13 under Rule 65 of the Rules
of Court, docketed as CA-G.R. SP No. 98224, assailing both the January 11, 2007
Show-Cause Order and Resolution No. 070382 dated February 26, 2007 as having
been issued with grave abuse of discretion amounting to excess or total absence of
jurisdiction. Prior to this, however, petitioner lodged an administrative/criminal
complaint against respondents Directors Racquel D.G. Buensalida (Chief of Staff,
Office of the CSC Chairman) and Lydia A. Castillo (CSC-RO IV) before the
Office of the Ombudsman, and a separate complaint for disbarment against
Director Buensalida.14

On April 17, 2007, petitioner received a notice of hearing from the CSC setting the
formal investigation of the case on April 30, 2007. On April 25, 2007, he filed in
the CA an Urgent Motion for the issuance of TRO and preliminary
injunction.15 Since he failed to attend the pre-hearing conference scheduled on
April 30, 2007, the CSC reset the same to May 17, 2007 with warning that the
failure of petitioner and/or his counsel to appear in the said pre-hearing conference
shall entitle the prosecution to proceed with the formal investigation ex-
parte.16 Petitioner moved to defer or to reset the pre-hearing conference, claiming
that the investigation proceedings should be held in abeyance pending the
resolution of his petition by the CA. The CSC denied his request and again
scheduled the pre-hearing conference on May 18, 2007 with similar warning on the
consequences of petitioner and/or his counsel’s non-appearance. 17 This prompted
petitioner to file another motion in the CA, to cite the respondents, including the
hearing officer, in indirect contempt. 18

On June 12, 2007, the CSC issued Resolution No. 07113419 denying petitioner’s
motion to set aside the denial of his motion to defer the proceedings and to inhibit
the designated hearing officer, Atty. Bernard G. Jimenez. The hearing officer was
directed to proceed with the investigation proper with dispatch.

In view of the absence of petitioner and his counsel, and upon the motion of the
prosecution, petitioner was deemed to have waived his right to the formal
investigation which then proceeded ex parte.

On July 24, 2007, the CSC issued Resolution No. 071420, 20 the dispositive part of
which reads:
WHEREFORE, foregoing premises considered, the Commission hereby finds
Briccio A. Pollo, a.k.a. Ricky A. Pollo GUILTY of Dishonesty, Grave Misconduct,
Conduct Prejudicial to the Best Interest of the Service and Violation of Republic
Act 6713. He is meted the penalty of DISMISSAL FROM THE SERVICE with all
its accessory penalties, namely, disqualification to hold public office, forfeiture of
retirement benefits, cancellation of civil service eligibilities and bar from taking
future civil service examinations.21

On the paramount issue of the legality of the search conducted on petitioner’s


computer, the CSC noted the dearth of jurisprudence relevant to the factual milieu
of this case where the government as employer invades the private files of an
employee stored in the computer assigned to him for his official use, in the course
of initial investigation of possible misconduct committed by said employee and
without the latter’s consent or participation. The CSC thus turned to relevant
rulings of the United States Supreme Court, and cited the leading case of
O’Connor v. Ortega22 as authority for the view that government agencies, in their
capacity as employers, rather than law enforcers, could validly conduct search and
seizure in the governmental workplace without meeting the "probable cause" or
warrant requirement for search and seizure. Another ruling cited by the CSC is the
more recent case of United States v. Mark L. Simons 23 which declared that the
federal agency’s computer use policy foreclosed any inference of reasonable
expectation of privacy on the part of its employees. Though the Court therein
recognized that such policy did not, at the same time, erode the respondent’s
legitimate expectation of privacy in the office in which the computer was installed,
still, the warrantless search of the employee’s office was upheld as valid because a
government employer is entitled to conduct a warrantless search pursuant to an
investigation of work-related misconduct provided the search is reasonable in its
inception and scope.

With the foregoing American jurisprudence as benchmark, the CSC held that
petitioner has no reasonable expectation of privacy with regard to the computer he
was using in the regional office in view of the CSC computer use policy which
unequivocally declared that a CSC employee cannot assert any privacy right to a
computer assigned to him. Even assuming that there was no such administrative
policy, the CSC was of the view that the search of petitioner’s computer
successfully passed the test of reasonableness for warrantless searches in the
workplace as enunciated in the aforecited authorities. The CSC stressed that it
pursued the search in its capacity as government employer and that it was
undertaken in connection with an investigation involving work-related misconduct,
which exempts it from the warrant requirement under the Constitution. With the
matter of admissibility of the evidence having been resolved, the CSC then ruled
that the totality of evidence adequately supports the charges of grave misconduct,
dishonesty, conduct prejudicial to the best interest of the service and violation of
R.A. No. 6713 against the petitioner. These grave infractions justified petitioner’s
dismissal from the service with all its accessory penalties.
In his Memorandum24 filed in the CA, petitioner moved to incorporate the above
resolution dismissing him from the service in his main petition, in lieu of the filing
of an appeal via a Rule 43 petition. In a subsequent motion, he likewise prayed for
the inclusion of Resolution No. 07180025 which denied his motion for
reconsideration.

By Decision dated October 11, 2007, the CA dismissed the petition for certiorari
after finding no grave abuse of discretion committed by respondents CSC officials.
The CA held that: (1) petitioner was not charged on the basis of the anonymous
letter but from the initiative of the CSC after a fact-finding investigation was
conducted and the results thereof yielded a prima facie case against him; (2) it
could not be said that in ordering the back-up of files in petitioner’s computer and
later confiscating the same, Chairperson David had encroached on the authority of
a judge in view of the CSC computer policy declaring the computers as
government property and that employee-users thereof have no reasonable
expectation of privacy in anything they create, store, send, or receive on the
computer system; and (3) there is nothing contemptuous in CSC’s act of
proceeding with the formal investigation as there was no restraining order or
injunction issued by the CA.

His motion for reconsideration having been denied by the CA, petitioner brought
this appeal arguing that –

THE HONORABLE COURT OF APPEALS GRIEVOUSLY ERRED AND


COMMITTED SERIOUS IRREGULARITY AND BLATANT ERRORS
IN LAW AMOUNTING TO GRAVE ABUSE OF DISCRETION WHEN
IT RULED THAT ANONYMOUS COMPLAINT IS ACTIONABLE
UNDER E.O. 292 WHEN IN TRUTH AND IN FACT THE CONTRARY
IS EXPLICITLY PROVIDED UNDER 2nd PARAGRAPH OF SECTION 8
OF CSC RESOLUTION NO. 99-1936, WHICH IS AN [AMENDMENT]
TO THE ORIGINAL RULES PER CSC RESOLUTION NO. 94-0521;

II

THE HONORABLE COURT GRIEVOUSLY ERRED AND


COMMITTED PALPABLE ERRORS IN LAW AMOUNTING TO
GRAVE ABUSE OF DISCRETION WHEN IT RULED THAT
PETITIONER CANNOT INVOKE HIS RIGHT TO PRIVACY, TO
UNREASONABLE SEARCH AND SEIZURE, AGAINST SELF-
INCRIMINATION, BY VIRTUE OF OFFICE MEMORANDUM NO. 10
S. 2002, A MERE INTERNAL MEMORANDUM SIGNED SOLELY
AND EXCLUSIVELY BY RESPONDENT DAVID AND NOT BY THE
COLLEGIAL COMMISSION CONSIDERING THAT POLICY
MATTERS INVOLVING SUB[S]TANTIAL RIGHTS CANNOT BE
COVERED BY AN OFFICE MEMORANDUM WHICH IS LIMITED TO
PROCEDURAL AND ROUTINARY INSTRUCTION;

III

THE HONORABLE COURT GRAVELY ERRED AND COMMITTED


GRAVE ABUSE OF DISCRETION WHEN IT RULED THAT MEMO
SEARCH DATED JANUARY 3, 2007 AND THE TAKING OF
DOCUMENTS IN THE EVENING THEREOF FROM 7:00 TO 10:00 P.M.
IS NOT GRAVE ABUSE OF DISCRETION LIMITING THE
DEFINITION [OF] GRAVE ABUSE OF DISCRETION TO ONE
INVOLVING AND TAINTED WITH PERSONAL HOSTILITY. IT
LIKEWISE ERRED IN HOLDING THAT DATA STORED IN THE
GOVERNMENT COMPUTERS ARE GOVERNMENT PROPERTIES
INCLUDING THE PERSONAL FILES WHEN THE CONTRARY IS
PROVIDED UNDER SECTION 14 OF OM. 10 s. 2002. AND
GRIEVOUSLY ERRED STILL WHEN IT RULED THAT RESPONDENT
DAVID BY VIRTUE OF O.M. 10 DID NOT ENCROACH ON THE
DUTIES AND FUNCTIONS OF A JUDGE PURSUANT TO ARTICLE
III, SECTION 2 OF THE 1987 PHILIPPINE CONSTITUTION;

IV

THE HONORABLE COURT ERRED WHEN IT FAILED TO CONSIDER


ALL OTHER NEW ARGUMENTS, ADDITIONAL EVIDENCE
HEREUNTO SUBMITTED AS WELL AS ITS FAILURE TO
EVALUATE AND TAKE ACTION ON THE 2 MOTIONS TO ADMIT
AND INCORPORATE CSC RESOLUTION NOS. 07-1420 DATED JULY
24, 2007 AND CSC RESOLUTION 07-1800 DATED SEPTEMBER 10,
2007. IT DID NOT RULE LIKEWISE ON THE FOUR URGENT
MOTION TO RESOLVE ANCILLARY PRAYER FOR TRO.26

Squarely raised by the petitioner is the legality of the search conducted on his
office computer and the copying of his personal files without his knowledge and
consent, alleged as a transgression on his constitutional right to privacy.

The right to privacy has been accorded recognition in this jurisdiction as a facet of
the right protected by the guarantee against unreasonable search and seizure under
Section 2, Article III of the 1987 Constitution, 27 which provides:

Sec. 2. The right of the people to be secure in their persons, houses, papers, and
effects against unreasonable searches and seizures of whatever nature and for any
purpose shall be inviolable, and no search warrant or warrant of arrest shall issue
except upon probable cause to be determined personally by the judge after
examination under oath or affirmation of the complainant and the witnesses he
may produce, and particularly describing the place to be searched and the persons
or things to be seized.

The constitutional guarantee is not a prohibition of all searches and seizures but
only of "unreasonable" searches and seizures. 28 But to fully understand this concept
and application for the purpose of resolving the issue at hand, it is essential that we
examine the doctrine in the light of pronouncements in another jurisdiction. As the
Court declared in People v. Marti29 :

Our present constitutional provision on the guarantee against unreasonable search


and seizure had its origin in the 1935 Charter which, worded as follows:

"The right of the people to be secure in their persons, houses, papers and effects
against unreasonable searches and seizures shall not be violated, and no warrants
shall issue but upon probable cause, to be determined by the judge after
examination under oath or affirmation of the complainant and the witnesses he
may produce, and particularly describing the place to be searched, and the persons
or things to be seized." (Sec. 1[3], Article III)

was in turn derived almost verbatim from the Fourth Amendment to the United
States Constitution. As such, the Court may turn to the pronouncements of the
United States Federal Supreme Court and State Appellate Courts which are
considered doctrinal in this jurisdiction.30

In the 1967 case of Katz v. United States, 31 the US Supreme Court held that the act
of FBI agents in electronically recording a conversation made by petitioner in an
enclosed public telephone booth violated his right to privacy and constituted a
"search and seizure". Because the petitioner had a reasonable expectation of
privacy in using the enclosed booth to make a personal telephone call, the
protection of the Fourth Amendment extends to such area. In the concurring
opinion of Mr. Justice Harlan, it was further noted that the existence of privacy
right under prior decisions involved a two-fold requirement: first, that a person has
exhibited an actual (subjective) expectation of privacy; and second, that the
expectation be one that society is prepared to recognize as reasonable (objective). 32

In Mancusi v. DeForte33 which addressed the reasonable expectations of private


employees in the workplace, the US Supreme Court held that a union employee
had Fourth Amendment rights with regard to an office at union headquarters that
he shared with other union officials, even as the latter or their guests could enter
the office. The Court thus "recognized that employees may have a reasonable
expectation of privacy against intrusions by police."

That the Fourth Amendment equally applies to a government workplace was


addressed in the 1987 case of O’Connor v. Ortega 34 where a physician, Dr. Magno
Ortega, who was employed by a state hospital, claimed a violation of his Fourth
Amendment rights when hospital officials investigating charges of
mismanagement of the psychiatric residency program, sexual harassment of female
hospital employees and other irregularities involving his private patients under the
state medical aid program, searched his office and seized personal items from his
desk and filing cabinets. In that case, the Court categorically declared that
"[i]ndividuals do not lose Fourth Amendment rights merely because they work for
the government instead of a private employer." 35 A plurality of four Justices
concurred that the correct analysis has two steps: first, because "some government
offices may be so open to fellow employees or the public that no expectation of
privacy is reasonable", a court must consider "[t]he operational realities of the
workplace" in order to determine whether an employee’s Fourth Amendment rights
are implicated; and next, where an employee has a legitimate privacy expectation,
an employer’s intrusion on that expectation "for noninvestigatory, work-related
purposes, as well as for investigations of work-related misconduct, should be
judged by the standard of reasonableness under all the circumstances." 36

On the matter of government employees’ reasonable expectations of privacy in


their workplace, O’Connor teaches:

x x x Public employees’ expectations of privacy in their offices, desks, and file


cabinets, like similar expectations of employees in the private sector, may be
reduced by virtue of actual office practices and procedures, or by legitimate
regulation. x x x The employee’s expectation of privacy must be assessed in the
context of the employment relation. An office is seldom a private enclave free from
entry by supervisors, other employees, and business and personal invitees. Instead,
in many cases offices are continually entered by fellow employees and other
visitors during the workday for conferences, consultations, and other work-related
visits. Simply put, it is the nature of government offices that others – such as
fellow employees, supervisors, consensual visitors, and the general public – may
have frequent access to an individual’s office. We agree with JUSTICE SCALIA
that "[c]onstitutional protection against unreasonable searches by the government
does not disappear merely because the government has the right to make
reasonable intrusions in its capacity as employer," x x x but some government
offices may be so open to fellow employees or the public that no expectation of
privacy is reasonable. x x x Given the great variety of work environments in
the public sector, the question of whether an employee has a reasonable
expectation of privacy must be addressed on a case-by-case basis. 37 (Citations
omitted; emphasis supplied.)

On the basis of the established rule in previous cases, the US Supreme Court
declared that Dr. Ortega’s Fourth Amendment rights are implicated only if the
conduct of the hospital officials infringed "an expectation of privacy that society is
prepared to consider as reasonable." Given the undisputed evidence that respondent
Dr. Ortega did not share his desk or file cabinets with any other employees, kept
personal correspondence and other private items in his own office while those
work-related files (on physicians in residency training) were stored outside his
office, and there being no evidence that the hospital had established any reasonable
regulation or policy discouraging employees from storing personal papers and
effects in their desks or file cabinets (although the absence of such a policy does
not create any expectation of privacy where it would not otherwise exist), the
Court concluded that Dr. Ortega has a reasonable expectation of privacy at least in
his desk and file cabinets.38

Proceeding to the next inquiry as to whether the search conducted by hospital


officials was reasonable, the O’Connor plurality decision discussed the following
principles:

Having determined that Dr. Ortega had a reasonable expectation of privacy in his
office, the Court of Appeals simply concluded without discussion that the
"search…was not a reasonable search under the fourth amendment." x x x "[t]o
hold that the Fourth Amendment applies to searches conducted by [public
employers] is only to begin the inquiry into the standards governing such
searches…[W]hat is reasonable depends on the context within which a search takes
place. x x x Thus, we must determine the appropriate standard of reasonableness
applicable to the search. A determination of the standard of reasonableness
applicable to a particular class of searches requires "balanc[ing] the nature and
quality of the intrusion on the individual’s Fourth Amendment interests against the
importance of the governmental interests alleged to justify the intrusion." x x x In
the case of searches conducted by a public employer, we must balance the
invasion of the employees’ legitimate expectations of privacy against the
government’s need for supervision, control, and the efficient operation of the
workplace.

xxxx

In our view, requiring an employer to obtain a warrant whenever the employer


wished to enter an employee’s office, desk, or file cabinets for a work-related
purpose would seriously disrupt the routine conduct of business and would be
unduly burdensome. Imposing unwieldy warrant procedures in such cases upon
supervisors, who would otherwise have no reason to be familiar with such
procedures, is simply unreasonable. In contrast to other circumstances in which we
have required warrants, supervisors in offices such as at the Hospital are hardly in
the business of investigating the violation of criminal laws. Rather, work-related
searches are merely incident to the primary business of the agency. Under these
circumstances, the imposition of a warrant requirement would conflict with the
"common-sense realization that government offices could not function if every
employment decision became a constitutional matter." x x x

xxxx
The governmental interest justifying work-related intrusions by public employers
is the efficient and proper operation of the workplace. Government agencies
provide myriad services to the public, and the work of these agencies would suffer
if employers were required to have probable cause before they entered an
employee’s desk for the purpose of finding a file or piece of office correspondence.
Indeed, it is difficult to give the concept of probable cause, rooted as it is in the
criminal investigatory context, much meaning when the purpose of a search is to
retrieve a file for work-related reasons. Similarly, the concept of probable cause
has little meaning for a routine inventory conducted by public employers for the
purpose of securing state property. x x x To ensure the efficient and proper
operation of the agency, therefore, public employers must be given wide latitude to
enter employee offices for work-related, noninvestigatory reasons.

We come to a similar conclusion for searches conducted pursuant to an


investigation of work-related employee misconduct. Even when employers conduct
an investigation, they have an interest substantially different from "the normal need
for law enforcement." x x x Public employers have an interest in ensuring that their
agencies operate in an effective and efficient manner, and the work of these
agencies inevitably suffers from the inefficiency, incompetence, mismanagement,
or other work-related misfeasance of its employees. Indeed, in many cases, public
employees are entrusted with tremendous responsibility, and the consequences of
their misconduct or incompetence to both the agency and the public interest can be
severe. In contrast to law enforcement officials, therefore, public employers are not
enforcers of the criminal law; instead, public employers have a direct and
overriding interest in ensuring that the work of the agency is conducted in a proper
and efficient manner. In our view, therefore, a probable cause requirement for
searches of the type at issue here would impose intolerable burdens on public
employers. The delay in correcting the employee misconduct caused by the
need for probable cause rather than reasonable suspicion will be translated
into tangible and often irreparable damage to the agency’s work, and
ultimately to the public interest. x x x

xxxx

In sum, we conclude that the "special needs, beyond the normal need for law
enforcement make the…probable-cause requirement impracticable," x x x for
legitimate, work-related noninvestigatory intrusions as well as investigations
of work-related misconduct. A standard of reasonableness will neither unduly
burden the efforts of government employers to ensure the efficient and proper
operation of the workplace, nor authorize arbitrary intrusions upon the privacy of
public employees. We hold, therefore, that public employer intrusions on the
constitutionally protected privacy interests of government employees for
noninvestigatory, work-related purposes, as well as for investigations of work-
related misconduct, should be judged by the standard of reasonableness under
all the circumstances. Under this reasonableness standard, both the inception
and the scope of the intrusion must be reasonable:

"Determining the reasonableness of any search involves a twofold inquiry: first,


one must consider ‘whether the…action was justified at its inception,’ x x x ;
second, one must determine whether the search as actually conducted ‘was
reasonably related in scope to the circumstances which justified the interference in
the first place,’" x x x

Ordinarily, a search of an employee’s office by a supervisor will be "justified at


its inception" when there are reasonable grounds for suspecting that the
search will turn up evidence that the employee is guilty of work-related
misconduct, or that the search is necessary for a noninvestigatory work-
related purpose such as to retrieve a needed file. x x x The search will be
permissible in its scope when "the measures adopted are reasonably related to
the objectives of the search and not excessively intrusive in light of …the
nature of the [misconduct]." x x x39 (Citations omitted; emphasis supplied.)

Since the District Court granted summary judgment without a hearing on the
factual dispute as to the character of the search and neither was there any finding
made as to the scope of the search that was undertaken, the case was remanded to
said court for the determination of the justification for the search and seizure, and
evaluation of the reasonableness of both the inception of the search and its scope.

In O’Connor the Court recognized that "special needs" authorize warrantless


searches involving public employees for work-related reasons. The Court thus laid
down a balancing test under which government interests are weighed against the
employee’s reasonable expectation of privacy. This reasonableness test implicates
neither probable cause nor the warrant requirement, which are related to law
enforcement.40

O’Connor was applied in subsequent cases raising issues on employees’ privacy


rights in the workplace. One of these cases involved a government employer’s
search of an office computer, United States v. Mark L. Simons 41 where the
defendant Simons, an employee of a division of the Central Intelligence Agency
(CIA), was convicted of receiving and possessing materials containing child
pornography. Simons was provided with an office which he did not share with
anyone, and a computer with Internet access. The agency had instituted a policy on
computer use stating that employees were to use the Internet for official
government business only and that accessing unlawful material was specifically
prohibited. The policy also stated that users shall understand that the agency will
periodically audit, inspect, and/or monitor the user’s Internet access as deemed
appropriate. CIA agents instructed its contractor for the management of the
agency’s computer network, upon initial discovery of prohibited internet activity
originating from Simons’ computer, to conduct a remote monitoring and
examination of Simons’ computer. After confirming that Simons had indeed
downloaded pictures that were pornographic in nature, all the files on the hard
drive of Simon’s computer were copied from a remote work station. Days later, the
contractor’s representative finally entered Simon’s office, removed the original
hard drive on Simon’s computer, replaced it with a copy, and gave the original to
the agency security officer. Thereafter, the agency secured warrants and searched
Simons’ office in the evening when Simons was not around. The search team
copied the contents of Simons’ computer; computer diskettes found in Simons’
desk drawer; computer files stored on the zip drive or on zip drive diskettes;
videotapes; and various documents, including personal correspondence. At his
trial, Simons moved to suppress these evidence, arguing that the searches of his
office and computer violated his Fourth Amendment rights. After a hearing, the
district court denied the motion and Simons was found guilty as charged.

Simons appealed his convictions. The US Supreme Court ruled that the searches of
Simons’ computer and office did not violate his Fourth Amendment rights and the
first search warrant was valid. It held that the search remains valid under the
O’Connor exception to the warrant requirement because evidence of the crime was
discovered in the course of an otherwise proper administrative inspection. Simons’
violation of the agency’s Internet policy happened also to be a violation of criminal
law; this does not mean that said employer lost the capacity and interests of an
employer. The warrantless entry into Simons’ office was reasonable under the
Fourth Amendment standard announced in O’Connor because at the inception of
the search, the employer had "reasonable grounds for suspecting" that the hard
drive would yield evidence of misconduct, as the employer was already aware that
Simons had misused his Internet access to download over a thousand pornographic
images. The retrieval of the hard drive was reasonably related to the objective of
the search, and the search was not excessively intrusive. Thus, while Simons had a
reasonable expectation of privacy in his office, he did not have such legitimate
expectation of privacy with regard to the files in his computer.

x x x To establish a violation of his rights under the Fourth Amendment, Simons


must first prove that he had a legitimate expectation of privacy in the place
searched or the item seized. x x x And, in order to prove a legitimate expectation of
privacy, Simons must show that his subjective expectation of privacy is one that
society is prepared to accept as objectively reasonable. x x x

xxxx

x x x We conclude that the remote searches of Simons’ computer did not violate
his Fourth Amendment rights because, in light of the Internet policy, Simons
lacked a legitimate expectation of privacy in the files downloaded from the
Internet. Additionally, we conclude that Simons’ Fourth Amendment rights were
not violated by FBIS’ retrieval of Simons’ hard drive from his office.
Simons did not have a legitimate expectation of privacy with regard to the
record or fruits of his Internet use in light of the FBIS Internet policy. The
policy clearly stated that FBIS would "audit, inspect, and/or monitor"
employees’ use of the Internet, including all file transfers, all websites visited,
and all e-mail messages, "as deemed appropriate." x x x This policy placed
employees on notice that they could not reasonably expect that their Internet
activity would be private. Therefore, regardless of whether Simons subjectively
believed that the files he transferred from the Internet were private, such a belief
was not objectively reasonable after FBIS notified him that it would be overseeing
his Internet use. x x x Accordingly, FBIS’ actions in remotely searching and
seizing the computer files Simons downloaded from the Internet did not violate the
Fourth Amendment.

xxxx

The burden is on Simons to prove that he had a legitimate expectation of


privacy in his office. x x x Here, Simons has shown that he had an office that he
did not share. As noted above, the operational realities of Simons’ workplace may
have diminished his legitimate privacy expectations. However, there is no evidence
in the record of any workplace practices, procedures, or regulations that had such
an effect. We therefore conclude that, on this record, Simons possessed a
legitimate expectation of privacy in his office.

xxxx

In the final analysis, this case involves an employee’s supervisor entering the
employee’s government office and retrieving a piece of government equipment in
which the employee had absolutely no expectation of privacy – equipment that the
employer knew contained evidence of crimes committed by the employee in the
employee’s office. This situation may be contrasted with one in which the criminal
acts of a government employee were unrelated to his employment. Here, there was
a conjunction of the conduct that violated the employer’s policy and the conduct
that violated the criminal law. We consider that FBIS’ intrusion into Simons’
office to retrieve the hard drive is one in which a reasonable employer might
engage. x x x42 (Citations omitted; emphasis supplied.)

This Court, in Social Justice Society (SJS) v. Dangerous Drugs Board 43 which
involved the constitutionality of a provision in R.A. No. 9165 requiring mandatory
drug testing of candidates for public office, students of secondary and tertiary
schools, officers and employees of public and private offices, and persons charged
before the prosecutor’s office with certain offenses, have also recognized the fact
that there may be such legitimate intrusion of privacy in the workplace.

The first factor to consider in the matter of reasonableness is the nature of the
privacy interest upon which the drug testing, which effects a search within the
meaning of Sec. 2, Art. III of the Constitution, intrudes. In this case, the office or
workplace serves as the backdrop for the analysis of the privacy expectation of the
employees and the reasonableness of drug testing requirement. The employees’
privacy interest in an office is to a large extent circumscribed by the company’s
work policies, the collective bargaining agreement, if any, entered into by
management and the bargaining unit, and the inherent right of the employer to
maintain discipline and efficiency in the workplace. Their privacy expectation in a
regulated office environment is, in fine, reduced; and a degree of impingement
upon such privacy has been upheld. (Emphasis supplied.)

Applying the analysis and principles announced in O’Connor and Simons to the
case at bar, we now address the following questions: (1) Did petitioner have a
reasonable expectation of privacy in his office and computer files?; and (2) Was
the search authorized by the CSC Chair, the copying of the contents of the hard
drive on petitioner’s computer reasonable in its inception and scope?

In this inquiry, the relevant surrounding circumstances to consider include "(1) the
employee’s relationship to the item seized; (2) whether the item was in the
immediate control of the employee when it was seized; and (3) whether the
employee took actions to maintain his privacy in the item." These factors are
relevant to both the subjective and objective prongs of the reasonableness inquiry,
and we consider the two questions together. 44 Thus, where the employee used a
password on his computer, did not share his office with co-workers and kept the
same locked, he had a legitimate expectation of privacy and any search of that
space and items located therein must comply with the Fourth Amendment. 45

We answer the first in the negative. Petitioner failed to prove that he had an actual
(subjective) expectation of privacy either in his office or government-issued
computer which contained his personal files. Petitioner did not allege that he had a
separate enclosed office which he did not share with anyone, or that his office was
always locked and not open to other employees or visitors. Neither did he allege
that he used passwords or adopted any means to prevent other employees from
accessing his computer files. On the contrary, he submits that being in the public
assistance office of the CSC-ROIV, he normally would have visitors in his office
like friends, associates and even unknown people, whom he even allowed to use
his computer which to him seemed a trivial request. He described his office as "full
of people, his friends, unknown people" and that in the past 22 years he had been
discharging his functions at the PALD, he is "personally assisting incoming clients,
receiving documents, drafting cases on appeals, in charge of accomplishment
report, Mamamayan Muna Program, Public Sector Unionism, Correction of name,
accreditation of service, and hardly had anytime for himself alone, that in fact he
stays in the office as a paying customer."46 Under this scenario, it can hardly be
deduced that petitioner had such expectation of privacy that society would
recognize as reasonable.
Moreover, even assuming arguendo, in the absence of allegation or proof of the
aforementioned factual circumstances, that petitioner had at least a subjective
expectation of privacy in his computer as he claims, such is negated by the
presence of policy regulating the use of office computers, as in Simons.

Office Memorandum No. 10, S. 2002 "Computer Use Policy (CUP)" explicitly
provides:

POLICY

1. The Computer Resources are the property of the Civil Service


Commission and may be used only for legitimate business purposes.

2. Users shall be permitted access to Computer Resources to assist them in


the performance of their respective jobs.

3. Use of the Computer Resources is a privilege that may be revoked at any


given time.

xxxx

No Expectation of Privacy

4. No expectation of privacy. Users except the Members of the Commission


shall not have an expectation of privacy in anything they create, store, send,
or receive on the computer system.

The Head of the Office for Recruitment, Examination and Placement shall
select and assign Users to handle the confidential examination data and
processes.

5. Waiver of privacy rights. Users expressly waive any right to privacy


in anything they create, store, send, or receive on the computer through the
Internet or any other computer network. Users understand that the CSC may
use human or automated means to monitor the use of its Computer
Resources.

6. Non-exclusivity of Computer Resources. A computer resource is not a


personal property or for the exclusive use of a User to whom a
memorandum of receipt (MR) has been issued. It can be shared or operated
by other users. However, he is accountable therefor and must insure its care
and maintenance.

xxxx

Passwords
12. Responsibility for passwords. Users shall be responsible for
safeguarding their passwords for access to the computer system. Individual
passwords shall not be printed, stored online, or given to others. Users shall
be responsible for all transactions made using their passwords. No User may
access the computer system with another User’s password or account.

13. Passwords do not imply privacy. Use of passwords to gain access to the


computer system or to encode particular files or messages does not imply
that Users have an expectation of privacy in the material they create or
receive on the computer system. The Civil Service Commission has global
passwords that permit access to all materials stored on its networked
computer system regardless of whether those materials have been encoded
with a particular User’s password. Only members of the Commission shall
authorize the application of the said global passwords.

x x x x47 (Emphasis supplied.)

The CSC in this case had implemented a policy that put its employees on notice
that they have no expectation of privacy in anything they create, store, send or
receive on the office computers, and that the CSC may monitor the use of the
computer resources using both automated or human means. This implies that on-
the-spot inspections may be done to ensure that the computer resources were used
only for such legitimate business purposes.

One of the factors stated in O’Connor which are relevant in determining whether
an employee’s expectation of privacy in the workplace is reasonable is the
existence of a workplace privacy policy. 48 In one case, the US Court of Appeals
Eighth Circuit held that a state university employee has not shown that he had a
reasonable expectation of privacy in his computer files where the university’s
computer policy, the computer user is informed not to expect privacy if the
university has a legitimate reason to conduct a search. The user is specifically told
that computer files, including e-mail, can be searched when the university is
responding to a discovery request in the course of litigation. Petitioner employee
thus cannot claim a violation of Fourth Amendment rights when university
officials conducted a warrantless search of his computer for work-related
materials.49

As to the second point of inquiry on the reasonableness of the search conducted on


petitioner’s computer, we answer in the affirmative.

The search of petitioner’s computer files was conducted in connection with


investigation of work-related misconduct prompted by an anonymous letter-
complaint addressed to Chairperson David regarding anomalies in the CSC-ROIV
where the head of the Mamamayan Muna Hindi Mamaya Na division is
supposedly "lawyering" for individuals with pending cases in the CSC.
Chairperson David stated in her sworn affidavit:

8. That prior to this, as early as 2006, the undersigned has received several text
messages from unknown sources adverting to certain anomalies in Civil Service
Commission Regional Office IV (CSCRO IV) such as, staff working in another
government agency, "selling" cases and aiding parties with pending cases, all done
during office hours and involved the use of government properties;

9. That said text messages were not investigated for lack of any verifiable leads
and details sufficient to warrant an investigation;

10. That the anonymous letter provided the lead and details as it pinpointed the
persons and divisions involved in the alleged irregularities happening in CSCRO
IV;

11. That in view of the seriousness of the allegations of irregularities happening in


CSCRO IV and its effect on the integrity of the Commission, I decided to form a
team of Central Office staff to back up the files in the computers of the Public
Assistance and Liaison Division (PALD) and Legal Division;

x x x x50

A search by a government employer of an employee’s office is justified at


inception when there are reasonable grounds for suspecting that it will turn up
evidence that the employee is guilty of work-related misconduct. 51 Thus, in the
2004 case decided by the US Court of Appeals Eighth Circuit, it was held that
where a government agency’s computer use policy prohibited electronic messages
with pornographic content and in addition expressly provided that employees do
not have any personal privacy rights regarding their use of the agency information
systems and technology, the government employee had no legitimate expectation
of privacy as to the use and contents of his office computer, and therefore evidence
found during warrantless search of the computer was admissible in prosecution for
child pornography. In that case, the defendant employee’s computer hard drive was
first remotely examined by a computer information technician after his supervisor
received complaints that he was inaccessible and had copied and distributed non-
work-related e-mail messages throughout the office. When the supervisor
confirmed that defendant had used his computer to access the prohibited websites,
in contravention of the express policy of the agency, his computer tower and
floppy disks were taken and examined. A formal administrative investigation
ensued and later search warrants were secured by the police department. The initial
remote search of the hard drive of petitioner’s computer, as well as the subsequent
warrantless searches was held as valid under the O’Connor ruling that a public
employer can investigate work-related misconduct so long as any search is justified
at inception and is reasonably related in scope to the circumstances that justified it
in the first place.52

Under the facts obtaining, the search conducted on petitioner’s computer was
justified at its inception and scope. We quote with approval the CSC’s discussion
on the reasonableness of its actions, consistent as it were with the guidelines
established by O’Connor:

Even conceding for a moment that there is no such administrative policy, there is
no doubt in the mind of the Commission that the search of Pollo’s computer has
successfully passed the test of reasonableness for warrantless searches in the
workplace as enunciated in the above-discussed American authorities. It bears
emphasis that the Commission pursued the search in its capacity as a
government employer and that it was undertaken in connection with an
investigation involving a work-related misconduct, one of the circumstances
exempted from the warrant requirement. At the inception of the search, a complaint
was received recounting that a certain division chief in the CSCRO No. IV was
"lawyering" for parties having pending cases with the said regional office or in the
Commission. The nature of the imputation was serious, as it was grievously
disturbing. If, indeed, a CSC employee was found to be furtively engaged in the
practice of "lawyering" for parties with pending cases before the Commission
would be a highly repugnant scenario, then such a case would have shattering
repercussions. It would undeniably cast clouds of doubt upon the institutional
integrity of the Commission as a quasi-judicial agency, and in the process, render it
less effective in fulfilling its mandate as an impartial and objective dispenser of
administrative justice. It is settled that a court or an administrative tribunal must
not only be actually impartial but must be seen to be so, otherwise the general
public would not have any trust and confidence in it.

Considering the damaging nature of the accusation, the Commission had to


act fast, if only to arrest or limit any possible adverse consequence or fall-out.
Thus, on the same date that the complaint was received, a search was forthwith
conducted involving the computer resources in the concerned regional office. That
it was the computers that were subjected to the search was justified since
these furnished the easiest means for an employee to encode and store
documents. Indeed, the computers would be a likely starting point in ferreting
out incriminating evidence. Concomitantly, the ephemeral nature of computer
files, that is, they could easily be destroyed at a click of a button, necessitated
drastic and immediate action. Pointedly, to impose the need to comply with the
probable cause requirement would invariably defeat the purpose of the wok-related
investigation.

Worthy to mention, too, is the fact that the Commission effected the warrantless
search in an open and transparent manner. Officials and some employees of the
regional office, who happened to be in the vicinity, were on hand to observe the
process until its completion. In addition, the respondent himself was duly notified,
through text messaging, of the search and the concomitant retrieval of files from
his computer.

All in all, the Commission is convinced that the warrantless search done on
computer assigned to Pollo was not, in any way, vitiated with unconstitutionality.
It was a reasonable exercise of the managerial prerogative of the Commission as an
employer aimed at ensuring its operational effectiveness and efficiency by going
after the work-related misfeasance of its employees. Consequently, the evidence
derived from the questioned search are deemed admissible. 53

Petitioner’s claim of violation of his constitutional right to privacy must


necessarily fail. His other argument invoking the privacy of communication and
correspondence under Section 3(1), Article III of the 1987 Constitution is also
untenable considering the recognition accorded to certain legitimate intrusions into
the privacy of employees in the government workplace under the aforecited
authorities. We likewise find no merit in his contention that O’Connor and Simons
are not relevant because the present case does not involve a criminal offense like
child pornography. As already mentioned, the search of petitioner’s computer was
justified there being reasonable ground for suspecting that the files stored therein
would yield incriminating evidence relevant to the investigation being conducted
by CSC as government employer of such misconduct subject of the anonymous
complaint. This situation clearly falls under the exception to the warrantless
requirement in administrative searches defined in O’Connor.

The Court is not unaware of our decision in Anonymous Letter-Complaint against


Atty. Miguel Morales, Clerk of Court, Metropolitan Trial Court of
Manila54 involving a branch clerk (Atty. Morales) who was investigated on the
basis of an anonymous letter alleging that he was consuming his working hours
filing and attending to personal cases, using office supplies, equipment and
utilities. The OCA conducted a spot investigation aided by NBI agents. The team
was able to access Atty. Morales’ personal computer and print two documents
stored in its hard drive, which turned out to be two pleadings, one filed in the CA
and another in the RTC of Manila, both in the name of another lawyer. Atty.
Morales’ computer was seized and taken in custody of the OCA but was later
ordered released on his motion, but with order to the MISO to first retrieve the files
stored therein. The OCA disagreed with the report of the Investigating Judge that
there was no evidence to support the charge against Atty. Morales as no one from
the OCC personnel who were interviewed would give a categorical and positive
statement affirming the charges against Atty. Morales, along with other court
personnel also charged in the same case. The OCA recommended that Atty.
Morales should be found guilty of gross misconduct. The Court En Banc held that
while Atty. Morales may have fallen short of the exacting standards required of
every court employee, the Court cannot use the evidence obtained from his
personal computer against him for it violated his constitutional right against
unreasonable searches and seizures. The Court found no evidence to support the
claim of OCA that they were able to obtain the subject pleadings with the consent
of Atty. Morales, as in fact the latter immediately filed an administrative case
against the persons who conducted the spot investigation, questioning the validity
of the investigation and specifically invoking his constitutional right against
unreasonable search and seizure. And as there is no other evidence, apart from the
pleadings, retrieved from the unduly confiscated personal computer of Atty.
Morales, to hold him administratively liable, the Court had no choice but to
dismiss the charges against him for insufficiency of evidence.

The above case is to be distinguished from the case at bar because, unlike the
former which involved a personal computer of a court employee, the computer
from which the personal files of herein petitioner were retrieved is a government-
issued computer, hence government property the use of which the CSC has
absolute right to regulate and monitor. Such relationship of the petitioner with the
item seized (office computer) and other relevant factors and circumstances under
American Fourth Amendment jurisprudence, notably the existence of CSC MO 10,
S. 2007 on Computer Use Policy, failed to establish that petitioner had a
reasonable expectation of privacy in the office computer assigned to him.

Having determined that the personal files copied from the office computer of
petitioner are admissible in the administrative case against him, we now proceed to
the issue of whether the CSC was correct in finding the petitioner guilty of the
charges and dismissing him from the service.

Well-settled is the rule that the findings of fact of quasi-judicial agencies, like the
CSC, are accorded not only respect but even finality if such findings are supported
by substantial evidence. Substantial evidence is such amount of relevant evidence
which a reasonable mind might accept as adequate to support a conclusion, even if
other equally reasonable minds might conceivably opine otherwise. 55

The CSC based its findings on evidence consisting of a substantial number of


drafts of legal pleadings and documents stored in his office computer, as well as
the sworn affidavits and testimonies of the witnesses it presented during the formal
investigation. According to the CSC, these documents were confirmed to be
similar or exactly the same content-wise with those on the case records of some
cases pending either with CSCRO No. IV, CSC-NCR or the Commission Proper.
There were also substantially similar copies of those pleadings filed with the CA
and duly furnished the Commission. Further, the CSC found the explanation given
by petitioner, to the effect that those files retrieved from his computer hard drive
actually belonged to his lawyer friends Estrellado and Solosa whom he allowed the
use of his computer for drafting their pleadings in the cases they handle, as
implausible and doubtful under the circumstances. We hold that the CSC’s factual
finding regarding the authorship of the subject pleadings and misuse of the office
computer is well-supported by the evidence on record, thus:
It is also striking to note that some of these documents were in the nature of
pleadings responding to the orders, decisions or resolutions of these offices or
directly in opposition to them such as a petition for certiorari or a motion for
reconsideration of CSC Resolution. This indicates that the author thereof
knowingly and willingly participated in the promotion or advancement of the
interests of parties contrary or antagonistic to the Commission. Worse, the
appearance in one of the retrieved documents the phrase, "Eric N. Estr[e]llado,
Epal kulang ang bayad mo," lends plausibility to an inference that the preparation
or drafting of the legal pleadings was pursued with less than a laudable motivation.
Whoever was responsible for these documents was simply doing the same for the
money – a "legal mercenary" selling or purveying his expertise to the highest
bidder, so to speak.

Inevitably, the fact that these documents were retrieved from the computer of Pollo
raises the presumption that he was the author thereof. This is because he had a
control of the said computer. More significantly, one of the witnesses, Margarita
Reyes, categorically testified seeing a written copy of one of the pleadings found in
the case records lying on the table of the respondent. This was the Petition for
Review in the case of Estrellado addressed to the Court of Appeals. The said
circumstances indubitably demonstrate that Pollo was secretly undermining the
interest of the Commission, his very own employer.

To deflect any culpability, Pollo would, however, want the Commission to believe
that the documents were the personal files of some of his friends, including one
Attorney Ponciano Solosa, who incidentally served as his counsel of record during
the formal investigation of this case. In fact, Atty. Solosa himself executed a sworn
affidavit to this effect. Unfortunately, this contention of the respondent was
directly rebutted by the prosecution witness, Reyes, who testified that during her
entire stay in the PALD, she never saw Atty. Solosa using the computer assigned
to the respondent. Reyes more particularly stated that she worked in close
proximity with Pollo and would have known if Atty. Solosa, whom she personally
knows, was using the computer in question. Further, Atty. Solosa himself was
never presented during the formal investigation to confirm his sworn statement
such that the same constitutes self-serving evidence unworthy of weight and
credence. The same is true with the other supporting affidavits, which Pollo
submitted.

At any rate, even admitting for a moment the said contention of the respondent, it
evinces the fact that he was unlawfully authorizing private persons to use the
computer assigned to him for official purpose, not only once but several times
gauging by the number of pleadings, for ends not in conformity with the interests
of the Commission. He was, in effect, acting as a principal by indispensable
cooperation…Or at the very least, he should be responsible for serious misconduct
for repeatedly allowing CSC resources, that is, the computer and the electricity, to
be utilized for purposes other than what they were officially intended.
Further, the Commission cannot lend credence to the posturing of the appellant that
the line appearing in one of the documents, "Eric N. Estrellado, Epal kulang ang
bayad mo," was a private joke between the person alluded to therein, Eric N.
Estrellado, and his counsel, Atty. Solosa, and not indicative of anything more
sinister. The same is too preposterous to be believed. Why would such a statement
appear in a legal pleading stored in the computer assigned to the respondent, unless
he had something to do with it?56

Petitioner assails the CA in not ruling that the CSC should not have entertained an
anonymous complaint since Section 8 of CSC Resolution No. 99-1936 (URACC)
requires a verified complaint:

Rule II – Disciplinary Cases

SEC. 8. Complaint. - A complaint against a civil service official or employee shall


not be given due course unless it is in writing and subscribed and sworn to by the
complainant. However, in cases initiated by the proper disciplining authority,
the complaint need not be under oath.

No anonymous complaint shall be entertained unless there is obvious truth or


merit to the allegation therein or supported by documentary or direct evidence, in
which case the person complained of may be required to comment.

xxxx

We need not belabor this point raised by petitioner. The administrative complaint
is deemed to have been initiated by the CSC itself when Chairperson David, after a
spot inspection and search of the files stored in the hard drive of computers in the
two divisions adverted to in the anonymous letter -- as part of the disciplining
authority’s own fact-finding investigation and information-gathering -- found a
prima facie case against the petitioner who was then directed to file his comment.
As this Court held in Civil Service Commission v. Court of Appeals 57 --

Under Sections 46 and 48 (1), Chapter 6, Subtitle A, Book V of E.O. No. 292 and
Section 8, Rule II of Uniform Rules on Administrative Cases in the Civil Service, a
complaint may be initiated against a civil service officer or employee by the
appropriate disciplining authority, even without being subscribed and sworn to.
Considering that the CSC, as the disciplining authority for Dumlao, filed the
complaint, jurisdiction over Dumlao was validly acquired. (Emphasis supplied.)

As to petitioner’s challenge on the validity of CSC OM 10, S. 2002 (CUP), the


same deserves scant consideration. The alleged infirmity due to the said
memorandum order having been issued solely by the CSC Chair and not the
Commission as a collegial body, upon which the dissent of Commissioner
Buenaflor is partly anchored, was already explained by Chairperson David in her
Reply to the Addendum to Commissioner Buenaflor’s previous memo expressing
his dissent to the actions and disposition of the Commission in this case. According
to Chairperson David, said memorandum order was in fact exhaustively discussed,
provision by provision in the January 23, 2002 Commission Meeting, attended by
her and former Commissioners Erestain, Jr. and Valmores. Hence, the
Commission En Banc at the time saw no need to issue a Resolution for the purpose
and further because the CUP being for internal use of the Commission, the practice
had been to issue a memorandum order.58 Moreover, being an administrative rule
that is merely internal in nature, or which regulates only the personnel of the CSC
and not the public, the CUP need not be published prior to its effectivity. 59

In fine, no error or grave abuse of discretion was committed by the CA in affirming


the CSC’s ruling that petitioner is guilty of grave misconduct, dishonesty, conduct
prejudicial to the best interest of the service, and violation of R.A. No. 6713. The
gravity of these offenses justified the imposition on petitioner of the ultimate
penalty of dismissal with all its accessory penalties, pursuant to existing rules and
regulations.

WHEREFORE, the petition for review on certiorari is DENIED. The


Decision dated October 11, 2007 and Resolution dated February 29, 2008 of the
Court of Appeals in CA-G.R. SP No. 98224 are AFFIRMED.

With costs against the petitioner.

SO ORDERED.

E. Search warrants
1. Requisites
a. existence of probable cause
b. probable cause determined personally by the judge
c. examination under oath of the applicant and his witnesses

CASES:
Burgos v. Chief of Staff, G.R. No. L-64261, December 26, 1984
DOCTRINE AND/OR RELEVANT PROVISION: 
Relevant Provision: 
Article III, Section 2: The right of the people to be secure in their persons, houses,
papers, and effects against unreasonable searches and seizures of whatever nature
and for any purpose shall be inviolable, and no search warrant or warrant of arrest
shall issue except upon probable cause to be determined personally by the judge
after examination under oath or affirmation of the complainant and the witnesses
he may produce, and particularly describing the place to be searched and the
persons or things to be seized.

Existence of Probable Cause:


Probable cause for a search is defined as such facts and circumstances which
would lead a reasonably discreet and prudent man to believe that an offense has
been committed and that the objects sought in connection with the offense are in
the place sought to be searched. And when the search warrant applied for is
directed against a newspaper publisher or editor in connection with the publication
of subversive materials, as in the case at bar, the application and/or its supporting
affidavits must contain a specification, stating with particularity the alleged
subversive material he has published or is intending to publish. Mere
generalization will not suffice. 

Examination under Oath of the Applicant and His Witnesses:


In mandating that "no warrant shall issue except upon probable cause to be
determined by the judge, ... after examination under oath or affirmation of the
complainant and the witnesses he may produce; the Constitution requires no less
than personal knowledge by the complainant or his witnesses of the facts upon
which the issuance of a search warrant may be justified.

The Exclusionary Rule:


The exclusionary rule states that any evidence obtained in violation of the
constitutional mandates is inadmissible in any proceeding for any purpose.

Doctrine of the Fruit of the Poisonous Tree:


This doctrine extends the exclusionary rule by excluding any evidence exposed
through other evidence attained by an illegal search, seizure, or arrest.

Assailed in this petition for certiorari prohibition and mandamus with preliminary
mandatory and prohibitory injunction is the validity of two [2] search warrants
issued on December 7, 1982 by respondent Judge Ernani Cruz-Pano, Executive
Judge of the then Court of First Instance of Rizal [Quezon City], under which the
premises known as No. 19, Road 3, Project 6, Quezon City, and 784 Units C & D,
RMS Building, Quezon Avenue, Quezon City, business addresses of the
"Metropolitan Mail" and "We Forum" newspapers, respectively, were searched,
and office and printing machines, equipment, paraphernalia, motor vehicles and
other articles used in the printing, publication and distribution of the said
newspapers, as well as numerous papers, documents, books and other written
literature alleged to be in the possession and control of petitioner Jose Burgos, Jr.
publisher-editor of the "We Forum" newspaper, were seized.

Petitioners further pray that a writ of preliminary mandatory and prohibitory


injunction be issued for the return of the seized articles, and that respondents,
"particularly the Chief Legal Officer, Presidential Security Command, the Judge
Advocate General, AFP, the City Fiscal of Quezon City, their representatives,
assistants, subalterns, subordinates, substitute or successors" be enjoined from
using the articles thus seized as evidence against petitioner Jose Burgos, Jr. and the
other accused in Criminal Case No. Q- 022782 of the Regional Trial Court of
Quezon City, entitled People v. Jose Burgos, Jr. et al. 1

In our Resolution dated June 21, 1983, respondents were required to answer the
petition. The plea for preliminary mandatory and prohibitory injunction was set for
hearing on June 28, 1983, later reset to July 7, 1983, on motion of the Solicitor
General in behalf of respondents.

At the hearing on July 7, 1983, the Solicitor General, while opposing petitioners'
prayer for a writ of preliminary mandatory injunction, manifested that respondents
"will not use the aforementioned articles as evidence in the aforementioned case
until final resolution of the legality of the seizure of the aforementioned
articles. ..." 2 With this manifestation, the prayer for preliminary prohibitory
injunction was rendered moot and academic.

Respondents would have this Court dismiss the petition on the ground that
petitioners had come to this Court without having previously sought the quashal of
the search warrants before respondent judge. Indeed, petitioners, before impugning
the validity of the warrants before this Court, should have filed a motion to quash
said warrants in the court that issued them. 3 But this procedural flaw
notwithstanding, we take cognizance of this petition in view of the seriousness and
urgency of the constitutional issues raised not to mention the public interest
generated by the search of the "We Forum" offices, which was televised in
Channel 7 and widely publicized in all metropolitan dailies. The existence of this
special circumstance justifies this Court to exercise its inherent power to suspend
its rules. In the words of the revered Mr. Justice Abad Santos in the case of C. Vda.
de Ordoveza v. Raymundo, 4 "it is always in the power of the court [Supreme
Court] to suspend its rules or to except a particular case from its operation,
whenever the purposes of justice require it...".

Respondents likewise urge dismissal of the petition on ground of laches.


Considerable stress is laid on the fact that while said search warrants were issued
on December 7, 1982, the instant petition impugning the same was filed only on
June 16, 1983 or after the lapse of a period of more than six [6] months.

Laches is failure or negligence for an unreasonable and unexplained length of time


to do that which, by exercising due diligence, could or should have been done
earlier. It is negligence or omission to assert a right within a reasonable time,
warranting a presumption that the party entitled to assert it either has abandoned it
or declined to assert it. 5
Petitioners, in their Consolidated Reply, explained the reason for the delay in the
filing of the petition thus:

Respondents should not find fault, as they now do [p. 1, Answer, p. 3,


Manifestation] with the fact that the Petition was filed on June 16,
1983, more than half a year after the petitioners' premises had been
raided.

The climate of the times has given petitioners no other choice. If they
had waited this long to bring their case to court, it was because they
tried at first to exhaust other remedies. The events of the past eleven
fill years had taught them that everything in this country, from release
of public funds to release of detained persons from custody, has
become a matter of executive benevolence or largesse

Hence, as soon as they could, petitioners, upon suggestion of persons


close to the President, like Fiscal Flaminiano, sent a letter to President
Marcos, through counsel Antonio Coronet asking the return at least of
the printing equipment and vehicles. And after such a letter had been
sent, through Col. Balbino V. Diego, Chief Intelligence and Legal
Officer of the Presidential Security Command, they were further
encouraged to hope that the latter would yield the desired results.

After waiting in vain for five [5] months, petitioners finally decided
to come to Court. [pp. 123-124, Rollo]

Although the reason given by petitioners may not be flattering to our judicial
system, We find no ground to punish or chastise them for an error in judgment. On
the contrary, the extrajudicial efforts exerted by petitioners quite evidently negate
the presumption that they had abandoned their right to the possession of the seized
property, thereby refuting the charge of laches against them.

Respondents also submit the theory that since petitioner Jose Burgos, Jr. had used
and marked as evidence some of the seized documents in Criminal Case No. Q-
022872, he is now estopped from challenging the validity of the search warrants.
We do not follow the logic of respondents. These documents lawfully belong to
petitioner Jose Burgos, Jr. and he can do whatever he pleases with them, within
legal bounds. The fact that he has used them as evidence does not and cannot in
any way affect the validity or invalidity of the search warrants assailed in this
petition.

Several and diverse reasons have been advanced by petitioners to nullify the search
warrants in question.
1. Petitioners fault respondent judge for his alleged failure to conduct an
examination under oath or affirmation of the applicant and his witnesses, as
mandated by the above-quoted constitutional provision as wen as Sec. 4, Rule 126
of the Rules of Court .6 This objection, however, may properly be considered moot
and academic, as petitioners themselves conceded during the hearing on August 9,
1983, that an examination had indeed been conducted by respondent judge of Col.
Abadilla and his witnesses.

2. Search Warrants No. 20-82[a] and No. 20- 82[b] were used to search two
distinct places: No. 19, Road 3, Project 6, Quezon City and 784 Units C & D, RMS
Building, Quezon Avenue, Quezon City, respectively. Objection is interposed to
the execution of Search Warrant No. 20-82[b] at the latter address on the ground
that the two search warrants pinpointed only one place where petitioner Jose
Burgos, Jr. was allegedly keeping and concealing the articles listed therein, i.e.,
No. 19, Road 3, Project 6, Quezon City. This assertion is based on that portion of
Search Warrant No. 20- 82[b] which states:

Which have been used, and are being used as instruments and means
of committing the crime of subversion penalized under P.D. 885 as
amended and he is keeping and concealing the same at 19 Road 3,
Project 6, Quezon City.

The defect pointed out is obviously a typographical error. Precisely, two search
warrants were applied for and issued because the purpose and intent were to search
two distinct premises. It would be quite absurd and illogical for respondent judge
to have issued two warrants intended for one and the same place. Besides, the
addresses of the places sought to be searched were specifically set forth in the
application, and since it was Col. Abadilla himself who headed the team which
executed the search warrants, the ambiguity that might have arisen by reason of the
typographical error is more apparent than real. The fact is that the place for which
Search Warrant No. 20- 82[b] was applied for was 728 Units C & D, RMS
Building, Quezon Avenue, Quezon City, which address appeared in the opening
paragraph of the said warrant. 7 Obviously this is the same place that respondent
judge had in mind when he issued Warrant No. 20-82 [b].

In the determination of whether a search warrant describes the premises to be


searched with sufficient particularity, it has been held "that the executing officer's
prior knowledge as to the place intended in the warrant is relevant. This would
seem to be especially true where the executing officer is the affiant on whose
affidavit the warrant had issued, and when he knows that the judge who issued the
warrant intended the building described in the affidavit, And it has also been said
that the executing officer may look to the affidavit in the official court file to
resolve an ambiguity in the warrant as to the place to be searched." 8
3. Another ground relied upon to annul the search warrants is the fact that although
the warrants were directed against Jose Burgos, Jr. alone, articles b belonging to
his co-petitioners Jose Burgos, Sr., Bayani Soriano and the J. Burgos Media
Services, Inc. were seized.

Section 2, Rule 126 of the Rules of Court, enumerates the personal properties that
may be seized under a search warrant, to wit:

Sec. 2. Personal Property to be seized. — A search warrant may be


issued for the search and seizure of the following personal property:

[a] Property subject of the offense;

[b] Property stolen or embezzled and other proceeds or


fruits of the offense; and

[c] Property used or intended to be used as the means of


committing an offense.

The above rule does not require that the property to be seized should be owned by
the person against whom the search warrant is directed. It may or may not be
owned by him. In fact, under subsection [b] of the above-quoted Section 2, one of
the properties that may be seized is stolen property. Necessarily, stolen property
must be owned by one other than the person in whose possession it may be at the
time of the search and seizure. Ownership, therefore, is of no consequence, and it is
sufficient that the person against whom the warrant is directed has control or
possession of the property sought to be seized, as petitioner Jose Burgos, Jr. was
alleged to have in relation to the articles and property seized under the warrants.

4. Neither is there merit in petitioners' assertion that real properties were seized
under the disputed warrants. Under Article 415[5] of the Civil Code of the
Philippines, "machinery, receptables, instruments or implements intended by the
owner of the tenement for an industry or works which may be carried on in a
building or on a piece of land and which tend directly to meet the needs of the said
industry or works" are considered immovable property. In Davao Sawmill Co. v.
Castillo9 where this legal provision was invoked, this Court ruled that machinery
which is movable by nature becomes immobilized when placed by the owner of the
tenement, property or plant, but not so when placed by a tenant, usufructuary, or
any other person having only a temporary right, unless such person acted as the
agent of the owner.

In the case at bar, petitioners do not claim to be the owners of the land and/or
building on which the machineries were placed. This being the case, the
machineries in question, while in fact bolted to the ground remain movable
property susceptible to seizure under a search warrant.
5. The questioned search warrants were issued by respondent judge upon
application of Col. Rolando N. Abadilla Intelligence Officer of the P.C.
Metrocom.10 The application was accompanied by the Joint Affidavit of Alejandro
M. Gutierrez and Pedro U. Tango, 11 members of the Metrocom Intelligence and
Security Group under Col. Abadilla which conducted a surveillance of the
premises prior to the filing of the application for the search warrants on December
7, 1982.

It is contended by petitioners, however, that the abovementioned documents could


not have provided sufficient basis for the finding of a probable cause upon which a
warrant may validly issue in accordance with Section 3, Article IV of the 1973
Constitution which provides:

SEC. 3. ... and no search warrant or warrant of arrest shall issue


except upon probable cause to be determined by the judge, or such
other responsible officer as may be authorized by law, after
examination under oath or affirmation of the complainant and the
witnesses he may produce, and particularly describing the place to be
searched and the persons or things to be seized.

We find petitioners' thesis impressed with merit. Probable cause for a search is
defined as such facts and circumstances which would lead a reasonably discreet
and prudent man to believe that an offense has been committed and that the objects
sought in connection with the offense are in the place sought to be searched. And
when the search warrant applied for is directed against a newspaper publisher or
editor in connection with the publication of subversive materials, as in the case at
bar, the application and/or its supporting affidavits must contain a specification,
stating with particularity the alleged subversive material he has published or is
intending to publish. Mere generalization will not suffice. Thus, the broad
statement in Col. Abadilla's application that petitioner "is in possession or has in
his control printing equipment and other paraphernalia, news publications and
other documents which were used and are all continuously being used as a means
of committing the offense of subversion punishable under Presidential Decree 885,
as amended ..." 12 is a mere conclusion of law and does not satisfy the
requirements of probable cause. Bereft of such particulars as would justify a
finding of the existence of probable cause, said allegation cannot serve as basis for
the issuance of a search warrant and it was a grave error for respondent judge to
have done so.

Equally insufficient as basis for the determination of probable cause is the


statement contained in the joint affidavit of Alejandro M. Gutierrez and Pedro U.
Tango, "that the evidence gathered and collated by our unit clearly shows that the
premises above- mentioned and the articles and things above-described were used
and are continuously being used for subversive activities in conspiracy with, and to
promote the objective of, illegal organizations such as the Light-a-Fire Movement,
Movement for Free Philippines, and April 6 Movement." 13

In mandating that "no warrant shall issue except upon probable cause to be
determined by the judge, ... after examination under oath or affirmation of the
complainant and the witnesses he may produce; 14 the Constitution requires no
less than personal knowledge by the complainant or his witnesses of the facts upon
which the issuance of a search warrant may be justified. In Alvarez v. Court of
First Instance, 15 this Court ruled that "the oath required must refer to the truth of
the facts within the personal knowledge of the petitioner or his witnesses, because
the purpose thereof is to convince the committing magistrate, not the individual
making the affidavit and seeking the issuance of the warrant, of the existence of
probable cause." As couched, the quoted averment in said joint affidavit filed
before respondent judge hardly meets the test of sufficiency established by this
Court in Alvarez case.

Another factor which makes the search warrants under consideration


constitutionally objectionable is that they are in the nature of general warrants. The
search warrants describe the articles sought to be seized in this wise:

1] All printing equipment, paraphernalia, paper, ink, photo


(equipment, typewriters, cabinets, tables, communications/recording
equipment, tape recorders, dictaphone and the like used and/or
connected in the printing of the "WE FORUM" newspaper and any
and all documents communication, letters and facsimile of prints
related to the "WE FORUM" newspaper.

2] Subversive documents, pamphlets, leaflets, books, and other


publication to promote the objectives and piurposes of the subversive
organization known as Movement for Free Philippines, Light-a-Fire
Movement and April 6 Movement; and,

3] Motor vehicles used in the distribution/circulation of the "WE


FORUM" and other subversive materials and propaganda, more
particularly,

1] Toyota-Corolla, colored yellow with Plate No. NKA


892;

2] DATSUN pick-up colored white with Plate No. NKV


969

3] A delivery truck with Plate No. NBS 524;


4] TOYOTA-TAMARAW, colored white with Plate
No. PBP 665; and,

5] TOYOTA Hi-Lux, pick-up truck with Plate No. NGV


427 with marking "Bagong Silang."

In Stanford v. State of Texas 16 the search warrant which authorized the search for
"books, records, pamphlets, cards, receipts, lists, memoranda, pictures, recordings
and other written instruments concerning the Communist Party in Texas," was
declared void by the U.S. Supreme Court for being too general. In like manner,
directions to "seize any evidence in connectionwith the violation of SDC 13-3703
or otherwise" have been held too general, and that portion of a search warrant
which authorized the seizure of any "paraphernalia which could be used to violate
Sec. 54-197 of the Connecticut General Statutes [the statute dealing with the crime
of conspiracy]" was held to be a general warrant, and therefore invalid. 17 The
description of the articles sought to be seized under the search warrants in question
cannot be characterized differently.

In the Stanford case, the U.S. Supreme Courts calls to mind a notable chapter in
English history: the era of disaccord between the Tudor Government and the
English Press, when "Officers of the Crown were given roving commissions to
search where they pleased in order to suppress and destroy the literature of dissent
both Catholic and Puritan Reference herein to such historical episode would not be
relevant for it is not the policy of our government to suppress any newspaper or
publication that speaks with "the voice of non-conformity" but poses no clear and
imminent danger to state security.

As heretofore stated, the premises searched were the business and printing offices
of the "Metropolitan Mail" and the "We Forum newspapers. As a consequence of
the search and seizure, these premises were padlocked and sealed, with the further
result that the printing and publication of said newspapers were discontinued.

Such closure is in the nature of previous restraint or censorship abhorrent to the


freedom of the press guaranteed under the fundamental law, 18 and constitutes a
virtual denial of petitioners' freedom to express themselves in print. This state of
being is patently anathematic to a democratic framework where a free, alert and
even militant press is essential for the political enlightenment and growth of the
citizenry.

Respondents would justify the continued sealing of the printing machines on the
ground that they have been sequestered under Section 8 of Presidential Decree No.
885, as amended, which authorizes "the sequestration of the property of any
person, natural or artificial, engaged in subversive activities against the
government and its duly constituted authorities ... in accordance with implementing
rules and regulations as may be issued by the Secretary of National Defense." It is
doubtful however, if sequestration could validly be effected in view of the absence
of any implementing rules and regulations promulgated by the Minister of National
Defense.

Besides, in the December 10, 1982 issue of the Daily Express, it was reported that
no less than President Marcos himself denied the request of the military authorities
to sequester the property seized from petitioners on December 7, 1982. Thus:

The President denied a request flied by government prosecutors for


sequestration of the WE FORUM newspaper and its printing presses,
according to Information Minister Gregorio S. Cendana.

On the basis of court orders, government agents went to the We


Forum offices in Quezon City and took a detailed inventory of the
equipment and all materials in the premises.

Cendaña said that because of the denial the newspaper and its
equipment remain at the disposal of the owners, subject to the
discretion of the court. 19

That the property seized on December 7, 1982 had not been sequestered is further
confirmed by the reply of then Foreign Minister Carlos P. Romulo to the letter
dated February 10, 1983 of U.S. Congressman Tony P. Hall addressed to President
Marcos, expressing alarm over the "WE FORUM " case. 20 In this reply dated
February 11, 1983, Minister Romulo stated:

2. Contrary to reports, President Marcos turned down the


recommendation of our authorities to close the paper's printing
facilities and confiscate the equipment and materials it uses. 21

IN VIEW OF THE FOREGOING, Search Warrants Nos. 20-82[a] and 20-82[b]


issued by respondent judge on December 7, 1982 are hereby declared null and void
and are accordingly set aside. The prayer for a writ of mandatory injunction for the
return of the seized articles is hereby granted and all articles seized thereunder are
hereby ordered released to petitioners. No costs.

SO ORDERED.

Fernando, C.J., Teehankee, Makasiar, Concepcion, Jr., Melencio-Herrera, Plana,


Relova, Gutierrez, Jr., De la Fuente and Cuevas, JJ., concur.

Aquino, J., took no part.

 
ISSUE/S: 

WON there is probable cause for the issuance of the two search warrants
(NO)

RULING:

No.
Probable cause for a search is defined as such facts and circumstances which
would lead a reasonably discreet and prudent man to believe that an offense has
been committed and that the objects sought in connection with the offense are in
the place sought to be searched. 
And when the search warrant applied for is directed against a newspaper publisher
or editor in connection with the publication of subversive materials, as in the case
at bar, the application and/or its supporting affidavits must contain a
specification, stating with particularity the alleged subversive material he has
published or is intending to publish. Mere generalization will not suffice. 
Thus, the broad statement in Col. Abadilla's application that petitioner "is in
possession or has in his control printing equipment and other paraphernalia, news
publications and other documents which were used and are all continuously being
used as a means of committing the offense of subversion punishable under
Presidential Decree 885, as amended ..."  is a mere conclusion of law and does
not satisfy the requirements of probable cause. 
Bereft of such particulars as would justify a finding of the existence of
probable cause, said allegation cannot serve as basis for the issuance of a
search warrant and it was a grave error for respondent judge to have done so.
Equally insufficient as basis for the determination of probable cause is the
statement contained in the joint affidavit of Alejandro M. Gutierrez and Pedro U.
Tango, "that the evidence gathered and collated by our unit clearly shows that the
premises above- mentioned and the articles and things above-described were used
and are continuously being used for subversive activities in conspiracy with, and to
promote the objective of, illegal organizations such as the Light-a-Fire Movement,
Movement for Free Philippines, and April 6 Movement." 
In mandating that "no warrant shall issue except upon probable cause to be
determined by the judge, ... after examination under oath or affirmation of the
complainant and the witnesses he may produce; the Constitution requires no less
than personal knowledge by the complainant or his witnesses of the facts upon
which the issuance of a search warrant may be justified. 
In Alvarez v. Court of First Instance, this Court ruled that "the oath required must
refer to the truth of the facts within the personal knowledge of the petitioner or his
witnesses, because the purpose thereof is to convince the committing magistrate,
not the individual making the affidavit and seeking the issuance of the warrant, of
the existence of probable cause." As couched, the quoted averment in said joint
affidavit filed before respondent judge hardly meets the test of sufficiency
established by this Court in Alvarez case.
Another factor which makes the search warrants under consideration
constitutionally objectionable is that they are in the nature of general
warrants. The search warrants describe the articles sought to be seized in this
wise:
1] All printing equipment, paraphernalia, paper, ink, photo (equipment,
typewriters, cabinets, tables, communications/recording equipment, tape recorders,
dictaphone and the like used and/or connected in the printing of the "WE FORUM"
newspaper and any and all documents communication, letters and facsimile of
prints related to the "WE FORUM" newspaper.
2] Subversive documents, pamphlets, leaflets, books, and other publication to
promote the objectives and piurposes of the subversive organization known as
Movement for Free Philippines, Light-a-Fire Movement and April 6 Movement;
and,
3] Motor vehicles used in the distribution/circulation of the "WE FORUM" and
other subversive materials and propaganda, more particularly,
1] Toyota-Corolla, colored yellow with Plate No. NKA 892;
2] DATSUN pick-up colored white with Plate No. NKV 969
3] A delivery truck with Plate No. NBS 524;
4] TOYOTA-TAMARAW, colored white with Plate No. PBP 665; and,
5] TOYOTA Hi-Lux, pick-up truck with Plate No. NGV 427 with marking
"Bagong Silang."
As heretofore stated, the premises searched were the business and printing offices
of the "Metropolitan Mail" and the "We Forum newspapers. As a consequence of
the search and seizure, these premises were padlocked and sealed, with the further
result that the printing and publication of said newspapers were discontinued.
Such closure is in the nature of previous restraint or censorship abhorrent to
the freedom of the press guaranteed under the fundamental law, and
constitutes a virtual denial of petitioners' freedom to express themselves in
print. 
Respondents would justify the continued sealing of the printing machines on the
ground that they have been sequestered under Section 8 of Presidential Decree No.
885, as amended, which authorizes "the sequestration of the property of any
person, natural or artificial, engaged in subversive activities against the
government and its duly constituted authorities ... in accordance with implementing
rules and regulations as may be issued by the Secretary of National Defense." It is
doubtful however, if sequestration could validly be effected in view of the
absence of any implementing rules and regulations promulgated by the
Minister of National Defense.
Besides, in the December 10, 1982 issue of the Daily Express, it was reported that
no less than President Marcos himself denied the request of the military authorities
to sequester the property seized from petitioners on December 7, 1982.

DISPOSITIVE PORTION:

IN VIEW OF THE FOREGOING, Search Warrants Nos. 20-82[a] and 20-82[b]


issued by respondent judge on December 7, 1982 are hereby declared null and void
and are accordingly set aside. The prayer for a writ of mandatory injunction for the
return of the seized articles is hereby granted and all articles seized thereunder are
hereby ordered released to petitioners. No costs.

On Exclusionary Rule and Doctrine of the Fruit of the Poisonous Tree (Taken from
People vs. Encinada):
Sec. 2. The right of the people to be secure in their persons, houses, papers, and
effects against unreasonable searches and seizures of whatever nature and for any
purpose shall be inviolable, and no search warrant or warrant of arrest shall issue
except upon probable cause to be determined personally by the judge after
examination under oath or affirmation of the complainant and the witnesses he
may produce, and particularly describing the place to be searched and the persons
or things to be seized.
Any evidence obtained in violation of this provision is legally inadmissible in
evidence as a "fruit of the poisonous tree." This principle is covered by this
exclusionary rule:
Sec. 3. . . .
(2) Any evidence obtained in violation of . . . the preceding section shall be
inadmissible for any purpose in any proceeding.
The plain import of the foregoing provision is that a search and seizure is normally
unlawful unless authorized by a validly issued search warrant or warrant of arrest.
This protection is based on the principle that, between a citizen and the police, the
magistrate stands as a mediator, nay, an authority clothed with power to issue or
refuse to issue search warrants or warrants or arrest.

Microsoft Corp. v. Maxicorp, Inc., G.R. No. 140946, September 13, 2004
This petition for review on certiorari 1 seeks to reverse the Court of Appeals’
Decision2 dated 23 December 1998 and its Resolution dated 29 November 1999 in
CA-G.R. SP No. 44777. The Court of Appeals reversed the Order 3 of the Regional
Trial Court, Branch 23, Manila ("RTC"), denying respondent Maxicorp, Inc.’s
("Maxicorp") motion to quash the search warrant that the RTC issued against
Maxicorp. Petitioners are the private complainants against Maxicorp for copyright
infringement under Section 29 of Presidential Decree No. 49 ("Section 29 of PD
49")4 and for unfair competition under Article 189 of the Revised Penal Code
("RPC").5

Antecedent Facts

On 25 July 1996, National Bureau of Investigation ("NBI") Agent Dominador


Samiano, Jr. ("NBI Agent Samiano") filed several applications for search warrants
in the RTC against Maxicorp for alleged violation of Section 29 of PD 49 and
Article 189 of the RPC. After conducting a preliminary examination of the
applicant and his witnesses, Judge William M. Bayhon issued Search Warrants
Nos. 96-451, 96-452, 96-453 and 96-454, all dated 25 July 1996, against
Maxicorp.

Armed with the search warrants, NBI agents conducted on 25 July 1996 a search of
Maxicorp’s premises and seized property fitting the description stated in the search
warrants.

On 2 September 1996, Maxicorp filed a motion to quash the search warrants


alleging that there was no probable cause for their issuance and that the warrants
are in the form of "general warrants." The RTC denied Maxicorp’s motion on 22
January 1997. The RTC also denied Maxicorp’s motion for reconsideration.

The RTC found probable cause to issue the search warrants after examining NBI
Agent Samiano, John Benedict Sacriz ("Sacriz"), and computer technician
Felixberto Pante ("Pante"). The three testified on what they discovered during their
respective visits to Maxicorp. NBI Agent Samiano also presented certifications
from petitioners that they have not authorized Maxicorp to perform the witnessed
activities using petitioners’ products.

On 24 July 1997, Maxicorp filed a petition for certiorari with the Court of Appeals
seeking to set aside the RTC’s order. On 23 December 1998, the Court of Appeals
reversed the RTC’s order denying Maxicorp’s motion to quash the search warrants.
Petitioners moved for reconsideration. The Court of Appeals denied petitioners’
motion on 29 November 1999.

The Court of Appeals held that NBI Agent Samiano failed to present during the
preliminary examination conclusive evidence that Maxicorp produced or sold the
counterfeit products. The Court of Appeals pointed out that the sales receipt NBI
Agent Samiano presented as evidence that he bought the products from Maxicorp
was in the name of a certain "Joel Diaz."

Hence, this petition.

The Issues

Petitioners seek a reversal and raise the following issues for resolution:

1. WHETHER THE PETITION RAISES QUESTIONS OF LAW;

2. WHETHER PETITIONERS HAVE LEGAL PERSONALITY TO FILE


THE PETITION;

3. WHETHER THERE WAS PROBABLE CAUSE TO ISSUE THE


SEARCH WARRANTS;

4. WHETHER THE SEARCH WARRANTS ARE "GENERAL


WARRANTS."

The Ruling of the Court

The petition has merit.

On Whether the Petition Raises Questions of Law

Maxicorp assails this petition as defective since it failed to raise questions of law.
Maxicorp insists that the arguments petitioners presented are questions of fact,
which this Court should not consider in a Rule 45 petition for review. Petitioners
counter that all the issues they presented in this petition involve questions of law.
Petitioners point out that the facts are not in dispute.

A petition for review under Rule 45 of the Rules of Court should cover questions
of law.6 Questions of fact are not reviewable. As a rule, the findings of fact of the
Court of Appeals are final and conclusive and this Court will not review them on
appeal,7 subject to exceptions as when the findings of the appellate court conflict
with the findings of the trial court.8

The distinction between questions of law and questions of fact is settled. A


question of law exists when the doubt or difference centers on what the law is on a
certain state of facts. A question of fact exists if the doubt centers on the truth or
falsity of the alleged facts. Though this delineation seems simple, determining the
true nature and extent of the distinction is sometimes problematic. For example, it
is incorrect to presume that all cases where the facts are not in dispute
automatically involve purely questions of law.
There is a question of law if the issue raised is capable of being resolved without
need of reviewing the probative value of the evidence. 9 The resolution of the issue
must rest solely on what the law provides on the given set of circumstances. Once
it is clear that the issue invites a review of the evidence presented, the question
posed is one of fact.10 If the query requires a re-evaluation of the credibility of
witnesses, or the existence or relevance of surrounding circumstances and their
relation to each other, the issue in that query is factual. 11 Our ruling in Paterno v.
Paterno12 is illustrative on this point:

Such questions as whether certain items of evidence should be accorded


probative value or weight, or rejected as feeble or spurious, or whether or
not the proofs on one side or the other are clear and convincing and adequate
to establish a proposition in issue, are without doubt questions of fact.
Whether or not the body of proofs presented by a party, weighed and
analyzed in relation to contrary evidence submitted by adverse party, may be
said to be strong, clear and convincing; whether or not certain documents
presented by one side should be accorded full faith and credit in the face of
protests as to their spurious character by the other side; whether or not
inconsistencies in the body of proofs of a party are of such gravity as to
justify refusing to give said proofs weight – all these are issues of fact.

It is true that Maxicorp did not contest the facts alleged by petitioners. But
this situation does not automatically transform all issues raised in the
petition into questions of law. The issues must meet the tests outlined in
Paterno.

Of the three main issues raised in this petition – the legal personality of the
petitioners, the nature of the warrants issued and the presence of probable
cause – only the first two qualify as questions of law. The pivotal issue of
whether there was probable cause to issue the search warrants is a question
of fact. At first glance, this issue appears to involve a question of law since
it does not concern itself with the truth or falsity of certain facts. Still, the
resolution of this issue would require this Court to inquire into the probative
value of the evidence presented before the RTC. For a question to be one of
law, it must not involve an examination of the probative value of the
evidence presented by the litigants or any of them. 13

Yet, this is precisely what the petitioners ask us to do by raising arguments


requiring an examination of the TSNs and the documentary evidence presented
during the search warrant proceedings. In short, petitioners would have us
substitute our own judgment to that of the RTC and the Court of Appeals by
conducting our own evaluation of the evidence. This is exactly the situation which
Section 1, Rule 45 of the Rules of Court prohibits by requiring the petition to raise
only questions of law. This Court is not a trier of facts. It is not the function of this
court to analyze or weigh evidence.14 When we give due course to such situations,
it is solely by way of exception. Such exceptions apply only in the presence of
extremely meritorious circumstances.15

Indeed, this case falls under one of the exceptions because the findings of the Court
of Appeals conflict with the findings of the RTC. 16 Since petitioners properly
raised the conflicting findings of the lower courts, it is proper for this Court to
resolve such contradiction.

On Whether Petitioners have the Legal Personality to File this Petition

Maxicorp argues that petitioners have no legal personality to file this petition since
the proper party to do so in a criminal case is the Office of the Solicitor General as
representative of the People of the Philippines. Maxicorp states the general rule but
the exception governs this case.17 We ruled in Columbia Pictures Entertainment,
Inc. v. Court of Appeals18 that the petitioner-complainant in a petition for review
under Rule 45 could argue its case before this Court in lieu of the Solicitor General
if there is grave error committed by the lower court or lack of due process. This
avoids a situation where a complainant who actively participated in the prosecution
of a case would suddenly find itself powerless to pursue a remedy due to
circumstances beyond its control. The circumstances in Columbia Pictures
Entertainment are sufficiently similar to the present case to warrant the
application of this doctrine.

On Whether there was Probable Cause to Issue the Search Warrants

Petitioners argue that the Court of Appeals erred in reversing the RTC based on the
fact that the sales receipt was not in the name of NBI Agent Samiano. Petitioners
point out that the Court of Appeals disregarded the overwhelming evidence that the
RTC considered in determining the existence of probable cause. Maxicorp counters
that the Court of Appeals did not err in reversing the RTC. Maxicorp maintains
that the entire preliminary examination that the RTC conducted was defective.

The Court of Appeals based its reversal on two factual findings of the RTC. First,
the fact that the sales receipt presented by NBI Agent Samiano as proof that he
bought counterfeit goods from Maxicorp was in the name of a certain "Joel Diaz."
Second, the fact that petitioners’ other witness, John Benedict Sacriz, admitted that
he did not buy counterfeit goods from Maxicorp.

We rule that the Court of Appeals erred in reversing the RTC’s findings.

Probable cause means "such reasons, supported by facts and circumstances as will
warrant a cautious man in the belief that his action and the means taken in
prosecuting it are legally just and proper."19 Thus, probable cause for a search
warrant requires such facts and circumstances that would lead a reasonably prudent
man to believe that an offense has been committed and the objects sought in
connection with that offense are in the place to be searched. 20

The judge determining probable cause must do so only after personally examining
under oath the complainant and his witnesses. The oath required must refer to "the
truth of the facts within the personal knowledge of the petitioner or his witnesses,
because the purpose thereof is to convince the committing magistrate, not the
individual making the affidavit and seeking the issuance of the warrant, of the
existence of probable cause."21 The applicant must have personal knowledge of the
circumstances. "Reliable information" is insufficient. 22 Mere affidavits are not
enough, and the judge must depose in writing the complainant and his witnesses. 23

The Court of Appeals’ reversal of the findings of the RTC centers on the fact that
the two witnesses for petitioners during the preliminary examination failed to
prove conclusively that they bought counterfeit software from Maxicorp. The
Court of Appeals ruled that this amounted to a failure to prove the existence of a
connection between the offense charged and the place searched.

The offense charged against Maxicorp is copyright infringement under Section 29


of PD 49 and unfair competition under Article 189 of the RPC. To support these
charges, petitioners presented the testimonies of NBI Agent Samiano, computer
technician Pante, and Sacriz, a civilian. The offenses that petitioners charged
Maxicorp contemplate several overt acts. The sale of counterfeit products is but
one of these acts. Both NBI Agent Samiano and Sacriz related to the RTC how
they personally saw Maxicorp commit acts of infringement and unfair competition.

During the preliminary examination, the RTC subjected the testimonies of the
witnesses to the requisite examination. NBI Agent Samiano testified that he saw
Maxicorp display and offer for sale counterfeit software in its premises. He also
saw how the counterfeit software were produced and packaged within Maxicorp’s
premises. NBI Agent Samiano categorically stated that he was certain the products
were counterfeit because Maxicorp sold them to its customers without giving the
accompanying ownership manuals, license agreements and certificates of
authenticity.

Sacriz testified that during his visits to Maxicorp, he witnessed several instances
when Maxicorp installed petitioners’ software into computers it had assembled.
Sacriz also testified that he saw the sale of petitioners’ software within Maxicorp’s
premises. Petitioners never authorized Maxicorp to install or sell their software.

The testimonies of these two witnesses, coupled with the object and documentary
evidence they presented, are sufficient to establish the existence of probable cause.
From what they have witnessed, there is reason to believe that Maxicorp engaged
in copyright infringement and unfair competition to the prejudice of petitioners.
Both NBI Agent Samiano and Sacriz were clear and insistent that the counterfeit
software were not only displayed and sold within Maxicorp’s premises, they were
also produced, packaged and in some cases, installed there.

The determination of probable cause does not call for the application of rules and
standards of proof that a judgment of conviction requires after trial on the merits.
As implied by the words themselves, "probable cause" is concerned with
probability, not absolute or even moral certainty. The prosecution need not present
at this stage proof beyond reasonable doubt. The standards of judgment are those
of a reasonably prudent man,24 not the exacting calibrations of a judge after a full-
blown trial.

No law or rule states that probable cause requires a specific kind of evidence. No
formula or fixed rule for its determination exists. 25 Probable cause is determined in
the light of conditions obtaining in a given situation. 26 Thus, it was improper for
the Court of Appeals to reverse the RTC’s findings simply because the sales
receipt evidencing NBI Agent Samiano’s purchase of counterfeit goods is not in
his name.

For purposes of determining probable cause, the sales receipt is not the only proof
that the sale of petitioners’ software occurred. During the search warrant
application proceedings, NBI Agent Samiano presented to the judge the computer
unit that he purchased from Maxicorp, in which computer unit Maxicorp had pre-
installed petitioners’ software.27 Sacriz, who was present when NBI Agent
Samiano purchased the computer unit, affirmed that NBI Agent Samiano
purchased the computer unit.28 Pante, the computer technician, demonstrated to the
judge the presence of petitioners’ software on the same computer unit. 29 There was
a comparison between petitioners’ genuine software and Maxicorp’s software pre-
installed in the computer unit that NBI Agent Sambiano purchased. 30 Even if we
disregard the sales receipt issued in the name of "Joel Diaz," which petitioners
explained was the alias NBI Agent Samiano used in the operation, there still
remains more than sufficient evidence to establish probable cause for the issuance
of the search warrants.

This also applies to the Court of Appeals’ ruling on Sacriz’s testimony. The fact
that Sacriz did not actually purchase counterfeit software from Maxicorp does not
eliminate the existence of probable cause. Copyright infringement and unfair
competition are not limited to the act of selling counterfeit goods. They cover a
whole range of acts, from copying, assembling, packaging to marketing, including
the mere offering for sale of the counterfeit goods. The clear and firm testimonies
of petitioners’ witnesses on such other acts stand untarnished. The Constitution and
the Rules of Court only require that the judge examine personally and thoroughly
the applicant for the warrant and his witnesses to determine probable cause. The
RTC complied adequately with the requirement of the Constitution and the Rules
of Court.
Probable cause is dependent largely on the opinion and findings of the judge who
conducted the examination and who had the opportunity to question the applicant
and his witnesses.31 For this reason, the findings of the judge deserve great weight.
The reviewing court should overturn such findings only upon proof that the judge
disregarded the facts before him or ignored the clear dictates of reason. 32 Nothing
in the records of the preliminary examination proceedings reveal any impropriety
on the part of the judge in this case. As one can readily see, here the judge
examined thoroughly the applicant and his witnesses. To demand a higher degree
of proof is unnecessary and untimely. The prosecution would be placed in a
compromising situation if it were required to present all its evidence at such
preliminary stage. Proof beyond reasonable doubt is best left for trial.

On Whether the Search Warrants are in the Nature of General Warrants

A search warrant must state particularly the place to be searched and the objects to
be seized. The evident purpose for this requirement is to limit the articles to be
seized only to those particularly described in the search warrant. This is a
protection against potential abuse. It is necessary to leave the officers of the law
with no discretion regarding what articles they shall seize, to the end that no
unreasonable searches and seizures be committed. 33

In addition, under Section 4, Rule 126 of the Rules of Criminal Procedure, a search
warrant shall issue "in connection with one specific offense." The articles
described must bear a direct relation to the offense for which the warrant is
issued.34 Thus, this rule requires that the warrant must state that the articles subject
of the search and seizure are used or intended for use in the commission of a
specific offense.

Maxicorp argues that the warrants issued against it are too broad in scope and lack
the specificity required with respect to the objects to be seized. After examining the
wording of the warrants issued, the Court of Appeals ruled in favor of Maxicorp
and reversed the RTC’s Order thus:

Under the foregoing language, almost any item in the petitioner’s store can
be seized on the ground that it is "used or intended to be used" in the illegal
or unauthorized copying or reproduction of the private respondents’
software and their manuals.35

The Court of Appeals based its reversal on its perceived infirmity of paragraph (e)
of the search warrants the RTC issued. The appellate court found that similarly
worded warrants, all of which noticeably employ the phrase "used or intended to
be used," were previously held void by this Court. 36 The disputed text of the search
warrants in this case states:
a) Complete or partially complete reproductions or copies of Microsoft
software bearing the Microsoft copyrights and/or trademarks owned by
MICROSOFT CORPORATION contained in CD-ROMs, diskettes and hard
disks;

b) Complete or partially complete reproductions or copies of Microsoft


instruction manuals and/or literature bearing the Microsoft copyrights and/or
trademarks owned by MICROSOFT CORPORATION;

c) Sundry items such as labels, boxes, prints, packages, wrappers,


receptacles, advertisements and other paraphernalia bearing the copyrights
and/or trademarks owned by MICROSOFT CORPORATION;

d) Sales invoices, delivery receipts, official receipts, ledgers, journals,


purchase orders and all other books of accounts and documents used in the
recording of the reproduction and/or assembly, distribution and sales, and
other transactions in connection with fake or counterfeit products bearing
the Microsoft copyrights and/or trademarks owned by MICROSOFT
CORPORATION;

e) Computer hardware, including central processing units including


hard disks, CD-ROM drives, keyboards, monitor screens and diskettes,
photocopying machines and other equipment or paraphernalia used or
intended to be used in the illegal and unauthorized copying or
reproduction of Microsoft software and their manuals, or which
contain, display or otherwise exhibit, without the authority of
MICROSOFT CORPORATION, any and all Microsoft trademarks
and copyrights; and

f) Documents relating to any passwords or protocols in order to access all


computer hard drives, data bases and other information storage devices
containing unauthorized Microsoft software. 37 (Emphasis supplied)

It is only required that a search warrant be specific as far as the circumstances will
ordinarily allow.38 The description of the property to be seized need not be
technically accurate or precise. The nature of the description should vary according
to whether the identity of the property or its character is a matter of
concern.39 Measured against this standard we find that paragraph (e) is not a
general warrant. The articles to be seized were not only sufficiently identified
physically, they were also specifically identified by stating their relation to the
offense charged. Paragraph (e) specifically refers to those articles used or intended
for use in the illegal and unauthorized copying of petitioners’ software. This
language meets the test of specificity. 40
The cases cited by the Court of Appeals are inapplicable. In those cases, the Court
found the warrants too broad because of particular circumstances, not because of
the mere use of the phrase "used or intended to be used." In Columbia Pictures,
Inc. v. Flores, the warrants ordering the seizure of "television sets, video cassette
recorders, rewinders and tape cleaners x x x" were found too broad since the
defendant there was a licensed distributor of video tapes. 41 The mere presence of
counterfeit video tapes in the defendant’s store does not mean that the machines
were used to produce the counterfeit tapes. The situation in this case is different.
Maxicorp is not a licensed distributor of petitioners. In Bache & Co. (Phil.), Inc.,
et al. v. Judge Ruiz, et al., the Court voided the warrants because they authorized
the seizure of records pertaining to "all business transactions" of the
defendant.42 And in 20th Century Fox Film Corp. v. Court of Appeals, the Court
quashed the warrant because it merely gave a list of articles to be seized,
aggravated by the fact that such appliances are "generally connected with the
legitimate business of renting out betamax tapes." 43

However, we find paragraph (c) of the search warrants lacking in particularity.


Paragraph (c) states:

c) Sundry items such as labels, boxes, prints, packages, wrappers,


receptacles, advertisements and other paraphernalia bearing the copyrights
and/or trademarks owned by MICROSOFT CORPORATION;

The scope of this description is all-embracing since it covers property used for
personal or other purposes not related to copyright infringement or unfair
competition. Moreover, the description covers property that Maxicorp may have
bought legitimately from Microsoft or its licensed distributors. Paragraph (c)
simply calls for the seizure of all items bearing the Microsoft logo, whether
legitimately possessed or not. Neither does it limit the seizure to products used in
copyright infringement or unfair competition.

Still, no provision of law exists which requires that a warrant, partially defective in
specifying some items sought to be seized yet particular with respect to the other
items, should be nullified as a whole. A partially defective warrant remains valid as
to the items specifically described in the warrant. 44 A search warrant is severable,
the items not sufficiently described may be cut off without destroying the whole
warrant.45 The exclusionary rule found in Section 3(2) of Article III of the
Constitution renders inadmissible in any proceeding all evidence obtained through
unreasonable searches and seizure. Thus, all items seized under paragraph (c) of
the search warrants, not falling under paragraphs a, b, d, e or f, should be returned
to Maxicorp.

WHEREFORE, we PARTIALLY GRANT the instant petition. The Decision of


the Court of Appeals dated 23 December 1998 and its Resolution dated 29
November 1999 in CA-G.R. SP No. 44777 are REVERSED and SET
ASIDE except with respect to articles seized under paragraph (c) of Search
Warrants Nos. 96-451, 96-452, 96-453 and 96-454. All articles seized under
paragraph (c) of the search warrants, not falling under paragraphs a, b, d, e or f, are
ordered returned to Maxicorp, Inc. immediately.

SO ORDERED.

ISSUES:
1. WHETHER THERE WAS PROBABLE CAUSE TO ISSUE THE
SEARCH WARRANTS (YES)
2. WHETHER THE SEARCH WARRANTS ARE "GENERAL
WARRANTS" (NO)

RULING:
WHEREFORE, we PARTIALLY GRANT the instant petition. The Decision of
the Court of Appeals dated 23 December 1998 and its Resolution dated 29
November 1999 in CA-G.R. SP No. 44777 are REVERSED and SET ASIDE
except with respect to articles seized under paragraph (c) of Search Warrants Nos.
96-451, 96-452, 96-453 and 96-454. All articles seized under paragraph (c) of the
search warrants, not falling under paragraphs a, b, d, e or f, are ordered returned to
Maxicorp, Inc. immediately.

RATIO:
1. The testimonies of these two witnesses, coupled with the object and
documentary evidence they presented, are sufficient to establish the
existence of probable cause. From what they have witnessed, there is reason
to believe that Maxicorp engaged in copyright infringement and unfair
competition to the prejudice of petitioners. Both NBI Agent Samiano and
Sacriz were clear and insistent that the counterfeit software were not only
displayed and sold within Maxicorp’s premises, they were also produced,
packaged and in some cases, installed there.
The determination of probable cause does not call for the application of
rules and standards of proof that a judgment of conviction requires after trial
on the merits. As implied by the words themselves, "probable cause" is
concerned with probability, not absolute or even moral certainty. The
prosecution need not present at this stage proof beyond reasonable doubt.
The standards of judgment are those of a reasonably prudent man, not the
exacting calibrations of a judge after a full-blown trial.
No law or rule states that probable cause requires a specific kind of
evidence. No formula or fixed rule for its determination exists. Probable
cause is determined in the light of conditions obtaining in a given
situation.Thus, it was improper for the Court of Appeals to reverse the
RTC’s findings simply because the sales receipt evidencing NBI Agent
Samiano’s purchase of counterfeit goods is not in his name.

2. e) Computer hardware, including central processing units including hard


disks, CD-ROM drives, keyboards, monitor screens and diskettes,
photocopying machines and other equipment or paraphernalia used or
intended to be used in the illegal and unauthorized copying or
reproduction of Microsoft software and their manuals, or which contain,
display or otherwise exhibit, without the authority of MICROSOFT
CORPORATION, any and all Microsoft trademarks and copyrights

It is only required that a search warrant be specific as far as the


circumstances will ordinarily allow. The description of the property to be
seized need not be technically accurate or precise. The nature of the
description should vary according to whether the identity of the property or
its character is a matter of concern. Measured against this standard we find
that paragraph (e) is not a general warrant. The articles to be seized were not
only sufficiently identified physically, they were also specifically identified
by stating their relation to the offense charged. Paragraph (e) specifically
refers to those articles used or intended for use in the illegal and
unauthorized copying of petitioners’ software. This language meets the test
of specificity. 

Castro v. Pabalan, G.R. No. L-28642, April 30, 1976

MARIA CASTRO and CO LING petitioners,


vs.
HONORABLE JAVIER PABALAN, Judge of the Court of First Instance of
La Union, and SGT. ERNESTO LUMANG, respondents.

Marcelino B. Florentino for petitioners.

Jose L Yumang for respondent Sgt. Ernesto Lumang. Hon. Javier Pabalan for and
in his own behalf.

FERNANDO, J.:

This Court is confronted anew in this certiorari proceeding with the claim that a


search warrant issued without complying with the requisites of the
Constitution 1 and the Rules of Court 2 should have been nullified, but was not in
the challenged order of respondent Judge Javier Pabalan. 3 More specifically, it
was the failure of the application for the search warrant as well as the search
warrant itself to specify the specific offense, to examine the applicant as well as his
witnesses on the part of respondent Judge, and to describe with particularity the
place to be searched and the things to be seized, that were singled out to justify the
assertion of illegality. When required to answer, respondent Judge did not bother to
refute specifically the allegations of the petition for certiorari, but merely contented
himself with inviting attention to the challenged order as well as the resolutions
denying the motion for reconsideration and with the statement that he "has no
particular prayer to ask the Supreme Court," an assertion thereafter repeated in the
second paragraph of his two-paragraph answer that he "has no request to make in
this particular case leaving the issues entirely to the discretion of the Supreme
Court." 4 The tone of diffidence, almost of apology, is easy to understand. It is
difficult to resist the thought that respondent Judge failed to pay heed to
authoritative decisions of this Court. The most cursory perusal of the application
for search warrant 5 by respondent Lumang and the search warrant itself, 6 yields
no other conclusion. Respondent Judge ignored what the Constitution requires on
two points, the existence of a probable cause and the particular description of the
things to be seized. The limitation as to the specific offense as mandated by the
Rules of Court was not observed either. Even on the assumption then that he could
not

relevant According to the former: "A search warrant shall not issue but upon
probable cause in connection with one specific offense to be determined by the
municipal or city judge after examination under oath or affirmation of the
complainant and the witnesses he may produce, and particularly describing the
place to be searched and the persons or things to be seized. No search warrant shall
issue for more than one specific offense." Section 4 provides: "The municipal or
city judge must, before issuing the warrant, personally examine on oath or
affirmation the complainant and any witnesses he may produce and take their
depositions in writing, and attach them to the record, in addition to any affidavits
presented to him." be held chargeable with knowledge of the leading Stonehill
decision, 7 announced barely twenty days before the search warrant in question was
issued, still from Alverez v. Court of First Instance 8 the first to be decided under
the 1935 Constitution, promulgated in 1937, to Oca v. Marquez, 9 that came out in
1965, this Court had adhered firmly to the view that for a search warrant to escape
the imputation of being unreasonable, there should be strict conformity with the
requirements of the Constitution and the applicable procedural rules. The finding
then should have been against the validity of the search warrant. Nonetheless,
insofar as such order limited itself to requiring the return solely of the liquor, the
pack of playing cards, the bottle of distilled water and five bottles of Streptomycin,
all of which may be considered as personal effects of petitioners, with the rest of
the goods taken falling under the category of things forbidden by law and therefore
need not be restored, 10 it can be sustained. So we rule.
In the opening paragraph of the application for search warrant, respondent Ernesto
I. Lumang admitted that "he has been informed" and therefore was of the belief
that petitioners Maria Castro and Co Ling, whose place of residence was not even
indicated, although subsequently mention was made of their being at Barrio
Padasil, Bangar, La Union, "have in possession narcotics and other
contraband." 11 There is a claim that he had verified the report and that therefore he
had "reasons to believe that a Search Warrant should be issued to enable the
undersigned to take possession" of such narcotics and other contraband. 12 The
application was accompanied by the joint affidavit of a Sergeant Francisco C.
Molina and a Corporal Lorenzo G. Apilado of the Philippine
Constabulary. 13 Again, mention was merely made of their information about
narcotics and other contraband being kept by petitioners. They did allege therein
that they conducted rigid surveillance, but all they could come out with is that
petitioner Co Ling is an overstaying alien for almost ten years conducting such
traffic and that after verification, he was not registered in the Immigration
Office. 14 Then, on the very same day, July 10, 1967, the search warrant was issued
for illegal traffic of narcotics and contraband. 15 Again, there was reference to the
possession by petitioners of such forbidden goods. As to the complete and detailed
description of the properties to be seized, the search warrant merely mentioned
illegal traffic of narcotics and contraband inside the warehouse and premises of
petitioners. 16 In the resolution upholding the validity of the search warrant,
respondent Judge did state the following: "On July 10, 1967, Ernesto Lumang, Sgt.
of the PC, with a long service behind, appeared in chamber before the Presiding
Judge of Branch I of this Court. With him were Sgt. Molina and Cpl. Apilado both
of the PC Command of La Union. The three submitted to the Presiding Judge in
chamber an application for search warrant which is Exhibit I in this case and a joint
affidavit supporting the search warrant asked. As Sgt. Lumang said, testifying
regarding this incident, those appearing were asked, although not in writing and
not recorded, some questions by the Presiding Judge regarding their request of the
search warrant on the knowledge of Molina and Apilado on the facts stated on the
application and on the joint affidavit. The inquiry was brief. The barrio to be
searched was handwritten in ink, Maria Cristina cancelling the typewritten name
Padasil. But this correction was not done in the duplicates. Anyhow Padasil and
Maria Cristina are adjoining barrios. After the routine taking of their oath and
examination questions and answers, the Presiding Judge of this Branch signed the
application for search warrant, the joint affidavits, and forthwith issued the search
warrant which is Exhibit C." 17

As set forth at the outset, failure to abide by both the Constitution and the
procedural law in terms of the existence of a probable cause, a particular
description of the property to be seized and the requirement that there be only one
specific offense, is quite manifest.

1. This excerpt from the epochal opinion of former Chief Justice Concepcion in
Stonehill v. Diokno 18 is highly relevant: "Two points must be stressed in
connection with — this constitutional mandate, namely: (1) that no warrant shall
issue but upon probable cause, to be determined by the judge in the manner set
forth in said provision; and (2) that the warrant shall particularly describe the
things to be seized. None of these requirements has been complied with in the
contested warrants. Indeed, the same were issued upon applications stating that the
natural and juridical persons therein named had committed a 'violation of Central
Bank Laws, Tariff and Customs Laws, Internal Revenue (Code) and Revised Penal
Code.' In other words, no specific offense had been alleged in said applications.
The averments thereof with respect to the offense committed were abstract. As a
consequence, it was impossible for the judges who issued the warrants to have
found the existence of probable cause, for the same presupposes the introduction of
competent proof that the party against whom it is sought has
performed particular acts, or committed specific omissions, violating a given
provision of our criminal laws. As a matter of fact, the applications involved in this
case do not allege any specific acts performed by herein petitioners. It would be a
legal heresy, of the highest order, to convict anybody of a 'violation of Central
Bank Laws, Tariff and Customs Laws, Internal Revenue (Code) and Revised Penal
Code,' — as alleged in the aforementioned applications — without reference to any
determinate provision of said laws or codes." 19 That same approach is reflected in
the two subsequent cases of Bache & Co. (Phil.), Inc. v. Ruiz  20 and Asian Surety
& Insurance Co., Inc. v. Herrera. 21 It bears repeating, as was emphasized in
Stonehill v. Diokno, that the averments as to the alleged commission of the
offenses imputed to petitioner were abstract. As admitted in the challenged order,
the inquiry was brief. Subsequently, reference was made to "the routine taking of
[their oath] and examination questions and answers ..." 22 Nor can such perfunctory
manner in which respondent Judge conducted the required "examination under
oath" be justified merely because respondent Lumang was "a Sergeant of the PC,
with a long service behind [him]." 23 Moreover, contrary to the Rules of Court, he
did not even bother to take the depositions of the witnesses in writing, attaching
them to the record. 24 There was thus a manifest and palpable violation of the
constitutional standard as to the quantum of proof to show the existence of
probable cause, as so clearly enunciated in Stonehill.

2. Then again, the Constitution requires, for the validity of a search warrant, that
there be a particular description of "the place to be searched and the persons or
things to be seized." 25 As was admitted by the judge in the challenged resolution,
there was a mistake concerning the residence of petitioners, which was set forth in
the search warrant as being in Barrio Padasil when in fact it is in Barrio Maria
Cristina. He would gloss over such inaccuracy by saying that they were, anyway,
adjoining barrios. As to the premises to be searched, it may be admitted that the
deficiency in the writ is not of sufficient gravity to call for its invalidation.
Nonetheless, and again in line with Stonehill v. Diokno, the Constitution is quite
explicit that there be a particular description of the things to be seized. That
requisite was not complied with in this case. That would explain why the searching
party felt it had a free hand and did take possession of various kinds of goods,
including personal effects, which respondent Judge himself would have them
return. What was aptly characterized as a "major objective" of this constitutional
provision, the elimination of general warrants, was thus frustrated. It need not be
stressed anew that this Court is resolutely committed to the doctrine that this
constitutional provision is of a mandatory character and therefore must be strictly
complied with. 26 To quote from the landmark American decision of Boyd v.
United States: 27 "It is the duty of courts to be watchful for the constitutional rights
of the citizen, and against any stealthy encroachments thereon. Their motto should
be obsta principis." 28

3. Another infirmity was the failure to comply with the basic procedural requisite
that a search warrant "shall not issue but upon probable cause in connection with
one specific offense." 29 Here reference was made to "an illegal traffic of narcotics
and contraband." The latter is a generic term covering all goods exported from or
imported into the country contrary to applicable statutes. Necessarily then, more
than one offense could arise from the activity designated as illegal traffic of
narcotics and contraband. As a matter of fact, in the challenged order, reference
was made to at least three charges having been filed, the violation of Section 203
of the Internal Revenue Code, its Section 1039 on tax evasion, as well as illegal
possession of opium. It would seem that once again what was correctly pointed out
by Chief Justice Concepcion in Stonehill v. Diokno as unjustified and unwarranted
finds application. Nor can there be any plausibility to the possible excuse, to repeat
what was said before, that the Stonehill opinion having been rendered only twenty
days previous to the issuance of the search warrant, respondent Judge could not be
held chargeable with a knowledge thereof, considering that as far back as July 30,
1965, two years earlier, in Oca v. Marquez, 30 this Court, through the then Justice J.
P. Bengzon, enunciated: "The decision herein has applied the provisions of th Old
Rules of Court since this case arose under said Rules. Attention of the Bench and
Bar is however called to the fact that effective January 1, 1964 the issuance of
search warrants is governed by Section 3, Rule 126 of the Revised Rules of Court
which among other things requires that a search warrant must be in connection
with one specific offense." 31

4. As was made clear at the outset, though, the illegality of the search warrant does
not call for the return of the things seized, the possession of which is prohibited by
law. This is the established doctrine in this jurisdiction. As far back as Uy Kheytin
v. Villareal, 32 a 1920 decision, it was held: "That although in the issuance of the
search warrant in question the judge did not comply with the requirements of
section 98 of General Orders No. 58, the petitioners are not entitled to the return of
the opium. and its paraphernalia which were found and seized under said warrant,
and much less are they entitled to be exonerated because of such omission of the
judge." 33 Among the authorities cited is Cooley: "'Search-warrants have heretofore
been allowed to search for stolen goods, for goods supposed to have been
smuggled into the country in violation of the revenue laws, for implements of
gaming or counterfeiting, for lottery tickets or Prohibited liquors kept for sale
contrary to law, for obscene books and papers kept for sale or circulation, and for
powder or other explosive and dangerous material so kept as to endanger the public
safety.'" 34 So, also, in Yee Sue Koy v. Almeda, 35 handed down in 1940, Justice
Laurel, speaking for this Court, stated: "If it be true, furthermore, without,
however, deciding the point, that as alleged by the respondents the articles in
question constitute the corpus delicti of the Usury Law, their return to the
petitioners cannot be ordered." 36 Magoncia v. Palacios, 37 promulgated in 1948,
reiterated such a doctrine. Thus: "En el asunto de Uy Kheytin contra Villareal (42
Jur. Fil. 935), los recurrentes pidieron la devolucion del opio de que se incautaron
los constabularies al registrar su casa armados con un mandamiento de registro
expedido sin cumplir las disposiciones de los articulos 96 y 98 de la Orden General
No. 58; sostenian que los requisites exigidos por dichos articulos no se habian
cumplido, y por tanto, el mandamiento de registro era ilegal, como si no existiera;
que al registro se ha hecho sin mandamiento de registro debidamente expedido.
Este Tribunal denego la peticion, declarando que la irregularidad de la expedicion
del mandamiento de registro ne era suficiente causa para ordenar la devolucion del
opio. El Hon. Juez recurrido no abuso de su discrecion al denegar la devolucion al
acusado del paltik, 42 municiones y una granada de mano, tampoco abuso de su
sana discrecion al denegar la peticion del acusado de que se prohiba al Fiscal
Provincial y al Jefe de Policia de Asingan, Pangasinan a presentar tales efectos
como prueba en la vista." 38

5. This decision leaves open the question of the legality of any possible use that
may be made by the prosecuting authorities of the articles seized under an invalid
search warrant. Here, again, the Yee Sue Koy opinion of Justice Laurel is
illuminating, especially in view of the inadmissibility of evidence illegally seized
under the present Constitution 39 At this stage, the question does not have to be
faced. The words of Justice Laurel follow: "While we reiterate the rule that the
seizure of books and documents by means of a search warrant ' for the purpose of
using them as evidence in a criminal case against the person in whose possession
they were found is unconstitutional because it makes the warrant unreasonable, and
it is equivalent to a violation of the constitutional provision prohibiting the
compulsion of an accused to testify against himself ..., the said rule has no
applicable force in the present case. ... In the application for the issuance of the
search warrant in question, it was alleged that the articles seized were 'being used
by it (Sam Sing & Co.) in connection with its activities of lending money at
usurious rate of interest in violation of the Usury Law,' and it is now suggested
(memoranda of respondents) that the only object of the agents of the Anti-Usury
Board in keeping the articles is to prevent the petitioners from employing them as a
means of further violations of the Usury Law. In this state of the record, without
deciding the question whether the petitioners will in fact use the articles in
question, if returned, for illegal purposes, we are not prepared to order the return
prayed for by the petitioners. (Cf. People v. Rubio, 57 Phil. 384, 394-395.)" 40
WHEREFORE, the writ of certiorari is granted and the order of September 12,
1967 denying the motion of petitioners to annul the search warrant as well as the
resolutions of October 26, 1967 and January 29, 1968 denying the motions for
reconsiderations are reversed, the decision of this Court being that the search
warrant in question is tainted by illegality for being violative both of the
Constitution and the Rules of Court. It is likewise the decision of this Court that
notwithstanding the illegality of such search warrant, the challenged order of
respondent Judge can be sustained only insofar as it would limit the return of the
articles seized to the liquor, the pack of playing cards, the bottle of distilled water
and five bottles of Streptomycin taken under such search warrant. No costs.

Barred, Antonio, Aquino and Concepcion, Jr., JJ., concur.

Asian Surety v. Herrera, G.R. No. L-25232, December 20, 1973

ESGUERRA, J.:

Petition to quash and annul a search warrant issued by respondent Judge Jose
Herrera of the City Court of Manila, and to command respondents to return
immediately the documents, papers, receipts and records alleged to have been
illegally seized thereunder by agents of the National Bureau of Investigation (NBI)
led by respondent Celso Zoleta, Jr.

On October 27, 1965, respondent Judge Herrera, upon the sworn application of
NBI agent Celso Zoleta, Jr. supported by the deposition of his witness, Manuel
Cuaresma, issued a search warrant in connection with an undocketed criminal case
for estafa, falsification, insurance fraud, and tax evasion, against the Asian Surety
and Insurance Co., a corporation duly organized and existing under the laws of the
Philippines, with principal office at Room 200 Republic Supermarket Bldg., Rizal
Avenue, Manila. The search warrant is couched in the following
language:jgc:chanrobles.com.ph

"It appearing to the satisfaction of the undersigned, after examining under oath
NBI Agent Celso J. Zoleta, Jr. and his witness Manuel Cuaresma that there are
good and sufficient reasons to believe that Mr. William Li Yao or his employees
has/have in his/their control in premises No. 2nd Floor Republic Supermarket
Building, in Rizal Avenue district of Sta. Cruz, Manila, property (Subject of the
offense; stolen or embezzled and proceeds or fruits of the offense used or intended
to be used as the means of committing the offense) should be seized and brought to
the undersigned.

"You are hereby commanded to make an immediate search at any time in the
_____ of the premises above-described and forthwith seize and take possession of
the following personal property to wit: Fire Registers, Loss Bordereau, Adjusters
Report including subrogation receipt and proof of loss, Loss Registers, Books of
Accounts, including cash receipts and disbursements and general ledger, check
vouchers, income tax returns, and other papers connected therewith . . . for the
years 1961 to 1964 to be dealt with as the law directs."cralaw virtua1aw library

Armed with the search warrant Zoleta and other agents assigned to the Anti-Graft
Division of the NBI entered the premises of the Republic Supermarket Building
and served the search warrant upon Atty. Alidio of the insurance company, in the
presence of Mr. William Li Yao, president and chairman of the board of directors
of the insurance firm. After the search they seized and carried away two (2)
carloads of documents, papers and receipts.

Petitioner assails the validity of the search warrant, claiming that it was issued in
contravention of the explicit provisions of the Constitution and the Rules of Court,
particularly Section 1, of Art. III of the 1935 Constitution, now Section 3, of Art.
IV of the new Constitution, and Sections 3, 5, 8 and 10 of Rule 126 of the Rules of
Court, hereunder quoted for convenience of reference, viz:jgc:chanrobles.com.ph

"Sec. 3. The rights of the people to be secure in their persons, houses, papers and
effects, against unreasonable searches and seizures shall not be violated, and no
warrant shall issue but upon probable cause to be determined by the judge after
examination under oath or affirmation of the complainant and the witnesses he
may produce, and particularly describing the place to be searched, and the persons,
or things to be seized." (Art. IV, Section 3, New Constitution)

"Sec. 3. Requisites for issuing search warrant. — A search warrant shall not issue
but upon probable cause in connection with one specific offense to be determined
by the judge or justice of the peace after examination under oath or affirmation of
the complainant and the witnesses he may produce, and particularly describing the
place to be searched and the persons or things to be seized.

No search warrant shall issue for more than one specific offense." (Sec. 3, Rule
126, Rules of Court)

"Sec. 5. Issuance and form of search warrant . — If the judge or justice of the
peace is thereupon satisfied of the existence of facts upon which the application is
based, or that there is probable cause to believe that they exist, he must issue the
warrant in the form prescribed by these rules." (Sec. 5, Rule 126)

"Sec. 8. Time of making search .— The warrant must direct that it be served in the
day time, unless the affidavit asserts that the property is on the person or in the
place ordered to be searched, in which case a direction may be inserted that it he
served at any time of the night or day." (Sec. 8, Rule 126)
"Sec. 10 Receipt for property seized. — The officer seizing property under the
warrant must give a detailed receipt for the same to the person on whom or in
whose possession it was found, or in the absence of any person, must, in the
presence of at least one witness, leave a receipt in the place in which he found the
seized property." (Sec. 10, Rule 126)

"Of all the rights of a citizen, few are of greater importance or more essential to his
peace and happiness than the right of personal security, and that involves the
exemption of his private affairs, books, and papers from the inspection and
scrutiny of others. 1 While the power to search and seize is necessary to the public
welfare, still it must be exercised and the law enforced without transgressing the
constitutional rights of the citizens, for the enforcement of no statute is of sufficient
importance to justify indifference to the basic principles of government (People v.
Elias, 147 N.E. 472)."cralaw virtua1aw library

In the case at bar, the search warrant was issued for four separate and distinct
offenses of: (1) estafa, (2) falsification, (3) tax evasion and (4) insurance fraud, in
contravention of the explicit command of Section 3, Rule 126, of the Rules
providing that: "no search warrant shall issue for more than one specific offense."
The aforequoted provision, which is found in the last paragraph of the same
section, is something new. "There is no precedent on this amendment —
prohibition against the issuance of a search warrant for more than one specific
offense — either in the American books on Criminal Procedure or in American
decisions." 2 It was applied in the celebrated case of Harry S. Stonehill v. Secretary
of Justice 3 where this Court said:jgc:chanrobles.com.ph

"To uphold the validity of the warrants in question would be to wipe out
completely one of the most fundamental rights guaranteed in our Constitution, for
it would place the sanctity of the domicile and the privacy of communication and
correspondence at the mercy of the whims, caprice or passion of peace officers.
This is precisely the evil sought to be remedied by the constitutional provision
abovequoted — to outlaw the so-called general warrants. It is not difficult to
imagine what would happen in times of keen political strife, when the party in
power feels that the minority is likely to wrest it, even though by legal means.

"Such is the seriousness of the irregularities committed in connection with the


disputed search warrants, that this Court deemed it fit to amend section 3 of Rule
122 of the former Rules of Court by providing in its counterpart, under the Revised
Rules of Court, that a search warrant shall not issue but upon probable cause in
connection with one specific offense. Not satisfied with this qualification, the court
added thereto a paragraph, directing that ‘no search warrant shall issue for more
than one specific offense."cralaw virtua1aw library

II

Petitioner likewise contests the validity of the search warrant on the ground that it
authorized the search and seizure of personal properties so vaguely described and
not particularized, thereby infringing the constitutional mandate requiring
particular description of the place to be searched and the persons or things to be
seized. It also assails the noncompliance with the above-requirement as likewise
openly violative of Section 2 of Rule 126 which provides:jgc:chanrobles.com.ph

"SEC. 2. A search warrant may be issued for the search and seizure of the
following personal property:chanrob1es virtual 1aw library

(a) Property subject of the offense;

(b) Property stolen or embezzled and other proceeds or fruits of the offense; and

(c) Property used or intended to be used as the means of committing an


offense."cralaw virtua1aw library

The search warrant herein involved reads in part: ". . . property (Subject of the
offense, stolen or embezzled and proceeds or fruits of the offense used or intended
to be used as the means of committing the offense) should be seized and brought to
the undersigned." The claim of respondents that by not cancelling the description
of one or two of the classes of property contained in the form when not applicable
to the properties sought to be seized, the respondent judge intended the search to
apply to all the three classes of property. This is a patent impossibility because the
description of the property to be searched and seized, viz: Fire Registers, Loss
Bordereau, Adjusters Report, including subrogation receipts and proof of loss,
Loss Registers, Books of Accounts including cash receipts and disbursements and
general ledger, etc. and the offenses alleged to have been committed by the
corporation to wit: estafa, falsification, tax evasion and insurance fraud, render it
impossible for Us to see how the above-described property can simultaneously be
contraband goods, stolen or embezzled and other proceeds or fruits of one and the
same offense. What is plain and clear is the fact that the respondent Judge made no
attempt to determine whether the property he authorized to be searched and seized
pertains specifically to any one of the three classes of personal property that may
be searched and seized under a search warrant under Rule 126, Sec. 2 of the Rules.
The respondent Judge simply authorized search and seizure under an omnibus
description of the personal properties to be seized. Because of this all embracing
description which includes all conceivable records of petitioner corporation, which
if seized (as it was really seized in the case at bar), could possibly paralyze its
business, 4 petitioner in several motions filed for early resolution of this case,
manifested that the seizure of TWO carloads of their papers has paralyzed their
business to the grave prejudice of not only the company, its workers, agents,
employees but also of its numerous insured and beneficiaries of bonds issued by it,
including the government itself, and of the general public. 5 And correlating the
same to the charges for which the warrant was issued, We have before Us the
infamous general warrants of old. In the case of Uy Kheytin, Et Al., v. Villareal, 42
Phil. 896, cited with approval in the Bache case, supra, We had occasion to explain
the purpose of the requirement that the warrant should particularly describe the
place to be searched and the things to be seized, to wit:jgc:chanrobles.com.ph

". . . Both the Jones Law (sec. 3) and General Orders No. 58 (sec. 97) specifically
require that a search warrant should particularly describe the place to be searched
and the things to be seized. The evident purpose and intent of this requirement is to
limit the things to be seized to those, and only those, particularly described in the
search warrant — to leave the officers of the law with no discretion regarding what
articles they shall seize, to the end that `unreasonable searches and seizures’ may
not be made. That this is the correct interpretation of this constitutional provision is
borne out by American authorities."cralaw virtua1aw library

The purpose as thus explained could, surely and effectively, be defeated under the
search warrant issued in this case.

III.

Moreover, as contended by petitioner, respondents in like manner transgressed


Section 10 of Rule 126 of the Rules for failure to give a detailed receipt of the
things seized. Going over the receipts (Annexes "B", "B-1", "B-2", "B-3" and "B-
4" of the Petition) issued, We found the following: one bordereau of reinsurance, 8
fire registers, 1 marine register, four annual statements, folders described only as
Bundle gm-1 red folders; bundle 17-22 big carton folders; folders of various sizes,
etc., without stating therein the nature and kind of documents contained in the
folders of which there were about a thousand of them that were seized. In the
seizure of two carloads of documents and other papers, the possibility that the
respondents took away private papers of the petitioner, in violation of his
constitutional rights, is not remote, for the NBI agents virtually had a field day
with the broad and unlimited search warrant issued by respondent Judge as their
passport.

IV.

The search warrant violated the specific injunctions of Section 8 of Rule 126. 6
Annex "A" of the Petition which is the search warrant in question left blank the
"time" for making search, while actual search was conducted in the evening of
October 27, 1965, at 7:30 p.m., until the wee hours of the morning of October 28,
1965, thus causing untold inconveniences to petitioners herein. Authorities 7 are of
the view that where a search is to be made during the night time, the authority for
executing the same at that time should appear in the directive on the face of the
warrant.

In their Memorandum 8 respondents, relying on the case of Moncado v. Peoples


Court (80 Phil. 1), argued:jgc:chanrobles.com.ph

"Even assuming that the search warrant in question is null and void, the illegality
thereof would not render the incriminating documents inadmissible in
evidence."cralaw virtua1aw library

This Court has reverted to the old rule and abandoned the Moncado ruling
(Stonehill case, supra). Most common law jurisdictions have already given up this
approach and eventually adopted the exclusionary rule, realizing that this is the
only practical means of enforcing the constitutional injunction against
unreasonable searches and seizures. Thus the Supreme Court of the United States
declared: 9

"If letters and private documents can thus be seized and held and used in evidence
against a citizen accused of an offense, the protection of the 4th Amendment,
declaring his right to be secured against such searches and seizures is of no value,
and so far as those thus placed are concerned, might as well be stricken from the
Constitution. The efforts of the courts and their officials to bring the guilty to
punishment, praise-worthy as they are, are not to be aided by the sacrifice of those
great principles established by years of endeavor and suffering which have resulted
in their embodiment in the fundamental law of the land."cralaw virtua1aw library

Moreover, the criminal charges filed by the NBI have all been dismissed and/or
dropped by the Court or by the office of the City Fiscal of Manila in 1968, as
manifested in the petition filed by petitioner dated October 24, 1972, for early
resolution of this case.

V.

It has likewise been observed that the offenses alleged took place from 1961 to
1964, and the application for search warrant was made on October 27, 1965. The
time of the application is so far remote in time as to make the probable cause of
doubtful veracity and the warrant vitally defective. Thus Mr. Joseph Varon, an
eminent authority on Searches, Seizures and Immunities, has this to say on this
point:jgc:chanrobles.com.ph
"From the examination of the several cases touching upon this subject, the
following general rules are said to apply to affidavits for search
warrants:chanrob1es virtual 1aw library

(1) . . .

(2) Such statement as to the time of the alleged offense must be clear and definite
and must not be too remote from the time of the making of the affidavit and
issuance of the search warrant.

(3) There is no rigid rule for determining whether the stated time of observation of
the offense is too remote from the time when the affidavit is made or the search
warrant issued but, generally speaking, a lapse of time of less than three weeks will
be held not to invalidate the search warrant, while a lapse of four weeks will be
held to be so.

A good and practical rule of thumb to measure the nearness of time given in the
affidavit as to the date of the alleged offense, and the time of making the affidavit
is thus expressed: The nearer the time at which the observation of the offense is
alleged to have been made, the more reasonable the conclusion of establishment of
probable cause." [Emphasis ours]

PREMISES CONSIDERED, petition is hereby granted; the search warrant of


October 27, 1965, is nullified and set aside and the respondents are hereby ordered
to return immediately all documents, papers and other objects seized or taken
thereunder. Without costs.

Makalintal, C.J., Castro, Fernandez ** , and Muñoz Palma, JJ., concur.

Makasiar, J., concurs in the result.


Alvarez v. CFI, G.R. No. L-45358, January 29, 1937
Issue: WON the search warrant is unreasonable.

Ruling: Yes. 

Section 1(par. 3), Article III of the Constitution provides that no warrants shall
issue but upon probable cause, to be determined by the judge after examination
under oath or affirmation of the complainant and the witnesses he may produce.
Section 98 of General Orders, No. 58 provides that the judge or justice must,
before issuing the warrant, examine under oath the complainant and any witnesses
he may produce and take their depositions in writing.

The purpose of both in requiring the presentation of depositions is nothing more


than to satisfy the committing magistrate of the existence of probable cause.
Therefore, if the affidavit of the applicant or complainant is sufficient, the judge
may dispense with that of other witnesses. 

Inasmuch as the affidavit of the agent in this case was insufficient because his
knowledge of the facts was not personal but merely hearsay, it is the duty of the
judge to require the affidavit of one or more witnesses for the purpose of
determining the existence of probable cause to warrant the issuance of the
search warrant. 

When the affidavit of the applicant or complainant contains sufficient facts within
his personal and direct knowledge, it is sufficient if the judge is satisfied that there
exists probable cause; when the applicant's knowledge of the facts is mere
hearsay, the affidavit of one or more witnesses having a personal knowledge of
the facts is necessary. 

In the case at bar, the judge who issued the search warrant in this case, relied
exclusively upon the affidavit made by agent Mariano G. Almeda and that he did
not require nor take the deposition of any other witness.

Therefore, the Court concluded that the warrant issued is illegal because it was
based only on the affidavit of the agent who had no personal knowledge of the
facts necessary to determine the existence or non-existence of probable cause.

The search warrant and the seizure are declared illegal and are set aside, and it is
ordered that the judge presiding over the CFI of Tayabas direct the immediate
return to the petitioner of the nineteen (19) documents.

CASE: Salazar v. Achacoso, G.R. No. 81510, March 14, 1990

This concerns the validity of the power of the Secretary of Labor to issue warrants
of arrest and seizure under Article 38 of the Labor Code, prohibiting illegal
recruitment.

The facts are as follows:

x x x           x x x          x x x

1. On October 21, 1987, Rosalie Tesoro of 177 Tupaz Street,


Leveriza, Pasay City, in a sworn statement filed with the Philippine
Overseas Employment Administration (POEA for brevity) charged
petitioner Hortencia Salazar, viz:
04. T: Ano ba ang dahilan at ikaw ngayon
ay narito at
nagbibigay ng salaysay.

S: Upang ireklamo sa dahilan ang aking PECC Card ay


ayaw ibigay sa akin ng dati kong manager. — Horty
Salazar — 615 R.O. Santos, Mandaluyong, Mla.

05. T: Kailan at saan naganap and


ginawang panloloko sa
iyo ng tao/mga taong inireklamo mo?

S. Sa bahay ni Horty Salazar.

06. T: Paano naman naganap ang


pangyayari?

S. Pagkagaling ko sa Japan ipinatawag


niya ako. Kinuha
ang PECC Card ko at sinabing hahanapan
ako ng
booking sa Japan. Mag 9 month's na ako sa
Phils. ay
hindi pa niya ako napa-alis. So lumipat ako
ng ibang
company pero ayaw niyang ibigay and
PECC Card
ko.

2. On November 3, 1987, public respondent Atty. Ferdinand Marquez


to whom said complaint was assigned, sent to the petitioner the
following telegram:

YOU ARE HEREBY DIRECTED TO APPEAR


BEFORE FERDIE MARQUEZ POEA ANTI ILLEGAL
RECRUITMENT UNIT 6TH FLR. POEA BLDG.
EDSA COR. ORTIGAS AVE. MANDALUYONG MM
ON NOVEMBER 6, 1987 AT 10 AM RE CASE FILED
AGAINST YOU. FAIL NOT UNDER PENALTY OF
LAW.

4. On the same day, having ascertained that the petitioner had no


license to operate a recruitment agency, public respondent
Administrator Tomas D. Achacoso issued his challenged CLOSURE
AND SEIZURE ORDER NO. 1205 which reads:
HORTY SALAZAR
No. 615 R.O. Santos St.
Mandaluyong, Metro Manila

Pursuant to the powers vested in me under Presidential Decree No.


1920 and Executive Order No. 1022, I hereby order the CLOSURE of
your recruitment agency being operated at No. 615 R.O. Santos St.,
Mandaluyong, Metro Manila and the seizure of the documents and
paraphernalia being used or intended to be used as the means of
committing illegal recruitment, it having verified that you have —

(1) No valid license or authority from the Department of


Labor and Employment to recruit and deploy workers
for overseas employment;

(2) Committed/are committing acts prohibited under


Article 34 of the New Labor Code in relation to Article
38 of the same code.

This ORDER is without prejudice to your criminal


prosecution under existing laws.

Done in the City of Manila, this 3th day of November,


1987.

5. On January 26, 1988 POEA Director on Licensing and Regulation


Atty. Estelita B. Espiritu issued an office order designating
respondents Atty. Marquez, Atty. Jovencio Abara and Atty. Ernesto
Vistro as members of a team tasked to implement Closure and
Seizure Order No. 1205. Doing so, the group assisted by
Mandaluyong policemen and mediamen Lito Castillo of the People's
Journal and Ernie Baluyot of News Today proceeded to the residence
of the petitioner at 615 R.O. Santos St., Mandaluyong, Metro Manila.
There it was found that petitioner was operating Hannalie Dance
Studio. Before entering the place, the team served said Closure and
Seizure order on a certain Mrs. Flora Salazar who voluntarily allowed
them entry into the premises. Mrs. Flora Salazar informed the team
that Hannalie Dance Studio was accredited with Moreman
Development (Phil.). However, when required to show credentials,
she was unable to produce any. Inside the studio, the team chanced
upon twelve talent performers — practicing a dance number and saw
about twenty more waiting outside, The team confiscated assorted
costumes which were duly receipted for by Mrs. Asuncion Maguelan
and witnessed by Mrs. Flora Salazar.
6. On January 28, 1988, petitioner filed with POEA the following
letter:

Gentlemen:

On behalf of Ms. Horty Salazar of 615 R.O. Santos, Mandaluyong,


Metro Manila, we respectfully request that the personal properties
seized at her residence last January 26, 1988 be immediately returned
on the ground that said seizure was contrary to law and against the
will of the owner thereof. Among our reasons are the following:

1. Our client has not been given any prior notice or


hearing, hence the Closure and Seizure Order No. 1205
dated November 3, 1987 violates "due process of law"
guaranteed under Sec. 1, Art. III, of the Philippine
Constitution.

2. Your acts also violate Sec. 2, Art. III of the Philippine


Constitution which guarantees right of the people "to be
secure in their persons, houses, papers, and effects
against unreasonable searches and seizures of whatever
nature and for any purpose."

3. The premises invaded by your Mr. Ferdi Marquez and


five (5) others (including 2 policemen) are the private
residence of the Salazar family, and the entry, search as
well as the seizure of the personal properties belonging
to our client were without her consent and were done
with unreasonable force and intimidation, together with
grave abuse of the color of authority, and constitute
robbery and violation of domicile under Arts. 293 and
128 of the Revised Penal Code.

Unless said personal properties worth around TEN


THOUSAND PESOS (P10,000.00) in all (and which
were already due for shipment to Japan) are returned
within twenty-four (24) hours from your receipt hereof,
we shall feel free to take all legal action, civil and
criminal, to protect our client's interests.

We trust that you will give due attention to these


important matters.

7. On February 2, 1988, before POEA could answer the letter,


petitioner filed the instant petition; on even date, POEA filed a
criminal complaint against her with the Pasig Provincial Fiscal,
docketed as IS-88-836.1

On February 2, 1988, the petitioner filed this suit for prohibition. Although the acts
sought to be barred are already fait accompli, thereby making prohibition too late,
we consider the petition as one for certiorari in view of the grave public interest
involved.

The Court finds that a lone issue confronts it: May the Philippine Overseas
Employment Administration (or the Secretary of Labor) validly issue warrants of
search and seizure (or arrest) under Article 38 of the Labor Code? It is also an issue
squarely raised by the petitioner for the Court's resolution.

Under the new Constitution, which states:

. . . no search warrant or warrant of arrest shall issue except upon


probable cause to be determined personally by the judge after
examination under oath or affirmation of the complainant and the
witnesses he may produce, and particularly describing the place to be
searched and the persons or things to be seized. 2

it is only a judge who may issue warrants of search and arrest. 3 In one case, it was
declared that mayors may not exercise this power:

x x x           x x x          x x x

But it must be emphasized here and now that what has just been
described is the state of the law as it was in September, 1985. The law
has since been altered. No longer does the mayor have at this time the
power to conduct preliminary investigations, much less issue orders
of arrest. Section 143 of the Local Government Code, conferring this
power on the mayor has been abrogated, rendered functus officio by
the 1987 Constitution which took effect on February 2, 1987, the date
of its ratification by the Filipino people. Section 2, Article III of the
1987 Constitution pertinently provides that "no search warrant or
warrant of arrest shall issue except upon probable cause to be
determined personally by the judge after examination under oath or
affirmation of the complainant and the witnesses he may produce, and
particularly describing the place to be searched and the person or
things to be seized." The constitutional proscription has thereby been
manifested that thenceforth, the function of determining probable
cause and issuing, on the basis thereof, warrants of arrest or search
warrants, may be validly exercised only by judges, this being
evidenced by the elimination in the present Constitution of the phrase,
"such other responsible officer as may be authorized by law" found in
the counterpart provision of said 1973 Constitution, who, aside from
judges, might conduct preliminary investigations and issue warrants
of arrest or search warrants. 4

Neither may it be done by a mere prosecuting body:

We agree that the Presidential Anti-Dollar Salting Task Force


exercises, or was meant to exercise, prosecutorial powers, and on that
ground, it cannot be said to be a neutral and detached "judge" to
determine the existence of probable cause for purposes of arrest or
search. Unlike a magistrate, a prosecutor is naturally interested in the
success of his case. Although his office "is to see that justice is done
and not necessarily to secure the conviction of the person accused,"
he stands, invariably, as the accused's adversary and his accuser. To
permit him to issue search warrants and indeed, warrants of arrest, is
to make him both judge and jury in his own right, when he is neither.
That makes, to our mind and to that extent, Presidential Decree No.
1936 as amended by Presidential Decree No. 2002, unconstitutional. 5

Section 38, paragraph (c), of the Labor Code, as now written, was entered as an
amendment by Presidential Decrees Nos. 1920 and 2018 of the late President
Ferdinand Marcos, to Presidential Decree No. 1693, in the exercise of his
legislative powers under Amendment No. 6 of the 1973 Constitution. Under the
latter, the then Minister of Labor merely exercised recommendatory powers:

(c) The Minister of Labor or his duly authorized representative shall


have the power to recommend the arrest and detention of any person
engaged in illegal recruitment. 6

On May 1, 1984, Mr. Marcos promulgated Presidential Decree No. 1920, with the
avowed purpose of giving more teeth to the campaign against illegal recruitment.
The Decree gave the Minister of Labor arrest and closure powers:

(b) The Minister of Labor and Employment shall have the power to
cause the arrest and detention of such non-licensee or non-holder of
authority if after proper investigation it is determined that his
activities constitute a danger to national security and public order or
will lead to further exploitation of job-seekers. The Minister shall
order the closure of companies, establishment and entities found to be
engaged in the recruitment of workers for overseas employment,
without having been licensed or authorized to do so. 7

On January 26, 1986, he, Mr. Marcos, promulgated Presidential Decree No. 2018,
giving the Labor Minister search and seizure powers as well:
(c) The Minister of Labor and Employment or his duly authorized
representatives shall have the power to cause the arrest and detention
of such non-licensee or non-holder of authority if after investigation it
is determined that his activities constitute a danger to national
security and public order or will lead to further exploitation of job-
seekers. The Minister shall order the search of the office or premises
and seizure of documents, paraphernalia, properties and other
implements used in illegal recruitment activities and the closure of
companies, establishment and entities found to be engaged in the
recruitment of workers for overseas employment, without having
been licensed or authorized to do so. 8

The above has now been etched as Article 38, paragraph (c) of the Labor Code.

The decrees in question, it is well to note, stand as the dying vestiges of


authoritarian rule in its twilight moments.

We reiterate that the Secretary of Labor, not being a judge, may no longer issue
search or arrest warrants. Hence, the authorities must go through the judicial
process. To that extent, we declare Article 38, paragraph (c), of the Labor Code,
unconstitutional and of no force and effect.

The Solicitor General's reliance on the case of Morano v. Vivo 9 is not well-
taken. Vivo involved a deportation case, governed by Section 69 of the defunct
Revised Administrative Code and by Section 37 of the Immigration Law. We have
ruled that in deportation cases, an arrest (of an undesirable alien) ordered by the
President or his duly authorized representatives, in order to carry out a final
decision of deportation is valid. 10 It is valid, however, because of the recognized
supremacy of the Executive in matters involving foreign affairs. We have held: 11

x x x           x x x          x x x

The State has the inherent power to deport undesirable aliens


(Chuoco Tiaco vs. Forbes, 228 U.S. 549, 57 L. Ed. 960, 40 Phil.
1122, 1125). That power may be exercised by the Chief Executive
"when he deems such action necessary for the peace and domestic
tranquility of the nation." Justice Johnson's opinion is that when the
Chief Executive finds that there are aliens whose continued presence
in the country is injurious to the public interest, "he may, even in the
absence of express law, deport them". (Forbes vs. Chuoco Tiaco and
Crossfield, 16 Phil. 534, 568, 569; In re McCulloch Dick, 38 Phil.
41).

The right of a country to expel or deport aliens because their


continued presence is detrimental to public welfare is absolute and
unqualified (Tiu Chun Hai and Go Tam vs. Commissioner of
Immigration and the Director of NBI, 104 Phil. 949, 956). 12

The power of the President to order the arrest of aliens for deportation is,
obviously, exceptional. It (the power to order arrests) can not be made to extend to
other cases, like the one at bar. Under the Constitution, it is the sole domain of the
courts.

Moreover, the search and seizure order in question, assuming, ex gratia argumenti,
that it was validly issued, is clearly in the nature of a general warrant:

Pursuant to the powers vested in me under Presidential Decree No.


1920 and Executive Order No. 1022, I hereby order the CLOSURE of
your recruitment agency being operated at No. 615 R.O. Santos St.,
Mandaluyong, Metro Manila and the seizure of the documents and
paraphernalia being used or intended to be used as the means of
committing illegal recruitment, it having verified that you have —

(1) No valid license or authority from the Department of


Labor and Employment to recruit and deploy workers
for overseas employment;

(2) Committed/are committing acts prohibited under


Article 34 of the New Labor Code in relation to Article
38 of the same code.

This ORDER is without prejudice to your criminal prosecution under


existing laws. 13

We have held that a warrant must identify clearly the things to be seized,
otherwise, it is null and void, thus:

x x x           x x x          x x x

Another factor which makes the search warrants under consideration


constitutionally objectionable is that they are in the nature of general
warrants. The search warrants describe the articles sought to be seized
in this wise:

1) All printing equipment, paraphernalia, paper, ink,


photo equipment, typewriters, cabinets, tables,
communications/ recording equipment, tape recorders,
dictaphone and the like used and/or connected in the
printing of the "WE FORUM" newspaper and any and
all documents/communications, letters and facsimile of
prints related to the "WE FORUM" newspaper.

2) Subversive documents, pamphlets, leaflets, books,


and other publications to promote the objectives and
purposes of the subversive organizations known as
Movement for Free Philippines, Light-a-Fire Movement
and April 6 Movement; and

3) Motor vehicles used in the distribution/circulation of


the "WE FORUM" and other subversive materials and
propaganda, more particularly,

1) Toyota-Corolla, colored yellow with Plate No. NKA


892;

2) DATSUN, pick-up colored white with Plate No.


NKV 969;

3) A delivery truck with Plate No. NBS 542;

4) TOYOTA-TAMARAW, colored white with Plate


No. PBP 665; and

5) TOYOTA Hi-Lux, pick-up truck with Plate No. NGV


472 with marking "Bagong Silang."

In Stanford v. State of Texas, the search warrant which authorized the


search for "books, records, pamphlets, cards, receipts, lists,
memoranda, pictures, recordings and other written instruments
concerning the Communist Parties of Texas, and the operations of the
Community Party in Texas," was declared void by the U.S. Supreme
Court for being too general. In like manner, directions to "seize any
evidence in connection with the violation of SDC 13-3703 or
otherwise" have been held too general, and that portion of a search
warrant which authorized the seizure of any "paraphernalia which
could be used to violate Sec. 54-197 of the Connecticut General
Statutes (the statute dealing with the crime of conspiracy)" was held
to be a general warrant, and therefore invalid. The description of the
articles sought to be seized under the search warrants in question
cannot be characterized differently.

In the Stanford case, the U.S. Supreme court calls to mind a notable
chapter in English history; the era of disaccord between the Tudor
Government and the English Press, when "Officers of the Crown
were given roving commissions to search where they pleased in order
to suppress and destroy the literature of dissent both Catholic and
Puritan." Reference herein to such historical episode would not be
relevant for it is not the policy of our government to suppress any
newspaper or publication that speaks with "the voice of non-
conformity" but poses no clear and imminent danger to state
security. 14

For the guidance of the bench and the bar, we reaffirm the following principles:

1. Under Article III, Section 2, of the l987 Constitution, it is only


judges, and no other, who may issue warrants of arrest and search:

2. The exception is in cases of deportation of illegal and undesirable


aliens, whom the President or the Commissioner of Immigration may
order arrested, following a final order of deportation, for the purpose
of deportation.

WHEREFORE, the petition is GRANTED. Article 38, paragraph (c) of the Labor
Code is declared UNCONSTITUTIONAL and null and void. The respondents are
ORDERED to return all materials seized as a result of the implementation of
Search and Seizure Order No. 1205.
d. particularity of description

e. issued in connection with one specific offense only

2. Partially defective SWs: legal effect


3. Illegality of search and seizure does not ipso jure result in return of seized items

CASES:
People v. Marcos, G.R. No. L-31757, October 29, 1982

Petitioner, through the Solicitor General, filed this petition to review on certiorari
the order dated October 13, 1969 of respondent Judge of the Court of First Instance
of Baguio and Benguet Province, declaring the issuance by the City Court of
Search Warrant No. 459, as contrary to law and ordering the National Bureau of
Investigation (NBI) agents and any person in possession of the articles seized by
virtue of the search warrant to deliver and return the same to Yu Cua Sio, owner
and/or manager of the Suntory Grocery.

On June 15, 1969, NBI Supervising Agent Jose Vicente filed an application for the
issuance of a search warrant with the City Court of Baguio City which provides,
among others, the following:

That he has been reliably informed and verily believes that the owner
and/or manager of the Suntory Grocery, of 36 Rajah Soliman St.,
Baguio City has in his possession and control stocks of San Miguel
Gin, product of the La Tondeña, Inc., Manila, which is adulterated,
bearing fake auxiliary stamps, and using crown caps not produced by
the company, which is in violation of the provisions of Articles 188
and 189 of the Revised Penal Code.

... and therefore has reason enough to believe that a search warrant
should be issued to enable the undersigned to take possession and
control and bring it to this Court said stock, as evidence in the above
case under investigation.

City Judge Patricio Perez of Baguio City, acting on the aforesaid application issued
on June 15, 1967, Search Warrant No. 459, under the following terms:

Whereas, after examination under oath of Supervising Agent, NBI


Jose Vicente and Cesar de Leon, both of the NBI Regional Office, at
Dagupan City, this Court finds that there is probable cause to believe
that the owner and/or manager of the Suntory Grocery, located at No.
36 Rajah Soliman St., Baguio City, is in possession such stock of La
Tondena product, San Miguel Gin, which is adulterated, bearing
auxiliary stamps which is tampered and possession of falsified or fake
crown caps, which is now under investigation by the National Bureau
of Investigation, in this City.

Therefore, you are hereby commanded during the day only to make
an immediate search on the premises of the store and/or grocery
owned by the Manager and/or proprietor of the Suntory Grocery
which is located in No. 36 Rajah Soliman, of this City and if you
should find the same to bring it forthwith before me in the City Court
of Baguio to be dealt with as the law directs.

Armed with said search warrant, the NBI agents, on June 19, 1967, searched the
premises of the Suntory Grocery located at 36 Rajah Soliman St., Baguio City,
owned and managed by private respondent Yu Cua Sio. During the search
conducted in the presence of private respondent and his wife, the NBI agents
seized the following articles which were properly inventoried and receipted:

(1) One (1) bundle consisting of rubber siphon, trainer and funnel;

(2) One (1) galvanized tank, 15" in diameter, 2 feet in height;

(3) One (1) mounted crown cap sealer with accessories;

(4) Six (6) cases of San Miguel Gin, round bottles, fined with
suspected adulterated Gin and bearing BIR stamps dated 6 April
1967;

(5) Two(2)cases of San Miguel Gin empty bottles;

(6) One (1) case containing ten (10) San Miguel Gin (adulterated) and
14 empty bottles, without stamps;

(7) One (1) bundle consisting of two (2) bottles believed to be


containing genuine San Miguel Gin which was marked as standard;
and

(8) One (1) paper bag containing 122 auxiliary stamps dated 6 April
1967, without perforation and bearing successive serial numbers
starting from 43,000,000.

Private respondent Yu Cua Sio filed a motion to quash Search Warrant No. 459
with the City Court of Baguio. In an Order dated February 3, 1969, the inferior
court ordered the NBI agents who seized and confiscated the various articles from
the store of Yu Cua Sio to return the same immediately to him.

On appeal, the Court of First Instance of Baguio City, reversed the decision of the
City Court and sustained the validity of the questioned search warrant. However,
upon motion for reconsideration filed by private respondent Yu Cua Sio,
respondent Judge, on October 13, 1969, reconsidered his decision and declared the
issuance of Search Warrant No. 459 as contrary to law and, forthwith, ordered the
NBI agents to deliver and return the articles seized by virtue of the search warrant
to private respondent Yu Cua Sio.

The only legal issue posed for resolution in this case is the validity of Search
Warrant No. 459 issued by City Judge Patricio Perez of Baguio City which
respondent Judge declared as null and void in his order dated October 13, 1969 on
the grounds that (1) the same was issued "for more than one specific offense in
violation of Section 3, Rule 126 of the New Rules of Court which states 'no search
warrant shall issue for more than one specific offense' "; and, (2) that the search
warrant was issued to fish for evidence.

Petitioner claims that the lower court erred (1) when it held that the questioned
search warrant violates the provisions of Section 3, Rule 126 of the New Rules of
Court; (2) in holding that the search warrant in question was issued to fish for
evidence; and (3) in declaring Search Warrant No. 459 as contrary to law and in
ordering the return of the articles seized by virtue of said search warrant to
respondent Yu Cua Sio.

We find merit in the petition. The search warrant issued by the City Court did not
mention any specific offense deemed to have been violated by respondent Yu Cua
Sio. It is in the application filed by the NBI agents which states that the owner
and/or manager of the Suntory Grocery has in his possession and control stocks of
San Miguel Gin which are adulterated and therefore, violative of the provisions of
Articles 188 and 189 of the Revised Penal Code. These articles of the Revised
Penal Code are entitled: "Substituting and Altering Trademarks, Tradenames, or
Service marks" and "Unfair Competition and Fraudulent Registration of
Trademark and Tradename," respectively. As aptly stated by the Solicitor General,
"the specific acts defining said offenses and mentioned in said articles are closely
allied to each other that in a sense, the punishable acts defined in one of them can
be considered as including, or necessarily included in the other."

There is no merit also in the pronouncement by respondent Judge that the search
warrant was issued to fish for evidence just because the application for search
warrant states that its purpose is "to take possession and control of the articles to be
used as evidence in the above case under investigation."
The search warrant as issued mentions that respondent has in his possession and
control stocks of La Tondeña product which are adulterated, with fake auxiliary
stamps, and are using crown caps which are not produced by the company. The
articles seized in the premises of respondent Yu Cua Sio show that he was in
possession of these articles mentioned in the application for search warrant and in
the search warrant itself. Possession of said fake stamps is illegal and the same
should not be returned to respondent Yu Cua Sio.

ACCORDINGLY, the order of respondent Judge, dated October 13, 1969, is SET
ASIDE, and private respondent Yu Cua Sio is hereby ordered to return the articles
seized if they had been delivered to him by the NBI agents.

SO ORDERED.

Mata v. Bayona, G.R. No. L- 50720 March 26, 1984

ISSUE:

WON the judge must before issuing the warrant personally examine on oath or
affirmation the complainant and any witnesses he may produce and take their
depositions in writing, and attach them to the record, in addition to any affidavits
presented to him?

HELD:

YES. Under the Constitution “no search warrant shall issue but upon probable
cause to be determined by the Judge or such other responsible officer as may be
authorized by law after examination under oath or affirmation of the complainant
and the witnesses he may produce.” More emphatic and detailed is the
implementing rule of the constitutional injunction, The Rules provide that the
judge must before issuing the warrant personally examine on oath or affirmation
the complainant and any witnesses he may produce and take their depositions in
writing, and attach them to the record, in addition to any affidavits presented to
him.

Mere affidavits of the complainant and his witnesses are thus not sufficient. The
examining Judge has to take depositions in writing of the complainant and the
witnesses he may produce and to attach them to the record. Such written deposition
is necessary in order that the Judge may be able to properly determine the existence
or nonexistence of the probable cause, to hold liable for perjury the person giving it
if it will be found later that his declarations are false.
We, therefore, hold that the search warrant is tainted with illegality by the failure
of the Judge to conform to the essential requisites of taking the depositions in
writing and attaching them to the record, rendering the search warrant invalid.

Nolasco v. Paño, G.R. No. L-69803, October 8, 1985 (Read also the dissents of
JJ. Cuevas & Teehankee)
[G.R. No. 50720. March 26, 1984.]

SORIANO MATA, Petitioner, v. HON. JOSEPHINE K. BAYONA, in her


capacity as Presiding Judge of the City Court of Ormoc, BERNARDO
GOLES and REYNALDO MAYOTE, Respondents.

Valeriano R. Ocubillo for Petitioner.

The Solicitor General for Respondents.

SYLLABUS

1. CONSTITUTIONAL LAW; BILL OF RIGHTS; RIGHT AGAINST


UNLAWFUL SEARCH AND SEIZURE; REQUISITES FOR ISSUANCE OF
SEARCH WARRANT. — Under the Constitution "no search warrant shall issue
but upon probable cause to be determined by the Judge or such other responsible
officer as may be authorized by law after examination under oath or affirmation of
the complainant and the witnesses he may produce." More emphatic and detailed is
the implementing rule of the constitutional injunction, Section 4 of Rule 126 which
provides that the judge must before issuing the warrant personally examine on oath
or affirmation the complainant and any witnesses he may produce and take their
depositions in writing, and attach them to the record, in addition to any affidavits
presented to him.

2. ID.; ID.; ID.; ID.; INSUFFICIENCY OF AFFIDAVITS OF COMPLAINANT


AND HIS WITNESSES IN THE CASE AT BAR. — Before issuing a search
warrant, the examining Judge has to take depositions in writing of the complainant
and the witnesses he may produce and to attach them to the record. Such written
deposition is necessary in order that the Judge may be able to properly determine
the existence or non-existence of the probable cause, and to hold liable for perjury
the person giving it if it will be found later that his declarations are false. Mere
affidavits of the complainant and his witnesses are thus not sufficient.

3. ID.; ID.; ID.; ID.; NO "DEPOSITION IN WRITING" ATTACHED TO


RECORDS OF CASE IN CASE AT BAR. — The judge’s insistence that she
examined the complainants under oath has become dubious by petitioner’s claim
that at the particular time when he examined all the relevant papers connected with
the issuance of the questioned search warrant, after he demanded the same from the
lower court since they were not attached to the records, he did not find any
certification at the back of the joint affidavit of the complainants. Before he filed
his motion to quash the search warrant and for the return of the articles seized, he
was furnished, upon his request, certified true copies of the said affidavits by the
Clerk of Court but which certified true copies do not bear any certification at the
back. Petitioner likewise claims that his xerox copy of the said joint affidavit
obtained at the outset of this case does not show also the certification of respondent
judge. This doubt becomes more confirmed by respondent Judge’s own admission,
while insisting that she did examine thoroughly the applicants, that "she did not
take the deposition of Mayote and Goles because to have done so would be to hold
a judicial proceeding which will be open and public", such that, according to her,
the persons subject of the intended raid will just disappear and move his illegal
operations somewhere else. Could it be that the certification was made belatedly to
cure the defect of the warrant? Be that as it may, there was no "deposition in
writing" attached to the records of the case in palpable disregard of the statutory
prohibition heretofore quoted.

4. ID.; ID.; ID.; ID.; DEPOSITIONS, HOW TAKEN. — The searching questions
propounded to the applicants of the search warrant and his witnesses must depend
to a large extent upon the discretion of the Judge just as long as the answers
establish a reasonable ground to believe the commission of a specific offense and
that the applicant is one authorized by law, and said answers particularly describe
with certainty the place to be searched and the persons or things to be seized. The
examination or investigation which must be under oath may not be in public. It
may even be held in the secrecy of his chambers. Far more important is that the
examination or investigation is not merely routinary but one that is thorough and
elicit the required information. To repeat, it must be under oath and must be in
writing.

5. ID.; ID.; ID.; ID.; MUST BE STRICTLY COMPLIED WITH; CASE AT BAR.
— Nothing can justify the issuance of the search warrant but the fulfillment of the
legal requisites. Thus, in issuing a search warrant the Judge must strictly comply
with the requirements of the Constitution and the statutory provisions. In the case
at bar, the search warrant is tainted with illegality by the failure of the Judge to
conform with essential requisites of taking the depositions in writing and attaching
them to record, rendering the search warrant invalid.

6. ID.; ID.; ID.; ALTHOUGH ILLEGAL, THINGS SEIZED CANNOT BE


RETURNED; CASE AT BAR. — While the search warrant is illegal, the return of
the things seized cannot be ordered. In Castro v. Pabalan (70 SCRA 478), it was
held that the illegality of the search warrant does not call for the return of the
things seized, the possession of which is prohibited.
DECISION

DE CASTRO, J.:

The validity of the search warrant issued by respondent Judge (not reappointed) is
challenged by petitioner for its alleged failure to comply with the requisites of the
Constitution and the Rules of Court.

Specifically, the contention is that the search warrant issued by respondent Judge
was based merely on the application for search warrant and a joint affidavit of
private respondents which were wrongfully it is alleged subscribed, and sworn to
before the Clerk of Court of respondent Judge. Furthermore, there was allegedly a
failure on the part of respondent Judge to attach the necessary papers pertinent to
the issuance of the search warrant to the records of Criminal Case No. 4298-CC
wherein petitioner is accused under PD 810, as amended by PD 1306, the
information against him alleging that Soriano Mata offered, took and arranged bets
on the Jai Alai game by "selling illegal tickets known as ‘Masiao tickets’ without
any authority from the Philippine Jai Alai & Amusement Corporation or from the
government authorities concerned." 1

Petitioner claims that during the hearing of the case, he discovered that nowhere
from the records of the said case could be found the search warrant and other
pertinent papers connected to the issuance of the same, so that he had to inquire
from the City Fiscal its whereabouts, and to which inquiry respondent Judge
replied, "it is with the court." The Judge then handed the records to the Fiscal who
attached them to the records.chanrobles.com : virtual law library

This led petitioner to file a motion to quash and annul the search warrant and for
the return of the articles seized, citing and invoking, among others, Section 4 of
Rule 126 of the Revised Rules of Court. The motion was denied by respondent
Judge on March 1, 1979, stating that the court has made a thorough investigation
and examination under oath of Bernardo U. Goles and Reynaldo T. Mayote,
members of the Intelligence Section of 352nd PC Co./Police District II INP; that in
fact the court made a certification to that effect; and that the fact that documents
relating to the search warrant were not attached immediately to the record of the
criminal case is of no moment, considering that the rule does not specify when
these documents are to be attached to the records. 2 Petitioner’s motion for
reconsideration of the aforesaid order having been denied, he came to this Court,
with the instant petition, praying, among others, that this Court declare the search
warrant to be invalid and all the articles confiscated under such warrant as
inadmissible as evidence in the case, or in any proceedings on the matter.
We hold that the search warrant is tainted with illegality for being violative of the
Constitution and the Rules of Court.

Under the Constitution "no search warrant shall issue but upon probable cause to
be determined by the Judge or such other responsible officer as may be authorized
by law after examination under oath or affirmation of the complainant and the
witnesses he may produce." More emphatic and detailed is the implementing rule
of the constitutional injunction, Section 4 of Rule 126 which provides that the
judge must before issuing the warrant personally examine on oath or affirmation
the complainant and any witnesses he may produce and take their depositions in
writing, and attach them to the record, in addition to any affidavits presented to
him.

Mere affidavits of the complainant and his witnesses are thus not sufficient. The
examining Judge has to take depositions in writing of the complainant and the
witnesses he may produce and to attach them to the record. Such written deposition
is necessary in order that the Judge may be able to properly determine the existence
or non-existence of the probable cause, to hold liable for perjury the person giving
it if it will be found later that his declarations are false.

We, therefore, hold that the search warrant is tainted with illegality by the failure
of the Judge to conform with the essential requisites of taking the depositions in
writing and attaching them to the record, rendering the search warrant
invalid.chanroblesvirtualawlibrary

The judge’s insistence that she examined the complainants under oath has become
dubious by petitioner’s claim that at the particular time when he examined all the
relevant papers connected with the issuance of the questioned search warrant, after
he demanded the same from the lower court since they were not attached to the
records, he did not find any certification at the back of the joint affidavit of the
complainants. As stated earlier, before he filed his motion to quash the search
warrant and for the return of the articles seized, he was furnished, upon his request,
certified true copies of the said affidavits by the Clerk of Court but which certified
true copies do not bear any certification at the back. Petitioner likewise claims that
his xerox copy of the said joint affidavit obtained at the outset of this case does not
show also the certification of respondent judge. This doubt becomes more
confirmed by respondent Judge’s own admission, while insisting that she did
examine thoroughly the applicants, that "she did not take the deposition of Mayote
and Goles because to have done so would be to hold a judicial proceeding which
will be open and public", 3 such that, according to her, the persons subject of the
intended raid will just disappear and move his illegal operations somewhere else.

Could it be that the certification was made belatedly to cure the defect of the
warrant? Be that as it may, there was no "deposition in writing" attached to the
records of the case in palpable disregard of the statutory prohibition heretofore
quoted.

Respondent Judge impresses this Court that the urgency to stop the illegal
gambling that lures every man, woman and child, and even the lowliest laborer
who could hardly make both ends meet justifies her action. She claims that in order
to abate the proliferation of this illegal "masiao" lottery, she thought it more
prudent not to conduct the taking of deposition which is done usually and publicly
in the court room.

Two points must be made clear. The term "depositions" is sometimes used in a
broad sense to describe any written statement verified by oath; but in its more
technical and appropriate sense the meaning of the word is limited to written
testimony of a witness given in the course of a judicial proceeding in advance of
the trial or hearing upon oral examination. 4 A deposition is the testimony of a
witness, put or taken in writing, under oath or affirmation before a commissioner,
examiner or other judicial officer, in answer to interlocutory and cross
interlocutory, and usually subscribed by the witnesses. 5 The searching questions
propounded to the applicants of the search warrant and his witnesses must depend
to a large extent upon the discretion of the Judge just as long as the answers
establish a reasonable ground to believe the commission of a specific offense and
that the applicant is one authorized by law, and said answers particularly describe
with certainty the place to be searched and the persons or things to be seized. The
examination or investigation which must be under oath may not be in public. It
may even be held in the secrecy of his chambers. Far more important is that the
examination or investigation is not merely routinary but one that is thorough and
elicit the required information. To repeat, it must be under oath and must be in
writing.cralawnad

The other point is that nothing can justify the issuance of the search warrant but the
fulfillment of the legal requisites. It might be well to point out what has been said
in Asian Surety & Insurance Co., Inc. v. Herrera:jgc:chanrobles.com.ph

"It has been said that of all the rights of a citizen, few are of greater importance or
more essential to his peace and happiness than the right of personal security, and
that involves the exemption of his private affairs, books, and papers from
inspection and scrutiny of others. While the power to search and seize is necessary
to the public welfare, still it must be exercised and the law enforced without
transgressing the constitutional rights of the citizens, for the enforcement of no
statute is of sufficient importance to justify indifference to the basic principles of
government." 6

Thus, in issuing a search warrant the Judge must strictly comply with the
requirements of the Constitution and the statutory provisions. A liberal
construction should be given in favor of the individual to prevent stealthy
encroachment upon, or gradual depreciation of the rights secured by the
Constitution. 7 No presumption of regularity are to be invoked in aid of the process
when an officer undertakes to justify it. 8

While We hold that the search warrant is illegal, the return of the things seized
cannot be ordered. In Castro v. Pabalan, 9 it was held that the illegality of the
search warrant does not call for the return of the things seized, the possession of
which is prohibited.

WHEREFORE, the writ of certiorari is granted and the order of March 1, 1979
denying the motion to annul the search warrant as well as the order of March 21,
1979 denying the motion for reconsideration are hereby reversed, the search
warrant, being declared herein as illegal. Notwithstanding such illegality, the
things seized under such warrant, such as stock of "masiao" tickets; "masiao" issue
tickets; bet money; control pad or "masiao" numbers; stamping pad with rubber
stamp marked Ormoc City Jai-Alai," cannot be returned as sought by petitioner.
No costs.

SO ORDERED.

Roan v. Gonzales, G.R. No. 71410, November 25, 1986

G.R. No. 71410 November 25, 1986

JOSEFINO S. ROAN, petitioner,
vs.
THE HONORABLE ROMULO T. GONZALES, PRESIDING JUDGE,
REGIONAL TRIAL COURT OF MARINDUQUE, BRANCH XXXVIII;
THE PROVINCIAL FISCAL OF MARINDUQUE; THE PROVINCIAL
COMMANDER, PC-INP MARINDUQUE, respondents.

CRUZ, J:

Once again we are asked to annul a search warrant on the ground that it violates the
Constitution. As we can do no less if we are to be true to the mandate of the
fundamental law, we do annul.

One of the most precious rights of the citizen in a free society is the right to be left
alone in the privacy of his own house. That right has ancient roots, dating back
through the mists of history to the mighty English kings in their fortresses of
power. Even then, the lowly subject had his own castle where he was monarch of
all he surveyed. This was his humble cottage from which he could bar his
sovereign lord and all the forces of the Crown.

That right has endured through the ages albeit only in a few libertarian regimes.
Their number, regrettably, continues to dwindle against the onslaughts of
authoritarianism. We are among the fortunate few, able again to enjoy this right
after the ordeal of the past despotism. We must cherish and protect it all the more
now because it is like a prodigal son returning.

That right is guaranteed in the following provisions of Article IV of the 1973


Constitution:

SEC. 3. The right of the people to be secure in their persons, houses,


papers and effects against unreasonable searches and seizures of
whatever nature and for any purpose shall not be violated, and no
search warrant or warrant of arrest shall issue except upon probable
cause to be determined by the judge, or such other responsible officer
as may be authorized by law, after examination under oath or
affirmation of the complainant and the witnesses he may produce, and
particularly describing the place to be searched, and the persons or
things to be seized.

SEC. 4. (1) The privacy of communication and cor- respondence shag


be inviolable except upon lawful order of the court, or when public
safety and order require otherwise.

(2) Any evidence obtained in violation of this or the preceding section


shall be inadmissible for any purpose in any proceeding.

Invoking these provisions, the petitioner claims he was the victim of an illegal
search and seizure conducted by the military authorities. The articles seized from
him are sought to be used as evidence in his prosecution for illegal possession of
firearms. He asks that their admission be temporarily restrained (which we
have) 1 and thereafter permanently enjoined.

The challenged search warrant was issued by the respondent judge on May 10,
1984.2 The petitioner's house was searched two days later but none of the articles
listed in the warrant was discovered. 3 However, the officers conducting the search
found in the premises one Colt Magnum revolver and eighteen live bullets which
they confiscated. They are now the bases of the charge against the petitioner. 4

To be valid, a search warrant must be supported by probable cause to be


determined by the judge or some other authorized officer after examining the
complainant and the witnesses he may produce. No less important, there must be a
specific description of the place to be searched and the things to be seized, to
prevent arbitrary and indiscriminate use of the warrant. 5

Probable cause was described by Justice Escolin in Burgos v. Chief of Staff 6 as


referring to "such facts and circumstances which would lead a reasonably discreet
and prudent man to believe that an offense has been committed and that the objects
sought in connection with the offense are in the place sought to be searched." As
held in a long line of decisions, the probable cause must refer to only one specific
offense.7

The inclusion of the requirement for the "examination under oath or affirmation of
the complainant and the witnesses he may produce" was a refinement proposed by
Delegate Vicente J. Francisco in the1934 Constitutional Convention. His purpose
was the strengthening of the guaranty against unreasonable searches and seizures.
Although the condition did not appear in the corresponding provision of the federa
Constitution of the United States which served as our model it was then already
embodied in the Code of Criminal Procedure. Nevertheless, Delegate Jose P.
Laurel, Chairman of the Committee on the Bill of Rights of that body, readily
accepted the proposal and it was thereafter, following a brief debate, approved by
the Convention.8

Implementing this requirement, the Rules of Court provided in what was then Rule
126:

SEC. 4. Examination of the applicant. — The municipal or city judge


must, before issuing the warrant, personally examine on oath or
affirmation the complainant and any witnesses he may produce and
take their depositions in writing, and attach them to the record, in
addition to any affidavits presented to him.

The petitioner claims that no depositions were taken by the respondent judge in
accordance with the above rule, but this is not entirely true. As a matter of fact,
depositions were taken of the complainant's two witnesses in addition to the
affidavit executed by them. 9 It is correct to say, however, that the complainant
himself was not subjected to a similar interrogation.

Commenting on this matter, the respondent judge declared:

The truth is that when PC Capt. Mauro P. Quinosa personally filed


his application for a search warrant on May 10, 1984, he appeared
before me in the company of his two (2) witnesses, Esmael Morada
and Jesus Tohilida, both of whom likewise presented to me their
respective affidavits taken by Pat. Josue V. Lining, a police
investigator assigned to the PC-INP command at Camp Col. Maximo
Abad. As the application was not yet subscribed and sworn to, I
proceeded to examine Captain Quillosa on the contents thereof to
ascertain, among others, if he knew and understood the same.
Afterwards, he subscribed and swore to the same before me. 10

By his own account, an he did was question Captain Quillosa on the contents of his
affidavit only "to ascertain, among others, if he knew and understood the same,"
and only because "the application was not yet subscribed and swom to." The
suggestion is that he would not have asked any questions at all if the affidavit had
already been completed when it was submitted to him. In any case, he did not ask
his own searching questions. He limited himself to the contents of the affidavit. He
did not take the applicant's deposition in writing and attach them to the record,
together with the affidavit presented to him.

As this Court held in Mata v. Bayona: 11

Mere affidavits of the complainant and his witnesses are thus not
sufficient. The examining Judge has to take depositions in writing of
the complainant and the witnesses he niay produce and attach them to
the record. Such written deposition is necessary in order that the
Judge may be able to properly determine the existence or non-
existence of the probable cause, to hold liable for perjury the person
giving it if it wifl be found later that his declarations are false.

We, therefore, hold that the search warrant is tainted with illegality by
the failure of the Judge to conform with the essential requisites of
taking the depositions in writing and attaching them to the record,
rendering the search warrant invalid.

The respondent judge also declared that he "saw no need to have applicant
Quillosa's deposition taken considering that he was applying for a search warrant
on the basis of the information provided by the aforenamed witnesses whose
depositions as aforementioned had already been taken by the undersigned." 12

In other words, the applicant was asking for the issuance of the search warrant on
the basis of mere hearsay and not of information personally known to him, as
required by settled jurisprudence." 13 The rationale of the requirement, of course, is
to provide a ground for a prosecution for perjury in case the applicant's
declarations are found to be false. His application, standing alone, was insufficient
to justify the issuance of the warrant sought. It was therefore necessary for the
witnesses themselves, by their own personal information, to establish the
apphcant's claims. 14

Even assuming then that it would have sufficed to take the depositions only of the
witnesses and not of the applicant himself, there is still the question of the
sufficiency of their depositions.
It is axiomatic that the examination must be probing and exhaustive, not merely
routinary or pro-forma, if the claimed probable cause is to be established. The
examining magistrate must not simply rehash the contents of the affidavit but must
make his own inquiry on the intent and justification of the application. 15

A study of the depositions taken from witnesses Esmael Morada and Jesus
Tohilida, who both claimed to be "intelligence informers," shows that they were in
the main a mere restatement of their allegations in their affidavits, except that they
were made in the form of answers to the questions put to them by the respondent
judge. Significantly, the meaningful remark made by Tohilida that they were
suspicious of the petitioner because he was a follower of the opposition candidate
in the forthcoming election (a "Lecarista") 16 did not excite the respondent judge's
own suspicions. This should have put him on guard as to the motivations of the
witnesses and alerted him to possible misrepresentations from them.

The respondent judge almost unquestioningly received the witnesses' statement


that they saw eight men deliver arms to the petitioner in his house on May 2,
1984. 17 This was supposedly done overtly, and Tohilida said he saw everything
through an open window of the house while he was near the gate. 18 He could even
positively say that six of the weapons were.45 caliber pistols and two were.38
caliber revolvers. 19

One may well wonder why it did not occur to the respondent judge to ask how the
witness could be so certain even as to the caliber of the guns, or how far he was
from the window, or whether it was on the first floor or a second floor, or why his
presence was not noticed at all, or if the acts related were really done openly, in the
full view of the witnesses, considering that these acts were against the law. These
would have been judicious questions but they were injudiciously omitted. Instead,
the declarations of the witnesses were readily accepted and the search warrant
sought was issued forthwith.

The above-discussed defects have rendered the search warrant invalid.


Nonetheless, the Solicitor General argues that whatever defect there was, was
waived when the petitioner voluntarily submitted to the search and manifested his
conformity in writing. 20

We do not agree. What we see here is pressure exerted by the military authorities,
who practically coerced the petitioner to sign the supposed waiver as a guaranty
against a possible challenge later to the validity of the search they were conducting.
Confronted with the armed presence of the military and the presumptive authority
of a judicial writ, the petitioner had no choice but to submit. This was not, as we
held in a previous case,21 the manifestation merely of our traditional Filipino
hospitality and respect for authority. Given the repressive atmosphere of the
Marcos regime, there was here, as we see it, an intimidation that the petitioner
could not resist.
The respondents also argue that the Colt Magnum pistol and the eighteen have
bullets seized from the petitioner were illegal per se and therefore could have been
taken by the military authorities even without a warrant. Possession of the said
articles, it is urged, was violative of P.D. 1866 and considered malum
prohibitum. Hence, the Wegal articles could be taken even without a warrant.

Prohibited articles may be seized but only as long as the search is valid. In this
case, it was not because: 1) there was no valid search warrant; and 2) absent such a
warrant, the right thereto was not validly waived by the petitioner. In short, the
military officers who entered the petitioner's premises had no right to be there and
therefore had no right either to seize the pistol and bullets.

It does not follow that because an offense is malum prohibitum, the subject thereof
is necessarily illegal per se. Motive is immaterial in mala prohibita, but the
subjects of this kind of offense may not be summarily seized simply because they
are prohibited. A search warrant is still necessary. If the rule were otherwise, then
the military authorities could have just entered the premises and looked for the
guns reportedly kept by the petitioner without bothering to first secure a search
warrant. The fact that they did bother to do so indicates that they themselves
recognized the necessity of such a warrant for the seizure of the weapons the
petitioner was suspected of possessing.

It is true that there are certain instances when a search may be validly made
without warrant and articles may be taken validly as a result of that search. For
example, a warrantless search may be made incidental to a lawful arrest, 22 as when
the person being arrested is frished for weapons he may otherwise be able to use
against the arresting officer. Motor cars may be inspected at borders to prevent
smuggling of aliens and contraband 23 and even in the interior upon a showing of
probable cause. 24 Vessels and aircraft are also traditionally removed from the
operation of the rule because of their mobility and their relative ease in fleeing the
state's jurisdiction. 25 The individual may knowingly agree to be searched or waive
objections to an illegal search. 26 And it has also been held that prohibited articles
may be taken without warrant if they are open to eye and hand and the peace
officer comes upon them inadvertently. 27

Clearly, though, the instant case does not come under any of the accepted
exceptions. The respondents cannot even claim that they stumbled upon the pistol
and bullets for the fact is that these things were deliberately sought and were not in
plain view when they were taken. Hence, the rule having been violated and no
exception being applicable, the conclusion is that the petitioner's pistol and bullets
were confiscated illegally and therefore are protected by the exclusionary principle.

Stonehill v. Diokno established this rule which was later expressly affirmed in the
1973 Constitution. While conceding that there may be occasions when the criminal
might be allowed to go free because "the constable has blundered," Chief Justice
Concepcion observed that the exclusionary rule was nonetheless "the only practical
means of enforcing the constitutional injunction" against abuse. The decision cited
Judge Learned Hand's justification that "only in case the prosecution which itself
controls the seizing officials, know that it cannot profit by their wrong, will the
wrong be repressed. "

The pistol and bullets cannot, of course, be used as evidence against the petitioner
in the criminal action against him for illegal possession of firearms. Pending
resolution of that case, however, the said articles must remain in custodia legis.

Finally, it is true that the petitioner should have, before coming to this Court, filed
a motion for the quashal of the search warrant by the respondent judge in
accordance with the normal procedure. But as we said and did in Burgos, "this
procedural flaw notwithstanding, we take cognizance of this petition in view of the
seriousness and urgency of the constitutional issues raised. 28

WHEREFORE, Search Warrant No. 1-84 issued by the respondent judge on May
10, 1984, is hereby declared null and void and accordingly set aside. Our
restraining order of August 6,1985, is made permanent. No costs.

SO ORDERED.

Burgos v. Chief of Staff, supra


20th Century Fox Film v. CA, G.R. Nos. 76649-51, August 19, 1988
ISSUE:
WON the search warrant was invalid

RULING:
Yes.

In the case of Burgos, Sr. v. Chief of Staff, AFP (133 SCRA 800), we defined
probable cause for a valid search "as such facts and circumstances which would
lead a reasonably discreet and prudent man to believe that an offense has been
committed and that the objects sought in connection with the offense are in the
place sought to be searched." This constitutional provision also demands "no less
than personal knowledge by the complainant or his witnesses of the facts upon
which the issuance of a search warrant may be justified" in order to convince the
judge, not the individual making the affidavit and seeking the issuance of the
warrant, of the existence of a probable cause. (Alvarez v. Court of First Instance,
64 Phil. 33; Burgos, Sr. v. Chief of Staff, AFP, supra). 

In this instant case, the lower court lifted the three questioned search warrants
against the private respondents on the ground that the same was granted on the
misrepresentations of applicant NBI and its witnesses that they had personal
knowledge that infringement of copyright or a piracy of a particular film have been
committed; hence no real probable cause. 

Moreover, the description of the things to be searched and seized was general in
character and made it a general warrant, which is invalid.The testimonies of the
NBI witnesses believe the fact that they had personal knowledge. In fact only the
petitioner’s witness had real personal knowledge but this was not given credence,
as the master tapes allegedly pirated tapes were not shown to the court during the
application. The presentation of the master tapes of the copyrighted films from
which the pirated films were allegedly copied, was necessary for the validity of
search warrants against those who have in in their possession the pirated films. The
court cannot presume that duplicate or copied tapes were necessarily reproduced
from master tapes that it owns.

Columbia Pictures, Inc. v. CA, G.R. No. 110318, August 28, 1996 (re-
examination of 20th Century Fox ruling)
Before us is a petition for review on certiorari of the decision of the Court of
Appeals 1 promulgated on July 22, 1992 and its resolution 2 of May 10, 1993
denying petitioners’ motion for reconsideration, both of which sustained the order
3 of the Regional Trial Court, Branch 133, Makati, Metro Manila, dated November
22, 1988 for the quashal of Search Warrant No. 87-053 earlier issued per its own
order 4 on September 5, 1988 for violation of section 56 of Presidential Decree No.
49, as amended, otherwise known as the "Decree on the Protection of Intellectual
Property."cralaw virtua1aw library

The material facts found by respondent appellate court are as follows:chanrob1es


virtual 1aw library

Complainants thru counsel lodged a formal complaint with the National Bureau of
Investigation for violation of PD No. 49, as amended, and sought its assistance in
their anti-film piracy drive. Agents of the NBI and private researchers made
discreet surveillance on various video establishments in Metro Manila including
Sunshine Home Video Inc. (Sunshine for brevity), owned and operated by Danilo
A. Pelindario with address at No. 6 Mayfair Center, Magallanes, Makati, Metro
Manila.

On November 14, 1987, NBI Senior Agent Lauro C. Reyes applied for a search
warrant with the court a quo against Sunshine seeking the seizure, among others, of
pirated video tapes of copyrighted films all of which were enumerated in a list
attached to the application; and, television sets, video cassettes and/or laser disc
recordings equipment and other machines and paraphernalia used or intended to be
used in the unlawful exhibition, showing, reproduction, sale, lease or disposition of
videograms tapes in the premises above described. In the hearing of the
application, NBI Senior Agent Lauro C. Reyes, upon questions by the court a quo,
reiterated in substance his averments in his affidavit. His testimony was
corroborated by another witness, Mr. Rene C. Baltazar. Atty. Rico V. Domingo’s
deposition was also taken. On the basis of the affidavits depositions of NBI Senior
Agent Lauro C. Reyes, Rene C. Baltazar and Atty. Rico V. Domingo, Search
Warrant No 87-053 for violation of Section 56 of PD No. 9, as amended, was
issued by the court a quo.

The search warrant was served at about 1:45 p.m. on December 14, 1987 to
Sunshine and/or their representatives. In the course of the search of the premises
indicated in the search warrant, the NBI Agents found and seized various video
tapes of duly copyrighted motion pictures/films owned or exclusively distributed
by private complainants, and machines, equipment, television sets, paraphernalia,
materials, accessories all of which were included in the receipt for properties
accomplished by the raiding team. Copy of the receipt was furnished and/or
tendered to Mr. Danilo A. Pelindario, registered owner-proprietor of Sunshine
Home Video.

On December 16, 1987, a "Return of Search Warrant" was filed with the Court.

A "Motion To Lift the Order of Search Warrant" was filed but was later denied for
lack of merit (p. 280, Records).

A Motion for reconsideration of the Order of denial was filed. The court a quo
granted the said motion for reconsideration and justified it in this
manner:jgc:chanrobles.com.ph

"It is undisputed that the master tapes of the copyrighted films from which the
pirated films were allegedly copies (sic), were never presented in the proceedings
for the issuance of the search warrants in question. The orders of the Court
granting the search warrants and denying the urgent motion to lift order of search
warrants were, therefore, issued in error. Consequently, they must be set aside." (p.
13, Appellant’s Brief) 5

Petitioners thereafter appealed the order of the trial, court granting private
respondents’ motion for reconsideration, thus lifting the search warrant which it
had therefore issued, to the Court of Appeals. As stated at the outset, said appeal
was dismissed and the motion for reconsideration thereof was denied. Hence, this
petition was brought to this Court particularly challenging the validity of
respondent court’s retroactive application of the ruling in 20th Century Fox Film
Corporation v. Court of Appeals, Et Al., 6 in dismissing petitioners’ appeal and
upholding the quashal of the search warrant by the trial court.

I
Inceptively, we shall settle the procedural considerations on the matter of and the
challenge to petitioners’ legal standing in our courts, they being foreign
corporations not licensed to do business in the Philippines.

Private respondents aver that being foreign corporations, petitioners should have
such license to be able to maintain an action in Philippines courts. In so
challenging petitioners’ personality to sue, private respondents point to the fact that
petitioners are the copyright owners or owners of exclusive rights of distribution in
the Philippines of copyrighted motion pictures or films, and also to the
appointment of Atty. Rico V. Domingo as their attorney-at-fact, as being
constitutive of "doing business in the Philippines" under Section 1(f)(1) and (2),
Rule 1 of the Rules of the Board of Investments. As foreign corporations doing
business in the Philippines, Section 133 of Batas Pambansa Bldg. 68, or the
Corporation Code of the Philippines, denies them the right to maintain a suit in
Philippine courts in the absence of a license to do business. Consequently, they
have no right to ask for the issuance of a search warrant. 7

In refutation, petitioners flatly deny that they are doing business in the Philippines,
8 and contend that private respondents have not adduced evidence to prove that
petitioners are doing such business here, as would require them to be licensed by
the Securities and Exchange Commission, other than averments in the quoted
portions of petitioners’ "Opposition to Urgent Motion to Lift Order of Search
Warrant" dated April 28, 1988 and Atty. Rico V. Domingo’s affidavit of December
14, 1987. Moreover, an exclusive right to distribute a product or the ownership of
such exclusive right does not conclusively prove the act of doing business nor
establish the presumption of doing business. 9

The Corporation Code provides:chanrob1es virtual 1aw library

Sec. 133. Doing business without a license. — No foreign corporation transacting


business in the Philippines without a license, or its successors or assigns, shall be
permitted to maintain or intervene in any action, suit or proceeding in any court or
administrative agency of the Philippines; but such corporation may be sued or
proceeded against before Philippine courts or administrative tribunals on any valid
cause of action recognized under Philippine laws.

The obtainment of a license prescribed by Section 125 of the Corporation Code is


not a condition precedent to the maintenance of any kind of action in Philippine
courts by a foreign corporation. However, under the aforequoted provision, no
foreign corporation shall be permitted to transact business in the Philippines, as
this phrase is understood under the Corporation Code, unless it shall have the
license required by law, and until it complies with the law in transacting business
here, it shall not be permitted to maintain any suit in local courts. 10 As thus
interpreted, any foreign corporation doing business in the Philippines may maintain
an action in our courts upon any cause of action, provided that the subject matter
and the defendant are within the jurisdiction of the court. It is not the absence of
the prescribed license bud "doing business" in the Philippines without such license
which debars the foreign corporation from access to our courts. In other words,
although a foreign corporation is without license to transact business in the
Philippines, it does not follow that it has no capacity to bring an action. Such
license is not necessary if it is not engaged in business in the Philippines. 11

Statutory provisions in many jurisdictions are determinative of what constitutes


"doing business" or "transacting business" within that forum, in which case said
provisions are controlling there. In others where no such definition or qualification
is laid down regarding acts or transactions falling within its purview, the question
rests primarily on facts and intent. It is thus held that all the combined acts of a
foreign corporation in the State must be considered, and every circumstance is
material which indicates a purpose on the part of the corporation to engage in some
part of its regular business in the State. 12

No general rule or governing principles can be laid down as to what constitutes


"doing" or "engaging in" or "transacting" business. Each case must be judged in
the light of its own peculiar environmental circumstances. 13 The true tests,
however, seem to be whether the foreign corporation is continuing the body or
substance of the business or enterprise for which it was organized or whether it has
substantially retired from it and turned it over to another. 14

As a general proposition upon which many authorities agree in principle, subject to


such modifications as may be necessary in view of the particular issue or of the
terms of the statute involved, it is recognized that a foreign corporation is "doing,"
"transacting," "engaging in," or "carrying on" business in the State when, and
ordinarily only when, it has entered the State by its agents and is there engaged in
carrying on and transacting through them some substantial part of its ordinary or
customary business, usually continuous in the sense that it may be distinguished
from merely casual, sporadic, or occasional transactions and isolated acts. 15

The Corporation Code does not itself define or categorize what acts constitute
doing or transacting business in the Philippines. Jurisprudence has, however, held
that the term implies a continuity of commercial dealings and arrangements, and
contemplates, to that extent, the performance of acts or works or the exercise of
some of the functions normally incident to or in progressive prosecution of the
purpose and subject of its organization. 16

This traditional case law definition has evolved into a statutory definition, having
been adopted with some qualifications in various pieces of legislation in our
jurisprudence.

For instance, Republic Act No. 5455 17 provides:chanrob1es virtual 1aw library

SECTION 1. Definitions and scope of this Act. — (1) . . .; and the phrase "doing
business" shall include soliciting orders, purchases, service contracts, opening
offices, whether called "liaison" offices or branches; appointing representatives or
distributors who are domiciled in the Philippines or who in any calendar year stay
in the Philippines for a period or periods totalling one hundred eighty days or
more; participating in the management, supervision or control of any domestic
business firm, entity or corporation in the Philippines; and any other act or acts that
imply a continuity of commercial dealings or arrangements, and contemplate to
that extent the performance of acts or works, or the exercise of some of the
functions normally incident to, and in-progressive prosecution of, commercial gain
or of the purpose and object of the business organization.

Presidential Decree No. 1789, 18 in Article 65 thereof, defines "doing business" to


include soliciting orders, purchases, service contracts, opening offices, whether
called "liaison" offices or branches; appointing representatives or distributors who
are domiciled in the Philippines or who in any calendar year stay in the Philippines
for a period or periods totalling one hundred eighty days or more; participating in
the management, supervision or control of any domestic business firm, entity or
corporation in the Philippines, and any other act or acts that imply a continuity of
commercial dealings or arrangements and contemplate to that extent the
performance of acts or works, or the exercise of some of the functions normally
incident to, and in progressive prosecution of, commercial gain or of the purpose
and object of the business organization.

The implementing rules and regulations of said presidential decree conclude the
enumeration of acts constituting "doing business" with a catch-all definition,
thus:chanrob1es virtual 1aw library

Sec. 1(g).’Doing Business’ shall be any act or combination of acts enumerated in


Article 65 of the Code. In particular ‘doing business’ includes:chanrob1es virtual
1aw library

x       x       x

(10) Any other act or acts which imply a continuity of commercial dealings or
arrangements, and contemplate to the extent the performance of acts or, works, or
the exercise of some of the functions normally incident to, or in the progressive
prosecution of, commercial gain or of the purpose and object of the business
organization.
Finally, Republic Act No. 7042 19 embodies such concept in this wise:chanrob1es
virtual 1aw library

SEC. 3. Definitions. — As used in this Act:chanrob1es virtual 1aw library

x       x       x

(d) the phrase "doing business shall include soliciting orders, service contracts,
opening offices, whether called ‘liaison’ offices or branches; appointing
representatives or distributors domiciled in the Philippines or who in any calendar
year stay in the country for a period or periods totalling one hundred eight(y) (180)
days or more; participating in the management, supervision or control of any
domestic business, firm, entity or corporation in the Philippines; and any other act
or acts that imply a continuity of commercial dealings or arrangements, and
contemplate to that extent the performance of acts or works, or the exercise of
some of the functions normally incident to, and in progressive prosecution of,
commercial gain or of the purpose and object of the business organization:
Provided, however, That the phrase "doing business" shall not be deemed to
include mere investment as a shareholder by a foreign entity in domestic
corporations duly registered to do business, and/or the exercise of rights as such
investors; nor having a nominee director or officer to represent its interests in such
corporation; nor appointing a representative or distributor domiciled in the
Philippines which transacts business in its own name and for its own account.

Based on Article 133 of the Corporation Code and gauged by such statutory
standards, petitioners are not barred from maintaining the present action. There is
no showing that, under our statutory or case law, petitioners are doing, transacting,
engaging in or carrying on business in the Philippines as would require obtention
of a license before they can seek redress from our courts. No evidence has been
offered to show that petitioners have performed any of the enumerated acts or any
other specific act indicative of an intention to conduct or transact business in the
Philippines.

Accordingly, the certification issued by the Securities and Exchange Commission


20 stating that its records do not show the registration of petitioner film companies
either as corporations or partnerships or that they have been licensed to transact
business in the Philippines, while undeniably true, is of no consequence to
petitioners’ right to bring action in the Philippines. Verily, no record of such
registration by petitioners can be expected to be found for, as aforestated, said
foreign film corporations do not transact or do business in the Philippines and,
therefore, do not need to be licensed in order to take recourse to our courts.
Although Section 1(g) of the Implementing Rules and Regulations of the Omnibus
Investments Code lists, among others —

(1) Soliciting orders, purchases (sales) or service contracts. Concrete and specific
solicitations by a foreign firm, or by an agent of such foreign firm, not acting
independently of the foreign firm amounting to negotiations or fixing of the terms
and conditions of sales or service contracts, regardless of where the contracts are
actually reduced to writing, shall constitute doing business even if the enterprise
has no office or fixed place of business in the Philippines. The arrangements
agreed upon as to manner, time and terms of delivery of the goods or the transfer
of title thereto is immaterial. A foreign firm which does business through the
middlemen acting in their own names, such as indentors, commercial brokers or
commission merchants, shall not be deemed doing business in the Philippines. But
such indentors, commercial brokers or commission merchants shall be the ones
deemed to be doing business in the Philippines.

(2) Appointing a representative or distributor who is domiciled in the Philippines,


unless said representative or distributor has an independent status, i.e., it transacts
business in its name and for its own account, and not in the name or for the account
of a principal. Thus, where a foreign firm is represented in the Philippines by a
person or local company which does not act in its name but in the name of the
foreign firm, the latter is doing business in the Philippines.

as acts constitutive of "doing business," the fact that petitioners are admittedly
copyright owners or owners of exclusive distribution rights in the Philippines of
motion pictures or films does not convert such ownership into an indicium of doing
business which would require them to obtain a license before they can use upon a
cause of action in local courts.

Neither is the appointment of Atty. Rico V. Domingo as attorney-in-fact of


petitioners, with express authority pursuant to a special power of attorney, inter alia

To lay criminal complaints with the appropriate authorities and to provide evidence
in support of both civil and criminal proceedings against any person or persons
involved in the criminal infringement of copyright, or concerning the unauthorized
importation, duplication, exhibition or distribution of any cinematographic work(s)
— films or video cassettes — of which . . . is the owner of copyright or the owner
of exclusive rights of owner or copyright or the owner of exclusive rights of
distribution in the Philippines pursuant to any agreement(s) between . . . and the
respective owners of copyright in such cinematographic work(s), to initiate and
prosecute on behalf of . . . criminal or civil actions in the Philippines against any
person or persons unlawfully distributing, exhibiting, selling or offering for sale
any films or video cassettes of which . . . is the owner of copyright or the owner of
exclusive rights of distribution in the Philippines pursuant to any agreement(s)
between . . . and the respective owners of copyright in such works. 21

tantamount to doing business in the Philippines. We fail to see how exercising


one’s legal and property rights and taking steps for the vigilant protection of said
rights, particularly the appointment of an attorney-in-fact, can be deemed by and of
themselves to be doing business here.

As a general rule, a foreign corporation will not be regarded as doing business in


the State simply because it enters into contracts with residents of the State, where
such contracts are consummated outside the State. 22 In fact, a view is taken that a
foreign corporation is not doing business in the state merely because sales of its
product are made there or other business furthering its interests is transacted there
by an alleged agent, whether a corporation or a natural person, where such
activities are not under the direction and control of the foreign corporation but are
engaged in by the alleged agent as an independent business. 23

It is generally held that sales made to customers in the State by an independent


dealer who has purchased and obtained title from the corporation to the products
sold are not a doing of business by the corporation. 24 Likewise, a foreign
corporation which sells its products to persons styled "distributing agents" in the
State, for distribution by then, is not doing business in the State so as to render it
subject to service of process therein, where the contract with these purchasers is
that they shall buy exclusively from the foreign corporation such goods as it
manufactures and shall sell them at trade prices established by it.25cralaw:red

It has moreover been held that the act of a foreign corporation in engaging an
attorney to represent it in a Federal court sitting in a particular State is not doing
business within the scope of the minimum contact test. 26 With much more reason
should this doctrine apply to the mere retainer of Atty. Domingo for legal
protection against contingent acts of intellectual piracy.

In accordance with the rule that "doing business" imports only acts in furtherance
of the purposes for which a foreign corporation was organized, it is held that the
mere institution and prosecution or defense of a suit, particularly if the transaction
which is the basis of the suit took place out of the State, do not amount to the doing
of business in the State. The institution of a suit or the removal thereof is neither
the making of a contract nor the doing of business within a constitutional provision
placing foreign corporations licensed to do business in the State under the same
regulations, limitations and liabilities with respect to such acts as domestic
corporations. Merely engaging in litigation has been considered as not a sufficient
minimum contact to warrant the exercise of jurisdiction over a foreign corporation.
27
As a consideration aside, we have perforce to comment on private respondents’
basis for arguing that petitioners are barred from maintaining suit in the
Philippines. For allegedly being foreign corporations doing business in the
Philippines without a license, private respondents repeatedly maintain in all their
pleadings that petitioners have thereby no legal personality to bring an action
before Philippine courts. 28

Among the grounds for a motion to dismiss under the Rules of Court are lack of
legal capacity to sue 29 and that the complaint states no cause of action. 30 Lack of
legal capacity to sue means that the plaintiff is not in the exercise of his civil rights,
or does not have the necessary qualification to appear in the case, or does not have
the character or representation he claims. 31 On the other hand, a case is
dismissible for lack of personality to sue upon proof that the plaintiff is not the real
party in interest, hence grounded on failure to state a cause of action. 32 The term
"lack of capacity to sue" should not be confused with the term "lack of personality
to sue." While the former refers to a plaintiff’s general disability to sue, such as on
account of minority, insanity, incompetence, lack of juridical personality or any
other general disqualifications of a party, the latter refers to the fact that the
plaintiff is not the real party in interest. Correspondingly, the first can be a ground
for a motion to dismiss based on the ground of lack of legal capacity to sue, 33
whereas the second can be used as a ground for a motion to dismiss based on the
fact that the complaint, on the face thereof, evidently states no cause of action. 34

Applying the above discussion to the instant petition, the ground available for
barring recourse to our courts by an unlicensed foreign corporation doing or
transacting business in the Philippines should properly be "lack of capacity to sue,"
not "lack of personality to sue." Certainly, a corporation whose legal rights have
been violated is undeniable such, if not the only, real party in interest to bring suit
thereon although, for failure to comply with the licensing requirement, it is not
capacitated to maintain any suit before our courts.

Lastly, on this point, we reiterate this Court’s rejection of the common procedural
tactics of erring local companies which, when sued by unlicensed foreign
corporations not engaged in business in the Philippines, invoke the latter’s
supposed lack of capacity to sue. The doctrine of lack of capacity to sue based on
failure to first acquired a local license is based on considerations of public policy.
It was never intended to favor nor insulate from suit unscrupulous establishments
or nationals in case of breach of valid obligations or violations of legal rights of
unsuspecting foreign firms or entities simply because they are not licensed to do
business in the country. 35

II
We now proceed to the main issue of the retroactive application to the present
controversy of the ruling in 20th Century Fox Film Corporation v. Court of
Appeals, Et Al., promulgated on August 19, 1988, 36 that for the determination of
probable cause to support the issuance of a search warrant in copyright
infringement cases involving videograms, the production of the master tape for
comparison with the allegedly pirated copies is necessary.

Petitioners assert that the issuance of a search warrant is addressed to the discretion
of the court subject to the determination of probable cause in accordance with the
procedure prescribed therefor under Section 3 and 4 of Rule 126. as of the time of
the application for the search warrant in question, the controlling creation for the
finding of probable cause was that enunciated in Burgos v. Chief of Staff 37 stating
that:chanrob1es virtual 1aw library

Probable cause for a search warrant is defined as such facts and circumstances
which would lead a reasonably discrete and prudent man to believe that an offense
has been committed and that the objects sought in connection with the offense are
in the place sought to be searched.

According to petitioners, after complying with what the law then required, the
lower court determined that there was probable cause for the issuance of a search
warrant, and which determination in fact led to the issuance and service on
December 14, 1987 of Search Warrant No. 87-053. It is further argued that any
search warrant so issued in accordance with all applicable legal requirements is
valid, for the lower court could not possibly have been expected to apply, as the
basis for a finding of probable cause for the issuance of a search warrant in
copyright infringement cases involving videograms, a pronouncement which was
not existent at the time of such determination, on December 14, 1987, and is, the
doctrine in the 20th Century Fox case that was promulgated only on August 19,
1988, or over eight months later.

Private respondents predictably argue in support of the ruling of the Court of


Appeals sustaining the quashal of the search warrant by the lower court on the
strength of that 20th Century Fox ruling which, they claim, goes into the very
essence of probable cause. At the time of the issuance of the search warrant
involved here, although the 20th Century Fox case had not yet been decided,
Section 2, Article III of the Constitution and Section 3, Rule 126 of the 1985 Rules
on Criminal Procedure embodied the prevailing and governing law on the matter.
The ruling in 20th Century Fox was merely an application of the law on probable
cause. Hence, they posit that there was no law that was retrospectively applied,
since the law had been there all along. To refrain from applying the 20th Century
Fox ruling, which had supervened as a doctrine promulgated at the time of the
solution of private respondents’ motion for reconsideration seeking the quashal of
the search warrant for failure of the trial court to require presentation of the master
tapes prior to the issuance of the search warrant, would have constituted grave
abuse of discretion. 38

Respondent court upheld the retroactive application of the 20th Century Fox ruling
by the trial court in resolving petitioners’ motion for reconsideration in favor of the
quashal of the search warrant, on this renovated thesis:chanrob1es virtual 1aw
library

And whether this doctrine should apply retroactively, it must be noted that in the
20th Century Fox case, the lower court quashed the earlier search warrant it issued.
On certiorari, the Supreme Court affirmed the quashal on the ground among others
that the master tapes or copyrighted films were not presented for comparison with
the purchased evidence of the video tapes to determine whether the latter is an
unauthorized reproduction of the former.

If the lower court in the Century Fox case did not quash the warrant, it is Our view
that the Supreme Court would have invalidated the warrant just the same
considering the very strict requirement set by the Supreme Court for the
determination of ‘probable cause’ in copyright infringement cases as enunciated in
this 20th Century Fox case. This is so because, as was stated by the Supreme Court
in the said case, the master tapes and the pirate tapes must be presented for
comparison to satisfy the requirement of ‘probable cause.’ So it goes back to the
very existence of probable cause. . . . 39

Mindful as we are of the ramifications of the doctrine of stare decisis and the
rudiments of fair play, it is our considered view that the 20th Century Fox ruling
cannot be retroactively applied to the instant case to justify the quashal of Search
Warrant No. 87-053. Herein petitioners’ consistent position that the order of the
lower court of September 5, 1988 denying therein defendants’ motion to lift the
order of search warrant was properly issued, there having been satisfactory
compliance with the then prevailing standards under the law for determination of
probable cause, is indeed well taken. The lower court could not possibly have
expected more evidence from petitioners in their application for a search warrant
other than what the law and jurisprudence, then existing and judicially accepted,
required with respect to the finding of probable cause.

Article 4 of the Civil Code provides that" (l)aws shall have no retroactive effect,
unless the contrary is provided. Correlatively, Article 8 of the same Code declares
that" (j)udicial decisions applying the laws or the Constitution shall form part of
the legal system of the Philippines."cralaw virtua1aw library

Jurisprudence, in our system of government, cannot be considered as an


independent source of law; it cannot create law. 40 While it is true that judicial
decisions which apply or interpret the Constitution or the laws are part of the legal
system of the Philippines, still they are not laws. Judicial decisions, though not
laws, are nonetheless evidence of what the laws mean, and it is for this reason that
they are part of the legal system of the Philippines. 41 Judicial decisions of the
Supreme Court assume the same authority as the statute itself. 42

Interpreting the aforequoted correlated provisions of the Civil Code and in light of
the above disquisition, this Court emphatically declared in Co v. Court of Appeals,
Et. Al. 43 That the principle of prospectivity applies not only to originator
amendatory statutes and administrative rulings and circulars, but also, and properly
so, to judicial decisions. Our holding in the earlier case of People v. Jubinal 44
echoes the rationale for this judicial declaration, vis.:chanrob1es virtual 1aw library

Decisions of this Court, although in themselves not laws, are nevertheless evidence
of what the laws mean, and this is the reason why under Article 8 of the New Civil
Code, "Judicial decisions applying or interpreting the laws or the Constitution shall
form part of the legal system." The interpretation upon a law by this Court
constitutes, in a way, a part of the law as of the date that the law was originally
passed, since this Court’s construction merely establishes the contemporaneous
legislative intent that the law thus construed intends to effectuate. The settled rule
supported by numerous authorities is a restatement of the legal maxim "legis
interpretation legis vim obtinet" — the interpretation placed upon the written law
by a competent court has the force of law. . . ., but when a doctrine of this Court is
overruled and a different view is adopted, the new doctrine should be applied
prospectively, and should not apply to parties who had relied on the old doctrine
and acted on the faith thereof . . . . (Emphasis supplied).

This was forcefully reiterated in Spouses Benzonan v. Court of Appeals, Et Al., 45


where the Court expounded:chanrob1es virtual 1aw library

. . . But while our decisions form part of the law of the land, they are also subject to
Article 4 of the Civil Code which provides that "laws shall have no retroactive
effect unless the contrary is provided." This is expressed in the familiar legal
maximum lex prospicit, non respicit, the law looks forward not backward. The
rationale against retroactivity is easy to perceive. The retroactive application of a
law usually divests rights that have already become vested or impairs the
obligations of contract and hence, is unconstitutional (Francisco v. Certeza, 3
SCRA (565 [1961]). The same consideration underlies our rulings giving only
prospective effect to decisions enunciating new doctrines. . . .

The reasoning behind Senarillos v. Hermosisima 46 that judicial interpretation of a


statute constitutes part of the law as of the date it was originally passed, since the
Court’s construction merely establishes the contemporaneous legislative intent that
the interpreted law carried into effect, is all too familiar. Such judicial doctrine
does not amount to the passage of a new law but consists merely of a construction
or interpretation of a pre-existing one, and that is precisely the situation obtaining
in this case.

It is consequently clear that a judicial interpretation becomes a part of the law as of


the date that law was originally passed, subject only to the qualification that when
a doctrine of this Court is overruled and a different view is adopted, and more so
when there is a reversal thereof, the new doctrine should be applied prospectively
and should not apply to parties who relied on the old doctrine and acted in good
faith. 47 To hold otherwise would be to deprive the law of its quality of fairness
and justice then, if there is no recognition of what had transpired prior to such
adjudication. 48

There is merit in petitioners’ impassioned and well-founded


argumentation:chanrob1es virtual 1aw library

The case of 20th Century Fox Film Corporation v. Court of Appeals, Et Al., 164
SCRA 655 (August 19, 1988) (hereinafter 20th Century Fox) was inexistent in
December of 1987 when Search Warrant 87-053 was issued by the lower court.
Hence, it boggles the imagination how the lower court could be expected to apply
the formulation of 20th Century Fox in finding probable cause when the
formulation was yet non-existent.

x       x       x

In short, the lower court was convinced at that time after conducting searching
examination questions of the applicant and his witnesses that "an offense had been
committed and that the objects sought in connection with the offense (were) in the
place sought to be searched" (Burgos v. Chief of Staff, Et Al., 133 SCRA 800). It
is indisputable, therefore, that at the time of the application, or on December 14,
1987, the lower court did not commit any error nor did it fail to comply with any
legal requirement for the valid issuance of search warrant.

. . . (W)e believe that the lower court should be considered as having followed the
requirements of the law in issuing Search Warrant No. 87-053. The search warrant
is therefore valid and binding. It must be noted that nowhere is it found in the
allegations of the Respondents that the lower court failed to apply the law as then
interpreted in 1987. Hence, we find it absurd that it is (sic) should be seen
otherwise, because it is simply impossible to have required the lower court to apply
a formulation which will only be defined six months later.

Furthermore, it is unjust and unfair to require compliance with legal and/or


doctrinal requirements which are inexistent at the time they were supposed to have
been complied with.
x       x       x

. . . If the lower court’s reversal will be sustained, what encouragement can be


given to courts and litigants to respect the law and rules if they can expect with
reasonable certainty that upon the passage of a new rule, their conduct can still be
open to question? This certainly breeds instability in our system of dispensing
justice. For Petitioners who took special effort to redress their grievances and to
protect their property rights by resorting to the remedies provided by the law, it is
most unfair that fealty to the rules and procedures then obtaining would bear but
fruits of injustice. 49

Withal, even the proposition that the prospectivity of judicial decisions imports
application thereof not only to future cases but also to cases still ongoing or not yet
final when the decision was promulgated, should not be countenanced in the jural
sphere on account of its inevitably unsettling repercussions. More to the point, it is
felt that the reasonableness of the added requirement in 20th Century Fox calling
for the production of the master tapes of the copyrighted films for determination of
probable cause in copyright infringement cases needs revisiting and clarification.

It will be recalled that the 20th Century Fox case arose from search warrant
proceedings in anticipation of the filing of a case for the unauthorized sale or
renting out of copyrighted films in videotape format in violation of Presidential
Decree No. 49. It revolved around the meaning of probable cause within the
context of the constitutional provision against illegal searches and seizures, as
applied to copyright infringement cases involving videotapes.

Therein it was ruled that —

The presentation of master tapes of the copyrighted films from which the pirated
films were allegedly copied, was necessary for the validity of search warrants
against those who have in their possession the pirated films. The petitioner’s
argument to the effect that the presentation of the master tapes at the time of
application may not be necessary as these would be merely evidentiary in nature
and not determinative of whether or not a probable cause exists to justify the
issuance of the search warrants is not meritorious. The court cannot presume that
duplicate or copied tapes were necessarily reproduced from master tapes that it
owns.

The application for search warrants was directed against video tape outlets which
allegedly were engaged in the unauthorized sale and renting out of copyrighted
films belonging to the petitioner pursuant to P.D. 49.

The essence of a copyright infringement is the similarity or at least substantial


similarity of the purported pirated works to the copyrighted work. Hence, the
applicant must present to the court the copyrighted films to compare them with the
purchased evidence of the video tapes allegedly pirated to determine whether the
latter is an unauthorized reproduction of the former. This linkage of the
copyrighted films to the pirated films must be established to satisfy the
requirements of probable cause. Mere allegations as to the existence of the
copyrighted films cannot serve as basis for the issuance of a search warrant.

For a closer and more perspicuous appreciation of the factual antecedents of 20th
Century Fox, the pertinent portions of the decision therein are quoted hereunder, to
wit:jgc:chanrobles.com.ph

"In the instant case, the lower court lifted the three questioned search warrants
against the private respondents on the ground that it acted on the application for the
issuance of the said search warrants and granted it on the misrepresentations of
applicant NBI and its witnesses that infringement of copyright or a piracy of a
particular film have been committed. Thus the lower court stated in its questioned
order dated January 2, 1986:jgc:chanrobles.com.ph

"According to the movant, all three witnesses during the proceedings in the
application for the three search warrants testified of their own personal knowledge.
Yet, Atty. Albino Reyes of the NBI stated that the counsel or representative of the
Twentieth Century Fox Corporation will testify on the video cassettes that were
pirated, so that he did not have personal knowledge of the alleged piracy. The
witness Bacani also said that the video cassettes were pirated without stating the
manner it was pirated and that it was Atty. Domingo that has knowledge of that
fact.

"On the part of Atty. Domingo, he said that the re-taping of the allegedly pirated
tapes was from master tapes allegedly belonging to the Twentieth Century Fox,
because, according to him it is of his personal knowledge.

"At the hearing of the Motion for Reconsideration, Senior NBI Agent Atty. Albino
Reyes testified that when the complaint for infringement was brought to the NBI,
the master tapes of the allegedly pirated tapes were shown to him and he made
comparisons of the tapes with those purchased by their man Bacani. Why the
master tapes or at least the film reels of the allegedly pirated tapes were not shown
to the Court during the application gives some misgivings as to the truth of that
bare statement of the NBI agent on the witness stand.

"Again as the application and search proceedings is a prelude to the filing of


criminal cases under PD 49, the copyright infringement law, and although what is
required for the issuance thereof is merely the presence of probable cause, that
probable cause must be satisfactory to the Court, for it is a time-honored precept
that proceedings to put a man to task as an offender under our laws should be
interpreted in strictissimi juris against the government and liberally in favor of the
alleged offender.

x       x       x

"This doctrine has never been overturned, and as a matter of fact it had been
enshrined in the Bill of Rights in our 1973 Constitution.

"So that lacking in persuasive effect, the allegation that master tapes were viewed
by the NBI and were compared to the purchased and seized video tapes from the
respondents’ establishments, it should be dismissed as not supported by competent
evidence and for that matter the probable cause hovers in that grey debatable
twilight zone between black and white resolvable in favor of respondents herein.

"But the glaring fact is that ‘Cocoon,’ the first video tape mentioned in the search
warrant, was not even duly registered or copyrighted in the Philippines. (Annex C
of Opposition p. 152 record.) So, that lacking in the requisite presentation to the
Court of an alleged master tape for purposes of comparison with the purchased
evidence of the video tapes allegedly pirated and those seized from respondents,
there was no way to determine whether there really was piracy, or copying of the
film of the complainant Twentieth Century Fox."cralaw virtua1aw library

x       x       x

The lower court, therefore, lifted the three (3) questioned search warrants in the
absence of probable cause that the private respondents violated P.D. 49. As found
by the court, the NBI agents who acted as witnesses did not have personal
knowledge of the subject matter of their testimony which was the alleged
commission of the offense by the private respondents. Only the petitioner’s
counsel who was also a witness during the application for the issuance of the
search warrants stated that he had personal knowledge that the confiscated tapes
owned by the private, respondents were pirated tapes taken from master tapes
belonging to the petitioner. However, the lower court did not give much credence
to his testimony in view of the fact that the master tapes of the allegedly pirated
tapes were not shown to the court during the application." (Emphasis ours).

The italicized passages readily expose the reason why the trial court therein
required the presentation of the master tapes of the allegedly pirated films in order
to convince itself of the existence of probable cause under the factual milieu
peculiar to that case. In the case at bar, respondent appellate court itself
observed:chanrob1es virtual 1aw library
We feel that the rationale behind the aforequoted doctrine is that the pirated copies
as well as the master tapes, unlike the other types of personal properties which may
be seized, were available for presentation to the court at the time of the application
for a search warrant to determine the existence of the linkage of the copyrighted
films with the pirated ones. Thus, there is no reason not the present them (Italics
supplied for emphasis). 50

In fine, the supposed pronunciamento in said case regarding the necessity for the
presentation of the master tapes of the copyrighted films for the validity of search
warrants should at most be understood to merely serve as a guidepost in
determining the existence of probable cause in copyright infringement cases where
there is doubt as to the true nexus between the master tape and the pirated copies.
An objective and careful reading of the decision in said case could lead to no other
conclusion than that said directive was hardly intended to be a sweeping and
inflexible requirement in all or similar copyright infringement cases. Judicial dicta
should always be construed within the factual matrix of their parturition, otherwise
a careless interpretation thereof could unfairly fault the writer with the vice of
overstatement and the reader with the fallacy of undue generalization.

In the case at bar, NBI Senior Agent Lauro C. Reyes who filed the application for
search warrant with the lower court following a formal complaint lodged by
petitioners, judging from his affidavit 51 and his deposition, 52 did testify on
matters within his personal knowledge based on said complaint of petitioners as
well as his own investigation and surveillance of the private respondents’ video
rental shop. Likewise, Atty. Rico V. Domingo, in his capacity as attorney-at-fact,
stated in his affidavit 53 and further expounded in his deposition 54 that he
personally knew of the fact that private respondents had never been authorized by
his clients to reproduce, lease and possess for the purpose of selling any of the
copyrighted films.

Both testimonies of Agent Reyes and Atty. Domingo were corroborated by Rene
C. Baltazar, a private researcher retained by Motion Pictures Association of
America, Inc. (MPAA, Inc.), who was likewise presented as a witness during the
search warrant proceedings. 55 The records clearly reflect that the testimonies of
the above named witnesses were straightforward and stemmed from matters within
their personal knowledge. They displayed none of the ambivalence and uncertainty
that the witnesses in the 20th Century Fox case exhibited. This categorical
forthrightness in their statements, among others, was what initially and correctly
convinced the trial court to make a finding of the existence of probable cause.

There is no originality in the argument of private respondents against the validity


of the search warrant, obviously borrowed from 20th Century Fox, that petitioners’
witnesses — NBI Agent Lauro C. Reyes, Atty. Rico V. Domingo and Rene C.
Baltazar — did not have personal knowledge of the subject matter of their
respective testimonies and that said witnesses’ claim that the video tapes were
pirated, without stating the manner by which these were pirated, is a conclusion of
fact without basis. 56 The difference, it must be pointed out, is that the records in
the present case reveal that (1) there is no allegation of misrepresentation, much
less a finding thereof by the lower court, on the part of petitioners’ witnesses; (2)
there is no denial on the part of private respondents that the tapes seized were
illegitimate copies of the copyrighted ones nor have they shown that they were
given any authority by petitioners to copy, sell, lease, distribute or circulate, or at
least, to offer for sale, lease, distribution or circulation the said video tapes; and (3)
a discreet but extensive surveillance of the suspected area was undertaken by
petitioner’s witnesses sufficient to enable them to execute trustworthy affidavits
and depositions regarding matters discovered in the course thereof and of which
they have personal knowledge.

It is evidently incorrect to suggest, as the ruling in 20th Century Fox may appear to
do, that in copyrighted films is always necessary to meet the requirement of
probable cause and that, in the absence thereof, there can be no finding of probable
cause for the issuance of a search warrant. It is true that such master tapes are
object evidence, with the merit that in this class of evidence the ascertainment of
the controverted fact is made through demonstrations involving the direct use of
the senses of the presiding magistrate. 57 Such auxiliary procedure, however, does
not rule out the use of testimonial or documentary evidence, depositions,
admissions or other classes of evidence tending to prove the factum probandum, 58
especially where the production in court of object evidence would result in delay,
inconvenience or expenses out of proportion to its evidentiary value. 59

Of course, as a general rule, constitutional and statutory provisions relating to


search warrants prohibits their issuance except on a showing of probable cause,
supported by oath or affirmation. These provisions prevent the issuance of warrants
on loose, vague, or doubtful bases of fact, and emphasize the purpose to protect
against all general searches. 60 Indeed, Article III of our Constitution mandates in
Sec. 2 thereof that no search warrant shall issue except upon probable cause to be
determined personally by the judge after examination under oath or affirmation of
the complainant and the witnesses he may produce, and particularly describing the
place to be searched and the things to be seized; and Sec. 3 thereof provides that
any evidence obtained in violation of the preceding section shall be inadmissible
for any purpose in any proceeding.

These constitutional strictures are implemented by the following provisions of


Rule 126 of the Rules of Court:chanrob1es virtual 1aw library

Sec. 3. Requisites for issuing search warrant. — A search warrant shall not issue
but upon probable cause in connection with one specific offense to be determined
personally by the judge after examination under oath or affirmation of the
complainant and the witnesses he may produce, and particularly describing the
place to be searched and the things to be seized.

Sec. 4. Examination of complainant; record. — The judge must, before issuing the
warrant, personally examine in the form of searching questions and answers, in
writing and under oath the complainant and any witnesses he may produce on facts
personally known to them and attach to the record their sworn statements together
with any affidavits submitted.

Sec. 5. Issuance and form of search warrant. — If the judge is thereupon satisfied
of the existence of facts upon which the application is based, or that there is
probable cause to believe that they exist, he must issue the warrant, which must be
substantially in the form prescribed by these Rules.

The constitutional and statutory provisions of various jurisdictions requiring a


showing of probable cause before a search warrant can be issued are mandatory
and must be complied with, and such a showing has been held to be an unqualified
condition precedent to the issuance of a warrant. A search warrant not based on
probable cause is a nullity, or is void, and the issuance thereof is, in legal
contemplation, arbitrary. 61 It behooves us, then, to review the concept of probable
cause, firstly, from representative holdings in the American jurisdiction from
which we patterned our doctrines on the matter.

Although the term "probable cause" has been said to have a well-defined meaning
in the law, the term is exceedingly difficult to define, in this case, with any degree
of precision; indeed, no definition of it which would justify the issuance of a search
warrant can be formulated which would cover every state of facts which might
arise, and no formula or standard, or hard and fast rule, may be laid down which
may be applied to the facts of every situation. 62 As to what acts constitute
probable cause seem incapable of definition. 63 There is, of necessity, no exact
test. 64

At best, the term "probable cause" has been understood to mean a reasonable
ground of suspicion, supported by circumstances sufficiently strong in themselves
to warrant a cautious man in the belief that the person accused is guilty of the
offense with which he is charged; 65 or the existence of such facts and
circumstances as would excite an honest belief in a reasonable mind acting on all
the facts and circumstances within the knowledge of the magistrate that the charge
made by the applicant for the warrant is true. 66

Probable cause does not mean actual and positive cause, nor does it important
absolute certainty. The determination of the existence of probable cause is not
concerned with the question of whether the offense charged has been or is being
committed in fact, or whether the accused is guilty or innocent, but only whether
the affiant has reasonable grounds for his belief. 67 The requirement is less than
certainty or proof, but more than suspicion or possibility. 68

In Philippine jurisprudence, probable cause has been uniformly defined as such


facts and circumstances which would lead a reasonable, discreet and prudent man
to believe that an offense has been committed, and that the objects sought in
connection with the offense are in the place sought to be searched. 69 It being the
duty of the issuing officer to issue, or refuse to issue, the warrant as soon as
practicable after the application therefor is filed, 70 the facts warranting the
conclusion of probable cause must be assessed at the time of such judicial
determination by necessarily using legal standards then set forth in law and
jurisprudence, and not those that have yet to be crafted thereafter.

As already stated, the definition of probable cause enunciated in Burgos, Sr. v.


Chief of Staff, Et Al., supra, vis-a-vis the provisions of Sections 3 and 4 of Rule
126, were the prevailing and controlling legal standards, as they continue to be, by
which a finding or probable cause is tested. Since the proprietary of the issuance of
a search warrant is to be determined at the time of the application therefor, which
in turn must not be too remote in time from the occurrence of the offense alleged to
have been committed, the issuing judge, in determining the existence of probable
cause, can and should logically look to the touchstones in the laws therefore
enacted and the decisions already promulgated at the time, and not to those which
had not yet even been conceived or formulated.

It is worth noting that neither the Constitution nor the Rules of Court attempt to
define probable cause, obviously for the purpose of leaving such matter to the
court’s discretion within the particular facts of each case. Although the
Constitution prohibits the issuance of a search warrant in the absence of probable
cause, such constitutional inhibition does not command the legislature to establish
a definition or formula for determining what shall constitute probable cause. 71
Thus, Congress, despite its broad authority to fashion standards of reasonableness
for searches and seizures, 72 does not venture to make such a definition or standard
formulation of probable cause, nor categorize what facts and circumstances make
up the same, much less limit the determination thereof to and within the
circumscription of a particular class of evidence, all in deference to judicial
discretion and probity. 73

Accordingly, to restrict the exercise of discretion by a judge by adding a particular


requirement (the presentation of master tapes, as intimated by 20th Century Fox)
not provided nor implied in the law for a finding of probable cause is beyond the
realm of judicial competence or statesmanship. It serves no purpose but to stultify
and constrict the judicious exercise of a court’s prerogatives and to denigrate the
judicial duty of determining the existence of probable cause to a mere ministerial
or mechanical function. There is, to repeat, no law or rule which requires that the
existence of probable cause is or should be determined solely by a specific kind of
evidence. Surely, this could not have been contemplated by the framers of the
Constitution, and we do not believe that the Court intended the statement in 20th
Century Fox regarding master tapes as the dictum for all seasons and reasons in
infringement cases.

Turning now to the case at bar, it can be gleaned from the records that the lower
court followed the prescribed procedure for the issuance of a search warrant: (1)
the examination under oath or affirmation of the complainant and his witnesses,
with them particularly describing the place to be searched and the things to be
seized; (2) an examination personally conducted by the judge in the form of
searching questions and answers, in writing and under oath of the complainant and
witnesses on facts personally known to them; and, (3) the taking of sworn
statements, together with the affidavits submitted, which were duly attached to the
records.

Thereafter, the court a quo made the following factual findings leading to the
issuance of the search warrant now subject to this controversy:chanrob1es virtual
1aw library

In the instant case, the following facts have been established: (1) copyrighted video
tapes bearing titles enumerated in Search Warrant No. 87-053 were being sold,
leased, distributed or circulated, or offered for sale, lease, distribution, or
transferred or caused to be transferred by defendants at their video outlets, without
the written consent of the private complainants or their assignee; (2) recovered or
confiscated from defendants’ possession were video tapes containing copyrighted
motion picture films without the authority of the complainant; (3) the video tapes
originated from spurious or unauthorized persons; and (4) said video tapes were
exact reproductions of the film listed in the search warrant whose copyrights or
distribution rights were owned by complainants.

The basis of these facts are the affidavits and depositions of NBI Senior Agent
Lauro C. Reyes, Atty. Rico V. Domingo, and Rene C. Baltazar. Motion Pictures
Association of America, Inc. (MPAA) thru their counsel, Atty. Rico V. Domingo,
filed a complaint with the National Bureau of Investigation against certain video
establishments one of which is defendant, for violation of PD No. 49 as amended
by PD No. 1988. Atty. Lauro C. Reyes led a team to conduct discreet surveillance
operations on said video establishments. Per information earlier gathered by Atty.
Domingo, defendants were engaged in the illegal sale, rental, distribution,
circulation or public exhibition of copyrighted films of MPAA without its written
authority or its members. Knowing that defendant Sunshine Home Video and its
proprietor, Mr. Danilo Pelindario, were not authorized by MPAA to reproduce,
lease, and possess for the purpose of selling any of its copyrighted motion pictures,
he instructed his researcher, Mr. Rene Baltazar to rent two video cassettes from
said defendants on October 21, 1987. Rene C. Baltazar proceeded to Sunshine
Home Video and rented tapes containing Little Shop of Horror. He was issued
rental slip No. 26362 dated October 21 1987 for P10.00 with a deposit of P100.00.
Again, on December 11, 1987, he returned to Sunshine Home Video and rented
Robocop with rental slip No. 25271 also for P10.00. On the basis of the complaint
of MPAA thru counsel, Atty. Lauro C. Reyes personally went to Sunshine Home
Video at No. 6 Mayfair Center, Magallanes Commercial Center, Makati. His last
visit was on December 7, 1987. There, he found the video outlet renting, leasing,
distributing video cassette tapes whose titles were copyrighted and without the
authority of MPAA.

Given these facts, a probable cause exists. . . . 74

The lower court subsequently executed a volte-face, despite its prior detailed and
substantiated findings, by stating in its order of November 22, 1988 denying
petitioners’ motion for reconsideration and quashing the search warrant that —

. . . The two (2) cases have a common factual milieu; both involve alleged pirated
copyrighted films of private complainants which were found in the possession or
control of the defendants. Hence, the necessity of the presentation of the master
tapes from which the pirated films were allegedly copied is necessary in the instant
case, to establish the existence of probable cause. 75

Being based solely on an unjustifiable and improper retroactive application of the


master tape requirement generated by 20th Century Fox upon a factual situation
completely different from that in the case at bar, and without anything more, this
later order clearly defies elemental fair play and is a gross reversible error. in fact,
this observation of the Court in La Chemise Lacoste, S.A. v. Fernandez, Et Al.,
supra, may just as easily apply to the present case:chanrob1es virtual 1aw library

A review of the grounds invoked . . . in his motion to quash the search warrants
reveals the fact that they are not appropriate for quashing a warrant. They are
matters of defense which should be ventilated during the trial on the merits of case.
...

As correctly pointed out by petitioners, a blind espousal of the requisite of


presentation of the master tapes in copyright infringement cases, as the prime
determinant of probable cause, is too exacting and impracticable a requirement to
be complied with in a search warrant application which, it must not be overlooked,
is only an ancillary proceeding. Further, on realistic considerations, a strict
application of said requirement militates against the elements of secrecy and speed
which underlie covert investigative and surveillance operations in police
enforcement campaigns against all forms of criminality, considering that the
master tapes of a motion picture required to be presented before the court consists
of several reels contained in circular steel casings which, because of their bulk, will
definitely draw attention, unlike diminutive objects like video tapes which can be
easily concealed. 76 With hundreds of titles being pirated, this onerous and tedious
imposition would be multiplied a hundredfold by judicial fiat, discouraging and
preventing legal recourses in foreign jurisdictions.

Given the present international awareness and furor over violations in large scale
of intellectual property rights, calling for transnational sanctions, it bears calling to
mind the Court’s admonition also in La Chemise Lacoste, supra, that —

. . . Judges all over the country are well advised to remember that court processes
should not be used as instruments to, unwittingly or otherwise, aid counterfeiters
and intellectual pirates, tie the hands of the law as it seeks to protect the Filipino
consuming public and frustrate executive and administrative implementation of
solemn commitments pursuant to international conventions and treaties.

III

The amendment of Section 56 of Presidential Decree No. 49 by Presidential


Decree No. 1987, 77 which would here be publicized judicially, brought about the
revision of its penalty structure and enumerated additional acts considered violative
of said decree on intellectual property, namely, (1) directly or indirectly
transferring or causing to be transferred any sound recording or motion picture or
other audio-visual works so recorded with intent to sell, lease, publicly exhibit or
cause to be sold, leased or publicly exhibited, or to use or cause to be used for
profit such articles on which sounds, motion pictures, or other audio-visual works
are so transferred without the written consent of the owner or his assignee; (2)
selling, leasing, distributing, circulating, publicly exhibiting, or offering for sale,
lease, distribution, or possessing for the purpose of sale, lease, distribution,
circulation or public exhibition any of the abovementioned articles, without the
written consent of the owner or his assignee; and, (3) directly or indirectly offering
or making available for a fee, rental, or any other form of compensation any
equipment, machinery, paraphernalia or any material with the knowledge that such
equipment, machinery, paraphernalia or material will be used by another to
reproduce, without the consent of the owner, any phonograph record, disc, wire,
tape, film or other article on which sounds, motion pictures or other audio-visual
recordings may be transferred, and which provide distinct bases for criminal
prosecution, being crimes independently punishable under Presidential Decree No.
49, as amended, aside from the act of infringing or aiding or abetting such
infringement under Section 29.

The trial court’s finding that private respondents committed acts in blatant
transgression of Presidential Decree No. 49 all the more bolsters its findings of
probable cause, which determination can be reached even in the absence of master
tapes by the judge in the exercise of sound discretion. The executive concern and
resolve expressed in the foregoing amendments to the decree for the protection of
intellectual property rights should be matched by corresponding judicial vigilance
and activism, instead of the apathy of submitting to technicalities in the face of
ample evidence of guilt.

The essence of intellectual piracy should be essayed in conceptual terms in order to


underscore its gravity by an appropriate understanding thereof. Infringement of a
copyright is a trespass on a private domain owned and occupied by the owner of
the copyright, and, therefore, protected by law, and infringement of copyright, or
piracy, which is a synonymous term in this connection, consists in the doing by
any person, without the consent of the owner of the copyright, of anything the sole
right to do which is conferred by statute on the owner of the copyright. 78

A copy of a piracy is an infringement of the original, and it is no defense that the


pirate, in such cases, did not know what works he was indirectly copying, or did
not know whether or not he was infringing any copyright; he at least knew that
what he was copying was not his, and he copied at his peril. In determining the
question of infringement, the amount of matter copied from the copyrighted work
is an important consideration. To constitute infringement, it is not necessary that
the whole or even a large portion of the work shall have been copied. If so much is
taken that the value of the original is sensibly diminished, or the labors of the
original author are substantially and to an injurious extent appropriated by another,
that is sufficient in point of law to constitute a piracy. 79 The question of whether
there has been an actionable infringement of a literary, musical, or artistic work in
motion pictures, radio or television being one of fact, 80 it should properly be
determined during the trial. That is the stage calling for conclusive or
preponderating evidence, and not the summary proceeding for the issuance of a
search warrant wherein both lower courts erroneously require the master tapes.

In disregarding private respondent’s argument that Search Warrant No. 87-053 is a


general warrant, the lower court observed that "it was worded in a manner that the
enumerated seizable items bear direct relation to the offense of violation of Sec. 56
of PD 49 as amended. It authorized only the seizur(e) of articles used or intended
to be used in the unlawful sale, lease and other unconcerted acts in violation of PD
49 as amended. . . ." 81

On this point, Bache and Co., (Phil.), Inc., Et. Al. v. Ruiz, Et Al., 82 instructs and
enlightens:chanrob1es virtual 1aw library

As search warrant may be said to particularly describe the things to be seized when
the description therein is as specific as the circumstances will ordinarily allow
(People v. Rubio, 57 Phil. 384); or when the description expresses a conclusion of
fact — not of law — by which the warrant officer may be guided in making the
search and seizure (idem., dissent of Abad Santos, J.,); or when the things
described are limited to those which bear direct relation to the offense for which
the warrant is being issued (Sec 2, Rule 126, Revised Rules of Court). . . . If the
articles desired to be seized have any direct relation to an offense committed, the
applicant must necessarily have some evidence, other than those articles, to prove
the said offense; and the articles subject of search and seizure should come in
handy merely to strengthen such evidence. . . .

On private respondents’ averment that the search warrant was made applicable to
more than one specific offense on the ground that there are as many offenses of
infringement as there are rights protected and, therefore, to issue one search
warrant for all the movie titles allegedly pirated violates the rule that a search
warrant must be issued only in connection with one specific offense, the lower
court said:chanrob1es virtual 1aw library

. . . As the face of the search warrant itself indicates, it was issued for violation of
Section 56, PD 49 as amended only. The specifications therein (in Annex A)
merely refer to the titles of the copyrighted motion pictures/films belonging to
private complainants which defendants were in control/possession for sale, lease,
distribution or public exhibition in contravention of Sec. 56, PD 49 as amended. 83

That there were several counts of the offense of copyright infringement and the
search warrant uncovered several contraband items in the form of pirate video
tapes is not to be confused with the number of offenses charged. The search
warrant herein issued does not violate the one-specific-offense rule.

It is pointless for private respondents to insist on compliance with the registration


and deposit requirements under presidential Decree No. 49 as prerequisites for
invoking the court’s protective mantle in copyright infringement cases. As
explained by the court below:chanrob1es virtual 1aw library

Defendants-movants contend that PD 49 as amended covers only producers who


have complied with the requirements of deposit and notice (in other words
registration) under Sections 49 and 50 thereof. Absent such registration, as in this
case, there was no right created, hence, no infringement under PD 49 as amended.
This is not well-taken.

As correctly pointed out by private complainants-oppositors, the Department of


Justice has resolved this legal question as far back as December 12, 1978 in its
Opinion No. 191 of the then Secretary of Justice Vicente Abad Santos which stated
that Sections 26 and 50 do not apply to cinematographic works and PD No. 49
"had done away with the registration and deposit of cinematographic works" and
that "even without prior registration and deposit of a work which may be entitled to
protection under the Decree, the creator can file action for infringement of its
rights." He cannot demand, however, payment of damages arising from
infringement. The same opinion stressed that "the requirements of registration and
deposit are thus retained under the Decree, not as conditions for the acquisition of
copyright and other rights, but as prerequisites to a suit for damages." The statutory
interpretation of the Executive Branch being correct, is entitled (to) weight and
respect.

x       x       x

Defendants-movants maintain that complainant and his witnesses led the Court to
believe that a crime existed when in fact there was none. This is wrong. As earlier
discussed, PD 49 as amended, does not require registration and deposit for a
creator to be able to file an action for infringement of his rights. These conditions
are merely pre-requisites to an action for damages. So, as long as the proscribed
acts are shown to exist, an action for infringement may be initiated. 84

Accordingly, the certifications 85 from the Copyright Section of the National


Library, presented as evidence by private respondents to show non-registration of
some of the films of petitioners, assume no evidentiary weight or significance,
whatsoever.

Furthermore, a closer review of Presidential Decree No. 49 reveals that even with
respect to works which are required under Section 26 thereof to be registered and
with copies to deposited with the National Library, such as books, including
composite and cyclopedic works, manuscripts, directories and gazetteers; and
periodicals, including pamphlets and newspapers; lectures, sermons, addresses,
dissertations prepared for oral delivery; and letters, the failure to comply with said
requirements does not deprive the copyright owner of the right to sue for
infringement. Such non-compliance merely limits the remedies available to him
and subjects him to the corresponding sanction.

The reason for this is expressed in Section 2 of the decree which prefaces its
enumeration of copyrightable works with the explicit statement that "the rights
granted under this Decree shall, from the moment of creation, subsist with respect
to any of the following classes of works." This means that under the present state
of the law, the copyright for a work is acquired by an intellectual creator from the
moment of creation even in the absence of registration and deposit. As has been
authoritatively clarified:chanrob1es virtual 1aw library

The registration and deposit of two complete copies or reproductions of the work
with the National library within three weeks after the first public dissemination or
performance of the work, as provided for in Section 26 (P.D. No. 49, as amended),
is not for the purpose of securing a copyright of the work, but rather to avoid the
penalty for non-compliance of the deposit of said two copies and in order to
recover damages in an infringement suit. 86

One distressing observation. This case has been fought on the basis of, and its
resolution long delayed by resort to, technicalities to a virtually abusive extent by
private respondents, without so much as an attempt to adduce any credible
evidence showing that they conduct their business legitimately and fairly. The fact
that private respondents could not show proof of their authority or that there was
consent from the copyright owners for them to sell, lease, distribute or circulate
petitioners’ copyrighted films immeasurably bolsters the lower court’s initial
finding of probable cause. That private respondents are licensed by the Videogram
Regulatory Board does not insulate them from criminal and civil liability for their
unlawful business practices. What is more deplorable is that the reprehensible acts
of some unscrupulous characters have stigmatized the Philippines with an
unsavory reputation as a hub for intellectual piracy in this part of the globe,
formerly in the records of the General Agreement on Tariffs and Trade and, now,
of the World Trade Organization. Such acts must not be glossed over but should be
denounced and repressed lest the Philippines become an international pariah in the
global intellectual community.

WHEREFORE, the assailed judgment and resolution of respondent Court of


Appeals, and necessarily inclusive of the order of the lower court dated November
22, 1988, are hereby REVERSED and SET ASIDE. The order of the court a quo of
September 5, 1988 upholding the validity of Search Warrant No. 87-053 is hereby
REINSTATED, and said court is DIRECTED to take and expeditiously proceed
with such appropriate proceedings as may be called for in this case. Trebles costs
are further assessed against private respondents.

SO ORDERED

People v. Tee, G. R. Nos. 140546-47, January 20, 2003

For automatic review is the consolidated judgment 1 of the Regional Trial Court
(RTC) of Baguio City, Branch 6, dated September 17, 1999, in Criminal Cases
Nos. 15800-R and 15822-R, involving violations of Section 8, Article II, of the
Dangerous Drugs Law.2 Since appellant was acquitted in the second case, we focus
on the first case, where appellant has been found guilty and sentenced to death and
fined one million pesos.

The decretal portion of the trial court’s decision reads:

WHEREFORE, judgment is hereby rendered, as follows:


1. In Crim. Case No. 15800-R, the Court finds the accused Modesto Tee
guilty beyond reasonable doubt of the offense of illegal possession of
marijuana of about 591.81 kilos in violation of Section 8, Article II of RA
6425 as amended by Section 13 of RA 7659 as charged in the Information,
seized by virtue of a search warrant and sentences him to the supreme
penalty of death and to pay a fine of 1 million pesos without subsidiary
imprisonment in case of insolvency.

The 591.81 kilos of marijuana contained in 26 boxes and one yellow sack
(Exhibits U-1 to U-27) are ordered forfeited in favor of the State to be
destroyed immediately in accordance with law.

2. In Crim. Case No. 15822-R, the Court finds that the prosecution failed to
prove the guilt of accused Modesto Tee beyond reasonable doubt and hereby
acquits him of the charge of illegal possession of marijuana in violation of
Section 8, Art. 2 of RA 6425 as amended by Section 13 of RA 7659 as
charged in the Information since the marijuana confiscated have to be
excluded in evidence as a product of unreasonable search and seizure.

The 336.93 kilos of marijuana contained in 13 sacks and four boxes (Exh. B
to S and their component parts) although excluded in evidence as the
product(s) of unreasonable search and seizure, are nevertheless ordered
forfeited in favor of the State to be destroyed immediately in accordance
with law considering that they are prohibited articles.

The City Jail Warden is, therefore, directed to release the accused Modesto
Tee in connection with Crim. Case No. 15822-R unless held on other
charges.

COST(S) DE OFFICIO.

SO ORDERED.3

Appellant is a Chinese national in his forties, a businessman, and a resident of


Baguio City. A raid conducted by operatives of the National Bureau of
Investigation (NBI) and Philippine National Police Narcotics Command (PNP
NARCOM) at premises allegedly leased by appellant and at his residence yielded
huge quantities of marijuana.

On July 20, 1998, appellant moved to quash the search warrant on the ground that
it was too general and that the NBI had not complied with the requirements for the
issuance of a valid search warrant. The pendency of said motion, however, did not
stop the filing of the appropriate charges against appellant. In an information dated
July 24, 1998, docketed as Criminal Case No. 15800-R, the City Prosecutor of
Baguio City charged Modesto Tee, alias "Estoy Tee," with illegal possession of
marijuana, allegedly committed as follows:

That on or about the 1st day of July, 1998 in the City of Baguio, Philippines,
and within the jurisdiction of this Honorable Court, the above-named
accused, did then and there willfully, unlawfully, feloniously and knowingly
have in his possession the following, to wit:

1. Ninety-two (92) bricks of dried flowering tops separately contained in


four (4) boxes; and

2. One hundred fifty-eight (158) bricks, twenty-one (21) blocks, and twenty-
three (23) bags of dried flowering tops separately contained in thirteen (13)
sacks, with a total weight of 336.93 kilograms; and

3 Six hundred two (602) bricks of dried flowering tops separately contained
in twenty-six (boxes) and a yellow sack, weighing 591.81 kilograms,

all having a grand total weight of 928.74 kilograms, a prohibited drug,


without the authority of law to possess, in violation of the above-cited
provision of law.

CONTRARY TO LAW.4

On August 7, 1998, the prosecution moved to "amend" the foregoing charge sheet
"considering that subject marijuana were seized in two (2) different places." 5

As a result, the information in Criminal Case No. 15800-R was amended to read as
follows:

That on or about the 1st day of July, 1998, in the City of Baguio,
Philippines, and within the jurisdiction of this Honorable Court, the above-
named accused, did then and there willfully, unlawfully, feloniously and
knowingly have in his possession the following, to wit:

- Six hundred two (602) bricks of dried flowering tops separately contained
in twenty-six (26) boxes and a yellow sack, weighing 591.81 kilograms

a prohibited drug, without the authority of law to possess, in violation of the


above-cited provision of law.

CONTRARY TO LAW.6

A separate amended information docketed as Criminal Case No. 15822-R was


likewise filed, the accusatory portion of which reads:
That on or about the 1st day of July, 1998 in the City of Baguio, Philippines,
and within the jurisdiction of this Honorable Court, the above-named
accused, did then and there willfully, unlawfully, feloniously and knowingly
have in his possession the following, to wit:

1. Ninety-two (92) bricks of dried flowering tops separately contained


in four (4) boxes; and

2. hundred fifty-eight (158) bricks, twenty-one (21) blocks, and


twenty-three (23) bags of dried flowering tops separately contained in
thirteen (13) sacks, with a total weight of 336.93 kilograms;

a prohibited drug, without the authority of law to possess, in violation of the


above-cited provision of law.

CONTRARY TO LAW.7

On September 4, 1998, the trial court denied the motion to quash the search
warrant and ordered appellant’s arraignment.

When arraigned in Criminal Cases Nos. 15800-R and 15822-R, appellant refused
to enter a plea. The trial court entered a plea of not guilty for him. 8 Trial on the
merits then ensued.

The facts of this case, as gleaned from the records, are as follows:

Prosecution witness Danilo Abratique, a Baguio-based taxi driver, and the


appellant Modesto Tee are well acquainted with each other, since Abratique’s wife
is the sister of Tee’s sister-in-law.9

Sometime in late June 1998, appellant asked Abratique to find him a place for the
storage of smuggled cigarettes.10 Abratique brought appellant to his friend, Albert
Ballesteros, who had a house for rent in Bakakeng, Baguio City. After negotiating
the terms and conditions, Ballesteros agreed to rent out his place to appellant.
Appellant then brought several boxes of purported "blue seal" cigarettes to the
leased premises.

Shortly thereafter, however, Ballesteros learned that the boxes stored in his place
were not "blue seal" cigarettes but marijuana. Fearful of being involved,
Ballesteros informed Abratique. Both later prevailed upon appellant to remove
them from the premises.11

Appellant then hired Abratique’s taxi and transported the boxes of cannabis from
the Ballesteros place to appellant’s residence at Km. 6, Dontogan, Green Valley,
Sto. Tomas, Baguio City.12
On June 30, 1998, appellant hired Abratique to drive him to La Trinidad, Benguet
on the pretext of buying and transporting strawberries. Upon reaching La Trinidad,
however, appellant directed Abratique to proceed to Sablan, Benguet, where
appellant proceeded to load several sacks of marijuana in Abratique’s taxi. He then
asked Abratique to find him a place where he could store the contraband. 13

Abratique brought appellant to his grandmother’s house at No. 27 Dr. Cariño St.,
QM Subdivision, Baguio City, which was being managed by Abratique’s aunt,
Nazarea Abreau. Nazarea agreed to rent a room to appellant. Abratique and
appellant unloaded and stored there the sacks of marijuana brought from
Sablan.14 Abratique was aware that they were transporting marijuana as some of
the articles in the sacks became exposed in the process of loading. 15

Eventually, Abratique and Nazarea were bothered by the nature of the goods stored
in the rented room. She confided to her daughter, Alice Abreau Fianza, about their
predicament. As Alice Fianza’s brother-in-law, Edwin Fianza, was an NBI agent,
Alice and Abratique phoned him and disclosed what had transpired. 16

On the morning of July 1, 1998, alerted by information that appellant would


retrieve the sacks of prohibited drugs that day, Edwin Fianza and other NBI
operatives conducted a stake out at No. 27, Dr. Cariño St. While the NBI agents
were conducting their surveillance, they noticed that several PNP NARCOM
personnel were also watching the place.17 The NBI then learned that the PNP
NARCOM had received a tip from one of their informers regarding the presence of
a huge amount of drugs in that place. The NBI and PNP NARCOM agreed to have
a joint operation.

As the day wore on and appellant did not show up, the NBI agents became
apprehensive that the whole operation could be jeopardized. They sought the
permission of Nazarea Abreau to enter the room rented by appellant. She acceded
and allowed them entry. The NBI team then searched the rented premises and
found four (4) boxes and thirteen (13) sacks of marijuana, totaling 336.93
kilograms.18

Later that evening, NBI Special Agent Darwin Lising, with Abratique as his
witness, applied for a search warrant from RTC Judge Antonio Reyes at his
residence.19 Judge Reyes ordered the NBI agents to fetch the Branch Clerk of
Court, Atty. Delilah Muñoz, so the proceedings could be properly recorded. After
Atty. Muñoz arrived, Judge Reyes questioned Lising and Abratique. Thereafter,
the judge issued a warrant directing the NBI to search appellant’s residence at Km.
6, Dontogan, Green Valley, Baguio City for marijuana. 20

The NBI operatives, with some PNP NARCOM personnel in tow, proceeded to
appellant’s residence where they served the warrant upon appellant himself. 21 The
search was witnessed by appellant, members of his family, barangay officials, and
members of the media.22 Photographs were taken during the actual search. 23 The
law enforcers found 26 boxes and a sack of dried marijuana 24 in the water tank,
garage, and storeroom of appellant’s residence. 25 The total weight of the haul was
591.81 kilograms.26 Appellant was arrested for illegal possession of marijuana.

The seized items were then submitted to the NBI laboratory for testing. NBI
Forensic Chemist Maria Carina Madrigal conducted the tests. Detailed microscopic
and chromatographic examinations of the items taken from appellant’s rented room
at No. 27, Dr. Cariño St., as well as those from his residence at Green Valley,
showed these to be marijuana.27

In his defense, appellant contended that the physical evidence of the prosecution
was illegally obtained, being the products of an unlawful search, hence
inadmissible. Appellant insisted that the search warrant was too general and the
process by which said warrant was acquired did not satisfy the constitutional
requirements for the issuance of a valid search warrant. Moreover, Abratique’s
testimony, which was heavily relied upon by the judge who issued the warrant, was
hearsay.

In Criminal Case No. 15822-R, the trial court agreed with appellant that the taking
of the 336.93 kilograms of marijuana was the result of an illegal search and hence,
inadmissible in evidence against appellant. Appellant was accordingly acquitted of
the charge. However, the trial court found that the prosecution’s evidence was
more than ample to prove appellant’s guilt in Criminal Case No. 15800-R and as
earlier stated, duly convicted him of illegal possession of marijuana and sentenced
him to death.

Hence, this automatic review.

Before us, appellant submits that the trial court erred in:

1…UPHOLDING THE LEGALITY OF THE SEARCH WARRANT


DESPITE LACK OF COMPLIANCE OF (sic) SEVERAL
REQUIREMENTS BEFORE IT SHOULD HAVE BEEN ISSUED AND IT
BEING A GENERAL WARRANT;

2….GRAVELY ABUSED ITS DISCRETION IN REOPENING THE


CASE AND ALLOWING ABRITIQUE TO TESTIFY AGAINST
APPELLANT;

3…GIVING CREDENCE TO THE TESTIMONY OF ABRITIQUE;

4. NOT ACQUITTING THE ACCUSED IN BOTH CASES AND


SENTENCING HIM TO DEATH DESPITE THE ILLEGALLY
OBTAINED EVIDENCE AS FOUND IN THE FIRST CASE.28
We find that the pertinent issues for resolution concern the following: (1) the
validity of the search conducted at the appellant’s residence; (2) the alleged
prejudice caused by the reopening of the case and absences of the prosecution
witness, on appellant’s right to speedy trial; (3) the sufficiency of the prosecution’s
evidence to sustain a finding of guilt with moral certainty; and (4) the propriety of
the penalty imposed.

1. On the Validity of the Search Warrant; Its Obtention and Execution

Appellant initially contends that the warrant, which directed the peace officers to
search for and seize "an undetermined amount of marijuana," was too general and
hence, void for vagueness. He insists that Abratique could already estimate the
amount of marijuana supposed to be found at appellant’s residence since Abratique
helped to transport the same.

For the appellee, the Office of the Solicitor General (OSG) counters that a search
warrant is issued if a judge finds probable cause that the place to be searched
contains prohibited drugs, and not that he believes the place contains a specific
amount of it. The OSG points out that, as the trial court observed, it is impossible
beforehand to determine the exact amount of prohibited drugs that a person has on
himself.

Appellant avers that the phrase "an undetermined amount of marijuana" as used in
the search warrant fails to satisfy the requirement of Article III, Section 2 29 of the
Constitution that the things to be seized must be particularly described. Appellant’s
contention, in our view, has no leg to stand on. The constitutional requirement of
reasonable particularity of description of the things to be seized is primarily meant
to enable the law enforcers serving the warrant to: (1) readily identify the
properties to be seized and thus prevent them from seizing the wrong items; 30 and
(2) leave said peace officers with no discretion regarding the articles to be seized
and thus prevent unreasonable searches and seizures. 31 What the Constitution seeks
to avoid are search warrants of broad or general characterization or sweeping
descriptions, which will authorize police officers to undertake a fishing expedition
to seize and confiscate any and all kinds of evidence or articles relating to an
offense.32 However, it is not required that technical precision of description be
required,33 particularly, where by the nature of the goods to be seized, their
description must be rather general, since the requirement of a technical description
would mean that no warrant could issue.34

Thus, it has been held that term "narcotics paraphernalia" is not so wanting in
particularity as to create a general warrant. 35 Nor is the description "any and all
narcotics" and "all implements, paraphernalia, articles, papers and records
pertaining to" the use, possession, or sale of narcotics or dangerous drugs so broad
as to be unconstitutional.36 A search warrant commanding peace officers to seize "a
quantity of loose heroin" has been held sufficiently particular. 37
Tested against the foregoing precedents, the description "an undetermined amount
of marijuana" must be held to satisfy the requirement for particularity in a search
warrant. Noteworthy, what is to be seized in the instant case is property of a
specified character, i.e., marijuana, an illicit drug. By reason of its character and
the circumstances under which it would be found, said article is illegal. A further
description would be unnecessary and ordinarily impossible, except as to such
character, the place, and the circumstances. 38 Thus, this Court has held that the
description "illegally in possession of undetermined quantity/amount of dried
marijuana leaves and Methamphetamine Hydrochloride (Shabu) and sets of
paraphernalia" particularizes the things to be seized. 39

The search warrant in the present case, given its nearly similar wording,
"undetermined amount of marijuana or Indian hemp," in our view, has satisfied the
Constitution’s requirements on particularity of description. The description therein
is: (1) as specific as the circumstances will ordinarily allow; (2) expresses a
conclusion of fact – not of law – by which the peace officers may be guided in
making the search and seizure; and (3) limits the things to be seized to those which
bear direct relation to the offense for which the warrant is being issued. 40 Said
warrant imposes a meaningful restriction upon the objects to be seized by the
officers serving the warrant. Thus, it prevents exploratory searches, which might be
violative of the Bill of Rights.

Appellant next assails the warrant for merely stating that he should be searched, as
he could be guilty of violation of Republic Act No. 6425. Appellant claims that
this is a sweeping statement as said statute lists a number of offenses with respect
to illegal drugs. Hence, he contends, said warrant is a general warrant and is thus
unconstitutional.

For the appellee, the OSG points out that the warrant clearly states that appellant
has in his possession and control marijuana or Indian hemp, in violation of Section
8 of Republic Act No. 6425.

We have carefully scrutinized Search Warrant No. 415 (7-98), 41 and we find that it
is captioned "For Violation of R.A. 6425, as amended." 42 It is clearly stated in the
body of the warrant that "there is probable cause to believe that a case for violation
of R.A. 6425, as amended, otherwise known as the Dangerous Drugs Act of 1972,
as further amended by R.A. 7659 has been and is being committed by one
MODESTO TEE a.k.a. ESTOY TEE of Km. 6, Dontogan Bgy., Green Valley, Sto.
Tomas, Baguio City by having in his possession and control an
UNDETERMINED AMOUNT OF MARIJUANA or INDIAN HEMP in violation
of the aforementioned law."43 In an earlier case, we held that though the specific
section of the Dangerous Drugs Law is not pinpointed, "there is no question at all
of the specific offense alleged to have been committed as a basis for the finding of
probable cause."44 Appellant’s averment is, therefore, baseless. Search Warrant No.
415 (7-98) appears clearly issued for one offense, namely, illegal possession of
marijuana.

Appellant next faults the Judge who issued Search Warrant No. 415 (7-98) for his
failure to exhaustively examine the applicant and his witness. Appellant points out
that said magistrate should not have swallowed all of Abratique’s statements – –
hook, line, and sinker. He points out that since Abratique consented to assist in the
transport of the marijuana, the examining judge should have elicited from
Abratique his participation in the crime and his motive for squealing on appellant.
Appellant further points out that the evidence of the NBI operative who applied for
the warrant is merely hearsay and should not have been given credit at all by Judge
Reyes.

Again, the lack of factual basis for appellant’s contention is apparent. The OSG
points out that Abratique personally assisted appellant in loading and transporting
the marijuana to the latter’s house and to appellant’s rented room at No. 27 Dr.
Cariño St., Baguio City. Definitely, this indicates personal knowledge on
Abratique’s part. Law enforcers cannot themselves be eyewitnesses to every crime;
they are allowed to present witnesses before an examining judge. In this case,
witness Abratique personally saw and handled the marijuana. Hence, the NBI did
not rely on hearsay information in applying for a search warrant but on personal
knowledge of the witness, Abratique.

Before a valid search warrant is issued, both the Constitution 45 and the 2000
Revised Rules of Criminal Procedure46 require that the judge must personally
examine the complainant and his witnesses under oath or affirmation. The personal
examination must not be merely routinary or pro forma, but must be probing and
exhaustive.47 In the instant case, it is not disputed that Judge Antonio Reyes
personally examined NBI Special Investigator III Darwin A. Lising, the applicant
for the search warrant as well as his witness, Danilo G. Abratique. Notes of the
proceedings were taken by Atty. Delilah Muñoz, Clerk of Court, RTC of Baguio
City, Branch 61, whom Judge Reyes had ordered to be summoned. In the letter of
transmittal of the Clerk of Court of the RTC of Baguio City, Branch 61 to Branch 6
of said court, mention is made of "notes" at "pages 7-11." 48 We have thoroughly
perused the records of Search Warrant No. 415 (7-98) and nowhere find said
"notes." The depositions of Lising and Abratique were not attached to Search
Warrant No. 415 (7-98) as required by the Rules of Court. We must stress,
however, that the purpose of the Rules in requiring depositions to be taken is to
satisfy the examining magistrate as to the existence of probable cause. 49 The Bill of
Rights does not make it an imperative necessity that depositions be attached to the
records of an application for a search warrant. Hence, said omission is not
necessarily fatal, for as long as there is evidence on the record showing what
testimony was presented.50 In the testimony of witness Abratique, Judge Reyes
required Abratique to confirm the contents of his affidavit; 51 there were instances
when Judge Reyes questioned him extensively. 52 It is presumed that a judicial
function has been regularly performed,53 absent a showing to the contrary. A
magistrate’s determination of probable cause for the issuance of a search warrant is
paid great deference by a reviewing court, 54 as long as there was substantial basis
for that determination.55 Substantial basis means that the questions of the
examining judge brought out such facts and circumstances as would lead a
reasonably discreet and prudent man to believe that an offense has been
committed, and the objects in connection with the offense sought to be seized are
in the place sought to be searched.

On record, appellant never raised the want of adequate depositions to support


Warrant No. 415 (7-98) in his motion to quash before the trial court. Instead, his
motion contained vague generalities that Judge Reyes failed to ask searching
questions of the applicant and his witness. Belatedly, however, he now claims that
Judge Reyes perfunctorily examined said witness. 56 But it is settled that when a
motion to quash a warrant is filed, all grounds and objections then available,
existent or known, should be raised in the original or subsequent proceedings for
the quashal of the warrant, otherwise they are deemed waived. 57

In this case, NBI Special Investigator Lising’s knowledge of the illicit drugs stored
in appellant’s house was indeed hearsay. But he had a witness, Danilo Abratique,
who had personal knowledge about said drugs and their particular location.
Abratique’s statements to the NBI and to Judge Reyes contained credible and
reliable details. As the NBI’s witness, Abratique was a person on whose statements
Judge Reyes could rely. His detailed description of appellant’s activities with
respect to the seized drugs was substantial. In relying on witness Abratique, Judge
Reyes was not depending on casual rumor circulating in the underworld, but on
personal knowledge Abratique possessed.

In Alvarez vs. Court of First Instance of Tayabas, 64 Phil. 33, 44 (1937), we held
that:

The true test of sufficiency of a deposition or affidavit to warrant issuance of


a search warrant is whether it has been drawn in such a manner that perjury
could be charged thereon and affiant be held liable for damages caused. 58

Appellant argues that the address indicated in the search warrant did not clearly
indicate the place to be searched. The OSG points out that the address stated in the
warrant is as specific as can be. The NBI even submitted a detailed sketch of the
premises prepared by Abratique, thus ensuring that there would be no mistake.

A description of the place to be searched is sufficient if the officer serving the


warrant can, with reasonable effort, ascertain and identify the place intended 59 and
distinguish it from other places in the community. 60 A designation or description
that points out the place to be searched to the exclusion of all others, and on
inquiry unerringly leads the peace officers to it, satisfies the constitutional
requirement of definiteness.

Appellant finally harps on the use of unnecessary force during the execution of the
search warrant. Appellant fails, however, to point to any evidentiary matter in the
record to support his contention. Defense witness Cipriana Tee, appellant’s mother,
testified on the search conducted but she said nothing that indicated the use of
force on the part of the NBI operatives who conducted the search and
seizure.61 What the record discloses is that the warrant was served on
appellant,62 who was given time to read it,63 and the search was witnessed by the
barangay officials, police operatives, members of the media, and appellant’s kith
and kin.64 No breakage or other damage to the place searched is shown. No injuries
sustained by appellant, or any witness, appears on record. The execution of the
warrant, in our view, has been orderly and peaceably performed.

2. On The Alleged Violation of Appellant’s Substantive Rights

Appellant insists that the prosecution’s unjustified and willful delay in presenting
witness Abratique unduly delayed the resolution of his case. He points out that a
total of eight (8) scheduled hearings had to be reset due to the failure or willful
refusal of Abratique to testify against him. Appellant insists that said lapse on the
prosecution’s part violated Supreme Court Circular No. 38-98. 65 Appellant now
alleges that the prosecution deliberately resorted to delaying the case to cause him
untold miseries.

For the appellee, the OSG points out that the two-month delay in the trial is not
such a great length of time as to amount to a violation of appellant’s right to a
speedy trial. A trial is always subject to reasonable delays or postponements, but
absent any showing that these delays are capricious and oppressive, the State
should not be deprived of a reasonable opportunity to prosecute the criminal
action.

On record, the trial court found that prosecution witness Danilo G. Abratique failed
to appear in no less than eighteen (18) hearings, namely those set for February 1, 2,
3, 4, 8, 9, 10, and 24; March 9, 15, 22, and 23; April 6, 7, 8, 16, and 19, all in
1999.66 No less than four (4) warrants of arrest were issued against him to compel
him to testify.67 The NBI agent who supposedly had him in custody was found
guilty of contempt of court for failing to produce Abratique at said hearings and
sanctioned.68 The prosecution had to write the NBI Regional Director in Baguio
City and NBI Director in Manila regarding the failure of the Bureau’s agents to
bring Abratique to court.69 Nothing on record discloses the reason for Abratique’s
aforecited absences. On the scheduled hearing of June 7, 1999, he was again absent
thus causing the trial court to again order his arrest for the fifth time.70 He also
failed to show up at the hearing of June 8, 1999. 71
Appellant now stresses that the failure of Abratique to appear and testify on twenty
(20) hearing dates violated appellant’s constitutional 72 and statutory right to a
speedy trial.

A speedy trial means a trial conducted according to the law of criminal procedure
and the rules and regulations, free from vexatious, capricious, and oppressive
delays.73 In Conde v. Rivera and Unson, 45 Phil. 650, 652 (1924), the Court held
that "where a prosecuting officer, without good cause, secures postponements of
the trial of a defendant against his protest beyond a reasonable period of time, as in
this instance, for more than a year, the accused is entitled to relief by a proceeding
in mandamus to compel a dismissal of the information, or if he be restrained of his
liberty, by habeas corpus to obtain his freedom."

The concept of speedy trial is necessarily relative. A determination as to whether


the right has been violated involves the weighing of several factors such as the
length of the delay, the reason for the delay, the conduct of the prosecution and the
accused, and the efforts exerted by the defendant to assert his right, as well as the
prejudice and damage caused to the accused. 74

The Speedy Trial Act of 1998, provides that the trial period for criminal cases in
general shall be one hundred eighty (180) days. 75 However, in determining the
right of an accused to speedy trial, courts should do more than a mathematical
computation of the number of postponements of the scheduled hearings of the
case.76 The right to a speedy trial is deemed violated only when: (1) the
proceedings are attended by vexatious, capricious, and oppressive delays; 77 or (2)
when unjustified postponements are asked for and secured; 78 or (3) when without
cause or justifiable motive a long period of time is allowed to elapse without the
party having his case tried.79

In the present case, although the absences of prosecution witness Abratique totaled
twenty (20) hearing days, there is no showing whatsoever that prosecution
capriciously caused Abratique’s absences so as to vex or oppress appellant and
deny him his rights. On record, after Abratique repeatedly failed to show up for the
taking of his testimony, the prosecution went to the extent of praying that the trial
court order the arrest of Abratique to compel his attendance at trial. The
prosecution likewise tried to get the NBI to produce Abratique as the latter was in
the Bureau’s custody, but to no avail. Eventually, the trial court ordered the
prosecution to waive its right to present Abratique and rest its case on the evidence
already offered.80

Nor do we find a delay of twenty (20) hearing days to be an unreasonable length of


time. Delay of less than two months has been found, in fact, to be not an
unreasonably lengthy period of time.81
Moreover, nothing on record shows that appellant Modesto Tee objected to the
inability of the prosecution to produce its witness. Under the Rules, appellant could
have moved the trial court to require that witness Abratique post bail to ensure that
the latter would testify when required.82 Appellant could have moved to have
Abratique found in contempt and duly sanctioned. Appellant did neither. It is a bit
too late in the day for appellant to invoke now his right to speedy trial.

No persuasive reason supports appellant’s claim that his constitutional right to


speedy trial was violated. One must take into account that a trial is always subject
to postponements and other causes of delay. But in the absence of a showing that
delays were unreasonable and capricious, the State should not be deprived of a
reasonable opportunity of prosecuting an accused. 83

Appellant next contends that the trial court gravely abused its discretion, and
exhibited partiality, when it allowed the reopening of the case after the prosecution
had failed to present Abratique on several occasions and had been directed to rest
its case. Appellant stresses that the lower court’s order to reopen the case to receive
Abratique’s further testimony is an indication that the trial court favored the
prosecution and unduly prejudiced appellant.

On appellee’s behalf, the Solicitor General points out that the trial court’s order
was in the interest of substantial justice and hence, cannot be termed as an abuse of
discretion. The OSG points out that the prosecution had not formally rested its case
and had yet to present its formal offer of evidence, hence, the submission of
additional testimony by the same witness cannot be prejudicial to the accused, it
being but the mere continuation of an uncompleted testimony. Furthermore,
appellant did not properly oppose the prosecution’s motion to reopen the case.

At the time Criminal Cases Nos. 15800-R and 15822-R were being tried, the 1985
Rules of Criminal Procedure were in effect. There was no specific provision at that
time governing motions to reopen.84 Nonetheless, long and established usage has
led to the recognition and acceptance of a motion to reopen. In view of the absence
of a specific procedural rule, the only controlling guideline governing a motion to
reopen was the paramount interests of justice. As a rule, the matter of reopening of
a case for reception of further evidence after either prosecution or defense has
rested its case is within the discretion of the trial court. 85 However, a concession to
a reopening must not prejudice the accused or deny him the opportunity to
introduce counter evidence.86

Strictly speaking, however, there was no reopening of the cases in the proceedings
below. A motion to reopen may properly be presented only after either or both
parties have formally offered and closed their evidence, but before judgment. 87 In
the instant case, the records show that on April 19, 1999, the prosecution was
directed to close its evidence and given 15 days to make its formal offer of
evidence.88 This order apparently arose from the manifestation of the prosecution
on April 16, 1999 that should they fail to produce witness Abratique on the next
scheduled hearing the prosecution would rest its case. 89 On April 19, 1999, which
was the next scheduled hearing after April 16, 1999, Abratique was absent
notwithstanding notices, orders, and warrants of arrest. However, on April 27,
1999, or before the prosecution had formally offered its evidence, Abratique was
brought to the trial court by the NBI. In its order of said date, the trial court pointed
out that the prosecution could move to "reopen" the case for the taking of
Abratique’s testimony.90 On May 7, 1999, the prosecution so moved, stressing that
it had not yet formally offered its evidence and that the substantial rights of the
accused would not be prejudiced inasmuch as the latter had yet to present his
evidence. Appellant filed no opposition to the motion. The trial court granted the
motion six days later. Plainly, there was nothing to reopen, as the prosecution had
not formally rested its case. Moreover, the taking of Abratique’s testimony was not
for the purpose of presenting additional evidence, but more properly for
the completion of his unfinished testimony. In U.S. vs. Base,91 we held that a trial
court is not in error, if it opts to reopen the proceedings of a case, even after both
sides had rested and the case submitted for decision, by the calling of additional
witnesses or recalling of witnesses so as to satisfy the judge’s mind with reference
to particular facts involved in the case. A judge cannot be faulted should he require
a material witness to complete his testimony, which is what happened in this case.
It is but proper that the judge’s mind be satisfied on any and all questions presented
during the trial, in order to serve the cause of justice.

Appellant’s claim that the trial court’s concession to "reopen" the case unduly
prejudiced him is not well taken. We note that appellant had every opportunity to
present his evidence to support his case or to refute the prosecution’s evidence
point-by-point, after the prosecution had rested its case. In short, appellant was
never deprived of his day in court. A day in court is the touchstone of the right to
due process in criminal justice.92 Thus, we are unable to hold that a grave abuse of
discretion was committed by the trial court when it ordered the so-called
"reopening" in order to complete the testimony of a prosecution witness.

3. On the Sufficiency of the Prosecution’s Evidence

In bidding for acquittal, appellant assails the credibility of Abratique as a witness.


Appellant insists that Abratique’s testimony is profuse with lies, contrary to human
nature, hence incredible. According to appellant, Abratique was evasive from the
outset with respect to certain questions of the trial court. He adds that it appeared
the court entertained in particular the suspicion that witness Abratique had
conspired with appellant in committing the crime charged. Appellant questions
Abratique’s motive in informing the NBI about his activities related to the
marijuana taking, transfer, and warehousing.

The OSG contends that Abratique’s testimony, taken as a whole, is credible. It


points out that Abratique testified in a straightforward manner as to his knowledge
of the huge cache of prohibited drugs stashed by appellant in two different places.
His testimony, said the OSG, when fused with the physical evidence consisting of
591.81 kilograms of marijuana found by law enforcers at appellant’s residence,
inexorably leads to the inculpation of appellant.

It is the bounden duty of the courts to test the prosecution evidence rigorously, so
that no innocent person is made to suffer the unusually severe penalties meted out
for drug offenses.93 Though we scrutinized minutely the testimony of Abratique,
we find no cogent reason to disbelieve him. From his account, Abratique might
appear aware treading the thin line between innocence and feeling guilty, with
certain portions of his story tending to be self-exculpatory. However, his whole
testimony could not be discredited. The established rule is that testimony of a
witness may be believed in part and disbelieved in other parts, depending on the
corroborative evidence and the probabilities and improbabilities of the case. But it
is accepted, as a matter of common sense, that if certain parts of a witness’
testimony are found true, his testimony cannot be disregarded entirely. 94

Abratique testified in open court that appellant rented the taxicab he was driving,
and he helped appellant transport huge amounts of marijuana to appellant’s rented
room at No. 27 Dr. Cariño St., Baguio City and to appellant’s residence at Km. 6,
Dontogan, Green Valley, Sto. Tomas, Baguio City. He also declared on the witness
stand that out of fear of being involved, he decided to divulge his knowledge of
appellant’s possession of large caches of marijuana to the NBI. When the places
referred to by Abratique were searched by the authorities, marijuana in staggering
quantities was found and seized by the law enforcers. Stated plainly, the physical
evidence in this case corroborated Abratique’s testimony on material points.

Appellant imputes questionable motives to Abratique in an effort to discredit him.


He demands that Abratique should likewise be prosecuted. However, by no means
is the possible guilt of Abratique a tenable defense for appellant. Nor would
Abratique’s prosecution mean appellant’s absolution.

In a prosecution for illegal possession of dangerous drugs, the following facts must
be proven with moral certainty: (1) that the accused is in possession of the object
identified as prohibited or regulated drug; (2) that such possession is not authorized
by law; and (3) that the accused freely and consciously possessed the said drug. 95

We find the foregoing elements proven in Criminal Case No. 15800-R beyond
reasonable doubt.

In said case, the testimony of Abratique and the recovery of 591.81 kilograms of
marijuana from appellant’s residence served to prove appellant’s possession of a
prohibited drug. Tests conducted by the NBI forensic chemist proved the seized
articles to be marijuana. These articles were seized pursuant to a valid search
warrant and hence, fully admissible in evidence.
In People v. de los Reyes, 239 SCRA 439 (1994), we held that the Dangerous
Drugs Act applies generally to all persons and proscribes the sale of dangerous
drugs by any person, and no person is authorized to sell such drugs. Said doctrine
is equally applicable with respect to possession of prohibited drugs. Republic Act
No. 6425, which penalizes the possession of prohibited drugs, applies equally to all
persons in this jurisdiction and no person is authorized to possess said articles,
without authority of law.

Anent the third element, we have held that to warrant conviction, possession of
illegal drugs must be with knowledge of the accused or that animus
possidendi existed together with the possession or control of said
articles.96 Nonetheless, this dictum must be read in consonance with our ruling that
possession of a prohibited drug per se constitutes prima facie evidence of
knowledge or animus possidendi sufficient to convict an accused absent a
satisfactory explanation of such possession.97 In effect, the onus probandi is shifted
to accused to explain the absence of knowledge or animus possidendi98 in this
situation.

Appellant Modesto Tee opted not to testify in his defense. Instead, he presented his
mother as his lone witness, who testified on matters totally irrelevant to his case.
We can only conclude that, failing to discharge the burden of the evidence on the
possession of prohibited drug, appellant’s guilt in Criminal Case No. 15800-R was
established beyond reasonable doubt.

4. On The Proper Penalty

Under Republic Act No. 6425 as amended by Republic Act No. 7659, the penalty
of reclusion perpetua to death and a fine ranging from five hundred thousand pesos
(P500,000.00) to ten million pesos (P10,000,000.00)99 shall be imposed if the
quantity of marijuana involved in a conviction for possession of marijuana or
Indian hemp shall be 750 grams or more.100

In the present case, the quantity of marijuana involved has been shown by the
prosecution to be far in excess of 750 grams, as stressed by the trial court:

The volume is rather staggering. It is almost one whole house or one whole
room. In fact, when they were first brought to the court, it took hours to load
them on the truck and hours also to unload them prompting the court to
direct that the boxes and sack of marijuana be instead kept at the NBI office
in Baguio. And the identification of said marijuana during the trial was
made in the NBI premises itself by the witnesses since it was physically
cumbersome and inconvenient to keep bringing them to the court during
every trial.101
In sentencing appellant to death, the trial court noted not only the huge quantity of
marijuana bales involved, but also "the acts of accused of hiding them in different
places…and transferring them from place to place and making them appear as
boxes of cigarettes to avoid and evade apprehension and detection." They showed
his being a big supplier, said the trial court, [whose] criminal perversity and craft
that "deserve the supreme penalty of death." 102

We are unable to agree, however, with the penalty imposed by the trial court. The
legislature never intended that where the quantity involved exceeds those stated in
Section 20 of Republic Act No. 6425 the maximum penalty of death shall
automatically be imposed.103 The statute prescribes two indivisible
penalties: reclusion perpetua and death. Hence, the penalty to be imposed must
conform with Article 63104 of the Revised Penal Code. As already held, the death
penalty law, Republic Act No. 7659 did not amend Article 63 of the Revised Penal
Code.105 The rules in Article 63 apply although the prohibited drugs involved are in
excess of the quantities provided for in Section 20 of Republic Act No.
6425.106 Thus, finding neither mitigating nor aggravating circumstances in the
present case, appellant’s possession of 591.81 kilograms of marijuana in Criminal
Case No. 15800-R, does not merit capital punishment but only the lesser penalty
of reclusion perpetua.

The trial court imposed a fine on appellant in the sum of One Million Pesos
(P1,000,000.00), without subsidiary imprisonment in case of insolvency. The
imposition of a fine is mandatory in cases of conviction of possession of illegal
drugs. This being within the limits allowed by the law, the amount of the fine must
be sustained. All these sanctions might not remedy all the havoc wrought by
prohibited drugs on the moral fiber of our society, especially the youth. 107 But these
penalties should warn peddlers of prohibited drugs that they cannot ply their trade
in our streets with impunity.

WHEREFORE, the decision of the Regional Trial Court of Baguio City, Branch 6,
in Criminal Case No. 15800-R, convicting appellant MODESTO TEE alias
"ESTOY" TEE of violation of Section 8 of Republic Act No. 6425, as amended, is
AFFIRMED with the MODIFICATION that appellant is hereby sentenced to
suffer the penalty of reclusion perpetua. The fine of ONE MILLION
(P1,000,000.00) PESOS imposed on him is sustained. Appellant is likewise
directed to pay the costs of suit.

SO ORDERED.

CASES:
People v. Tuan, G.R. No. 176066, August 11, 2010

For review is the Decision1 dated September 21, 2006 of the Court of Appeals in
CA-G.R. CR.-H.C. No. 00381, which affirmed with modification the
Decision2 dated April 9, 2002 of the Regional Trial Court (RTC), Branch 6, Baguio
City, finding accused-appellant Estela Tuan y Baludda guilty in Criminal Case No.
17619-R, of illegal possession of marijuana under Article II, Section 8 of Republic
Act No. 6425, otherwise known as "The Dangerous Drugs Act of 1972," as
amended; and in Criminal Case No. 17620-R, of violating Presidential Decree No.
1866, otherwise known as the "Illegal Possession of Firearms," as amended.

On April 5, 2000, two separate Informations were filed before the RTC against
accused-appellant for illegal possession of marijuana and illegal possession of
firearm. The Informations read:

Criminal Case No. 17619-R

The undersigned Public Prosecutor accuses ESTELA TUAN Y BALUDDA of the


crime of VIOLATION OF SEC. 8, ART. II OF REPUBLIC ACT 6425, AS
AMENDED (Illegal Possession of Marijuana), committed as follows:

That on or about 24th day of January 2000, at Barangay Gabriela Silang, in the
City of Baguio, Philippines, and within the jurisdiction of this Honorable Court,
the above-named accused did then and there willfully and unlawfully have in her
possession, custody, and control the following, to wit:

a) Nine (9) bricks of dried Marijuana leaves with an approximate total


weight of 18.750 kgs., and

b) One (1) plastic bag containing dried Marijuana leaves weighing


approximately .3 kg.

without any authority of law to do so in violation of the above-cited provision of


law.3

Criminal Case No. 17620-R

The undersigned Public Prosecutor accuses ESTELA TUAN Y BALUDDA of the


crime of VIOLATION OF PRESIDENTIAL DECREE 1866, AS AMENDED
(Illegal Possession of Firearm), committed as follows:

That on or about the 24th day of January 2000, at Barangay Gabriela Silang, in the
City of Baguio, Philippines, and within the jurisdiction of this Honorable Court,
the above-named accused did then and there willfully and unlawfully have in her
possession, custody, and control one (1) Cal. .357 S & W revolver, a high-powered
firearm, without any license, permit or authority duly issued by the government to
possess or keep the same in violation of the above-cited law. 4
Upon her arraignment on April 18, 2000, accused-appellant, assisted by her
counsel de parte, pleaded "NOT GUILTY" to both charges. 5 Pre-trial and trial
proper then ensued.

During trial, the prosecution presented four witnesses: Senior Police Officer (SPO)
1 Modesto F. Carrera (Carrera), Police Officer (PO) 2 Jaime Chavez (Chavez),
SPO2 Fernando Fernandez (Fernandez), and Forensic Chemist II Marina Carina
Madrigal (Madrigal).

The events, as recounted by the prosecution, are as follows:

At around nine o’clock in the morning on January 24, 2000, two male informants
namely, Jerry Tudlong (Tudlong) and Frank Lad-ing (Lad-ing) arrived at the office
of the 14th Regional CIDG (Criminal Investigation and Detention Group) at DPS
Compound, Marcoville, Baguio City, and reported to SPO2 Fernandez, Chief of
the Station Drug Enforcement Unit (SDEU), that a certain "Estela Tuan" had been
selling marijuana at Barangay Gabriela Silang, Baguio City. Present at that time
were Police Superintendent Isagani Neres, Regional Officer of the 14th Regional
CIDG; Chief Inspector Reynaldo Piay, Deputy Regional Officer; and other police
officers.6

SPO2 Fernandez set out to verify the report of Tudlong and Lad-ing. At around
one o’clock in the afternoon of the same day, he gave Tudlong and Lad-ing
₱300.00 to buy marijuana, and then accompanied the two informants to the
accused-appellant’s house. Tudlong and Lad-ing entered accused-appellant’s
house, while SPO2 Fernandez waited at the adjacent house. After thirty minutes,
Tudlong and Lad-ing came out of accused-appellant’s house and showed SPO2
Fernandez the marijuana leaves they bought. After returning to the CIDG regional
office, SPO2 Fernandez requested the laboratory examination of the leaves bought
from accused-appellant. When said laboratory examination yielded positive results
for marijuana, SPO2 Fernandez prepared an Application for Search Warrant for
accused-appellant’s house.

SPO2 Fernandez, together with Tudlong and Lad-ing, filed the Application for a
Search Warrant before Judge Iluminada Cabato-Cortes (Judge Cortes) of the
Municipal Trial Court in Cities (MTCC), Baguio City, Branch IV, at about one
o’clock in the afternoon on January 25, 2000. Two hours later, at around three
o’clock, Judge Cortes personally examined SPO2 Fernandez, Tudlong, and Lad-
ing, after which, she issued a Search Warrant, being satisfied of the existence of
probable cause. The Search Warrant read:

TO ANY PEACE OFFICER:

GREETINGS:
It appearing to the satisfaction of the undersigned of the existence of facts upon
which the application for Search Warrant is based, after personally examining by
searching questions under oath SPO2 Fernando V. Fernandez of the CAR Criminal
Investigation and Detection Group with office address at DPS Compound, Utility
Road, Baguio City and his witnesses namely: Frank Lad-ing of Happy Hallow,
Baguio City and Jerry Tudlong, of Barangay Kitma, Baguio City, after having been
duly sworn to, who executed sworn statements and deposition as witneses, that
there is a probable cause to believe that a Violation of R.A. 6425 as amended by
R.A. 7659 has been committed and that there are good and sufficient reasons to
believe that Estela Tuan, has in her possession and control at her resident at Brgy.
Gabriela Silang, Baguio City, the following:

- Undetermined Quantity of Marijuana Dried Leaves and/or Marijuana Hashish

xxxx

which are subject of the offense which should be seized and brought to the
undersigned.

You are hereby commanded to make an immediate search at anytime in the day the
house of the accused Estela Tuan at Brgy. Gabriela Silang, Baguio City, and
forthwith seize and take possession of the following:

- Undetermined Quantity of Marijuana Dried Leaves and/or Marijuana Hashish

x x x nothing follows x x x

and bring said items to the undersigned to be dealt with as the law directs.

This Search Warrant shall be valid for ten (10) days from date of issue, thereafter,
it shall be void.

The officers must conduct the search and seize the above-mentioned personal items
in the presence of the lawful occupant thereof or any member of her family or in
the absence of the latter, in the presence of two witnesses of sufficient age and
discretion residing in the same locality.

The officers seizing the items must give a detailed receipt for the same to the
lawful occupant of the house in whose presence the search and seizure were made,
or in the absence of such occupant, must, in the presence of the 2 witnesses
mentioned, leave a receipt in the place in which the seized items were found;
thereafter, deliver the items seized to the undersigned judge together with a true
inventory thereof duly verified under oath.

Baguio City, Philippines, this 25th day of January, 2000.


(SGD)ILUMINADA CABATO-CORTES
Executive Judge

MTCC, Branch IV7

Upon receipt of the Search Warrant, SPO2 Fernandez, his team supervisor Police
Senior Inspector Rodolfo Castel, SPO1 Carrera, Police Senior Inspector Ricarte
Marquez and PO2 Chavez implemented the warrant. Before going to the accused-
appellant’s house, SPO2 Fernandez invited barangay officials to be present when
the Search Warrant was to be served, but since no one was available, he requested
one Eliza Pascual (Pascual), accused-appellant’s neighbor, to come along.

The CIDG team thereafter proceeded to accused-appellant’s house. Even though


accused-appellant was not around, the CIDG team was allowed entry into the
house by Magno Baludda (Magno), accused-appellant’s father, after he was shown
a copy of the Search Warrant. SPO2 Fernandez and Police Senior Inspector Ricarte
Marquez guarded the surroundings of the house, 8 while SPO1 Carrera and PO2
Chavez searched inside.

SPO1 Carrera and PO2 Chavez began searching the rooms on the first floor in the
presence of Magno and Pascual. They continued their search on the second floor.
They saw a movable cabinet in accused-appellant’s room, below which they found
a brick of marijuana and a firearm. At around six o’clock that evening, accused-
appellant arrived with her son. The police officers asked accused-appellant to open
a built-in cabinet, in which they saw eight more bricks of marijuana. 9 PO2 Chavez
issued a receipt for the items confiscated from accused-appellant 10 and a
certification stating that the items were confiscated and recovered from the house
and in accused-appellant’s presence.

The nine bricks of marijuana were brought to the National Bureau of Investigation
(NBI) for examination.

The defense, on the other hand, had an entirely different version of what transpired
that day. It presented four witnesses, namely, accused-appellant herself; Beniasan
Tuan (Beniasan), accused-appellant’s husband; Magno, accused-appellant’s father;
and Mabini Maskay (Maskay), the Barangay Captain of Barangay Gabriela
Silang.1avvphi1

In her testimony, accused-appellant declared that she worked as a vendor at Hangar


Market. Sometime in January 2000, while she was selling vegetables at Hangar
Market, her son arrived with two police officers who asked her to go home because
of a letter from the court.11 At about six o’clock in the afternoon, she and her
husband Beniasan reached their residence and found a green paper bag with
marijuana in their sala. According to the police officers, they got the bag from a
room on the first floor of accused-appellant’s house. Accused-appellant explained
that the room where the bag of marijuana was found was previously rented by
boarders. The boarders padlocked the room because they still had things inside and
they had paid their rent up to the end of January 2000. 12 The police officers also
informed accused-appellant that they got a gun from under a cabinet in the latter’s
room, which accused-appellant disputed since her room was always left open and it
was where her children play.13 Accused-appellant alleged that a Search Warrant
was issued for her house because of a quarrel with her neighbor named Lourdes
Estillore (Estillore). Accused-appellant filed a complaint for the demolition of
Estillore’s house which was constructed on the road. 14

Beniasan supported the testimony of his wife, accused-appellant. He narrated that


he and accused-appellant were at their Hangar Market stall when two police
officers came and asked them to go home. Beniasan and accused-appellant arrived
at their residence at around six o’clock in the evening and were shown the
marijuana the police officers supposedly got from the first floor of the house. The
police officers then made Beniasan sign a certification of the list of items
purportedly confiscated from the house.15

Magno testified that he resided at the first floor of accused-appellant’s residence.


He was present when the search was conducted but denied that the Search Warrant
was shown to him.16 He attested that the confiscated items were found from the
vacant room at the first floor of accused-appellant’s house which was previously
occupied by boarders. Said room was padlocked but was forced open by the police
officers. In the course of the police officers’ search, they pulled something from
under the bed that was wrapped in green cellophane, but Magno did not know the
contents thereof.17 The police officers also searched the rooms of accused-appellant
and her children at the second floor of the house, during which they allegedly
found a gun under the cabinet in accused-appellant’s room. Magno claimed that he
did not personally witness the finding of the gun and was merely informed about it
by the police officers.18

Maskay, the Barangay Captain of Barangay Gabriela Silang, Baguio City, was the
last to testify for the defense. He corroborated accused-appellant’s allegation that
the latter had a quarrel with Estillore, and this could be the reason behind the filing
of the present criminal cases. He further remembered that the members of the
CIDG went to his office on January 24, 2000 to ask about the location of accused-
appellant’s house.19

The RTC, in its Decision dated April 9, 2002, found accused-appellant guilty as
charged and adjudged thus:

WHEREFORE, judgment is hereby rendered as follows:

1. In Criminal Case No. 17619-R, the Court finds the accused Estela Tuan guilty
beyond reasonable doubt of the offense of illegal possession of marijuana (nine [9]
bricks of dried marijuana leaves with an approximate weight of 18.750 kilograms
and the one [1] plastic bag containing the dried marijuana weighing about .3
kilograms) in violation of Section 8, Article II of Republic Act No. 6425 as
amended by Section 13 of Republic Act 7659 as charged in the information and
sentences her to the penalty of reclusion perpetua and to pay a fine of ₱500,000.00
without subsidiary imprisonment in case of insolvency.

The nine (9) bricks of dried marijuana leaves with an approximate weight of
18.750 kilograms and one (1) plastic bag containing dried marijuana leaves
weighing approximately .3 kilograms (Exhibit F, F-1, F-1-A to F-1-J) are ordered
confiscated and forfeited in favor of the State to be destroyed immediately in
accordance with law.

The accused Estela Tuan being a detention prisoner is entitled to be credited 4/5 of
her preventive imprisonment in the service of her sentence in accordance with
Article 29 of the Revised Penal Code; and

2. In Criminal Case No. 17620-R, the Court finds the accused Estela Tuan guilty
beyond reasonable doubt of the offense of illegal possession of firearms (one [1]
caliber .357 S & W revolver), a high powered firearm, without any license, permit
or authority issued by the Government to keep the same in violation of Section 1,
Republic Act No. 8294 which amended Section 1 of PD 1866 as charged in the
information and hereby sentences her, applying the Indeterminate Sentence Law, to
imprisonment ranging from 4 years 9 months and 10 days of prision correccional
in its maximum period as Minimum to 6 years and 8 months of prision mayor in its
minimum period as Maximum and a fine of ₱30,000.00 without subsidiary
imprisonment in case of insolvency.

The firearm caliber .357 S & W revolver without serial number is ordered forfeited
in favor of the State to be disposed of immediately in accordance with law.

The accused Estela Tuan being a detention prisoner is entitled to be credited 4/5 of
her preventive imprisonment in the service of her sentence in accordance with
Article 29 of the Revised Penal Code.20

The records of the two criminal cases were forwarded to this Court by the RTC,
but the Court issued a Resolution21 dated October 13, 2004 transferring said
records to the Court of Appeals pursuant to People v. Mateo. 22

On September 21, 2006, the Court of Appeals promulgated its Decision.

The Court of Appeals held that the contested search and consequent seizure of the
marijuana bricks were done pursuant to the Search Warrant validly issued by the
MTCC. There was no showing of procedural defects or lapses in the issuance of
said Search Warrant as the records support that the issuing judge determined
probable cause only after conducting the searching inquiry and personal
examination of the applicant and the latter’s witnesses, in compliance with the
requirements of the Constitution. Hence, the appellate court affirmed the
conviction of accused-appellant for illegal possession of marijuana.

The Court of Appeals, however, modified the appealed RTC judgment by


acquitting accused-appellant of the charge for illegal possession of firearm.
According to the appellate court, the records were bereft of evidence that the gun
supposedly confiscated from accused-appellant was unlicensed. The absence of a
firearm license was simply presumed by the police officers because the gun was a
defective paltik with no serial number. That the said condition of the gun did not
dispense with the need for the prosecution to establish that it was unlicensed
through the testimony or certification of the appropriate officer from the Board of
the Firearms and Explosives Bureau of the Philippine National Police.

In the end, the Court of Appeals decreed:

WHEREFORE, premises considered, the instant appeal is PARTLY GRANTED.


The assailed Decision of the RTC of Baguio City, Branch 6, dated April 9, 2002, is
hereby MODIFIED such that the conviction of accused-appellant for Violation of
Section 8, Art. II, RA 6425, as amended, is AFFIRMED while her conviction for
Violation of PD 1866, as amended, is REVERSED and SET ASIDE. Accused-
appellant is accordingly ACQUITTED of the latter offense. 23

In its Resolution dated October 20, 2006, the Court of Appeals gave due course to
accused-appellant’s Partial Notice of Appeal and accordingly forwarded the
records of the case to this Court.

This Court then issued a Resolution24 dated February 28, 2007 directing the parties
to file their respective supplemental briefs, if they so desired, within 30 days from
notice. Accused-appellant25 opted not to file a supplemental brief and manifested
that she was adopting her arguments in the Appellant’s Brief since the same had
already assiduously discussed her innocence of the crime charged. The
People26 likewise manifested that it would no longer file a supplemental brief as the
issues have all been addressed in its Appellee’s Brief.

Accused-appellant raised the following assignment of errors in her Brief: 27

THE TRIAL COURT GRAVELY ERRED IN GIVING FULL FAITH AND


CREDENCE TO THE INCREDIBLE AND CONTRADICTORY TESTIMONIES
OF THE POLICE OFFICERS.

THE TRIAL COURT GRAVELY ERRED IN CONVICTING THE ACCUSED-


APPELLANT OF THE CRIMES CHARGED DESPITE FAILURE OF THE
PROSECUTION TO PROVE HER GUILT BEYOND REASONABLE DOUBT.
THE TRIAL COURT ERRED IN NOT CONSIDERING AS VOID THE
SEARCH WARRANT ISSUED AGAINST THE ACCUSED-APPELLANT.

Given that accused-appellant was already acquitted of the charge of violation of


Presidential Decree No. 1866 on the ground of reasonable doubt in Criminal Case
No. 17620-R, her instant appeal relates only to her conviction for illegal possession
of prohibited or regulated drugs in Criminal Case No. 17619-R. The Court can no
longer pass upon the propriety of accused-appellant’s acquittal in Criminal Case
No. 17620-R because of the rule that a judgment acquitting the accused is final and
immediately executory upon its promulgation, and that accordingly, the State may
not seek its review without placing the accused in double jeopardy. Such acquittal
is final and unappealable on the ground of double jeopardy whether it happens at
the trial court or on appeal at the Court of Appeals. 28

In a prosecution for violation of the Dangerous Drugs Law, such as Criminal Case
No. 17619-R, a case becomes a contest of credibility of witnesses and their
testimonies. In such a situation, this Court generally relies upon the assessment by
the trial court, which had the distinct advantage of observing the conduct or
demeanor of the witnesses while they were testifying. Hence, its factual findings
are accorded respect – even finality – absent any showing that certain facts of
weight and substance bearing on the elements of the crime have been overlooked,
misapprehended or misapplied.29

The Court finds no reason to deviate from the general rule in the case at bar.

Illegal possession of prohibited or regulated drugs is committed when the


following elements concur: (1) the accused is in possession of an item or object
which is identified to be a prohibited drug; (2) such possession is not authorized by
law; and (3) the accused freely and consciously possesses the said drug. 30

All the foregoing elements were duly proven to exist in Criminal Case No. 17619-
R. The search conducted by SPO1 Carrera and PO2 Chavez in accused-appellant’s
house yielded nine bricks of marijuana. Marijuana is a prohibited drug, thus,
accused-appellant’s possession thereof could not have been authorized by law in
any way. Accused-appellant evidently possessed the marijuana freely and
consciously, even offering the same for sale. The bricks of marijuana were found
in accused-appellant’s residence over which she had complete control. In fact,
some of the marijuana were found in accused-appellant’s own room.

Accused-appellant challenges the judgment of the RTC, affirmed by the Court of


Appeals, finding her guilty of illegal possession of marijuana, by pointing out
certain inconsistencies in the testimonies of prosecution witnesses that supposedly
manifested their lack of credibility, i.e., the date of the test buy and the manner by
which the doors of the rooms of the house were opened.
These alleged inconsistencies and contradictions pertain to minor details and are so
inconsequential that they do not in any way affect the credibility of the witnesses
nor detract from the established fact of illegal possession of marijuana by accused-
appellant at her house. The Court has previously held that discrepancies and
inconsistencies in the testimonies of witnesses referring to minor details, and not in
actuality touching upon the central fact of the crime, do not impair their credibility.
Testimonies of witnesses need only corroborate each other on important and
relevant details concerning the principal occurrence. 31

Inconsistencies as to minor details and collateral matters do not affect the


credibility of the witnesses nor the veracity or weight of their testimonies. Such
minor inconsistencies may even serve to strengthen their credibility as they negate
any suspicion that the testimonies have been rehearsed. 32

Accused-appellant further questions the non-presentation as witnesses of Lad-ing


and Tudlong, the informants, and Pascual, the neighbor who supposedly witnessed
the implementation of the Search Warrant, during the joint trial of Criminal Case
Nos. 17619-R and 17620-R before the RTC. This Court though is unconvinced that
such non-presentation of witnesses is fatal to Criminal Case No. 17619-R.

The prosecution has the exclusive prerogative to determine whom to present as


witnesses. The prosecution need not present each and every witness but only such
as may be needed to meet the quantum of proof necessary to establish the guilt of
the accused beyond reasonable doubt. The testimonies of the other witnesses may,
therefore, be dispensed with if they are merely corroborative in nature. The Court
has ruled that the non-presentation of corroborative witnesses does not constitute
suppression of evidence and is not fatal to the prosecution’s case. 33

Although Criminal Case No. 17619-R involves illegal possession of marijuana, the
following pronouncement of this Court in People v. Salazar, 34 relating to the illegal
sale of the same drug, still rings true:

Neither is her right to confront witnesses against her affected by the prosecution's
failure to present the informer who pointed to her as a drug pusher. The
presentation of an informant in an illegal drugs case is not essential for conviction
nor is it indispensable for a successful prosecution because his testimony would be
merely corroborative and cumulative. In a case involving the sale of illegal drugs,
what should be proven beyond reasonable doubt is the fact of the sale itself. Hence,
like the non-presentation of the marked money used in buying the contraband, the
non-presentation of the informer on the witness stand would not necessarily create
a hiatus in the prosecutions' evidence. (Emphasis ours.)

Lastly, accused-appellant insists that the items allegedly seized from her house are
inadmissible as evidence because the Search Warrant issued for her house was
invalid for failing to comply with the constitutional and statutory requirements.
Accused-appellant specifically pointed out the following defects which made said
Search Warrant void: (1) the informants, Lad-ing and Tudlong, made
misrepresentation of facts in the Application for Search Warrant filed with the
MTCC; (2) Judge Cortes of the MTCC failed to consider the informants’
admission that they themselves were selling marijuana; and (3) the Search Warrant
failed to particularly describe the place to be searched because the house was a
two-storey building composed of several rooms.

The right of a person against unreasonable searches and seizure is recognized and
protected by no less than the Constitution, particularly, Sections 2 and 3(2) of
Article III which provide:

SEC. 2. The right of the people to be secure in their persons, houses, papers, and
effects against unreasonable searches and seizures of whatever nature and for any
purpose shall be inviolable, and no search warrant or warrant of arrest shall issue
except upon probable cause to be determined personally by the judge after
examination under oath or affirmation of the complainant and the witnesses he
may produce, and particularly describing the place to be searched and the persons
or things to be seized.

SEC. 3. x x x

(2) Any evidence obtained in violation of this or the preceding section shall be
inadmissible for any purpose in any proceeding. (Emphases ours.)

Accordingly, Sections 4 and 5, Rule 126 of the Revised Rules on Criminal


Procedure laid down the following requisites for the issuance of a valid search
warrant:

SEC. 4. Requisites for issuing search warrant. – A search warrant shall not issue
except upon probable cause in connection with one specific offense to be
determined personally by the judge after examination under oath or affirmation of
the complainant and the witnesses he may produce, and particularly describing the
place to be searched and the things to be seized which may be anywhere in the
Philippines.

SEC. 5. Examination of complainant; record. – The judge must, before issuing the
warrant, personally examine in the form of searching questions and answers, in
writing and under oath, the complainant and the witnesses he may produce on facts
personally known to them and attach to the record their sworn statements, together
with the affidavits submitted.

Therefore, the validity of the issuance of a search warrant rests upon the following
factors: (1) it must be issued upon probable cause; (2) the probable cause must be
determined by the judge himself and not by the applicant or any other person; (3)
in the determination of probable cause, the judge must examine, under oath or
affirmation, the complainant and such witnesses as the latter may produce; and (4)
the warrant issued must particularly describe the place to be searched and persons
or things to be seized.35

There is no dispute herein that the second and third factors for a validly issued
search warrant were complied with, i.e., personal determination of probable cause
by Judge Cortes; and examination, under oath or affirmation, of SPO2 Fernandez
and the two informants, Lad-ing and Tudlong, by Judge Cortes. What is left for the
Court to determine is compliance with the first and fourth factors, i.e., existence of
probable cause; and particular description of the place to be searched and things to
be seized.

In People v. Aruta,36 the Court defined probable cause as follows:

Although probable cause eludes exact and concrete definition, it generally signifies
a reasonable ground of suspicion supported by circumstances sufficiently strong in
themselves to warrant a cautious man to believe that the person accused is guilty of
the offense with which he is charged. It likewise refers to the existence of such
facts and circumstances which could lead a reasonably discreet and prudent man to
believe that an offense has been committed and that the item(s), article(s) or
object(s) sought in connection with said offense or subject to seizure and
destruction by law is in the place to be searched.

It ought to be emphasized that in determining probable cause, the average man


weighs facts and circumstances without resorting to the calibrations of our rules of
evidence of which his knowledge is technically nil. Rather, he relies on the
calculus of common sense which all reasonable men have in abundance. The same
quantum of evidence is required in determining probable cause relative to search.
Before a search warrant can be issued, it must be shown by substantial evidence
that the items sought are in fact seizable by virtue of being connected with criminal
activity, and that the items will be found in the place to be searched.

A magistrate’s determination of probable cause for the issuance of a search warrant


is paid great deference by a reviewing court, as long as there was substantial basis
for that determination. Substantial basis means that the questions of the examining
judge brought out such facts and circumstances as would lead a reasonably discreet
and prudent man to believe that an offense has been committed, and the objects in
connection with the offense sought to be seized are in the place sought to be
searched.37 Such substantial basis exists in this case.

Judge Cortes found probable cause for the issuance of the Search Warrant for
accused-appellant’s residence after said judge’s personal examination of SPO2
Fernandez, the applicant; and Lad-ing and Tudlong, the informants.
SPO2 Fernandez based his Application for Search Warrant not only on the
information relayed to him by Lad-ing and Tudlong. He also arranged for a test
buy and conducted surveillance of accused-appellant. He testified before Judge
Cortes:

COURT:

Q. You are applying for a Search Warrant and you alleged in your
application that Estela Tuan of Brgy. Gabriela Silang, Baguio City, is in
possession of dried marijuana leaves and marijuana hashish, how did you
come to know about this matter?

A. Through the two male persons by the name of Frank Lad-ing and Jerry
Tudlong, Your Honor.

Q. When did these two male persons report to your office?

A. January 22, Your Honor.

Q. This year?

A. Yes, your honor.

Q. To whom did they report?

A. To me personally, Your Honor.

Q. How did they report the matter?

A. They reported that a certain Estela Tuan is selling dried Marijuana leaves
and marijuana hashish, Your Honor.

Q. What else?

A. She is not only selling marijuana but also selling vegetables at the
Trading Post in La Trinidad, Your Honor.

Q. They just told you, she is selling marijuana and selling vegetables, that is
already sufficient proof or sufficient probable cause she is in possession of
marijuana, what else did they report?

A. That they are also selling marijuana in large volume at their house.

Q. What did you do when you asked them regarding that matter?
A. They had a test buy and they were able to buy some commodities
yesterday, Your honor.

Q. Who bought?

A. Tudlong and Lad-ing, Your Honor.

Q. How did you go about it?

A. I accompanied the said persons and kept watch over them and gave them
money after which, they were able to purchase and when they purchased the
said items or drugs, they were even informed that if you wanted to sell then
you could come and get. Your Honor.

COURT:

Q. Where is that ₱300.00?

A. It is with them, Your Honor.

Q. You did not entrap her?

A. No, Your Honor, because it is only a test buy.

Q: And that was January 22. Why did you not apply immediately for search
warrant?

A: Because we still have to look at the area and see to it that there are really
some buyers or people who would go and leave the place, Your Honor.

Q: What did you observe?

A: Well, there are persons who would go inside and after going inside, they
would come out bringing along with them something else.

Q: Did you not interview these people?

A: No, Your Honor. We did not bother.38

Lad-ing and Tudlong affirmed before Judge Cortes that they were the ones
who informed SPO2 Fernandez that accused-appellant was keeping and
selling marijuana at her house, and that they took part in the test buy.

Lad-ing narrated:

COURT:
Q: Mr. Lad-ing, you said that you are working at the Trading Post. What
kind of work do you have there?

A: I am a middleman of the vegetable dealers, Your Honor.

COURT:

Q: Did you come to know of this person Estela Tuan?

A: Yes, Your Honor, because there was an incident wherein we were


conducting our line of business when they came and joined us and we
became partners, Your Honor.

Q: You said, they, how many of you?

A: A certain Jerry Tudlong, Estela Tuan and myself, Your Honor.

Q: In other words, Estela Tuan went with you and later on she became your
partner in that business?

A: Yes, Your Honor.

Q: And so what happened when she became a partner of your business?

A: When we were about to divide our profit, we then went at their residence
at Gabriela Silang, Baguio City, Your Honor.

Q: What happened?

A: While we then sitted ourselves at the sala, she told us that if we wanted to
earn some more, she told us that she has in her possession marijuana which
could be sold, Your Honor.

Q: And so, what happened?

A: After which, she showed the marijuana, Your Honor.

Q: Where was the marijuana?

A: It was placed in a cellophane, in a newspaper, Your Honor.

Q: How big?

A: A dimension of 10 x 4 inches, Your Honor.

Q: With that size, where did she show you the box of this cellophane?
A: At the place where we were sitted at the receiving room, Your Honor.

Q: In other words, she went to get it and then presented or showed it to you?

A: Yes, Your Honor.

Q: Where did she go, if you know?

A: Because at the sala, there is a certain room located at the side that is the
place where she got the same, Your Honor.

Q: Where is this house of Estela Tuan located, is it along the road or inside
the road or what?

A: It is near the road but you have to walk in a little distance, Your Honor.

Q: Will you describe the place where Estela Tuan is residing?

A: Well, it is a two storey house, the walls are made of galvanized iron
Sheets, Your Honor.

COURT:

Q: Do you know who are staying there?

A: I do not know who is living with her, however, that is her residence,
Your Honor.

Q: How many times did you go there?

A: It was my second time to go at that time we were sent by PO Fernandez


to purchase marijuana, Your Honor.

Q: Where is the marijuana now?

A: It is in the possession of PO Fernandez, Your Honor.

Q: Where is the marijuana placed?

A: In a newspaper, Your Honor.

Q: What happened next?

A: We handed to her the amount of P300.00, your Honor.

Q: And she gave you that marijuana?


A: Yes, Your Honor.

xxxx

Q: How many rooms are there in the first floor of the house of Estela Tuan?

A: Three rooms, Your Honor, it has a dining room and beside the place is
the receiving room where we sitted ourselves, Your Honor.

Q: When you already bought marijuana from her, what did she tell you, if
any?

A; Well, if we would be interested to buy more, I still have stocks here,


Your Honor.39

Tudlong recounted in more detail what happened during the test buy:

COURT:

Q: My question is, when she told you that she has some substance for sale
for profit and you mentioned marijuana, did you talk immediately with
Frank or what did you do?

A: We reported the matter to the Criminal Investigation and Detection


Group, your Honor.

xxxx

Q: What time?

A: We went to the office at 9:00 – 9:30 o’clock in the morning, Your Honor.

Q: When you went there, what did you do?

A: The amount of ₱300.00 was given to Frank and we were instructed to


purchase, Your Honor.

Q: Did you go?

A: Yes, Your Honor.

xxxx

Q: Will you tell what happened when you went to the house of the woman?
A: Well, we were allowed to go inside the house after which, we were made
to sit down at the receiving area or sala, Your Honor.

Q: When you went there, you were allowed to enter immediately?

A: Yes, Your Honor.

Q: Who allowed you to enter?

A: The female person, Your Honor.

Q: What happened when you were asked to be sitted?

A: During that time, Frank and the female person were the ones conferring,
Your Honor.

Q: Did you hear what they were talking about?

A: That Frank was purchasing marijuana, Your Honor.

Q: What did the woman tell you?

A: After we handed the money, a plastic which was transparent, was then
handed to Frank, it was a plastic and there was a newspaper inside, Your
Honor.

xxxx

Q: So, you did not actually see what is in the newspaper?

A: No, Your Honor, however, I know that that is marijuana.

Q: Why?

A: Because that was our purpose, to buy marijuana, Your Honor.

Q: And you have not gotten marijuana without Estela Tuan informing you?

A: Yes, Your Honor.

Q: Will you tell us what kind of materials were used in the house of Estela
Tuan?

A: Two storey, the walls are made of GI sheets, Your Honor.\

Q: Is the house beside the road or do you have to walk?


A: It is near the road. Upon reaching the road, you still have to walk a short
distance, Your Honor.

Q: Where did Estela Tuan get the newspaper placed in a transparent plastic?

A: She got it from a room because were then made to wait at the sala, Your
Honor.

Q: Did she tell you how much she can sell marijuana?

A: She told us, Your Honor.

Q: What?

A: Well, the marijuana that we purchased was worth ₱300.00[.] However,


we could divide it into two small packs and we could sell it at ₱20.00 per
piece so that you can also have some gain.

COURT:

Q: After that, to whom did you sell?

A: We did not sell the marijuana, Your Honor.

Q: I thought you are going to sell marijuana and so you went there?

A: We were just instructed by PO Fernandez to verify what we are telling


him was true, Your Honor.40

Accused-appellant’s contention that MTCC Judge Cortes failed to consider the


informants’ admission that they themselves were selling marijuana is utterly
without merit. First, even after carefully reviewing the testimonies of Lad-ing and
Tudlong before Judge Cortes, this Court did not find a categorical admission by
either of the two informants that they themselves were selling marijuana. In fact,
Tudlong expressly denied that he and Lad-ing sold the marijuana, having only
bought the same from the accused-appellant for the test buy. Moreover, even if the
informants were also selling marijuana, it would not have affected the validity of
the Search Warrant for accused-appellant’s house. The criminal liabilities of
accused-appellant and the informants would be separate and distinct. The
investigation and prosecution of one could proceed independently of the other.

Equally without merit is accused-appellant’s assertion that the Search Warrant did
not describe with particularity the place to be searched.

A description of the place to be searched is sufficient if the officer serving the


warrant can, with reasonable effort, ascertain and identify the place intended and
distinguish it from other places in the community. A designation or description that
points out the place to be searched to the exclusion of all others, and on inquiry
unerringly leads the peace officers to it, satisfies the constitutional requirement of
definiteness.41 In the case at bar, the address and description of the place to be
searched in the Search Warrant was specific enough. There was only one house
located at the stated address, which was accused-appellant’s residence, consisting
of a structure with two floors and composed of several rooms.

In view of the foregoing, the Court upholds the validity of the Search Warrant for
accused-appellant’s house issued by MTCC Judge Cortes, and any items seized as
a result of the search conducted by virtue thereof, may be presented as evidence
against the accused-appellant.

Since it is beyond any cavil of doubt that the accused-appellant is, indeed, guilty of
violation of Article II, Section 8 of Republic Act No. 6425, as amended, the Court
shall now consider the appropriate penalty to be imposed upon her.

Article II, Section 8, in relation to Section 20(3), of Republic Act No. 6425, as
amended, provides:

SEC. 8. Possession or Use of Prohibited Drugs.- The penalty of reclusion


perpetua to death and a fine ranging from five hundred thousand pesos to ten
million pesos shall be imposed upon any person who, unless authorized by law,
shall possess or use any prohibited drug subject to the provisions of Section 20
hereof. (As amended by R.A. 7659)

Sec. 20. Application of Penalties, Confiscation and Forfeiture of the Proceeds or


Instruments of the Crime. - The penalties for offenses under Sections 3, 4, 7, 8 and
9 of Article II and Sections 14, 14-A, 15 and 16 of Article III of this Act shall be
applied if the dangerous drugs involved is in any of the following quantities:

1. 40 grams or more of opium;

2. 40 grams or more of morphine;

3. 200 grams or more of shabu or methylamphetamine hydrochloride;

4. 40 grams or more of heroin;

5. 750 grams or more of Indian hemp or marijuana;

6. 50 grams or more of marijuana resin or marijuana resin oil;

7. 40 grams or more of cocaine or cocaine hydrocholoride; or


8. In the case of other dangerous drugs, the quantity of which is far beyond
therapeutic requirements, as determined and promulgated by the Dangerous
Drugs Board, after public consultations/hearings conducted for the purpose.
(Emphasis supplied.)

Pursuant to Article II, Section 8 of Republic Act No. 6425, as amended, illegal
possession of 750 grams or more of the prohibited drug marijuana is punishable by
reclusion perpetua to death. Accused-appellant had in her possession a total of
19,050 grams of marijuana, for which she was properly sentenced to reclusion
perpetua by the RTC, affirmed by the Court of Appeals.

In the same vein, the fine of ₱500,000.00 imposed upon accused-appellant by the
RTC, affirmed by the Court of Appeals, is also correct, as the same is still within
the range of fines imposable on any person who possessed prohibited drugs
without any authority, under Article II, Section 8 of Republic Act No. 6425, as
amended.

WHEREFORE, premises considered, the Decision dated September 21, 2006 of


the Court of Appeals in CA-G.R. CR.-H.C. No. 00381, is hereby AFFIRMED in
toto. No costs.

SO ORDERED.

Stonehill v. Diokno, G.R. No. L-19550, June 19, 1967

Upon application of the officers of the government named on the margin 1 —
hereinafter referred to as Respondents-Prosecutors — several judges 2 —
hereinafter referred to as Respondents-Judges — issued, on different dates, 3 a total
of 42 search warrants against petitioners herein 4 and/or the corporations of which
they were officers,5 directed to the any peace officer, to search the persons above-
named and/or the premises of their offices, warehouses and/or residences, and to
seize and take possession of the following personal property to wit:

Books of accounts, financial records, vouchers, correspondence, receipts,


ledgers, journals, portfolios, credit journals, typewriters, and other
documents and/or papers showing all business transactions including
disbursements receipts, balance sheets and profit and loss statements and
Bobbins (cigarette wrappers).

as "the subject of the offense; stolen or embezzled and proceeds or fruits of the
offense," or "used or intended to be used as the means of committing the offense,"
which is described in the applications adverted to above as "violation of Central
Bank Laws, Tariff and Customs Laws, Internal Revenue (Code) and the Revised
Penal Code."
Alleging that the aforementioned search warrants are null and void, as
contravening the Constitution and the Rules of Court — because, inter alia: (1)
they do not describe with particularity the documents, books and things to be
seized; (2) cash money, not mentioned in the warrants, were actually seized; (3) the
warrants were issued to fish evidence against the aforementioned petitioners in
deportation cases filed against them; (4) the searches and seizures were made in an
illegal manner; and (5) the documents, papers and cash money seized were not
delivered to the courts that issued the warrants, to be disposed of in accordance
with law — on March 20, 1962, said petitioners filed with the Supreme Court this
original action for certiorari, prohibition, mandamus and injunction, and prayed
that, pending final disposition of the present case, a writ of preliminary injunction
be issued restraining Respondents-Prosecutors, their agents and /or representatives
from using the effects seized as aforementioned or any copies thereof, in the
deportation cases already adverted to, and that, in due course, thereafter, decision
be rendered quashing the contested search warrants and declaring the same null
and void, and commanding the respondents, their agents or representatives to
return to petitioners herein, in accordance with Section 3, Rule 67, of the Rules of
Court, the documents, papers, things and cash moneys seized or confiscated under
the search warrants in question.

In their answer, respondents-prosecutors alleged, 6 (1) that the contested search


warrants are valid and have been issued in accordance with law; (2) that the defects
of said warrants, if any, were cured by petitioners' consent; and (3) that, in any
event, the effects seized are admissible in evidence against herein petitioners,
regardless of the alleged illegality of the aforementioned searches and seizures.

On March 22, 1962, this Court issued the writ of preliminary injunction prayed for
in the petition. However, by resolution dated June 29, 1962, the writ was partially
lifted or dissolved, insofar as the papers, documents and things seized from the
offices of the corporations above mentioned are concerned; but, the injunction was
maintained as regards the papers, documents and things found and seized in the
residences of petitioners herein.7

Thus, the documents, papers, and things seized under the alleged authority of the
warrants in question may be split into two (2) major groups, namely: (a) those
found and seized in the offices of the aforementioned corporations, and (b) those
found and seized in the residences of petitioners herein.

As regards the first group, we hold that petitioners herein have no cause of action
to assail the legality of the contested warrants and of the seizures made in
pursuance thereof, for the simple reason that said corporations have their respective
personalities, separate and distinct from the personality of herein petitioners,
regardless of the amount of shares of stock or of the interest of each of them in said
corporations, and whatever the offices they hold therein may be. 8 Indeed, it is well
settled that the legality of a seizure can be contested only by the party whose rights
have been impaired thereby,9 and that the objection to an unlawful search and
seizure is purely personal and cannot be availed of by third
parties. 10 Consequently, petitioners herein may not validly object to the use in
evidence against them of the documents, papers and things seized from the offices
and premises of the corporations adverted to above, since the right to object to the
admission of said papers in evidence belongs exclusively to the corporations, to
whom the seized effects belong, and may not be invoked by the corporate officers
in proceedings against them in their individual capacity. 11 Indeed, it has been held:

. . . that the Government's action in gaining possession of papers belonging


to the corporation did not relate to nor did it affect the personal defendants.
If these papers were unlawfully seized and thereby the constitutional rights
of or any one were invaded, they were the rights of the corporation and not
the rights of the other defendants. Next, it is clear that a question of the
lawfulness of a seizure can be raised only by one whose rights have been
invaded. Certainly, such a seizure, if unlawful, could not affect the
constitutional rights of defendants whose property had not been seized or
the privacy of whose homes had not been disturbed; nor could they claim for
themselves the benefits of the Fourth Amendment, when its violation, if any,
was with reference to the rights of another. Remus vs. United
States (C.C.A.)291 F. 501, 511. It follows, therefore, that the question of the
admissibility of the evidence based on an alleged unlawful search and
seizure does not extend to the personal defendants but
embraces only the corporation whose property was taken. . . . (A
Guckenheimer & Bros. Co. vs. United States, [1925] 3 F. 2d. 786, 789,
Emphasis supplied.)

With respect to the documents, papers and things seized in the residences of
petitioners herein, the aforementioned resolution of June 29, 1962, lifted the writ of
preliminary injunction previously issued by this Court, 12 thereby, in effect,
restraining herein Respondents-Prosecutors from using them in evidence against
petitioners herein.

In connection with said documents, papers and things, two (2) important questions
need be settled, namely: (1) whether the search warrants in question, and the
searches and seizures made under the authority thereof, are valid or not, and (2) if
the answer to the preceding question is in the negative, whether said documents,
papers and things may be used in evidence against petitioners herein.1äwphï1.ñët

Petitioners maintain that the aforementioned search warrants are in the nature of
general warrants and that accordingly, the seizures effected upon the authority
there of are null and void. In this connection, the Constitution 13 provides:

The right of the people to be secure in their persons, houses, papers, and
effects against unreasonable searches and seizures shall not be violated, and
no warrants shall issue but upon probable cause, to be determined by the
judge after examination under oath or affirmation of the complainant and the
witnesses he may produce, and particularly describing the place to be
searched, and the persons or things to be seized.

Two points must be stressed in connection with this constitutional mandate,


namely: (1) that no warrant shall issue but upon probable cause, to be determined
by the judge in the manner set forth in said provision; and (2) that the warrant
shall particularly describe the things to be seized.

None of these requirements has been complied with in the contested warrants.
Indeed, the same were issued upon applications stating that the natural and
juridical person therein named had committed a "violation of Central Ban Laws,
Tariff and Customs Laws, Internal Revenue (Code) and Revised Penal Code." In
other words, no specific offense had been alleged in said applications. The
averments thereof with respect to the offense committed were abstract. As a
consequence, it was impossible for the judges who issued the warrants to have
found the existence of probable cause, for the same presupposes the introduction of
competent proof that the party against whom it is sought has
performed particular acts, or committed specific omissions, violating a given
provision of our criminal laws. As a matter of fact, the applications involved in this
case do not allege any specific acts performed by herein petitioners. It would be the
legal heresy, of the highest order, to convict anybody of a "violation of Central
Bank Laws, Tariff and Customs Laws, Internal Revenue (Code) and Revised Penal
Code," — as alleged in the aforementioned applications — without reference to
any determinate provision of said laws or

To uphold the validity of the warrants in question would be to wipe out completely
one of the most fundamental rights guaranteed in our Constitution, for it would
place the sanctity of the domicile and the privacy of communication and
correspondence at the mercy of the whims caprice or passion of peace officers.
This is precisely the evil sought to be remedied by the constitutional provision
above quoted — to outlaw the so-called general warrants. It is not difficult to
imagine what would happen, in times of keen political strife, when the party in
power feels that the minority is likely to wrest it, even though by legal means.

Such is the seriousness of the irregularities committed in connection with the


disputed search warrants, that this Court deemed it fit to amend Section 3 of Rule
122 of the former Rules of Court 14 by providing in its counterpart, under the
Revised Rules of Court 15 that "a search warrant shall not issue but upon probable
cause in connection with one specific offense." Not satisfied with this qualification,
the Court added thereto a paragraph, directing that "no search warrant shall issue
for more than one specific offense."
The grave violation of the Constitution made in the application for the contested
search warrants was compounded by the description therein made of the effects to
be searched for and seized, to wit:

Books of accounts, financial records, vouchers, journals, correspondence,


receipts, ledgers, portfolios, credit journals, typewriters, and other
documents and/or papers showing all business transactions including
disbursement receipts, balance sheets and related profit and loss statements.

Thus, the warrants authorized the search for and seizure of records pertaining to all
business transactions of petitioners herein, regardless of whether the transactions
were legal or illegal. The warrants sanctioned the seizure of all records of the
petitioners and the aforementioned corporations, whatever their nature, thus openly
contravening the explicit command of our Bill of Rights — that the things to be
seized be particularly described — as well as tending to defeat its major objective:
the elimination of general warrants.

Relying upon Moncado vs. People's Court (80 Phil. 1), Respondents-Prosecutors


maintain that, even if the searches and seizures under consideration were
unconstitutional, the documents, papers and things thus seized are admissible in
evidence against petitioners herein. Upon mature deliberation, however, we are
unanimously of the opinion that the position taken in the Moncado case must be
abandoned. Said position was in line with the American common law rule, that the
criminal should not be allowed to go free merely "because the constable has
blundered," 16 upon the theory that the constitutional prohibition against
unreasonable searches and seizures is protected by means other than the exclusion
of evidence unlawfully obtained, 17 such as the common-law action for damages
against the searching officer, against the party who procured the issuance of the
search warrant and against those assisting in the execution of an illegal search,
their criminal punishment, resistance, without liability to an unlawful seizure, and
such other legal remedies as may be provided by other laws.

However, most common law jurisdictions have already given up this approach and
eventually adopted the exclusionary rule, realizing that this is the only practical
means of enforcing the constitutional injunction against unreasonable searches and
seizures. In the language of Judge Learned Hand:

As we understand it, the reason for the exclusion of evidence competent as


such, which has been unlawfully acquired, is that exclusion is the only
practical way of enforcing the constitutional privilege. In earlier times the
action of trespass against the offending official may have been protection
enough; but that is true no longer. Only in case the prosecution which itself
controls the seizing officials, knows that it cannot profit by their wrong will
that wrong be repressed.18
In fact, over thirty (30) years before, the Federal Supreme Court had already
declared:

If letters and private documents can thus be seized and held and used in
evidence against a citizen accused of an offense, the protection of the 4th
Amendment, declaring his rights to be secure against such searches and
seizures, is of no value, and, so far as those thus placed are concerned, might
as well be stricken from the Constitution. The efforts of the courts and their
officials to bring the guilty to punishment, praiseworthy as they are, are not
to be aided by the sacrifice of those great principles established by years of
endeavor and suffering which have resulted in their embodiment in the
fundamental law of the land.19

This view was, not only reiterated, but, also, broadened in subsequent decisions on
the same Federal Court. 20 After reviewing previous decisions thereon, said Court
held, in Mapp vs. Ohio (supra.):

. . . Today we once again examine the Wolf's constitutional documentation


of the right of privacy free from unreasonable state intrusion, and after its
dozen years on our books, are led by it to close the only courtroom door
remaining open to evidence secured by official lawlessness in flagrant abuse
of that basic right, reserved to all persons as a specific guarantee against that
very same unlawful conduct. We hold that all evidence obtained by searches
and seizures in violation of the Constitution is, by that same authority,
inadmissible in a State.

Since the Fourth Amendment's right of privacy has been declared


enforceable against the States through the Due Process Clause of the
Fourteenth, it is enforceable against them by the same sanction of exclusion
as it used against the Federal Government. Were it otherwise, then just as
without the Weeks rule the assurance against unreasonable federal searches
and seizures would be "a form of words," valueless and underserving of
mention in a perpetual charter of inestimable human liberties, so
too, without that rule the freedom from state invasions of privacy would be
so ephemeral and so neatly severed from its conceptual nexus with the
freedom from all brutish means of coercing evidence as not to permit this
Court's high regard as a freedom "implicit in the concept of ordered
liberty." At the time that the Court held in Wolf that the amendment was
applicable to the States through the Due Process Clause, the cases of this
Court as we have seen, had steadfastly held that as to federal officers the
Fourth Amendment included the exclusion of the evidence seized in
violation of its provisions. Even Wolf "stoutly adhered" to that proposition.
The right to when conceded operatively enforceable against the States, was
not susceptible of destruction by avulsion of the sanction upon which its
protection and enjoyment had always been deemed dependent under the
Boyd, Weeks and Silverthorne Cases. Therefore, in extending the
substantive protections of due process to all constitutionally unreasonable
searches — state or federal — it was logically and constitutionally
necessarily that the exclusion doctrine — an essential part of the right to
privacy — be also insisted upon as an essential ingredient of the right newly
recognized by the Wolf Case. In short, the admission of the new
constitutional Right by Wolf could not tolerate denial of its most important
constitutional privilege, namely, the exclusion of the evidence which an
accused had been forced to give by reason of the unlawful seizure. To hold
otherwise is to grant the right but in reality to withhold its privilege and
enjoyment. Only last year the Court itself recognized that the purpose of the
exclusionary rule to "is to deter — to compel respect for the constitutional
guaranty in the only effectively available way — by removing the incentive
to disregard it" . . . .

The ignoble shortcut to conviction left open to the State tends to destroy the
entire system of constitutional restraints on which the liberties of the people
rest. Having once recognized that the right to privacy embodied in the
Fourth Amendment is enforceable against the States, and that the right to be
secure against rude invasions of privacy by state officers is, therefore
constitutional in origin, we can no longer permit that right to remain an
empty promise. Because it is enforceable in the same manner and to like
effect as other basic rights secured by its Due Process Clause, we can no
longer permit it to be revocable at the whim of any police officer who, in the
name of law enforcement itself, chooses to suspend its enjoyment. Our
decision, founded on reason and truth, gives to the individual no more than
that which the Constitution guarantees him to the police officer no less than
that to which honest law enforcement is entitled, and, to the courts, that
judicial integrity so necessary in the true administration of justice.
(emphasis ours.)

Indeed, the non-exclusionary rule is contrary, not only to the letter, but also, to the
spirit of the constitutional injunction against unreasonable searches and seizures.
To be sure, if the applicant for a search warrant has competent evidence to
establish probable cause of the commission of a given crime by the party against
whom the warrant is intended, then there is no reason why the applicant should not
comply with the requirements of the fundamental law. Upon the other hand, if he
has no such competent evidence, then it is not possible for the Judge to find that
there is probable cause, and, hence, no justification for the issuance of the warrant.
The only possible explanation (not justification) for its issuance is the necessity
of fishing evidence of the commission of a crime. But, then, this fishing expedition
is indicative of the absence of evidence to establish a probable cause.

Moreover, the theory that the criminal prosecution of those who secure an illegal
search warrant and/or make unreasonable searches or seizures would suffice to
protect the constitutional guarantee under consideration, overlooks the fact that
violations thereof are, in general, committed By agents of the party in power, for,
certainly, those belonging to the minority could not possibly abuse a power they do
not have. Regardless of the handicap under which the minority usually — but,
understandably — finds itself in prosecuting agents of the majority, one must not
lose sight of the fact that the psychological and moral effect of the possibility 21 of
securing their conviction, is watered down by the pardoning power of the party for
whose benefit the illegality had been committed.

In their Motion for Reconsideration and Amendment of the Resolution of this


Court dated June 29, 1962, petitioners allege that Rooms Nos. 81 and 91 of
Carmen Apartments, House No. 2008, Dewey Boulevard, House No. 1436,
Colorado Street, and Room No. 304 of the Army-Navy Club, should be included
among the premises considered in said Resolution as residences of herein
petitioners, Harry S. Stonehill, Robert P. Brook, John J. Brooks and Karl Beck,
respectively, and that, furthermore, the records, papers and other effects seized in
the offices of the corporations above referred to include personal belongings of
said petitioners and other effects under their exclusive possession and control, for
the exclusion of which they have a standing under the latest rulings of the federal
courts of federal courts of the United States. 22

We note, however, that petitioners' theory, regarding their alleged possession of


and control over the aforementioned records, papers and effects, and the alleged
"personal" nature thereof, has Been Advanced, not in their petition or amended
petition herein, but in the Motion for Reconsideration and Amendment of the
Resolution of June 29, 1962. In other words, said theory would appear to be
readjustment of that followed in said petitions, to suit the approach intimated in the
Resolution sought to be reconsidered and amended. Then, too, some of the
affidavits or copies of alleged affidavits attached to said motion for
reconsideration, or submitted in support thereof, contain either inconsistent
allegations, or allegations inconsistent with the theory now advanced by petitioners
herein.

Upon the other hand, we are not satisfied that the allegations of said petitions said
motion for reconsideration, and the contents of the aforementioned affidavits and
other papers submitted in support of said motion, have sufficiently established the
facts or conditions contemplated in the cases relied upon by the petitioners; to
warrant application of the views therein expressed, should we agree thereto. At any
rate, we do not deem it necessary to express our opinion thereon, it being best to
leave the matter open for determination in appropriate cases in the future.

We hold, therefore, that the doctrine adopted in the Moncado case must be, as it is
hereby, abandoned; that the warrants for the search of three (3) residences of herein
petitioners, as specified in the Resolution of June 29, 1962, are null and void; that
the searches and seizures therein made are illegal; that the writ of preliminary
injunction heretofore issued, in connection with the documents, papers and other
effects thus seized in said residences of herein petitioners is hereby made
permanent; that the writs prayed for are granted, insofar as the documents, papers
and other effects so seized in the aforementioned residences are concerned; that the
aforementioned motion for Reconsideration and Amendment should be, as it is
hereby, denied; and that the petition herein is dismissed and the writs prayed for
denied, as regards the documents, papers and other effects seized in the twenty-
nine (29) places, offices and other premises enumerated in the same Resolution,
without special pronouncement as to costs.

It is so ordered.

Reyes, J.B.L., Dizon, Makalintal, Bengzon, J.P., Zaldivar and Sanchez, JJ.,
concur.

CASTRO, J., concurring and dissenting:

From my analysis of the opinion written by Chief Justice Roberto Concepcion and
from the import of the deliberations of the Court on this case, I gather the
following distinct conclusions:

1. All the search warrants served by the National Bureau of Investigation in


this case are general warrants and are therefore proscribed by, and in
violation of, paragraph 3 of section 1 of Article III (Bill of Rights) of the
Constitution;

2. All the searches and seizures conducted under the authority of the said
search warrants were consequently illegal;

3. The non-exclusionary rule enunciated in Moncado vs. People, 80 Phil. 1,


should be, and is declared, abandoned;

4. The search warrants served at the three residences of the petitioners


are expressly declared null and void the searches and seizures therein made
are expressly declared illegal; and the writ of preliminary injunction
heretofore issued against the use of the documents, papers and effect seized
in the said residences is made permanent; and

5. Reasoning that the petitioners have not in their pleadings satisfactorily


demonstrated that they have legal standing to move for the suppression of
the documents, papers and effects seized in the places other than the three
residences adverted to above, the opinion written by the Chief
Justice refrains from expressly declaring as null and void the such warrants
served at such other places and as illegal the searches and seizures made
therein, and leaves "the matter open for determination in appropriate cases
in the future."

It is precisely the position taken by the Chief Justice summarized in the


immediately preceding paragraph (numbered 5) with which I am not in accord.

I do not share his reluctance or unwillingness to expressly declare, at this time, the
nullity of the search warrants served at places other than the three residences, and
the illegibility of the searches and seizures conducted under the authority thereof.
In my view even the exacerbating passions and prejudices inordinately generated
by the environmental political and moral developments of this case should not
deter this Court from forthrightly laying down the law not only for this case but as
well for future cases and future generations. All the search warrants, without
exception, in this case are admittedly general, blanket and roving warrants and are
therefore admittedly and indisputably outlawed by the Constitution; and the
searches and seizures made were therefore unlawful. That the petitioners, let us
assume in gratia argumente, have no legal standing to ask for the suppression of
the papers, things and effects seized from places other than their residences, to my
mind, cannot in any manner affect, alter or otherwise modify the intrinsic nullity of
the search warrants and the intrinsic illegality of the searches and seizures made
thereunder. Whether or not the petitioners possess legal standing the said warrants
are void and remain void, and the searches and seizures were illegal and remain
illegal. No inference can be drawn from the words of the Constitution that "legal
standing" or the lack of it is a determinant of the nullity or validity of a search
warrant or of the lawfulness or illegality of a search or seizure.

On the question of legal standing, I am of the conviction that, upon the pleadings
submitted to this Court the petitioners have the requisite legal standing to move for
the suppression and return of the documents, papers and effects that were seized
from places other than their family residences.

Our constitutional provision on searches and seizures was derived


almost verbatim from the Fourth Amendment to the United States Constitution. In
the many years of judicial construction and interpretation of the said constitutional
provision, our courts have invariably regarded as doctrinal the pronouncement
made on the Fourth Amendment by federal courts, especially the Federal Supreme
Court and the Federal Circuit Courts of Appeals.

The U.S. doctrines and pertinent cases on standing to move for the suppression or
return of documents, papers and effects which are the fruits of an unlawful search
and seizure, may be summarized as follows; (a) ownership of documents, papers
and effects gives "standing;" (b) ownership and/or control or possession — actual
or constructive — of premises searched gives "standing"; and (c) the "aggrieved
person" doctrine where the search warrant and the sworn application for search
warrant are "primarily" directed solely and exclusively against the "aggrieved
person," gives "standing."

An examination of the search warrants in this case will readily show that,
excepting three, all were directed against the petitioners personally. In some of
them, the petitioners were named personally, followed by the designation, "the
President and/or General Manager" of the particular corporation. The three
warrants excepted named three corporate defendants. But the
"office/house/warehouse/premises" mentioned in the said three warrants were also
the same "office/house/warehouse/premises" declared to be owned by or under the
control of the petitioners in all the other search warrants directed against the
petitioners and/or "the President and/or General Manager" of the particular
corporation. (see pages 5-24 of Petitioners' Reply of April 2, 1962). The searches
and seizures were to be made, and were actually made, in the
"office/house/warehouse/premises" owned by or under the control of the
petitioners.

Ownership of matters seized gives "standing."

Ownership of the properties seized alone entitles the petitioners to bring a motion
to return and suppress, and gives them standing as persons aggrieved by an
unlawful search and seizure regardless of their location at the time of
seizure. Jones vs. United States, 362 U.S. 257, 261 (1960) (narcotics stored in the
apartment of a friend of the defendant); Henzel vs. United States, 296 F. 2d. 650,
652-53 (5th Cir. 1961), (personal and corporate papers of corporation of which the
defendant was president), United States vs. Jeffers, 342 U.S. 48 (1951) (narcotics
seized in an apartment not belonging to the defendant); Pielow vs. United States, 8
F. 2d 492, 493 (9th Cir. 1925) (books seized from the defendant's sister but
belonging to the defendant); Cf. Villano vs. United States, 310 F. 2d 680, 683 (10th
Cir. 1962) (papers seized in desk neither owned by nor in exclusive possession of
the defendant).

In a very recent case (decided by the U.S. Supreme Court on December 12, 1966),
it was held that under the constitutional provision against unlawful searches and
seizures, a person places himself or his property within a constitutionally protected
area, be it his home or his office, his hotel room or his automobile:

Where the argument falls is in its misapprehension of the fundamental


nature and scope of Fourth Amendment protection. What the Fourth
Amendment protects is the security a man relies upon when he places
himself or his property within a constitutionally protected area, be it his
home or his office, his hotel room or his automobile. There he is protected
from unwarranted governmental intrusion. And when he puts some thing in
his filing cabinet, in his desk drawer, or in his pocket, he has the right to
know it will be secure from an unreasonable search or an unreasonable
seizure. So it was that the Fourth Amendment could not tolerate the
warrantless search of the hotel room in Jeffers, the purloining of the
petitioner's private papers in Gouled, or the surreptitious electronic
surveilance in Silverman. Countless other cases which have come to this
Court over the years have involved a myriad of differing factual contexts in
which the protections of the Fourth Amendment have been appropriately
invoked. No doubt, the future will bring countless others. By nothing we say
here do we either foresee or foreclose factual situations to which the Fourth
Amendment may be applicable. (Hoffa vs. U.S., 87 S. Ct. 408 (December
12, 1966). See also U.S. vs. Jeffers, 342 U.S. 48, 72 S. Ct. 93 (November 13,
1951). (Emphasis supplied).

Control of premises searched gives "standing."

Independent of ownership or other personal interest in the records and documents


seized, the petitioners have standing to move for return and suppression by virtue
of their proprietary or leasehold interest in many of the premises searched. These
proprietary and leasehold interests have been sufficiently set forth in their motion
for reconsideration and need not be recounted here, except to emphasize that the
petitioners paid rent, directly or indirectly, for practically all the premises searched
(Room 91, 84 Carmen Apts; Room 304, Army & Navy Club; Premises 2008,
Dewey Boulevard; 1436 Colorado Street); maintained personal offices within the
corporate offices (IBMC, USTC); had made improvements or furnished such
offices; or had paid for the filing cabinets in which the papers were stored (Room
204, Army & Navy Club); and individually, or through their respective spouses,
owned the controlling stock of the corporations involved. The petitioners'
proprietary interest in most, if not all, of the premises searched therefore
independently gives them standing to move for the return and suppression of the
books, papers and affects seized therefrom.

In Jones vs. United States, supra, the U.S. Supreme Court delineated the nature
and extent of the interest in the searched premises necessary to maintain a motion
to suppress. After reviewing what it considered to be the unduly technical standard
of the then prevailing circuit court decisions, the Supreme Court said (362 U.S.
266):

We do not lightly depart from this course of decisions by the lower courts.
We are persuaded, however, that it is unnecessarily and ill-advised to import
into the law surrounding the constitutional right to be free from
unreasonable searches and seizures subtle distinctions, developed and
refined by the common law in evolving the body of private property law
which, more than almost any other branch of law, has been shaped by
distinctions whose validity is largely historical. Even in the area from which
they derive, due consideration has led to the discarding of those distinctions
in the homeland of the common law. See Occupiers' Liability Act, 1957, 5
and 6 Eliz. 2, c. 31, carrying out Law Reform Committee, Third Report,
Cmd. 9305. Distinctions such as those between "lessee", "licensee,"
"invitee," "guest," often only of gossamer strength, ought not be
determinative in fashioning procedures ultimately referable to constitutional
safeguards. See also Chapman vs. United States, 354 U.S. 610, 616-17
(1961).

It has never been held that a person with requisite interest in the premises searched
must own the property seized in order to have standing in a motion to return and
suppress. In Alioto vs. United States, 216 F. Supp. 48 (1963), a Bookkeeper for
several corporations from whose apartment the corporate records were seized
successfully moved for their return. In United States vs. Antonelli, Fireworks Co.,
53 F. Supp. 870, 873 (W D. N. Y. 1943), the corporation's president successfully
moved for the return and suppression is to him of both personal and corporate
documents seized from his home during the course of an illegal search:

The lawful possession by Antonelli of documents and property, "either his


own or the corporation's was entitled to protection against unreasonable
search and seizure. Under the circumstances in the case at bar, the search
and seizure were unreasonable and unlawful. The motion for the return of
seized article and the suppression of the evidence so obtained should be
granted. (Emphasis supplied).

Time was when only a person who had property in interest in either the place
searched or the articles seize had the necessary standing to invoke the protection of
the exclusionary rule. But in MacDonald vs. Unite States, 335 U.S. 461 (1948),
Justice Robert Jackson joined by Justice Felix Frankfurter, advanced the view that
"even a guest may expect the shelter of the rooftree he is under against criminal
intrusion." This view finally became the official view of the U.S. Supreme Court
and was articulated in United States vs. Jeffers, 432 U.S 48 (1951). Nine years
later, in 1960, in Jones vs. Unite States, 362 U.S. 257, 267, the U.S. Supreme Court
went a step further. Jones was a mere guest in the apartment unlawfully searched
but the Court nonetheless declared that the exclusionary rule protected him as well.
The concept of "person aggrieved by an unlawful search and seizure" was enlarged
to include "anyone legitimately on premise where the search occurs."

Shortly after the U.S. Supreme Court's Jones decision the U.S. Court of Appeals
for the Fifth Circuit held that the defendant organizer, sole stockholder and
president of a corporation had standing in a mail fraud prosecution against him to
demand the return and suppression of corporate property. Henzel vs. United States,
296 F 2d 650, 652 (5th Cir. 1961), supra. The court conclude that the defendant
had standing on two independent grounds: First — he had a sufficient interest in
the property seized, and second — he had an adequate interest in the premises
searched (just like in the case at bar). A postal inspector had unlawfully searched
the corporation' premises and had seized most of the corporation's book and
records. Looking to Jones, the court observed:

Jones clearly tells us, therefore, what is not required qualify one as a


"person aggrieved by an unlawful search and seizure." It tells us that
appellant should not have been precluded from objecting to the Postal
Inspector's search and seizure of the corporation's books and records merely
because the appellant did not show ownership or possession of the books
and records or a substantial possessory interest in the invade premises . . .
(Henzel vs. United States, 296 F. 2d at 651). .

Henzel was soon followed by Villano vs. United States, 310 F. 2d 680, 683, (10th
Cir. 1962). In Villano, police officers seized two notebooks from a desk in the
defendant's place of employment; the defendant did not claim ownership of either;
he asserted that several employees (including himself) used the notebooks. The
Court held that the employee had a protected interest and that there also was an
invasion of privacy. Both Henzel and Villano considered also the fact that the
search and seizure were "directed at" the moving defendant. Henzel vs. United
States, 296 F. 2d at 682; Villano vs. United States, 310 F. 2d at 683.

In a case in which an attorney closed his law office, placed his files in storage and
went to Puerto Rico, the Court of Appeals for the Eighth Circuit recognized his
standing to move to quash as unreasonable search and seizure under the Fourth
Amendment of the U.S. Constitution a grand jury subpoena duces tecum directed
to the custodian of his files. The Government contended that the petitioner had no
standing because the books and papers were physically in the possession of the
custodian, and because the subpoena was directed against the custodian. The court
rejected the contention, holding that

Schwimmer legally had such possession, control and unrelinquished


personal rights in the books and papers as not to enable the question of
unreasonable search and seizure to be escaped through the mere procedural
device of compelling a third-party naked possessor to produce and deliver
them. Schwimmer vs. United States, 232 F. 2d 855, 861 (8th Cir. 1956).

Aggrieved person doctrine where the search warrant s primarily directed against
said person gives "standing."

The latest United States decision squarely in point is United States vs. Birrell, 242
F. Supp. 191 (1965, U.S.D.C. S.D.N.Y.). The defendant had stored with an
attorney certain files and papers, which attorney, by the name of Dunn, was not, at
the time of the seizing of the records, Birrell's attorney. * Dunn, in turn, had stored
most of the records at his home in the country and on a farm which, according to
Dunn's affidavit, was under his (Dunn's) "control and management." The papers
turned out to be private, personal and business papers together with corporate
books and records of certain unnamed corporations in which Birrell did not even
claim ownership. (All of these type records were seized in the case at bar).
Nevertheless, the search in Birrell was held invalid by the court which held that
even though Birrell did not own the premises where the records were stored, he
had "standing" to move for the return of all the papers and properties seized. The
court, relying on Jones vs. U.S., supra; U.S. vs. Antonelli Fireworks Co., 53 F.
Supp. 870, Aff'd 155 F. 2d 631: Henzel vs. U.S., supra; and Schwimmer vs. U.S.,
supra, pointed out that

It is overwhelmingly established that the searches here in question were


directed solely and exclusively against Birrell. The only person suggested in
the papers as having violated the law was Birrell. The first search warrant
described the records as having been used "in committing a violation of
Title 18, United States Code, Section 1341, by the use of the mails by one
Lowell M. Birrell, . . ." The second search warrant was captioned: "United
States of America vs. Lowell M. Birrell. (p. 198)

Possession (actual or constructive), no less than ownership, gives standing


to move to suppress. Such was the rule even before Jones. (p. 199)

If, as thus indicated Birrell had at least constructive possession of the


records stored with Dunn, it matters not whether he had any interest in the
premises searched. See also Jeffers v. United States, 88 U.S. Appl. D.C. 58,
187 F. 2d 498 (1950), affirmed 432 U.S. 48, 72 S. Ct. 93, 96 L. Ed. 459
(1951).

The ruling in the Birrell case was reaffirmed on motion for reargument; the United
States did not appeal from this decision. The factual situation in Birrell is
strikingly similar to the case of the present petitioners; as in Birrell, many personal
and corporate papers were seized from premises not petitioners' family residences;
as in Birrell, the searches were "PRIMARILY DIRECTED SOLETY AND
EXCLUSIVELY" against the petitioners. Still both types of documents were
suppressed in Birrell because of the illegal search. In the case at bar, the petitioners
connection with the premises raided is much closer than in Birrell.

Thus, the petitioners have full standing to move for the quashing of all the warrants
regardless whether these were directed against residences in the narrow sense of
the word, as long as the documents were personal papers of the petitioners or (to
the extent that they were corporate papers) were held by them in a personal
capacity or under their personal control.

Prescinding a from the foregoing, this Court, at all events, should order the return
to the petitioners all personal and private papers and effects seized, no matter
where these were seized, whether from their residences or corporate offices or any
other place or places. The uncontradicted sworn statements of the petitioners in
their, various pleadings submitted to this Court indisputably show that amongst the
things seized from the corporate offices and other places
were personal and private papers and effects belonging to the petitioners.

If there should be any categorization of the documents, papers and things which
where the objects of the unlawful searches and seizures, I submit that the grouping
should be: (a) personal or private papers of the petitioners were they were
unlawfully seized, be it their family residences offices, warehouses and/or premises
owned and/or possessed (actually or constructively) by them as shown in all the
search and in the sworn applications filed in securing the void search warrants and
(b) purely corporate papers belonging to corporations. Under such categorization
or grouping, the determination of which unlawfully seized papers, documents and
things are personal/private of the petitioners or purely corporate papers will have
to be left to the lower courts which issued the void search warrants in ultimately
effecting the suppression and/or return of the said documents.

And as unequivocally indicated by the authorities above cited, the petitioners


likewise have clear legal standing to move for the suppression of purely
corporate papers as "President and/or General Manager" of the corporations
involved as specifically mentioned in the void search warrants.

Finally, I must articulate my persuasion that although the cases cited in my


disquisition were criminal prosecutions, the great clauses of the constitutional
proscription on illegal searches and seizures do not withhold the mantle of their
protection from cases not criminal in origin or nature.

Columbia Pictures v. Flores, G.R. No. 78631, June 29, 1993

Before us is a petition for certiorari seeking to set aside the order dated May 29,
1987 of the Regional Trial Court of the National Capital Region (Branch 167,
Pasig) directing the immediate release and return of television sets, video cassette
recorders, rewinders, tape head cleaners, accessories, equipment, and other
paraphernalia or pieces of machinery which had been seized by operatives of the
National Bureau of Investigation by virtue of a search warrant.chanrobles virtual
lawlibrary

Petitioners herein are all foreign corporations organized and existing under the
laws of the United States of America and represented in the Philippines by their
attorney-in-fact, Rebecca Benitez-Cruz of the Motion Picture Association of
America, Inc. (MPAA for brevity). Private respondent FGT Video Network, Inc. is
a merger of Fox, Galactic, and Technica Video. It is registered with and licensed
by the Videogram Regulatory Board as a distributor under License No. 1333
VMM. Technica Video, Inc. which is part of the merger, is registered with and
licensed as a reproducer by the said board under License No. 967 VMM (p. 11,
Rollo).

In a letter dated April 20, 1987, the MPAA, through counsel Rico V. Domingo,
lodged a complaint before then Director Antonio Carpio of the National Bureau of
Investigation (NBI) against certain video establishments for violation of
Presidential Decree No. 49 (Protection of Intellectual Property), as amended by
Presidential Decree No. 1988, in connection with its anti-piracy campaign.
Specifically complaining of the "unauthorized sale, rental, reproduction and/or
disposition of copyrighted film", the MPAA sought the NBI’s "urgent assistance in
the conduct of search and seizure operations in Metro Manila and elsewhere." (p.
29, Rollo.)

On the basis of said letter, NBI and private agents conducted discreet surveillance
operations on certain video establishments, among them private respondent FGT
Video Network, Inc. (FGT). Thus, on April 20, 1987, Danilo Manalang, a.k.a.
Ronaldo Lim, allegedly an NBI agent, went to the office of FGT to have the
copyrighted motion pictures "Cleopatra" owned by Twentieth Century Fox Film
Corp. and "The Ten Commandments" owned by Paramount Pictures, Inc.
reproduced or retaped in video format. For the reproduction services, FGT issued
Order Slip No. 3482 dated April 20, 1987 and Delivery Slip No. 118667 dated
April 22, 1987, for which services Danilo Manalang paid P45.00. On May 5, 1987,
Manalang also had MGM’s copyrighted film "Walk Like a Man" reproduced or
retaped by FGT for P15.00 (p. 5, Rollo).chanrobles.com.ph : virtual law library

Consequently, on May 14, 1987, NBI Agent III Lauro C. Reyes, with Manalang
and Rebecca Benitez-Cruz as witnesses, applied for a search warrant with the
Regional Trial Court in Pasig. Introduced as evidence in support of the application
were the following: the letter dated April 20, 1987 of the MPAA through Rico V.
Domingo (Exh. A): FGT’s Order Slip No. 3842 (Exh. B); FGT’s Delivery Slip No.
118667 (Exh. B-1); video cassettes containing the film "The Ten Commandments"
(Exh. B-1-A, B-1-B); video cassette containing the film "Cleopatra" (Exh. B-1-C);
video cassette containing the film "Walk Like a Man" (Exh. B-1-D); FGT’s Order
Slip No. 3923 dated May 5, 1987 (Exh. B-2); FGT’s Delivery Slip No. 123321
dated May 6, 1987 (Exh. B-3); list of copyrighted MPAA member company titles
(Exh. C); sketch of location of FGT’s office or premises (Exh. D); affidavit of
Rebecca Benitez-Cruz (Exh. E); special power of attorney designating Ms.
Benitez-Cruz as petitioners’ attorney-in-fact (Exh. F to F-8); and affidavit of
Danilo Manalang (Exh. G).chanrobles law library : red

Upon the offer of these pieces of evidence, Judge Alfredo C. Flores of the
aforesaid court, issued Search Warrant No. 45 which reads:chanrob1es virtual 1aw
library

TO ANY PEACE OFFICER:chanrob1es virtual 1aw library


GREETINGS:chanrob1es virtual 1aw library

It appearing to the satisfaction of the Undersigned after examining under oath NBI
Senior Agent Lauro C. Reyes and his witnesses Mr. Danilo Manalang and Ms.
Rebecca Benitez-Cruz, that there is a probable cause to believe that Violation of
Section 56 P.D. No. 49 as amended by P.D. No. 1988 (otherwise known as the
Decree on Protection of Intellectual Property) has been committed and that there
are good and sufficient reasons to believe that FGT Video Network, Inc., Manuel
Mendoza, Alfredo C. Ongyanco, Eric Apolonio, Susan Yang and Eduardo Yotoko
are responsible and have in control/possession at No. 4 Epifanio de los Santos
corner Connecticut, Greenhills, San Juan, Metro Manila (per attached sketch and
list of MPAA member Company Titles) the following properties to wit:chanrob1es
virtual 1aw library

(a) Pirated video tapes of the copyrighted motion pictures/films the titles of which
are mentioned in the attached list;

(b) Posters, advertising leaflets, flyers, brochures, invoices, lists of titles being
reproduced or retaped, journals, ledgers, jon (sic) order slips, delivery slips and
books of accounts bearing and/or mentioning the pirated films with titles (as per
attached list), or otherwise used in the reproduction/repating business of the
defendants;

(c) Television sets, video cassette recorders, rewinders, tape head cleaners,
accessories, equipment and other machines and paraphernalia or materials used or
intended to be used in the unlawful sale, lease, distribution, or possession for
purpose of sale, lease, distribution, circulation or public exhibition of the above-
mentioned pirated video tapes which they are keeping and concealing in the
premises above-described, which should be seized and brought to the Undersigned.

You are hereby commanded to make an immediate search at any time in the day
between 8:00 A.M. to 5:00 P.M. of the premises above-described and forthwith
seize and take possession of the above-enumerated personal properties, and bring
said properties to the undersigned immediately upon implementation to be dealt
with as the law directs.

WITNESS MY HAND this 14th day of May, 1987, at Pasig, Metro Manila. (pp.
30-31, Rollo; Emphasis supplied.)

At or about high noon of the same day, agents from the NBI, led by Lauro C.
Reyes and Mamerto Espartero, with the assistance of the personnel of the
Videogram Regulatory Board headed by Elmer San Pascual, duly served Search
Warrant No. 45 on the operators or representatives of FGT. In the course of the
search of the premises of FGT, the NBI agents found and seized various video
tapes of duly copyrighted motion pictures or films owned and exclusively
distributed by petitioners. Also seized were machines and equipment, television
sets, paraphernalia, materials, accessories, rewinders, tape head cleaners,
statements of order, return slips, video prints, flyers, production orders, and
posters. Inventories of these seized articles were then prepared and copies thereof
were furnished Jess Ayson, production manager of FGT. On May 18, 1987, the
NBI agents filed a return of the search warrant with a motion to retain custody of
the seized items (p. 32, Rollo).

Meanwhile, FGT filed an urgent motion for the immediate release of equipment
and accessories "not covered" by the search warrant, without prejudice to the filing
of a motion to quash the said search warrant (p. 101, Rollo). It argued that as a
licensed video reproducer, FGT had the right to maintain possession of the seized
reproduction equipment and paraphernalia which are not contraband or illegal per
se, but are rather "exclusively used and intended to be used for reproduction" and
not in the "sale, lease, distribution or possession for purposes of sale, lease
distribution, circulation or public exhibition of pirated video tapes." (p. 102,
Rollo.)chanrobles law library

Petitioners opposed the motion, asserting that the seized articles were all lawfully
taken. They explained that since FGT was a videogram distributor and not a
reproducer, "it may be logically concluded that such 634 VCRs, accessories, etc."
were "used or intended to be used in the unlawful sale, lease, distribution or
possession for purposes of sale, lease, distribution, circulation or public exhibition
of, at the very least, the 310 videocassette tapes containing the copyrighted
films/motion pictures." They asserted that Search Warrant No. 45 was issued upon
the proper determination of probable cause and that, therefore, it is not for FGT "to
second-guess the wisdom" of the court’s directive to seize the questioned VCRs
and accessories "as an inquiry thereon would involve evidentiary matters which are
better ventilated in the criminal prosecution proper." (pp. 107-116, Rollo.)

Finding that FGT was a "registered and duly licensed distributor and in certain
instances and under special instructions and conditions . reproducer of videograms"
and that, therefore, its right to possess and use the seized equipment had been
"placed in serious doubt", the lower court resolved the doubt "against the
Government and in favor of a lawful business enterprise." Applying the
constitutional precept of presumption of innocence and considering that the seized
articles are not contraband, respondent court ruled that to allow the Government
"to keep possession of the equipment(s) and machines where there is no actual
criminal charge" would amount to a "confiscation in violation of the due process
clause of the constitution, notwithstanding the filing by the Director of the NBI of
a letter to the Department of Justice recommending that the defendants be charged
with violation of Section 56 of P.D. No. 49, as amended by P.D. No. 1988." (pp.
131-132, Rollo.)

Thus, in its order on May 29, 1987, the lower court granted FGT’s motion and
ordered the immediate release and return of the "television sets, video cassette
recorders, rewinders, tape head cleaners, accessories, equipment and other
machines or paraphernalias, as reflected in the ‘Receipt for Properties Seized’
attached to the records of the case beginning from page 84 to page 130, to the
defendants, excluding video cassette tapes reflected in the ‘Receipts for Properties
Seized’, beginning from page 132 to page 146 of the records." Respondent court
also ordered the inventory of all articles returned with individual descriptions "to
evidence their existence" copies of which inventory should be furnished the NBI
and the court (p. 132, Rollo).

Hence, the present recourse.

As prayed for by petitioners, on June 17, 1987, the Court issued a temporary
restraining order enjoining respondents from implementing the lower court’s order
of May 29, 1987 upon a bond in the amount of P750,000.00 which petitioners
accordingly posted on June 19, 1987. (pp. 138-141,
Rollo.)chanrobles.com:cralaw:red

The sole issue to be resolved is whether or not the lower court acted with grave
abuse of discretion amounting to lack of jurisdiction in ordering the immediate
release and return of some of the items seized by virtue of the search warrant.

Petitioners insist that the search warrant was issued upon due determination of
probable cause. They argue that FGT’s act of illegally reproducing copyrighted
films had been clearly established by evidence on record and that FGT’s principal
ground in praying for the immediate release of the seized articles is a matter of
defense which should be ventilated at the trial of the case on the merits.chanrobles
virtual lawlibrary

Private respondents, on the other hand, claim that the issuance of Search Warrant
No. 45 is tainted with illegality as no particular or specific acts or omissions
constituting the offense charged had been alleged in the application for its
issuance.

The right to security against unreasonable searches and seizures is guaranteed


under Section 2, Article III of the 1987 Constitution which provides:chanrob1es
virtual 1aw library

Sec. 2. The right of the people to be secure in their persons, houses, papers and
effects against unreasonable searches and seizures of whatever nature and for any
purpose shall be inviolable, and no search warrant or warrant of arrest shall issue
except upon probable cause to be determined by the judge after examination under
oath or affirmation of the complainant and the witnesses he may produce, and
particularly describing the place to be searched and the persons or things to be
seized.

Thus, Sections 3 and 4 of Rule 126 of the Rules of Court provide for the requisites
in the issuance of search warrants:chanrob1es virtual 1aw library

SEC. 3. Requisites for issuing search warrant. — A search warrant shall not issue
but upon probable cause in connection with one specific offense to be determined
personally by the judge after examination under oath or affirmation of the
complainant and the witnesses he may produce, and particularly describing the
place to be searched and the things to be seized.

SEC. 4. Examination of complainant; record. — The judge must, before issuing the
warrant, personally examine in the form of searching questions and answers, in
writing and under oath the complainant and the witnesses he may produce on facts
personally known to them and attach to the record their sworn statements together
with any affidavits submitted.

In issuing a search warrant, the judge must strictly comply with the constitutional
and statutory requirements. He must determine the existence of probable cause by
personally examining the applicant and his witnesses in the form of searching
questions (Silva v. Presiding Judge, RTC of Negros Oriental, Br. XXXIII (203
SCRA 140 [1991]). The search warrant must contain a specific description of the
place to be searched and the articles sought to be seized must be described with
particularity (Pendon v. Court of Appeals, 191 SCRA 429 [1990]).

Withal, measured by the foregoing constitutional and legal provisions as well as


the existing jurisprudence on the matter, we find that Search Warrant No. 45 fails
to satisfy the test of legality. More so because the Court has previously decided a
case dealing with virtually the same search warrant.

In 20th Century Fox Film Corp. v. Court of Appeals (164 SCRA 655 [1988]),
wherein therein petitioner is also one of the petitioners herein, we upheld the
legality of the order of the lower court lifting the search warrant issued under
circumstances similar to those obtaining in the case at bar.

A striking similarity between the case at bar and 20th Century Fox is the fact that
Search Warrant No. 45, specifically paragraph (c) thereof describing the articles to
be seized, contains an almost identical description as the warrant issued in the 20th
Century Fox case, to wit:chanrob1es virtual 1aw library

(c) Television sets, Video Cassettes Recorders, rewinders, tape head cleaners,
accessories, equipments and other machines used or intended to be used in the
unlawful reproduction, sale, rental/lease, distribution of the above-mentioned video
tapes which she is keeping and concealing in the premises above-described. (at p.
664.)

On the propriety of the seizure of the articles above-described, we held in said


case:chanrob1es virtual 1aw library

Television sets, video cassette recorders, rewinders and tape cleaners are articles
which can be found in a video tape store engaged in the legitimate business of
lending or renting out betamax tapes. In short, these articles and appliances are
generally connected with, or related to a legitimate business not necessarily
involving piracy of intellectual property or infringement of copyright laws. Hence,
including these articles without specification and/or particularity that they were
really instruments in violating an Anti-Piracy law makes the search warrant too
general which could result in the confiscation of all items found in any video store.
(at p. 665.)

The language used in paragraph (c) of Search Warrant No. 45 is thus too all-
embracing as to include all the paraphernalia of FGT in the operation of its
business. As the search warrant is in the nature of a general one, it is
constitutionally objectionable (Corro v. Lising, 137 SCRA 541 [1985]).

In consequence, respondent court was merely correcting its own erroneous


conclusions in issuing Search Warrant No. 45 when it ordered the return of the
seized television sets and other paraphernalia specified in the motion filed by FGT.
This can be gleaned from its statement that" .. the machines and equipment could
have been used or intended to be used in the illegal reproduction of tapes of the
copyrighted motion pictures/films, yet, it cannot be said with moral certainty that
the machines or equipment(s) were used in violating the law by the mere fact that
pirated video tapes of the copyrighted motion pictures/films were reproduced. As
already stated, FGT Video Network, Inc. is a registered and duly licensed
distributor and in certain instances and under special instructions .. reproducer of
videograms, and as such, it has the right to keep in its possession, maintain and
operate reproduction equipment(s) and paraphernalia(s)." (pp. 131-132, Rollo.)

Far from being despotic or arbitrary, respondent judge must be commended for
rectifying his error when he found that his initial conclusions were inaccurate and
erroneous, colliding as they did with the constitutional rights of
private Respondent.

Much has been said in the media about piracy of films and videotapes and that
violators of the law must be brought to the courts but, as the Court said in
Bagalihog v. Fernandez (198 SCRA 614 [1991])," [z]eal in the pursuit of criminals
cannot ennoble the use of arbitrary methods that the Constitution itself abhors." (at
p. 622.)

WHEREFORE, the petition is DISMISSED, the assailed order of May 29, 1987
AFFIRMED, and the temporary restraining order issued on June 18, 1987, vacated
and lifted.

Issue: WON the search warrant (Search Warrant No. 45) is invalid for lacking
particular or specific acts or omissions constituting the offense charged had been
alleged in the application for its issuance.

Ruling: Yes, the search warrant (Search Warrant No. 45) is invalid.

The search warrant must contain a specific description of the place to be


searched and the articles sought to be seized must be described with
particularity (Pendon vs. Court of Appeals, 191 SCRA 429 [1990]).

The Court has previously decided a case dealing with virtually the same search
warrant. Search Warrant No. 45, specifically paragraph (c) thereof describing the
articles to be seized, contains an almost identical description as the warrant issued
in 20th Century Fox Film Corp. vs. Court of Appeals. In the said case, the Court
held:
Television sets, video cassette recorders, rewinders and tape cleaners are
articles which can be found in a video tape store engaged in the legitimate
business of lending or renting out betamax tapes. In short, these articles and
appliances are generally connected with, or related to a legitimate
business not necessarily involving piracy of intellectual property or
infringement of copyright laws. Hence, including these articles without
specification and/or particularity that they were really instruments in
violating an Anti-Piracy law makes the search warrant too general which
could result in the confiscation of all items found in any video store.

The language used in paragraph (c) of Search Warrant No. 45 is thus too all-
embracing as to include all the paraphernalia of FGT in the operation of its
business. 

In consequence, respondent court was merely correcting its own erroneous


conclusions in issuing Search Warrant No. 45 when it ordered the return of the
seized television sets and other paraphernalia specified in the motion filed by FGT.

This can be gleaned from its statement that ".. the machines and equipment could
have been used or intended to be used in the illegal reproduction of tapes of the
copyrighted motion pictures/films, yet, it cannot be said with moral certainty that
the machines or equipment(s) were used in violating the law by the mere fact that
pirated video tapes of the copyrighted motion pictures/films were reproduced. As
already stated, FGT Video Network, Inc. is a registered and duly licensed
distributor and in certain instances and under special instructions .. reproducer
of videograms, and as such, it has the right to keep in its possession, maintain
and operate reproduction equipment(s) and paraphernalia(s)."

The SC held that Search Warrant No. 45 fails to satisfy the test of legality. Hence,
the petition was dismissed.

CASE: Prudente v. Dayrit, G.R. No. 82870 December 14, 1989

This is a petition for certiorari to annul and set aside the order of respondent Judge
dated 9 March 1988 which denied the petitioner's motion to quash Search Warrant
No. 87-14, as well as his order dated 20 April 1988 denying petitioner's motion for
reconsideration of the earlier order.

It appears that on 31 October 1987, P/Major Alladin Dimagmaliw, Chief of the


Intelligence Special Action Division (ISAD) of the Western Police District (WPD)
filed with the Regional Trial Court (RTC) of Manila, Branch 33, presided over by
respondent Judge Abelardo Dayrit, now Associate Justice of the Court of Appeals.
an application 1 for the issuance of a search warrant, docketed therein as SEARCH
WARRANT NO. 87-14, for VIOLATION OF PD NO. 1866 (Illegal Possession of
Firearms, etc.) entitled "People of the Philippines, Plaintiff, versus Nemesis E.
Prudente, Defendant." In his application for search warrant, P/Major Alladin
Dimagmaliw alleged, among others, as follows:

1. That he has been informed and has good and sufficient reasons to
believe that NEMESIO PRUDENTE who may be found at the
Polytechnic University of the Philippines, Anonas St. Sta. Mesa,
Sampaloc, Manila, has in his control or possession firearms,
explosives handgrenades and ammunition which are illegally
possessed or intended to be used as the means of committing an
offense which the said NEMESIO PRUDENTE is keeping and
concealing at the following premises of the Polytechnic University of
the Philippines, to wit:

a. Offices of the Department of Military Science and


Tactics at the ground floor and other rooms at the
ground floor;

b. Office of the President, Dr. Nemesio Prudente at


PUP, Second Floor and other rooms at the second floor;
2. That the undersigned has verified the report and found it to be a
fact, and therefore, believes that a Search Warrant should be issued to
enable the undersigned or any agent of the law to take possession and
bring to this Honorable Court the following described properties:

a. M 16 Armalites with ammunitions;

b. .38 and .45 Caliber handguns and pistols;

c. explosives and handgrenades; and,

d. assorted weapons with ammunitions.

In support of the application for issuance of search warrant, P/Lt. Florenio C.


Angeles, OIC of the Intelligence Section of (ISAD) executed a "Deposition of
Witness" dated 31 October 1987, subscribed and sworn to before respondent
Judge. In his deposition, P/Lt. Florenio Angeles declared, inter alia, as follows:

Q: Do you know P/Major Alladin Dimagmaliw, the


applicant for a Search Warrant?

A: Yes, sir, he is the Chief, Intelligence and Special


Action Division, Western Police District.

Q: Do you know the premises of Polytechnic University


of the Philippines at Anonas St., Sta. Mesa, Sampaloc,
Manila

A: Yes, sir, the said place has been the subject of our
surveillance and observation during the past few days.

Q: Do you have personal knowledge that in the said


premises is kept the following properties subject of the
offense of violation of PD No. 1866 or intended to be
used as a means of committing an offense:

a. M 16 Armalites with ammunitions;

b. .38 and 45 Caliber handguns and pistols;

c. explosives and handgrenades; and d. Assorted weapons with


ammunitions?

A: Yes sir.
Q: Do you know who is or who are the person or
persons who has or have control of the above-described
premises?

A: Yes sir, it is Dr. Nemesio Prudente, President of the


Polytechnic University of the Philippines.

Q: How do you know that said property is subject of the


offense of violation of Pres. Decree No. 1866 or
intended to be used as the means of committing an
offense?

A: Sir, as a result of our continuous surveillance


conducted for several days, we gathered information
from verified sources that the holder of said firearms
and explosives as well as ammunitions aren't licensed to
possess said firearms and ammunition. Further, the
premises is a school and the holders of these firearms
are not students who were not supposed to possess
firearms, explosives and ammunition.

On the same day, 31 October 1987, respondent Judge issued Search Warrant No.
87-14, 3 the pertinent portions of which read as follows:

It appearing to the satisfaction of the undersigned, after examining


under oath applicant ALLADIN M. DIMAGMALIW and his witness
FLORENIO C. ANGELES that there are good and sufficient reasons
to believe (probable cause) that NEMESIO PRUDENTE has in his
control in the premises of Polytechnic University of the Philippines,
Anonas St., Sta. Mesa, Sampaloc, Manila, properties which are
subject of the above offense or intended to be used as the means of
committing the said offense.

You are hereby commanded to make an immediate search at any time


in the day or night of the premises of Polytechnic University of the
Philippines, more particularly (a) offices of the Department of
Military Science and Tactics at the ground floor and other rooms at
the ground floor; (b) office of the President, Dr. Nemesio Prudente at
PUP, Second Floor and other rooms at the second floor, and forthwith
seize and take possession of the following personal properties, to wit:

a. M 16 Armalites with ammunition;

b. .38 and .45 Caliber handguns and pistols;


c. explosives and hand grenades; and

d. assorted weapons with ammunitions.

and bring the above described properties to the undersigned to be


dealt with as the law directs.

On 1 November 1987, a Sunday and All Saints Day, the search warrant was
enforced by some 200 WPD operatives led by P/Col. Edgar Dula Torre, Deputy
Superintendent, WPD, and P/Major Romeo Maganto, Precinct 8 Commander.

In his affidavit, 4 dated 2 November 1987, Ricardo Abando y Yusay, a member of


the searching team, alleged that he found in the drawer of a cabinet inside the wash
room of Dr. Prudente's office a bulging brown envelope with three (3) live
fragmentation hand grenades separately wrapped with old newspapers, classified
by P/Sgt. J.L. Cruz as follows (a) one (1) pc.—M33 Fragmentation hand grenade
(live); (b) one (11) pc.—M26 Fragmentation hand grenade (live); and (c) one (1)
pc.—PRB—423 Fragmentation hand grenade (live).

On 6 November 1987, petitioner moved to quash the search warrant. He claimed


that (1) the complainant's lone witness, Lt. Florenio C. Angeles, had no personal
knowledge of the facts which formed the basis for the issuance of the search
warrant; (2) the examination of the said witness was not in the form of searching
questions and answers; (3) the search warrant was a general warrant, for the reason
that it did not particularly describe the place to be searched and that it failed to
charge one specific offense; and (4) the search warrant was issued in violation of
Circular No. 19 of the Supreme Court in that the complainant failed to allege under
oath that the issuance of the search warrant on a Saturday was urgent. 5

The applicant, P/Major Alladin Dimagmaliw thru the Chief, Inspectorate and Legal
Affairs Division, WPD, opposed the motion. 6 After petitioner had filed his
reply 7 to the opposition, he filed a supplemental motion to quash. 8

Thereafter, on 9 March 1988, respondent Judge issued an order, 9denying the


petitioner's motion and supplemental motion to quash. Petitioner's motion for
reconsideration 10 was likewise denied in the order 11 dated 20 April 1988.

Hence, the present recourse, petitioner alleging that respondent Judge has decided
a question of substance in a manner not in accord with law or applicable decisions
of the Supreme Court, or that the respondent Judge gravely abused his discretion
tantamount to excess of jurisdiction, in issuing the disputed orders.

For a valid search warrant to issue, there must be probable cause, which is to be
determined personally by the judge, after examination under oath or affirmation of
the complainant and the witnesses he may produce, and particularly describing the
place to be searched and the persons or things to be seized. 12 The probable cause
must be in connection with one specific offense 13 and the judge must, before
issuing the warrant, personally examine in the form of searching questions and
answers, in writing and under oath, the complainant and any witness he may
produce, on facts personally known to them and attach to the record their sworn
statements together with any affidavits submitted. 14

The "probable cause" for a valid search warrant, has been defined "as such facts
and circumstances which would lead a reasonably discreet arid prudent man to
believe that an offense has been committed, and that objects sought in connection
with the offense are in the place sought to be searched." 15 This probable cause
must be shown to be within the personal knowledge of the complainant or the
witnesses he may produce and not based on mere hearsay. 16

Petitioner assails the validity of Search Warrant No. 87-14 on the ground that it
was issued on the basis of facts and circumstances which were not within the
personal knowledge of the applicant and his witness but based on hearsay
evidence. In his application for search warrant, P/Major Alladin Dimagmaliw
stated that "he has been informed" that Nemesio Prudente "has in his control and
possession" the firearms and explosives described therein, and that he "has verified
the report and found it to be a fact." On the other hand, in his supporting
deposition, P/Lt. Florenio C. Angeles declared that, as a result of their continuous
surveillance for several days, they "gathered informations from verified sources"
that the holders of the said fire arms and explosives are not licensed to possess
them. In other words, the applicant and his witness had no personal knowledge of
the facts and circumstances which became the basis for issuing the questioned
search warrant, but acquired knowledge thereof only through information from
other sources or persons.

While it is true that in his application for search warrant, applicant P/Major
Dimagmaliw stated that he verified the information he had earlier received that
petitioner had in his possession and custody the t there is nothing in the record to
show or indicate how and when said applicant verified the earlier information
acquired by him as to justify his conclusion that he found such information to be a
fact. He might have clarified this point if there had been searching questions and
answers, but there were none. In fact, the records yield no questions and answers,
whether searching or not, vis-a-vis the said applicant.

What the records show is the deposition of witness, P/Lt. Angeles, as the only
support to P/Major Dimagmaliw's application, and the said deposition is based on
hearsay. For, it avers that they (presumably, the police authorities) had conducted
continuous surveillance for several days of the suspected premises and, as a result
thereof, they "gathered information from verified sources" that the holders of the
subject firearms and explosives are not licensed to possess them.
In Alvarez vs. Court of First Instance, 17 this Court laid the following test in
determining whether the allegations in an application for search warrant or in a
supporting deposition, are based on personal knowledge or not—

The true test of sufficiency of a deposition or affidavit to warrant


issuance of a search warrant is whether it has been drawn in a manner
that perjury could be charged thereon and the affiant be held liable for
damage caused. The oath required must refer to the truth of the facts
within the personal knowledge of the applicant for search warrant,
and/or his witnesses, not of the facts merely reported by a person
whom one considers to be reliable.

Tested by the above standard, the allegations of the witness, P/Lt. Angeles, in his
deposition, do not come up to the level of facts of his personal knowledge so much
so that he cannot be held liable for perjury for such allegations in causing the
issuance of the questioned search warrant.

In the same Alvarez case, 18 the applicant stated that his purpose for applying for a
search warrant was that: "It had been reported to me by a person whom I consider
to be reliable that there are being kept in said premises books, documents, receipts,
lists, chits and other papers used by him in connection with his activities as a
money lender, challenging usurious rate of interests, in violation of law." The
Court held that this was insufficient for the purpose of issuing a search warrant.

In People vs. Sy Juco, 19 where the affidavit contained an allegation that there had
been a report to the affiant by a person whom lie considered reliable that in said
premises were "fraudulent books, correspondence and records," this was likewise
held as not sufficient for the purpose of issuing a search warrant. Evidently, the
allegations contained in the application of P/ Major Alladin Dimagmaliw and the
declaration of P/Lt. Florenio C. Angeles in his deposition were insufficient basis
for the issuance of a valid search warrant. As held in the Alvarez case:

The oath required must refer to the truth of the facts within the
personal knowledge of the petitioner or his witnesses, because the
purpose thereof is to convince the committing magistrate, not the
individual making the affidavit and seeking the issuance of the
warrant, of the existence of probable cause.

Besides, respondent Judge did not take the deposition of the applicant as required
by the Rules of Court. As held in Roan v. Gonzales, 20 "(m)ere affidavits of the
complainant and his witnesses are thus not sufficient. The examining Judge has to
take depositions in writing of the complainant and the witnesses he may produce
and attach them to the record."
Moreover, a perusal of the deposition of P/Lt. Florenio Angeles shows that it was
too brief and short. Respondent Judge did not examine him "in the form of
searching questions and answers." On the contrary, the questions asked were
leading as they called for a simple "yes" or "no" answer. As held in Quintero vs.
NBI," 21 the questions propounded by respondent Executive Judge to the applicant's
witness are not sufficiently searching to establish probable cause. Asking of
leading questions to the deponent in an application for search warrant, and
conducting of examination in a general manner, would not satisfy the requirements
for issuance of a valid search warrant."

Manifestly, in the case at bar, the evidence failed to show the existence of probable
cause to justify the issuance of the search warrant. The Court also notes post
facto that the search in question yielded, no armalites, handguns, pistols, assorted
weapons or ammunitions as stated in the application for search warrant, the
supporting deposition, and the search warrant the supporting hand grenades were
itself Only three (3) live fragmentation found in the searched premises of the PUP,
according to the affidavit of an alleged member of the searching party.

The Court avails of this decision to reiterate the strict requirements for
determination of "probable cause" in the valid issuance of a search warrant, as
enunciated in earlier cases. True, these requirements are stringent but the purpose
is to assure that the constitutional right of the individual against unreasonable
search and seizure shall remain both meaningful and effective.

Petitioner also assails the validity of the search warrant on the ground that it failed
to particularly describe the place to be searched, contending that there were several
rooms at the ground floor and the second floor of the PUP.

The rule is, that a description of a place to be searched is sufficient if the officer
with the warrant can, with reasonable effort, ascertain and Identify the place
intended .22 In the case at bar, the application for search warrant and the search
warrant itself described the place to be searched as the premises of the Polytechnic
University of the Philippines, located at Anonas St., Sta. Mesa, Sampaloc, Manila
more particularly, the offices of the Department of Military Science and Tactics at
the ground floor, and the Office of the President, Dr. Nemesio Prudente, at PUP,
Second Floor and other rooms at the second floor. The designation of the places to
be searched sufficiently complied with the constitutional injunction that a search
warrant must particularly describe the place to be searched, even if there were
several rooms at the ground floor and second floor of the PUP.

Petitioner next attacks the validity of the questioned warrant, on the ground that it
was issued in violation of the rule that a search warrant can be issued only in
connection with one specific offense. The search warrant issued by respondent
judge, according to petitioner, was issued without any reference to any particular
provision of PD No. 1866 that was violated when allegedly P.D. No. 1866
punishes several offenses.

In Stonehill vs. Diokno, 23 Where the warrants involved were issued upon


applications stating that the natural and juridical persons therein named had
committed a "violation of Central Bank Laws, Tariff and Customs Laws, Internal
Revenue Code and Revised Penal Code," the Court held that no specific offense
had been alleged in the applications for a search warrant, and that it would be a
legal hearsay of the highest order to convict anybody of a "Violation of Central
Bank Laws, Tariff and Customs Laws, Internal Revenue Code and Revised Penal
Code" without reference to any determinate provision of said laws and codes.

In the present case, however, the application for search warrant was captioned:
"For Violation of PD No. 1866 (Illegal Possession of Firearms, etc.) While the said
decree punishes several offenses, the alleged violation in this case was, qualified
by the phrase "illegal possession of firearms, etc." As explained by respondent
Judge, the term "etc." referred to ammunitions and explosives. In other words, the
search warrant was issued for the specific offense of illegal possession of firearms
and explosives. Hence, the failure of the search warrant to mention the particular
provision of PD No. 1-866 that was violated is not of such a gravity as to call for
its invalidation on this score. Besides, while illegal possession of firearms is
penalized under Section 1 of PD No. 1866 and illegal possession of explosives is
penalized under Section 3 thereof, it cannot be overlooked that said decree is a
codification of the various laws on illegal possession of firearms, ammunitions and
explosives; such illegal possession of items destructive of life and property are
related offenses or belong to the same species, as to be subsumed within the
category of illegal possession of firearms, etc. under P.D. No. 1866. As observed
by respondent Judge: 24

The grammatical syntax of the phraseology comparative with the title


of PD 1866 can only mean that illegal possession of firearms,
ammunitions and explosives, have been codified under Section 1 of
said Presidential Decree so much so that the second and third are
forthrightly species of illegal possession of firearms under Section (1)
thereof It has long been a practice in the investigative and prosecution
arm of the government, to designate the crime of illegal possession of
firearms, ammunitions and explosives as 'illegal possession of
firearms, etc.' The Constitution as well as the Rules of Criminal
Procedure does not recognize the issuance of one search warrant for
illegal possession of firearms, one warrant for illegal possession of
ammunitions, and another for illegal possession of explosives.
Neither is the filing of three different informations for each of the
above offenses sanctioned by the Rules of Court. The usual practice
adopted by the courts is to file a single information for illegal
possession of firearms and ammunitions. This practice is considered
to be in accordance with Section 13, Rule 110 of the 1985 Rules on
Criminal Procedure which provides that: 'A complaint or information
must charge but one offense, except only in those cases in which
existing laws prescribe a single punishment for various
offenses. Describably, the servers did not search for articles other than
firearms, ammunitions and explosives. The issuance of Search
Warrant No. 87-14 is deemed profoundly consistent with said rule
and is therefore valid and enforceable. (Emphasis supplied)

Finally, in connection with the petitioner's contention that the failure of the
applicant to state, under oath, the urgent need for the issuance of the search
warrant, his application having been filed on a Saturday, rendered the questioned
warrant invalid for being violative of this Court's Circular No. 19, dated 14 August
1987, which reads:

3. Applications filed after office hours, during Saturdays, Sundays


and holidays shall likewise be taken cognizance of and acted upon by
any judge of the court having jurisdiction of the place to be searched,
but in such cases the applicant shall certify and state the facts under
oath, to the satisfaction of the judge, that the issuance is urgent.

it would suffice to state that the above section of the circular merely provides for a
guideline, departure from which would not necessarily affect the validity of an
otherwise valid search warrant.

WHEREFORE, all the foregoing considered, the petition is GRANTED. The


questioned orders dated 9 March 1988 and 20 April 1988 as well as Search
Warrant No. 87-14 are hereby ANNULLED and SET ASIDE.

The three (3) live fragmentation hand grenades which, according to Ricardo Y.
Abando, a member of the searching team, were seized in the washroom of
petitioner's office at the PUP, are ordered delivered to the Chief, Philippine
Constabulary for proper disposition.

SO ORDERED.

Issue Related to the Topic: Whether the warrant was issued in violation of the
rule that a search warrant can be issued only in connection with one specific
offense, given that it  was issued without any reference to any particular
provision of PD No. 1866 that was violated when allegedly P.D. No. 1866
punishes several offenses.

Ruling: No, the failure of the search warrant to mention the particular provision
of PD No. 1-866 that was violated is not of such a gravity as to call for its
invalidation on this score.
Explanation by the Supreme Court:

In Stonehill vs. Diokno, Where the warrants involved were issued upon
applications stating that the natural and juridical persons therein named had
committed a "violation of Central Bank Laws, Tariff and Customs Laws, Internal
Revenue Code and Revised Penal Code," the Court held that no specific offense
had been alleged in the applications for a search warrant, and that it would be a
legal hearsay of the highest order to convict anybody of a "Violation of Central
Bank Laws, Tariff and Customs Laws, Internal Revenue Code and Revised Penal
Code" without reference to any determinate provision of said laws and codes.

In this case, however, the application for search warrant was captioned: "For
Violation of PD No. 1866 (Illegal Possession of Firearms, etc.) While the said
decree punishes several offenses, the alleged violation in this case was qualified by
the phrase "illegal possession of firearms, etc." As explained by the respondent
Judge, the term "etc." referred to ammunition and explosives. In other words, the
search warrant was issued for the specific offense of illegal possession of
firearms and explosives. Hence, the failure of the search warrant to mention
the particular provision of PD No. 1-866 that was violated is not of such a
gravity as to call for its invalidation on this score. Besides, while illegal
possession of firearms is penalized under Section 1 of PD No. 1866 and illegal
possession of explosives is penalized under Section 3 thereof, it cannot be
overlooked that said decree is a codification of the various laws on illegal
possession of firearms, ammunitions and explosives; such illegal possession of
items destructive of life and property are related offenses or belong to the same
species, as to be subsumed within the category of illegal possession of firearms,
etc. under P.D. No. 1866. As observed by respondent Judge: 

The grammatical syntax of the phraseology comparative with the title


of PD 1866 can only mean that illegal possession of firearms,
ammunitions and explosives, have been codified under Section 1 of
said Presidential Decree so much so that the second and third are
forthrightly species of illegal possession of firearms under Section (1)
thereof It has long been a practice in the investigative and prosecution
arm of the government, to designate the crime of illegal possession of
firearms, ammunitions and explosives as 'illegal possession of
firearms, etc.' The Constitution as well as the Rules of Criminal
Procedure does not recognize the issuance of one search warrant for
illegal possession of firearms, one warrant for illegal possession of
ammunitions, and another for illegal possession of explosives.
Neither is the filing of three different informations for each of the
above offenses sanctioned by the Rules of Court. The usual practice
adopted by the courts is to file a single information for illegal
possession of firearms and ammunitions. This practice is considered
to be in accordance with Section 13, Rule 110 of the 1985 Rules on
Criminal Procedure which provides that: 'A complaint or information
must charge but one offense, except only in those cases in which
existing laws prescribe a single punishment for various offenses.
Describably, the servers did not search for articles other than
firearms, ammunitions and explosives. The issuance of Search
Warrant No. 87-14 is deemed profoundly consistent with said rule
and is therefore valid and enforceable. (Emphasis supplied)

CASE: People v. Salanguit, G.R. No. 133254-55, April 19, 2001

This is an appeal from the decision,1 dated January 27, 1998, of the Regional Trial
Court, Branch 96, Quezon City, finding accused-appellant Roberto Salanguit y Ko
guilty of violation of §16 of Republic Act No. 6425, as amended, and sentencing
him accordingly to suffer imprisonment ranging from six (6) months of arresto
mayor, as minimum, to four (4) years and two (2) months of prision
correccional, as maximum, and of §8 of the same law and sentencing him for such
violation to suffer the penalty of reclusion perpetua and to pay a fine of P700,
000.00.

Charges against accused-appellant for violations of R.A. No. 6425 were filed on
December 28, 1995. In Criminal Case No. Q-95-64357, the information alleged:

That on or about the 26th day of December 1995, in Quezon City,


Philippines, the said accused, did then and there willfully, unlawfully and
knowingly possess and/or use 11.14 grams of Methamphetamine
Hydrochloride (Shabu) a regulated drug, without the necessary license
and/or prescription therefor, in violation of said law.

CONTRARY TO LAW .2

In Criminal Case No. Q-95-64358, the information charged:

That on or about the 26th day of December 1995, in Quezon City,


Philippines, the said accused not being authorized by law to possess or use
any prohibited drug, did, then and there willfully, unlawfully and knowingly
have in his possession and under his custody and control 1,254 grams of
Marijuana, a prohibited drug.

CONTRARY TO LAW ,3

When arraigned on May 21, 1996, accused-appellant pleaded not


guilty4 whereupon he was tried.
Three witnesses were presented by the prosecution: P/Insp. Sonia S. Ludovico,
forensic chemist and chief of the Physical Science Branch of the Philippine
National Police Crime Laboratory, Senior Inspector Rodolfo Aguilar of the
Narcotics Command, Camp Crame, Quezon City, and PO3 Rolando Duazo of
Station 10, Kamuning, Quezon City, a field operative. The prosecution evidence
established the following:

On December 26, 1995, Sr. Insp. Aguilar applied for a warrant 5 in the Regional
Trial Court, Branch 90, Dasmariñias, Cavite, to search the residence of accused-
appellant Robert Salanguit y Ko on Binhagan St., Novaliches, Quezon City. He
presented as his witness SPO1 Edmund Badua, who testified that as a poseur-
buyer, he was able to purchase 2.12 grams of shabu from accused-appellant. The
sale took place in accused-appellant's room, and Badua saw that the shabu was
taken by accused-appellant from a cabinet inside his room. The application was
granted, and a search warrant was later issued by Presiding Judge Dolores L.
Español.

At about 10:30 p.m. of December 26, 1995, a group of about 10 policemen, along
with one civilian informer, went to the residence of accused-appellant to serve the
warrant.6

The police operatives knocked on accused-appellant’s door, but nobody opened it.
They heard people inside the house, apparently panicking. The police operatives
then forced the door open and entered the house. 7

After showing the search warrant to the occupants of the house, Lt. Cortes and his
group started searching the house.8 They found 12 small heat-sealed transparent
plastic bags containing a white crystalline substance, a paper clip box also
containing a white crystalline substance, and two bricks of dried leaves which
appeared to be marijuana wrapped in newsprint 9 having a total weight of
approximately 1,255 grams.10 A receipt of the items seized was prepared, but the
accused-appellant refused to sign it. 11

After the search, the police operatives took accused-appellant with them to Station
10, EDSA, Kamuning, Quezon City, along with the items they had seized. 12

PO3 Duazo requested a laboratory examination of the confiscated evidence. 13 The


white crystalline substance with a total weight of 2.77 grams and those contained
in a small box with a total weight of 8.37 grams were found to be positive for
methamphetamine hydrochloride. On the other hand, the two bricks of dried
leaves, one weighing 425 grams and the other 850 grams, were found to be
marijuana.14

For the defense, accused-appellant testified in his own behalf. His testimony was
corroborated by his mother-in-law, Soledad Arcano.
Accused-appellant testified that on the night of December 26, 1995, as they were
about to leave their house, they heard a commotion at the gate and on the roof of
their house. Suddenly, about 20 men in civilian attire, brandishing long firearms,
climbed over the gate and descended through an opening in the roof. 15

When accused-appellant demanded to be shown a search warrant, a piece of paper


inside a folder was waved in front of him. As accused-appellant fumbled for his
glasses, however, the paper was withdrawn and he had no chance to read it. 16

Accused-appellant claimed that he was ordered to stay in one place of the house
while the policemen conducted a search, forcibly opening cabinets and taking his
bag containing money, a licensed .45 caliber firearm, jewelry , and canned goods. 17

The policemen left at around 12:30 a.m. of December 27, 1995, and, after putting
handcuffs on accused-appellant, took him with them to the NARCOM on EDSA,
Quezon City, where accused-appellant was detained. 18

Accused-appellant's mother-in law, Soledad Arcano, corroborated his testimony.


Arcano testified that the policemen ransacked their house, ate their food, and took
away canned goods and other valuables. 19

After hearing, the trial court rendered its decision, the dispositive portion of which
reads:

WHEREFORE, judgment is hereby rendered:

1. In Criminal Case No. Q-95-64357, for violation of Sec. 16, Republic Act
No. 6425, as amended, finding the accused ROBERTO SALANGUIT y KO
guilty beyond reasonable doubt of the crime charged and he is hereby
accordingly sentenced to suffer an indeterminate sentence with a minimum
of six (6) months of arresto mayor and a maximum of four (4) years and
two (2) months of prision correccional; and,

2. In Criminal Case No. Q-95-64358, for violation of Sec. 8, Republic Act


No. 6425, as amended, finding the accused ROBERTO SALANGUIT y KO
guilty beyond reasonable doubt of the crime charged and he is hereby
accordingly sentenced to suffer reclusion perpetua and to pay a fine of
P700,000.00.

The accused shall further pay the costs of suit.

The 11.14 grams of methamphetamine hydrochloride and the 1,254 grams


of marijuana bricks are hereby confiscated and condemned for disposition
according to law. The evidence custodian of this Court is hereby directed to
turn such substances over to the National Bureau of Investigation pursuant
to law.

SO ORDERED.20

Hence this appeal. Accused-appellant contends that -

THE COURT A QUO GRAVELY ERRED IN DECLARING THE


SEARCH WARRANT VALID

THE COURT A QUO ERRED IN CONVICTING ACCUSED-


APPELLANT FOR ILLEGAL POSSESSION OF METHAMPHETAMINE
HYDRO-CHLORIDE(SHABU)

THE COURT A QUO GRAVELY ERRED IN CONVICTING


ACCUSSED-APPELLANT FOR VIOLATION §8, R.A. No. 6425

THE COURT A QUO ERRED IN ADMITTING IN EVIDENCE THE


TWO (2) BRICKS OF MARIJUANA

THE COURT A QUO ERRED IN NOT FINDING THAT THE


POLICEMEN USED EXCESSIVE FORCE IN ENFORCING THE
SEARCH WARRANT.

Accused-appellant is contesting his conviction on three grounds. First, the


admissibility of the shabu allegedly recovered from his residence as evidence
against him on the ground that the warrant used in obtaining it was invalid.
Second, the admissibility in evidence of the marijuana allegedly seized from
accused-appellant pursuant to the "plain view" doctrine. Third, the employment of
unnecessary force by the police in the execution of the warrant.

First. Rule 126, §4 of the Revised Rules on Criminal Procedure 21 provides that a


search warrant shall not issue except upon probable cause in connection with one
specific offense to be determined personally by the judge after examination under
oath or affirmation of the complainant and the witnesses he may produce, and
particularly describing the place to be searched and the things to be seized which
may be anywhere in the Philippines.

In issuing a search warrant, judges must comply strictly with the requirements of
the Constitution and the Rules of Criminal Procedure. No presumption of
regularity can be invoked in aid of the process when an officer undertakes to justify
its issuance.22 Nothing can justify the issuance of the search warrant unless all the
legal requisites are fulfilled.

In this case, the search warrant issued against accused-appellant reads:


SE
AR
CH
W
AR
RA
NT
N
O.
16
0
For
:
Vi
ola
tio
n
of
RA
64
25

SEARCH WARRANT

TO ANY PEACE OFFICER:

G R E E T I N G S:

It appearing to the satisfaction of the undersigned after examining under


oath SR. INSP. RODOLFO V. AGUILAR, PNP and his witness SPO1
EDMUND M. BADUA, PNP that there is probable cause to believe that
ROBERT SALANGUIT has in his possession and control in his premises
Binhagan St., San Jose, Quezon City as shown in Annex "A", the properties
to wit:

UNDETERMINED QUANTITY OF SHABU AND DRUG


PARAPHERNALIA

which should be seized and brought to the undersigned.

You are hereby commanded to make an immediate search anytime of the


day/night of the premises above-described and forthwith seize and take
possession of the above-stated properties and bring said properties to the
undersigned to be dealt with as the law directs.
GIVEN UNDER MY HAND this 26th day of December 1995 at Imus,
Cavite, Philippines.

(S
G
D.)
D
OL
OR
ES
L.
ES
PA
Ñ
OL

        Judg
e

Accused-appellant assails the validity of the warrant on three grounds: (1) that
there was no probable cause to search for drug paraphernalia; (2) that the search
warrant was issued for more than one specific offense; and (3) that the place to be
searched was not described with sufficient particularity.

Existence of Probable Cause

The warrant authorized the seizure of "undetermined quantity of shabu and drug
paraphernalia." Evidence was presented showing probable cause of the existence of
methamphetamine hydrochloride or shabu. Accused-appellant contends, however,
that the search warrant issued is void because no evidence was presented showing
the existence of drug paraphernalia and the same should not have been ordered to
be seized by the trial court.23

The contention has no merit. To be sure, SPO1 Edmund Badua, the intelligence
officer who acted as a poseur-buyer, did not testify in the proceedings for the
issuance of a search warrant on anything about drug paraphernalia. He stated:

Q -Being a member of the Intelligence and Operation Section, NMDU,


NARCOM, do you remember if you were assigned into a monitoring or
surveillance work?

A -Yes, sir.

Q – Of what particular assignment or area were you assigned for monitoring


or surveillance?
A – Its within the Quezon City area particularly a house without a number
located at Binhagan St., San Jose Quezon City, Sir.

Q – Do You know the person who occupies the specific place?

A – Yes, sir, he is ROBERT SALANGUIT @ Robert.

Q – Are you familiar with that place?

A – Yes, sir, as part of my surveillance, I was able to penetrate inside the


area and established contract with ROBERT SALANGUIT alias Robert
through my friend who introduced me to the former.

Q – In what particular occasion did you meet ROBERT SALANGUIT alias


Robert?

A – When I was introduced by my friend as a good buyer and drug pusher


of shabu, sir .

Q – Were you able to buy at that time?

A – Yes, sir.

Q – How much if you can still remember the amount involved?

A – I was able to buy two point twelve (2.12) grams of shabu in the amount
of Two Thousand Seven Hundred Fifty (P2,750.00) pesos, sir .

Q – Having established contact with ROBERT SALANGUIT @ Robert, do


you know where the stuff (shabu) were being kept?

A – Yes, sir, inside a cabinet inside his room.

Q – How were you able to know the place where he kept the stuff?

A – When I first bought the 2.12 grams of shabu from him, it was done
inside his room and I saw that the shabu was taken by him inside his
cabinet.

Q – Do you know who is in control of the premises?

A – Yes, sir, it was ROBERT SALANGUIT @ Robert.

Q – How sure are you, that the shabu that you bought from ROBERT
SALANGUIT @ Robert is genuine shabu?
A – After I left the house of ROBERT SALANGUIT @ Robert, I proceeded
back to our office and reported the progress of my mission to our Chief and
presented to him the 2.12, grams of shabu I bought from the subject. Then
afterwards, our Chief formally requested the Chief PNP Central Crime
Laboratory Services, NPDC, for Technical Analysis which yielded positive
result for shabu, a regulated drug as shown in the attached certification of
PNP CLS result No. D-414-95 dated 19 December 95.

Q – Do you have anything more to add or retract from your statement?

A – Yes, sir, I was offered by him (ROBERT SALANGUIT @ Robert) that


anything I wish to buy bigger quantity of shabu, he is willing to transact to
me on cash basis at his price of One Thousand Seven Hundred Fifty
(P1,750.00) pesos per gram.

Q – Are you willing to sign your statement freely and voluntarily?

A – Yes, sir.24

However, the fact that there was no probable cause to support the application for
the seizure of drug paraphernalia does not warrant the conclusion that the search
warrant is void. This fact would be material only if drug paraphernalia was in fact
seized by the police. The fact is that none was taken by virtue of the search warrant
issued. If at all, therefore, the search warrant is void only insofar as it authorized
the seizure of drug paraphernalia, but it is valid as to the seizure of
methamphetamine hydrochloride as to which evidence was presented showing
probable cause as to its existence. Thus, in Aday v. Superior Court,25  the warrant
properly described two obscene books but improperly described other articles. It
was held:

Although the warrant was defective in the respects noted, it does not follow
that it was invalid as a whole. Such a conclusion would mean that the
seizure of certain articles, even though proper if viewed separately, must be
condemned merely because the warrant was defective with respect to other
articles. The invalid portions of the warrant are severable from the
authorization relating to the named books, which formed the principal basis
of the charge of obscenity. The search for and seizure of these books, if
otherwise valid, were not rendered illegal by the defects concerning other
articles. ...In so holding we do not mean to suggest that invalid portions "of
a warrant will be treated as severable under all circumstances. We recognize
the danger that warrants might be obtained which are essentially general in
character but as to minor items meet the requirement of particularity, and
that wholesale seizures might be made under them, in the expectation that
the seizure would in any event be upheld as to the property specified. Such
an abuse of the warrant procedure, of course, could not be tolerated.
It would be a drastic remedy indeed if a warrant, which was issued on probable
cause and particularly describing the items to be seized on the basis thereof, is to
be invalidated in toto because the judge erred in authorizing a search for other
items not supported by the evidence.26 Accordingly, we hold that the first part of
the search warrant, authorizing the search of accused-appellant's house for an
undetermined quantity of shabu, is valid, even though the second part, with respect
to the search for drug paraphernalia, is not.

Specificity of the Offense Charged

Accused-appellant contends that the warrant was issued for more than one specific
offense because possession or use of methamphetamine hydrochloride and
possession of drug paraphernalia are punished under two different provisions of
R.A. No. 6425.27 It will suffice to quote what this Court said in a similar case to
dispose of this contention:

While it is true that the caption of the search warrant states that it is in
connection with "Violation of R.A. 6425, otherwise known as the
Dangerous Drugs Act of 1972," it is clearly recited in the text thereof that
"There is probable cause to believe that Adolfo Olaes alias 'Debie' and alias
'Baby' of No.628 Comia St., Filtration, Sta. Rita, Olongapo City, has in their
session and control and custody of marijuana dried stalks/leaves/
seeds/cigarettes and other regulated/prohibited and exempt narcotics
preparations which is the subject of the offense stated above." Although the
specific section of the Dangerous Drugs Act is not pinpointed, there is no
question at all of the specific offense alleged to have been committed as a
basis for the finding of probable cause. The search warrant also satisfies the
requirement in the Bill of Rights of the particularity of the description to be
made of the "place to be searched and the persons or things to be seized." 28

Indeed, in People v. Dichoso29 the search warrant was also for "Violation of R.A.
6425," without specifying what provisions of the law were violated, and it
authorized the search and seizure of "dried marijuana leaves and methamphetamine
hydrochloride (shabu) and sets of paraphernalias (sic)." This Court, however,
upheld the validity of the warrant:

Appellant's contention that the search warrant in question was issued for
more than (1) offense, hence, in violation of Section 3, Rule 126 of the
Rules of Court, is unpersuasive. He engages in semantic juggling by
suggesting that since illegal possession of shabu, illegal possession of
marijuana and illegal possession of paraphernalia are covered by different
articles and sections of the Dangerous Drugs Act of 1972, the search warrant
is clearly for more than one (1) specific offense. In short, following this
theory, there should have been three (3) separate search warrants, one for
illegal possession of shabu, the second for illegal possession of marijuana
and the third for illegal possession of paraphernalia. This argument is
pedantic. The Dangerous Drugs Act of 1972 is a special law that deals
specifically with dangerous drugs which are subsumed into "prohibited" and
"regulated" drugs and defines and penalizes categories of offenses which are
closely related or which belong to the same class or species. Accordingly,
one (1) search warrant may thus be validly issued for the said violations of
the Dangerous Drugs Act. 30

Similarly, in another case,31 the search warrant was captioned: "For Violation of


P .D. No.1866 (Illegal Possession of Firearms, etc.)." The validity of the warrant
was questioned on the ground that it was issued without reference to any particular
provision in P.D. No.1866, which punished several offenses. We held, however,
that while illegal possession of firearms is penalized under §1 of P.D. No.1866 and
illegal possession of explosives is penalized under §3 thereof, the decree is a
codification of the various laws on illegal possession of firearms, ammunitions,
and explosives which offenses are so related as to be subsumed within the category
of illegal possession of firearms, etc. under P.D. No.1866. Thus, only one warrant
was necessary to cover the violations under the various provisions of the said law.

Particularly of the Place

Accused-appellant contends that the search warrant failed to indicate the place to
be searched with sufficient particularity.

This contention is without merit. As the Solicitor General states:

.....While the address stated in the warrant is merely "Binhagan St., San
Jose, Quezon City," the trial court took note of the fact that the records of
Search Warrant Case No.160 contained several documents which identified
the premises to be searched, to wit: 1) the application for search warrant
which stated that the premises to be searched was located in between No.7
and 11 at Binhagan Street, San Jose, Quezon City; 2) the deposition of
witness which described the premises as "a house without a number located
at Binhagan St., San Jose, Quezon City; and 3) the pencil sketch of the
location of the premises to be searched. In fact, the police officers who
raided appellant's house under the leadership of Police Senior Inspector
Rodolfo Aguilar could not have been mistaken as Inspector Aguilar resides
in the same neighborhood in Binhagan where appellant lives and in fact
Aguilar's place is at the end of appellant's place in Binhagan. Moreover, the
house raided by Aguilar's team is undeniably appellant'.s house and it was
really appellant who was the target. The raiding team even first ascertained
through their informant that appellant was inside his residence before they
actually started their operation.32
The rule is that a description of the place to be searched is sufficient if the officer
with the warrant can, with reasonable effort, ascertain and identify the place
intended to be searched.33 For example, a search warrant authorized a search of
Apartment Number 3 of a building at 83 Pleasant Street, Malborough,
Massachusetts. As it turned out, there were five apartments in the basement and six
apartments on both the ground and top floors and that there was an Apartment
Number 3 on each floor. However, the description was made determinate by a
reference to the affidavit supporting the warrant that the apartment was occupied
by the accused "Morris Ferrante of 83 Pleasant Street, Malboro Mass." 34 In this
case, the location of accused-appellant's house being indicated by the evidence on
record, there can be no doubt that the warrant described the place to be searched
with sufficient particularity.1âwphi1.nêt

In sum, we hold that with respect to the seizure of shabu from accused-appellant's


residence, Search Warrant No.160 was properly issued, such warrant being
founded on probable cause personally determined by the judge under oath or
affirmation of the deposing witness and particularly describing the place to be
searched and the things to be seized.

Second. The search warrant authorized the seizure of methamphetamine


hydrochloride or shabu but not marijuana. However, seizure of the latter drug is
being justified on the ground that the drug was seized within the "plain view" of
the searching party. This is contested by accused-appellant.

Under the "plain view doctrine," unlawful objects within the "plain view" of an
officer who has the right to be in the position to have that view are subject to
seizure and may be presented in evidence.35 For this doctrine to apply, there must
be: (a) prior justification; (b ) inadvertent discovery of the evidence; and (c)
immediate apparent illegality of the evidence before the police. 36 The question is
whether these requisites were complied with by the authorities in seizing the
marijuana in this case.

Prior Justification and Discovery by Inadvertence

Because the location of the shabu was indicated in the warrant and thus known to
the police operatives, it is reasonable to assume that the police found the packets of
the shabu first. Once the valid portion of the search warrant has been executed, the
"plain view doctrine" can no longer provide any basis -for admitting the other
items subsequently found. As has been explained:

What the 'plain view' cases have in common is that the police officer in each
of them had a prior justification for an intrusion in the course of which he
came inadvertently across a piece of evidence incriminating the accused.
The doctrine serves to supplement the prior justification -whether it be a
warrant for another object, hot pursuit, search incident to lawful arrest, or
some other legitimate reason for being present unconnected with a search
directed against the accused -and permits the warrantless seizure. Of course,
the extension of the original justification is legitimate only where it is
immediately apparent to the police that they have evidence before them; the
'plain view' doctrine may not be used to extend a general exploratory search
from one object to another until something incriminating at last emerges. 37

The only other possible justification for an intrusion by the police is the conduct of
a search pursuant to "accused-appellant's lawful arrest for possession
of shabu. However, a search incident to a lawful arrest is limited to the person of
the one arrested and the premises within his immediate control. 18 The rationale for
permitting such a search is to prevent the person arrested from obtaining a weapon
to commit violence, or to reach for incriminatory evidence and destroy it.

The police failed to allege in this case the time when the marijuana was
found, i.e., whether prior to, or contemporaneous with, the shabu subject of the
warrant, or whether it was recovered on accused-appellant's person or in an area
within his immediate control. Its recovery, therefore, presumably during the search
conducted after the shabu had been recovered from the cabinet, as attested to by
SPO1 Badua in his depostion, was invalid.

Apparent Illegality of the Evidence

The marijuana bricks were wrapped in newsprint. There was no apparent illegality
to justify their seizure. This case is similar to People. v. Musa39  in which we
declared inadmissible the marijuana recovered by NARCOM agents because the
said drugs were contained in plastic bag which gave no indication of its contents.
We explained:

Moreover, when the NARCOM agents saw the plastic bag hanging in one
corner of the kitchen, they had no clue as to its contents. They had to ask the
appellant what the bag contained. When the appellant refused to respond,
they opened it and found the marijuana. Unlike Ker v. California, where the
marijuana was visible to the police officer's eyes, the NARCOM agents in
this case could not have discovered the inculpatory nature of the contents of
the bag had they not forcibly opened it; Even assuming then, that the
NARCOM agents inadvertently came across the plastic bag because it was
within their "plain view," what may be said to be the object in their "plain
view" was just the plastic bag and not the marijuana. The incriminating
nature of the contents of the plastic bag was not immediately apparent from
the "plain view" of said object. It cannot be claimed that the plastic bag
clearly betrayed its contents, whether by its distinctive configuration, is
transparency, or otherwise, that its contents are obvious to an observer . 40
No presumption of regularity may be invoked by an officer in aid of the process
when he undertakes to justify an encroachment of rights secured by the
Constitution.41 In this case, the marijuana allegedly found in the possession of
accused-appellant was in the form of two bricks wrapped in newsprint. Not being
in a transparent container, the contents wrapped in newsprint could not have been
readily discernible as marijuana. Nor was there mention of the time or manner
these items were discovered. Accordingly, for failure of the prosecution to prove
that the seizure of the marijuana without a warrant was conducted in accordance
with the "plain view doctrine," we hold that the marijuana is inadmissible in
evidence against accused-appellant. However, the confiscation of the drug must be
upheld.

Third. Accused-appellant claims that undue and unnecessary force was employed


by the searching party in effecting the raid.

Rule 126, §7 of the Revised Rules on Criminal Procedure 42 provides:

Right to break door or window to effect search. - The officer, if refused


admittance to the place of directed search after giving notice of his purpose
and authority, may break open any outer or inner door or window of a house
or any part of a house or anything therein to execute the warrant or liberate
himself or any person lawfully aiding him when unlawfully detained therein.

Accused-appellant's claim that the policemen had clambered up the roof of his
house to gain entry and had broken doors and windows in the process is
unsupported by reliable and competent proof. No affidavit or sworn statement of
disinterested persons, like the barangay officials or neighbors, has been presented
by accused-appellant to attest to the truth of his claim.

In contrast, Aguilar and Duano's claim that they had to use some force in order to
gain entry cannot be doubted. The occupants of the house, especially accused-
appellant, refused to open the door despite the fact that the searching party knocked
on the door several times. Furthermore, the agents saw the suspicious movements
of the people inside the house. These circumstances justified the searching party's
forcible entry into the house, founded as it is on the apprehension that the
execution of their mission would be frustrated unless they do so.

WHEREFORE, in Criminal Case No. Q-95-64357, the decision of the Regional


Trial Court, Branch 96, Quezon City, finding accused-appellant Roberto Salanguit
y Ko guilty of possession of illegal drugs under §16 of R.A. No.6425, otherwise
known as the Dangerous Drugs Act, as amended, and sentencing him to suffer a
prison term ranging from six (6) months of arresto mayor, as minimum, and four
(4) years and two (2) months of prision correccional, as maximum, and ordering
the confiscation of 11.14 grams of methamphetamine hydrochloride is
AFFIRMED .
In Criminal Case No. Q-95-64358, the decision of the same court finding accused-
appellant Roberto Salanguit y Ko guilty of possession of prohibited drugs under §8
of R.A. No. 6425, as amended, and sentencing him to suffer the penalty
of reclusion perpetua and to pay a fine of Pl00,000.00 is hereby REVERSED and
SET ASIDE and accused- appellant is ACQUITTED of the crime charged.
However, the confiscation of the 1,254 grams of marijuana, as well as the 11.14
grams of methamphetamine hydrochloride, and its disposition as ordered by the
trial court is AFFIRMED .

CASES:
Alih v. Castro, G.R. No. L-69401, June 23, 1987

On November 25, 1984, a contingent of more than two hundred Philippine marines
and elements of the home defense forces raided the compound occupied by the
petitioners at Gov. Alvarez street, Zamboanga City, in search of loose firearms,
ammunition and other explosives. 1

The military operation was commonly known and dreaded as a "zona," which was
not unlike the feared practice of the kempeitai during the Japanese Occupation of
rounding up the people in a locality, arresting the persons fingered by a hooded
informer, and executing them outright (although the last part is not included in the
modern refinement).

The initial reaction of the people inside the compound was to resist the invasion
with a burst of gunfire. No one was hurt as presumably the purpose was merely to
warn the intruders and deter them from entering. Unfortunately, as might be
expected in incidents like this, the situation aggravated soon enough. The soldiers
returned fire and a bloody shoot-out ensued, resulting in a number of casualties. 2

The besieged compound surrendered the following morning, and sixteen male
occupants were arrested, later to be finger-printed, paraffin-tested and
photographed over their objection. The military also inventoried and confiscated
nine M16 rifles, one M14 rifle, nine rifle grenades, and several rounds of
ammunition found in the premises. 3

On December 21, 1984, the petitioners came to this Court in a petition for
prohibition and mandamus with preliminary injunction and restraining order. Their
purpose was to recover the articles seized from them, to prevent these from being
used as evidence against them, and to challenge their finger-printing,
photographing and paraffin-testing as violative of their right against self-
incrimination.4

The Court, treating the petition as an injunction suit with a prayer for the return of
the articles alleged to have been illegally seized, referred it for hearing to Judge
Omar U. Amin of the regional trial court, Zamboanga City. 5 After receiving the
testimonial and documentary evidence of the parties, he submitted the report and
recommendations on which this opinion is based. 6

The petitioners demand the return of the arms and ammunition on the ground that
they were taken without a search warrant as required by the Bill of Rights. This is
confirmed by the said report and in fact admitted by the respondents, "but with
avoidance. 7

Article IV, Section 3, of the 1973 Constitution, which was in force at the time of
the incident in question, provided as follows:

Sec. 3. The right of the people to be secure in their persons, houses, papers,
and effects against unreasonable searches and seizures of whatever nature
and for any purpose shall not be violated, and no search warrant or warrant
of arrest shall issue except upon probable cause to be determined by the
judge, or such other responsible officer as may be authorized by law, after
examination under oath or affirmation of the complainant and the witnesses
he may produce, and particularly describing the place to be searched, and
the persons or things to be seized.

It was also declared in Article IV, Section 4(2) that-

Sec. 4(2) Any evidence obtained in violation of this or the preceding section
shall be inadmissible for any purpose in any proceeding.

The respondents, while admitting the absence of the required such warrant, sought
to justify their act on the ground that they were acting under superior
orders. 8 There was also the suggestion that the measure was necessary because of
the aggravation of the peace and order problem generated by the assassination of
Mayor Cesar Climaco. 9

Superior orders" cannot, of course, countermand the Constitution. The fact


that the petitioners were suspected of the Climaco killing did not excuse the
constitutional short-cuts the respondents took. As eloquently affirmed by the
U.S. Supreme Court in Ex parte Milligan: 10

The Constitution is a law for rulers and people, equally in war and in peace,
and covers with the shield of its protection all classes of men, at all times
and under all circumstances. No doctrine, involving more pernicious
consequences, was ever invented by the wit of man than that any of its
provisions can be suspended during any of the great exigencies of
government.
The precarious state of lawlessness in Zamboanga City at the time in question
certainly did not excuse the non-observance of the constitutional guaranty against
unreasonable searches and seizures. There was no state of hostilities in the area to
justify, assuming it could, the repressions committed therein against the petitioners.

It is so easy to say that the petitioners were outlaws and deserved the arbitrary
treatment they received to take them into custody; but that is a criminal argument.
It is also fallacious. Its obvious flaw lies in the conclusion that the petitioners were
unquestionably guilty on the strength alone of unsubstantiated reports that they
were stockpiling weapons.

The record does not disclose that the petitioners were wanted criminals or fugitives
from justice. At the time of the "zona," they were merely suspected of the mayor's
slaying and had not in fact even been investigated for it. As mere suspects, they
were presumed innocent and not guilty as summarily pronounced by the military.

Indeed, even if were assumed for the sake of argument that they were guilty, they
would not have been any less entitled to the protection of the Constitution, which
covers both the innocent and the guilty. This is not to say, of course, that the
Constitution coddles criminals. What it does simply signify is that, lacking the
shield of innocence, the guilty need the armor of the Constitution, to protect them,
not from a deserved sentence, but from arbitrary punishment. Every person is
entitled to due process. It is no exaggeration that the basest criminal, ranged
against the rest of the people who would condemn him outright, is still, under the
Bill of Rights, a majority of one.

If the respondents did not actually disdain the Constitution when they made their
illegal raid, they certainly gave every appearance of doing so. This is truly
regrettable for it was incumbent on them, especially during those tense and tindery
times, to encourage rather than undermine respect for the law, which it was their
duty to uphold.

In acting as they did, they also defied the precept that "civilian authority is at all
times supreme over the military" so clearly proclaimed in the 1973
Constitution. 11 In the instant case, the respondents simply by-passed the civil
courts, which had the authority to determine whether or not there was probable
cause to search the petitioner's premises. Instead, they proceeded to make the raid
without a search warrant on their own unauthorized determination of the
petitioner's guilt.

The respondents cannot even plead the urgency of the raid because it was in fact
not urgent. They knew where the petitioners were. They had every opportunity to
get a search warrant before making the raid. If they were worried that the weapons
inside the compound would be spirited away, they could have surrounded the
premises in the meantime, as a preventive measure. There was absolutely no reason
at all why they should disregard the orderly processes required by the Constitution
and instead insist on arbitrarily forcing their way into the petitioner's premises with
all the menace of a military invasion.

Conceding that the search was truly warrantless, might not the search and seizure
be nonetheless considered valid because it was incidental to a legal arrest? Surely
not. If all the law enforcement authorities have to do is force their way into any
house and then pick up anything they see there on the ground that the occupants
are resisting arrest, then we might as well delete the Bill of Rights as a fussy
redundancy.

When the respondents could have easily obtained a search warrant from any of the
TEN civil courts then open and functioning in Zamboanga City, 12 they instead
simply barged into the beleaguered premises on the verbal order of their superior
officers. One cannot just force his way into any man's house on the illegal orders of
a superior, however lofty his rank. Indeed, even the humblest hovel is protected
from official intrusion because of the ancient rule, revered in all free regimes, that
a man's house is his castle.

It may be frail; its roof may shake; the wind may enter; the rain may enter.
But the King of England may not enter. All the forces of the Crown dare not
cross the threshold of the ruined tenement. 13

If the arrest was made under Rule 113, Section 5, of the Rules of Court in
connection with a crime about to be committed, being committed, or just
committed, what was that crime? There is no allegation in the record of such a
justification. Parenthetically, it may be observed that under the Revised Rule 113,
Section 5(b), the officer making the arrest must have personal knowledge of the
ground therefor as stressed in the recent case of People v. Burgos. 14

If follows that as the search of the petitioners' premises was violative of the
Constitution, all the firearms and ammunition taken from the raided compound are
inadmissible in evidence in any of the proceedings against the petitioners. These
articles are "fruits of the poisonous tree. 15 As Judge Learned Hand observed,
"Only in case the prosecution which itself controls the seizing officials, knows that
it cannot profit by their wrong, will the wrong be repressed. 16 Pending
determination of the legality of such articles, however, they shall remain
in custodia legis, subject to such appropriate disposition as the corresponding
courts may decide. 17

The objection to the photographing, fingerprinting and paraffin-testing of the


petitioners deserves slight comment. The prohibition against self-incrimination
applies to testimonial compulsion only. As Justice Holmes put it in Holt v. United
States, 18 "The prohibition of compelling a man in a criminal court to be a witness
against himself is a prohibition of the use of physical or moral compulsion to extort
communications from him, not an exclusion of his body as evidence when it may
be material."

The fearful days of hamleting salvaging, "zona" and other dreaded operations
should remain in the past, banished with the secret marshals and their covert
license to kill without trial. We must be done with lawlessness in the name of law
enforcement. Those who are supposed to uphold the law must not be the first to
violate it. As Chief Justice Claudio Teehankee stressed in his concurring opinion
in Lacanilao v. De Leon, 19 "It is time that the martial law regime's legacy of the
law of force be discarded and that there be a return to the force and rule of law."

All of us must exert efforts to make our country truly free and democratic, where
every individual is entitled to the full protection of the Constitution and the Bill of
Rights can stand as a stolid sentinel for all, the innocent as well as the guilty,
including the basest of criminals.

WHEREFORE, the search of the petitioners' premises on November 25, 1984, is


hereby declared ILLEGAL and all the articles seized as a result thereof are
inadmissible in evidence against the petitioners in any proceedings. However, the
said articles shall remain in custodia legis pending the outcome of the criminal
cases that have been or may later be filed against the petitioners.

Uy Kheytin v. Villareal, G.R. No. 16009, September 21, 1920

This is an original petition, filed in this court, for the writs of injunction and
prohibition. It appears from the record that on April 30, 1919, one Ramon
Gayanilo, corporal of the Philippine Constabulary, presented to the judge of the
Court of First Instance of Iloilo an application for search warrant, the said Ramon
Gayanilo stating in his application; "That in the house of Chino Uy Kheytin, Sto.
Niño St., No. 20, Iloilo, under the writing desk in his store, there is kept a certain
amount of opium." The application was subscribed and sworn to by the said
complainant before the Honorable L. M. Southworth, judge of the Twenty-third
Judicial District.

Upon that application the said judge, on the same day, issued a search warrant in
the following terms:

The United States, to any officer of the law.

Whereas on this day proof , by affidavit, having been presented before me


by Corporal Ramon Gayanilo, Philippine Constabulary, that there is
probable cause to believe that in the house of Chino Uy Kheytin, Sto. Niño
St., No. 20, under the desk for writing in his store there is kept a certain
amount of opium.
Therefore, you are hereby commanded during day or night to make an
immediate search on the person of Uy Kheytin or in the house, Sto. Niño
St., No. 20, for the following property opium and, if you find the same or
any part thereof, to bring it forthwith before me in the Court of First
Instance of Iloilo.

Witness my hand this 30th day of April, 1919.

(Sgd.) L. M. SOUTHWORTH,
Judge of the Court of Iloilo.

Armed with that search warrant, the respondent M. S. Torralba, lieutenant of the
Philippine Constabulary, accompanied by some of his subordinates, on the same
day (April 30th) searched the house of the petitioner Uy Kheytin and found therein
60 small cans of opium. They wanted to search also the bodega on the ground-
floor of the house, but Uy Kheytin positively denied that it was his or that he
rented it. Lieutenant Torralba wanted to be sure, and for this reason he placed a
guard in the premises to see that nothing was removed therefrom, and then went
away to find out who the owner of the bodega was. The next morning he learned
from the owner of the house, one Segovia, of the town of Molo, that the Chinaman
Uy Kheytin was the one who was renting the bodega. Thereupon Lieutenant
Torralba and his subordinates resumed the search and then and there found and
seized the following articles:

No. 2. — One wrap of paper containing a broken bottle of opium liquid,
which is kept in a tin box No. 1.

No. 3. — One wrap of paper containing an opium pipe, complete, one
opium container, one wrap of opium ashes, one rag soaked in opium and one
thimble with opium.

No. 4. — One leather hand bag containing 7 small bottle containing opium,
with two cedulas belonging to Tian Liong, with key.

No. 5. — One wooden box containing 75 empty cans, opium containers.

No. 6. — One tin box containing 23 small empty cans, opium containers.

No. 7. — One cardboard box containing 3 pieces of wood, one old chisel,
one file, one piece of soldering lead, one box of matches, 5 pieces of iron
plates, and several other tin plates.

No. 8. — One roll of 7 ½ sheets of brass.

No. 9. — Three soldering outfits.


No. 10. — One hammer.

No. 11. — One Chinese scale for opium.

No. 12. — Twelve small bottles empty.

No. 13. — Two bottles containing opium.

No. 14. — One bundle of Chinese books of accounts with several personal
letter of Chine Uy Kheytin.

No. 15. — One tin box containing 60 cans of molasses, with 1 small bottle
containing molasses.

Thereafter a criminal complaint was filed in the court of the justice of the peace of
Iloilo against all the petitioners herein, charging them with a violation of the
Opium Law. They were duly arrested, and a preliminary investigation was
conducted by the justice of the peace, after which he found that there was probable
cause for believing that the crime complained of had been committed and that the
defendants were the persons responsible therefor. The cause was duly transmitted
to the Court of First Instance.

While said cause was in the Court of First Instance, pending the filing of a
complaint by the provincial fiscal, the defendants, petitioners herein, through their
attorney, filed a petition in the Court of First Instance, asking for the return of
"private papers, books and other property" which the Constabulary officers had
seized from said defendants, upon the ground that they had been so seized illegally
and in violation of the constitutional rights of the defendants. It was urged (1) that
the search warrant of April 30th was illegal because the requisites prescribed by
the General Orders No. 58 had not been complied with in its issuance; (2) that the
searches and seizures made on May 1st had been made without any semblance of
authority and hence illegal; and (3) that the seizure of the defendants' books and
letters was a violation of the provisions of the Jones Law providing that no person
shall be compelled to testify against himself, and protecting him against
unreasonable searches and seizures.

After a hearing upon said motion, the Honorable Antonio Villareal, judge, in a
very carefully prepared opinion, reached the conclusion that the searches and
seizures complained of had been legally made, and consequently, denied the
defendants' petition.

Thereafter and on November 22, 1919, the said defendants, petitioners herein, filed
the present petition in this court, praying as follows:
Wherefore, in view of the foregoing allegations, it is respectfully prayed that
a preliminary injunction issue.

First, restraining the respondent judge, and his successors from making any
cognizance of any action of any kind which has or may be brought against
these petitioners which have resulted directly or indirectly from the unlawful
searches and seizures above-mentioned;

Second, restraining the respondent clerk of the court, the respondent fiscal,
the respondent commandant of the Constabulary, and the successors of any
of them, and the assistants of any of them, from any further examination of
the private papers, books, and other property unlawfully seized as above
alleged; from making or using the same for the purpose or in such a manner
that the character or reputation of these petitioners might be injured; from
making or using any copies, memorandum, notes, or extracts obtained from
the books, papers, etc., so seized; from making any examinations of any of
the property thus obtained or from using any reports or from publishing in
any manner any reports already prepared as a result of the examination of
such property; or from making any other use of the property and papers so
obtained until orders are received from this court regarding the disposition
of the same.

It is further requested, that a writ of prohibition issue, restraining the


respondent judge from at any time taking cognizance of any action or
prosecution growing out of the unlawful searches and seizures above-
mentioned, and directing such judge or his successor to order the immediate
return to these petitioners of all of the papers and other property thus
unlawfully obtained, together with all copies, extracts, memorandum, notes,
photographs, reports, samples, or evidence obtained by reason of such
searches and seizures whereby the reputation and character of petitioners
may be further damaged; furthermore enjoining all of the respondents and
their assistants from divulging any of the secrets or information which they
have thus unlawfully obtained from these petitioners; and especially
ordering the respondent judge to dismiss all actions or prosecutions already
filed before him or which may hereafter come before him as a result of the
unlawful acts herein alleged.

THE SEARCH WARRANT OF APRIL 30TH

The petitioners contend that the search warrant of April 30, 1919, was illegal, (1)
because it was not issued upon either of the grounds mentioned in section 96 of
General Orders No. 58, and (2) because the judge who issued it did not determine
the probable cause by examining witnesses under oath, a required by section 98 of
said General Orders No. 58.

Section 96 of General Orders No. 58 is as follows:

SEC. 96. It (a search warrant) may be issued upon either of the following
grounds:

1. When the property was stolen or embezzled.

2. When it was used or when the intent exists to use it as the means of
committing a felony.

In support of their first contention the petitioners argue that the property ordered to
be seized, namely, opium, under the said search warrant, had not been stolen or
embezzled, nor had it been used or intended to be used as the means of committing
a felony; that the word "felony" is applicable only to a serious crime which
is malum per se and not to one which is merely malum prohibitum, such as the
possession of opium.

For the purpose of this decision we deem it unnecessary to draw the distinction
between the words "felony" and "misdemeanor" a used in the common law. Suffice
it to say that, whatever may be the technical common-law meaning of the word
"felony," which is used in paragraph 2 of section 96 above quoted, we believe it
would be the height of absurdity to hold, upon technical grounds, that a search
warrant is illegal which is issued to search for and seize property the very
possession of which is forbidden by law and constitutes a crime. Opium is such
property. "Search-warrants have heretofore been allowed to search for stolen
goods, for goods supposed to have been smuggled into the country in violation of
the revenue laws, for implements of gaming or counterfeiting, for lottery tickets or
prohibited liquors kept for sale contrary to law, for obscene books and paper kept
for sale or circulation, and for powder or other explosive and dangerous material so
kept as to endanger the public safety." (Cooley on Constitutional Limitations, 7th
ed., p. 432.)

In support of their second contention, the petitioners invoke section 98 of General


Orders No. 58, which provides a follow:

SEC. 98. The judge or justice must, before issuing the warrant, examine on
oath the complainant and any witnesses he may produce and take their
depositions in writing.

Section 97 provides that "a search warrant shall not issue except for probable
cause" and section 98 above quoted provides the manner in which that probable
cause shall be determined by the judge issuing the warrant. In the present case,
however, the judge did not examine any witness under oath but relied solely upon
the sworn application of the Constabulary officer in determining whether there was
probable cause. In that application the complainant swore positively: "That in the
house of Chino Uy Kheytin, Sto. Niño St., No. 20, Iloilo, under the writing desk in
his store, there is kept a certain amount of opium." This statement was found to be
true by the subsequent finding and seizure of a considerable quantity of opium in
the place mentioned. The question now is, whether the omission of the judge to
comply with the requirements of section 98 would, under the circumstances, justify
the court in declaring that the search warrant in question was illegal and ordering
the return of the opium found and seized under said warrant.

A search warrant may be likened to a warrant of arrest. The issuance of both is


restricted by the same provision of the Jones Law (sec. 3) which is as follows:

That no warrant shall issue but upon probable cause, supported by oath or
affirmation, and particularly describing the place to be searched and the
person or thing to be seized.

A person, then, is protected from unreasonable arrests just as much as he is


protected from unreasonable searches. But suppose he happened to be arrested
without any warrant, or upon a warrant which had been issued by a judge without
first properly determining whether there was probable cause, and upon
investigation it should be found, from his own admission, that he was the author of
the crime, — should he be released upon the ground that he had not been legally
arrested? In the case of Ker vs. Illinois (119 U. S., 436) Ker having committed the
crime of larceny, escaped and went to Peru. He was kidnapped in Peru and brought
back to the State of Illinois without any pretense of authority. Passing upon the
question of the constitutionality of the arrest of Ker, the Supreme Court of the
United States, speaking through Mr. Justice Miller, said:

We do not intend to say that there may not be proceedings previous to the
trial in regard to which the prisoner could invoke in some manner the
provisions of this clause of the Constitution; but for mere irregularities in
the manner in which he may be brought into the custody of the law, we do
not think he is entitled to say that he should not be tried at all for the crime
with which he is charged in a regular indictment. He may be arrested for a
very heinous offense by persons without any warrant, or without any
previous complaint, and brought before a proper officer, and this may be in
some sense said to be "without due process of law." But it would hardly be
claimed that after the case had been investigated, and the defendant held by
the proper authorities to answer for the crime, he could plead that he was
first arrested "without due process of law." (Followed in U. S. vs. Grant and
Kennedy, 18 Phil., 122, 146; U. S. vs. Wilson, 4 Phil., 317.)
In the present case there was an irregularity in the issuance of the search warrant in
question in that the judge did not first examine the complainant or any witnesses
under oath, as required by section 98 of General Orders No. 58. But the property
sought to be searched for and seized having been actually found in the place
described by the complainant, reasoning by analogy from the case of an improper
arrest, we are of the opinion that that irregularity is not sufficient cause for
ordering the return of the opium found and seized under said warrant, to the
petitioners, and exonerating the latter.

II

THE SEARCH MADE ON MAY 1ST

Petitioners content that this was made without any search warrant and without any
authority of law; that the search warrant of April 30th could not be used on May
1st because that warrant had been executed on the day of its issuance. In support of
this contention counsel for the petitioners, in the lower court, argued that:

While it is true that a warrant is good for 10 days after the date of issuance,
this cannot be interpreted to mean that a search warrant can be used every
day for 10 days, and for a different purpose each day. This would be absurd.
It is admitted, for sake of argument, that if upon a search, under a legally
issued warrant, some other prohibited articles than those named in the
warrant should be found, these articles might be seized. Also, it might
possibly be true, that if a warrant was issued to search for a certain article
and it was not found after the first search, that another search could be made
sometime within the 10 days. But this is certainly the furthest possible
extreme the doctrine could be carried. It certainly could not be interpreted to
allow a search to be made, and after the articles for which the warrant was
issued had been seized, to use this same warrant as authority to make
another search.

We agree with counsel that a search warrant cannot be used every day for ten days,
"and for a different purpose each day," and that after the articles for which the
warrant was issued have been seized the same warrant cannot be used as authority
to make another search. But this argument is not applicable to the facts in this
case. It appears from the oral evidence adduced during the hearing of the
petitioners' motion in the court below that the search for opium, the property
mentioned in the warrant, was not completed on April 30th; it was interrupted by
the necessity to ascertain who the owner of the bodega on the ground-floor was,
because the petitioner Uy Kheytin falsely disclaimed ownership thereof. In other
words, the search of May 1st was not made "for a different purpose," nor could it
be considered "another search," but was really a continuation of the search begun
on April 30th. This is shown by the fact that during the interval between the two
searches the premises in question were guarded by Constabulary soldiers, and the
petitioners were made to understand on April 30th that the authorities were not yet
through with the search and would continue the same as soon as they found out
that the bodega was also occupied by the petitioner Uy Kheytin. We are, therefore,
of the opinion that the search made on May 1st was authorized under the search
warrant of April 30th

III

THE SEIZURE OF BOOKS, LETTERS, ETC.

The important question that remains to be decided is whether, under a search


warrant for opium, the officers of the law were authorized to seize books, personal
letters, and other property having a remote or no connection with opium. The
respondent M. S. Torralba, lieutenant of the Constabulary, testified that he seized
these articles because he believed or suspected that they had some relation with the
opium in question; in other words, he thought that they might be used as evidence
against the petitioners when they are prosecuted for a violation of the Opium Law.
The respondents contend that this was a sufficient justification under the law for
the seizure of such articles under the same warrant for opium.

We are of the opinion that the respondent's contention in untenable. Both the Jones
Law (sec. 3) and General Orders No. 58 (sec. 97) specifically require that a search
warrant should particularly describe the place to be searched and the things to be
seized. The evident purpose and intent of this requirement is to limit the things to
be seized to those, and only those, particularly described in the search warrant —
to leave the officers of the law with no discretion regarding what articles they shall
seize, to the end that "unreasonable searches and seizures" may not be made, —
that abuses may not be committed. That this is the correct interpretation of this
constitutional provision is borne out by American authorities.

In order to comply with the constitutional provisions regulating the issuance


of search warrants, the property to be seized under a warrant must be
particularly described therein and no other property can be taken
thereunder. The goods to be seized must be described with such certainty as
to identify them, and the description must be so particular that the officer
charged with the execution of the warrant will be left with no discretion
respecting the property to be taken. . . . Under a warrant to search a person
for stolen goods, the officer cannot lawfully take from the person a letter,
such letter not being particularly described in the warrant as property to be
searched for. (24 R. C. L., 714, 715.)

It is a violation of the declaration of rights respecting searches and seizures


for an officer, while searching one's person under a search warrant for stolen
goods, to take from it, against the party's will, a letter written to him. (State
vs. Slamon, 87 Am. St. Rep., 711.)
We have said that if the officer follows the command of his warrant, he is
protected; and this is so even when the complaint proves to have been
unfounded. But if he exceed the command by searching in places not
described therein, or by seizing persons or articles not commanded, he is not
protected by the warrant, and can only justify himself as in other cases
where he assumes to act without process. Obeying strictly the command of
his warrant, he may break open outer or inner doors, and his justification
does not depend upon his discovering that for which he is to make search.
(Cooley on Constitutional Limitations, 7th ed., p. 434)

That the officers of the law believed that the books, papers, etc., which they seized
might be used as evidence against the petitioners herein a criminal action against
them for a violation of the Opium Law, is no reason or justification under the law
for the seizure: First, because they were not "particularly described" or even
mentioned in the search warrant; second, because, even if they had been mentioned
in the search warrant, they could not be legally seized, for a search warrant cannot
be used for the purpose of obtaining evidence; and third, because to compel a
person to produce his private papers to be used in evidence against him would be
equivalent to compelling him to be a witness against himself.

1. The authorities for the first proposition have already been given above.

2. It may be said that —

Books of account, private documents, and private papers are property which
men may lawfully possess. It is not believed that the stature (subsection 2 of
section 96, G. O. 58) was intended to cover property of this class. Granting
that property of which men may lawfully possess themselves has been used
in the commission of a crime and not possessed nor created purely for the
purpose of committing a crime, and not likely to be used again, then
certainly its seizure can only be for the purpose of using the same as
evidence to prove the commission of the crime already committed. This
purpose is not contemplated by the provision of the law. The finding of
evidence can not be the immediate reason for issuing the search warrant. To
use a search warrant for the purpose of obtaining possession of property for
this purpose would be an "unreasonable" use of the remedy by search
warrant, which is prohibited by law. (Regidor vs. Araullo, 5 Off. Gaz., 955,
961, 962; U. S. vs. De los Reyes and Esguerra, 20 Phil., 467.)

Judge Cooley in his work on Constitutional Limitations, 7th ed., p. 431, says:

The warrant is not allowed for the purpose of obtaining evidence of an


intended crime; but only after the lawful evidence of an offense actually
committed. Nor even then is it allowable to invade one's privacy for the sole
purpose of obtaining evidence against him, except in a few special cases
where that which is the subject of the crime is supposed to be concealed, and
the public or the complainant has an interest in it on its destruction.

3. In the case of Boyd vs. United States (116 U. S., 616), the Supreme Court of the
United States, speaking through Mr. Justice Bradley, said:

The seizure or compulsory production of a man's private papers to be used


in evidence against him is equivalent to compelling him to be a witness
against himself, and, in a prosecution for a crime, penalty or forfeiture, is
equally within the prohibition of the Fifth Amendment.

Both amendments (fourth and fifth) relate to the personal security of the
citizen. They nearly run into and mutually throw light upon each other.
When the thing forbidden in the Fifth Amendment, namely, compelling a
man to be a witness against himself, is the object of a search and seizure of
his private papers, it is an "unreasonable search and seizure" within the
Fourth Amendment.

Search and seizure of a man's private papers to be used in evidence for the
purpose of convicting him of a crime, recovering a penalty, or of forfeiting
his property, is totally different from the search and seizure of stolen goods,
dutiable articles on which the duties have not been paid, and the like, which
rightfully belong to the custody of the law. (Se also Silverthorne Lumber
Co. vs. United States, decided Jan. 26, 1920, by the Supreme Court of the
United States.)

The seizure of a person's private papers, to be used in evidence against him,


is equivalent to compelling him to be a witness against himself. (State vs.
Slamon, 73 Vt., 212; 87 Am. St. Rep., 711.)

From all of the foregoing our conclusions are:

1. That although in the issuance of the search warrant in question the judge did not
comply with the requirements of section 98 of General Orders No. 58, the
petitioners are not entitled to the return of the opium and its paraphernalia which
were found and seized under said warrant, and much less are they entitled to be
exonerated because of such omission of the judge.

2. That the search made on May 1st was a continuation of the search begun on the
previous day, and, therefore, did not require another search warrant.

3. That the seizure of the petitioner's books, letters, telegrams, and other articles
which have no inherent relation with opium and the possession of which is not
forbidden by law, was illegal and in violation of the petitioners' constitutional
rights.
Therefore, it is hereby ordered and decreed that each and all of the respondents
herein, their assistants or successors, be, and they hereby are, forbidden from
examining or making any use of said books, letters, telegrams, etc., namely, the
articles described in items Nos. 7, 8, 9, 10, 12, 14, and 15 of the sheriff's return
(Exhibit 3, reproduced at the top of page 3 of this decision 1) and they are hereby
ordered to immediately return the said articles to the petitioners. So ordered.

Araullo and Villamor, JJ., concur.


Mapa, C.J., concurs in the result.

F. Warrantless search and seizure


1. Search incident to a lawful arrest
2. Stop-and-frisk (“Terry” search)
3. Consented warrantless search
4. Customs search; search of vessels
5. The plain view doctrine
6. Warrantless search of moving land vehicles
7. Warrantless search under urgent and exigent circumstances

CASES:
Malacat v. CA, G.R. No. 123595, December 12, 1997
[G.R. No. 123595. December 12, 1997.]

SAMMY MALACAT y MANDAR, Petitioner, v. COURT OF APPEALS, and


PEOPLE OF THE PHILIPPINES, Respondents.

DECISION

DAVIDE, JR., J.:

In an Information 1 filed on 30 August 1990, in Criminal Case No. 90-86748


before the Regional Trial Court (RTC) of Manila, Branch 5, petitioner Sammy
Malacat y Mandar was charged with violating Section 3 of Presidential Decree No.
1866, 2 as follows:chanrobles virtualawlibrary chanrobles.com:chanrobles.com.ph

That on or about August 27, 1990, in the City of Manila, Philippines, the said
accused did then and there willfully, unlawfully and knowingly keep, possess
and/or acquire a hand grenade, without first securing the necessary license and/or
permit therefor from the proper authorities.
At arraignment 3 on 9 October 1990, Petitioner, assisted by counsel de officio,
entered a plea of not guilty.

At pre-trial on 11 March 1991, petitioner admitted the existence of Exhibits "A,"


"A-1," and "A-2," 4 while the prosecution admitted that the police authorities were
not armed with a search warrant nor warrant of arrest at the time they arrested
petitioner. 5

At trial on the merits, the prosecution presented the following police officers as its
witnesses: Rodolfo Yu, the arresting officer; Josefino C. Serapio, the investigating
officer; and Orlando Ramilo, who examined the grenade.

Rodolfo Yu of the Western Police District, Metropolitan Police Force of the


Integrated National Police, Police Station No. 3, Quiapo, Manila, testified that on
27 August 1990, at about 6:30 p.m., in response to bomb threats reported seven
days earlier, he was on foot patrol with three other police officers (all of them in
uniform) along Quezon Boulevard, Quiapo, Manila, near the Mercury Drug store
at Plaza Miranda. They chanced upon two groups of Muslim-looking men, with
each group, comprised of three to four men, posted at opposite sides of the corner
of Quezon Boulevard near the Mercury Drug Store. These men were acting
suspiciously with" [t]their eyes . . . moving very fast." 6

Yu and his companions positioned themselves at strategic points and observed both
groups for about thirty minutes. The police officers then approached one group of
men, who then fled in different directions. As the policemen gave chase, Yu caught
up with and apprehended petitioner. Upon searching petitioner, Yu found a
fragmentation grenade tucked inside petitioner’s "front waist line." 7 Yu’s
companion, police officer Rogelio Malibiran, apprehended Abdul Casan from
whom a .38 caliber revolver was recovered. Petitioner and Casan were then
brought to Police Station No. 3 where Yu placed an "X" mark at the bottom of the
grenade and thereafter gave it to his commander. 8

On cross-examination, Yu declared that they conducted the foot patrol due to a


report that a group of Muslims was going to explode a grenade somewhere in the
vicinity of Plaza Miranda. Yu recognized petitioner as the previous Saturday, 25
August 1990, likewise at Plaza Miranda, Yu saw petitioner and 2 others attempt to
detonate a grenade. The attempt was aborted when Yu and other policemen chased
petitioner and his companions; however, the former were unable to catch any of the
latter. Yu further admitted that petitioner and Casan were merely standing on the
corner of Quezon Boulevard when Yu saw them on 27 August 1990. Although
they were not creating a commotion, since they were supposedly acting
suspiciously, Yu and his companions approached them. Yu did not issue any
receipt for the grenade he allegedly recovered from petitioner. 9
Josefino C. Serapio declared that at about 9:00 a.m. of 28 August 1990, petitioner
and a certain Abdul Casan were brought in by Sgt. Saquilla 10 for investigation.
Forthwith, Serapio conducted the inquest of the two suspects, informing them of
their rights to remain silent and to be assisted by competent and independent
counsel. Despite Serapio’s advice, petitioner and Casan manifested their
willingness to answer questions even without the assistance of a lawyer. Serapio
then took petitioner’s uncounselled confession (Exh. "E"), there being no PAO
lawyer available, wherein petitioner admitted possession of the grenade.
Thereafter, Serapio prepared the affidavit of arrest and booking sheet of petitioner
and Casan. Later, Serapio turned over the grenade to the Intelligence and Special
Action Division (ISAD) of the Explosive Ordinance Disposal Unit for
examination. 11

On cross-examination, Serapio admitted that he took petitioner’s confession


knowing it was inadmissible in evidence. 12

Orlando Ramilo, a member of the Bomb Disposal Unit, whose principal duties
included, among other things, the examination of explosive devices, testified that
on 22 March 1991, he received a request dated 19 March 1991 from Lt. Eduardo
Cabrera and PO Diosdado Diotoy for examination of a grenade. Ramilo then
affixed an orange tag on the subject grenade detailing his name, the date and time
he received the specimen. During the preliminary examination of the grenade, he
"found that [the] major components consisting of [a] high filler and fuse assembly
[were] all present," and concluded that the grenade was" [l]ive and capable of
exploding." On even date, he issued a certification stating his findings, a copy of
which he forwarded to Diotoy on 11 August 1991. 13

Petitioner was the lone defense witness. He declared that he arrived in Manila on
22 July 1990 and resided at the Muslim Center in Quiapo, Manila. At around 6:30
in the evening of 27 August 1990, he went to Plaza Miranda to catch a breath of
fresh air. Shortly after, several policemen arrived and ordered all males to stand
aside. The policemen searched petitioner and two other men, but found nothing in
their possession. However, he was arrested with two others, brought to and
detained at Precinct No. 3, where he was accused of having shot a police officer.
The officer showed the gunshot wounds he allegedly sustained and shouted at
petitioner" [i]to ang tama mo sa akin." This officer then inserted the muzzle of his
gun into petitioner’s mouth and said," [y]ou are the one who shot me."cralaw
virtua1aw library

Petitioner denied the charges and explained that he only recently arrived in Manila.
However, several other police officers mauled him, hitting him with benches and
guns. Petitioner was once again searched, but nothing was found on him. He saw
the grenade only in court when it was presented. 14
The trial court ruled that the warrantless search and seizure of petitioner was akin
to a "stop and frisk," where a "warrant and seizure can be effected without
necessarily being preceded by an arrest" and "whose object is either to maintain the
status quo momentarily while the police officer seeks to obtain more information."
15 Probable cause was not required as it was not certain that a crime had been
committed, however, the situation called for an investigation, hence to require
probable cause would have been "premature." 16 The RTC emphasized that Yu
and his companions were" [c]onfronted with an emergency, in which the delay
necessary to obtain a warrant, threatens the destruction of evidence" 17 and the
officers" [h]ad to act in haste," as petitioner and his companions were acting
suspiciously, considering the time, place and "reported cases of bombing." Further,
petitioner’s group suddenly ran away in different directions as they saw the
arresting officers approach, thus" [i]t is reasonable for an officer to conduct a
limited search, the purpose of which is not necessarily to discover evidence of a
crime, but to allow the officer to pursue his investigation without fear of violence."
18

The trial court then ruled that the seizure of the grenade from petitioner was
incidental to a lawful arrest, and since petitioner" [l]ater voluntarily admitted such
fact to the police investigator for the purpose of bombing the Mercury Drug Store,"
concluded that sufficient evidence existed to establish petitioner’s guilt beyond
reasonable doubt.

In its decision 19 dated 10 February 1994 but promulgated on 15 February 1994,


the trial court thus found petitioner guilty of the crime of illegal possession of
explosives under Section 3 of P.D. No. 1866, and sentenced him to
suffer:chanrob1es virtual 1aw library

[T]he penalty of not less than SEVENTEEN (17) YEARS, FOUR (4) MONTHS
AND ONE (1) DAY OF RECLUSION TEMPORAL, as minimum, and not more
than THIRTY (30) YEARS OF RECLUSION PERPETUA, as maximum.

On 18 February 1994, petitioner filed a notice of appeal 20 indicating that he was


appealing to this Court. However, the record of the case was forwarded to the
Court of Appeals which docketed it as CA-G.R. CR No. 15988 and issued a notice
to file briefs. 21

In his Appellant’s Brief 22 filed with the Court of Appeals, petitioner asserted
that:chanrob1es virtual 1aw library

1. THE LOWER COURT ERRED IN HOLDING THAT THE SEARCH UPON


THE PERSON OF ACCUSED-APPELLANT AND THE SEIZURE OF THE
ALLEGED HANDGRENADE FROM HIM "WAS AN APPROPRIATE
INCIDENT TO HIS ARREST."cralaw virtua1aw library
2. THE LOWER COURT ERRED IN ADMITTING AS EVIDENCE AGAINST
ACCUSED-APPELLANT THE HANDGRENADE ALLEGEDLY SEIZED
FROM HIM AS IT WAS A PRODUCT OF AN UNREASONABLE AND
ILLEGAL SEARCH.

In sum, petitioner argued that the warrantless arrest was invalid due to absence of
any of the conditions provided for in Section 5 of Rule 113 of the Rules of Court,
citing People v. Mengote. 23 As such, the search was illegal, and the hand grenade
seized, inadmissible in evidence.

In its Brief for the Appellee, the Office of the Solicitor General agreed with the
trial court and prayed that its decision be affirmed in toto. 24

In its decision of 24 January 1996, 25 the Court of Appeals affirmed the trial court,
noting, first, that petitioner abandoned his original theory before the court a quo
that the grenade was "planted" by the police officers; and second, the factual
finding of the trial court that the grenade was seized from petitioner’s possession
was not raised as an issue. Further, respondent court focused on the admissibility in
evidence of Exhibit "D," the hand grenade seized from petitioner. Meeting the
issue squarely, the Court of Appeals ruled that the arrest was lawful on the ground
that there was probable cause for the arrest as petitioner was "attempting to commit
an offense," thus:chanrob1es virtual 1aw library

We are at a loss to understand how a man, who was in possession of a live grenade
and in the company of other suspicious character[s] with unlicensed firearm[s]
lurking in Plaza Miranda at a time when political tension ha[d] been enkindling a
series of terroristic activities, [can] claim that he was not attempting to commit an
offense. We need not mention that Plaza Miranda is historically notorious for being
a favorite bomb site especially during times of political upheaval. As the mere
possession of an unlicensed grenade is by itself an offense, Malacat’s posture is
simply too preposterous to inspire belief.

In so doing, the Court of Appeals took into account petitioner’s failure to rebut the
testimony of the prosecution witnesses that they received intelligence reports of a
bomb threat at Plaza Miranda; the fact that PO Yu chased petitioner two days prior
to the latter’s arrest, or on 27 August 1990; and that petitioner and his companions
acted suspiciously, the "accumulation" of which was more than sufficient to
convince a reasonable man that an offense was about to be committed. Moreover,
the Court of Appeals observed:chanrob1es virtual 1aw library

The police officers in such a volatile situation would be guilty of gross negligence
and dereliction of duty, not to mention of gross incompetence, if they [would] first
wait for Malacat to hurl the grenade, and kill several innocent persons while
maiming numerous others, before arriving at what would then be an assured but
moot conclusion that there was indeed probable cause for an arrest. We are in
agreement with the lower court in saying that the probable cause in such a situation
should not be the kind of proof necessary to convict, but rather the practical
considerations of everyday life on which a reasonable and prudent mind, and not
legal technicians, will ordinarily act.

Finally, the Court of Appeals held that the rule laid down in People v. Mengote, 26
which petitioner relied upon, was inapplicable in light of" [c]rucial differences," to
wit:chanrob1es virtual 1aw library

[In Mengote] the police officers never received any intelligence report that
someone [at] the corner of a busy street [would] be in possession of a prohibited
article. Here the police officers were responding to a [sic] public clamor to put a
check on the series of terroristic bombings in the Metropolis, and, after receiving
intelligence reports about a bomb threat aimed at the vicinity of the historically
notorious Plaza Miranda, they conducted foot patrols for about seven days to
observe suspicious movements in the area. Furthermore, in Mengote, the police
officers [had] no personal knowledge that the person arrested has committed, is
actually committing, or is attempting to commit an offense. Here, PO3 Yu [had]
personal knowledge of the fact that he chased Malacat in Plaza Miranda two days
before he finally succeeded in apprehending him.

Unable to accept his conviction, petitioner forthwith filed the instant petition and
assigns the following errors:chanrob1es virtual 1aw library

1. THE RESPONDENT COURT ERRED IN AFFIRMING THE FINDING OF


THE TRIAL COURT THAT THE WARRANTLESS ARREST OF PETITIONER
WAS VALID AND LEGAL.

2. THE RESPONDENT COURT ERRED IN HOLDING THAT THE RULING IN


PEOPLE VS. MENGOTE DOES NOT FIND APPLICATION IN THE INSTANT
CASE.

In support thereof, petitioner merely restates his arguments below regarding the
validity of the warrantless arrest and search, then disagrees with the finding of the
Court of Appeals that he was "attempting to commit a crime," as the evidence for
the prosecution merely disclosed that he was "standing at the corner of Plaza
Miranda and Quezon Boulevard" with his eyes "moving very fast" and "looking at
every person that come (sic) nearer (sic) to them." Finally, petitioner points out the
factual similarities between his case and that of People v. Mengote to demonstrate
that the Court of Appeals miscomprehended the latter.

In its Comment, the Office of the Solicitor General prays that we affirm the
challenged decision.

For being impressed with merit, we resolved to give due course to the petition.

The challenged decision must immediately fall on jurisdictional grounds. To


repeat, the penalty imposed by the trial court was:chanrob1es virtual 1aw library

[N]ot less than SEVENTEEN (17) YEARS, FOUR (4) MONTHS AND ONE (1)
DAY OF RECLUSION TEMPORAL, as minimum, and not more than THIRTY
(30) YEARS OF RECLUSION PERPETUA, as maximum.

The penalty provided by Section 3 of P.D. No. 1866 upon any person who shall
unlawfully possess grenades is reclusion temporal in its maximum period
to reclusion perpetua.

For purposes of determining appellate jurisdiction in criminal cases, the maximum


of the penalty, and not the minimum, is taken into account. Since the maximum of
the penalty is reclusion perpetua, the appeal therefrom should have been to us, and
not the Court of Appeals, pursuant to Section 9(3) of the Judiciary Reorganization
Act of 1980 (B.P. Blg. 129), 27 in relation to Section 17 of the Judiciary Act of
1948, 28 Section 5(2) of Article VIII of the Constitution 29 and Section 3(c) of
Rule 122 of the Rules of Court. 30 The term "life imprisonment" as used in Section
9 of B.P. Blg. 129, the Judiciary Act of 1948, and Section 3 of Rule 122 must be
deemed to include reclusion perpetua in view of Section 5(2) of Article VIII of the
Constitution.

Petitioner’s Notice of Appeal indicated that he was appealing from the trial court’s
decision to this Court, yet the trial court transmitted the record to the Court of
Appeals and the latter proceeded to resolve the appeal.

We then set aside the decision of the Court of Appeals for having been rendered
without jurisdiction, and consider the appeal as having been directly brought to us,
with the petition for review as petitioner’s Brief for the Appellant, the comment
thereon by the Office of the Solicitor General as the Brief for the Appellee and the
memoranda of the parties as their Supplemental Briefs.

Deliberating on the foregoing pleadings, we find ourselves convinced that the


prosecution failed to establish petitioner’s guilt with moral certainty.

First, serious doubt surrounds the story of police officer Yu that a grenade was
found in and seized from petitioner’s possession. Notably, Yu did not identify, in
court, the grenade he allegedly seized. According to him, he turned it over to his
commander after putting an "X" mark at its bottom; however, the commander was
not presented to corroborate this claim. On the other hand, the grenade presented in
court and identified by police officer Ramilo referred to what the latter received
from Lt. Eduardo Cabrera and police officer Diotoy not immediately after
petitioner’s arrest, but nearly seven (7) months later or on 19 March 1991; further,
there was no evidence whatsoever that what Ramilo received was the very same
grenade seized from petitioner. In his testimony, Yu never declared that the
grenade passed on to Ramilo was the grenade the former confiscated from
petitioner. YU did not, and was not made to, identify the grenade examined by
Ramilo, and the latter did not claim that the grenade he examined was that seized
from petitioner. Plainly, the law enforcement authorities failed to safeguard and
preserve the chain of evidence so crucial in cases such as these.

Second, if indeed petitioner had a grenade with him, and that two days earlier he
was with a group about to detonate an explosive at Plaza Miranda, and YU and his
fellow officers chased, but failed to arrest them, then considering that Yu and his
three fellow officers were in uniform and therefore easily cognizable as police
officers, it was then unnatural and against common experience that petitioner
simply stood there in proximity to the police officers. Note that Yu observed
petitioner for thirty minutes and must have been close enough to petitioner in order
to discern petitioner’s eyes "moving very fast."cralaw virtua1aw library

Finally, even assuming that petitioner admitted possession of the grenade during
his custodial investigation by police officer Serapio, such admission was
inadmissible in evidence for it was taken in palpable violation of Section 12(1) and
(3) of Article III of the Constitution, which provide as follows:chanrob1es virtual
1aw library

SEC. 12 (1). Any person under investigation for the commission of an offense shall
have the right to be informed of his right to remain silent and to have competent
and independent counsel preferably of his own choice. If the person cannot afford
the services of counsel, he must be provided with one. These rights cannot be
waived except in writing and in the presence of counsel.

x       x       x

(3) Any confession or admission obtained in violation of this or Section 17 hereof


shall be inadmissible in evidence against him.

Serapio conducted the custodial investigation on petitioner the day following his
arrest. No lawyer was present and Serapio could not have requested a lawyer to
assist petitioner as no PAO lawyer was then available. Thus, even if petitioner
consented to the investigation and waived his rights to remain silent and to
counsel, the waiver was invalid as it was not in writing, neither was it executed in
the presence of counsel.
Even granting ex gratia that petitioner was in possession of a grenade, the arrest
and search of petitioner were invalid, as will be discussed below.

The general rule as regards arrests, searches and seizures is that a warrant is needed
in order to validly effect the same. 31 The Constitutional prohibition against
unreasonable arrests, searches and seizures refers to those effected without a
validly issued warrant, 32 subject to certain exceptions. As regards valid
warrantless arrests, these are found in Section 5, Rule 113 of the Rules of Court,
which reads, in part:chanrob1es virtual 1aw library

Sec. 5. Arrest, without warrant; when lawful. — A peace officer or a private person
may, without a warrant, arrest a person:chanrob1es virtual 1aw library

(a) When, in his presence, the person to be arrested has committed, is actually
committing, or is attempting to commit an offense;

(b) When an offense has in fact just been committed, and he has personal
knowledge of facts indicating that the person to be arrested has committed it; and

(c) When the person to be arrested is a prisoner who has escaped . . .

A warrantless arrest under the circumstances contemplated under Section 5(a) has
been denominated as one "in flagrante delicto," while that under Section 5(b) has
been described as a "hot pursuit" arrest.

Turning to valid warrantless searches, they are limited to the following: (1)
customs searches; (2) search of moving vehicles; (3) seizure of evidence in plain
view; (4) consent searches; 33 (5) a search incidental to a lawful arrest; 34 and (6)
a "stop and frisk." 35

In the instant petition, the trial court validated the warrantless search as a "stop and
frisk" with "the seizure of the grenade from the accused [as] an appropriate
incident to his arrest," hence necessitating a brief discussion on the nature of these
exceptions to the warrant requirement.

At the outset, we note that the trial court confused the concepts of a "stop-and-
frisk" and of a search incidental to a lawful arrest. These two types of warrantless
searches differ in terms of the requisite quantum of proof before they may be
validly effected and in their allowable scope.

In a search incidental to a lawful arrest, as the precedent arrest determines the


validity of the incidental search, the legality of the arrest is questioned in a large
majority of these cases, e.g., whether an arrest was merely used as a pretext for
conducting a search. 36 In this instance, the law requires that there first be a lawful
arrest before a search can be made — the process cannot be reversed. 37 At
bottom, assuming a valid arrest, the arresting officer may search the person of the
arrestee and the area within which the latter may reach for a weapon or for
evidence to destroy, and seize any money or property found which was used in the
commission of the crime, or the fruit of the crime, or that which may be used as
evidence, or which might furnish the arrestee with the means of escaping or
committing violence. 38

Here, there could have been no valid in flagrante delicto or hot pursuit arrest
preceding the search in light of the lack of personal knowledge on the part of Yu,
the arresting officer, or an overt physical act, on the part of petitioner, indicating
that a crime had just been committed, was being committed or was going to be
committed.

Having thus shown the invalidity of the warrantless arrest in this case, plainly, the
search conducted on petitioner could not have been one incidental to a lawful
arrest.

We now proceed to the justification for and allowable scope of a "stop-and-frisk"


as a "limited protective search of outer clothing for weapons," as laid down in
Terry; thus:chanrob1es virtual 1aw library

We merely hold today that where a police officer observes unusual conduct which
leads him reasonably to conclude in light of his experience that criminal activity
may be afoot and that the persons with whom he is dealing may be armed and
presently dangerous, where in the course of investigating this behavior he
identifies himself as a policeman and makes reasonable inquiries, and where
nothing in the initial stages of the encounter serves to dispel his reasonable fear for
his own or others’ safety, he is entitled for the protection of himself and others in
the area to conduct a carefully limited search of the outer clothing of such persons
in an attempt to discover weapons which might be used to assault him. Such a
search is a reasonable search under the Fourth Amendment . . . 39

Other notable points of Terry are that while probable cause is not required to
conduct a "stop and frisk," 40 it nevertheless holds that mere suspicion or a hunch
will not validate a "stop and frisk." A genuine reason must exist, in light of the
police officer’s experience and surrounding conditions, to warrant the belief that
the person detained has weapons concealed about him. 41 Finally, a "stop-and-
frisk" serves a two-fold interest: (1) the general interest of effective crime
prevention and detection, which underlies the recognition that a police officer may,
under appropriate circumstances and in an appropriate manner, approach a person
for purposes of investigating possible criminal behavior even without probable
cause; and (2) the more pressing interest of safety and self-preservation which
permit the police officer to take steps to assure himself that the person with whom
he deals is not armed with a deadly weapon that could unexpectedly and fatally be
used against the police officer.

Here, there are at least three (3) reasons why the "stop-and-frisk" was
invalid:chanrob1es virtual 1aw library

First, we harbor grave doubts as to Yu’s claim that petitioner was a member of the
group which attempted to bomb Plaza Miranda two days earlier. This claim is
neither supported by any police report or record nor corroborated by any other
police officer who allegedly chased that group. Aside from impairing Yu’s
credibility as a witness, this likewise diminishes the probability that a genuine
reason existed so as to arrest and search petitioner. If only to further tarnish the
credibility of Yu’s testimony, contrary to his claim that petitioner and his
companions had to be chased before being apprehended, the affidavit of arrest
(Exh. "A") expressly declares otherwise, i.e., upon arrival of five (5) other police
officers, petitioner and his companions were "immediately collared."cralaw
virtua1aw library

Second, there was nothing in petitioner’s behavior or conduct which could have
reasonably elicited even mere suspicion other than that his eyes were "moving very
fast" — an observation which leaves us incredulous since Yu and his teammates
were nowhere near petitioner and it was already 6:30 p.m., thus presumably dusk.
Petitioner and his companions were merely standing at the corner and were not
creating any commotion or trouble, as Yu explicitly declared on cross-
examination:chanrobles virtual lawlibrary

Q And what were they doing?

A They were merely standing.

Q You are sure of that?

A Yes, sir.

Q And when you saw them standing, there were nothing or they did not create any
commotion?

A None, sir.

Q Neither did you see them create commotion?

A None, sir. 42

Third, there was at all no ground, probable or otherwise, to believe that petitioner
was armed with a deadly weapon. None was visible to Yu, for as he admitted, the
alleged grenade was "discovered" "inside the front waistline" of petitioner, and
from all indications as to the distance between Yu and petitioner, any telltale bulge,
assuming that petitioner was indeed hiding a grenade, could not have been visible
to Yu. In fact, as noted by the trial court:chanrob1es virtual 1aw library

When the policemen approached the accused and his companions, they were not
yet aware that a handgrenade was tucked inside his waistline. They did not see any
bulging object in [sic] his person. 43

What is unequivocal then in this case are blatant violations of petitioner’s rights
solemnly guaranteed in Sections 2 and 12(1) of Article III of the Constitution.

WHEREFORE, the challenged decision of the Seventeenth Division of the Court


of Appeals in CA-G.R. CR No. 15988 is SET ASIDE for lack of jurisdiction on the
part of said Court and, on ground of reasonable doubt, the decision of 10 February
1994 of Branch 5 of the Regional Trial Court of Manila is REVERSED and
petitioner SAMMY MALACAT y MANDAR is hereby ACQUITTED and
ORDERED immediately released from detention, unless his further detention is
justified for any other lawful cause.

People v. Chua, G.R. No. 128222, June 17, 1999

Chua Ho San @ Tsay Ho San (hereafter CHUA) prays for his acquittal and the
reversal of the judgment of 10 February 1997 of the Regional Trial Court (RTC) of
San Fernando, La Union, Branch 66, finding him guilty of transporting, without
appropriate legal authority, the regulated substance methamphetamine
hydrochloride, in violation of Section 15, 1 Article III of Republic Act No. 6425,
otherwise known as the Dangerous Drugs Act of 1972 as further amended by R.A.
No. 7659, 2 and sentencing him to "die by lethal injection." In view thereof, the
judgment was brought to this Court for automatic review pursuant to Article 47 of
the Revised Penal Code, as amended by Section 11 of R.A. No.
7659.chanrobles.com:cralaw:red

In response to reports of rampant smuggling of firearms and other contraband, Jim


Lagasca Cid (hereafter CID), as Chief of Police of the Bacnotan Police Station, of
La Union began patrolling the Bacnotan coastline with his officers. While
monitoring the coastal area of Barangay Bulala on 29 March 1995, he intercepted a
radio call at around 12:45 p.m. from Barangay Captain Juan Almoite (hereafter
ALMOITE) of Barangay Tammocalao requesting police assistance regarding an
unfamiliar speedboat the latter had spotted. According to ALMOITE, the vessel
looked different from the boats ordinarily used by fisherfolk of the area and was
poised to dock at Tammocalao shores. CID and six of his men led by his Chief
Investigator, SPO1 Reynoso Badua (hereafter BADUA), proceeded forthwith to
Tammocalao beach and there conferred with ALMOITE. CID then observed that
the speedboat ferried a lone male passenger. As it was routine for CID to deploy
his men in strategic places when dealing with similar situations, he ordered his
men to take up positions thirty meters from the coastline. When the speedboat
landed, the male passenger alighted, and using both hands, carried what appeared a
multicolored strawbag. He then walked towards the road. By this time, ALMOITE,
CID and BADUA, the latter two conspicuous in their uniform and issued side-
arms, became suspicious of the man as he suddenly changed direction and broke
into a run upon seeing the approaching officers. BADUA, however, prevented the
man from fleeing by holding on to his right arm. Although CID introduced
themselves as police officers, the man appeared impassive. Speaking in English,
CID then requested the man to open his bag, but he seemed not to understand. CID
thus tried speaking Tagalog, then Ilocano, but still to no avail. CID then resorted to
what he termed "sign language;" he motioned with his hands for the man to open
the bag. This time, the man apparently understood and acceded to the request. A
search of the bag yielded several transparent plastic packets containing yellowish
crystalline substances. CID then gestured to the man to close the bag, which he did.
As CID wished to proceed to the police station, he signaled the man to follow, but
the latter did not to comprehend. Hence, CID placed his arm around the shoulders
of the man and escorted the latter to the police headquarters.

At the police station, CID surmised, after having observed the facial features of the
man, that he was probably Taiwanese. CID then "recited and informed the man of
his constitutional rights" to remain silent, to have the assistance of a counsel, etc.
Eliciting no response from the man, CID ordered his men to find a resident of the
area who spoke Chinese to act as an interpreter. In the meantime, BADUA opened
the bag and counted twenty-nine (29) plastic packets containing yellowish
crystalline substances which he and CID suspected was shabu. The interpreter, Mr.
Go Ping Guan, finally arrived, through whom the man was "apprised of his
constitutional rights." The police authorities were satisfied that the man and the
interpreter perfectly understood each other despite their uncertainty as to what
language was spoken. But when the policemen asked the man several questions, he
retreated to his obstinate reticence and merely showed his I.D. with the name Chua
Ho San printed thereon. CHUA’s bag and its contents were sent to the PNP Crime
Laboratory at Camp Diego Silang, Carlatan, San Fernando, La Union for
laboratory examination. In the meantime, CHUA was detained at the Bacnotan
Police Station.

Later that same day, Police Chief Inspector and Forensic Chemist Theresa Ann
Bugayong Cid of the Philippine National Police, Region I, received a letter request
3 from CID — incidentally her husband — to conduct a laboratory examination of
twenty-nine (29) plastic packets placed inside a multicolored strawbag. In her
Chemistry Report No. D-025-95, 4 she stated that her qualitative examination
established the contents of the plastic packets, weighing 28.7 kilos, to be positive
of methamphetamine hydrochloride or shabu, a regulated
drug.chanroblesvirtuallawlibrary

CHUA was initially charged with illegal possession of methamphetamine


hydrochloride before the RTC which docketed the case as Criminal Case No. 4037.
However, pursuant to the recommendation of the Office of the Provincial
Prosecutor of San Fernando, La Union, that the facts of the case could support an
indictment for illegal transport of a regulated drug, the information was
subsequently amended to allege that CHUA "willfully, unlawfully and feloniously
transpor(ted) 28.7 kilos of [m]ethamphetamine [h]ydrochloride (shabu) without the
necessary permit or authority to transport the same" in violation of Section 15,
Article III of R.A. 6425 as amended by R.A. 7659.

At his arraignment on 31 July 1995, CHUA entered a plea of not guilty. The RTC
was satisfied that CHUA understood the amended information read to him in
Fukien by the Fukien-speaking interpreter, Thelma Sales Go.

Thereafter, the RTC exerted all efforts to obtain the services of a Taiwanese
Interpreter through the auspices of the Department of Foreign Affairs. However, it
was only after directing the request to the Taipei Economic and Cultural Office in
the Philippines that interpreters were assigned to CHUA.

Trial finally ensued. The State presented evidence tending to establish the above
narration of facts which were culled chiefly from the testimony of CID, its first
witness, and whose testimony, in turn, was substantially corroborated by witnesses
BADUA and ALMOITE.

Expert witness Theresa Ann Cid, confirmed the entries of her chemistry report in
that the contents of the 29 plastic packets weighing 28.7 kilos sent to her for
chemical analysis were pure, unadulterated methamphetamine hydrochloride or
shabu. She also explained that they were unwashed, hence they appeared
yellowish.

For the defense, CHUA testified in his own behalf through interpreter Steven Yu.
He disclosed that he hails from Taiwan and was employed in a shipbuilding and
repairing company. On 21 March 1995, he was instructed by his employer Cho
Chu Rong (hereafter RONG) to board the latter’s 35-tonner ship which would
embark for Nan Au Port, Mainland China where they would buy fish. Upon arrival
at their destination, RONG left the ship, came back without the fish, but with two
bags, the contents of which he never divulged to CHUA. RONG then showed to
CHUA a document purportedly granting them authority to fish on Philippine
waters. So they sailed towards the Philippines and reached Dagupan, Pangasinan
on 29 March 1995. At around 10:30 a.m., they disembarked on a small speedboat
with the two bags RONG brought with him from China. While sailing, RONG
made several phone calls using his mobile phone. CHUA heard RONG asked the
person on the other side of the line if he could see the speedboat they were riding.
Apparently, the person on shore could not see them so they cruised over the waters
for about five hours more when finally, low on fuel and telephone battery, they
decided to dock. CHUA anchored the boat while RONG carried the bags to shore.
The tasks completed, RONG left to look for a telephone while CHUA rested and
sat one and half (1 1/2) meters away from one bag. A child thereafter pointed out
to him that one bag was missing much to RONG’s dismay when he learned of it.
When a crowd started to mill around them, the police arrived. CHUA then realized
that RONG was nowhere to be found. The police immediately approached CHUA,
and with nary any spoken word, only gestures and hand movements, they escorted
him to the precinct where he was handcuffed and tied to a chair. Later, the police,
led by an officer who CHUA guessed as the Chief of Police arrived with the motor
engine of the speedboat and a bag. They presented the bag to him, opened it,
inspected and weighed the contents, then proclaimed them as methamphetamine
hydrochloride.

CHUA denounced the prosecution’s story as a distortion of the truth. He denied he


was ever favored with an interpreter or informed of his "constitutional rights,"
particularly of his right to counsel. Consequently, his arrest was tainted with
illegality and the methamphetamine hydrochloride found in the bag should have
been regarded inadmissible as evidence. He also maintained that CID never graced
the occasion of his setting foot for the first time at Tammocalao beach. BADUA
certainly never prevented him from running away, as such thought failed to make
an impression in his mind. Most significantly, he denied ownership and knowledge
of the contents of the bag, emphasizing that RONG alone exercised dominion over
the same.chanrobles virtual lawlibrary

Elmer Parong, (hereafter PARONG) a Sangguniang Bayan member, recalled that


on the date in question, he arrived at the beach with the police. He saw CHUA
standing with a bag beside him. He also remembered hearing from the people
congregating at the beach that CHUA arrived with a companion and a certain
policeman Anneb had chased the latter’s car. He additionally claimed that when
the crowd became unruly, the police decided to bring CHUA to police
headquarters. There, the mayor took charge of the situation — he opened CHUA’s
bag with the assistance of the police, he called for a forensic chemist surnamed
CID to take a sample of the contents of the bag, and he ordered his officials to find
an interpreter. Throughout the proceedings, photographers were busy taking
pictures to document the event.

Last to testify was Arsenio CRAIG, a farmer and resident of Tammocalao who
narrated that he was standing with CHUA on the beach when two men and a lady
arrived. They were about to get a bag situated near CHUA when they detected the
arrival of the local police. They quickly disappeared. CRAIG then noticed
ALMOITE and PARONG at the beach but not CID.

In a decision promulgated on 10 February 1997, the RTC found that the


prosecution successfully discharged its burden of proving that CHUA transported
28.7 kilos of methamphetamine hydrochloride without legal authority to do so.
Invoking People v. Tagliben 5 as authority, the RTC characterized the search as
incidental to a valid in flagrante delicto arrest, hence it allowed the admission of
the methamphetamine hydrochloride as corpus delicti. The RTC also noted the
futility of informing CHUA of his constitutional rights to remain silent, and to
have competent and independent counsel preferably of his own choice, considering
the language barrier and the observation that such irregularity was "rectified when
accused was duly arraigned and . . . (afterwards) participated in the trial of this
case." The RTC then disregarded the inconsistencies and contradictions in the
testimonies of the prosecution witnesses as these referred to minor details which
did not impair the credibility of the witnesses or tarnish the credence conferred on
the testimonies thus delivered.

The RTC also believed that CHUA conspired not only with his alleged employer
RONG and the Captain of the 35-tonner vessel in the illegal trade of prohibited
drugs on Philippine shores, but with several other members of an organized
syndicate bent on perpetrating said illicit traffic. Such predilection was plainly
evident in the dispositive portion, to wit:chanrob1es virtual 1aw library

WHEREFORE, and in view of all the foregoing, as proven and established by


convincing and satisfactory evidence that the accused had conspired and acted in
concert with one Cho Chu Rong, not to mention Chen Ho Fa, the Skipper of the
35-tonner ship they used in coming to the Country from China and Taiwan, this
Court finds the accused Chua Ho San @ Tsay Ho San guilty beyond reasonable
doubt of the offense of Violation of Sec. 15, Art. III of R.A. No. 6425, as amended
by R.A. No. 7659 as charged in the Information, and considering the provisions of
Sec. 20 of R.A. No. 7659 that the maximum penalty shall be imposed if the
quantity sold/possessed/transported is ‘200 grams or more’ in the case of Shabu,
and considering, further that the quantity involved in this case is 28.7 kilograms
which is far beyond the weight ceiling specified in said Act, coupled with the
findings of conspiracy or that accused is a member of an organized syndicated
crime group, this Court, having no other recourse but to impose the maximum
penalty to accused, this Court hereby sentences the said accused Chua Ho San @
Tsay Ho San to die by lethal injection; to pay a fine of Ten Million Pesos
(P10,000,000.00); and to pay the costs.

The Court hereby orders Director Ricareido [sic] Sarmiento of the Philippine
National Police to immediately form an investigating Committee to be composed
by [sic] men of unimpeachable integrity, who will conduct an exhaustive
investigation regarding this case to determine whether there was negligence or
conspiracy in the escape of Cho Chu Rong and the two (2) or three (3) persons
who approached the accused in the seashore of Tammocalao, Bacnotan, La Union,
and attempted to take the remaining bag from accused, as well as the whereabouts
of the other bag; and to furnish this Court a copy of the report/result of the said
investigation in order to show compliance herewith sixty (60) days from receipt
hereof.

The confiscated 28.7 kilograms of Methamphetamine Hydrochloride or Shabu is


ordered turned over immediately to the Dangerous Drugs Board for destruction in
accordance with the law.

The fiberglass boat with its motor engine is hereby ordered confiscated in favor of
the government and to be turned over to the Philippine National Police, La Union
Command, for use in their Bantay-Dagat operations against all illegal seaborne
activities.

SO ORDERED. 6

Before this Court, CHUA posits that the RTC erred in (1) admitting as competent
evidence the 29 plastic packets of methamphetamine hydrochloride since they were
indubitably "forbidden fruits;" (2) granting weight and credence to the testimonies
of prosecution witnesses despite glaring inconsistencies on material points; and in
(3) appreciating conspiracy between him and an organized syndicate in the illicit
commerce of prohibited drugs since this was not alleged in the
information.chanrobles lawlibrary : rednad

The Solicitor General traverses CHUA’s contentions by asserting that: (1) the
search was licitly conducted despite the absence of search and seizure warrants as
circumstances immediately preceding to and contemporaneous with the search
necessitated and validated the police action; and (2) that there was an effective and
valid waiver of CHUA’s right against unreasonable searches and seizures since he
consented to the search.

We reverse the RTC.

Enshrined in the Constitution is the inviolable right to privacy of home and person.
It explicitly ordains that people have the right to be secure in their persons, houses,
papers and effects against unreasonable searches and seizures of whatever nature
and for any purpose. 7 Inseparable, and not merely corollary or incidental to said
right and equally hallowed in and by the Constitution, is the exclusionary principle
which decrees that any evidence obtained in violation of said right is inadmissible
for any purpose in any proceeding. 8
The Constitutional proscription against unreasonable searches and seizures does
not, of course, forestall reasonable searches and seizure. What constitutes a
reasonable or even an unreasonable search in any particular case is purely a
judicial question, determinable from a consideration of the circumstances involved.
9 Verily, the rule is, the Constitution bars State intrusions to a person’s body,
personal effects or residence except if conducted by virtue of a valid search
warrant issued in compliance with the procedure outlined in the Constitution and
reiterated in the Rules of Court; "otherwise such search and seizure become
‘unreasonable’ within the meaning of the aforementioned constitutional provision."
10 This interdiction against warrantless searches and seizures, however, is not
absolute and such warrantless searches and seizures have long been deemed
permissible by jurisprudence 11 in instances of (1) search of moving vehicles, (2)
seizure in plain view, (3) customs searches, (4) waiver or consent searches, (5) stop
and frisk situations (Terry search), 12 and (6) search incidental to a lawful arrest.
The last includes a valid warrantless search and seizure pursuant to an equally
valid warrantless arrest, for, while as a rule, an arrest is considered legitimate if
effected with a valid warrant of arrest, the Rules of Court recognize permissible
warrantless arrests, to wit: (1) arrests in flagrante delicto, (2) arrests effected in hot
pursuit, and (3) arrests of escaped prisoners. 13

This Court is therefore tasked to determine whether the warrantless arrest, search
and seizure conducted under the facts of the case at bar constitute a valid
exemption from the warrant requirement. Expectedly and quite understandably, the
prosecution and the defense painted extremely divergent versions of the incident.
But this Court is certain that CHUA was arrested and his bag searched without the
benefit of a warrant.

In cases of in flagrante delicto arrests, a peace officer or a private person may


without a warrant, arrest a person, when, in his presence, the person to be arrested
has committed, is actually committing, or is attempting to commit an offense. The
arresting officer, therefore, must have personal knowledge of such fact 14 or as
recent case law 15 adverts to, personal knowledge of facts or circumstances
convincingly indicative or constitutive of probable cause. The term probable cause
had been understood to mean a reasonable ground of suspicion supported by
circumstances sufficiently strong in themselves to warrant a cautious man’s belief
that the person accused is guilty of the offense with which he is charged. 16
Specifically with respect to arrests, it is such facts and circumstances which would
lead a reasonably discreet and prudent man to believe that an offense has been
committed by the person sought to be arrested. 17 In People v. Montilla, 18 the
Court acknowledged that "the evidentiary measure for the propriety of filing
criminal charges, and correlatively, for effecting warrantless arrest, has been
reduced and liberalized." Noting that the previous statutory and jurisprudential
evidentiary standard was" prima facie evidence" and that it had been dubiously
equated with probable cause, the Court explained:chanroblesvirtual|awlibrary
[F]elicitously, those problems and confusing concepts (referring to prima facie
evidence and probable cause) were clarified and set aright, at least on the issue
under discussion, by the 1985 amendment of the Rules of Court which provides in
Rule 112 thereof that the quantum of evidence required in preliminary
investigation is such evidence as suffices to ‘engender as well founded belief’ as to
the fact of the commission of the crime and the respondent’s probable guilt thereof.
It has the same meaning as the related phraseology used in other parts of the same
Rule, that is, that the investigating fiscal ‘finds cause to hold the respondent for
trial,’ or where ‘a probable cause exists.’ It should, therefore, be in that sense,
wherein the right to effect a warrantless arrest should be considered as legally
authorized." (Emphasis supplied) 19

Guided by these principles, this Court finds that there are no facts on record
reasonably suggestive or demonstrative of CHUA’s participation in an ongoing
criminal enterprise that could have spurred police officers from conducting the
obtrusive search. The RTC never took the pains of pointing to such facts, but
predicated mainly its decision on the finding that "accused was caught red-handed
carrying the bagful of [s]habu when apprehended." In short, there is no probable
cause. At least in People v. Tangliben, the Court agreed with the lower court’s
finding that compelling reasons (e.g., Accused was acting suspiciously, on the spot
identification by an informant that accused was transporting prohibitive drug, and
the urgency of the situation) constitutive of probable cause impelled police officers
from effecting an in flagrante delicto arrest. In the case at bar, the Solicitor General
proposes that the following details are suggestive of probable cause — persistent
reports of rampant smuggling of firearm and other contraband articles, CHUA’s
watercraft differing in appearance from the usual fishing boats that commonly
cruise over the Bacnotan seas, CHUA’s illegal entry into the Philippines (he lacked
the necessary travel documents or visa), CHUA’s suspicious behavior, i.e. he
attempted to flee when he saw the police authorities, and the apparent ease by
which CHUA can return to and navigate his speedboat with immediate dispatch
towards the high seas, beyond the reach of Philippine laws.

This Court, however, finds that these do not constitute "probable cause." None of
the telltale clues, e.g., bag or package emanating the pungent odor of marijuana or
other prohibited drug, 20 confidential report and/or positive identification by
informers of courier(s) of prohibited drug and/or the time and place where they
will transport/deliver the same, 21 suspicious demeanor or behavior 22 and
suspicious bulge in the waist 23 — accepted by this Court as sufficient to justify a
warrantless arrest exists in this case. There was no classified information that a
foreigner would disembark at Tammocalao beach bearing prohibited drug on the
date in question. CHUA was not identified as a drug courier by a police informer
or agent. The fact that the vessel that ferried him to shore bore no resemblance to
the fishing boats of the area did not automatically mark him as in the process of
perpetrating an offense. And despite claims by CID and BADUA that CHUA
attempted to flee, ALMOITE testified that the latter was merely walking and
oblivious to any attempt at conversation when the officers approached him. This
cast serious doubt on the truthfulness of the claim, thus:chanrob1es virtual 1aw
library

Q How far were you when the accused put the bag on his shoulder?

A We were then very near him about three meters away from the male person
carrying the bag.

Q To what direction was he facing when he put the bag on his shoulder?

A To the east direction.

Q In relation to you, where were you.

A With the company of Sgt. Reynoso and Maj. Cid we approached the accused and
when Maj. Cid went near him, he spoke in Tagalog, English and Ilocano which
accused did not understand because he did not respond.

Q When Maj. Cid was talking, what was the accused doing at that time?

A He was walking.

Q To what direction he was walking?

A He was walking to the east direction. (sic)

Q He was walking away from you or going near you?

A He was going away from us. That is why Sgt. Reynoso held the right arm of the
accused.

Q Was Sgt. Badua able to hold the right arm of the accused?

A Yes sir and he stopped. 24

True, CHUA entered Philippine territory without a visa. This was not obvious to
the police. But gossamer to the officers’ sense perception and view were CHUA
disembarking from a speedboat, CHUA walking casually towards the road, and
CHUA carrying a multicolored strawbag. These acts did not convey any
impression that he illegally entered Philippine shores. Neither were these overt
manifestations of an ongoing felonious activity nor of CHUA’s criminal behavior
as clearly established in CID’s testimony, thus:chanrob1es virtual 1aw library

Q Was the accused committing a crime when you introduced


yourselves:chanrob1es virtual 1aw library

A No, sir.

Q No, so there was no reason for you to approach the accused because he was not
doing anything wrong?

A No, sir, that is our objective, to approach the person and if ever or whatever
assistance that we can give we will give.25cralaw:red

The search cannot therefore be denominated as incidental to an arrest. While a


contemporaneous search of a person arrested may be effected to deliver dangerous
weapons or proofs or implements used in the commission of the crime and which
search may extend to the area within his immediate control where he might gain
possession of a weapon or evidence he can destroy, 26 a valid arrest must precede
the search. The process cannot be reversed.

In a search incidental to a lawful arrest, as the precedent arrest determines the


validity of the incidental search, the legality of the arrest is questioned in a large
majority of these cases, e.g., whether an arrest was merely used as a pretext for
conducting a search. In this instance, the law requires that there be first a lawful
arrest before a search can be made - the process cannot be reversed. 27

To reiterate, the search was not incidental to an arrest. There was no warrant of
arrest and the warrantless arrest did not fall under the exemptions allowed by the
Rules of Court 28 as already shown. From all indications, the search was nothing
but a fishing expedition. It is worth mentioning here that after introducing
themselves, the police officers immediately inquired about the contents of the bag.
What else could have impelled the officers from displaying such inordinate interest
in the bag but to ferret out evidence and discover if a felony had indeed been
committed by CHUA — in effect to "retroactively establish probable cause and
validate an illegal search and seizure." chanrobles.com : virtual law library

The State then attempted to persuade this Court that there was a consented search,
a legitimate waiver of the constitutional guarantee against obtrusive searches. It is
fundamental, however, that to constitute a waiver, it must first appear that the right
exists; secondly, that the person involved had knowledge, actual or constructive, of
the existence of such a right; and lastly, that said person had an actual intention to
relinquish the right. 29 CHUA never exhibited that he knew, actually or
constructively of his right against unreasonable searches or that he intentionally
conceded the same. This can be inferred from the manner by which the search was
performed, thus:chanrob1es virtual 1aw library

Q Together with your Chief Investigator, what was the first thing that you did
when you approached him (CHUA)?

A We introduced ourselves as police officers, sir.

Q Okey, in the first place why did you introduce yourselves?

A That is normal practice in our part, sir.

x       x       x

Q If it is possible . Okey (sic) now, after introducing yourselves what did you do?

A He did not answer me and he did not utter any word,

Q When he did not utter any word. What else did he do?

A I asked again a question that if he can open his bag sir.

Q And did he understand your question when you requested him to open his bag?

A No, sir, there is no answer.

Q No answer?

A Yes, sir, no answer.

Q And when there was no answer what did you do next?

A I used sign language sir.

Q Will you demonstrate to this Honorable Court how you demonstrated that sign
language of opening the bag mr. (sic) witness?

A I pointed to the zipper of the bag and then made an action like this sir.

x       x       x

SHERIFF:chanrob1es virtual 1aw library

The witness demonstrating (sic) by pointing to the straw bag and then manifesting
a sign to open the zipper of the straw bag moving his right hand from left to right
or from the opening to the end of the zipper.

COURT:chanrob1es virtual 1aw library

From the start of the zipper where you open it up to the end of the zipper.

Witness:chanrob1es virtual 1aw library

Yes, sir, and then I made a motion like this.

(The witness repeating the motion described on record.)

COURT:chanrob1es virtual 1aw library

Did you open that personally?

WITNESS:chanrob1es virtual 1aw library

A No, your honor.

Q Now, mr. (sic) witness, why did you request the accused to open the bag?

A Because it is our duty also to inspect his belongings sir.

Q Why, why was it — no, I reform my question your honor. Is it normal procedure
for you to examine anybody or to request anybody to open his bag?

A The fact that he was a foreigner, sir, it is also our duty to inspect the baggage, it
is our routine duty of a police (sic), sir.

Q Is that the normal duty of a police officer to request a person to open his bag?

A yes, sir.

Q Okey, (sic) you did not ask the accused, mr. (sic) witness, to open his bag?

A No, sir.

Q But you simply requested him to open the nag?

A Yes, sir. 30

CHUA obviously failed to understand the events that overran and overwhelmed
him. The police officers already introduced themselves to CHUA in three
languages, but he remained completely deadpan. The police hence concluded that
CHUA failed to comprehend the three languages. When CHUA failed to respond
again to the police’s request to open the bag, they resorted to what they called
"sign language." They claimed that CHUA finally understood their hand motions
and gestures. This Court disagrees. If CHUA could not understand what was orally
articulated to him, how could he understand the police’s "sign language." More
importantly, it cannot logically be inferred from his alleged cognizance of the "sign
language" that he deliberately, intelligently, and consciously waived his right
against such an intrusive search. This Court is not unmindful of cases upholding
the validity of consented warrantless searches and seizure. But in these cases, the
police officers’ request to search personnel effects was orally articulated to the
accused and in such language that left no room for doubt that the latter fully
understood what was requested. In some instances, the accused even verbally
replied to the request demonstrating that he also understood the nature and
consequences of such request. 31

It was eventually discovered that the bag contained the regulated substance. But
this is a trifling matter. If evidence obtained during an illegal search even if tending
to confirm or actually confirming initial information or suspicion of felonious
activity is absolutely considered inadmissible for any purpose in any proceeding,
the same being the fruit of a poisonous tree 32 how much more of "forbidden
fruits" which did not confirm any initial suspicion of criminal enterprise as in this
case — because the police admitted that they never harbored any initial suspicion.
Casting aside the regulated substance as evidence, the remaining evidence on
record are insufficient, feeble and ineffectual to sustain CHUA’s
conviction.chanroblesvirtualawlibrary

Indeed, the likelihood of CHUA having actually transported methamphetamine


hydrochloride cannot be quickly dispelled. But the constitutional guarantee against
unreasonable searches and seizures cannot be so carelessly disregarded as
overzealous police officers are sometimes wont to do. Fealty to the Constitution
and the rights it guarantees should be paramount in their minds, otherwise their
good intentions will remain as such simply because they have blundered. "There
are those who say that . . .’the criminal is to go free because the constable has
blundered.’… In some cases this will undoubtedly be the result. But . . .’there is
another consideration — the imperative of judicial integrity.’. . . The criminal goes
free, if he must, but it is the law that sets him free. Nothing can destroy a
government more quickly than its failure to observe its own laws, or worse, its
disregard of the charter of its own existence." 33

As to the averred glaring inconsistencies in the testimonies of the prosecution


witnesses, this Court considers them trivial as they refer to insignificant details
which will not affect the outcome of the case. On a passing note, this Court calls
the attention of the trial court regarding its erroneous appreciation of conspiracy.
This aggravating circumstance is without question unsupported by the records.
Conspiracy was not included in the indictment nor raised in the pleadings or
proceedings of the trial court. It is also fundamental that conspiracy must be proven
just like any other criminal accusation, that is, independently and beyond
reasonable doubt. 34

WHEREFORE, for all the foregoing, the decision of the Regional Trial Court,
Branch 66, San Fernando, La Union in Criminal Case No. 4037 is hereby
REVERSED and SET ASIDE and accused-appellant CHUA HO SAN @ TSAY
HO SAN is hereby ACQUITTED of the crime charged, the evidence not being
sufficient to establish his guilt beyond reasonable doubt.

Costs de oficio.
People v. Cuizon, G.R. No. 109287, April 18, 1996
In deciding the case at bench, the Court reiterates doctrines on illegal searches and
seizures, and the requirements for a valid warrantless search incident to a valid
warrantless arrest. While the Court appreciates and encourages pro-active law
enforcement, it nonetheless upholds the sacredness of constitutional rights and
repeats the familiar maxim, "the end never justifies the means" .

This is an appeal from the Decision 1 dated January 5, 1993 (Criminal Case No.
92-0230) of the Regional Trial Court, Branch 116, 2 Pasay City finding appellants
guilty of violating Section 15 of R.A. 6425, otherwise known as the Dangerous
Drugs Act of 1972.

On March 10, 1992, an Information 3 was filed against the appellants charging
them as follows:jgc:chanrobles.com.ph

"That on or about February 21, 1992 in Pasay City, Philippines and within the
jurisdiction of this Honorable Court, the above-named accused, conspiring,
confederating and mutually helping one another, did then and there, willfully,
unlawfully and feloniously carry and transport into the country, without lawful
authority, 16 kilograms, more or less, of METHAMPHETAMINE
HYDROCHLORIDE, also popularly known as ‘SHABU’, a regulated drug.

CONTRARY TO LAW."cralaw virtua1aw library

Upon arraignment, appellant Antolin Cuizon, assisted by counsel de parte, pleaded


not guilty. During the arraignment of appellants Paul Lee and Steve Pua, the latter
translated the Information into Chinese-Cantonese for the understanding of
appellant Lee, who does not speak nor understand English, Pilipino or any other
Philippine dialect. Both of them, duly assisted by their counsel, also pleaded not
guilty. 4 Trial ensued and on January 5, 1993, the court a quo found appellants
guilty as charged and rendered the following disposition: 5

"WHEREFORE, Accused Antolin Cuizon y Ortega, Steve Pua y Clofas alias


Stephen Po y Uy or Tommy Sy, and Paul Lee y Wong, alias Paul Leung, are found
guilty beyond reasonable doubt of transporting, without legal authority,
methamphetamine hydrochloride, or ‘shabu’, a regulated drug, as charged in the
aforequoted Information; and they are each sentenced to suffer the penalty of life
imprisonment and to pay a fine of 20,000.00.

"The methamphetamine hydrochloride or ‘shabu’ involved in this case is declared


forfeited in favor of the government and is ordered turned over to the Dangerous
Drug Board for proper disposal."cralaw virtua1aw library

The Facts

According to the Prosecution

The facts as summarized by the trial court and adopted by the Solicitor General,
who added the page references to the transcript of stenographic notes as indicated
in brackets, are as follows: 6

"In January 1992, the Reaction Group of the National Bureau of Investigation
(NBI) gathered an information regarding the drug activities of accused Antolin
Cuizon y Ortega and his wife, Susan Cuizon. A surveillance was conducted on
them. The residence of the spouses was traced to Caloocan City (tsn, May 19,
1992, pp. 17-18, 21).

"In the morning of February 21, 1992, the Reaction Group received a report from
its informant in Hong Kong that accused Cuizon, together with his wife, was
arriving on the same day at the Ninoy Aquino International Airport (NAIA) in
Pasay City, Metro Manila, from the British crown colony, carrying with him a big
quantity of ‘shabu’. A team was organized to intercept the suspects. Heading the
team was Jose Yap, with Ernesto Diño, Marcelino Amurao, Jose Bataller and
Alfredo Jacinto, as members. Some belonged to the Narcotics Division and the
others to the Reaction Group of the NBI (tsn, May 19, 1992, pp. 4, 18).

"Arriving at the NAIA shortly before 12:00 noon of February 21, 1992, Diño
positioned himself at the Arrival Area, while Yap and the other members of the
team posted themselves at the parking area of the airport. At about 12:45 in the
afternoon of the same date, Accused Cuizon and his wife, who had just returned
from Hong Kong, after passing through the Immigration and Customs Areas at the
NAIA, proceeded to the Arrival Area of the airport preparatory to their boarding a
car. While there, Accused Cuizon, together with his wife, handed four (4)
travelling bags to accused Steve Pua y Clofas and accused Paul Lee y Wong, who
were at the vicinity of the Arrival Area. Accused Pua and Lee loaded the bags in a
taxicab which they boarded in leaving the airport. Accused Cuizon and his wife
took another vehicle (tsn, May 19, 1992, pp. 4-5, 8-9).

"At this juncture, Diño, who was observing the activities of the accused, radioed
the group of Yap at the parking area, describing the vehicle boarded by accused
Pua and Lee so that Yap and his companions could apprehend the two. However,
the message of Diño was not completely received by his teammates as the radio he
was using ran short of battery power (tsn, May 19, 1992, pp. 25-26).

"Immediately after the vehicle boarded by Pua and Lee had left, Diño proceeded to
the place where his companions were stationed for the purpose of giving assistance
to them, believing that they were already in the process of apprehending accused
Pua and Lee. When he realized that the two accused were not apprehended, Diño
told the group of Yap to follow him as he was following the vehicle taken by Pua
and Lee which, according to an earlier tip he learned, was proceeding to the Manila
Peninsula Hotel in Makati, Metro Manila (tsn, May 19, 1992 pp. 25-26; tsn, May
21, 1992 pp. 6, 15).

"Upon arriving at about 2:00 p.m. of the same date of February 21, 1992, in the
Manila Peninsula Hotel, in whose premises the taxicab boarded by accused Pua
and Lee entered, Diño and the other members of the team coordinated with Col.
Regino Arellano, Chief Security Officer of the hotel, for the purpose of
apprehending the two accused. A verification made by the Chief Security Officer
showed that accused Pua and Lee occupied Room 340 of the hotel. The two
accused allowed Diño and Yap, together with Col. Arellano, to enter their room.
Found inside Room 340 were four (4) travelling bags, which were similar to the
ones handed by accused Cuizon to accused Pua and Lee at the Arrival Area of the
NAIA. After having introduced themselves as NBI agents, Diño and Yap were
permitted by accused Pua and Lee to search their bags in the presence of Col.
Arellano. The permission was made in writing.(Exh. I). Three (3) of the four (4)
bags each yielded a plastic package containing a considerable quantity of white
crystalline substance suspected to be methamphetamine hydrochloride or ‘shabu’.
Each package was sandwiched between two (2) pieces of board which appear to be
‘lawanit’ placed at the bottom of each of the three (3) bags. The suspected ‘shabu’
contained in one bag weighed 2.571 kilos, that found in the other had a weight of
2.768 kilos, and the suspected ‘shabu’ retrieved from the third bag weighed 2.970
kilos. Pua and Lee were then apprehended by Diño and his companions (tsn, May
20, 1992, pp. 9-13; tsn, May 7, 1992, p. 9, Exh. "F-2", p. 75, Records.

"Immediately thereafter, Diño and the other members of the team proceeded to the
house of accused Cuizon in Caloocan City, taking with them accused Pua and Lee
and the bags with their contents of suspected dangerous drugs. They reached the
place at about 5:50 in the afternoon of the same date of February 21, 1992.
Retrieved from accused Cuizon in his residence was another bag also containing a
white crystalline substance weighing 2.695 kilos, likewise believed to be
methamphetamine hydrochloride or ‘shabu’. In addition, a .38 Cal. firearm was
taken from accused Cuizon (tsn, May 19, 1992, pp. 10-11).

"Pua, Lee, Cuizon and his wife were then brought by the arresting officers to the
NBI headquarters at Taft Avenue, Manila, for further investigation. They were
subsequently referred to the Prosecution Division of the Department of Justice for
inquest. However, only the present three accused were charged in court (tsn, May
19, 1992, pp. 12-13, 16-17).

"In the meantime, at about 5:30 p.m. of the same date of February 21, 1992,
Joselito Soriano, roomboy of the Manila Peninsula Hotel, while cleaning Room
340, observed that a portion of the ceiling was misaligned. While fixing it, he
discovered in the ceiling a laundry bag containing suspected ‘shabu’ of more than
five (5) kilos (Exh.’X,’ p. 110). Informed of the discovery while they were already
in their office in the NBI, Yap and some companions returned to the hotel. The
suspected ‘shabu’ was turned over to them (tsn, May 20, 1992, pp. 19-22).

"When examined in the Forensic Chemistry Section of the NBI, the white
crystalline substance taken from the three (3) travelling bags found in the room of
accused Pua and Lee in the Manila Peninsula Hotel, the white crystalline substance
retrieved from the bag confiscated from accused Cuizon in his house in Caloocan
City, and the white crystalline substance hidden in the ceiling of Room 340 of the
hotel were confirmed to be methamphetamine hydrochloride or ‘shabu’, a
regulated drug. (Board Regulation No. 6, dated December 11, 1972, of the
Dangerous Drugs Board) (tsn, May 7, 1992, p. 12)."cralaw virtua1aw library

The Defense’s Version(s)

Appellant Pua, on his part, interposed the defense of alibi. On direct examination,
he testified that at the time of the alleged commission of the offense, he and his co-
appellant Lee were in their room at the Manila Peninsula Hotel. 7 His version of
what happened on February 21, 1992 can be summarized as follows:chanrob1es
virtual 1aw library

At around 9:30 in the morning, he accompanied appellant Paul Lee to check-in at


the Manila Peninsula Hotel for and in behalf of the latter’s personal friend named
Leong Chong Chong or Paul Leung, who was expected to arrive that evening
because of a delayed flight. Appellant Pua was engaged by appellant Lee to act as
interpreter as Lee does not know how to speak English and the local language. 8

While in Room 340, past 1:00 in the afternoon, they received a call from the lobby
informing them of the arrival of Paul Leung’s luggage. At Pua’s instructions, the
said luggage were brought to the room by a bellboy. Thereafter, two persons
knocked on their door, accompanied by a "tomboy" and a thin man with curly hair.
The two men identified themselves as NBI agents and asked appellant Pua to let
them in. He declined since he did not know who they were. However, when Col.
Arellano, the Chief Security Officer of the hotel, arrived and identified the two
NBI agents, he and Lee relented and permitted them to enter. Thereafter, he and
Lee were told by the agents to sign a piece of paper. Made to understand that they
were merely giving their consent for the agents to enter their room, Pua and Lee
signed the same. Whereupon, the agents told them that they will open Paul Leung’s
bags. Again appellant Pua refused, saying that the bags did not belong to them. Just
the same, the agents, without appellants Pua and Lee’s consent, opened the bags
and found the shabu. Pua and Lee were then apprehended and brought to the NBI
headquarters. 9

Appellant Cuizon, on the other hand, flatly rejected the prosecution’s version of
the incident. While admitting that on February 21, 1992, he and his wife Susan did
arrive from Hong Kong with several pieces of luggage, he denied that he met Pua
and Lee at the arrival area of the airport, much less passed to them the four pieces
of luggage. According to him, only his two-year old son, accompanied by his
cousin, Ronald Allan Ong, met them outside the airport. Ong fetched them from
the airport and brought them to their home in Caloocan City. They arrived at their
house around 3:00 in the afternoon. 10

About two hours later, while he was resting together with his wife and son on his
bed, two NBI agents suddenly barged in and poked a gun at him. They manhandled
him in front of his wife and son. His hands were tied with a necktie and he was
forcibly brought out of their house while the NBI agents ransacked the place
without any warrant. He, his wife Susan, and his cousin Ronald Allan Ong, were
afterwards brought to the NBI Headquarters in Manila and there the NBI agents
continued mauling him. 11

Appellant Cuizon’s wife Susan, his cousin Ronald Allan Ong, and his nephew
Nestor Dalde, testified in his favor basically reiterating or confirming his
testimony. 12

Unfortunately, appellant Paul Lee, who does not speak or understand a word of
English or Pilipino and only knows Chinese-Cantonese, was not able to take the
witness stand for lack of an interpreter who would translate his testimony to
English. In the hearing set on October 28, 1992, the last trial date allotted to the
defense for the reception of Lee’s testimony, his counsel, although notified of the
proceedings, did not appear. Thus, the trial court deemed him and Pua to have
waived their right to present additional evidence, 13 and the case was considered
submitted for decision after the filing of memoranda. The counsel for Pua and Lee
did not ask for the reconsideration of such ruling; neither did he submit any
memorandum. Only accused Cuizon, who was assisted by another counsel, was
able to submit his memorandum.

The Issues

In their brief, appellants Pua and Lee made the following assignments of errors: 14

"I. The trial court erred in finding conspiracy among the accused.

"II. The trial court erred in giving credence to the testimonies of prosecution
witnesses Marcelino Amurao, Jose Yap and Ernesto Diño despite contradictions
made on material points.

"III. The trial court erred in not giving accused Paul Lee the opportunity to present
his evidence in his defense in violation of his constitutional right to due
process."cralaw virtua1aw library

Appellant Cuizon, in a separate brief, essentially reiterates the first two


assignments of errors above-quoted, and in addition challenges the legality and
validity of his warrantless arrest and the search and seizure incidental thereto. 15

As this Court sees it, the resolution of this case hinges on the pivotal question of
the legality of the arrest and search of herein appellants effected by the NBI
operatives. Put differently, were the warrantless arrests and the warrantless
searches conducted by the NBI legal and constitutional?

The answer to this threshold question determines whether the judgment of the
court a quo will stand or fall. Consequently, there is a need to resolve first this
issue before endeavoring to consider the other issues raised by appellants.

A necessary side issue to be considered is, assuming the searches and arrests to
have been illegal, whether failure by appellants Pua and Lee to explicitly assign the
same as errors before this Court amounted to a waiver of their constitutional rights
against such illegal searches and arrests.

The Court’s Ruling

General Rule on Warrantless

Arrests, Searches, & Seizures

Well entrenched in this country is the rule that no arrest, search and seizure can be
made without a valid warrant issued by a competent judicial authority. So sacred is
this right that no less than the fundamental law of the land 16 ordains
it:jgc:chanrobles.com.ph

"The right of the people to be secure in their persons, houses, papers and effects
against unreasonable searches and seizures of whatever nature and for any purpose,
shall be inviolable, and no search warrant or warrant of arrest shall issue except
upon probable cause to be determined personally by the judge after examination
under oath or affirmation of the complainant and the witnesses he may produce,
and particularly describing the place to be searched, and the persons or things to be
seized."cralaw virtua1aw library

It further decrees that any evidence obtained in violation of said right shall be
inadmissible for any purpose in any proceeding. 17

However, the right against warrantless arrest and search and seizure is not absolute.
Thus, under Section 5 of Rule 113 of the Revised Rules of Court, an arrest without
a warrant may be lawfully made by a peace officer or a private
person:jgc:chanrobles.com.ph

"a) When, in his presence, the person to be arrested has committed, is actually
committing, or is attempting to commit an offense;

"(b) When an offense has in fact just been committed, and he has personal
knowledge of facts indicating that the person to be arrested has committed it; and

"(c) When the person to be arrested is a prisoner who has escaped from a penal
establishment or place where he is serving final judgment or temporarily confined
while his case is pending, or has escaped while being transferred from one
confinement to another."cralaw virtua1aw library

On the occasion of any of the aforementioned instances of legitimate arrest without


warrant, the person arrested may be subjected to a search of his body and of his
personal effects or belongings, "for dangerous weapons or anything which may be
used as proof of the commission of an offense," likewise without need of a search
warrant. 18

However, where a person is searched without a warrant, and under circumstances


other than those justifying a warrantless arrest, as discussed above, upon a mere
suspicion that he has embarked on some criminal activity, and/or for the purpose of
discovering if indeed a crime has been committed by him, then the search made of
such person as well as his arrest are deemed illegal. 19 Consequently, any evidence
which may have been obtained during such search, even if tending to confirm or
actually confirming such initial suspicion, is absolutely inadmissible for any
purpose and in any proceeding, 20 the same being "the fruit of the poisonous tree."
21 Emphasis is to be laid on the fact that the law requires that the search be
incident to a lawful arrest, in order that the search itself may likewise be
considered legal. Therefore, it is beyond cavil that a lawful arrest must precede the
search of a person and his belongings. Were a search first undertaken, then an
arrest effected based on evidence produced by the search, both such search and
arrest would be unlawful, for being contrary to law.

The Instant Case Does Not Fall Under

The Exceptions for Warrantless Searches, etc.

Re-assessing the factual backdrop of the case at bench, this Court cannot agree
with and accept the conclusion of the trial court that the appellants were caught in
flagrante delicto which would justify the search without a warrant. The shaky
reasoning of the court a quo gives away the baselessness of its findings and
conclusion:jgc:chanrobles.com.ph

". . . the search conducted on their bags in the hotel room could still be regarded as
valid for being incidental to a lawful arrest. . . . The arrest of accused Pua and Lee
without a warrant of arrest was lawful, as they could be considered to have
committed the crime of transporting ‘shabu’ in the presence of the arresting
officers from the time they received the bags containing the regulated drug in the
airport up to the time they brought the bags to the hotel. Or their arrest without a
warrant was legal as falling under the situation where an offense had in fact just
been committed, and the arresting officers had personal knowledge of facts
indicating that the said accused were the ones who committed it. . . ." 22

Scrutinizing the provisions of Sec. 5 of Rule 113 of the Rules of Court on lawful
arrests without warrant, we note that par. (c) of said section is obviously
inapplicable, the appellants not being escapees from a penal institution at the time
of arrest. Par. (a) on the other hand requires that the person be arrested (i) after he
has committed or while he is actually committing or is at least attempting to
commit an offense, (ii) in the presence of the arresting officer(s). These
requirements are not present in the case at bench, for at the time of their arrest,
appellants Pua and Lee were merely resting in their hotel room, and appellant
Cuizon for his part was in bed resting with his wife and child inside his home. No
offense had just been committed, or was being actually committed or being
attempted by any of the accused in the presence of the lawmen. 23

Par. (b) Or the same provision is likewise inapplicable since its equally exacting
requirements have also not been met. The prosecution failed to establish that at the
time of the arrest, an offense had in fact just been committed and the arresting
officers had personal knowledge of facts indicating that the accused-appellants had
committed it. Appellant Cuizon could not, by the mere act of handing over four
pieces of luggage to the other two appellants, be considered to have committed the
offense of "carrying and transporting" prohibited drugs. Under the circumstances
of the case, there was no sufficient probable cause for the arresting officers to
believe that the accused were then and there committing a crime. The act per se of
handing over the baggage, assuming the prosecution’s version to be true, cannot in
any way be considered a criminal act. It was not even an act performed under
suspicious circumstances as indeed, it took place in broad daylight, practically at
high noon, and out in the open, in full view of the public. 24 Furthermore, it can
hardly be considered unusual, in an airport setting, for travellers and/or their
welcomers to be passing, handing over and delivering pieces of baggage,
especially considering the somewhat obsessive penchant of our fellow countrymen
for sending along ("pakikipadala") things and gifts through friends and relatives.
Moreover, one cannot determine from the external appearance of the luggage that
they contained "shabu" hidden beneath some secret panel or false bottom. The only
reason why such act of parting with luggage took on the color and dimensions of a
felonious deed, at least as far as the lawmen were concerned, was the alleged tip
that the NBI agents purportedly received that morning, to the effect that appellant
Cuizon would be arriving that same day with a shipment of shabu. To quote from
another decision of like import," (A)ll they had was hearsay information (from the
telephone caller), and about a crime that had yet to be committed."25cralaw:red

In the leading case of People v. Burgos, 26 this Court laid down clear guidelines,
as follows:jgc:chanrobles.com.ph

"Under Section 6(a) of Rule 113, the officer arresting a person who has just
committed, is committing, or is about to commit an offense must have personal
knowledge of that fact. The offense must also be committed in his presence or
within his view. (Sayo v. Chief of Police, 80 Phil. 859)."cralaw virtua1aw library

The same decision is highly instructive as it goes on to state:jgc:chanrobles.com.ph

"The Solicitor General is of the persuasion that the arrest may still be considered
lawful under Section 6(b) using the test of reasonableness. He submits that the
information given by Cesar Masamlok was sufficient to induce a reasonable
ground (for belief) that a crime has been commit-ted and that the accused is
probably guilty thereof.

"In arrests without a warrant under Section 6(b), however, it is not enough that
there is reasonable ground to believe that the person to be arrested has committed a
crime. A crime must in fact or actually have been committed first. That a crime has
actually been committed is an essential precondition. It is not enough to suspect
that a crime may have been committed. The fact of the commission of the offense
must be undisputed. The test of reasonable ground applies only to the identity of
the perpetrator.
"In this case, the accused was arrested on the sole basis of Masamlok’s verbal
report. Masamlok led the authorities to suspect that the accused had committed
crime. They were still fishing for evidence of a crime not yet ascertained. The
subsequent recovery of the subject firearm on the basis of information from the lips
of a frightened wife cannot make the arrest lawful. . . ."cralaw virtua1aw library

The foregoing doctrine was affirmed in the case of Alih v. Castro, 27 where this
Court ruled that." . . under the Revised Rule 113, Section 5(b), the officer making
the arrest must have personal knowledge of the ground therefor as stressed in the
recent case of People v. Burgos."cralaw virtua1aw library

In the case at bench, not only did the NBI agents rely merely on hearsay
information ("tips"), but they were completely uncertain that anything was really
"going down" that day. That much is undisputed, from a reading of the testimony
of Agent Diño:jgc:chanrobles.com.ph

"Q Now, but you were informed by the personnel of the airport that the spouses
Cuizon were going to bring in or transport into the country shabu on February 21,
1992?

A Yes, sir.

Q Now, you were not sure or your group was not sure that they indeed would bring
in shabu, is it not? That was only the information relayed to your group?

A Yes, sir.

x       x       x

Q But then you were jumping ahead. You were not sure is it not that they were
bringing in shabu?

A Yes, sir." (TSN, May 19, 1992, pp. 37-38.)

In his testimony, NBI Investigator Jose Justo Yap, who was with Agent Diño
during the operation, likewise admitted in substantially the same tenor their
uncertainty regarding the commission of the offense (cf . TSN, May 20, 1992, pp.
29 & 34.).

We therefore hold that under the circumstances obtaining, the prosecution failed to
establish that there was sufficient and reasonable ground for the NBI agents to
believe that appellants had committed a crime at the point when the search and
arrest of Pua and Lee were made; hence, said search and arrest do not come under
the exception in par. (b) of Sec. 5 of Rule 113, and therefore should be deemed
illegal. We might add that the search conducted on Pua and Lee was not incident to
a lawful warrantless arrest, having preceded the same and produced the
justification therefor. On the other hand, the search on Cuizon’s residence, without
the benefit of a search warrant, was clearly illegal and the ‘shabu’ seized thereat
cannot but be considered inadmissible in evidence. More on these points later.

Comparison Between The Present Case

and Earlier Decisions of This Court

For clarity’s sake, it is imperative to compare the foregoing holding with previous
decisions by this Court in various drug cases, in which apparently different
conclusions were reached, in order to distinguish them from the instant case and
avoid any potential misunderstanding of the foregoing holding as well as the
constitutional and legal principles on which it is based.

1. In People v. Claudio, 28 the accused, a passenger on a bus bound for Baguio


City, was arrested by a policeman on the same bus because of the distinctive odor
of marijuana emanating from the plastic bag she was carrying. The Court held the
warrantless arrest under the circumstances to be lawful, the search justified and the
evidence thus discovered admissible in evidence.

2. In People v. Tangliben, 29 the accused, carrying a travelling bag at a bus


terminal, was noticed by lawmen to be acting suspiciously, and was also positively
fingered by an informer as carrying marijuana, and so he was accosted by
policemen who happened to be on a surveillance mission; the lawmen asked him to
open the bag, in which was found a package of marijuana leaves. It was held that
there was a valid warrantless arrest and search incident thereto. The Court in effect
considered the evidence on hand sufficient to have enabled the law enforcers to
secure a search warrant had there been time, but as the case "presented urgency,"
and there was actually no time to obtain a warrant since the accused was about to
board a bus, and inasmuch as an informer had given information "on the spot" that
the accused was carrying marijuana, the search of his person and effects was thus
considered valid.

3. In Posadas v. Court of Appeals, 30 the accused was seen acting suspiciously,


and when accosted by two members of the Davao INP who identified themselves
as lawmen, he suddenly fled, but was pursued, subdued and placed in custody. The
buri bag he was carrying yielded an unlicensed revolver, live ammunition and a
tear gas grenade. This Court upheld his conviction for illegal possession of
firearms, holding that there was under the circumstances sufficient probable cause
for a warrantless search.
4. In People v. Moises Maspil, Jr., Et Al., 31 agents of the Narcotics Command set
up a checkpoint on a highway in Atok, Benguet, to screen vehicular traffic on the
way to Baguio City due to confidential reports from informers that Maspil and a
certain Bagking would be transporting a large quantity of marijuana. At about 2
a.m. of November 1, 1986, the two suspects, riding a jeepney, pulled up to the
checkpoint and were made to stop. The officers noticed that the vehicle was loaded
with some sacks and tin cans, which, when opened, were seen to contain marijuana
leaves. The Court upheld the search thus conducted as being incidental to a valid
warrantless arrest.

5. In People v. Lo Ho Wing, Et Al., 32 the Court ruled that the search of the
appellants’ moving vehicles and the seizure of ‘shabu’ therefrom was legal, in
view of the intelligence information, including notably, clandestine reports by a
planted deep penetration agent or spy who was even participating in the drug
smuggling activities of the syndicate, to the effect that appellants were bringing in
prohibited drugs into the country. The Court also held that it is not practicable to
secure a search warrant in cases of smuggling with the use of a moving vehicle to
transport contraband, because the vehicle can be quickly moved out of the locality
or jurisdiction in which the warrant must be sought.

6. In People v. Malmstedt, 33 NARCOM agents stationed at Camp Dangwa,


Mountain Province, set up a temporary checkpoint to check vehicles coming from
the Cordillera Region, due to persistent reports that vehicles from Sagada were
transporting marijuana and other drugs, and because of particular information to
the effect that a Caucasian would be travelling from Sagada that day with
prohibited drugs. The bus in which accused was riding was stopped at the
checkpoint. While conducting an inspection, one of the NARCOM men noticed
that accused, the only foreigner on board, had a bulge at the waist area. Thinking it
might be a gun, the officer sought accused’s passport or other identification papers.
When the latter failed to comply, the lawman directed him to bring out whatever it
was that was bulging at his waist. It was a pouch bag which, when opened by the
accused, was found to contain packages of hashish, a derivative of marijuana.
Invited for questioning, the accused disembarked from the bus and brought along
with him two pieces of luggage; found inside were two teddy bears stuffed with
more hashish. The Court held that there was sufficient probable cause in the
premises for the lawmen to believe that the accused was then and there committing
a crime and/or trying to hide something illegal from the authorities. Said probable
cause arose not only from the persistent reports of the transport of prohibited drugs
from Sagada, and the "tip" received by the NARCOM that same day that a
Caucasian coming from Sagada would be bringing prohibited drugs, but also from
the failure of the accused to present his passport or other identification papers
when confronted by the lawmen, which only triggered suspicion on the part of the
law enforcers that accused was trying to hide his identity, it being the normal thing
expected of an innocent man with nothing to hide, that he readily present
identification papers when asked to do so. The warrantless arrest and search were
thus justified.

In all the cases discussed hereinabove, there were facts which were found by the
Court to provide probable cause justifying warrantless arrests and searches, i.e.,
distinct odor of marijuana, reports about drug transporting or positive identification
by informers, suspicious behaviour, attempt to flee, failure to produce
identification papers, and so on. Too, urgency attended the arrests and searches
because each of the above-mentioned cases involved the use of motor vehicles and
the great likelihood that the accused would get away long before a warrant can be
procured. And, lest it be overlooked, unlike in the case before us now, the law
enforcers in the aforementioned cases acted immediately on the information
received, suspicions raised, and probable causes established, and effected the
arrests and searches without any delay.

Unexplained Matters in the Instant Case

In the case before us, the NBI agents testified that they purportedly decided against
arresting the accused-appellants inside the airport as they allegedly wanted to
discover the identities of the airport immigration, security or customs personnel
who might be protecting the accused or otherwise involved in the drug smuggling
activities, and also in order to avoid the possibility of an armed encounter with
such protectors, which might result in injuries to innocent bystanders. These
excuses are simply unacceptable. They are obviously after-thoughts concocted to
justify their rank failure to effect the arrest within constitutional limits. Indeed, the
NBI men failed to explain how come they did not apprehend the appellants at the
moment Cuizon handed over the baggage to Pua and Lee, or even afterwards, in
relative safety. Such arrest would have been consistent with the settled
constitutional, legal and jurisprudential precedents earlier cited.

The spouses Cuizon had already passed through the airport security checks
allegedly with their contraband cargo undetected in their luggage. Apparently, the
NBI agents did not see (as indeed they did not testify that they saw) anyone from
the airport immigration, security or customs who could have escorted the spouses
Cuizon, and therefore, there was no danger of any "live ammo encounter" with
such group(s). The alleged drug couriers had already made their way outside the
NAIA, had allegedly made contact with the accused Pua and Lee, and were in the
very act of handing over the luggage to the latter. Why the NBI men did not move
in and pounce on them at that very instant has not been satisfactorily explained.
Instead one of the agents, Diño, merely watched as Pua and Lee loaded the luggage
into a cab and took off for Makati. Furthermore, it taxes the imagination too much
to think that at the most critical and climactic moment, when agent Diño radioed
his companions for help to close in on the suspects, the most amazing and
stupendous thing actually happened: Murphy’s Lay Kicked in — whatever could
go wrong, did, and at the worst possible time — the batteries in Agent Diño’s
hand-held radio supposedly went dead and his message was not transmitted. Thus
the departing Pua and Lee proceeded merrily and unimpeded to the Peninsula
Hotel, while the spouses Cuizon simultaneously sped off to their residence in
Caloocan City, leaving the lawmen empty-handed and scampering madly to catch
up. Such absolutely astounding and incredible happenstance might find a place in a
fourth-rate movie script, but expecting the courts to swallow it — hook, line and
sinker — is infinite naivete, if not downright malevolence.

Even granting arguendo that the radio really went dead, nevertheless, the agents
were not thereby rendered helpless or without recourse. The NBI agents,
numbering five in all, not counting their so-called informant, claimed to have piled
into three cars (TSN, May 19, 1992) and tailed the suspects Pua and Lee into
Makati, keeping a safe two-car distance behind (TSN, May 20, 1992). The lawmen
and the prosecutors failed to explain why the agents did not intercept the vehicle in
which Pua and Lee were riding, along the way, pull them over, arrest them and
search the luggage. And since the agents were in three (3) cars, they also could
have easily arranged to have agents in one vehicle follow, intercept and apprehend
the Cuizons while the others went after Pua and Lee. All or any of these possible
moves are mere ordinary, common-sense steps, not requiring a great deal of
intelligence. The NBI men who testified claimed to have conducted or participated
in previous drug busts or similar operations and therefore must have been familiar
with contingency planning, or at least should have known what to do in this
situation where their alleged original plan fell through. At any rate, what the
lawmen opted to do, i.e., allow Pua and Lee to freely leave the airport, allegedly
bringing the drug cache to the hotel, and Cuizon to leave unimpededly the airport
and reach his residence with one of the luggage, increased significantly the risk of
the suspects (and/or the drugs) slipping through the lawmen’s fingers, and puts into
question the regularity of performance of their official functions. The agents’
alleged actions in this case compare poorly with the forthright and decisive steps
taken by lawmen in the cases earlier cited where this Court held the arrests and
seizures to be valid.

Had the arrests and searches been made in transitu, i.e., had the agents intercepted
and collared the suspects on the way to Makati and Caloocan, or better yet, at the
very moment of the hand-over, then there would not have been any question at all
as to the legality of their arrest and search, as they would presumably have been
caught red-handed with the evidence, and consequently for that reason and by the
very nature and manner of commission of the offense charged, there would have
been no doubt also as to the existence of conspiracy among the appellant to
transport the drugs. However, because of the way the operation actually turned out,
there is no sufficient proof of conspiracy between Pua and Lee on the one hand,
and Cuizon on the other, inasmuch as there is no clear and convincing evidence
that the four (4) bags handed by Cuizon to Pua and Lee at the airport were the very
same ones found in the possession of the latter in Room 340 of the Peninsula
Hotel. Not one of the NBI agents when testifying could definitely and positively
state that the bags seized from Room 340 were the very same ones passed by
Cuizon at the airport; at best, they could only say that they "looked like" the ones
they saw at the airport. And even assuming them to be the same bags, there
remains doubt and uncertainty as to the actual ownership of the said bags as at the
alleged turnover vis-a-vis the time they were seized by the agents. For these
reason, we cannot sustain the finding of conspiracy as between Cuizon on the one
hand and Pua and Lee on the other. Well-settled is the rule that conspiracy must be
prove independently and beyond reasonable doubt. 34

Additionally, in light of the foregoing discussion, we find it extremely difficult to


subscribe to the trial court’s finding as to the existence and sufficiency of probable
cause in this case, one major component of which would have been the alleged
information or "tip" purportedly received by the agents as to the expected arrival of
the spouses Cuizon that fateful day with a large cache of ‘shabu’. The question that
defies resolution in our minds is why, if indeed the information or "tip" was
genuine and from a highly reliable source as claimed by the government agents,
did they not act on it? Throw in the alleged month-long surveillance supposedly
conducted by some of the NBI people on the Cuizon couple, and the mystery only
deepens. Even with the so-called tip and the results of surveillance, the government
officers were still seemingly hesitant, reluctant, uncertain, or perhaps afraid, to
arrest and search the accused appellants, so much so that the NBI agents who went
after Pua and Lee at the Peninsula Hotel, instead of outrightly cuffing and
searching them, as they were supposed to, opted instead to play it safe and meekly
beseeched the two to sign a written consent for the agents to search their personal
effects! Indeed, this is one for the books. If this is how confident the agents were
about their "hot tips", reliable informers and undercover surveillance, then we
cannot be blamed for failing to appreciate the existence/sufficiency of probable
cause to justify a warrantless arrest and search in this case. There is a whole lot
more that can be said on this score, but we shall leave it at that for now. We shall
now dispose of the appeals of the accused-appellants individually.

Re: Appellant Antolin Cuizon

The search of the house of appellant Cuizon, having been conducted without any
warrant, and not on the occasion or as an incident of a valid warrantless arrest, was
indubitably illegal, and the shabu seized thereat could not be admissible in
evidence. That is why even the trial judge did not make an effort to hold him liable
under such seizure. He lamely argued:" (A)t any rate, Accused Cuizon is not held
criminally liable in this case in connection with the bag containing ‘shabu’
confiscated from his residence. His responsibility is based on the bags containing
‘shabu’ which he handed to Pua and Lee at the NAIA. Consequently, even if the
bag and its contents of ‘shabu’ taken from his house were not admitted in evidence,
the remaining proofs of the prosecution would still be sufficient to establish the
charge against him." However, contrary to the trial judge’s conclusion, we hold
that insofar as Cuizon is concerned, all the evidence seized are considered fruit of
the poisonous tree and are inadmissible as against him, and thus, he should be
acquitted, since, as shown hereinabove, (i) the warrantless search conducted on Pua
and Lee was clearly illegal per se, not being incident to a valid warrantless arrest
either; (ii) and even if the search on Pua and Lee were not illegal, conspiracy as
between Cuizon on the one hand and appellants Pua and Lee on the other had not
been established by sufficient proof beyond reasonable doubt; and (iii) appellant
Cuizon had timely raised before this Court the issue of the illegality of his own
arrest and the search and seizure conducted at his residence, and questioned the
admission of the seized shabu in evidence.

Re: Appellant Steve Pua @ "Tommy Sy"

What has been said for Cuizon cannot, alas, be said for appellant Pua. While the
search and arrest carried out on him and Lee may have been illegal for not being
incident to a lawful warrantless arrest, the unfortunate fact is that appellant Pua
failed to challenge the validity of his arrest and search, as well as the admission of
the evidence obtained thereby; he did not raise the issue or assign the same as an
error before this Court. Accordingly, any possible challenge thereto based on
constitutional grounds is deemed waived. This Court has upheld and recognized
waivers of constitutional rights, including, particularly, the right against
unreasonable searches and seizures, in cases such People v. Malasugui 35 and De
Garcia v. Locsin. 36

Additionally, the prosecution had argued and the trial court agreed that by virtue of
the handwritten consent (Exhibit "I") secured by the arresting officers from
appellants Pua and Lee, the latter freely gave their consent to the search of their
baggage, and thus, the drugs discovered as a result of the consented search is
admissible in evidence. The said written permission is in English, and states
plainly that they (Pua and Lee) freely consent to the search of their luggage to be
conducted by NBI agents to determine if Pua and Lee are carrying shabu. It
appears that appellant Pua understands both English and Tagalog; he is born of a
Filipino mother, had resided in Vito Cruz, Manila, and gave his occupation as that
of salesman. He admitted that he was asked to sign the written consent, and that he
did in fact sign it (TSN, May 28, 1992, pp. 33-34). His barefaced claim made
during his direct and cross examinations to the effect that he did not really read the
consent but signed it right away, and that by signing it he only meant to give
permission for the NBI agents to enter the room (and not to search) is hardly
worthy of belief, considering that prior to the search, he seemed to have been extra
careful about who to let into the hotel room.
Thus, the full weight of the prosecution’s testimonial evidence plus the large
amount of prohibited drugs found, must be given full force vis-a-vis Pua’s claim of
innocent presence in the hotel room, which is weak and not worthy of credence.

Re: Appellant Paul Lee @ "Paul Leung"

Appellant Lee’s situation is different from that of Pua. We agree with the Solicitor
General when he noted that the trial judge did not exert sufficient effort to make
available compulsory process and to see to it that accused appellant Lee was given
his day in court. It is clear that appellant Lee was effectively denied his right to
counsel, for although he was provided with one, he could not understand and
communicate with him concerning his defense such that, among other things, no
memorandum was filed on his behalf; further, he was denied his right to have
compulsory process to guarantee the availability of witnesses and the production of
evidence on his behalf, including the services of a qualified and competent
interpreter to enable him to present his testimony. 37 In sum, he was denied due
process. For this reason, we hold that the case as against Lee must be remanded to
the court of origin for a re-trial.

Epilogue

It is evident and clear to us that the NBI agents gravely mishandled the drug bust
operation and in the process violated the constitutional guarantees against unlawful
arrests and illegal searches and seizures. Because of the large haul of illegal drugs
that the government officers claimed to have recovered, this Court agonized over
the case before us and struggled to apply the law with an even hand. In the final
analysis, we in the administration of justice would have no right to expect ordinary
people to be law-abiding if we do not insist on the full protection of their rights.
Some lawmen, prosecutors and judges may still tend to gloss over an illegal search
and seizure as long as the law enforcers show the alleged evidence of the crime
regardless of the methods by which they were obtained. This kind of attitude
condones law-breaking in the name of law enforcement. Ironically, it only fosters
the more rapid breakdown of our system of justice, and the eventual denigration of
society. While this Court appreciates and encourages the efforts of law enforcers to
uphold the law and to preserve the peace and security of society, we nevertheless
admonish them to act with deliberate care and within the parameters set by the
Constitution and the law. Truly, the end never justifies the means.

WHEREFORE, in view of the foregoing considerations, Accused-appellant


Antolin Cuizon y Ortega is hereby ACQUITTED on constitutional grounds. His
immediate release is ordered unless he is detained for other valid causes. Accused-
appellant Steve Pua y Clofas is hereby found GUILTY of the crime of Illegal
Transport of Regulated Drugs, penalized under Section 15, R.A. No. 6425, as
amended, and is hereby sentenced to suffer the penalty of reclusion perpetua; the
Decision appealed from, as herein modified, is hereby affirmed as to appellant Pua.
Finally, the case as to appellant Lee is hereby ordered REMANDED to the trial
court in order that said accused may be given his day in court. The Decision
appealed from is also AFFIRMED with respect to the disposition of the prohibited
drugs involved in the case.

People v. Encinada, G.R. No. 116720, October 2, 1997


In acquitting the appellant, the Court reiterates the constitutional proscription that
evidence (in this case, prohibited drugs) seized without a valid search warrant is
inadmissible in any proceeding. A yield of incriminating evidence will not
legitimize an illegal search. Indeed, the end never justifies the means.

The Case

This principle is stressed in this appeal from the Judgment, 1 promulgated on July
15, 1994 by the Regional Trial Court of Surigao City, Branch 32, 2 in Criminal
Case No. 3668, convicting Appellant Roel Encinada of illegal transportation of
prohibited drugs under Section 4 of Republic Act No. 6425, as amended by Batas
Pambansa Blg. 179.

An Information, 3 dated May 22, 1992, was filed by Third Asst. Surigao City
Prosecutor Virgilio M. Egay charging appellant of said crime allegedly committed
as follows:jgc:chanrobles.com.ph

"That on or about May 21, 1992, in the City of Surigao, Philippines, and within the
jurisdiction of this Honorable Court, the above-named accused, in gross disregard
of the prohibition of the provisions of Republic Act No. 6425 as amended by Batas
Pambansa Bilang 179, did then and there willfully, unlawfully and feloniously
have in his possession, custody and control dried marijuana leaves weighing 800
grams, more or less, which he transported to Surigao City from Cebu City aboard a
passenger ship, well knowing that such acts are expressly prohibited by
law."cralaw virtua1aw library

Before arraignment, appellant, assisted by Counsel Antonio Casurra, offered to


plead guilty to a lesser offense, i.e., illegal possession of prohibited drugs. 4 The
trial court requested the prosecution to study the offer, 5 but the records do not
show any agreement on such proposal.

Upon his arraignment, appellant pleaded "not guilty" to the charge. 6 After the
prosecution presented its evidence, the defense filed, with leave of court, 7 a
"Demurrer to Evidence" dated September 1, 1993, 8 questioning the admissibility
of the evidence which allegedly was illegally seized from appellant. The court a
quo denied the motion, ruling: 9

"For resolution is the demurrer to evidence dated September 1, 1993 of the


accused, Roel Encinada, praying that he be acquitted of the crime charged on the
ground of the inadmissibility of the evidence for the prosecution consisting of the
marijuana (seized) from him by the police. The accused raised the following issues,
to wit: (1) Whether the arrest and search of the accused without a warrant would
fall under the doctrine of warrantless search as an incident to a lawful arrest; and,
(2) Whether the subject marijuana is admissible in evidence against the accused.

x       x       x

A scrutiny of the evidence for the prosecution shows that the events leading to the
arrest of the accused started when SPO4 Nicolas Bolonia, chief of the PNP vice
control section, received a tip from his informer that the accused, Roel Encinada
would be arriving on board the M/V Sweet Pearl at about seven o’clock in the
morning of May 21, 1992. On cross-examination SPO4 Bolonia testified that the
information was given to him by his asset at about four o’clock in the afternoon of
May 20, 1992. After receiving the tip he relayed the information to SPO4 Cipriano
Iligan, Jr., PNP chief of intelligence. SPO4 Bolonia further declared that he would
have applied for a search warrant but there was simply no time for it.

x       x       x

In the later case of People v. Tangliben (184 SCRA 220) the Supreme Court
modified its ruling in the Aminuddin case when it held that the arrest and search is
lawful when the police had to act quickly and there was no more time to secure a
search warrant. It is noted that the tip was given to SPO4 Bolonia by his informant
at about the closing time of the offices of the various courts. He still had to inform
SPO4 Iligan in order to coordinate with him. The boat carrying the accused was
scheduled to dock in Surigao City at seven o’clock the following morning when
the courts had not yet opened.

It is therefore quite obvious that the police did not have enough time to apply for a
search warrant in the interim. The police cannot be faulted for acting on the tip and
for stopping and searching the accused even without a warrant.

In the case at bar, the accused was caught in flagrante delicto in actual possession
of the marijuana. The search made upon his personal effects falls squarely under
paragraph (a) of Rule 113, Section 5 of the 1985 Rules on Criminal Procedure
which allows a warrantless search as an incident to a lawful arrest (People v.
Malmstedt, 198 SCRA 401).
x       x       x

WHEREFORE, premises considered, the demurrer to evidence in question is


denied for lack of merit."cralaw virtua1aw library

After trial in due course, the assailed Judgment was rendered, the decretal portion
of which reads:jgc:chanrobles.com.ph

"WHEREFORE, premises considered, the Court finds the accused, Roel Encinada,
guilty beyond reasonable doubt of the violation of Section 4, Article II, of Republic
Act No. 6425 as amended by Batas Pambansa Bilang 179, and hereby sentences
him to suffer the penalty of life imprisonment and to pay a fine of twenty thousand
pesos (P20,000.00) without subsidiary imprisonment in case of insolvency; and to
pay the costs.

The marijuana (Exhibit B) involved in this case is hereby forfeited to the


government to be destroyed or disposed of pursuant to present rules and
regulations. The two plastic chairs (Exhibits D and D-1) are also forfeited to the
government."cralaw virtua1aw library

The Facts

Version of the Prosecution

The Solicitor General, in the Appellee’s Brief, recounts the events leading to
appellant’s arrest, as follows: 10

"At around 4 p.m. of May 20, 1992, SPO4 Nicolas Bolonia was in his house when
he received a tip from an informant that Roel Encinada would be arriving in
Surigao City from Cebu City in the morning of May 21, 1992 on board the M/V
Sweet Pearl bringing with him ‘marijuana.’ Bolonia was then Chief of the Vice
Control Squad of the Surigao City Police (pp. 27-29; TSN, November 27, 1992,
34-40; p. 10, TSN, May 14, 1993).

Bolonia already knew Encinada because the latter previously was engaged in
illegal gambling known as ‘buloy-buloy.’ After receiving the tip, Bolonia notified
the members of his team — SPO3 Marcial Tiro, SPO3 Glen Abot and SPO3
Charlito Duero — as well as his colleague SPO4 Cipriano Iligan, Jr., the chief of
the Intelligence and Investigation Division, of the information he received.
Because the information came late, there was no more time to secure a search
warrant (pp. 38; TSN, November 27, 1992, May 14, 1993, p. 13; pp. 4, 19; TSN,
March 3, 1993).
In the early morning of May 21, 1992, Bolonia, Iligan and other police officers
deployed themselves in different strategic points at the city wharf to intercept
Encinada. At about 8:15 a.m. of the same day, the M/V Sweet Pearl finally docked.
The police officers saw Encinada walk briskly down the gangplank, carrying two
small colored plastic baby chairs in his hand (p. 11 TSN, May 14, 1993; pp. 4, 5,
15-16 TSN, March 3, 1993; pp. 29-30 TSN, November 27, 1992, pp. 29-30).

From their various positions, the police officers followed Encinada immediately
boarded a tricycle at Borromeo Street, still holding the plastic chairs. As the
tricycle slowly moved forward, Bolonia chased it and ordered the driver to stop
after identifying himself as a police officer. When the vehicle stopped, Bolinia
identified himself to Encinada and ordered him to alight from the tricycle. Bolonia
asked Encinada to hand over the plastic chairs, to which the latter complied (pp. 5,
6, 17 TSN, March 3, 1993, pp. 30-32, 35 TSN, November 27, 1992).

Bolonia noticed that there were two small chairs, one green and the other blue,
stacked together and tied with a piece of string. Between the stack of chairs, there
was a bulky package. Bolonia examined it closely and smelled the peculiar scent of
marijuana. Making a small tear in the cellophane cover, Bolonia could see and
smell the what appeared to be ‘marijuana,’ a prohibited drug (pp. 6-9 TSN, March
3, 1993, Exh.’B’, ‘D’ and sub-markings; pp. 32-34. 35-39 TSN, November 27,
1992).

Encinada was brought to the central police station. Bolonia, in the presence of one
Nonoy Lerio who is a member of the local media and a friend of Encinada, opened
the package. It was discovered that indeed, the contents consisted of dried leaves
known as marijuana. In the course of the investigation, Encinada surrendered to
Bolonia his passenger ticket issued by M/V Sweet Pearl (pp. 9-11 TSN, March 3,
1993, Exh.’E’; pp. 34-35, 39-40 TSN, November 27, 1992).

On July 13, 1992, Bolonia brought the package of dried leaves for examination at
the PNP Crime Laboratory at Camp Evangelista, Cagayan de Oro City. The
forensic chemist, Inspector Vicente Armada, tested the leaves and confirmed that
they were positive for marijuana. However, the marijuana only weighed 610
grams, which Armada opined to be probably due to shrinkage and moisture loss
(pp. 12-17, 19-21, 24-40, 41; TSN, November 27, 1992, Exh.’A’, ‘B’, ‘C’ and sub-
markings.)"

Version of the Defense

Appellant sets up denial as his defense. In his brief, he denied ownership and
possession of said plastic baby chairs, as follows: 11
"1) In the morning of May 21, 1992, at around 8:00 o’clock in the morning, more
or less, the accused was seen to have disembarked from M/V Sweet Pearl after an
overnight trip from Cebu City;

2) The accused proceeded to the Surigao PPA Gate and boarded a motorela bound
for his residence at Little Tondo, (within the City Proper), Surigao City. The
Motorela was fully loaded with passengers, with the accused as the fourth
passenger;

3) When the motorela was already able to travel a distance of about ten (10) meters
more or less, the same was forcibly stopped by persons who ordered the passengers
to disembarked (sic). Thereafter, all the (baggage) of the passengers and the driver
were ordered to stand in a line for which a body search was made individually
(sic);

4) After the search was made, the accused was singled out in the line and ordered
to board the service vehicle of the police and was brought to the PNP Police
Station.

Before however the accused boarded the jeep, he was openly protesting to the
action taken by the police authorities and demanded from the apprehending
officers a copy of a search warrant and/or warrant of arrest for the search made and
for his apprehension;

5) In the police headquarters, the accused was made to undergo custodial


investigation for which a plastic bag was presented to him allegedly containing the
subject marijuana leaves. The accused denied that the said plastic bag belonged to
him.

The denial was witnessed by Mr. Daniel ‘Nonoy’ Lerio, Jr. a member of the
Surigao City Press, who was invited by the Police Investigators to witness the
presentation of the alleged marijuana leaves, during the said
investigation;chanrobles virtual lawlibrary

6) After the custodial investigation, the accused was placed immediately behind
bars and the Information for Violation of RA 6425 as amended by Batas Pambansa
Blg. 179 was filed before the Court;

x       x       x"

Aside from appellant, the defense also presented five (5) other witnesses whose
testimony allegedly established the following: 12

"8.a) Ruben Concha — the driver of the motorela who testified that he was
surprised when the motorela he was driving was forcibly stopped (while already in
motion) by the police authorities while directing his four (4) passengers, (3 males
and 1 female) to disembarked (sic) together with their (baggage).

That after the search was made, the accused was singled out, and despite the
protests made, was ordered to board the Police service vehicle, while the 2 other
male passengers just left the scene while the female passenger continued to board
the motorela who directed him to proceed to the residence of Baby Encinada to
verify whether the person picked up by the police authorities was related to the
latter;

8.b) Josephine Nodalo — testified that she is a beautician, and that she was one of
the four (4) passengers of the motorela driven by Ruben Concha, which motorela
was forcibly stopped by men who are chasing it after travelling a distance of 5 to
10 meters away from its loading area near the PPA Gate.

All the four (4) passengers were ordered to disembarked (sic) from the motorela
whereupon they were all subjected to body search including their (baggage).

That it was the male passenger who was sitting at the rear portion of the motorela
who was picked up by the Police Authorities and despite the protests made was
ordered to board the Police service vehicle.

Upon learning from the persons who were gathered at the scene, that the one who
was picked up was the son of Mr. Encinada, the latter boarded back the motorela
and directed the driver to proceed to the residence of the Encinada’s at Little
Tondo to verify whether it was really their son who was picked up by the police
authorities. She made this, as Mrs. Encinada, (the mother of the accused) is his
(regular) customer;

8.c) Mr. Daniel ‘Nonoy’ Lerio, Jr. — testified that, being a member of the Press,
he was requested by the police authorities to witness the custodial investigation
conducted upon the person of the accused, who, during the entire proceedings of
the investigation vehemently denied having any knowledge about the marijuana
leaves placed inside the plastic bag;

8.d) Isabelita Encinada — testified that she was informed by her manicurist
(Josephine Nodalo) about the arrest . . . (of) her son, somewhere at the PPA Port
Area and upon being informed, she and her husband immediately went to the
Surigao PNP Headquarters to verify the (news) . . .;"

x       x       x"

Ruling of the Trial Court


The trial court rejected appellant’s claim that he was merely an innocent passenger
and that his package contained mango and otap samples, not marijuana.
Emphasizing that the Surigao City Police had no ill motive against appellant, the
trial court gave credence to SPO4 Bolonia’s story that he actually received from
his police asset the information regarding appellant’s arrival in Surigao City. The
trial court further emphasized that appellant was caught carrying marijuana in
flagrante delicto. Hence, the warrantless search following his lawful arrest was
valid and the marijuana obtained was admissible in evidence.

Assignment of Errors

In his Brief, appellant submits the following assignment of errors: 13

"I. The lower court erred in finding that the accused was caught in flagranti (sic)
delicto in possession of the subject marijuana leaves and is the one responsible in
transporting the same;

II. The lower court gravely erred in finding that search and the arrest of the accused
without a warrant would fall under the doctrine of warrantless search as incident to
a lawful arrest —

III. The lower court gravely erred in finding that the subject marijuana leaves is
admissible in evidence —"

In short, the main issues are (1) the sufficiency of the evidence showing possession
of marijuana by appellant and (2) the validity of the search conducted on the
person and belongings of the Appellant.

The Court’s Ruling

The petition is meritorious.

First Issue: Illegal Possession of Prohibited Drugs

Appellant claims that the prosecution failed to prove his possession and ownership
of the plastic baby chairs. He contends that the testimonies of Bolonia and Iligan
conflicted as to the number of passengers riding the motorela. Such alleged conflict
is peripheral and irrelevant. Hence, it deserves scant consideration. Appellant adds
that such testimonies also conflicted as to the place where appellant sat inside the
motorela. This claim, aside from being flimsy, is also not supported by the
transcript of stenographic notes.
In his testimony, appellant vehemently denied possession of the plastic baby
chairs, stressing that he was not holding them when the search was conducted.
However, his denial is easily rebutted by Bolonia’s testimony: 14

"Q: When you saw Roel Encinada who disembarked from M/V Sweet Pearl, what
did you observe in his person, if any?

A: He was carrying a (sic) baby chairs.

Q: What kind of chairs?

A: A (sic) plastic chairs.

x       x       x

Q: After you saw Roel Encinada disembarked (sic) from the boat, what did you
and your companions do?

A: We followed him behind because we posted in the different direction(s) in the


wharf.

x       x       x

Q: You said you followed Roel Encinada, what happened next when you followed
him?

A: I saw Roel Encinada took (sic) a ride with a motorcycle so I chased him and let
him stopped (sic).

x       x       x

Q: By the way, where was (sic) this (sic) two plastic chairs placed in the motorize
tricycle?

A: He was sitting at the back of the motor at the right portion of the seat and the
chairs was (sic) placed besides him. ([W]itness indicating that he was sitting (sic)
an imaginary seat at the back of the motor and holding an (sic) imaginary chairs
with his left arm)."cralaw virtua1aw library

Between these two contentions, the choice of the trial court prevails because this is
a matter that involves credibility of witnesses. On this subject of credibility, the
opinion of the trial court deserves great respect as it was in a better position to
observe the demeanor and deportment of the witnesses on the stand; 15 hence, it
was in a superior situation to assess their testimonies.

Furthermore, proof of ownership of the marijuana is not necessary in the


prosecution of illegal drug cases; 16 it is sufficient that such drug is found in
appellant’s possession.

Second Issue: Illegal Search and Seizure

Based on the foregoing discussion, appellant’s conviction could have been


affirmed by this Court. However, the very evidence implicating him — the
prohibited drugs found in his possession — cannot be used against him in this case
or, for that matter, in "any proceeding."cralaw virtua1aw library

Generally, a search and seizure must be validated by a previously secured warrant;


otherwise, such search and seizure is subject to challenge. 17 Section 2, Article III
of the 1987 Constitution, is apropos:jgc:chanrobles.com.ph

"SEC. 2. The right of the people to be secure in their persons, houses, papers, and
effects against unreasonable searches and seizures of whatever nature and for any
purpose shall be inviolable, and no search warrant or warrant of arrest shall issue
except upon probable cause to be determined personally by the judge after
examination under oath or affirmation of the complainant and the witnesses he
may produce, and particularly describing the place to be searched and the persons
or things to be seized." chanroblesvirtualawlibrary

Any evidence obtained in violation of this provision is legally inadmissible in


evidence as a "fruit of the poisonous tree." This principle is covered by this
exclusionary rule:jgc:chanrobles.com.ph

"SEC. 3. . . .

(2) Any evidence obtained in violation of . . . the preceding section shall be


inadmissible for any purpose in any proceeding."cralaw virtua1aw library

The plain import of the foregoing provision is that a search and seizure is normally
unlawful unless authorized by a validly issued search warrant or warrant of arrest.
This protection is based on the principle that, between a citizen and the police, the
magistrate stands as a mediator, nay, an authority clothed with power to issue or
refuse to issue search warrants or warrants of arrest. 18

The right against warrantless searches, however, is subject to legal and judicial
exceptions, as follows: (1) search incidental to a lawful arrest, (2) search of moving
vehicles, (3) seizure in plain view, (4) customs searches, and (5) waiver by the
accused themselves of their right against unreasonable search and seizure. 19 In
these cases, the search and seizure may be made only upon probable cause as the
essential requirement. Although the term eludes exact definition, probable cause
signifies a reasonable ground of suspicion supported by circumstances sufficiently
strong in themselves to warrant a cautious man’s belief that the person accused is
guilty of the offense with which he is charged; or the existence of such facts and
circumstances which could lead a reasonably discreet and prudent man to believe
that an offense has been committed and that the item(s), article(s) or object(s)
sought in connection with said offense or subject to seizure and destruction by law
is in the place to be searched. 20

In this case, Bolonia received at 4:00 p.m. on May 20, 1992 an intelligence report
that appellant who was carrying marijuana would arrive the next morning aboard
the M/V Sweet Pearl. Although such report could have been the basis of probable
cause, Bolonia explained that he could not secure a warrant because the courts in
Surigao City were already closed for the day. Thus, he and the other lawmen had
no choice but to proceed the next morning to the port area. After appellant
disembarked from the ship and rode a motorela, Bolonia stopped the motor vehicle
and conducted the search. He rummaged through the two strapped plastic baby
chairs which were held by appellant and found inserted between them a package of
marijuana wrapped in a small plastic envelope.

Appellant contended before the lower court that the warrantless search of his
belongings was proscribed by the Constitution. But the trial judge rejected this
contention, opining that appellant was caught in flagrante delicto at the time of his
arrest. Hence, it concluded that the warrantless search conducted after his "lawful
arrest" was valid and that the marijuana was admissible in evidence.

Rule 113, Section 5, discusses the instances when a warrantless arrest may be
effected, as follows:jgc:chanrobles.com.ph

"SEC. 5. Arrest without warrant; when lawful. — A peace officer or a private


person may, without a warrant, arrest a person:chanrob1es virtual 1aw library

(a) When, in his presence, the person to be arrested has committed, is actually
committing, or is attempting to commit an offense;

(b) When an offense has in fact just been committed, and he has personal
knowledge of facts indicating that the person to be arrested has committed it; and

(c) When the person to be arrested is a prisoner who has escaped from a penal
establishment or place where he is serving final judgment or temporarily confined
while his case is pending, or has escaped while being transferred from one
confinement to another.
x       x       x."cralaw virtua1aw library

In this case, appellant was not committing a crime in the presence of the Surigao
City policemen. Moreover, the lawmen did not have personal knowledge of facts
indicating that the person to be arrested had committed an offense. The search
cannot be said to be merely incidental to a lawful arrest. Raw intelligence
information is not a sufficient ground for a warrantless arrest. Bolonia’s testimony
shows that the search preceded the arrest: 21

"Q: You said you followed Roel Encinada, what happened next when you followed
him?

A: I saw Roel Encinada took (sic) a ride with a motorcycle so I chased him and let
him stopped (sic).

x       x       x

Q: You said you stopped the motor tricycle in which Roel Encinada (sic) riding,
what did you do?

A: At first I identified myself to the driver and to some of the passengers.

x       x       x

Q: And after that, what happened next?

A: I requested Roel Encinada to disembark from the motor tricycle because of that
information given to us in his possession;

Q: Possession of what?

A: Possession of marijuana, Sir.

Q: And Roel Encinada alighted from the motor vehicle?

A: Yes, Sir.

Q: After Roel Encinada alighted from the motor tricycle, what happened next?

A: I requested to him to see his chairs that he carried."cralaw virtua1aw library

Contrary to the trial court’s ruling, People v. Tangliben 22 is factually inapplicable


to the case at bar. The prosecution’s evidence did not show any suspicious
behavior when the appellant disembarked from the ship or while he rode the
motorela. No act or fact demonstrating a felonious enterprise could be ascribed to
appellant under such bare circumstances.

We disagree with the trial court’s justification for the search:jgc:chanrobles.com.ph

"The arrest of the accused without warrant was lawful because there was a
probable cause or ground for his apprehension. The police had received reliable,
albeit confidential information from their informant that Roel Encinada would be
bringing in marijuana from Cebu City on board the M/V Sweet Pearl.
Unfortunately there was no more time for the police to apply for and secure a
search warrant as the information was received late in the afternoon of May 20,
1992 and the accused was expected to arrive at seven o’clock the following
morning. The different courts were closed by then. Nevertheless the police felt
constrained to act on the valuable piece of information."cralaw virtua1aw library

Even if the information was received by Bolonia about 4:00 p.m. of May 20, 1992
at his house, there was sufficient time to secure a warrant of arrest, as the M/V
Sweet Pearl was not expected to dock until 7:00 a m. the following day.
Administrative Circular No. 13 allows applications for search warrants even after
court hours:jgc:chanrobles.com.ph

"3. Rafflling shall be strictly enforced, except only in case where an application for
search warrant may be filed directly with any judge in whose jurisdiction the place
to be searched is located, after office hours, or during Saturdays, Sundays, and
legal holidays, in which case the applicant is required to certify under oath the
urgency of the issuance thereof after office hours, or during Saturdays, Sundays
and legal holidays; (Emphasis supplied)

The same procedural dispatch finds validation and reiteration in Circular No. 19,
series of 1987, entitled "Amended Guidelines and Procedures on Applications for
Search Warrants for Illegal Possession of Firearms and Other Serious Crimes Filed
in Metro Manila Courts and Other Courts with Multiple
Salas" :jgc:chanrobles.com.ph

"This Court has received reports of delay while awaiting raffle, in acting on
applications for search warrants in the campaign against loose firearms and other
serious crimes affecting peace and order. There is a need for prompt action on such
applications for search warrant. Accordingly, these amended guidelines in the
issuance of a search warrant are issued:chanrob1es virtual 1aw library

1. All applications for search warrants relating to violation of the Anti-subversion


Act, crimes against public order as defined in the Revised Penal Code, as amended,
illegal possession of firearms and/or ammunition and violations of the Dangerous
Drugs Act of 1972, as amended, shall no longer be raffled and shall immediately
be taken cognizance of and acted upon by the Executive Judge of the Regional
Trial Court, Metropolitan Trial Court, and Municipal Trial Court under whose
jurisdiction the place to be searched is located.

2. In the absence of the Executive Judge, the Vice-Executive Judge shall take
cognizance of and personally act on the same. In the absence of the Executive
Judge or Vice-Executive Judge, the application may be taken cognizance of and
acted upon by any judge of the Court where the application is filed.chanrobles
virtual lawlibrary

3. Applications filed after office hours, during Saturdays, Sundays and holidays,
shall likewise be taken cognizance of and acted upon by any judge of the Court
having jurisdiction of the place to be searched, but in such cases the applicant shall
certify and state the facts under oath, to the satisfaction of the judge, that its
issuance is urgent.

4. Any judge acting on such application shall immediately and without delay
personally conduct the examination of the applicant and his witnesses to prevent
the possible leakage of information. He shall observe the procedures, safeguards,
and guidelines for the issuance of search warrants provided for in this Court’s
Administrative Circular No. 13, dated October 1, 1985."cralaw virtua1aw library

In People v. Aminnudin, the Court declared as inadmissible in evidence the


marijuana found in appellant’s possession during a search without a warrant,
because it had been illegally seized. The Court firmly struck down the policemen’s
cavalier disregard for the Bill of Rights, explaining:jgc:chanrobles.com.ph

"The present case presented no urgency. From the conflicting declarations of the
PC witnesses, it is clear that they had at least two days within which they could
have obtained a warrant to arrest and search Aminnudin who was coming to Iloilo
on the M/V Wilcon 9. His name was known. The vehicle was identified. The date
of its arrival was certain. And from the information they had received, they could
have persuaded a judge that there was probable cause, indeed, to justify the
issuance of a warrant. Yet they did nothing. No effort was made to comply with the
law. The Bill of Rights was ignored altogether because the PC lieutenant who was
the head of the arresting team, had determined on his own authority that a ‘search
warrant was not necessary.’"

Lawmen cannot be allowed to violate the very law they are expected to enforce.
Bolonia’s receipt of the intelligence information regarding the culprit’s identity,
the particular crime he allegedly committed and his exact whereabouts underscored
the need to secure a warrant for his arrest. But he failed or neglected to do so. Such
failure or neglect cannot excuse him from violating a constitutional right of
the Appellant.

It is significant that the Solicitor General does not share the trial judge’s opinion.
Taking a totally different approach to justify the search, the Republic’s counsel
avers that appellant voluntarily handed the chairs containing the package of
marijuana to the arresting officer and thus effectively waived his right against the
warrantless search. This, he gleaned from Bolonia’s testimony: 23

"Q: After Roel Encinada alighted from the motor tricycle, what happened next?

A: I requested to him to see his chairs that he carried.

Q: Are you referring to the two plastic chairs?

A: Yes, Sir.

Q: By the way, when Roel Encinada agreed to allow you to examine the two
plastic chairs that he carried, what did you do next?

A: I examined the chairs and I noticed that something inside in between the two
chairs."cralaw virtua1aw library

We are not convinced. While in principle we agree that consent will validate an
otherwise illegal search, we believe that appellant — based on the transcript quoted
above — did not voluntarily consent to Bolonia’s search of his belongings.
Appellant’s silence should not be lightly taken as consent to such search. 24 the
implied acquiescence to the search, if there was any, could not have been more
than mere passive conformity given under intimidating or coercive circumstances
and is thus considered no consent at all within the purview of the constitutional
guarantee. 25 Furthermore, considering that the search was conducted irregularly,
i.e, without a warrant, we cannot appreciate consent based merely on the
presumption of regularity of the performance of duty.

Appellant’s alleged acquiescence should be distinguished from the consent


appreciated in the recent case of People v. Lacerna. 26 In said case, the search was
conducted at a validly established checkpoint and was made in the regular
performance of the policemen’s duty. Although it became intrusive when the
policemen opened his baggage, it was validated by the consent of appellant, who
testified in open court that he allowed such search because he had nothing to hide.
In the present case, there was no checkpoint established. The policemen stopped
the motorela and forthwith subjected the passengers to a search of their persons
and baggage. In contrast to the accused in Lacerna, herein appellant testified that
he openly objected to the search by asking for a warrant.
Without the illegally seized prohibited drug, the appellant’s conviction cannot
stand. There is simply no sufficient evidence remaining to convict him. That the
search disclosed a prohibited substance in appellant’s possession, and thus
confirmed the police officers’ initial information and suspicion, did not cure its
patent illegality. An illegal search cannot be undertaken and then an arrest effected
on the strength of the evidence yielded by the search.

We should stress that the Court is not unmindful of the difficulties of law
enforcement agencies in suppressing the illegal traffic of dangerous drugs.
However, quick solutions of crimes and apprehensions of malefactors do not
justify a callous disregard of the Bill of Rights. Law enforcers are required to
follow the law and to respect the people’s rights. Otherwise, their efforts become
counterproductive. We remind them of this recent exhortation by this Court: 27

". . . In the final analysis, we in the administration of justice would have no right to
expect ordinary people to be law-abiding if we do not insist on the full protection
of their rights. Some lawmen, prosecutors and judges may still tend to gloss over
an illegal search and seizure as long as the law enforcers show the alleged evidence
of the crime regardless of the methods by which they were obtained. This kind of
attitude condones law-breaking in the name of law enforcement. Ironically, it only
fosters the more rapid breakdown of our system of justice, and the eventual
denigration of society. While this Court appreciates and encourages the efforts of
law enforcers to uphold the law and to preserve the peace and security of society,
we nevertheless admonish them to act with deliberate care and within the
parameters set by the Constitution and the law. Truly, the end never justifies the
means."cralaw virtua1aw library

WHEREFORE, the appeal is hereby GRANTED. The assailed Decision is


REVERSED and SET ASIDE. Appellant is ACQUITTED. Unless convicted for
any other crime or detained for some lawful reason, Appellant Roel Encinada is
ORDERED RELEASED immediately.

SO ORDERED.

Espano v. CA, G.R. No. 120431, April 1, 1998

This is a petition for review of the decision of the Court of Appeals in CA-G.R. CR
No. 13976 dated January 16, 1995,1 which affirmed in toto the judgment of the
Regional Trial Court of Manila, Branch 1, convincing petitioner Rodolfo Espano
for violation of Article II, Section 8 of Republic Act No. 6425, as amended,
otherwise known as the Dangerous Drugs Act.

Petitioner was charged under the following information:


That on or about July 14, 1991, in the City of Manila, Philippines, the said
accused not being authorized by law to possess or use any prohibited drug,
did then and there willfully, unlawfully and knowingly have in his
possession and under his custody and control twelve (12) plastic cellophane
(bags) containing crushed flowering tops, marijuana weighing 5.5 grams
which is a prohibited drug.

Contrary to law. 2

The evidence for the prosecution, based on the testimony of Pat. Romeo Pagilagan,
shows that on July 14, 1991, at about 12:30 a.m., he and other police officers,
namely, Pat. Wilfredo Aquino, Simplicio Rivera, and Erlindo Lumboy of the
Western Police District (WPD), Narcotics Division went to Zamora and Pandacan
Streets, Manila to confirm reports of drug pushing in the area. They saw petitioner
selling "something" to another person. After the alleged buyer left, they
approached petitioner, identified themselves as policemen, and frisked him. The
search yielded two plastic cellophane tea bags of marijuana. When asked if he had
more marijuana, he replied that there was more in his house. The policemen went
to his residence where they found ten more cellophane tea bags of marijuana.
Petitioner was brought to the police headquarters where he was charged with
possession of prohibited drugs. On July 24, 1991, petitioner posted bail 3 and the
trial court issued his order of release on July 29, 1991. 4

Annabelle Alip, forensic chemist of the WPD Criminal Investigation Laboratory


Section, testified that the articles sent to her by Pat. Wilfredo Aquino regarding the
apprehension of a certain Rodolfo Espano for examination tested positive for
marijuana, with a total weight of 5.5 grams.

By way of defense, petitioner testified that on said evening, he was sleeping in his
house and was awakened only when the policemen handcuffed him. He alleged
that the policemen were looking for his brother-in-law Lauro, and when they could
not find the latter, he was instead brought to the police station for investigation and
later indicted for possession of prohibited drugs. His wife Myrna corroborated his
story.

The trial court rejected petitioner's, defense as a "mere afterthought" and found the
version of the prosecution "more credible and trustworthy."

Thus, on August 14, 1992, the trial court rendered a decision, convicting petitioner
of the crime charged, the dispositive portion of which reads:

WHEREFORE there being proof beyond reasonable doubt, the court finds
the accused Rodolfo Espano y Valeria guilty of the crime of violation of
Section 8, Article II, in relation to Section 2 (e-L) (I) of Republic Act No.
6425 as amended by Batas Pambansa Blg. 179, and pursuant to law hereby
sentences him to suffer imprisonment of six (6) years and one (1) day to
twelve (12) years and to pay a fine of P6,000.00 with subsidiary
imprisonment in case of default plus costs.

The marijuana is declared forfeited in favor of government and shall be


turned over to the Dangerous Drugs Board without delay.

SO ORDERED. 5

Petitioner appealed the decision to the Court of Appeals. The appellate court,
however, affirmed the decision of the trial court in toto.

Hence, this petition.

Petitioner contends that the trial and appellate courts erred in convicting him on the
basis of the following: (a) the pieces of evidence seized were inadmissible; (b) the
superiority of his constitutional right to be presumed innocent over the doctrine of
presumption of regularity, (c) he was denied the constitutional right of
confrontation and to compulsory process; and (d) his conviction was based on
evidence which was irrelevant and not properly identified.

After a careful examination of the records of the case, this Court finds no
compelling reason sufficient to reverse the decisions of the trial and appellate
courts.

First, it is a well settled doctrine that findings of trial courts on the credibility of
witnesses deserve a high degree of respect. Having observed the deportment of
witnesses during the trial, the trial judge is in a better position to determine the
issue of credibility and, thus, his findings will not be disturbed during appeal in the
absence of any clear showing that he had overlooked, misunderstood or misapplied
some facts or circumstances of weight and substance which could have altered the
conviction of the appellants.6

In this case, the findings of the trial court that the prosecution witnesses were more
credible than those of the defense must stand. Petitioner failed to show that Pat.
Pagilagan, in testifying against him, was motivated by reasons other than his duty
to curb drug abuse and had any intent to falsely impute to him such a serious crime
as possession of prohibited drugs. In the absence of such ill motive, the
presumption of regularity in the performance of his official duty must prevail.

In People v. Velasco,7 this Court reiterated the doctrine of presumption of


regularity in the performance of official duty which provides:

. . . Appellant failed to establish that Pat. Godoy and the other members of
the buy-bust team are policemen engaged in mulcting or other unscrupulous
activities who were motivated either by the desire to extort money or exact
personal vengeance, or by sheer whim and caprice, when they entrapped her.
And in the absence of proof of any intent on the part of the police authorities
to falsely impute such a serious crime against appellant, as in this case, the
presumption of regularity in the performance of official duty, . . . , must
prevail over the self-serving and uncorroborated claim of appellant that she
had been framed. 8

Furthermore, the defense set up by petitioner does not deserve any consideration.
He simply contended that he was in his house sleeping at the time of the incident.
This Court has consistently held that alibi is the weakest of all defenses; and for it
to prosper, the accused has the burden of proving that he was not at the scene of the
crime at the time of its commission and that it was physically impossible for him to
be there. Moreover, the "claim of a 'frame-up', like alibi, is a defense that has been
invariably viewed by the Court with disfavor for it can just as easily be concocted
but difficult to prove, and is a common and standard line of defense in most
prosecutions arising from violations of the Dangerous Drugs Act." 9 No clear and
convincing evidence was presented by petitioner to prove his defense of alibi.

Second, petitioner contends that the prosecution's failure to present the alleged
informant in court cast a reasonable doubt which warrants his acquittal. This is
again without merit, since failure of the prosecution to produce the informant in
court is of no moment especially when he is not even the best witness to establish
the fact that a buy-bust operation had indeed been conducted. In this case, Pat.
Pagilagan, one of the policemen who apprehended petitioner, testified on the actual
incident of July 14, 1991, and identified him as the one they caught in possession
of prohibited drugs. Thus,

We find that the prosecution had satisfactorily proved its case against
appellants. There is no compelling reason for us to overturn the finding of
the trial court that the testimony of Sgt. Gamboa, the lone witness for the
prosecution, was straightforward spontaneous and convincing. The
testimony of a sole witness, if credible and positive and satisfies the court
beyond reasonable doubt, is sufficient to convict. 10

Thus on the basis of Pat. Pagilagan's testimony, the prosecution was able to prove
that petitioner indeed committed the crime charged; consequently, the finding of
conviction was proper.

Lastly, the issue on the admissibility of the marijuana seized should likewise be
ruled upon. Rule 113 Section 5(a) of the Rules of Court provides:

A peace officer or a private person may, without a warrant, arrest a person:


a. when, in his presence, the person to be arrested has committed, is actually
committing, or is attempting to commit an offense;

x x x           x x x          x x x

Petitioner's arrest falls squarely under the aforecited rule. He was caught in
flagranti as a result of a buy-bust operation conducted by police officers on the
basis of information received regarding the illegal trade of drugs within the area of
Zamora and Pandacan Streets, Manila. The police officer saw petitioner handing
over something to an alleged buyer. After the buyer left, they searched him and
discovered two cellophanes of marijuana. His arrest was, therefore, lawful and the
two cellophane bags of marijuana seized were admissible in evidence, being the
fruits of the crime.

As for the ten cellophane bags of marijuana found at petitioner's residence,


however, the same are inadmissible in evidence.

The 1987 Constitution guarantees freedom against unreasonable searches and


seizures under Article III, Section 2 which provides:

The right of the people to be secure in their persons, houses, papers and
effects against unreasonable searches and seizures of whatever nature and
for any purpose shall be inviolable, and no search warrant or warrant of
arrest shall issue except upon probable cause to be determined personally by
the judge after examination under oath or affirmation of the complainant and
the witnesses he may produce, and particularly describing the place to be
searched and the persons or things to be seized.

An exception to the said rule is a warrantless search incidental to a lawful arrest for
dangerous weapons or anything which may be used as proof of the commission of
an offense.11 It may extend beyond the person of the one arrested to include the
premises or surroundings under his immediate control. In this case, the ten
cellophane bags of marijuana seized at petitioner's house after his arrest at
Pandacan and Zamora Streets do not fall under the said exceptions.

In the case of People v. Lua,12 this Court held:

As regards the brick of marijuana found inside the appellant's house, the trial
court correctly ignored it apparently in view of its inadmissibility. While
initially the arrest as well as the body search was lawful, the warrantless
search made inside the appellant's house became unlawful since the police
operatives were not armed with a search warrant. Such search cannot fall
under "search made incidental to a lawful arrest," the same being limited to
body search and to that point within reach or control of the person arrested,
or that which may furnish him with the means of committing violence or of
escaping. In the case at bar, appellant was admittedly outside his house
when he was arrested. Hence, it can hardly be said that the inner portion of
his house was within his reach or control.

The articles seized from petitioner during his arrest were valid under the doctrine
of search made incidental to a lawful arrest. The warrantless search made in his
house, however, which yielded ten cellophane bags of marijuana became unlawful
since the police officers were not armed with a search warrant at the time.
Moreover, it was beyond the reach and control of petitioner.

In sum, this Court finds petitioner Rodolfo Espano guilty beyond reasonable doubt
of violating Article II, Section 8, in relation to Section 2 (e-L) (I) of Republic Act
No. 6425, as amended. Under the said provision, the penalty imposed is six years
and one day to twelve years and a fine ranging from six thousand to twelve
thousand pesos. With the passage of Republic Act No. 7659, which took effect on
December 31, 1993, the imposable penalty shall now depend on the quantity of
drugs recovered. Under the provisions of Republic Act No. 7629, Section 20, and
as interpreted in People v. Simon13 and People v. Lara,14 if the quantity of
marijuana involved is less than 750 grams, the imposable penalty ranges
from prision correccional to reclusion temporal. Taking into consideration that
petitioner is not a habitual delinquent, the amendatory provision is favorable to him
and the quantity of marijuana involved is less than 750 grams, the penalty imposed
under Republic Act No. 7659 should be applied. There being no mitigating nor
aggravating circumstances, the imposable penalty shall be prision correccional in
its medium period. Applying the Indeterminate Sentence Law, the maximum
penalty shall be taken from the medium period of prision correccional, which is
two (2) years, four (4) months and one (1) day to four (4) years and two (2)
months, while the minimum shall be taken from the penalty next lower in degree,
which is one (1) month and one (1) day to six (6) months of arresto mayor.

WHEREFORE, the instant petition is hereby DENIED. The decision of the Court
of Appeals in C.A.-G.R. CR No. 13976 dated January 16, 1995 is AFFIRMED
with the MODIFICATION that petitioner Rodolfo Espano is sentenced to suffer an
indeterminate penalty of TWO (2) months and ONE (1) day of arresto mayor, as
minimum to TWO (2) years, FOUR (4) months and ONE (1) day of prision
correccional, as maximum.

SO ORDERED.

People v. Claudio, G.R. No. 72564, April 15, 1988 [CRITIQUE]

This is an appeal from the decision of the Regional Trial Court of Olongapo City,
Branch 73 finding the accused Anita Claudio y Bagtang guilty beyond reasonable
doubt of violating Sec. 4, Rep. Act No. 6425 (Dangerous Drugs Act of 1972 as
amended) and sentencing her to serve the penalty of reclusion perpetua, to pay a
fine of P20,000.00, and to pay the costs.

The information filed against the accused alleged:jgc:chanrobles.com.ph

"That on or about the 21st day of July 1981, in the City of Olongapo, Philippines
and within the jurisdiction of this Honorable Court, the above-named accused
without being lawfully authorized, did then and there wilfully, unlawfully and
knowingly transport 1.1 kilos of Marijuana dried leaves, which are prohibited
drugs for the purpose of selling the same from Baguio City to Olongapo City."
(Rollo, p. 13)

The lower court established her guilt beyond reasonable doubt on the basis of the
prosecution’s evidence as follows:jgc:chanrobles.com.ph

"To prove the guilt of the accused, the prosecution offered the following
documentary and testimonial evidence as follows: Exhibit "A" — Letter request
for Examination of suspected marijuana dried leaves weighing approximately 1.1
kilos dated July 25, 1981; "B" — plastic container; "B" marijuana contained in the
plastic container; "B" -1-a" — another plastic container; "C" — Chemistry Report
No. D-668-81; "C" Findings: Positive for marijuana; "D, "D-1," D-2 and "D-3;"
"E" and "E" photographs of accused with Pat. Daniel Obiña and Paulino Tiongco
showing the marijuana, "F — Victory Liner Ticket No. 84977; "G" — Sworn
Statement of Pat. Daniel Obiña, "H" — Request for Field Test on suspected
marijuana from accused by P/Lt. Antonio V. Galindo; "H" -1 date of receipt of the
request; "L" — Certificate of Field Test dated duly 22, 1981; "B-2" and "B-2-a" —
additional Wrapping paper; and the testimonies of witnesses of the prosecution,
Theresa Ann Bugayong; Pat. Daniel Obiña, Cpl. Paulino Tiongco, Cpl. Ernesto
Abello and Sgt. Leoncio Bagang.

"Theresa Ann Bugayong - 22 years old, single, Forensic Chemist and a resident of
1150 Sampaloc, Metro Manila, testified that she received a request from the Task
Force Bagong Buhay, Olongapo City, dated July 25, 1981, on specimen of
marijuana submitted for examination. The specimen consisted of 900 grams of
suspected dried marijuana flowering tops wrapped in a newspaper placed in a
plastic bag with a marking "MB Store" (Exh. "B").

"The examination conducted by her proved to be positive for marijuana. After her
examination, she prepared Chemistry Report No. D-668-81 dated July 29, 1981
(Exhs. "C" and "C-1"). She conducted three examinations; microscopic
examination, the duguenoi levine test and thirdly, the confirmatory examination of
thin layer chromatographic test. The said specimen was submitted to them by OIC
Danilo Santiago, a representative of the CANU, Olongapo City.cralawnad
"The second witness for the prosecution was Daniel Obiña, 37 years old, married,
policeman and residing at 34 Corpuz St., East Tapinac, Olongapo City. Obiña
testified that he has been a member of the INP, since 1970 up to the present. He
was assigned in June, 1972 at the Investigation Division as operative. His job then
was among other things to follow up reports in their office, recover stolen items
and apprehend suspects on July 21, 1981, he was on Detached Service with the
ANTI-NARCOTICS Unit; and that on that date, he came from Baguio City and
arrived in Olongapo City at about 1:30 o’clock in the Afternoon having left Baguio
at about 8:30 o’clock in the morning. He took the Victory Liner in going back to
Olongapo City. His family lives in Baguio City. On board the Victory Liner, he
was seated on the second seat at the back. While he was thus seated, suspect Anita
Claudio boarded the same bus and took the seat in front of him after putting a bag
which she was carrying at the back of the seat of Obiña. The bag placed by suspect
behind his seat was a wooven buri bag made of plastic containing some vegetables.
The act of the accused putting her bag behind Pat. Obiña seat aroused his suspicion
and made him felt (sic) nervous. With the feeling that there was something
unusual, he had the urge to search the woven plastic bag. But it was only at San
Fernando, Pampanga when he was able to go to the bag. He inserted one of his
fingers in a plastic bag located at the bottom of the woven bag and smelt
marijuana. The plastic woven bag appearing to contain camote tops on the top has
a big bundle of plastic of marijuana at the bottom. He could recognize the smell of
marijuana because he was assigned at that time at the ANTI-NARCOTICS Unit.
He did not, however, do anything after he discovered that there was marijuana
inside the plastic bag of the accused until they reached Olongapo City and the
accused alighted from the bus in front of the Caltex Gasoline Station in Sta. Rita.
Right after the accused alighted from the bus, policeman Obiña intercepted her and
showed her his ID identifying himself as a policeman and told her he will search
her bag because of the suspicion that she was carrying marijuana inside said bag.
In reply, Accused told him, "Please go with me, let us settle this at home."
However, the witness did not heed her plea and instead handcuffed her right hand
and with her, boarded a tricycle right away and brought the suspect to the police
headquarters with her bag appearing to contain vegetables.

"At the police headquarters Investigation Section. the bag was searched in the
presence of Investigator Cpl. Tiongco; Pat. Obiña, the accused and Sgt. Leoncio
Bagang Inside the plastic bag was found a big bundle of plastic containing
marijuana weighing about one kilo. Witness stated that he could detect marijuana
even before the application of chemicals because of his one year and a half
assignment with the CANU. After the marijuana was taken from the bag of the
accused, photographs were taken of the accused and the marijuana confiscated
from her possession with Pat. Obiña and that of Investigator Tiongco, Accused and
himself identified photographs shown to him in open Court. (Exhs. "D," "D-1," "D-
2" and "D-3"). Witness was likewise shown a plastic bag of marijuana contained in
a plastic container (Exhs. "B," "B-1" and "B-1-a") and identified it as the one
confiscated from the accused and pointed to his initials on the newspaper wrapping
which also shows the date and time, although the wrapper at the time he testified
appeared to be soiled already. The marijuana was allegedly still fresh when
confiscated.

"To prove further that the accused transported the confiscated marijuana from
Baguio City to Olongapo City, witness identified Victory Liner Ticket No. 684977
which was confiscated from the accused and for identification purposes, the
witness presented the body number of the bus he wrote at the back of the ticket
which is "309" (Exhs. "F" and "F-1"). Regarding himself, he did not pay his fare
from Baguio City because as a policeman, he used his badge and a free ride.

"On cross-examination, witness stated that he went to Baguio City on July 15,
1981 and underwent treatment of his heart while he was there. He was given a
furlough for medical treatment. He stayed in Baguio City for about five days and
returned to Olongapo City or July 21, 1981. Prior to July 21, 1981, witness never
knew the accused, and the first time he saw her was in Baguio when she boarded
the same Victory Liner he took. Wi en the accused who was bringing with her a
woven plastic bag placed the bag right behind his seat instead of placing it in front
of her or beside her seat Witness Obiña became suspicious and his suspicion was
confirmed when they reached San Fernando, Pampanga, after he checked the buri
bag. The bus stopped at said town to load some gasoline. Witness inserted one of
his fingers inside the buri bag and thereafter smelt marijuana. He confirmed his
testimony on direct that when witness confronted accused he was invited to go
with her in order to settle the matter to which he refused. Accused further testified
that from the time the accused placed her bag behind his seat from Baguio City, he
felt so nervous and had to take his medicine at the Tarlac Station. It was only after
having taken his medicine that his apprehension was contained and thus was able
to insert his right hand inside the buri bag in San Fernando, Pampanga. His fingers
reached the very bottom of the bag. He identified his sworn statement regarding
this incident given on July 21, 1981 which is Exhibit "G." Witness likewise
identified accused Anita Claudio in open court.

"Paulino Tiongco, 52 years old, married and resident of 31 Canada St., East Bajac
Bajac, Olongapo City, testified that as a policeman on the afternoon of July 21,
1981, he was inside the Investigation Division of the Police Station, Olongapo
City. As Duty Investigator, between 1:46 and 2:00 o’clock in the afternoon of the
same day, Pat. Daniel Obiña arrived at the Police Station with a woman and
identified her in the courtroom as Anita Claudio. Pat. Obiña reported to him that he
apprehended Anita Claudio inside the Victory Liner bus for possession of
marijuana dried leaves. The marijuana leaves were contained in a buri bag with
some vegetables such as camote tops, bananas and some other vegetable." The
marijuana was placed in a plastic wrapper with the name National Book Store
colored black and white. Witness identified the wrapper (Exh. "B-2"). The bag
contained the markings of Pat. Obiña which are his initials, (Exh. "B-2-a"), and
numbers 210781 representing the date which was placed by Pat. Obiña after Cpl.
Tiongco examined the suspected marijuana.chanrobles virtual lawlibrary

"After examining and seeing the marijuana together with the vegetables, he
interviewed apprehending officer Obiña and reduced his statements in writing. Cpl.
Tiongco identified the sworn statement of Obiña (Exh.’G). He also interviewed
accused Anita Claudio who was all the while inside the Investigation room seated
on a chair. After appraising her of her constitutional rights, he asked the accused
whether she was willing to give her written statements to which the accused
refused. Hence, no statements were taken of her. However, pictures were taken
inside the investigation room. Exhs. "D" and "E," series which were already
previously identified by Pat. Obiña. Witness identified the persons appearing in the
pictures as that of Pat. Obiña and the accused and also of himself. Thereafter, the
marijuana contained in the plastic bag were turned over to Lt. Galindo and Anita
Claudio was detained.

"Ernesto Abello, 41 years old, married and residing at No. 29 Alba Street, East
Tapinac, Olongapo City, testified he was since March 1972 a policeman and was
stationed at Police Station 21, Olongapo City, Metrodiscom. However, in 1981, he
was already assigned to the CANU-General Anti-NARCOTICS Unit. On July 22,
1981, he reported for work at the CANU and received from Lt. Galindo more than
a kilo of suspected marijuana dried leaves. As requested by Lt. Galindo, he
conducted a field test on this marijuana which he received from Lt. Galindo, as
evidenced by a request signed by him dated July 22, 1981 (Exh. "H").

"In connection with the field test conducted by him on the specimen, he prepared a
Certificate of Field Test dated July 22, 1981 (Exhs. "I"). The Certificate of Field
Test indicated the presence of tetra-hydrocannabinol (THC), an active substance
that can only be found in marijuana, a prohibited drug. Cpl. Abello identified a
plastic bag of marijuaua received from Lt. Galindo which he later give to CIC
Danilo Santiago, the Evidence Custodian, for the latter to bring the specimen to the
PC Crime Laboratory.

"The last witness for the prosecution was Leoncio Bagang, 40 years old, married,
residing at No. 27 Jones St., East Tapinac, Olongapo City, a policeman of
Olongapo City, assigned with Police Station "21." He has been a policeman since
1966 up to the present. In July, 1981, he was then assigned at the Patrol Division
and his duty was to patrol the city proper from Magsaysay Drive up to east Bajac
Bajac.

"He narrated that on July 21, 1981, between the hours of 1:00 and 2:00 o’clock in
the afternoon, he was at the Caltex Gasoline Station, East Bajac Bajac, Olongapo
City along Rizal Avenue. He was then on duty patrol using a motorcycle. While he
was at the said place, he saw Pat. Obiña alighted from the Victory Liner bus
ordering somebody to alight from the same bus. When he heard Pat. Obiña, he
approached him and asked him what was happening. Pat. Obiña told him he
apprehended a certain woman possessing dried marijuana. The woman was shall
then inside the bus. Pat. Obiña then brought the woman to the police department
who was bringing with her a buri bag. They boarded a tricycle, the woman riding
inside the tricycle while Pat. Obiña sat behind the driver. He then followed in his
motorcycle the said tricycle to police station. He went inside the Investigation
Section of the Police Station and he was there when Pat. Obiña reported to Cpl.
Tiongco his apprehension of the woman possessing marijuana. He saw the
marijuana for the first time inside the Investigation Section placed in a buri bag
covered with newspaper. He witnessed the taking out of the marijuana from inside
the bag by Pat. Obiña in the presence of Cpl. Tiongco and the woman or the
accused in this case, and himself Policeman Bagang identified the accused in open
Court. When asked about the nature of the marijuana when it was brought out from
the bag, he said that the marijuana was dried but not well dried. Aside from the
marijuana inside the buri bag, there were vegetables and bananas. Witness
identified in open Court, the marijuana he saw found in the buri bag of the accused.
His means of identification was the signature of Pat. Obiña, (Exh. "B-1"). He
likewise identified a newspaper wrapping which was already torn.

"While in the Investigation Division, witness Bagang heard the accused’s answer
to Cpl. Tiongco’s question that she was going to deliver the marijuana to Sta. Rita.
He, however, did not linger long at the investigation Division. After he saw the
marijuana and heard the answer of the accused to Cpl. Tiongco’s question the place
of delivery of the marijuana, he left the police station. Witness likewise identified
an initial DO-21-07-81 already marked as Exhibit "B-2." DO which is an initial,
and not a signature, stands for Daniel Obiña. After the testimony of Leoncio
Bagang, the prosecution rested its case." (Rollo, pp. 42-47)

Accused Claudio raised the following assignments of errors in this


appeal:chanrob1es virtual 1aw library

"CONVICTION UNDER SECTION 4, ART. II OF R.A. 6425 IS IMPROPER IF


ONE OR SOME OF THE ELEMENTS OF THE OFFENSE IS OR ARE
ABSENT.

II

"CONVICTION CAN NOT BE HAD UNDER SECTION 4, ART. II OF R.A.


6425 IF THE ALLEGED BUYMAN WAS NOT PRESENTED TO TESTIFY.

III

"APPELLANT’S CONVICTION FOR DELIVERY (SEC. 4, ART II, OF R.A.


6424) IS WRONG BECAUSE SOME MATERIAL FACTS WERE
OVERLOOKED AND NOT CONSIDERED IN FAVOR OF APPELLANT.
(Rollo, p. 91)

The accused alleges that she is only liable, at the most, for possession under Sec. 8,
Art. II of Rep. Act No. 6425 and not for violating Sec. 4 of the same Act.

The latter section, Sec. 4 provides:jgc:chanrobles.com.ph

"Sec. 4. Sale, Administration, Delivery Distribution and Transportation of


Prohibited Drugs. — The penalty of life imprisonment to death and a fine ranging
from twenty thousand to thirty thousand pesos shall be imposed upon any person
who, unless authorized by law, shall sell, administer, deliver, give away to another,
distribute, dispatch in transit or transport any prohibited drug, or shall act as a
broker in any of such transactions. If the victim of the offense is a minor, or should
a prohibited drug involved in any offense under this Section be the proximate
cause of the death of a victim thereof, the maximum penalty herein provided shall
be imposed."cralaw virtua1aw library

Claudio contends that there was no delivery as there was no recipient of the
prohibited drugs. Therefore, she may not be convicted under Sec. 4 of Rep. Act
No. 6425.chanrobles.com:cralaw:red

The contention is without merit. A closer perusal of the subject provision shows
that it is not only delivery which is penalized but also the sale, administration,
distribution and transportation of prohibited drugs. Claudio was caught
transporting 1.1 kilos of marijuana, thus the lower court did not err in finding her
guilty of violating Sec. 4.

The accused also alleges that before the completion of delivery, the intention of the
possessor is unknown.

This allegation is also unavailing. It is undisputed that Claudio had in her


possession 1.1 kilos of marijuana. This is a considerable quantity. As held in the
case of People v. Toledo, (140 SCRA 259, 267) "the possession of such
considerable quantity as three plastic bags of marijuana leaves and seeds coupled
with the fact that he is not a user of prohibited drugs cannot indicate anything
except the intention of the accused to sell, distribute and deliver said
marijuana."cralaw virtua1aw library

The accused next contends the warrantless search, seizure and apprehension as
unlawful.

The applicable provisions on this issue are found in the 1985 Rules on Criminal
Procedure.

Rule 113, Sec. 5(a) of the said Rules provides:jgc:chanrobles.com.ph

". . . A peace officer or a private person may, without a warrant, arrest a


person:jgc:chanrobles.com.ph

"(a) When, in his presence, the person to be arrested has committed, is actually
committing, or is attempting to commit an offense.

x       x       x

Meanwhile, its Rule 126, Sec. 12 provides:jgc:chanrobles.com.ph

"Section 12. Search incident to lawful arrest. — A person lawfully arrested may be
searched for dangerous weapons or anything which may be used as proof of the
commission of an offense, without a search warrant. (12a)"

Appellant Claudio was caught transporting prohibited drugs. Pat. Daniel Obiña did
not need a warrant to arrest Claudio as the latter was caught in flagrante delicto.
The warrantless search being an incident to a lawful arrest is in itself lawful.
(Nolasco v. Pano, 147 SCRA 509). Therefore, there was no infirmity in the seizure
of the 1.1 kilos of marijuana.

The accused takes inconsistent positions in her appellant’s brief At first, she does
not deny having had with her marijuana at the time of her arrest. Instead, she
claims that she should just be guilty of possession. In a complete turnabout, in the
latter portion of said brief, she claims that the evidence against her were mere
fabrications and the marijuana allegedly found in her possession was only planted.

We have carefully examined the records of the case and we find no ground to alter
the trial court’s findings and appreciation of the evidence presented.

Credence is accorded to the prosecution’s evidence, more so as it consisted mainly


of testimonies of policemen. Law enforcers are presumed to have regularly
performed their duty in the absence of proof to the contrary (People v. De Jesus,
145 SCRA 521). We also find no reason from the records why the prosecution
witnesses should fabricate their testimonies and implicate appellant in such a
serious crime (See People v. Bautista, 147 SCRA 500).chanrobles virtual
lawlibrary

The accused testified that she was not on that bus that came from Baguio City but
rather she was in Olongapo City all that time. She alleged that she was arrested by
Pat. Obiña for no reason at all.

In the case at bar, alibi does not deserve much credit as it was established only by
the accused herself (People v. De la Cruz, 148 SCRA 582).

Moreover, it is a well-established rule that alibi cannot prevail over positive


testimony (People v. De La Cruz, supra).

WHEREFORE, the judgment appealed from is AFFIRMED.

People v. Aminnudin, G.R. No. 74869 July 6, 1988

The accused-appellant claimed his business was selling watches but he was
nonetheless arrested, tried and found guilty of illegally transporting marijuana. The
trial court, disbelieving him, held it was high time to put him away and sentenced
him to life imprisonment plus a fine of P20,000.00. 1

Idel Aminnudin was arrested on June 25, 1984, shortly after disembarking from the
M/V Wilcon 9 at about 8:30 in the evening, in Iloilo City. The PC officers who
were in fact waiting for him simply accosted him, inspected his bag and finding
what looked liked marijuana leaves took him to their headquarters for
investigation. The two bundles of suspect articles were confiscated from him and
later taken to the NBI laboratory for examination. When they were verified as
marijuana leaves, an information for violation of the Dangerous Drugs Act was
filed against him. 2 Later, the information was amended to include Farida Ali y
Hassen, who had also been arrested with him that same evening and likewise
investigated. 3 Both were arraigned and pleaded not guilty. 4 Subsequently, the
fiscal filed a motion to dismiss the charge against Ali on the basis of a sworn
statement of the arresting officers absolving her after a "thorough investigation." 5
The motion was granted, and trial proceeded only against the accused-appellant,
who was eventually convicted. 6

According to the prosecution, the PC officers had earlier received a tip from one of
their informers that the accused-appellant was on board a vessel bound for Iloilo
City and was carrying marijuana. 7 He was identified by name. 8 Acting on this
tip, they waited for him in the evening of June 25, 1984, and approached him as he
descended from the gangplank after the informer had pointed to him. 9 They
detained him and inspected the bag he was carrying. It was found to contain three
kilos of what were later analyzed as marijuana leaves by an NBI forensic examiner,
10 who testified that she conducted microscopic, chemical and chromatographic
tests on them. On the basis of this finding, the corresponding charge was then filed
against Aminnudin.

In his defense, Aminnudin disclaimed the marijuana, averring that all he had in his
bag was his clothing consisting of a jacket, two shirts and two pairs of pants. 11 He
alleged that he was arbitrarily arrested and immediately handcuffed. His bag was
confiscated without a search warrant. At the PC headquarters, he was manhandled
to force him to admit he was carrying the marijuana, the investigator hitting him
with a piece of wood in the chest and arms even as he parried the blows while he
was still handcuffed. 12 He insisted he did not even know what marijuana looked
like and that his business was selling watches and sometimes cigarettes. 13 He also
argued that the marijuana he was alleged to have been carrying was not properly
identified and could have been any of several bundles kept in the stock room of the
PC headquarters. 14

The trial court was unconvinced, noting from its own examination of the accused
that he claimed to have come to Iloilo City to sell watches but carried only two
watches at the time, traveling from Jolo for that purpose and spending P107.00 for
fare, not to mention his other expenses. 15 Aminnudin testified that he kept the two
watches in a secret pocket below his belt but, strangely, they were not discovered
when he was bodily searched by the arresting officers nor were they damaged as a
result of his manhandling. 16 He also said he sold one of the watches for P400.00
and gave away the other, although the watches belonged not to him but to his
cousin, 17 to a friend whose full name he said did not even know. 18 The trial
court also rejected his allegations of maltreatment, observing that he had not
sufficiently proved the injuries sustained by him. 19

There is no justification to reverse these factual findings, considering that it was


the trial judge who had immediate access to the testimony of the witnesses and had
the opportunity to weigh their credibility on the stand. Nuances of tone or voice,
meaningful pauses and hesitation, flush of face and dart of eyes, which may reveal
the truth or expose the lie, are not described in the impersonal record. But the trial
judge sees all of this, discovering for himself the truant fact amidst the falsities.

The only exception we may make in this case is the trial court’s conclusion that the
accused-appellant was not really beaten up because he did not complain about it
later nor did he submit to a medical examination. That is hardly fair or realistic. It
is possible Aminnudin never had that opportunity as he was at that time under
detention by the PC authorities and in fact has never been set free since he was
arrested in 1984 and up to the present. No bail has been allowed for his release.

There is one point that deserves closer examination, however, and it is


Aminnudin’s claim that he was arrested and searched without warrant, making the
marijuana allegedly found in his possession inadmissible in evidence against him
under the Bill of Rights. The decision did not even discuss this point. For his part,
the Solicitor General dismissed this after an all-too-short argument that the arrest
of Aminnudin was valid because it came under Rule 113, Section 6(b) of the Rules
of Court on warrantless arrests. This made the search also valid as incidental to a
lawful arrest.

It is not disputed, and in fact it is admitted by the PC officers who testified for the
prosecution, that they had no warrant when they arrested Aminnudin and seized the
bag he was carrying. Their only justification was the tip they had earlier received
from a reliable and regular informer who reported to them that Aminnudin was
arriving in Iloilo by boat with marijuana. Their testimony varies as to the time they
received the tip, one saying it was two days before the arrest, 20 another two weeks
21 and a third "weeks before June 25." 22 On this matter, we may prefer the
declaration of the chief of the arresting team, Lt. Cipriano Querol, Jr., who testified
as follow:jgc:chanrobles.com.ph

"Q You mentioned an intelligence report, you mean with respect to the coming of
Idel Aminnudin on June 25, 1984?

"A Yes, sir.

"Q When did you receive this intelligence report?

"A Two days before June 25, 1984 and it was supported by reliable sources.

"Q Were you informed of the coming of the Wilcon 9 and the possible trafficking
of marijuana leaves on that date?

"A Yes, sir, two days before June 25,1984 when we received this information from
that particular informer, prior to June 25, 1984 we have already reports of the
particular operation which was being participated by Idel Aminnudin.

"Q You said you received an intelligence report two days before June 25,1984 with
respect to the coming of Wilcon 9?

"A Yes, sir.

"Q Did you receive any other report aside from this intelligence report?

"A Well, I have received also other reports but not pertaining to the coming of
Wilcon 9. For instance, report of illegal gambling operation.
"COURT:jgc:chanrobles.com.ph

"Q Previous to that particular information which you said two days before June 25,
1984, did you also receive any report regarding the activities of Idel Aminnudin?

"A Previous to June 25, 1984 we received reports on the activities of Idel
Aminnudin.

"Q What were those activities?

"A Purely marijuana trafficking.

"Q From whom did you get that information?

"A It came to my hand which was written in a required sheet of information,


maybe for security reason and we cannot identify the person.

"Q But you received it from your regular informer?

"A Yes, sir.

"ATTY. LLARIZA:jgc:chanrobles.com.ph

"Q Previous to June 25, 1984, you were more or less sure that Idel Aminnudin is
coming with drugs?

"A Marijuana, sir.

"Q And this information respecting Idel Aminnudin’s coming to Iloilo with
marijuana was received by you many days before you received the intelligence
report in writing?

"A Not a report of the particular coming of Aminnudin but his activities.

"Q You only knew that he was coming on June 25, 1984 two days before?

"A Yes, sir.

"Q You mean that before June 23, 1984 you did not know that Aminnudin was
coming?

"A Before June 23, 1984, I, in my capacity, did not know that he was coming but
on June 23, 1984 that was the time when I received the information that he was
coming. Regarding the reports on his activities, we have reports that he was
already consummated the act of selling and shipping marijuana stuff.

"COURT:jgc:chanrobles.com.ph

"Q And as a result of that report, you put him under surveillance?

"A Yes, sir.

"Q In the intelligence report, only the name of Idel Aminnudin was mentioned?

"A Yes, sir.

"Q Are you sure of that?

"A On the 23rd he will be coming with the woman.

"Q So that even before you received the official report on June 23, 1984, you had
already gathered information to the effect that Idel Aminnudin was coming to
Iloilo on June 25,1984?

"A Only on the 23rd of June.

"Q You did not try to secure a search warrant for the seizure or search of the
subject mentioned in your intelligence report?

"A No, more.

"Q Why not?

"A Because we were very very sure that our operation will yield positive result.

"Q Is that your procedure that whenever it will yield positive result you do not
need a search warrant anymore?

"A Search warrant is not necessary." 23

That last answer is a cavalier pronouncement, especially as it comes from a mere


lieutenant of the PC. The Supreme Court cannot countenance such a statement.
This is still a government of laws and not of men.

The mandate of the Bill of Rights is clear:jgc:chanrobles.com.ph

"Sec. 2. The right of the people to be secure in their persons, houses, papers, and
effects against unreasonable searches and seizures of whatever nature and for any
purpose shall be inviolable, and no search warrant or warrant of arrest shall issue
except upon probable cause to be determined personally by the judge after
examination under oath or affirmation of the complainant and the witnesses he
may produce, and particularly describing the place to be searched and the persons
or things to be seized."cralaw virtua1aw library

In the case at bar, there was no warrant of arrest or search warrant issued by a
judge after personal determination by him of the existence of probable cause.
Contrary to the averments of the government, the accused-appellant was not caught
in flagrante nor was a crime about to be committed or had just been committed to
justify the warrantless arrest allowed under Rule 113 of the Rules of Court. Even
expediency could not be invoked to dispense with the obtention of the warrant as
in the case of Roldan v. Arca, 24 for example. Here it was held that vessels and
aircraft are subject to warrantless searches and seizures for violation of the customs
law because these vehicles may be quickly moved out of the locality or jurisdiction
before the warrant can be secured.

The present case presented no such urgency. From the conflicting declarations of
the PC witnesses, it is clear that they had at least two days within which they could
have obtained a warrant to arrest and search Aminnudin who was coming Iloilo on
the M/V Wilcon 9. His name was known. The vehicle was identified. The date of
its arrival was certain. And from the information they had received, they could
have persuaded a judge that there was probable cause, indeed, to justify the
issuance of a warrant. Yet they did nothing. No effort was made to comply with the
law. The Bill of Rights was ignored altogether because the PC lieutenant who was
the head of the arresting team, had determined on his own authority that "search
warrant was not necessary."cralaw virtua1aw library

In the many cases where this Court has sustained the warrantless arrest of violators
of the Dangerous Drugs Act, it has always been shown that they were caught red-
handed, as result of what are popularly called "buy-bust" operations of the
narcotics agents. 25 Rule 113 was clearly applicable because at the precise time of
arrest the accused was in the act of selling the prohibited drug.

In the case at bar, the accused-appellant was not, at the moment of his arrest,
committing a crime nor was it shown that he was about to do so or that he had just
done so. What he was doing was descending the gangplank of the M/V Wilcon 9
and there was no outward indication that called for his arrest. To all appearances,
he was like any of the other passengers innocently disembarking from the vessel. It
was only when the informer pointed to him as the carrier of the marijuana that the
suddenly became suspect and so subject to apprehension. It was the furtive finger
that triggered his arrest. The identification by the informer was the probable cause
as determined by the officers (and not a judge) that authorized them to pounce
upon Aminnudin and immediately arrest him.
Now that we have succeeded in restoring democracy in our country after fourteen
years of the despised dictatorship, when any one could be picked up at will,
detained without charges and punished without trial, we will have only ourselves to
blame if that kind of arbitrariness is allowed to return, to once more flaunt its
disdain of the Constitution and the individual liberties its Bill of Rights guarantees.

While this is not to say that the accused-appellant is innocent, for indeed his very
own words suggest that he is lying, that fact alone does not justify a finding that he
is guilty. The constitutional presumption is that he is innocent, and he will be so
declared even if his defense is weak as long as the prosecution is not strong enough
to convict him.

Without the evidence of the marijuana allegedly seized from Aminnudin, the case
of the prosecution must fall. That evidence cannot be admitted, and should never
have been considered by the trial court for the simple fact is that the marijuana was
seized illegally. It is the fruit of the poisonous tree, to use Justice Holmes’
felicitous phrase. The search was not an incident of a lawful arrest because there
was no warrant of arrest and the warrantless arrest did not come under the
exceptions allowed by the Rules of Court. Hence, the warrantless search was also
illegal and the evidence obtained thereby was inadmissible.

The Court strongly supports the campaign of the government against drug
addiction and commends the efforts of our law-enforcement officers against those
who would inflict this malediction upon our people, especially the susceptible
youth. But as demanding as this campaign may be, it cannot be more so than the
compulsions of the Bill of Rights for the protection of the liberty of every
individual in the realm, including the basest of criminals. The Constitution covers
with the mantle of its protection the innocent and the guilty alike against any
manner of high-handedness from the authorities, however praiseworthy their
intentions.

Those who are supposed to enforce the law are not justified in disregarding the
rights of the individual in the name of order. Order is too high a price for the loss
of liberty. As Justice Holmes, again, said, "I think it a less evil that some criminal
should escape than that the government should play an ignoble part." It is simply
not allowed in the free society to violate a law to enforce another, especially if the
law violated is the Constitution itself.

We find that with the exclusion of the illegally seized marijuana as evidence
against the accused-appellant, his guilt has not been proved beyond reasonable
doubt and he must therefore be discharged on the presumption that he is innocent.

ACCORDINGLY, the decision of the trial court is REVERSE: and the accused-
appellant is ACQUITTED. It is so ordered.

Narvasa, Gancayco and Medialdea JJ. concur.


Issue: WON the search was an incident of a lawful arrest.

Ruling: 

No. The search was not an incident of a lawful arrest.

"Sec. 2. The right of the people to be secure in their persons, houses, papers, and
effects against unreasonable searches and seizures of whatever nature and for any
purpose shall be inviolable, and no search warrant or warrant of arrest shall issue
except upon probable cause to be determined personally by the judge after
examination under oath or affirmation of the complainant and the witnesses he
may produce, and particularly describing the place to be searched and the persons
or things to be seized."

In the case at bar, there was no warrant of arrest or search warrant issued by
a judge after personal determination by him of the existence of probable
cause. Contrary to the averments of the government, the accused-appellant was not
caught in flagrante nor was a crime about to be committed or had just been
committed to justify the warrantless arrest allowed under Rule 113 of the Rules of
Court. 

Even expediency could not be invoked to dispense with the obtention of the
warrant as in the case of Roldan v. Arca, for example. Here it was held that vessels
and aircraft are subject to warrantless searches and seizures for violation of the
customs law because these vehicles may be quickly moved out of the locality or
jurisdiction before the warrant can be secured.

The present case presented no such urgency. From the conflicting declarations of
the PC witnesses, it is clear that they had at least two days within which they could
have obtained a warrant to arrest and search Aminnudin. His name was known.
The vehicle was identified. The date of its arrival was certain. And from the
information they had received, they could have persuaded a judge that there
was probable cause, indeed, to justify the issuance of a warrant. Yet they did
nothing. No effort was made to comply with the law. The Bill of Rights was
ignored altogether because the PC lieutenant who was the head of the
arresting team, had determined on his own authority that "search warrant
was not necessary."

(Warrantless search during “on-the-spot apprehensions” with probable cause)


In the many cases where this Court has sustained the warrantless arrest of violators
of the Dangerous Drugs Act, it has always been shown that they were caught red-
handed, as result of what are popularly called "buy-bust" operations of the
narcotics agents. Rule 113 was clearly applicable because at the precise time of
arrest the accused was in the act of selling the prohibited drug.

In the case at bar, the accused-appellant was not, at the moment of his arrest,
committing a crime nor was it shown that he was about to do so or that he had
just done so. What he was doing was descending the gangplank of the M/V
Wilcon 9 and there was no outward indication that called for his arrest. To all
appearances, he was like any of the other passengers innocently disembarking from
the vessel. It was only when the informer pointed to him as the carrier of the
marijuana that he suddenly became suspect and so subject to apprehension. It was
the furtive finger that triggered his arrest. The identification by the informer was
the probable cause as determined by the officers (and not a judge) that authorized
them to pounce upon Aminnudin and immediately arrest him.

Conclusion:

Without the evidence of the marijuana allegedly seized from Aminnudin, the case
of the prosecution must fall. That evidence cannot be admitted, and should never
have been considered by the trial court for the simple fact that the marijuana was
seized illegally. It is the fruit of the poisonous tree, to use Justice Holmes'
felicitous phrase. 

The search was not an incident of a lawful arrest because there was no
warrant of arrest and the warrantless arrest did not come under the
exceptions allowed by the Rules of Court. Hence, the warrantless search was
also illegal and the evidence obtained thereby was inadmissible.

The Court held that with the exclusion of the illegally seized marijuana as evidence
against the accused-appellant, his guilt has not been proved beyond reasonable
doubt and he must therefore be discharged on the presumption that he is
innocent.

Dissenting:
GRIÑO-AQUINO, J., dissenting:
I respectfully dissent. I hold that the accused was caught in flagrante for he was
carrying marijuana leaves in his bag at the moment of his arrest. He was not
"innocently disembarking from the vessel." The unauthorized transportation of
marijuana (Indian hemp), which is a prohibited drug, is a crime, (Sec. 4, Rep. Act
No. 6425). Since he was committing a crime his arrest could be lawfully effected
without a warrant (Sec. 6-a, Rule 113, Rules of Court), and the search of his bag
(which yielded the marijuana leaves) without a search warrant was also lawful
(Sec. 12, Rule 126, Rules of Court). I vote to affirm the judgment of the trial court
finding him guilty of illegally transporting marijuana.
People v. Maspil, G.R. No. 85177, August 20, 1990 [CRITIQUE]

This petition is an appeal from the decision of the Regional Trial Court of Baguio
City, Branch 5, the dispositive portion of which reads:jgc:chanrobles.com.ph

"WHEREFORE, the Court finds and declare the accuse MOISES MASPIL, JR. y
WAYWAY and SALCEDO BAGKING y ALTAKI guilty beyond reasonable
doubt of the crime of illegal transportation of marijuana as charged and hereby
sentences EACH of them to suffer LIFE IMPRISONMENT; to pay a fine of
P20,000.00, without subsidiary imprisonment in case of insolvency; and to pay
their proportionate shares in the costs.

The confiscated marijuana (Exhibits "B", "B-1" to "B-23" ; "C", "C-1" to "C-16",
"D", "D-1" to "D-20" ; "E", "E-1", to "E-14" ; "F", "F-1" ; "G", "G-1") are hereby
declared forfeited in favor of the Government and upon the finality of this decision,
the Branch Clerk of Court is directed to turn over the same to the Dangerous Drugs
Board (NBI), through the Chief, PC Crime Laboratory, Regional Unit No. 1 Camp
Dangwa, La Trinidad, Benguet, for disposition in accordance with law." (Rollo, pp.
25-26)

In Criminal Case No. 4263-R, the information filed against the two accused
alleged:jgc:chanrobles.com.ph

"That on or about the 1st day of November, 1986, at Sayangan, Municipality of


Atok, Province of Benguet, Philippines, and within the jurisdiction of this
Honorable Court, the above-named accused, conspiring, confederating and
mutually aiding each other, and without any authority of law, did then and there
willfully, unlawfully and knowingly transport and carry in transit from Sinto,
Bauko, Mt. Province to Atok, Benguet One Hundred Eleven Kilos and Nine Grams
(111.9 kilos), more or less, of dried marijuana leaves which are sources of
dangerous and prohibited drugs and from which dangerous and prohibited drugs
nay be derived and manufactured, in violation of the said law." (Rollo, p. 11)

The narration of facts by the trial court is as follows:jgc:chanrobles.com.ph

"According to Jerry Veleroso, Sgt. Amador Ablang and Sgt. Florentino Baillo, all
members of the First Narcotics Regional Unit of the Narcotics Command stationed
in Baguio City, (See also Exhibit "I") on October 30, 1986, they established a
checkpoint in front of the Municipal Hall at Sayangan, Atok, Benguet, which is
along the Halsema Highway, to check on vehicles proceeding to Baguio City
because their Commanding Officer, Maj. Basilio Cablayan, had been earlier tipped
off by some confidential informers that the herein accused Maspil and Bagking
would be transporting a large volume of marijuana to Baguio City. The informers
went along with the operatives to Sayangan.

"At about 2:00 o’clock in the early morning of November 1, the operatives
intercepted a Sarao type jeep driven by Maspil with Bagking as his companion.
Upon inspection, the jeep was found loaded with two (2) plastic sacks (Exhibits
"B" and "D"), one (1) jute sack (Exhibit "C") and three (3) big round tin cans
(Exhibits "E", "F" and "G") which, when opened contained several bundles of
suspected dried marijuana leaves (Exhibits "B-1", to "B-23" ; "C-1" to "C-16" ;
"D-1" to "D-20" ; "E-1" to "E-14" ; "F-1" and "G-1").

"Maspil and Bagking were arrested and the suspected marijuana leaves were
confiscated.

"The confiscated items were later on referred to the PC Crime Laboratory,


Regional Unit I, for examination (Exhibit "A"). Forensic Chemist Carlos V.
Figuerroa performed the requested examination and determined that the specimen,
with an aggregate weight of 115.66 kilos, were positive to the standard tests for
marijuana.

"The accused admitted that the marijuana dried leaves were indeed confiscated
from the jeep being then driven by Maspil with Bagking as his helper. However,
they claimed that the prohibited drugs belonged to two of their passengers who
loaded them in the jeep as paying cargo for Baguio City without the accused
knowing that they were marijuana.

"The accused declared that on October 31, 1986, at the burned area along
Lakandula Street, Baguio City, a certain Mrs. Luisa Mendoza hired the jeep of
Maspil to transport her stock of dried fish and canned goods contained in cartons to
Abatan, Buguias, Benguet, because her own vehicle broke down. They left Baguio
City at about 1:00 o’clock in the afternoon (11:30 in the morning, according to
Bagking) with Mrs. Mendoza, her helper and salesgirls on board the jeep with
Maspil as driver and Bagking as his own helper. They arrived at Abatan at about
6:00 o’clock in the evening.

"After unloading their cargo, Maspil and Bagking repaired to a restaurant for their
dinner before undertaking the trip back to Baguio City. While thus eating, they
were approached by two persons, one of whom they would learn later on to be a
certain Danny Buteng. Buteng inquired if they were going to Baguio City and upon
being given an affirmative answer, he said that he would ride with them and that he
has some cargo. Asked what the cargo was, Buteng replied that they were flowers
in closed tin cans and sealed sacks for the commemoration of All Souls Day in
Baguio City. After Buteng had agreed to Maspil’s condition that he would pay for
the space to be occupied by his cargo, Buteng himself and his companion loaded
the cargo and fixed them inside Maspil’s jeep.
"Maspil and Bagking left Abatan at about 7:00 o’clock that same evening of
October 31. Aside from Buteng and companion they had four other passengers.
These four other passengers alighted at Natubleng, Buguias, Benguet.

"Upon reaching Sayangan, Atok, Benguet, Maspil stopped at the Marosan


Restaurant where they intended to take coffee. Their remaining passengers —
Buteng and companion — alighted and went to the restaurant. However, a soldier
waved at Maspil to drive to where he was, which Maspil did. The soldier secured
Maspil’s permission to inspect their cargo after which he grabbed Maspil on the
latter’s left shoulder and asked who owned the cargo. Maspil told the soldier that
the cargo belonged to their passengers who went to the restaurant. The soldier
called for his companions and they went to look for Maspil’s passengers in the
restaurant. Later on, they returned and placed Maspil and Bagking under arrest
since their cargo turned out to be marijuana.

"Lawrence Balonglong, alias Banawe, a radio reporter of DZWX Bombo Radio


who was invited by Lt. Valeroso to witness the operation, affirmed the
unsuccessful pursuit of the alleged two companions of Maspil and Bagking. He
recalled that he was awakened from his sleep at the town hall in Sayangan after the
arrest of Maspil and Bagking. When he went to the scene, the NARCOM
operatives boarded the jeep of Maspil to chase the two companions of Maspil and
Bagking. Balonglong climbed on top of the jeep with his camera to join the chase.
They proceeded towards the direction of Bontoc but failed to catch anyone. Hence,
they returned.

"Thereupon, Maspil and Bagking were taken to the town hall where they were
allegedly maltreated to admit ownership of the confiscated marijuana. At about
4:00 o’clock in the afternoon of November 1, the soldiers took them away from
Sayangan to be transferred to their station at Baguio City. On their way,
particularly at Km. 32 or 34, they met Mike Maspil, an elder brother of Moises
Maspil, and the soldiers called for him and then Lt. Valeroso and his men mauled
him on the road.

"Mike testified that between 3:00 and 4:00 o’clock in the afternoon of November
1, he was informed by a neighbor that his brother Moises was detained at the Atok
Municipal Jail. So he called for Jose Pos-el and James Longages, his driver and
helper, respectively, to go along with him to see Moises. They rode in his jeep. On
the way, they met the group of Lt. Valeroso. For no apparent reason, Lt. Valeroso
boxed and kicked him several times. Thereafter, Lt. Valeroso placed him under
arrest together with his driver and helper. They were all brought to a shoe store on
Gen. Luna Road, Baguio City, together with Moises and Bagking. There, Lt.
Valeroso got his wallet containing P210.00 and Seiko wrist watch but the receipt
(Exhibit "3") was issued by a certain Miss Pingil, a companion of Valeroso. He
was released after nine days. He then went to Lt. Valeroso to claim his wallet,
money and watch but he was told that they were with Miss Pingil. However, when
he went to Miss Pingil, the latter said that the items were with Lt. Valeroso. He
sought the assistance of then Tourism Deputy Minister Honorato Aquino who
assigned a lawyer to assist him. The lawyer advised him to file a case against Lt.
Valeroso but because of the intervening congressional elections, the matter has
never been pursued." (Rollo, p. 21-24)

The appellants raise the following assignment of errors in their appeal, to


wit:chanrob1es virtual 1aw library

THAT THE TRIAL COURT ERRED IN NOT FINDING THAT THE ALLEGED
MARIJUANA AS CHARGED IN THE INFORMATION IS DIFFERENT FROM
THAT PRESENTED FOR LABORATORY EXAMINATION.

II

THAT THE TRIAL COURT ERRED IN FINDING THAT THERE WERE ONLY
TWO OCCUPANTS, THE APPELLANTS, IN THE VEHICLE WHERE THE
ALLEGED MARIJUANA WAS CONFISCATED.

III

THAT THE TRIAL COURT ERRED IN FINDING THAT THE ACCUSED


KNEW THAT THE CARGO THEY WERE TRANSPORTING WAS
MARIJUANA.

IV

THAT THE TRIAL COURT ERRED IN ADMITTING IN EVIDENCE THE


ALLEGED CONFISCATED MARIJUANA.

THAT THE TRIAL COURT ERRED IN SHIFTING FROM THE


PROSECUTION THE BURDEN OF PROVING THE COMMISSION OF THE
OFFENSE CHARGED TO THE APPELLANTS TO PROVE THEIR
INNOCENCE." (Rollo, p. 40)
The main defense of the appellants is their claim that the prohibited drugs belonged
to their two passengers who loaded them in the jeep as paying cargo without the
appellants knowing that the cargo was marijuana.

In the second and third assignment of errors, the appellants claim that the trial
court erred in not appreciating their version of the facts.

The appellants state that the trial court’s reliance on Sgt. Baillo’s testimony that
they were the only ones in the jeep cannot be given credence as Sgt. Baillo’s
testimony is full of inconsistencies.

The appellants cite Sgt. Baillo’s inconsistencies as to the time of the arrest whether
morning or afternoon, the time the checkpoint was removed and the persons who
were with him at the time of arrest.

It has been ruled that inconsistencies in the testimonies of the prosecution


witnesses not on material points is not fatal. Moreover, minor inconsistencies are
to be expected but must be disregarded if they do not affect the basic credibility of
the evidence as a whole. (People v. Marcos, G.R. No. 83325, May 8, 1990)

The defense even state that there were a lot of policemen (T.S.N., December 1,
1987, p. 22) and it was but natural that there would be confusion on who was there
at the time of the arrest.

The trial court gave credence to the positive and categorical statement of Sgt.
Baillo that there were only two occupants, and these were the appellants inside the
jeepney at the time (T.S.N., June 30, 1987, p. 18). We see no cogent reason to
reverse this finding of fact.

There is nothing in the records to suggest that the arrest was motivated by any
reason other than the desire of the police officers to accomplish their mission.
Courts generally give full faith and credit to police officers when the facts and
circumstances surrounding then acts sustain the presumption that they have
performed their duties in a regular manner. (Rule 131, Section 5 (m), Rules of
Court; People v. Marcos, supra; People v. Yap and Mendoza, G.R. Nos. 87088-89,
May 9, 1990).

The appellants put forward the testimony of Lawrence Balonglong which


corroborates and affirms their stand that there were, indeed, passengers in the
jeepney.

However, a close perusal of said testimony reveals no such corroboration. The


pertinent portions of Balonglong’s testimony is as follows:jgc:chanrobles.com.ph
"x       x       x

Q Where were you when these two persons were apprehended?

A I was in the Municipal Hall asleep, sir.

Q How did you know then that these people were apprehended?

A It is like this, sir, on the night of October 31, I was then asleep and at around
11:00, I guess, p.m., they awakened me so I went and I saw these two guys being
apprehended by the Narcom operative.

Court:chanrob1es virtual 1aw library

Q You saw them being apprehended?

A No, sir . . . I saw them there.

Q Already apprehended?

A Already apprehended.

Atty. Fianza:chanrob1es virtual 1aw library

Q And when you saw these persons, what did you do, if any?

A What I recall is that when I went to the road, where these two guys were
apprehended, the operatives boarded the same jeep and I even climbed the jeep . . .
on top of the jeep holding my camera and tape recorder and we . . . I don’t know . .
. they chased, according to the operatives, they chased two companions of the two
arrested guys." (T.S.N., May 11, 1988, p. 4)

In their brief, the appellants even admit that "he (Balonglong) did not see the
passengers" and it was just his impression that there were other people present.
(Appellant’s Brief, p. 7)

The appellants maintain that they did not know what was in the cargo. Their main
concern was in going back to Baguio City and they saw no need to question their
two passengers on why flowers were being kept in closed cans and sacks. They
were apprehended after midnight. They traversed a lonely and reputedly dangerous
portion of the mountain highway.

The appellants’ version is not believable. It is inconceivable that the appellants


would not even bother to ask the names of the strangers who approached them in a
restaurant at night wanting to hire their jeepney, considering that they were
familiar with the identity of the passenger, Luisa Mendoza, who hired them to
transport her goods to Abatan, Buguias, Benguet.

It is likewise incredible that the appellants did not show the slightest curiousity as
to why flowers were being kept in closed tin cans and sealed sacks and cellophane.
On the other hand, the appellants had clear knowledge that Luisa Mendoza was
transporting cartons containing dried fish and canned goods on the trip out of
Baguio. It is contrary to human experience that the appellants would inquire about
the name of the passenger and the cargo she was loading on their jeep and not
doing the same about another who would transport goods on a midnight trip.

Well-settled is the rule that evidence to be believed, must not only proceed from
the mouth of a credible witness but it must be credible itself. No better test has yet
been found to measure the value of a witness than its conformity to the knowledge
and common experience of mankind. (People v. Maribung, 149 SCRA 292, 297
[1987]; People v. Aldana, G.R. No. 81817, July 27, 1989; People v. Pascua, G.R.
No. 82303, December 21, 1989).

The appellants further allege that if, indeed they knew about the contents of their
cargo, they would have adopted means to prevent detection or to evade arrest.

At the time the appellants were being motioned by the policemen to come nearer
the checkpoint, there was no way that the appellants could have evaded the arrest
without putting their lives in jeopardy. They decided to just brazen it out with
police and insist on their version of the story.

As for the other assigned errors, the appellants in the first assigned error, contend
that since there is a discrepancy of 3.76 between the number of kilos stated in the
information (111.9 kilos) and in the report (115.66 kilos) of the forensic chemist, it
is very likely that the marijuana presented as evidence was not the one confiscated
from the appellants or even if they were the same, it could have already been
tampered with. The appellants conclude that the marijuana then, cannot be
admitted as evidence.

The marijuana examined by the forensic chemist, which was contained in three big
round tin cans, two jute sacks (there was really only one jute sack colored light
green which was confiscated but since one of the plastic sacks [green] appeared to
be tattered, some of its contents were transferred to a white jute sack), (T.S.N.,
June 23, 1987, p. 5) and two plastic bags colored yellow and green (T.S.N., June
23, 1987, p. 3), was positively identified to be the same as those confiscated from
the appellants. This is very clear from the testimony of Lt. Valeroso who
stated:jgc:chanrobles.com.ph
"x       x       x

Q When you went down, where were these two suspects, as you said?

A They were sitted (sic) at the front seat.

Q Front seat of what?

A The jeep, sir.

Q And did you ask or see what was inside the jeep?

A Yes.

Q And what were those?

A It was all suspected marijuana dried leaves contained in three big cans, one sack
colored green, two sacks colored yellow and green." (Italics supplied, T.S.N.,
September 16, 1987, p. 4)

Lt. Valeroso further testified that Exhibits "B" (yellow plastic bag), "C" light green
jute sack, "D" (green plastic bag), "E" (one big can), "F" (second can), "G" (third
can) were, indeed, the same articles which he saw at the back of the jeepney of the
appellants. (T.S.N., September 16, 1987, p. 5)

One of the appellants, Moises Maspil, even admitted that the articles identified by
Lt. Valeroso in his testimony were indeed, the same articles confiscated from their
jeepney at Sayangan, Atok, Benguet. (T.S.N., February 24, 1988, pp. 34-35)

Moreover, the words "more or less" following the weight in kilos of the marijuana
in the questioned information declare that the number of kilos stated therein is just
an approximation. It can therefore be a little lighter or heavier. The slight
discrepancy is not material.

Another ground stated by the appellants for the inadmissibility in evidence of the
confiscated marijuana is that the marijuana allegedly seized from them was a
product of an unlawful search without a warrant.

In the case of Valmonte v. de Villa, G.R. No. 83988, September 29, 1989, the
Court held that:jgc:chanrobles.com.ph

"x       x       x
True, the manning of checkpoints by the military is susceptible of abuse by the
men in uniform, in the same manner that all governmental power is susceptible of
abuse. But at the cost of occasional inconvenience, discomfort and even irritation
to the citizen, the checkpoints during these abnormal times, when conducted within
reasonable limits are part of the price we pay for an orderly society and a peaceful
community."cralaw virtua1aw library

The search was conducted within reasonable limits. There was information that a
sizeable volume of marijuana will be transported to take advantage of the All
Saints Day holiday wherein there will be a lot of people going to and from Baguio
City (T.S.N., September 16, 1987, p. 6). In fact, during the three day (October 30,
1986 to November 1, 1986) duration of the checkpoint, there were also other drug
related arrests made aside from that of the two appellants.

But even without the Valmonte ruling, the search would still be valid. This case
involves a search incident to a lawful arrest which is one of the exceptions to the
general rule requiring a search warrant. This exception is embodied in Section 12
of Rule 126 of the 1985 Rules on Criminal Procedure which
provides:jgc:chanrobles.com.ph

"SEC. 12. Search incident to lawful arrest. — A person lawfully arrested may be
searched for dangerous weapons or anything which may be used as proof of the
commission of an offense, without a search warrant."cralaw virtua1aw library

and Rule 113, Section 5 (1) which state:jgc:chanrobles.com.ph

"SEC. 5. Arrest without warrant; when lawful. — A peace officer or a private


person may, without a warrant, arrest a person:chanrob1es virtual 1aw library

(a) When, in his presence, the person to be arrested has committed, is actually
committing, or is attempting to commit an offense."cralaw virtua1aw library

This case falls squarely within the exceptions. The appellants were caught in
flagrante delicto since they were transporting the prohibited drugs at the time of
their arrest. (People v. Tangliben, G.R. No. 63630, April 6, 1990) A crime was
actually being committed.

The appellants, however, cite the case of People v. Aminnudin, (163 SCRA 402
[1988]). In said case, the PC officers received information that the accused-
appellant, on board a vessel bound for Iloilo City, was carrying marijuana. When
the accused-appellant was descending the gangplank, the PC officers detained him
and inspected the bag that he was carrying and found marijuana. The Court ruled
that since the marijuana was seized illegally, it is inadmissible in evidence.
There are certain facts of the said case which are not present in the case before us.
In the Aminnudin case, the records showed that there was sufficient time and
adequate information for the PC officers to have obtained a warrant. The officers
knew the name of the accused, that the accused was on board M/V Wilcon 9,
bound to Iloilo and the exact date of the arrival of the said vessel.

On the other hand, in this case there was no information as to the exact description
of the vehicle and no definite time of the arrival. A jeepney cannot be equated with
a passenger ship on the high seas. The ruling in the Aminnudin case, is not
applicable to the case at bar.

As for the fifth and last assigned error we agree with the Solicitor General
that:jgc:chanrobles.com.ph

"Examination of the testimonies of appellants show that they admit the fact that the
confiscated marijuana was taken from their jeep while they were transporting it
from Abatan, Buguias, Benguet to Baguio City. This being so, the burden of the
prosecution to prove illegal transportation of prohibited drugs punished under
Section 4 of RA 6425, as amended, has been satisfactorily discharged. The rule in
civil as well as in criminal cases is that each party must prove his own affirmative
allegations. The prosecution avers the guilt of the accused who is presumed to be
innocent until the contrary is proved. Therefore, the prosecution must prove such
guilt by establishing the existence of all elements of the crime charged. But facts
judicially known, presumed, admitted or confessed need not be proved. (Rule 129,
Sec. 4, Rules on Evidence) (Appellee’s Brief, p. 26-27)

WHEREFORE, the guilt of the appellants having been proved beyond reasonable
doubt, the appealed decision is hereby AFFIRMED.

SO ORDERED.

First Issue: Whether or not the seized marijuana was a product of an


unlawful search without a warrant thus making it inadmissible as evidence.

Ruling: No, the search was conducted within reasonable limits. 

Explanation:

In the case of Valmonte v. de Villa, G.R. No. 83988, September 29, 1989, the Court
held that:

xxx xxx xxx


True, the manning of checkpoints by the military is susceptible of
abuse by the men in uniform, in the same manner that all government
power is susceptible of abuse. But at the cost of occasional
inconvenience, discomfort and even irritation to the citizen, the
checkpoints during these abnormal times, when conducted within
reasonable limits are part of the price we pay for an orderly society
and a peaceful community.

The search was conducted within reasonable limits. There was information that
a sizeable volume of marijuana will be transported to take advantage of the All
Saints Day holiday wherein there will be a lot of people going to and from Baguio
City. In fact, during the three day (October 30, 1986 to November 1, 1986)
duration of the checkpoint, there were also other drug related arrests made aside
from that of the two appellants.

But even without the Valmonte ruling, the search would still be valid. This
case involves a search incident to a lawful arrest which is one of the exceptions
to the general rule requiring a search warrant. This exception is embodied in
Section 12 of Rule 126 of the 1985 Rules on Criminal Procedure which
provides:

Sec. 12. Search incident to lawful arrest. A person lawfully


arrested may be searched for dangerous weapons or anything
which may be used as proof of the commission of an offense,
without a search warrant.

and Rule 113, Section 5 (11) which state:

Sec. 5. Arrest without warrant; when lawful. A peace officer or a


private person may, without a warrant, arrest a person:

(a) When, in his presence, the person to be arrested has


committed, is actually committing, or is attempting to commit an
offense.

This case falls squarely within the exceptions. The appellants were caught in
flagrante delicto since they were transporting the prohibited drugs at the time of
their arrest. 

Second Issue: Whether the rule of seizure in a ship would be applicable in a


seizure in a moving land vehicle.

Ruling: No, seizure in a ship is different from seizure in a moving land vehicle.
Explanation: 

The appellants, cited the case of People V. Aminnudin,. In said case, the PC
officers received information that the accused-appellant, on board a vessel bound
for Iloilo City, was carrying marijuana. When the accused-appellant was
descending the gangplank, the PC officers detained him and inspected the bag that
he was carrying and found marijuana. The Court ruled that since the marijuana was
seized illegally, it is inadmissible in evidence.

There are certain facts of the said case which are not present in the case
before us. In the Aminnudin case, the records showed that there was sufficient time
and adequate information for the PC officers to have obtained a warrant. The
officers knew the name of the accused, that the accused was on board M/V Wilcon
9, bound to Iloilo and the exact date of the arrival of the said vessel.

On the other hand, in this case there was no information as to the exact
description of the vehicle and no definite time of the arrival. A jeepney cannot
be equated with a passenger ship on the high seas. The ruling in the
Aminnudin case, is not applicable to the case at bar.

People v. Tangliben, G.R. No. L-63630 April 6, 1990 [CRITIQUE]

This is an appeal from the decision of the Regional Trial Court, Branch 41, Third
Judicial Region at San Fernando, Pampanga, Branch 41, finding appellant Medel
Tangliben y Bernardino guilty beyond reasonable doubt of violating Section 4,
Article II of Republic Act 6425 (Dangerous Drugs Act of 1972 as amended) and
sentencing him to life imprisonment, to pay a fine of P20,000 and to pay the costs.

The information filed against the appellant alleged:

That on or about the 2nd day of March, 1982, in the municipality of San
Fernando, Province of Pampanga, Philippines, and within the jurisdiction of
this Honorable Court, the above-named accused MEDEL TANGLIBEN y
BERNARDINO, knowing fully well that Marijuana is a prohibited drug, did
then and there willfully, unlawfully and feloniously have his possession,
control and custody one (1) bag of dried marijuana leaves with an
approximate weight of one (1) kilo and to transport (sic) the same to
Olongapo City, without authority of law to do so. (At p. 6, Rollo)

The prosecution's evidence upon which the finding of guilt beyond reasonable
doubt was based is narrated by the trial court as follows:

It appears from the evidence presented by the prosecution that in the late
evening of March 2, 1982, Patrolmen Silverio Quevedo and Romeo L.
Punzalan of the San Fernando Police Station, together with Barangay Tanod
Macario Sacdalan, were conducting surveillance mission at the Victory
Liner Terminal compound located at Barangay San Nicolas, San Fernando,
Pampanga; that the surveillance was aimed not only against persons who
may commit misdemeanors at the said place but also on persons who may
be engaging in the traffic of dangerous drugs based on informations supplied
by informers; that it was around 9:30 in the evening that said Patrolmen
noticed a person caring a traveling bag (Exhibit G) who was acting
suspiciously and they confronted him; that the person was requested by
Patrolmen Quevedo and Punzalan to open the red traveling bag but the
person refused, only to accede later on when the patrolmen identified
themselves; that found inside the bag were marijuana leaves (Exhibit B)
wrapped in a plastic wrapper and weighing one kilo, more or less; that the
person was asked of his name and the reason why he was at the said place
and he gave his name as Medel Tangliben and explained that he was waiting
for a ride to Olongapo City to deliver the marijuana leaves; that the accused
was taken to the police headquarters at San Fernando, Pampanga, for further
investigation; and that Pat. Silverio Quevedo submitted to his Station
Commander his Investigator's Report (Exhibit F).

It appears also from the prosecution's evidence that in the following morning
or on March 3, 1982, Pat. Silverio Quevedo asked his co-policeman Pat.
Roberto Quevedo, who happens to be his brother and who has had special
training on narcotics, to conduct a field test on a little portion of the
marijuana leaves and to have the remaining portion examined by the PCCL
at Camp Olivas, San Fernando, Pampanga; that Pat. Roberto Quevedo
conducted a field test (Exhibit H) on the marijuana leaves and found
positive result for marijuana (Exhibit E); that the remaining bigger quantity
of the marijuana leaves were taken to the PCCL at Camp Olivas by Pat.
Roberto Quevedo that same day of March 3, 1982 (Exhibit A and A-1) and
when examined, the same were also found to be marijuana (Exhibit C and
C-1). (At pp. 9-10, Rollo)

Only the accused testified in his defense. His testimony is narrated by the trial
court as follows:

The accused declared that he got married on October 25, 1981 and his wife
begot a child on June 10, 1982; that he was formerly employed in the
poultry farm of his uncle Alejandro Caluma in Antipolo, Rizal; that he is
engaged in the business of selling poultry medicine and feeds, including
chicks, and used to conduct his business at Taytay, Rizal; that he goes to
Subic at times in connection with his business and whenever he is in Subic,
he used to buy C-rations from one Nena Ballon and dispose the same in
Manila; that he never left his residence at Antipolo, Rizal, on March 2,
1982; that on March 3, 1982, he went to Subic to collect a balance of
P100.00 from a customer thereat and to buy C-rations; that he was able to
meet Nena Ballon at 6:00 o'clock in the evening and he stayed in Nena's
house up to 8:00 o'clock because he had a drinking spree with Nena's son;
that he tried to catch the 8:00 o'clock trip to Manila from Olongapo City but
he failed and was able to take the bus only by 9:00 o'clock that evening that
it was a Victory Liner Bus that he rode and because he was tipsy, he did not
notice that the bus was only bound for San Fernando, Pampanga; that upon
alighting at the Victory Liner Compound at San Fernando, Pampanga he
crossed the street to wait for a bus going to Manila; that while thus waiting
for a bus, a man whom he came to know later as Pat. Punzalan, approached
him and asked him if he has any residence certificate; that when he took out
his wallet, Pat. Punzalan got the wallet and took all the money inside the
wallet amounting to P545.00; that Pat. Punzalan told him that he'll be taken
to the municipal building for verification as he may be an NPA member;
that at the municipal building, he saw a policeman, identified by him later as
Pat. Silverio Quevedo, sleeping but was awakened when he arrived that Pat.
Quevedo took him upstairs and told him to take out everything from his
pocket saying that the prisoners inside the jail may get the same from him;
that inside his pocket was a fifty-peso bill and Pat. Quevedo took the same,
telling him that it shall be returned to him but that it was never returned to
him; that he was thereafter placed under detention and somebody told him
that he is being charged with possession of marijuana and if he would like to
be bailed out, somebody is willing to help him; and, that when he was
visited by his wife, he told his wife that Patrolman Silverio Quevedo took
away all his money but he told his wife not to complain anymore as it would
be useless. (Rollo, pp. 10-11)

Appellant, through counsel de oficio Atty. Enrique Chan, raised the lone
assignment of error in his appeal:

THE COURT A QUO ERRED IN CONVICTING THE ACCUSED-


APPELLANT AND FINDING HIM GUILTY OF THE CRIME
CHARGED ON INSUFFICIENT AND DOUBTFUL EVIDENCE. (At p.
48, Rollo)

The Solicitor-General likewise filed his brief, basically reiterating ating the lower
court's findings.

However, before this Court had the chance to act on appeal, counsel de oficio Atty.
Enrique Chan died. Thereafter, this court appointed a new counsel de oficio, Atty.
Katz Tierra and pursuant thereto, the Deputy Clerk of Court, in behalf of the Clerk
of Court, required the new counsel to file her appellant's brief. The latter complied
and, in her brief, raised the following assignment of errors:

I
THE LOWER COURT ERRED IN ADMITTING AS EVIDENCE THE
PACKAGE OF MARIJUANA ALLEGEDLY SEIZED FROM
DEFENDANT-APPELLANT AS IT WAS A PRODUCT OF AN
UNLAWFUL SEARCH WITHOUT A WARRANT.

II

THE LOWER COURT ERRED IN ADMITTING AS EVIDENCE THE


ALLEGED PACKAGE OF MARIJUANA LEAVES AS THE LEAVES
SUPPOSEDLY SEIZED FROM ACCUSED WHEN IT WAS NEVER
AUTHENTICATED.

III

THE LOWER COURT ERRED IN NOT RULING THAT THE


PROSECUTION FAILED TO PROVE THE GUILT OF DEFENDANT-
APPELLANT. (At pp. 92-93, Rollo)

It is contended that the marijuana allegedly seized from the accused was a product
of an unlawful search without a warrant and is therefore inadmissible in evidence.

This contention is devoid of merit.

One of the exceptions to the general rule requiring a search warrant is a search
incident to a lawful arrest. Thus, Section 12 of Rule 126 of the 1985 Rules on
Criminal Procedure provides:

Section 12. Search incident to a lawful arrest. A person lawfully arrested


may be searched for dangerous weapons or anything which may be used as
proof of the commission of an offense, without a search warrant.

Meanwhile, Rule 113, Sec. 5(a) provides:

. . . A peace officer or a private person may, without a warrant, arrest a


person:

(a) When, in his presence, the person to be arrested has committed, is


actually committing, or is attempting to commit an offense.

Accused was caught in flagrante, since he was carrying marijuana at the time of his
arrest. This case therefore falls squarely within the exception. The warrantless
search was incident to a lawful arrest and is consequently valid.

In the case of People v. Claudia, 160 SCRA 646, [1988] this Court, confronted
with the same issue, held that:
Appellant Claudio was caught transporting prohibited drugs. Pat. Daniel did
not need a warrant to arrest Claudio as the latter was caught in flagrante
delicto. The warrantless search being an incident to a lawful arrest is in itself
lawful. (Nolasco V. Paño, 147 SCRA 509). Therefore, there was no
infirmity in the seizure of the 1.1 kilos of marijuana.

We are not unmindful of the decision of this Court in People v. Amininudin, 163


SCRA 402 [1988]. In that case the PC officers had earlier received a tip from an
informer that accused-appellant. was on board a vessel bound for Iloilo City and
was carrying marijuana. Acting on this tip, they waited for him one evening,
approached him as he descended from the gangplank, detained him and inspected
the bag he was carrying. Said bag contained marijuana leaves. The Court held that
the marijuana could not be admitted in evidence since it was seized illegally. The
records show, however, that there were certain facts, not sing in the case before us,
which led the Court to declare the seizure as invalid. As stated therein:

The present case presented no such urgency From the conflicting


declarations of the PC witnesses, it is clear that they had at react two days
within which they could have obtained a warrant of arrest and search
Aminnudin who was coming to Iloilo on the M/V Wilcon 9. His name was
known. The vehicle was identified. The date of its arrival was certain. And
from the information they had received, they could have persuaded a judge
that there was probable cause, indeed, to justify the issuance of a warrant.
Yet they did nothing. No effort was made to comply with the law. The Bill
of Rights was ignored altogether because the PC lieutenant who was the
head of the arresting team, had determined on his own authority that a
"search warrant was not necessary."

In contrast, the case before us presented urgency. Although the trial court's
decision did not mention it, the transcript of stenographic notes reveals that there
was an informer who pointed to the accused-appellant as carrying marijuana.
(TSN, pp. 52-53) Faced with such on-the-spot information, the police officers had
to act quickly. There was not enough time to secure a search warrant. We cannot
therefore apply the ruling in Aminnudin to the case at bar. To require search
warrants during on-the-spot apprehensions of drug pushers, illegal possessors of
firearms, jueteng collectors, smugglers of contraband goods, robbers, etc. would
make it extremely difficult, if not impossible to contain the crimes with which
these persons are associated.

Accused-appellant likewise asserts that the package of marijuana leaves


supposedly seized from him was never authenticated and therefore should not have
been admitted as evidence. He capitalizes on the fact that the marijuana package
brought by patrolman Roberto Quevedo to the PC Crime Laboratory for
examination did not contain a tag bearing the name of the accused. We rule,
however, that since Patrolman Quevedo testified that he gave the marijuana
package together with a letter-request for examination, and the forensic chemist
Marilene Salangad likewise testified that she received the marijuana together with
the letter-request and said letter-request bore the name of the accused, then the
requirements of proper authentication of evidence were sufficiently complied with.
The marijuana package examined by the forensic checklist was satisfactorily
identified as the one seized from accused.

Even assuming arguendo that the marijuana sent to the PC Crime Laboratory was
not properly authenticated, still, we cannot discount the separate field test
conducted by witness Roberto Quevedo which yielded positive results for
marijuana.

Lastly, the appellant claims that the evidence upon which he was convicted was
insufficient and doubtful and that the prosecution failed to prove his guilt.

In attacking the sufficiency of evidence, the appellant avers that the informer
should have been presented before the lower court. We discard this argument as a
futile attempt to revive an already settled issue. This Court has ruled in several
cases that non-presentation of the informer, where his testimony would be merely
corroborative or cumulative, is not fatal to the prosecution's case. (People v. Asio,
G.R. No. 84960, September 1, 1989; (People v. Viola, G.R. No. 64262, March 16,
1989; People v. Capulong, 160 SCRA 533 [1988]; People v. Cerelegia, 147 SCRA
538).

As to doubtfulness of evidence, well-settled is the rule that findings of the trial


court on the issue of credibility of witnesses and their testimonies are entitled to
great respect and accorded the highest consideration by the appellate court. Since
credibility is a matter that is peculiarly within the province of the trial judge, who
had first hand opportunity to watch and observe the demeanor and behavior of
witnesses both for the prosecution and the defense at the time of their testimony
(People v. Tejada, G.R. No. 81520, February 21, 1989; People v. Turla, 167 SCRA
278), we find no reason to disturb the following findings:

The testimony of prosecution witnesses Patrolmen Silverio Quevedo and


Romeo Punzalan are positive and sufficiently clean to show the commission
by the accused of the offense herein chatted. These prosecution witnesses
have no motive to fabricate the facts and to foist a very serious offense
against the accused. The knowledge on what these witnesses testified to
were (sic) acquired by them in the official performance of their duties and
then, (sic) being no showing that they are prejudiced against the accused,
their testimonies deserve full credit.

The testimonies of the afore-mentioned petitioner that what they found in


the possession of the accused were marijuana leaves were corroborated by
the examination findings conducted by Pat. October to Salangad of the
PCCL, with station at camp Olivas, San Fernando, Pampanga (Exhibits C
and C-1). (Rollo, p. 11)

Moreover, if there is truth in the testimony of the accused to the effect that
Pat. Punzalan got all the money from his wallet when he was accosted at the
Victory Liner Terminal and was told just to keep quiet otherwise he will be
"salvaged" why will Pat. Punzalan still bring the accused to the municipal
Building for interrogation and/or verification? Would not Pat. Punzalan be
exposing his identity to the accused? This is unnatural. And this is also true
on the testimony to the accused that Pat. Silverio Quevedo got his fifty-peso
bill arid never returned the same to him. If the policemen really got any
money from the accused and that the marijuana leaves do not belong to the
accused, why will the two policemen still produce in Court as evidence that
expensive-looking traveling red bag (Exhibit G) taken from the accused and
which contained the marijuana in question if the instant case is a mere
fabrication?

As already stated, all the evidence, oral and documentary, presented by the
prosecution in this case were all based on personal knowledge acquired by
the prosecution witnesses in the regular performance of their official duties
and there is nothing in their testimonies to show that they are bias (sic) or
that they have any prejudice against the herein accused. Between the
testimonies of these prosecution witnesses and that of the uncorroborated
and self-serving testimony of the accused, the former should prevail. (Rollo,
p. 13)

Likewise, the appellant chose to limit his defense to his own testimony. He could
have availed himself through compulsory court processes of several witnesses to
buttress his defense. Since not one other witness was presented nor was any
justification for the non-appearance given, the inadequacy of his lone and
uncorroborated testimony remains. It cannot prevail vis-a-vis the positive
testimonies given by the prosecution witnesses.

Moreover, the appellant's having jumped bail is akin to flight which, as correctly
observed by the lower court, is an added circumstance tending to establish his
guilt.

We take exception, however, to the trial court's finding that:

The dried marijuana leaves found in the possession of the accused weighs
one (1) kilo, more or less. The intent to transport the same is clear from the
testimony of Pat. Silverio Quevedo who declared, among other things, that
when he confronted the accused that night, the latter told him that he
(accused) is bringing the marijuana leaves to Olongapo City. Moreover,
considering the quantity of the marijuana leaves found in the possession of
the accused and the place he was arrested which is at San Fernando,
Pampanga, a place where the accused is not residing, it can be said that the
intent to transport the marijuana leaves has been clearly established. (Rollo,
pp. 13-14)

The alleged extrajudicial confession of the accused which, on the other hand, he
categorically denied in court, that he is transporting the marijuana leaves to
Olongapo City cannot be relied upon. Even assuming it to be true, the extrajudicial
confession cannot be admitted because it does not appear in the records that the
accused, during custodial investigation, was apprised of his rights to remain silent
and to counsel and to be informed of such rights. In People v. Duero 104 SCRA
379 [1981], the Court pronounced that "inasmuch as the prosecution failed to prove
that before Duero made his alleged oral confession he was informed of his rights to
remain silent and to have counsel and because there is no proof that he knowingly
and intelligently waived those rights, his confession is inadmissible in evidence.
This ruling was reiterated in People v. Tolentino, 145 SCRA 597 [1986], where the
Court added that:

In effect, the Court not only abrogated the rule on presumption of regularity
of official acts relative to admissibility of statements taken during in-
custody interrogation but likewise dispelled any doubt as to the full adoption
of the Miranda doctrine in this jurisdiction. It is now incumbent upon the
prosecution to prove during a trial that prior to questioning, the confessant
was warned of his constitutionally protected rights.

The trial judge likewise found the marijuana to weigh one kilo, more or less, and
from this finding extracted a clear intent to transport the marijuana leaves. It may
be pointed out, however, that although the information stated the weight to be
approximately one kilo, the forensic chemist who examined the marijuana leaves
testified that the marijuana weighed only 600 grams Such amount is not a
considerable quantity as to conclusively confer upon the accused an intent to
transport the marijuana leaves.

Nor can it be said that the intent to transport is clearly established from the fact that
the accused was arrested at San Fernando, Pampanga, a place which is not his
residence. Conviction of a crime with an extremely severe penalty must be based
on evidence which is clearer and more convincing than the inferences in this case.

What was therefore proved beyond reasonable doubt is not his intent to transport
the marijuana leaves but his actual session.

The offense committed by the appellant is possession of marijuana under Section 8


of Republic Act No. 6425 (Dangerous Drugs Act of 1972 as amended).
WHEREFORE, the judgment of conviction by the trial court is hereby
AFFIRMED but MODIFIED. The appellant is sentenced to suffer the penalty of
imprisonment ranging from six (6) years and one (1) day to twelve (12) years and
fine of Six Thousand (P6,000.00) Pesos.

SO ORDERED.

CASES:
Terry v. Ohio, 392 U.S. 1 (Further reading: Floyd v. City of New York, 959 F.
Supp. 2d 540, August 12, 2013)

MR. CHIEF JUSTICE WARREN delivered the opinion of the Court.

This case presents serious questions concerning the role of the Fourth Amendment
in the confrontation on the street between the citizen and the policeman
investigating suspicious circumstances.

Petitioner Terry was convicted of carrying a concealed weapon and sentenced to


the statutorily prescribed term of one to three years in the penitentiary. [Footnote
1] Following

[5]

the denial of a pretrial motion to suppress, the prosecution introduced in evidence


two revolvers and a number of bullets seized from Terry and a codefendant,
Richard Chilton, [Footnote 2] by Cleveland Police Detective Martin McFadden. At
the hearing on the motion to suppress this evidence, Officer McFadden testified
that, while he was patrolling in plain clothes in downtown Cleveland at
approximately 2:30 in the afternoon of October 31, 1963, his attention was
attracted by two men, Chilton and Terry, standing on the corner of Huron Road
and Euclid Avenue. He had never seen the two men before, and he was unable to
say precisely what first drew his eye to them. However, he testified that he had
been a policeman for 39 years and a detective for 35, and that he had been assigned
to patrol this vicinity of downtown Cleveland for shoplifters and pickpockets for
30 years. He explained that he had developed routine habits of observation over the
years, and that he would "stand and watch people or walk and watch people at
many intervals of the day." He added: "Now, in this case, when I looked over, they
didn't look right to me at the time."

His interest aroused, Officer McFadden took up a post of observation in the


entrance to a store 300 to 400 feet

[6]
away from the two men. "I get more purpose to watch them when I seen their
movements," he testified. He saw one of the men leave the other one and walk
southwest on Huron Road, past some stores. The man paused for a moment and
looked in a store window, then walked on a short distance, turned around and
walked back toward the corner, pausing once again to look in the same store
window. He rejoined his companion at the corner, and the two conferred briefly.
Then the second man went through the same series of motions, strolling down
Huron Road, looking in the same window, walking on a short distance, turning
back, peering in the store window again, and returning to confer with the first man
at the corner. The two men repeated this ritual alternately between five and six
times apiece -- in all, roughly a dozen trips. At one point, while the two were
standing together on the corner, a third man approached them and engaged them
briefly in conversation. This man then left the two others and walked west on
Euclid Avenue. Chilton and Terry resumed their measured pacing, peering, and
conferring. After this had gone on for 10 to 12 minutes, the two men walked off
together, heading west on Euclid Avenue, following the path taken earlier by the
third man.

By this time, Officer McFadden had become thoroughly suspicious. He testified


that, after observing their elaborately casual and oft-repeated reconnaissance of the
store window on Huron Road, he suspected the two men of "casing a job, a stick-
up," and that he considered it his duty as a police officer to investigate further. He
added that he feared "they may have a gun." Thus, Officer McFadden followed
Chilton and Terry and saw them stop in front of Zucker's store to talk to the same
man who had conferred with them earlier on the street corner. Deciding that the
situation was ripe for direct action, Officer McFadden approached the three men,
identified

[7]

himself as a police officer and asked for their names. At this point, his knowledge
was confined to what he had observed. He was not acquainted with any of the three
men by name or by sight, and he had received no information concerning them
from any other source. When the men "mumbled something" in response to his
inquiries, Officer McFadden grabbed petitioner Terry, spun him around so that
they were facing the other two, with Terry between McFadden and the others, and
patted down the outside of his clothing. In the left breast pocket of Terry's
overcoat, Officer McFadden felt a pistol. He reached inside the overcoat pocket,
but was unable to remove the gun. At this point, keeping Terry between himself
and the others, the officer ordered all three men to enter Zucker's store. As they
went in, he removed Terry's overcoat completely, removed a .38 caliber revolver
from the pocket and ordered all three men to face the wall with their hands raised.
Officer McFadden proceeded to pat down the outer clothing of Chilton and the
third man, Katz. He discovered another revolver in the outer pocket of Chilton's
overcoat, but no weapons were found on Katz. The officer testified that he only
patted the men down to see whether they had weapons, and that he did not put his
hands beneath the outer garments of either Terry or Chilton until he felt their guns.
So far as appears from the record, he never placed his hands beneath Katz' outer
garments. Officer McFadden seized Chilton's gun, asked the proprietor of the store
to call a police wagon, and took all three men to the station, where Chilton and
Terry were formally charged with carrying concealed weapons.

On the motion to suppress the guns, the prosecution took the position that they had
been seized following a search incident to a lawful arrest. The trial court rejected
this theory, stating that it "would be stretching the facts beyond reasonable
comprehension" to find that Officer

[8]

McFadden had had probable cause to arrest the men before he patted them down
for weapons. However, the court denied the defendants' motion on the ground that
Officer McFadden, on the basis of his experience, "had reasonable cause to believe
. . . that the defendants were conducting themselves suspiciously, and some
interrogation should be made of their action." Purely for his own protection, the
court held, the officer had the right to pat down the outer clothing of these men,
who he had reasonable cause to believe might be armed. The court distinguished
between an investigatory "stop" and an arrest, and between a "frisk" of the outer
clothing for weapons and a full-blown search for evidence of crime. The frisk, it
held, was essential to the proper performance of the officer's investigatory duties,
for, without it, "the answer to the police officer may be a bullet, and a loaded pistol
discovered during the frisk is admissible."

After the court denied their motion to suppress, Chilton and Terry waived jury trial
and pleaded not guilty. The court adjudged them guilty, and the Court of Appeals
for the Eighth Judicial District, Cuyahoga County, affirmed. State v. Terry, 5 Ohio
App.2d 122, 214 N.E.2d 114 (1966). The Supreme Court of Ohio dismissed their
appeal on the ground that no "substantial constitutional question" was involved.
We granted certiorari, 387 U.S. 929 (1967), to determine whether the admission of
the revolvers in evidence violated petitioner's rights under the Fourth Amendment,
made applicable to the States by the Fourteenth. Mapp v. Ohio, 367 U. S.
643 (1961). We affirm the conviction.

The Fourth Amendment provides that "the right of the people to be secure in their
persons, houses, papers, and effects, against unreasonable searches and seizures,
shall not be violated. . . ." This inestimable right of

[9]
personal security belongs as much to the citizen on the streets of our cities as to the
homeowner closeted in his study to dispose of his secret affairs. For as this Court
has always recognized,

"No right is held more sacred, or is more carefully guarded, by the common law
than the right of every individual to the possession and control of his own person,
free from all restraint or interference of others, unless by clear and unquestionable
authority of law." Union Pac. R. Co. v. Botsford, 141 U. S. 250, 251 (1891).

We have recently held that "the Fourth Amendment protects people, not
places," Katz v. United States, 389 U. S. 347, 351 (1967), and wherever an
individual may harbor a reasonable "expectation of privacy," id. at 361 (MR.
JUSTICE HARLAN, concurring), he is entitled to be free from unreasonable
governmental intrusion. Of course, the specific content and incidents of this right
must be shaped by the context in which it is asserted. For "what the Constitution
forbids is not all searches and seizures, but unreasonable searches and
seizures." Elkins v. United States, 364 U. S. 206, 222 (1960). Unquestionably
petitioner was entitled to the protection of the Fourth Amendment as he walked
down the street in Cleveland. Beck v. Ohio, 379 U. S. 89 (1964); Rios v. United
States, 364 U. S. 253 (1960); Henry v. United States, 361 U. S. 98 (1959); United
States v. Di Re, 332 U. S. 581 (1948); Carroll v. United States, 267 U. S.
132 (1925). The question is whether, in all the circumstances of this on-the-street
encounter, his right to personal security was violated by an unreasonable search
and seizure.

We would be less than candid if we did not acknowledge that this question thrusts
to the fore difficult and troublesome issues regarding a sensitive area of police
activity -- issues which have never before been squarely

[10]

presented to this Court. Reflective of the tensions involved are the practical and
constitutional arguments pressed with great vigor on both sides of the public
debate over the power of the police to "stop and frisk" -- as it is sometimes
euphemistically termed -- suspicious persons.

On the one hand, it is frequently argued that, in dealing with the rapidly unfolding
and often dangerous situations on city streets, the police are in need of an
escalating set of flexible responses, graduated in relation to the amount of
information they possess. For this purpose, it is urged that distinctions should be
made between a "stop" and an "arrest" (or a "seizure" of a person), and between a
"frisk" and a "search." [Footnote 3] Thus, it is argued, the police should be allowed
to "stop" a person and detain him briefly for questioning upon suspicion that he
may be connected with criminal activity. Upon suspicion that the person may be
armed, the police should have the power to "frisk" him for weapons. If the "stop"
and the "frisk" give rise to probable cause to believe that the suspect has committed
a crime, then the police should be empowered to make a formal "arrest," and a full
incident "search" of the person. This scheme is justified in part upon the notion that
a "stop" and a "frisk" amount to a mere "minor inconvenience and petty indignity,"
[Footnote 4] which can properly be imposed upon the

[11]

citizen in the interest of effective law enforcement on the basis of a police officer's
suspicion. [Footnote 5]

On the other side, the argument is made that the authority of the police must be
strictly circumscribed by the law of arrest and search as it has developed to date in
the traditional jurisprudence of the Fourth Amendment. [Footnote 6] It is
contended with some force that there is not -- and cannot be -- a variety of police
activity which does not depend solely upon the voluntary cooperation of the
citizen, and yet which stops short of an arrest based upon probable cause to make
such an arrest. The heart of the Fourth Amendment, the argument runs, is a severe
requirement of specific justification for any intrusion upon protected personal
security, coupled with a highly developed system of judicial controls to enforce
upon the agents of the State the commands of the Constitution. Acquiescence by
the courts in the compulsion inherent

[12]

in the field interrogation practices at issue here, it is urged, would constitute an


abdication of judicial control over, and indeed an encouragement of, substantial
interference with liberty and personal security by police officers whose judgment is
necessarily colored by their primary involvement in "the often competitive
enterprise of ferreting out crime." Johnson v. United States, 333 U. S. 10, 14
(1948). This, it is argued, can only serve to exacerbate police-community tensions
in the crowded centers of our Nation's cities. [Footnote 7]

In this context, we approach the issues in this case mindful of the limitations of the
judicial function in controlling the myriad daily situations in which policemen and
citizens confront each other on the street. The State has characterized the issue here
as "the right of a police officer . . . to make an on-the-street stop, interrogate and
pat down for weapons (known in street vernacular as 'stop and frisk'). [Footnote 8]"
But this is only partly accurate. For the issue is not the abstract propriety of the
police conduct, but the admissibility against petitioner of the evidence uncovered
by the search and seizure. Ever since its inception, the rule excluding evidence
seized in violation of the Fourth Amendment has been recognized as a principal
mode of discouraging lawless police conduct. See Weeks v. United States, 232 U.
S. 383, 391-393 (1914). Thus, its major thrust is a deterrent one, see Linkletter v.
Walker, 381 U. S. 618, 629-635 (1965), and experience has taught that it is the
only effective deterrent to police misconduct in the criminal context, and that,
without it, the constitutional guarantee against unreasonable searches and seizures
would be a mere "form of words." Mapp v. Ohio, 367 U. S. 643, 655 (1961). The
rule also serves another vital function -- "the imperative of judicial
integrity." Elkins

[13]

v. United States, 364 U. S. 206, 222 (1960). Courts which sit under our
Constitution cannot and will not be made party to lawless invasions of the
constitutional rights of citizens by permitting unhindered governmental use of the
fruits of such invasions. Thus, in our system, evidentiary rulings provide the
context in which the judicial process of inclusion and exclusion approves some
conduct as comporting with constitutional guarantees and disapproves other
actions by state agents. A ruling admitting evidence in a criminal trial, we
recognize, has the necessary effect of legitimizing the conduct which produced the
evidence, while an application of the exclusionary rule withholds the constitutional
imprimatur.

The exclusionary rule has its limitations, however, as a tool of judicial control. It
cannot properly be invoked to exclude the products of legitimate police
investigative techniques on the ground that much conduct which is closely similar
involves unwarranted intrusions upon constitutional protections. Moreover, in
some contexts, the rule is ineffective as a deterrent. Street encounters between
citizens and police officers are incredibly rich in diversity. They range from wholly
friendly exchanges of pleasantries or mutually useful information to hostile
confrontations of armed men involving arrests, or injuries, or loss of life.
Moreover, hostile confrontations are not all of a piece. Some of them begin in a
friendly enough manner, only to take a different turn upon the injection of some
unexpected element into the conversation. Encounters are initiated by the police for
a wide variety of purposes, some of which are wholly unrelated to a desire to
prosecute for crime. [Footnote 9] Doubtless some

[14]

police "field interrogation" conduct violates the Fourth Amendment. But a stern
refusal by this Court to condone such activity does not necessarily render it
responsive to the exclusionary rule. Regardless of how effective the rule may be
where obtaining convictions is an important objective of the police, [Footnote 10]
it is powerless to deter invasions of constitutionally guaranteed rights where the
police either have no interest in prosecuting or are willing to forgo successful
prosecution in the interest of serving some other goal.
Proper adjudication of cases in which the exclusionary rule is invoked demands a
constant awareness of these limitations. The wholesale harassment by certain
elements of the police community, of which minority groups, particularly Negroes,
frequently complain, [Footnote 11] will not be

[15]

stopped by the exclusion of any evidence from any criminal trial. Yet a rigid and
unthinking application of the exclusionary rule, in futile protest against practices
which it can never be used effectively to control, may exact a high toll in human
injury and frustration of efforts to prevent crime. No judicial opinion can
comprehend the protean variety of the street encounter, and we can only judge the
facts of the case before us. Nothing we say today is to be taken as indicating
approval of police conduct outside the legitimate investigative sphere. Under our
decision, courts still retain their traditional responsibility to guard against police
conduct which is overbearing or harassing, or which trenches upon personal
security without the objective evidentiary justification which the Constitution
requires. When such conduct is identified, it must be condemned by the judiciary,
and its fruits must be excluded from evidence in criminal trials. And, of course, our
approval of legitimate and restrained investigative conduct undertaken on the basis
of ample factual justification should in no way discourage the employment of other
remedies than the exclusionary rule to curtail abuses for which that sanction may
prove inappropriate.

Having thus roughly sketched the perimeters of the constitutional debate over the
limits on police investigative conduct in general and the background against which
this case presents itself, we turn our attention to the quite narrow question posed by
the facts before us: whether it is always unreasonable for a policeman to seize a
person and subject him to a limited search for weapons unless there is probable
cause for an arrest.

[16]

Given the narrowness of this question, we have no occasion to canvass in detail the
constitutional limitations upon the scope of a policeman's power when he confronts
a citizen without probable cause to arrest him.

II

Our first task is to establish at what point in this encounter the Fourth Amendment
becomes relevant. That is, we must decide whether and when Officer McFadden
"seized" Terry, and whether and when he conducted a "search." There is some
suggestion in the use of such terms as "stop" and "frisk" that such police conduct is
outside the purview of the Fourth Amendment because neither action rises to the
level of a "search" or "seizure" within the meaning of the Constitution. [Footnote
12] We emphatically reject this notion. It is quite plain that the Fourth Amendment
governs "seizures" of the person which do not eventuate in a trip to the
stationhouse and prosecution for crime -- "arrests" in traditional terminology. It
must be recognized that, whenever a police officer accosts an individual and
restrains his freedom to walk away, he has "seized" that person. And it is nothing
less than sheer torture of the English language to suggest that a careful exploration
of the outer surfaces of a person's clothing all over his or her body in an attempt to
find weapons is not a "search." Moreover, it is simply fantastic to urge that such a
procedure

[17]

performed in public by a policeman while the citizen stands helpless, perhaps


facing a wall with his hands raised, is a "petty indignity." [Footnote 13] It is a
serious intrusion upon the sanctity of the person, which may inflict great indignity
and arouse strong resentment, and it is not to be undertaken lightly. [Footnote 14]

The danger in the logic which proceeds upon distinctions between a "stop" and an
"arrest," or "seizure" of the person, and between a "frisk" and a "search," is
twofold. It seeks to isolate from constitutional scrutiny the initial stages of the
contact between the policeman and the citizen. And, by suggesting a rigid all-or-
nothing model of justification and regulation under the Amendment, it obscures the
utility of limitations upon the scope, as well as the initiation, of police action as a
means of constitutional regulation. [Footnote 15] This Court has held, in

[18]

the past that a search which is reasonable at its inception may violate the Fourth
Amendment by virtue of its intolerable intensity and scope. Kremen v. United
States, 353 U. S. 346 (1957); Go-Bart Importing Co. v.

[19]

United States, 282 U. S. 344, 356-358 (1931); see United States v. Di Re, 332 U. S.


581, 586-587 (1948). The scope of the search must be "strictly tied to and justified
by" the circumstances which rendered its initiation permissible. Warden v.
Hayden, 387 U. S. 294, 310 (1967) (MR. JUSTICE FORTAS, concurring); see,
e.g., Preston v. United States, 376 U. S. 364, 367-368 (1964); Agnello v. United
States, 269 U. S. 20, 30-31 (1925).

The distinctions of classical "stop-and-frisk" theory thus serve to divert attention


from the central inquiry under the Fourth Amendment -- the reasonableness in all
the circumstances of the particular governmental invasion of a citizen's personal
security. "Search" and "seizure" are not talismans. We therefore reject the notions
that the Fourth Amendment does not come into play at all as a limitation upon
police conduct if the officers stop short of something called a "technical arrest" or a
"full-blown search."

In this case, there can be no question, then, that Officer McFadden "seized"
petitioner and subjected him to a "search" when he took hold of him and patted
down the outer surfaces of his clothing. We must decide whether, at that point, it
was reasonable for Officer McFadden to have interfered with petitioner's personal
security as he did. [Footnote 16] And, in determining whether the seizure and
search were "unreasonable," our inquiry

[20]

is a dual one -- whether the officer's action was justified at its inception, and
whether it was reasonably related in scope to the circumstances which justified the
interference in the first place.

III

If this case involved police conduct subject to the Warrant Clause of the Fourth
Amendment, we would have to ascertain whether "probable cause" existed to
justify the search and seizure which took place. However, that is not the case. We
do not retreat from our holdings that the police must, whenever practicable, obtain
advance judicial approval of searches and seizures through the warrant
procedure, see, e.g., Katz v. United States, 389 U. S. 347 (1967); Beck v. Ohio, 379
U. S. 89, 96 (1964); Chapman v. United States, 365 U. S. 610 (1961), or that, in
most instances, failure to comply with the warrant requirement can only be
excused by exigent circumstances, see, e.g., Warden v. Hayden, 387 U. S.
294 (1967) (hot pursuit); cf. Preston v. United States, 376 U. S. 364, 367-368
(1964). But we deal here with an entire rubric of police conduct -- necessarily swift
action predicated upon the on-the-spot observations of the officer on the beat --
which historically has not been, and, as a practical matter, could not be, subjected
to the warrant procedure. Instead, the conduct involved in this case must be tested
by the Fourth Amendment's general proscription against unreasonable searches and
seizures. [Footnote 17]

Nonetheless, the notions which underlie both the warrant procedure and the
requirement of probable cause remain fully relevant in this context. In order to
assess the reasonableness of Officer McFadden's conduct as a general proposition,
it is necessary "first to focus upon

[21]

the governmental interest which allegedly justifies official intrusion upon the
constitutionally protected interests of the private citizen," for there is "no ready test
for determining reasonableness other than by balancing the need to search [or
seize] against the invasion which the search [or seizure] entails." Camara v.
Municipal Court, 387 U. S. 523, 534-535, 536-537 (1967). And, in justifying the
particular intrusion, the police officer must be able to point to specific and
articulable facts which, taken together with rational inferences from those facts,
reasonably warrant that intrusion. [Footnote 18] The scheme of the Fourth
Amendment becomes meaningful only when it is assured that, at some point, the
conduct of those charged with enforcing the laws can be subjected to the more
detached, neutral scrutiny of a judge who must evaluate the reasonableness of a
particular search or seizure in light of the particular circumstances. [Footnote 19]
And, in making that assessment, it is imperative that the facts be judged against an
objective standard: would the facts

[22]

available to the officer at the moment of the seizure or the search "warrant a man
of reasonable caution in the belief" that the action taken was appropriate? Cf.
Carroll v. United States, 267 U. S. 132 (1925); Beck v. Ohio, 379 U. S. 89, 96-97
(1964). [Footnote 20] Anything less would invite intrusions upon constitutionally
guaranteed rights based on nothing more substantial than inarticulate hunches, a
result this Court has consistently refused to sanction. See, e.g., Beck v. Ohio,
supra; Rios v. United States, 364 U. S. 253 (1960); Henry v. United States, 361 U.
S. 98 (1959). And simple "'good faith on the part of the arresting officer is not
enough.' . . . If subjective good faith alone were the test, the protections of the
Fourth Amendment would evaporate, and the people would be 'secure in their
persons, houses, papers, and effects,' only in the discretion of the police." Beck v.
Ohio, supra, at 97.

Applying these principles to this case, we consider first the nature and extent of the
governmental interests involved. One general interest is, of course, that of effective
crime prevention and detection; it is this interest which underlies the recognition
that a police officer may, in appropriate circumstances and in an appropriate
manner, approach a person for purposes of investigating possibly criminal behavior
even though there is no probable cause to make an arrest. It was this legitimate
investigative function Officer McFadden was discharging when he decided to
approach petitioner and his companions. He had observed Terry, Chilton, and Katz
go through a series of acts, each of them perhaps innocent in itself, but which,
taken together, warranted further investigation. There is nothing unusual in two
men standing together on a street corner, perhaps waiting for someone. Nor is there
anything suspicious about people

[23]

in such circumstances strolling up and down the street, singly or in pairs. Store
windows, moreover, are made to be looked in. But the story is quite different
where, as here, two men hover about a street corner for an extended period of time,
at the end of which it becomes apparent that they are not waiting for anyone or
anything; where these men pace alternately along an identical route, pausing to
stare in the same store window roughly 24 times; where each completion of this
route is followed immediately by a conference between the two men on the corner;
where they are joined in one of these conferences by a third man who leaves
swiftly, and where the two men finally follow the third and rejoin him a couple of
blocks away. It would have been poor police work indeed for an officer of 30
years' experience in the detection of thievery from stores in this same
neighborhood to have failed to investigate this behavior further.

The crux of this case, however, is not the propriety of Officer McFadden's taking
steps to investigate petitioner's suspicious behavior, but, rather, whether there was
justification for McFadden's invasion of Terry's personal security by searching him
for weapons in the course of that investigation. We are now concerned with more
than the governmental interest in investigating crime; in addition, there is the more
immediate interest of the police officer in taking steps to assure himself that the
person with whom he is dealing is not armed with a weapon that could
unexpectedly and fatally be used against him. Certainly it would be unreasonable
to require that police officers take unnecessary risks in the performance of their
duties. American criminals have a long tradition of armed violence, and every year
in this country many law enforcement officers are killed in the line of duty, and
thousands more are wounded.

[24]

Virtually all of these deaths and a substantial portion of the injuries are inflicted
with guns and knives. [Footnote 21]

In view of these facts, we cannot blind ourselves to the need for law enforcement
officers to protect themselves and other prospective victims of violence in
situations where they may lack probable cause for an arrest. When an officer is
justified in believing that the individual whose suspicious behavior he is
investigating at close range is armed and presently dangerous to the officer or to
others, it would appear to be clearly unreasonable to deny the officer the power to
take necessary measures to determine whether the person is, in fact, carrying a
weapon and to neutralize the threat of physical harm.

We must still consider, however, the nature and quality of the intrusion on
individual rights which must be accepted if police officers are to be conceded the
right to search for weapons in situations where probable cause to arrest for crime is
lacking. Even a limited search of the outer clothing for weapons constitutes a
severe,

[25]
though brief, intrusion upon cherished personal security, and it must surely be an
annoying, frightening, and perhaps humiliating experience. Petitioner contends that
such an intrusion is permissible only incident to a lawful arrest, either for a crime
involving the possession of weapons or for a crime the commission of which led
the officer to investigate in the first place. However, this argument must be closely
examined.

Petitioner does not argue that a police officer should refrain from making any
investigation of suspicious circumstances until such time as he has probable cause
to make an arrest; nor does he deny that police officers, in properly discharging
their investigative function, may find themselves confronting persons who might
well be armed and dangerous. Moreover, he does not say that an officer is always
unjustified in searching a suspect to discover weapons. Rather, he says it is
unreasonable for the policeman to take that step until such time as the situation
evolves to a point where there is probable cause to make an arrest. When that point
has been reached, petitioner would concede the officer's right to conduct a search
of the suspect for weapons, fruits or instrumentalities of the crime, or "mere"
evidence, incident to the arrest.

There are two weaknesses in this line of reasoning, however. First, it fails to take
account of traditional limitations upon the scope of searches, and thus recognizes
no distinction in purpose, character, and extent between a search incident to an
arrest and a limited search for weapons. The former, although justified in part by
the acknowledged necessity to protect the arresting officer from assault with a
concealed weapon, Preston v. United States, 376 U. S. 364, 367 (1964), is also
justified on other grounds, ibid., and can therefore involve a relatively extensive
exploration of the person. A search for weapons in the absence of probable cause
to

[26]

arrest, however, must, like any other search, be strictly circumscribed by the
exigencies which justify its initiation. Warden v. Hayden, 387 U. S. 294, 310
(1967) (MR. JUSTICE FORTAS, concurring). Thus, it must be limited to that
which is necessary for the discovery of weapons which might be used to harm the
officer or others nearby, and may realistically be characterized as something less
than a "full" search, even though it remains a serious intrusion.

A second, and related, objection to petitioner's argument is that it assumes that the
law of arrest has already worked out the balance between the particular interests
involved here -- the neutralization of danger to the policeman in the investigative
circumstance and the sanctity of the individual. But this is not so. An arrest is a
wholly different kind of intrusion upon individual freedom from a limited search
for weapons, and the interests each is designed to serve are likewise quite different.
An arrest is the initial stage of a criminal prosecution. It is intended to vindicate
society's interest in having its laws obeyed, and it is inevitably accompanied by
future interference with the individual's freedom of movement, whether or not trial
or conviction ultimately follows. [Footnote 22] The protective search for weapons,
on the other hand, constitutes a brief, though far from inconsiderable, intrusion
upon the sanctity of the person. It does not follow that, because an officer may
lawfully arrest a person only when he is apprised of facts sufficient to warrant a
belief that the person has committed or is committing a crime, the officer is equally
unjustified, absent that kind of evidence, in making any intrusions short of an
arrest. Moreover, a perfectly reasonable apprehension of danger may arise long
before the officer is possessed of adequate information to justify taking a person
into custody for

[27]

the purpose of prosecuting him for a crime. Petitioner's reliance on cases which
have worked out standards of reasonableness with regard to "seizures" constituting
arrests and searches incident thereto is thus misplaced. It assumes that the interests
sought to be vindicated and the invasions of personal security may be equated in
the two cases, and thereby ignores a vital aspect of the analysis of the
reasonableness of particular types of conduct under the Fourth Amendment. See
Camara v. Municipal Court, supra.

Our evaluation of the proper balance that has to be struck in this type of case leads
us to conclude that there must be a narrowly drawn authority to permit a reasonable
search for weapons for the protection of the police officer, where he has reason to
believe that he is dealing with an armed and dangerous individual, regardless of
whether he has probable cause to arrest the individual for a crime. The officer need
not be absolutely certain that the individual is armed; the issue is whether a
reasonably prudent man, in the circumstances, would be warranted in the belief
that his safety or that of others was in danger. Cf. Beck v. Ohio, 379 U. S. 89, 91
(1964); Brinegar v. United States, 338 U. S. 160, 174-176 (1949); Stacey v.
Emery, 97 U. S. 642, 645 (1878). [Footnote 23] And in determining whether the
officer acted reasonably in such circumstances, due weight must be given not to his
inchoate and unparticularized suspicion or "hunch," but to the specific reasonable
inferences which he is entitled to draw from the facts in light of his experience. Cf.
Brinegar v. United States supra.

IV

We must now examine the conduct of Officer McFadden in this case to determine
whether his search and seizure of petitioner were reasonable, both at their inception

[28]
and as conducted. He had observed Terry, together with Chilton and another man,
acting in a manner he took to be preface to a "stick-up." We think, on the facts and
circumstances Officer McFadden detailed before the trial judge, a reasonably
prudent man would have been warranted in believing petitioner was armed, and
thus presented a threat to the officer's safety while he was investigating his
suspicious behavior. The actions of Terry and Chilton were consistent with
McFadden's hypothesis that these men were contemplating a daylight robbery --
which, it is reasonable to assume, would be likely to involve the use of weapons --
and nothing in their conduct from the time he first noticed them until the time he
confronted them and identified himself as a police officer gave him sufficient
reason to negate that hypothesis. Although the trio had departed the original scene,
there was nothing to indicate abandonment of an intent to commit a robbery at
some point. Thus, when Officer McFadden approached the three men gathered
before the display window at Zucker's store, he had observed enough to make it
quite reasonable to fear that they were armed, and nothing in their response to his
hailing them, identifying himself as a police officer, and asking their names served
to dispel that reasonable belief. We cannot say his decision at that point to seize
Terry and pat his clothing for weapons was the product of a volatile or inventive
imagination, or was undertaken simply as an act of harassment; the record
evidences the tempered act of a policeman who, in the course of an investigation,
had to make a quick decision as to how to protect himself and others from possible
danger, and took limited steps to do so.

The manner in which the seizure and search were conducted is, of course, as vital a
part of the inquiry as whether they were warranted at all. The Fourth Amendment
proceeds as much by limitations upon the

[29]

scope of governmental action as by imposing preconditions upon its


initiation. Compare Katz v. United States, 389 U. S. 347, 354-356 (1967). The
entire deterrent purpose of the rule excluding evidence seized in violation of the
Fourth Amendment rests on the assumption that "limitations upon the fruit to be
gathered tend to limit the quest itself." United States v. Poller, 43 F.2d 911, 914
(C.A.2d Cir.1930); see, e.g., Linkletter v. Walker, 381 U. S. 618, 629-635
(1965); Mapp v. Ohio, 367 U. S. 643 (1961); Elkins v. United States, 364 U. S.
206, 216-221 (1960). Thus, evidence may not be introduced if it was discovered by
means of a seizure and search which were not reasonably related in scope to the
justification for their initiation. Warden v. Hayden, 387 U. S. 294, 310 (1967)
(MR. JUSTICE FORTAS, concurring).

We need not develop at length in this case, however, the limitations which the
Fourth Amendment places upon a protective seizure and search for weapons. These
limitations will have to be developed in the concrete factual circumstances of
individual cases. See Sibron v. New York, post, p. 40, decided today. Suffice it to
note that such a search, unlike a search without a warrant incident to a lawful
arrest, is not justified by any need to prevent the disappearance or destruction of
evidence of crime. See Preston v. United States, 376 U. S. 364, 367 (1964). The
sole justification of the search in the present situation is the protection of the police
officer and others nearby, and it must therefore be confined in scope to an intrusion
reasonably designed to discover guns, knives, clubs, or other hidden instruments
for the assault of the police officer.

The scope of the search in this case presents no serious problem in light of these
standards. Officer McFadden patted down the outer clothing of petitioner and his
two companions. He did not place his hands in their pockets or under the outer
surface of their garments until he had

[30]

felt weapons, and then he merely reached for and removed the guns. He never did
invade Katz' person beyond the outer surfaces of his clothes, since he discovered
nothing in his pat-down which might have been a weapon. Officer McFadden
confined his search strictly to what was minimally necessary to learn whether the
men were armed and to disarm them once he discovered the weapons. He did not
conduct a general exploratory search for whatever evidence of criminal activity he
might find.

We conclude that the revolver seized from Terry was properly admitted in
evidence against him. At the time he seized petitioner and searched him for
weapons, Officer McFadden had reasonable grounds to believe that petitioner was
armed and dangerous, and it was necessary for the protection of himself and others
to take swift measures to discover the true facts and neutralize the threat of harm if
it materialized. The policeman carefully restricted his search to what was
appropriate to the discovery of the particular items which he sought. Each case of
this sort will, of course, have to be decided on its own facts. We merely hold today
that, where a police officer observes unusual conduct which leads him reasonably
to conclude in light of his experience that criminal activity may be afoot and that
the persons with whom he is dealing may be armed and presently dangerous,
where, in the course of investigating this behavior, he identifies himself as a
policeman and makes reasonable inquiries, and where nothing in the initial stages
of the encounter serves to dispel his reasonable fear for his own or others' safety,
he is entitled for the protection of himself and others in the area to conduct a
carefully limited search of the outer clothing of such persons in an attempt to
discover weapons which might be used to assault him.

[31]
Such a search is a reasonable search under the Fourth Amendment, and any
weapons seized may properly be introduced in evidence against the person from
whom they were taken.

Affirmed.

MR. JUSTICE BLACK concurs in the judgment and the opinion except where the
opinion quotes from and relies upon this Court's opinion in Katz v. United
States and the concurring opinion in Warden v. Hayden.

MR. JUSTICE HARLAN, concurring.

While I unreservedly agree with the Court's ultimate holding in this case, I am
constrained to fill in a few gaps, as I see them, in its opinion. I do this because
what is said by this Court today will serve as initial guidelines for law enforcement
authorities and courts throughout the land as this important new field of law
develops.

A police officer's right to make an on-the-street "stop" and an accompanying


"frisk" for weapons is, of course, bounded by the protections afforded by the
Fourth and Fourteenth Amendments. The Court holds, and I agree, that, while the
right does not depend upon possession by the officer of a valid warrant, nor upon
the existence of probable cause, such activities must be reasonable under the
circumstances as the officer credibly relates them in court. Since the question in
this and most cases is whether evidence produced by a frisk is admissible, the
problem is to determine what makes a frisk reasonable.

If the State of Ohio were to provide that police officers could, on articulable
suspicion less than probable cause, forcibly frisk and disarm persons thought to be
carrying concealed weapons, I would have little doubt that action taken pursuant to
such authority could be constitutionally reasonable. Concealed weapons create an
immediate

[32]

and severe danger to the public, and though that danger might not warrant routine
general weapons checks, it could well warrant action on less than a "probability." I
mention this line of analysis because I think it vital to point out that it cannot be
applied in this case. On the record before us, Ohio has not clothed its policemen
with routine authority to frisk and disarm on suspicion; in the absence of state
authority, policemen have no more right to "pat down" the outer clothing of
passers-by, or of persons to whom they address casual questions, than does any
other citizen. Consequently, the Ohio courts did not rest the constitutionality of this
frisk upon any general authority in Officer McFadden to take reasonable steps to
protect the citizenry, including himself, from dangerous weapons.
The state courts held, instead, that, when an officer is lawfully confronting a
possibly hostile person in the line of duty, he has a right, springing only from the
necessity of the situation, and not from any broader right to disarm, to frisk for his
own protection. This holding, with which I agree and with which I think the Court
agrees, offers the only satisfactory basis I can think of for affirming this
conviction. The holding has, however, two logical corollaries that I do not think
the Court has fully expressed.

In the first place, if the frisk is justified in order to protect the officer during an
encounter with a citizen, the officer must first have constitutional grounds to insist
on an encounter, to make a forcible stop. Any person, including a policeman, is at
liberty to avoid a person he considers dangerous. If and when a policeman has a
right instead to disarm such a person for his own protection, he must first have a
right not to avoid him, but to be in his presence. That right must be more than the
liberty (again, possessed by every citizen) to address questions to other persons, for
ordinarily the person

[33]

addressed has an equal right to ignore his interrogator and walk away; he certainly
need not submit to a frisk for the questioner's protection. I would make it perfectly
clear that the right to frisk in this case depends upon the reasonableness of a
forcible stop to investigate a suspected crime.

Where such a stop is reasonable, however, the right to frisk must be immediate and
automatic if the reason for the stop is, as here, an articulable suspicion of a crime
of violence. Just as a full search incident to a lawful arrest requires no additional
justification, a limited frisk incident to a lawful stop must often be rapid and
routine. There is no reason why an officer, rightfully but forcibly confronting a
person suspected of a serious crime, should have to ask one question and take the
risk that the answer might be a bullet.

The facts of this case are illustrative of a proper stop and an incident frisk. Officer
McFadden had no probable cause to arrest Terry for anything, but he had observed
circumstances that would reasonably lead an experienced, prudent policeman to
suspect that Terry was about to engage in burglary or robbery. His justifiable
suspicion afforded a proper constitutional basis for accosting Terry, restraining his
liberty of movement briefly, and addressing questions to him, and Officer
McFadden did so. When he did, he had no reason whatever to suppose that Terry
might be armed, apart from the fact that he suspected him of planning a violent
crime. McFadden asked Terry his name, to which Terry "mumbled something."
Whereupon McFadden, without asking Terry to speak louder and without giving
him any chance to explain his presence or his actions, forcibly frisked him.
I would affirm this conviction for what I believe to be the same reasons the Court
relies on. I would, however, make explicit what I think is implicit in affirmance on

[34]

the present facts. Officer McFadden's right to interrupt Terry's freedom of


movement and invade his privacy arose only because circumstances warranted
forcing an encounter with Terry in an effort to prevent or investigate a crime. Once
that forced encounter was justified, however, the officer's right to take suitable
measures for his own safety followed automatically.

Upon the foregoing premises, I join the opinion of the Court.

MR. JUSTICE WHITE, concurring.

I join the opinion of the Court, reserving judgment, however, on some of the
Court's general remarks about the scope and purpose of the exclusionary rule
which the Court has fashioned in the process of enforcing the Fourth Amendment.

Also, although the Court puts the matter aside in the context of this case, I think an
additional word is in order concerning the matter of interrogation during an
investigative stop. There is nothing in the Constitution which prevents a policeman
from addressing questions to anyone on the streets. Absent special circumstances,
the person approached may not be detained or frisked, but may refuse to cooperate
and go on his way. However, given the proper circumstances, such as those in this
case, it seems to me the person may be briefly detained against his will while
pertinent questions are directed to him. Of course, the person stopped is not
obliged to answer, answers may not be compelled, and refusal to answer furnishes
no basis for an arrest, although it may alert the officer to the need for continued
observation. In my view, it is temporary detention, warranted by the circumstances,
which chiefly justifies the protective frisk for weapons. Perhaps the frisk itself,
where proper, will have beneficial results whether questions are asked or not. If
weapons are found, an arrest will follow.

[35]

If none is found, the frisk may nevertheless serve preventive ends because of its
unmistakable message that suspicion has been aroused. But if the investigative stop
is sustainable at all, constitutional rights are not necessarily violated if pertinent
questions are asked and the person is restrained briefly in the process.

MR. JUSTICE DOUGLAS, dissenting.

I agree that petitioner was "seized" within the meaning of the Fourth Amendment. I
also agree that frisking petitioner and his companions for guns was a "search." But
it is a mystery how that "search" and that "seizure" can be constitutional by Fourth
Amendment standards unless there was "probable cause" [Footnote 1] to believe
that (1) a crime had been committed or (2) a crime was in the process of being
committed or (3) a crime was about to be committed.

The opinion of the Court disclaims the existence of "probable cause." If loitering
were in issue and that

[36]

was the offense charged, there would be "probable cause" shown. But the crime
here is carrying concealed weapons; [Footnote 2] and there is no basis for
concluding that the officer had "probable cause" for believing that that crime was
being committed. Had a warrant been sought, a magistrate would, therefore, have
been unauthorized to issue one, for he can act only if there is a showing of
"probable cause." We hold today that the police have greater authority to make a
"seizure" and conduct a "search" than a judge has to authorize such action. We
have said precisely the opposite over and over again. [Footnote 3]

[37]

In other words, police officers up to today have been permitted to effect arrests or
searches without warrants only when the facts within their personal knowledge
would satisfy the constitutional standard of probable cause. At the time of their
"seizure" without a warrant, they must possess facts concerning the person arrested
that would have satisfied a magistrate that "probable cause" was indeed present.
The term "probable cause" rings a bell of certainty that is not sounded by phrases
such as "reasonable suspicion." Moreover, the meaning of "probable cause" is
deeply imbedded in our constitutional history. As we stated in Henry v. United
States, 361 U. S. 98, 100-102:

"The requirement of probable cause has roots that are deep in our history. The
general warrant, in which the name of the person to be arrested was left blank, and
the writs of assistance, against which James Otis inveighed, both perpetuated the
oppressive practice of allowing the police to arrest and search on suspicion. Police
control took the place of judicial control, since no showing of 'probable cause'
before a magistrate was required."

"* * * *"

"That philosophy [rebelling against these practices] later was reflected in the
Fourth Amendment. And as the early American decisions both before and
immediately after its adoption show, common rumor or report, suspicion, or even
'strong reason to suspect' was not adequate to support a warrant

[38]

for arrest. And that principle has survived to this day. . . ."

". . . It is important, we think, that this requirement [of probable cause] be strictly
enforced, for the standard set by the Constitution protects both the officer and the
citizen. If the officer acts with probable cause, he is protected even though it turns
out that the citizen is innocent. . . . And while a search without a warrant is, within
limits, permissible if incident to a lawful arrest, if an arrest without a warrant is to
support an incidental search, it must be made with probable cause. . . . This
immunity of officers cannot fairly be enlarged without jeopardizing the privacy or
security of the citizen."

The infringement on personal liberty of any "seizure" of a person can only be


"reasonable" under the Fourth Amendment if we require the police to possess
"probable cause" before they seize him. Only that line draws a meaningful
distinction between an officer's mere inkling and the presence of facts within the
officer's personal knowledge which would convince a reasonable man that the
person seized has committed, is committing, or is about to commit a particular
crime. "In dealing with probable cause, . . . as the very name implies, we deal with
probabilities. These are not technical; they are the factual and practical
considerations of everyday life on which reasonable and prudent men, not legal
technicians, act." Brinegar v. United States, 338 U. S. 160, 175.

To give the police greater power than a magistrate is to take a long step down the
totalitarian path. Perhaps such a step is desirable to cope with modern forms of
lawlessness. But if it is taken, it should be the deliberate choice of the people
through a constitutional amendment.

[39]

Until the Fourth Amendment, which is closely allied with the Fifth, [Footnote 4] is
rewritten, the person and the effects of the individual are beyond the reach of all
government agencies until there are reasonable grounds to believe (probable cause)
that a criminal venture has been launched or is about to be launched.

There have been powerful hydraulic pressures throughout our history that bear
heavily on the Court to water down constitutional guarantees and give the police
the upper hand. That hydraulic pressure has probably never been greater than it is
today.
Yet if the individual is no longer to be sovereign, if the police can pick him up
whenever they do not like the cut of his jib, if they can "seize" and "search" him in
their discretion, we enter a new regime. The decision to enter it should be made
only after a full debate by the people of this country.

Footnotes

[Footnote 1]

Ohio Rev.Code § 2923.01 (1953) provides in part that "[n]o person shall carry a
pistol, bowie knife, dirk, or other dangerous weapon concealed on or about his
person." An exception is made for properly authorized law enforcement officers.

[Footnote 2]

Terry and Chilton were arrested, indicted, tried, and convicted together. They were
represented by the same attorney, and they made a joint motion to suppress the
guns. After the motion was denied, evidence was taken in the case against Chilton.
This evidence consisted of the testimony of the arresting officer and of Chilton. It
was then stipulated that this testimony would be applied to the case against Terry,
and no further evidence was introduced in that case. The trial judge considered the
two cases together, rendered the decisions at the same time, and sentenced the two
men at the same time. They prosecuted their state court appeals together through
the same attorney, and they petitioned this Court for certiorari together. Following
the grant of the writ upon this joint petition, Chilton died. Thus, only Terry's
conviction is here for review.

[Footnote 3]

Both the trial court and the Ohio Court of Appeals in this case relied upon such a
distinction. State v. Terry, 5 Ohio App.2d 122, 125-130, 214 N.E.2d 114, 117-120
(1966). See also, e.g., People v. Rivera, 14 N.Y.2d 441, 201 N.E.2d 32, 252
N.Y.S.2d 458 (1964), cert. denied, 379 U.S. 978 (1965); Aspen, Arrest and Arrest
Alternatives: Recent Trends, 1966 U.Ill.L.F. 241, 249-254; Warner, The Uniform
Arrest Act, 28 Va.L.Rev. 315 (1942); Note, Stop and Frisk in California, 18
Hastings L.J. 623, 629-632 (1967).

[Footnote 4]

People v. Rivera, supra, n 3, at 447, 201 N.E.2d at 36, 252 N.Y.S.2d at 464.

[Footnote 5]

The theory is well laid out in the Rivera opinion:

 
"[T]he evidence needed to make the inquiry is not of the same degree of
conclusiveness as that required for an arrest. The stopping of the individual to
inquire is not an arrest and the ground upon which the police may make the inquiry
may be less incriminating than the ground for an arrest for a crime known to have
been committed. . . ."

"* * * *"

"And as the right to stop and inquire is to be justified for a cause less conclusive
than that which would sustain an arrest, so the right to frisk may be justified as an
incident to inquiry upon grounds of elemental safety and precaution which might
not initially sustain a search. Ultimately, the validity of the frisk narrows down to
whether there is or is not a right by the police to touch the person questioned. The
sense of exterior touch here involved is not very far different from the sense of
sight or hearing -- senses upon which police customarily act." People v. Rivera, 14
N.Y.2d 441, 445, 447, 201 N.E.2d 32, 34, 35, 252 N.Y.S.2d 458, 461, 463
(1964), cert. denied, 379 U.S. 978 (1965).

[Footnote 6]

See, e.g., Foote, The Fourth Amendment: Obstacle or Necessity in the Law of
Arrest?, 51 J.Crim.L.C. & P.S. 402 (1960).

[Footnote 7]

See n 11, infra.

[Footnote 8]

Brief for Respondent 2.

[Footnote 9]

See L. Tiffany, D. McIntyre D. Rotenberg, Detection of Crime: Stopping and


Questioning, Search and Seizure, Encouragement and Entrapment 186 (1967). This
sort of police conduct may, for example, be designed simply to help an intoxicated
person find his way home, with no intention of arresting him unless he becomes
obstreperous. Or the police may be seeking to mediate a domestic quarrel which
threatens to erupt into violence. They may accost a woman in an area known for
prostitution as part of a harassment campaign designed to drive prostitutes away
without the considerable difficulty involved in prosecuting them. Or they may be
conducting a dragnet search of all teenagers in a particular section of the city for
weapons because they have heard rumors of an impending gang fight.

[Footnote 10]
See Tiffany, McIntyre & Rotenberg, supra, n 9, at 100-101; Comment, 47
Nw.U.L.Rev. 493, 497-499 (1952).

[Footnote 11]

The President's Commission on Law Enforcement and Administration of Justice


found that, "[i]n many communities, field interrogations are a major source of
friction between the police and minority groups." President's Commission on Law
Enforcement and Administration of Justice, Task Force Report: The Police 183
(1967). It was reported that the friction caused by "[m]isuse of field interrogations"
increases "as more police departments adopt 'aggressive patrol,' in which officers
are encouraged routinely to stop and question persons on the street who are
unknown to them, who are suspicious, or whose purpose for being abroad is not
readily evident." Id. at 184. While the frequency with which "frisking" forms a part
of field interrogation practice varies tremendously with the locale, the objective of
the interrogation, and the particular officer, see Tiffany, McIntyre &
Rotenberg, supra, n 9, at 47-48, it cannot help but be a severely exacerbating factor
in police-community tensions. This is particularly true in situations where the "stop
and frisk" of youths or minority group members is "motivated by the officers'
perceived need to maintain the power image of the beat officer, an aim sometimes
accomplished by humiliating anyone who attempts to undermine police control of
the streets." Ibid.

[Footnote 12]

In this case, for example, the Ohio Court of Appeals stated that "we must be
careful to distinguish that the 'frisk' authorized herein includes only a 'frisk' for a
dangerous weapon. It by no means authorizes a search for contraband, evidentiary
material, or anything else in the absence of reasonable grounds to arrest. Such a
search is controlled by the requirements of the Fourth Amendment, and probable
cause is essential." State v. Terry, 5 Ohio App.2d 122, 130, 214 N.E.2d 114, 120
(1966). See also, e.g., Ellis v. United States, 105 U.S.App.D.C. 86, 88, 264 F.2d
372, 374 (1959); Comment, 65 Col.L.Rev. 848, 860, and n. 81 (1965).

[Footnote 13]

Consider the following apt description:

"[T]he officer must feel with sensitive fingers every portion of the prisoner's body.
A thorough search must be made of the prisoner's arms and armpits, waistline and
back, the groin and area about the testicles, and entire surface of the legs down to
the feet." Priar & Martin, Searching and Disarming Criminals, 45 J.Crim.L.C. &
P.S. 481 (1954).

[Footnote 14]
See n 11, supra, and accompanying text.

We have noted that the abusive practices which play a major, though by no means
exclusive, role in creating this friction are not susceptible of control by means of
the exclusionary rule, and cannot properly dictate our decision with respect to the
powers of the police in genuine investigative and preventive situations. However,
the degree of community resentment aroused by particular practices is clearly
relevant to an assessment of the quality of the intrusion upon reasonable
expectations of personal security caused by those practices.

[Footnote 15]

These dangers are illustrated in part by the course of adjudication in the Court of
Appeals of New York. Although its first decision in this area, People v. Rivera, 14
N.Y.2d 441, 201 N.E.2d 32, 252 N.Y.S.2d 458 (1964), cert. denied, 379 U.S. 978
(1965), rested squarely on the notion that a "frisk" was not a "search," see nn. 3-
5 supra, it was compelled to recognize, in People v. Taggart, 20 N.Y.2d 335, 342,
229 N.E.2d 581, 586, 283 N.Y.S.2d 1, 8 (1967), that what it had actually
authorized in Rivera and subsequent decisions, see, e.g., People v. Pugach, 15
N.Y.2d 65, 204 N.E.2d 176, 255 N.Y.S.2d 833 (1964), cert. denied, 380 U.S. 936
(1965), was a "search" upon less than probable cause. However, in acknowledging
that no valid distinction could be maintained on the basis of its cases, the Court of
Appeals continued to distinguish between the two in theory. It still defined
"search" as it had in Rivera -- as an essentially unlimited examination of the person
for any and all seizable items -- and merely noted that the cases had upheld police
intrusions which went far beyond the original limited conception of a "frisk." Thus,
principally because it failed to consider limitations upon the scope of searches in
individual cases as a potential mode of regulation, the Court of Appeals in three
short years arrived at the position that the Constitution must, in the name of
necessity, be held to permit unrestrained rummaging about a person and his effects
upon mere suspicion. It did apparently limit its holding to "cases involving serious
personal injury or grave irreparable property damage," thus excluding those
involving "the enforcement of sumptuary laws, such as gambling, and laws of
limited public consequence, such as narcotics violations, prostitution, larcenies of
the ordinary kind, and the like." People v. Taggart, supra, at 340, 214 N.E.2d at
584, 283 N.Y.S.2d at 6.

In our view, the sounder course is to recognize that the Fourth Amendment
governs all intrusions by agents of the public upon personal security, and to make
the scope of the particular intrusion, in light of all the exigencies of the case, a
central element in the analysis of reasonableness. Cf. Brinegar v. United
States, 338 U. S. 160, 183 (1949) (Mr. Justice Jackson, dissenting). Compare
Camara v. Municipal Court, 387 U. S. 523, 537 (1967). This seems preferable to
an approach which attributes too much significance to an overly technical
definition of "search," and which turns in part upon a judge-made hierarchy of
legislative enactments in the criminal sphere. Focusing the inquiry squarely on the
dangers and demands of the particular situation also seems more likely to produce
rules which are intelligible to the police and the public alike than requiring the
officer in the heat of an unfolding encounter on the street to make a judgment as to
which laws are "of limited public consequence."

[Footnote 16]

We thus decide nothing today concerning the constitutional propriety of an


investigative "seizure" upon less than probable cause for purposes of "detention"
and/or interrogation. Obviously, not all personal intercourse between policemen
and citizens involves "seizures" of persons. Only when the officer, by means of
physical force or show of authority, has in some way restrained the liberty of a
citizen may we conclude that a "seizure" has occurred. We cannot tell with any
certainty upon this record whether any such "seizure" took place here prior to
Officer McFadden's initiation of physical contact for purposes of searching Terry
for weapons, and we thus may assume that, up to that point, no intrusion upon
constitutionally protected rights had occurred.

[Footnote 17]

See generally Leagre, The Fourth Amendment and the Law of Arrest, 54


J.Crim.L.C. & P.S. 393, 396 403 (1963).

[Footnote 18]

This demand for specificity in the information upon which police action is
predicated is the central teaching of this Court's Fourth Amendment
jurisprudence. See Beck v. Ohio, 379 U. S. 89, 96-97 (1964); Ker v. California, 374
U. S. 23, 34-37 (1963); Wong Sun v. United States, 371 U. S. 471, 479-484
(1963); Rios v. United States, 364 U. S. 253, 261-262 (1960); Henry v. United
States, 361 U. S. 98, 100-102 (1959); Draper v. United States, 358 U. S. 307, 312-
314 (1959); Brinegar v. United States, 338 U. S. 160, 175-178 (1949); Johnson v.
United States, 333 U. S. 10, 15-17 (1948); United States v. Di Re, 332 U. S. 581,
593-595 (1948); Husty v. United States, 282 U. S. 694, 700-701 (1931); Dumbra v.
United States, 268 U. S. 435, 441 (1925); Carroll v. United States, 267 U. S. 132,
159-162 (1925); Stacey v. Emery, 97 U. S. 642,6 45 (1878).

[Footnote 19]

See, e.g., Katz v. United States, 389 U. S. 347, 354-357 (1967); Berger v. New


York, 388 U. S. 41, 54-60 (1967); Johnson v. United States, 333 U. S. 10, 13-15
(1948); cf. Wong Sun v. United States, 371 U. S. 471, 479-480 (1963). See also
Aguilar v. Texas, 378 U. S. 108, 110-115 (1964).
[Footnote 20]

See also cases cited in n 18, supra.

[Footnote 21]

Fifty-seven law enforcement officers were killed in the line of duty in this country
in 1966, bringing the total to 335 for the seven-year period beginning with 1960.
Also in 1966, there were 23,851 assaults on police officers, 9,113 of which resulted
in injuries to the policemen. Fifty-five of the 57 officers killed in 1966 died from
gunshot wounds, 41 of them inflicted by handguns easily secreted about the
person. The remaining two murders were perpetrated by knives. See Federal
Bureau of Investigation, Uniform Crime Reports for the United States -- 1966, at
45-48, 152 and Table 51.

The easy availability of firearms to potential criminals in this country is well


known, and has provoked much debate. See, e.g., President's Commission on Law
Enforcement and Administration of Justice, The Challenge of Crime in a Free
Society 239-243 (1967). Whatever the merits of gun control proposals, this fact is
relevant to an assessment of the need for some form of self-protective search
power.

[Footnote 22]

See generally W. LaFave, Arrest -- The Decision to Take a Suspect into Custody
1-13 (1965).

[Footnote 23]

See also cases cited in n. 18, supra.

[Footnote 1]

The meaning of "probable cause" has been developed in cases where an officer has
reasonable grounds to believe that a crime has been or is being committed. See,
e.g., 70 U. S. 3 Wall. 155; Stacey v. Emery, 97 U. S. 642; Director General v.
Kastenbaum, 263 U. S. 25; Carroll v. United States, 267 U. S. 132; United States
v. Di Re, 332 U. S. 581; Brinegar v. United States, 338 U. S. 160; Draper v.
United States, 358 U. S. 307; Henry v. United States, 361 U. S. 98. In such cases,
of course, the officer may make an "arrest" which results in charging the individual
with commission of a crime. But while arresting persons who have already
committed crimes is an important task of law enforcement, an equally if not more
important function is crime prevention and deterrence of would-be criminals.
"[T]here is no war between the Constitution and common sense," Mapp v.
Ohio, 367 U. S. 643, 657. Police officers need not wait until they see a person
actually commit a crime before they are able to "seize" that person. Respect for our
constitutional system and personal liberty demands in return, however, that such a
"seizure" be made only upon "probable cause."

[Footnote 2]

Ohio Rev.Code § 2923.01.

[Footnote 3]

This Court has always used the language of "probable cause" in determining the
constitutionality of an arrest without a warrant. See, e.g., Carroll v. United
States, 267 U. S. 132, 156, 161-162; Johnson v. United States, 333 U. S. 10, 13-
15; McDonald v. United States, 335 U. S. 451, 455-456; Henry v. United
States, 361 U. S. 98; Wong Sun v. United States, 371 U. S. 471, 479-484. To give
power to the police to seize a person on some grounds different from or less than
"probable cause" would be handing them more authority than could be exercised
by a magistrate in issuing a warrant to seize a person. As we stated in Wong Sun v.
United States, 371 U. S. 471, with respect to requirements for arrests without
warrants:

"Whether or not the requirements of reliability and particularity of the information


on which an officer may act are more stringent where an arrest warrant is absent,
they surely cannot be less stringent than where an arrest warrant is obtained."

Id. at 479. And we said in Brinegar v. United States, 338 U. S. 160, 176:

"These long-prevailing standards [for probable cause] seek to safeguard citizens


from rash and unreasonable interferences with privacy and from unfounded
charges of crime. They also seek to give fair leeway for enforcing the law in the
community's protection. Because many situations which confront officers in the
course of executing their duties are more or less ambiguous, room must be allowed
for some mistakes on their part. But the mistakes must be those of reasonable men,
acting on facts leading sensibly to their conclusions of probability. The rule of
probable cause is a practical, nontechnical conception affording the best
compromise that has been found for accommodating these often opposing interests.
Requiring more would unduly hamper law enforcement. To allow less would be to
leave law-abiding citizens at the mercy of the officers' whim or caprice."

And see Johnson v. United States, 333 U. S. 10, 14-15; Wrightson v. United States,


95 U.S.App.D.C. 390, 393-394, 222 F.2d 556, 559-560 (1955).

[Footnote 4]

See Boyd v. United States, 116 U. S. 616, 633:


"For the 'unreasonable searches and seizures' condemned in the Fourth Amendment
are almost always made for the purpose of compelling a man to give evidence
against himself, which, in criminal cases, is condemned in the Fifth Amendment,
and compelling a man 'in a criminal case to be a witness against himself,' which is
condemned in the Fifth Amendment, throws light on the question as to what is an
'unreasonable search and seizure' within the meaning of the Fourth Amendment."

Manalili v. CA, G.R. No. 113447, October 9, 1997


When dealing with a rapidly unfolding and potentially criminal situation in the city
streets where unarguably there is no time to secure an arrest or a search warrant,
policemen should employ limited, flexible responses — like "stop-and-frisk" —
which are graduated in relation to the amount of information they possess, the
lawmen being ever vigilant to respect and not to violate or to treat cavalierly the
citizen’s constitutional rights against unreasonable arrest, search and seizure.

The Case

This rule is reiterated as we resolve this petition for review on certiorari under


Rule 45 of the Rules of Court, seeking the reversal of the Decision of the Court of
Appeals dated April 19, 1993 and its Resolution dated January 20, 1994 in CA
G.R. CR No. 07266, entitled "People of the Philippines v. Alain Manalili y
Dizon."cralaw virtua1aw library

In an Information dated April 11, 1988, 1 Petitioner Alain Manalili y Dizon was
charged by Assistant Caloocan City Fiscal E. Juan R. Bautista with violation of
Section 8, Article II of Republic Act No. 6425, allegedly committed as follows: 2

"That on or about the 11th day of April 1988 in Caloocan City, MM, Philippines
and within the jurisdiction of this Honorable Court, the above-named accused
without any authority of law, did then and there willfully, unlawfully and
feloniously have in his custody, possession and control crushed marijuana residue,
which is a prohibited drug and knowing the same to be such.

Contrary to Law."cralaw virtua1aw library

Upon his arraignment on April 21, 1988, appellant pleaded "not guilty" to the
charge. 3 With the agreement of the public prosecutor, appellant was released after
filing a P10,000.00 bail bond. 4 After trial in due course, the Regional Trial Court
of Caloocan City, Branch 124, acting as a Special Criminal Court, rendered on
May 19, 1989 a decision 5 convicting appellant of illegal possession of marijuana
residue. The dispositive portion of the decision reads: 6

"WHEREFORE, in view of all the foregoing, this Court finds the accused ALAIN
MANALILI Y DIZON guilty beyond reasonable doubt of violation of Section 8,
Article II, of Republic Act No. 6425, as amended (Illegal Possession of Marijuana
residue), and hereby sentences (sic) said accused to suffer imprisonment of SIX (6)
YEARS and ONE (1) DAY; and to pay a fine of P6,000.00; and to pay the costs.

x       x       x"

Appellant remained on provisional liberty. 7 Atty. Benjamin Razon, counsel for


the defense, filed a Notice of Appeal 8 dated May 31, 1989. On April 19, 1993,
Respondent Court 9 promulgated its assailed Decision, denying the appeal and
affirming the trial court: 10

"ACCORDINGLY, the decision appealed from dated May 19, 1989 is hereby
AFFIRMED in all respects. Costs against appellant."cralaw virtua1aw library

Respondent Court 11 denied reconsideration via its assailed Resolution dated


January 20, 1994, disposing:jgc:chanrobles.com.ph

"ACCORDINGLY, Accused-appellant’s motion for reconsideration is, as is hereby


DENIED."cralaw virtua1aw library

The Facts

Version of the Prosecution

The facts, as found by the trial court, are as follows: 12

"At about 2:10 o’clock in the afternoon of April 11, 1988, policemen from the
Anti-Narcotics Unit of the Kalookan City Police Station were conducting a
surveillance along A. Mabini street, Kalookan City, in front of the Kalookan City
Cemetery. The policemen were Pat. Romeo Espiritu and Pat. Anger Lumabas and a
driver named Arnold Enriquez was driving a Tamaraw vehicle which was the
official car of the Police Station of Kalookan City. The surveillance was being
made because of information that drug addicts were roaming the area in front of
the Kalookan City Cemetery.

Upon reaching the Kalookan City Cemetery, the policemen alighted from their
vehicle. They then chanced upon a male person in front of the cemetery who
appeared high on drugs. The male person was observed to have reddish eyes and to
be walking in a swaying manner. When this male person tried to avoid the
policemen, the latter approached him and introduced themselves as police officers.
The policemen then asked the male person what he was holding in his hands. The
male person tried to resist. Pat. Romeo Espiritu asked the male person if he could
see what said male person had in his hands. The latter showed the wallet and
allowed Pat. Romeo Espiritu to examine the same. Pat. Espiritu took the wallet and
examined it. He found suspected crushed marijuana residue inside. He kept the
wallet and its marijuana contents.chanrobles.com : virtual lawlibrary

The male person was then brought to the Anti-Narcotics Unit of the Kalookan City
Police Headquarters and was turned over to Cpl. Wilfredo Tamondong for
investigation. Pat. Espiritu also turned over to Cpl. Tamondong the confiscated
wallet and its suspected marijuana contents. The man turned out to be the accused
ALAIN MANALILI y DIZON.

Upon receipt of the confiscated suspected marijuana residue from Pat. Espiritu,
Cpl. Tamondong wrapped the same with a white sheet of paper on which he wrote
‘Evidence ‘A’ 4/11/88 Alain Manalili’. The white sheet of paper was marked as
Exhibit ‘E-3’. The residue was originally wrapped in a smaller sheet of folded
paper. (Exhibit ‘E-4’).

Cpl. Tamondong next prepared a referral slip addressed to the NBI Forensic
Chemistry Section requesting a chemical analysis of the subject marijuana residue
(Exhibit ‘D’). Cpl. Tamondong thereafter prepared a Joint Affidavit of the
apprehending policemen (Exhibit ‘A’). Pat. Angel Lumabas handcarried the
referral slip (Exhibit ‘D’) to the National Bureau of Investigation (NBI), including
the subject marijuana residue for chemical analysis. The signature of Pat. Lumabas
appears on the left bottom corner of Exhibit ‘D’.

The Forensic Chemistry Section of the NBI received the aforesaid referral slip and
the subject marijuana residue at 7:40 o’clock in the evening of April 11, 1988 as
shown on the stamped portion of Exhibit ‘D’.

It was NBI Aida Pascual who conducted the microscopic and chemical
examinations of the specimen which she identified. (Exhibit ‘E’) 13 Mrs. Pascual
referred to the subject specimen as ‘crushed marijuana leaves’ in her Certification
dated April 11, 1988 (Exhibit ‘F’). 14 These crushed marijuana leaves gave
positive results for marijuana, according to the Certificate.

Mrs. Pascual also conducted a chromatographic examination of the specimen. In


this examination, she also found that the ‘crushed marijuana leaves’ gave positive
results for marijuana. She then prepared a Final Report of her examinations
(Exhibit ‘G’).

After conducting the examinations, Ms. Pascual placed the specimen in a white
letter-envelope and sealed it. (Exhibit ‘E’). She then wrote identification notes on
this letter-envelope. (Exhibit ‘E-1’).
Pat. Lumabas carried the Certification marked as Exhibit ‘F’ from the NBI
Forensic Chemistry Section to Cpl. Tamondong. Upon receipt thereof, Cpl.
Tamondong prepared a referral slip addressed to the City Fiscal of Kalookan City.
(Exhibit ‘C’)"

On rebuttal, Pat. Espiritu testified that appellant was not riding a tricycle but was
walking in front of the cemetery when he was apprehended. 15

Version of the Defense

The trial court summarized the testimonies of the defense witnesses as follows: 16

"At about 2:00 o’clock in the afternoon of April 11, 1988, the accused ALAIN
MANALILI was aboard a tricycle at A. Mabini street near the Kalookan City
Cemetery on the way to his boarding house. Three policemen ordered the driver of
the tricycle to stop because the tricycle driver and his lone passenger were under
the influence of marijuana. The policemen brought the accused and the tricycle
driver inside the Ford Fiera which the policemen were riding in. The policemen
then bodily searched the accused and the tricycle driver. At this point, the accused
asked the policemen why he was being searched and the policemen replied that he
(accused) was carrying marijuana. However, nothing was found on the persons of
the accused and the driver. The policemen allowed the tricycle driver to go while
they brought the accused to the police headquarters at Kalookan City where they
said they would again search the accused.

On the way to the police headquarters, the accused saw a neighbor and signaled the
latter to follow him. The neighbor thus followed the accused to the Kalookan City
Police Headquarters. Upon arrival thereat, the accused was asked to remove his
pants in the presence of said neighbor and another companion. The policemen
turned over the pants of the accused over a piece of bond paper trying to look for
marijuana. However, nothing was found, except for some dirt and dust. This
prompted the companion of the neighbor of the accused to tell the policemen to
release the accused. The accused was led to a cell. The policemen later told the
accused that they found marijuana inside the pockets of his pants.

At about 5:00 o’clock in the afternoon on the same day, the accused was brought
outside the cell and was led to the Ford Fiera. The accused was told by the
policemen to call his parents in order to ‘settle’ the case. The policemen who led
the accused to the Ford Fiera were Pat. Lumabas, Pat. Espiritu and Cpl.
Tamondong. Pat. Lumabas was the policeman who told the accused to call his
parents. The accused did not call his parents and he told the policemen that his
parents did not have any telephone.

At about 5:30 o’clock in the afternoon of the same day, the accused was brought in
the office of an inquest Fiscal. There, the accused told the Fiscal that no marijuana
was found on his person but the Fiscal told the accused not to say anything. The
accused was then brought back to the Kalookan City Jail.

Loreto Medenilla, the tricycle driver who was allegedly with the accused when he
and the accused were stopped by policemen and then bodily searched on April 11,
1988, testified. He said that the policemen found nothing either on his person or on
the person of the accused when both were searched on April 11, 1988.

Roberto Abes, a neighbor of the accused, testified that he followed the accused at
the Kalookan City Police Headquarters on April 11, 1988. He said that the police
searched the accused who was made to take off his pants at the police headquarters
but no marijuana was found on the body of the accused" .

Appellant, who was recalled to the stand as sur-rebuttal witness, presented several
pictures showing that tricycles were allowed to ply in front of the Caloocan
Cemetery. 17

The Rulings of the Trial and the Appellate Courts

The trial court convicted petitioner of illegal possession of marijuana residue


largely on the strength of the arresting officers testimony. Patrolmen Espiritu and
Lumabas were "neutral and disinterested" witnesses, testifying only on what
transpired during the performance of their duties. Substantially, they asserted that
the appellant was found to be in possession of a substance which was later
identified as crushed marijuana residue.

The trial court disbelieved appellant’s defense that this charge was merely
"trumped up," because the appellant neither took any legal action against the
allegedly erring policemen nor moved for a reinvestigation before the city fiscal of
Kalookan City.

On appeal, Respondent Court found no proof that the decision of the trial court was
based on speculations, surmises or conjectures. On the alleged "serious"
discrepancies in the testimonies of the arresting officers, the appellate court ruled
that the said inconsistencies were insubstantial to impair the essential veracity of
the narration. It further found petitioner’s contention — that he could not be
convicted of illegal possession of marijuana residue — to be without merit,
because the forensic chemist reported that what she examined were marijuana
leaves.chanroblesvirtuallawlibrary:red

Issues
Petitioner assigns the following errors on the part of Respondent Court.

"I

The Court of Appeals erred in upholding the findings of fact of the trial court.

II

The Court of Appeals erred in upholding the conviction of (the) accused (and) in
ruling that the guilt of the accused had been proved (beyond) reasonable doubt.

III

The Court of Appeals erred in not ruling that the inconsistencies in the testimonies
of the prosecution witnesses were material and substantial and not minor.

IV

The Court of Appeals erred in not appreciating the evidence that the accused was
framed for the purpose of extorting money.

The Court of Appeals erred in not acquitting the accused when the evidence
presented is consistent with both innocence and guilt.

VI

The Court of Appeals erred in admitting the evidence of the prosecution which are
inadmissible in evidence."cralaw virtua1aw library

Restated more concisely, petitioner questions (1) the admissibility of the evidence
against him, (2) the credibility of prosecution witnesses and the rejection by the
trial and the appellate courts of the defense of extortion, and (3) the sufficiency of
the prosecution evidence to sustain his conviction.

The Court’s Ruling


The petition has no merit.

First Issue: Admissibility of the Evidence Seized

During a Stop-and-Frisk

Petitioner protests the admission of the marijuana leaves found in his possession,
contending that they were products of an illegal search. The Solicitor General, in
his Comment dated July 5, 1994, which was adopted as memorandum for
respondent, counters that the inadmissibility of the marijuana leaves was waived
because petitioner never raised this issue in the proceedings below nor did he
object to their admissibility in evidence. He adds that, even assuming arguendo
that there was no waiver, the search was legal because it was incidental to a
warrantless arrest under Section 5 (a), Rule 113 of the Rules of Court.

We disagree with petitioner and hold that the search was valid, being akin to a
stop-and-frisk. In the landmark case of Terry v. Ohio, 18 a stop-and-frisk was
defined as the vernacular designation of the right of a police officer to stop a
citizen on the street, interrogate him, and pat him for
weapon(s):jgc:chanrobles.com.ph

". . . (W)here a police officer observes an unusual conduct which leads him
reasonably to conclude in light of his experience that criminal activity may be
afoot and that the persons with whom he is dealing may be armed and presently
dangerous, where in the course of investigating this behavior he identified himself
as a policeman and makes reasonable inquiries, and where nothing in the initial
stages of the encounter serves to dispel his reasonable fear for his own or others’
safety, he is entitled for the protection of himself and others in the area to conduct
a carefully limited search of the outer clothing of such persons in an attempt to
discover weapons which might be used to assault him. Such a search is a
reasonable search under the Fourth Amendment, and any weapon seized may
properly be introduced in evidence against the person from whom they were
taken." 19

In allowing such a search, the United States Supreme Court held that the interest of
effective crime prevention and detection allows a police officer to approach a
person, in appropriate circumstances and manner, for purposes of investigating
possible criminal behavior even though there is insufficient probable cause to make
an actual arrest. This was the legitimate investigative function which Officer
McFadden discharged in that case, when he approached petitioner and his
companion whom he observed to have hovered alternately about a street corner for
an extended period of time, while not waiting for anyone; paused to stare in the
same store window roughly 24 times; and conferred with a third person. It would
have been sloppy police work for an officer of 30 years experience to have failed
to investigate this behavior further.

In admitting in evidence two guns seized during the stop-and-frisk, the US


Supreme Court held that what justified the limited search was the more immediate
interest of the police officer in taking steps to assure himself that the person with
whom he was dealing was not armed with a weapon that could unexpectedly and
fatally be used against him.

It did not, however, abandon the rule that the police must, whenever practicable,
obtain advance judicial approval of searches and seizures through the warrant
procedure, excused only by exigent circumstances.

In Philippine jurisprudence, the general rule is that a search and seizure must be
validated by a previously secured judicial warrant; otherwise, such search and
seizure is unconstitutional and subject to challenge. 20 Section 2, Article III of the
1987 Constitution, gives this guarantee:jgc:chanrobles.com.ph

"SEC. 2. The right of the people to be secure in their persons, houses, papers, and
effects against unreasonable searches and seizures of whatever nature and for any
purpose shall be inviolable, and no search warrant or warrant of arrest shall issue
except upon probable cause to be determined personally by the judge after
examination under oath or affirmation of the complainant and the witnesses he
may produce, and particularly describing the place to be searched and the persons
or things to be seized."cralaw virtua1aw library

Any evidence obtained in violation of the mentioned provision is legally


inadmissible in evidence as a "fruit of the poisonous tree," falling under the
exclusionary rule:jgc:chanrobles.com.ph

"SEC. 3. . . .

(2) Any evidence obtained in violation of . . . the preceding section shall be


inadmissible for any purpose in any proceeding."cralaw virtua1aw library

This right, however, is not absolute. 21 The recent case of People v. Lacerna
enumerated five recognized exceptions to the rule against warrantless search and
seizure, viz.:" (1) search incidental to a lawful arrest, (2) search of moving
vehicles, (3) seizure in plain view, (4) customs search, and (5) waiver by the
accused themselves of their right against unreasonable search and seizure." 22 In
People v. Encinada, 23 the Court further explained that" [in] these cases, the search
and seizure may be made only with probable cause as the essential requirement.
Although the term eludes exact definition, probable cause for a search is, at best,
defined as a reasonable ground of suspicion, supported by circumstances
sufficiently strong in themselves to warrant a cautious man in the belief that the
person accused is guilty of the offense with which he is charged; or the existence
of such facts and circumstances which could lead a reasonably discreet and prudent
man to believe that an offense has been committed and that the item(s), article(s) or
object(s) sought in connection with said offense or subject to seizure and
destruction by law is in the place to be searched."cralaw virtua1aw library

Stop-and-frisk has already been adopted as another exception to the general rule
against a search without a warrant. In Posadas v. Court of Appeals, 24 the Court
held that there were many instances where a search and seizure could be effected
without necessarily being preceded by an arrest, one of which was stop-and-frisk.
In said case, members of the Integrated National Police of Davao stopped
petitioner, who was carrying a buri bag and acting suspiciously. They found inside
petitioner’s bag one .38-cal. revolver with two rounds of live ammunition, two live
ammunitions for a .22-cal. gun and a tear gas grenade. In upholding the legality of
the search, the Court said that to require the police officers to search the bag only
after they had obtained a search warrant might prove to be useless, futile and much
too late under the circumstances. In such a situation, it was reasonable for a police
officer to stop a suspicious individual briefly in order to determine his identity or
to maintain the status quo while obtaining more information, rather than to simply
shrug his shoulders and allow a crime to occur.

In the case at hand, Patrolman Espiritu and his companions observed during their
surveillance that appellant had red eyes and was wobbling like a drunk along the
Caloocan City Cemetery, which according to police information was a popular
hangout of drug addicts. From his experience as a member of the Anti-Narcotics
Unit of the Caloocan City Police, such suspicious behavior was characteristic of
drug addicts who were "high." The policemen therefore had sufficient reason to
stop petitioner to investigate if he was actually high on drugs. During such
investigation, they found marijuana in petitioner’s possession:25cralaw:red

"FISCAL RALAR:chanrob1es virtual 1aw library

Q And why were you conducting surveillance in front of the Caloocan Cemetery,
Sangandaan, Caloocan City?

A Because there were some informations that some drug dependents were roaming
around at A. Mabini Street in front of the Caloocan Cemetery, Caloocan City.

x       x       x

Q While you were conducting your surveillance, together with Pat. Angel Lumabas
and one Arnold Enriquez, what happened, if any?
A We chanced upon one male person there in front of the Caloocan Cemetery then
when we called his attention he tried to avoid us, then prompting us to approach
him and introduce ourselves as police officers in a polite manner.chanroblesvirtual|
awlibrary

x       x       x

Q Could you describe to us the appearance of that person when you chanced upon
him?

A That person seems like he is high on drug.

Q How were you able to say Mr. Witness that that person that you chanced upon
was high on drug?

A Because his eyes were red and he was walking on a swaying manner.

Q What was he doing in particular when you chanced upon him?

A He was roaming around, sir.

Q You said that he avoided you, what did you do when he avoided you?

A We approached him and introduced ourselves as police officers in a polite


manner, sir.

Q How did you introduce yourselves?

A In a polite manner, sir.

Q What did you say when you introduced yourselves?

A We asked him what he was holding in his hands, sir.

Q And what was the reaction of the person when you asked him what he was
holding in his hands?

A He tried to resist, sir.

Q When he tried to resist, what did you do?

A I requested him if I can see what was he was (sic) holding in his hands.
Q What was the answer of the person upon your request?

A He allowed me to examine that something in his hands, sir.

x       x       x

Q What was he holding?

A He was holding his wallet and when we opened it, there was a marijuana (sic)
crushed residue."cralaw virtua1aw library

Furthermore, we concur with the Solicitor General’s contention that petitioner


effectively waived the inadmissibility of any evidence illegally obtained when he
filed to raise this issue or to object thereto during the trial. A valid waiver of a
right, more particularly of the constitutional right against unreasonable search,
requires the concurrence of the following requirements: (1) the right to be waived
existed; (2) the person waiving it had knowledge, actual or constructive, thereof;
and (3) he or she had an actual intention to relinquish the right. 26 Otherwise, the
Courts will indulge every reasonable presumption against waiver of fundamental
safeguards and will not deduce acquiescence from the failure to exercise this
elementary right. In the present case, however, petitioner is deemed to have waived
such right for his failure to raise its violation before the trial court. In petitions
under Rule 45, as distinguished from an ordinary appeal of criminal cases where
the whole case is opened for review, the appeal is generally limited to the errors
assigned by petitioner. Issues not raised below cannot be pleaded for the first time
on appeal. 27

Second Issue: Assessment of Evidence

Petitioner also contends that the two arresting officers’ testimony contained
"polluted, irreconcilable and unexplained" contradictions which did not support
petitioner’s conviction.

We disagree. Time and again, this Court has ruled that the trial court’s assessment
of the credibility of witnesses, particularly when affirmed by the Court of Appeals
as in this case, is accorded great weight and respect, since it had the opportunity to
observe their demeanor and deportment as they testified before it. Unless
substantial facts and circumstances have been overlooked or misappreciated by the
trial court which, if considered, would materially affect the result of the case, we
will not countenance a departure from this rule. 28

We concur with Respondent Court’s ruling:jgc:chanrobles.com.ph


"(e)ven assuming as contended by appellant that there had been some
inconsistencies in the prosecution witnesses’ testimonies, We do not find them
substantial enough to impair the essential veracity of their narration. In People v.
Avila, it was held that — "As long as the witnesses concur on the material points,
slight differences in their remembrance of the details, do not reflect on the essential
veracity of their statements."cralaw virtua1aw library

However, we find that, aside from the presumption of regularity in the performance
of duty, the bestowal of full credence on Pat. Espiritu’s testimony is justified by
tangible evidence on record. Despite Pat. Lumabas’ contradictory testimony, that
of Espiritu is supported by the Joint Affidavit 29 signed by both arresting
policemen. The question of whether the marijuana was found inside petitioner’s
wallet or inside a plastic bag is immaterial, considering that petitioner did not deny
possession of said substance. Failure to present the wallet in evidence did not
negate that marijuana was found in petitioner’s possession. This shows that such
contradiction is minor and does not destroy Espiritu’s credibility. 30

Third Issue: Sufficiency of Evidence

The elements of illegal possession of marijuana are: (a) the accused is in


possession of an item or object which is identified to be a prohibited drug; (b) such
possession is not authorized by law; and (c) the accused freely and consciously
possessed the said drug. 31

The substance found in petitioner’s possession was identified by NBI Forensic


Chemist Aida Pascual to be crushed marijuana leaves. Petitioner’s lack of authority
to possess these leaves was established. His awareness thereof was undeniable,
considering that petitioner was high on drugs when stopped by the policemen and
that he resisted when asked to show and identify the thing he was holding. Such
behavior clearly shows that petitioner knew that he was holding marijuana and that
it was prohibited by law.chanrobles.com : virtual lawlibrary

Furthermore, like the trial and the appellate courts, we have not been given
sufficient grounds to believe the extortion angle in this case. Petitioner did not file
any administrative or criminal case against the arresting officers or present any
evidence other than his bare claim. His argument that he feared for his life was
lame and unbelievable, considering that he was released on bail and continued to
be on bail as early as April 26, 1988. 32 Since then, he could have made the charge
in relative safety, as he was no longer in the custody of the police. His defense of
frame-up, like alibi, is viewed by this Court with disfavor, because it is easy to
concoct and fabricate. 33

The Proper Penalty


The trial and the appellate courts overlooked the Indeterminate Sentence Law (Act
No. 4103, as amended) by sentencing petitioner to a straight penalty of six years
and one day of imprisonment, aside from the imposed fine of six thousand pesos.
This Act requires the imposition of an indeterminate
penalty:jgc:chanrobles.com.ph

"SEC. 1. Hereafter, in imposing a prison sentence for an offense punished by the


Revised Penal Code, or its amendments, the court shall sentence the accused to an
indeterminate sentence the maximum term of which shall be that which, in view of
the attending circumstances, could be properly imposed under the rules of the said
Code, and the minimum which shall be within the range of the penalty next lower
to that prescribed by the Code for the offense; and if the offense is punished by any
other law, the court shall sentence the accused to an indeterminate sentence, the
maximum term of which shall not exceed the maximum fixed by said law and the
minimum shall not be less than the minimum term prescribed by the same. (As
amended by Act No. 4225.)

"SEC. 2. This Act shall not apply to persons convicted of offenses punished with
death penalty or life-imprisonment; to those convicted of treason; to those
convicted of misprision of treason, rebellion, sedition or espionage; to those
convicted of piracy; to those who are habitual delinquents; to those who shall have
escaped from confinement or evaded sentence; to those who having been granted
conditional pardon by the Chief Executive shall have violated the terms thereof; to
those whose maximum term of imprisonment does not exceed one year, not to
those already sentenced by final judgment at the time of approval of this Act,
except as provided in Section 5 hereof." (Emphasis supplied)

The Dangerous Drugs Law, R.A. 6425, as amended by B.P. 179, imposes the
following penalty for illegal possession of marijuana:jgc:chanrobles.com.ph

"Sec. 8. . . .

The penalty of imprisonment ranging from six years and one day to twelve years
and a fine ranging from six thousand to twelve thousand pesos shall be imposed
upon any person who, unless authorized by law, shall possess or use Indian
hemp."cralaw virtua1aw library

Prescinding from the foregoing, the Court holds that the proper penalty is an
indeterminate sentence of imprisonment ranging from six years and one day to
twelve years. 34

WHEREFORE, the assailed Decision and Resolution are hereby AFFIRMED with
MODIFICATION. Petitioner is sentenced to suffer IMPRISONMENT of SIX (6)
YEARS, as minimum, to TWELVE (12) YEARS, as maximum, and to PAY a
FINE of SIX THOUSAND PESOS. Costs against petitioner.

SO ORDERED.

Narvasa, C.J., Romero, Melo and Francisco, JJ., concur.

People v. Mengote, G.R. No. 87059 June 22, 1992


[G.R. No. 87059. June 22, 1992.]

THE PEOPLE OF THE PHILIPPINES, Plaintiff-Appellee, v. ROGELIO


MENGOTE Y TEJAS, Accused-Appellant.

CRUZ, J.:

Accused-appellant Rogelio Mengote was convicted of illegal possession of


firearms on the strength mainly of the stolen pistol found on his person at the
moment of his warrantless arrest. In this appeal, he pleads that the weapon was not
admissible as evidence against him because it had been illegally seized and was
therefore the fruit of the poisonous tree. The Government disagrees. It insists that
the revolver was validly received in evidence by the trial judge because its seizure
was incidental to an arrest that was doubtless lawful even if admittedly without
warrant.

The incident occurred shortly before noon of August 8, 1987, after the Western
Police District received a telephone call from an informer that there were three
suspicious-looking persons at the corner of Juan Luna and North Bay Boulevard in
Tondo, Manila. A surveillance team of plainclothesmen was forthwith dispatched
to the place. As later narrated at the trial by Patrolmen Rolando Mercado and
Alberto Juan, 1 they there saw two men "looking from side to side," one of whom
was holding his abdomen. They approached these persons and identified
themselves as policemen, whereupon the two tried to run away but were unable to
escape because the other lawmen had surrounded them. The suspects were then
searched. One of them, who turned out to be the accused-appellant, was found with
a .38 caliber Smith and Wesson revolver with six live bullets in the chamber. His
companion, later identified as Nicanor Morellos, had a fan knife secreted in his
front right pantspocket. The weapons were taken from them. Mengote and
Morellos were then turned over to police headquarters for investigation by the
Intelligence Division.chanrobles law library : red

On August 11, 1987, the following information was filed against the accused-
appellant before the Regional Trial Court of Manila:chanrob1es virtual 1aw library
The undersigned accuses ROGELIO MENGOTE y TEJAS of a violation of
Presidential Decree No. 1866, committed as follows:chanrob1es virtual 1aw library

That on or about August 8, 1987, in the City of Manila, Philippines, the said
accused did then and there wilfully, unlawfully and knowingly have in his
possession and under his custody and control a firearm, to wit:chanrob1es virtual
1aw library

one (1) cal. 38 "S & W" bearing

Serial No. 8720-T.

without first having secured the necessary license or permit therefor from the
proper authorities.

Besides the police officers, one other witness presented by the prosecution was
Rigoberto Danganan, who identified the subject weapon as among the articles
stolen from him during the robbery in his house in Malabon on June 13, 1987. He
pointed to Mengote as one of the robbers. He had duly reported the robbery to the
police, indicating the articles stolen from him, including the revolver. 2 For his
part, Mengote made no effort to prove that he owned the firearm or that he was
licensed to possess it and claimed instead that the weapon had been "planted" on
him at the time of his arrest. 3

The gun, together with the live bullets and its holster, were offered as Exhibits A,
B and C and admitted over the objection of the defense. As previously stated, the
weapon was the principal evidence that led to Mengote’s conviction for violation
of P.D. 1866. He was sentenced to reclusion perpetua. 4

It is submitted in the Appellant’s Brief that the revolver should not have been
admitted in evidence because of its illegal seizure, no warrant therefor having been
previously obtained. Neither could it have been seized as an incident of a lawful
arrest because the arrest of Mengote was itself unlawful, having been also effected
without a warrant. The defense also contends that the testimony regarding the
alleged robbery in Danganan’s house was irrelevant and should also have been
disregarded by the trial court.

The following are the pertinent provision of the Bill of Rights:chanrob1es virtual
1aw library

Sec. 2. The right of the people to be secure in their persons, houses, papers, and
effects against unreasonable searches and seizures of whatever nature and for any
purpose shall be inviolable, and no search warrant or warrant of arrest shall issue
except upon probable cause to be determined personally by the judge after
examination under oath or affirmation of the complainant and the witnesses he
may produce, and particularly describing the place to be searched and the persons
or things to be seized.

Sec. 3 (1). The privacy of communication and correspondence shall be inviolable


except upon lawful order of the court, or when public safety or order requires
otherwise as prescribed by law.

(2) Any evidence obtained in violation of this or the preceding section shall be
inadmissible for any purpose in any proceeding.

There is no question that evidence obtained as a result of an illegal search or


seizure is inadmissible in any proceeding for any purpose. That is the absolute
prohibition of Article III, Section 3(2), of the Constitution. This is the celebrated
exclusionary rule based on the justification given by Judge Learned Hand that
"only in case the prosecution, which itself controls the seizing officials, knows that
it cannot profit by their wrong will the wrong be repressed."cralaw virtua1aw
library

The Solicitor General, while conceding the rule, maintains that it is not applicable
in the case at bar. His reason is that the arrest and search of Mengote and the
seizure of the revolver from him were lawful under Rule 113, Section 5, of the
Rules of Court reading as follows:chanrob1es virtual 1aw library

Sec. 5. Arrest without warrant; when lawful. — A peace officer or private person
may without a warrant, arrest a person:chanrobles virtual lawlibrary

(a) When, in his presence, the person to be arrested has committed, is actually
committing, or is attempting to commit an offense;

(b) When an offense has in fact just been committed, and he has personal
knowledge of facts indicating that the person to be arrested has committed it; and

(c) When the person to be arrested is a prisoner who has escaped from a penal
establishment or place where he is serving final judgment or temporarily confined
while his case is pending, or has escaped while being transferred from one
confinement to another.

In cases falling under paragraphs (a) and (b) hereof, the person arrested without a
warrant shall be forthwith delivered to the nearest police station or jail, and he shall
be proceeded against in accordance with Rule 112, Section 7.

We have carefully examined the wording of this rule and cannot see how we can
agree with the prosecution.

Par. (c) of Section 5 is obviously inapplicable as Mengote was not an escapee from
a penal institution when he was arrested. We therefore confine ourselves to
determining the lawfulness of his arrest under either Par. (a) or Par. (b) of this
section.

Par. (a) requires that the person be arrested (1) after he has committed or while he
is actually committing or is at least attempting to commit an offense, (2) in the
presence of the arresting officer.

These requirements have not been established in the case at bar. At the time of the
arrest in question, the accused-appellant was merely "looking from side to side"
and "holding his abdomen," according to the arresting officers themselves. There
was apparently no offense that had just been committed or was being actually
committed or at least being attempted by Mengote in their presence.

The Solicitor General submits that the actual existence of an offense was not
necessary as long as Mengote’s acts "created a reasonable suspicion on the part of
the arresting officers and induced in them the belief that an offense had been
committed and that the accused-appellant had committed it." The question is, What
offense? What offense could possibly have been suggested by a person "looking
from side to side" and "holding his abdomen" and in a place not exactly forsaken?

These are certainly not sinister acts. And the setting of the arrest made them less
so, if at all. It might have been different if Mengote had been apprehended at an
ungodly hour and in a place where he had no reason to be, like a darkened alley at
3 o’clock in the morning. But he was arrested at 11:30 in the morning and in a
crowded street shortly after alighting from a passenger jeep with his companion.
He was not skulking in the shadows but walking in the clear light of day. There
was nothing clandestine about his being on that street at that busy hour in the blaze
of the noonday sun.

On the other hand, there could have been a number of reasons, all of them
innocent, why his eyes were darting from side to side and he was holding his
abdomen. If they excited suspicion in the minds of the arresting officers, as the
prosecution suggests, it has nevertheless not been shown what their suspicion was
all about. In fact, the policemen themselves testified that they were dispatched to
that place only because of the telephone call from the informer that there were
"suspicious-looking" persons in that vicinity who were about to commit a robbery
at North Bay Boulevard. The caller did not explain why he thought the men looked
suspicious nor did he elaborate on the impending crime.chanrobles law library : red

In the recent case of People v. Malmstedt, 5 the Court sustained the warrantless
arrest of the accused because there was a bulge in his waist that excited the
suspicion of the arresting officer and, upon inspection, turned out to be a pouch
containing hashish. In People v. Claudio, 6 the accused boarded a bus and placed
the buri bag she was carrying behind the seat of the arresting officer while she
herself sat in the seat before him. His suspicion aroused, he surreptitiously
examined the bag, which he found to contain marijuana. He then and there made
the warrantless arrest and seizure that we subsequently upheld on the ground that
probable cause had been sufficiently established.

The case before us is different because there was nothing to support the arresting
officers’ suspicion other than Mengote’s darting eyes and his hand on his
abdomen. By no stretch of the imagination could it have been inferred from these
acts that an offense had just been committed, or was actually being committed, or
was at least being attempted in their presence.

This case is similar to People v. Aminnudin, 7 where the Court held that the
warrantless arrest of the accused was unconstitutional. This was effected while he
was coming down a vessel, to all appearances no less innocent than the other
disembarking passengers. He had not committed nor was he actually committing or
attempting to commit an offense in the presence of the arresting officers. He was
not even acting suspiciously. In short, there was no probable cause that, as the
prosecution incorrectly suggested, dispensed with the constitutional requirement of
a warrant.

Par. (b) is no less applicable because its no less stringent requirements have also
not been satisfied. The prosecution has not shown that at the time of Mengote’s
arrest an offense had in fact just been committed and that the arresting officers had
personal knowledge of facts indicating that Mengote had committed it. All they
had was hearsay information from the telephone caller, and about a crime that had
yet to be committed.

The truth is that they did not know then what offense, if at all, had been committed
and neither were they aware of the participation therein of the Accused-Appellant.
It was only later, after Danganan had appeared at the police headquarters, that they
learned of the robbery in his house and of Mengote’s supposed involvement
therein. 8 As for the illegal possession or the firearm found on Mengote’s person,
the policemen discovered this only after he had been searched and the investigation
conducted later revealed that he was not its owners nor was he licensed to possess
it.

Before these events, the peace officers had no knowledge even of Mengote’
identity, let alone the fact (or suspicion) that he was unlawfully carrying a firearm
or that he was involved in the robbery of Danganan’s house.
In the landmark case of People v. Burgos, 9 this Court declared:chanrob1es virtual
1aw library

Under Section 6(a) of Rule 113, the officer arresting a person who has just
committed, is committing, or is about to commit an offense must have personal
knowledge of the fact. The offense must also be committed in is presence or within
his view. (Sayo v. Chief of Police, 80 Phil. 859). (Emphasis supplied)

x       x       x

In arrests without a warrant under Section 6(b), however, it is not enough that there
is reasonable ground to believe that the person to be arrested has committed a
crime. A crime must in fact or actually have been committed first. That a crime has
actually been committed is an essential precondition. It is not enough to suspect
that a crime may have been committed. The fact of the commission of the offense
must be undisputed. The test of reasonable ground applies only to the identity of
the perpetrator. (Emphasis supplied)

This doctrine was affirmed in Alih v. Castro, 10 thus:chanrob1es virtual 1aw


library

If the arrest was made under Rule 113, Section 5, of the Rules of Court in
connection with a crime about to be committed, being committed, or just
committed, what was that crime? There is no allegation in the record of such a
justification. Parenthetically, it may be observed that under the Revised Rule 113,
Section 5(b), the officer making the arrest must have personal knowledge of the
ground therefor as stressed in the recent case of People v. Burgos. (Emphasis
supplied)

It would be a sad day, indeed, if any person could be summarily arrested and
searched just because he is holding his abdomen, even if it be possibly because of a
stomach-ache, or if a peace officer-could clamp handcuffs on any person with a
shifty look on suspicion that he may have committed a criminal act or is actually
committing or attempting it. This simply cannot be done in a free society. This is
not a police state where order is exalted over liberty or, worse, personal malice on
the part of the arresting officer may be justified in the name of security.

There is no need to discuss the other issues raised by the accused-appellant as the
ruling we here make is sufficient to sustain his exoneration. Without the evidence
of the firearm taken from him at the time of his illegal arrest, the prosecution has
lost its most important exhibit and must therefore fail. The testimonial evidence
against Mengote (which is based on the said firearm) is not sufficient to prove his
guilt beyond reasonable doubt of the crime imputed to him.
We commend Atty. Violeta Calvo-Drilon for her able and spirited defense of the
accused-appellant not only in the brief but also in the reply brief, which she did not
have to file but did so just the same to stress the constitutional rights of her client.
The fact that she was acting only as a counsel de oficio with no expectation of
material reward makes her representation even more commendable.

The Court feels that if the peace officers had been more mindful of the provisions
of the Bill of Rights, the prosecution of the accused-appellant might have
succeeded. As it happened, they allowed their over-zealousness to get the better of
them, resulting in their disregard of the requirements of a valid search and seizure
that rendered inadmissible the vital evidence they had invalidly seized.chanrobles
law library : red

This should be a lesson to other peace officers. Their impulsiveness may be the
very cause of the acquittal of persons who deserve to be convicted, escaping the
clutches of the law because, ironically enough, it has not been observed by those
who are supposed to enforce it.

WHEREFORE, the appealed decision is REVERSED and SET ASIDE. The


accused-appellant is ACQUITTED and ordered released immediately unless he is
validly detained for other offenses. No costs.

SO ORDERED.

Griño-Aquino, Medialdea and Bellosillo, JJ., concur.

Posadas v. CA, G.R. No. 89139, August 2, 1990

The validity of a warrantless search on the person of petitioner is put into issue in
this case.

On October 16, 1986 at about 10:00 o'clock in the morning Pat. Ursicio Ungab and
Pat. Umbra Umpar, both members of the Integrated National Police (INP) of the
Davao Metrodiscom assigned with the Intelligence Task Force, were conducting a
surveillance along Magallanes Street, Davao City. While they were within the
premises of the Rizal Memorial Colleges they spotted petitioner carrying a "buri"
bag and they noticed him to be acting suspiciously.

They approached the petitioner and identified themselves as members of the INP.
Petitioner attempted to flee but his attempt to get away was thwarted by the two
notwithstanding his resistance.

They then checked the "buri" bag of the petitioner where they found one (1) caliber
.38 Smith & Wesson revolver with Serial No. 770196 1 two (2) rounds of live
ammunition for a .38 caliber gun 2 a smoke (tear gas) grenade,3 and two (2) live
ammunitions for a .22 caliber gun. 4 They brought the petitioner to the police
station for further investigation. In the course of the same, the petitioner was asked
to show the necessary license or authority to possess firearms and ammunitions
found in his possession but he failed to do so. He was then taken to the Davao
Metrodiscom office and the prohibited articles recovered from him were indorsed
to M/Sgt. Didoy the officer then on duty. He was prosecuted for illegal possession
of firearms and ammunitions in the Regional Trial Court of Davao City wherein
after a plea of not guilty and trial on the merits a decision was rendered on October
8, 1987 finding petitioner guilty of the offense charged as follows:

WHEREFORE, in view of all the foregoing, this Court , finds the


accused guilty beyond reasonable doubt of the offense charged.

It appearing that the accuse d was below eighteen (18) years old at the
time of the commission of the offense (Art. 68, par. 2), he is hereby
sentenced to an indeterminate penalty ranging from TEN (10)
YEARS and ONE (1) DAY of prision mayor to TWELVE (12)
Years, FIVE (5) months and Eleven (11) days of Reclusion Temporal,
and to pay the costs.

The firearm, ammunitions and smoke grenade are forfeited in favor of


the government and the Branch Clerk of Court is hereby directed to
turn over said items to the Chief, Davao Metrodiscom, Davao City. 5

Not satisfied therewith the petitioner interposed an appeal to the Court of Appeals
wherein in due course a decision was rendered on February 23, 1989 affirming in
toto the appealed decision with costs against the petitioner. 6

Hence, the herein petition for review, the main thrust of which is that there being
no lawful arrest or search and seizure, the items which were confiscated from the
possession of the petitioner are inadmissible in evidence against him.

The Solicitor General, in justifying the warrantless search of the buri bag then
carried by the petitioner, argues that under Section 12, Rule 136 of the Rules of
Court a person lawfully arrested may be searched for dangerous weapons or
anything used as proof of a commission of an offense without a search warrant. It
is further alleged that the arrest without a warrant of the petitioner was lawful
under the circumstances.

Section 5, Rule 113 of the 1985 Rules on Criminal Procedure provides as follows:

SEC. 5. Arrest without warrant; when lawful — A peace officer or a


private person may, without a warrant, arrest a person:
(a) When in his presence, the person to be arrested has committed is
actually committing, or is attempting to commit an offense;

(b) When an offense has in fact just been committed, and he has
personal knowledge of facts indicating that the person to be arrested
has committed it; and

(c) When the person to be arrested is a prisoner who has escaped from
a penal establishment or place where he is serving final judgment or
temporarily confined while his case is pending, or has escaped while
being transferred from one confinement to another.

In cases falling under paragraphs (a) and (b) hereof, the person
arrested without a warrant shall be forthwith delivered to the nearest
police station or jail, and he shall be proceeded against in accordance
with Rule 112, Section 7. (6a, 17a)

From the foregoing provision of law it is clear that an arrest without a warrant may
be effected by a peace officer or private person, among others, when in his
presence the person to be arrested has committed, is actually committing, or is
attempting to commit an offense; or when an offense has in fact just been
committed, and he has personal knowledge of the facts indicating that the person
arrested has committed it.

The Solicitor General argues that when the two policemen approached the
petitioner, he was actually committing or had just committed the offense of illegal
possession of firearms and ammunitions in the presence of the police officers and
consequently the search and seizure of the contraband was incidental to the lawful
arrest in accordance with Section 12, Rule 126 of the 1985 Rules on Criminal
Procedure. We disagree.

At the time the peace officers in this case identified themselves and apprehended
the petitioner as he attempted to flee they did not know that he had committed, or
was actually committing the offense of illegal possession of firearms and
ammunitions. They just suspected that he was hiding something in the buri bag.
They did now know what its contents were. The said circumstances did not justify
an arrest without a warrant.

However, there are many instances where a warrant and seizure can be effected
without necessarily being preceded by an arrest, foremost of which is the "stop and
search" without a search warrant at military or police checkpoints, the
constitutionality or validity of which has been upheld by this Court in Valmonte
vs. de Villa, 7 as follows:
Petitioner Valmonte's general allegation to the effect that he had been
stopped and searched without a search warrant by the military
manning the checkpoints, without more, i.e., without stating the
details of the incidents which amount to a violation of his light
against unlawful search and seizure, is not sufficient to enable the
Court to determine whether there was a violation of Valmonte's right
against unlawful search and seizure. Not all searches and seizures
are prohibited. Those which are reasonable are not forbidden. A
reasonable search is not to be determined by any fixed formula but is
to be resolved according to the facts of each case.

Where, for example, the officer merely draws aside the curtain of a
vacant vehicle which is parked on the public fair grounds, or simply
looks into a vehicle or flashes a light therein, these do not constitute
unreasonable search.

The setting up of the questioned checkpoints in Valenzuela (and


probably in other areas) may be considered as a security measure to
enable the NCRDC to pursue its mission of establishing effective
territorial defense and maintaining peace and order for the benefit of
the public. Checkpoints may also be regarded as measures to thwart
plots to destabilize the government in the interest of public security.
In this connection, the Court may take judicial notice of the shift to
urban centers and their suburbs of the insurgency movement, so
clearly reflected in the increased killings in cities of police and
military men by NPA "sparrow units," not to mention the abundance
of unlicensed firearms and the alarming rise in lawlessness and
violence in such urban centers, not all of which are reported in media,
most likely brought about by deteriorating economic conditions —
which all sum up to what one can rightly consider, at the very least,
as abnormal times. Between the inherent right of the state to protect
its existence and promote public welfare and an individual's right
against a warrantless search which is however reasonably
conducted, the former should prevail.

True, the manning of checkpoints by the military is susceptible of


abuse by the men in uniform in the same manner that all
governmental power is susceptible of abuse. But, at the cost of
occasional inconvenience, discomfort and even irritation to the
citizen, the checkpoints during these abnormal times, when conducted
within reasonable limits, are part of the price we pay for an orderly
society and a peaceful community. (Emphasis supplied).

Thus, as between a warrantless search and seizure conducted at military or police


checkpoints and the search thereat in the case at bar, there is no question that,
indeed, the latter is more reasonable considering that unlike in the former, it was
effected on the basis of a probable cause. The probable cause is that when the
petitioner acted suspiciously and attempted to flee with the buri bag there was a
probable cause that he was concealing something illegal in the bag and it was the
right and duty of the police officers to inspect the same.

It is too much indeed to require the police officers to search the bag in the
possession of the petitioner only after they shall have obtained a search warrant for
the purpose. Such an exercise may prove to be useless, futile and much too late.

In People vs. CFI of Rizal, 8 this Court held as follows:

. . . In the ordinary cases where warrant is indispensably necessary,


the mechanics prescribed by the Constitution and reiterated in the
Rules of Court must be followed and satisfied. But We need not argue
that there are exceptions. Thus in the extraordinary events where
warrant is not necessary to effect a valid search or seizure, or when
the latter cannot be performed except without warrant, what
constitutes a reasonable or unreasonable search or seizure becomes
purely a judicial question, determinable from the uniqueness of the
circumstances involved, including the purpose of the search or
seizure, the presence or absence of probable cause, the manner in
which the search and seizure was made, the place or thing searched
and the character of the articles procured.

The Court reproduces with approval the following disquisition of the Solicitor
General:

The assailed search and seizure may still be justified as akin to a "stop
and frisk" situation whose object is either to determine the identity of
a suspicious individual or to maintain the status quo momentarily
while the police officer seeks to obtain more information. This is
illustrated in the case of Terry vs. Ohio, 392 U.S. 1 (1968). In this
case, two men repeatedly walked past a store window and returned to
a spot where they apparently conferred with a third man. This aroused
the suspicion of a police officer. To the experienced officer, the
behaviour of the men indicated that they were sizing up the store for
an armed robbery. When the police officer approached the men and
asked them for their names, they mumbled a reply. Whereupon, the
officer grabbed one of them, spun him around and frisked him.
Finding a concealed weapon in one, he did the same to the other two
and found another weapon. In the prosecution for the offense of
carrying a concealed weapon, the defense of illegal search and seizure
was put up. The United States Supreme Court held that "a police
officer may in appropriate circumstances and in an appropriate
manner approach a person for the purpose of investigating possible
criminal behaviour even though there is no probable cause to make an
arrest." In such a situation, it is reasonable for an officer rather than
simply to shrug his shoulder and allow a crime to occur, to stop a
suspicious individual briefly in order to determine his identity or
maintain the status quo while obtaining more information. . . .

Clearly, the search in the case at bar can be sustained under the exceptions
heretofore discussed, and hence, the constitutional guarantee against unreasonable
searches and seizures has not been violated. 9

WHEREFORE, the petition is DENIED with costs against petitioner.

SO ORDERED.

People v. Cogaed, G.R. No. 200334, July 30, 2014

LEONEN, J.:

The mantle of protection upon one's person and one's effects through Article III,
Section 2 of the Constitution is essential to allow citizens to evolve their autonomy
and, hence, to avail themselves of their right to privacy. The alleged compromise
with the battle against dangerous drugs is more apparent than real. Often, the
compromise is there because law enforcers neglect to perform what could have
been done to uphold the Constitution as they pursue those who traffic this scourge
of society.

Squarely raised in· this appeal1 is the admissibility of the evidence seized as a
result of a warrantless arrest. The police officers identified the alleged perpetrator
through facts that were not based on their personal knowledge. The information as
to the accused’s whereabouts was sent through a text message. The accusedwho
never acted suspicious was identified by a driver. The bag that allegedly contained
the contraband was required to be opened under intimidating circumstances and
without the accused having been fully apprised of his rights. This was not a
reasonable search within the meaning of the Constitution. There was no reasonable
suspicion that would allow a legitimate "stop and frisk" action. The alleged waiver
of rights by the accused was not done intelligently, knowingly, and without
improper pressure or coercion.

The evidence, therefore, used against the accused should be excluded consistent
with Article III, Section 3 (2) of the Constitution. There being no possible
admissible evidence, the accused should be acquitted.

I
According to the prosecution, at about 6:00 a.m. of November 25, 2005, Police
Senior Inspector Sofronio Bayan (PSI Bayan) of the San Gabriel Police Station in
San Gabriel,La Union, "received a text message from an unidentified civilian
informer"2 that one Marvin Buya (also known as Marvin Bugat) "[would]be
transporting marijuana"3 from Barangay LunOy, San Gabriel, La Union to the
Poblacion of San Gabriel, La Union.4

PSI Bayan organized checkpoints in order "to intercept the suspect." 5 PSI Bayan
ordered SPO1 Jaime Taracatac, Jr. (SPO1 Taracatac), a member of the San Gabriel
Police, to set up a checkpoint in the waiting area of passengers from San Gabriel
bound for San Fernando City.6 A passenger jeepney from Barangay Lun-Oy
arrived at SPO1 Taracatac’s checkpoint. 7 The jeepney driver disembarked and
signalled to SPO1 Taracatac indicating the two male passengers who were carrying
marijuana.8 SPO1 Taracatac approached the two male passengers who were later
identified as Victor RomanaCogaed and Santiago Sacpa Dayao. 9 Cogaed was
carrying a blue bag and a sack while Dayao was holding a yellow bag. 10

SPO1 Taracatac asked Cogaed and Dayao about the contents of their
bags.11 Cogaed and Dayao told SPO1 Taracatac that they did not know since they
were transporting the bags as a favor for their barriomatenamed Marvin. 12 After
this exchange, Cogaed opened the blue bag, revealing three bricks of what looked
like marijuana.13 Cogaed then muttered, "nagloko daytoy nga Marvinen, kastoymet
gayam ti nagyanna,"which translates to "Marvin is a fool, this is what [is]
contained in the bag."14 "SPO1 Taracatac arrested [Cogaed] and . . . Dayao and
brought them to the police station."15 Cogaed and Dayao "were still carrying their
respective bags"16 inside the station.17

While at the police station, the Chief of Police and Investigator PO3 Stanley
Campit (PO3 Campit) requested Cogaed and Dayao to empty their bags. 18 Inside
Cogaed’s sack was "four (4) rolled pieces of suspected marijuana fruiting
tops,"19 and inside Dayao’s yellow bag was a brick of suspected marijuana. 20

PO3 Campit prepared the suspected marijuana for laboratory testing. 21 PSI Bayan
personally delivered the suspected marijuana to the PNP Crime
Laboratory.22 Forensic Chemical Officer Police Inspector Valeriano Panem Laya II
performed the tests and found that the objects obtained were indeed
marijuana.23 The marijuana collected from Cogaed’s blue bag had a total weight of
8,091.5 grams.24 The marijuana from Cogaed’s sack weighed 4,246.1 grams. 25 The
marijuana collected from Dayao’s bag weighed 5,092 grams. 26 A total of 17,429.6
grams werecollected from Cogaed’s and Dayao’s bags. 27

According to Cogaed’s testimony during trial, he was at Balbalayan, La Union,


"waiting for a jeepney to take him"28 to the Poblacion of San Gabriel so he could
buy pesticide.29 He boarded a jeepney and recognized Dayao, his younger brother’s
friend.30 Upon arrival at the Poblacion of San Gabriel, Dayao and Cogaed alighted
from the jeepney.31 Dayao allegedly "asked for [Cogaed’s] help in carrying his
things, which included a travelling bag and a sack." 32 Cogaed agreed because they
were both going to the market.33 This was when SPO1 Taracatac approached them,
and when SPO1 Taracatac asked Cogaed what was inside the bags, Cogaed replied
that he did not know.34 SPO1 Taracatac then talked to Dayao, however, Cogaed
was not privy to their conversation.35 Thereafter, SPO1 Taracatac arrested Dayao
and Cogaed and brought them to the police station. 36 These facts were corroborated
by an eyewitness,Teodoro Nalpu-ot, who was standing across the parking lot
where Cogaed was apprehended.37

At the police station, Cogaed said that "SPO1 Taracatac hit [him] on the
head."38 The bags were also opened, but Cogaed never knew what was inside. 39

It was only later when Cogaed learned that it was marijuana when he and Dayao
were charged with illegal possession of dangerous drugs under Republic Act No.
9165.40 The information against them states:

That on or about the 25th day of November, 2005, in the Municipality of San
Gabriel, Province of La Union, and within the jurisdiction of this Honorable Court,
the above-named accused VICTOR COGAED Y ROMANA and SANTIAGO
DAYAO Y SACPA (who acted with discernment) and JOHN DOE,conspiring,
confederating and mutually helping one another, did then there wilfully,
unlawfully, feloniously and knowingly, without being authorized by law, have in
their control, custody and possession dried marijuana, a dangerous drug, with a
total weight of seventeen thousand,four hundred twenty-nine and sixtenths (17,
429.6) grams.

CONTRARY TO Section 11 (Possession of Dangerous Drugs), Article II, of


Republic Act No. 9165 (otherwise known as the "Comprehensive Dangerous
Drugs Act of 2002").41

The case was raffled to Regional Trial Court, Branch 28 of San Fernando City, La
Union.42 Cogaed and Dayao pleaded not guilty.43 The case was dismissed against
Dayao because he was only 14 years old at that time and was exempt from criminal
liability under the Juvenile Justice and Welfare Act of 2006 or Republic Act No.
9344.44 Trial against Cogaed ensued. In a decision 45 dated May 21, 2008, the
Regional Trial Court found Cogaed guilty. The dispositive portion of the decision
states:

WHEREFORE, the Court finds accused Victor Cogaed y Romana GUILTY


beyond reasonable doubt for Violation of Section 11, Article II of Republic Act
No. 9165 (otherwise known as the "Comprehensive Dangerous Drugs Act of
2002") and sentences him to suffer life imprisonment, and to pay a fine of one
million pesos (Php 1,000,000.00).46
The trial court judge initiallyfound Cogaed’s arrest illegal considering that
"Cogaed at that time was not, at the moment of his arrest, committing a crime nor
was shown that hewas about to do so or that had just done so. He just alighted from
the passenger jeepney and there was no outward indication that called for his
arrest."47 Since the arrest was illegal, the warrantless search should also be
considered illegal.48 However, the trial court stated that notwithstanding the
illegality of the arrest, Cogaed "waived his right to object to such
irregularity"49 when "he did not protest when SPO1 Taracatac, after identifying
himself, asked him to open his bag."50

Cogaed appealed51 the trial court’s decision.However, the Court of Appeals denied


his appeal and affirmed the trial court’s decision. 52 The Court of Appeals found
that Cogaed waived his right against warrantless searches when "[w]ithout any
prompting from SPO1 Taracatac, [he] voluntarily opened his bag." 53 Hence, this
appeal was filed.

The following errors were assigned by Cogaed in his appellant’s brief:

THE TRIAL COURT GRAVELY ERRED IN ADMITTING THE SEIZED


DANGEROUS DRUGS AS EVIDENCE AGAINST THE ACCUSED-
APPELLANT DESPITE BEING THE RESULT OF AN UNLAWFUL
WARRANTLESS SEARCH AND SEIZURE.

II

THE TRIAL COURT GRAVELY ERRED IN CONVICTING THE ACCUSED-


APPELLANT DESPITE THE ARRESTING OFFICER’S NON-COMPLIANCE
WITH THE REQUIREMENTS FOR THE PROPER CUSTODY OF SEIZED
DANGEROUS DRUGS UNDER REPUBLIC ACT NO. 9165.

III

THE TRIAL COURT GRAVELY ERRED IN CONVICTING THE ACCUSED-


APPELLANT DESPITE THE ARRESTING OFFICER’S FAILURE TO
PRESERVE THE INTEGRITY AND EVIDENTIARY VALUE OF THE SEIZED
DANGEROUS DRUGS.54

For our consideration are the following issues: (1) whether there was a valid search
and seizure of marijuana as against the appellant; (2) whether the evidence
obtained through the search should be admitted; and (3) whether there was enough
evidence to sustain the conviction of the accused.
In view of the disposition of this case, we deem that a discussion with respect to
the requirements on the chain of custody of dangerous drugs unnecessary. 55

We find for the accused.

II

The right to privacy is a fundamental right enshrined by implication in our


Constitution. It has many dimensions. One of its dimensions is its protection
through the prohibition of unreasonable searches and seizures in Article III,
Section 2 of the Constitution:

The right of the people to be secure in their persons, houses, papers, and effects
against unreasonable searches and seizures of whatever nature and for any purpose
shall be inviolable, and no search warrant or warrant of arrest shall issue except
upon probable cause to be determinedpersonally by the judge after examination
under oath or affirmation of the complainant and the witnesses he may produce,
and particularly describing the place to be searched and the persons or things to be
seized.

This provision requires that the court examine with care and diligence whether
searches and seizures are "reasonable." As a general rule, searches conducted with
a warrant that meets all the requirements of this provision are reasonable. This
warrant requires the existence of probable cause that can only be determined by a
judge.56 The existence of probable cause must be established by the judge after
asking searching questions and answers.57 Probable cause at this stage can only
exist if there is an offense alleged to be committed. Also, the warrant frames the
searches done by the law enforcers. There must be a particular description of the
place and the things to be searched.58

However, there are instances when searches are reasonable even when
warrantless.59 In the Rules of Court, searchesincidental to lawful arrests are
allowed even without a separate warrant. 60 This court has taken into account the
"uniqueness of circumstances involved including the purpose of the search or
seizure, the presence or absence of probable cause, the manner in which the search
and seizure was made, the place or thing searched, and the character of the articles
procured."61 The known jurisprudential instances of reasonable warrantless
searches and seizures are:

1. Warrantless search incidental to a lawful arrest. . . ;

2. Seizure of evidence in "plain view," . . . ;

3. Search of a moving vehicle. Highly regulated by the government, the


vehicle’s inherent mobility reduces expectation of privacy especially when
its transit in public thoroughfares furnishes a highly reasonable suspicion
amounting to probable cause that the occupant committed a criminal
activity;

4. Consentedwarrantless search;

5. Customs search;

6. Stop and frisk; and

7. Exigent and emergency circumstances. 62 (Citations omitted)

III

One of these jurisprudential exceptionsto search warrants is "stop and frisk". "Stop
and frisk" searches are often confused with searches incidental to lawful arrests
under the Rules of Court.63 Searches incidental to a lawful arrest require that a
crime be committed in flagrante delicto, and the search conducted within the
vicinity and withinreach by the person arrested is done to ensure that there are no
weapons, as well as to preserve the evidence. 64

On the other hand, "stop and frisk"searches are conducted to prevent the
occurrence of a crime. For instance, the search in Posadas v. Court of
Appeals65 was similar "to a ‘stop and frisk’ situation whose object is either to
determine the identity of a suspicious individual or to maintain the status
quomomentarily while the police officer seeks to obtain more information." 66 This
court stated that the "stop and frisk" search should be used "[w]hen dealing with a
rapidly unfolding and potentially criminal situation in the city streets where
unarguably there is no time to secure . . . a search warrant." 67

The search involved in this case was initially a "stop and frisk" search, but it did
not comply with all the requirements of reasonability required by the Constitution.

"Stop and frisk" searches (sometimes referred to as Terrysearches 68) are necessary
for law enforcement. That is, law enforcers should be given the legal arsenal to
prevent the commission of offenses. However, this should be balanced with the
need to protect the privacy of citizens in accordance with Article III, Section 2 of
the Constitution.

The balance lies in the concept of"suspiciousness" present in the situation where
the police officer finds himself or herself in. This may be undoubtedly based on the
experience ofthe police officer. Experienced police officers have personal
experience dealing with criminals and criminal behavior. Hence, they should have
the ability to discern — based on facts that they themselves observe — whether an
individual is acting in a suspicious manner. Clearly, a basic criterion would be that
the police officer, with his or her personal knowledge, must observe the facts
leading to the suspicion of an illicit act.

In Manalili v. Court of Appeals,69 the police officers were initially informed about


a place frequented by people abusing drugs. 70 When they arrived, one of the police
officers saw a man with "reddish eyes and [who was] walking in a swaying
manner."71 The suspicion increased when the man avoided the police
officers.72 These observations led the police officers to conclude that the man was
high on drugs.73 These were sufficient facts observed by the police officers "to
stop[the] petitioner [and] investigate."74

In People v. Solayao,75 police officers noticed a man who appeared drunk. 76 This


man was also "wearing a camouflage uniform or a jungle suit." 77 Upon seeing the
police, the man fled.78 His flight added to the suspicion.79 After stopping him, the
police officers found an unlicensed "homemade firearm" 80 in his possession.81 This
court ruled that "[u]nder the circumstances, the government agents could not
possibly have procured a search warrant first." 82 This was also a valid search.

In these cases, the police officers using their senses observed facts that led to the
suspicion. Seeing a man with reddish eyes and walking in a swaying manner, based
on their experience, is indicative of a person who uses dangerous and illicit drugs.
A drunk civilian in guerrilla wear is probably hiding something as well.

The case of Cogaed was different. He was simply a passenger carrying a bag and
traveling aboarda jeepney. There was nothing suspicious, moreover, criminal,
about riding a jeepney or carrying a bag. The assessment of suspicion was not
made by the police officer but by the jeepney driver. It was the driver who
signalled to the police that Cogaed was "suspicious."

This is supported by the testimony of SPO1 Taracatac himself:

COURT:

Q So you don’t know what was the content while it was still being carried by him
in the passenger jeep?

WITNESS:

A Not yet, Your Honor.83

SPO1 Taracatac likewise stated:

COURT:

Q If the driver did not make a gesture pointing to the accused, did you have reason
to believe that the accused were carrying marijuana?
WITNESS:

A No, Your Honor.84

The jeepney driver had to point toCogaed. He would not have been identified by
the police officers otherwise.

It is the police officer who should observe facts that would lead to a reasonable
degree of suspicion of a person. The police officer should not adopt the suspicion
initiated by another person. This is necessary to justify that the person suspected be
stopped and reasonably searched.85 Anything less than this would be an
infringementupon one’s basic right to security of one’s person and effects.

IV

Normally, "stop and frisk" searches do not give the law enforcer an opportunity to
confer with a judge to determine probable cause. In Posadas v. Court of
Appeals,86 one of the earliest cases adopting the "stop and frisk" doctrine in
Philippine jurisprudence, this court approximatedthe suspicious circumstances as
probable cause:

The probable causeis that when the petitioner acted suspiciously and attempted to
flee with the buri bag there was a probable cause that he was concealing something
illegal in the bag and it was the right and duty of the police officers to inspect the
same.87 (Emphasis supplied)

For warrantless searches, probable cause was defined as "a reasonable ground of
suspicionsupported by circumstances sufficiently strong in themselves to warrant a
cautious man to believe that the person accused is guilty of the offense with which
he is charged."88

Malacat v. Court of Appeals89 clarifies the requirement further. It does not have to


be probable cause,but it cannot be mere suspicion. 90 It has to be a "genuine
reason"91 to serve the purposes of the "stop and frisk" exception: 92

Other notable points of Terryare that while probable cause is not required to
conduct a "stop and frisk," it nevertheless holds that mere suspicion or a hunch will
not validate a "stop and frisk." A genuine reason must exist, in light of the police
officer’s experience and surrounding conditions, to warrant the belief that the
person detained has weapons concealed about him. 93 (Emphasis supplied, footnotes
omitted)

In his dissent for Esquillo v. People,94 Justice Bersamin reminds us that police


officers must not rely on a single suspicious circumstance. 95 There should be
"presence of more than oneseemingly innocent activity, which, taken together,
warranted a reasonable inference of criminal activity." 96 The Constitution prohibits
"unreasonable searches and seizures."97 Certainly, reliance on only one suspicious
circumstance or none at all will not result in a reasonable search. 98

There was not a single suspicious circumstance in this case, and there was no
approximation for the probable cause requirement for warrantless arrest. The
person searched was noteven the person mentioned by the informant. The
informant gave the name of Marvin Buya, and the person searched was Victor
Cogaed. Even if it was true that Cogaed responded by saying that he was
transporting the bag to Marvin Buya, this still remained only as one circumstance.
This should not have been enough reason to search Cogaed and his belongings
without a valid search warrant.

Police officers cannot justify unbridled searches and be shielded by this exception,
unless there is compliance with the "genuine reason" requirement and that the
search serves the purpose of protecting the public. As stated in Malacat:

[A] "stop-and-frisk" serves a two-fold interest: (1) the general interest of effective
crime prevention and detection, which underlies the recognition that a police
officer may, under appropriate circumstances and in an appropriate manner,
approach a person for purposes of investigating possible criminal behavior even
without probable cause; and (2) the more pressing interest of safety and self-
preservationwhich permit the police officer to take steps to assure himself that the
person with whom he deals is not armed with a deadly weapon that could
unexpectedly and fatally be used against the police officer. 99 (Emphasis supplied)

The "stop and frisk" searchwas originally limited to outer clothing and for the
purpose of detecting dangerous weapons.100 As in Manalili,101 jurisprudence also
allows "stop and frisk" for cases involving dangerous drugs.

The circumstances of thiscase are analogous to People v. Aruta. 102 In that case, an
informant told the police that a certain "Aling Rosa" would be bringing in drugs
from Baguio City by bus.103 At the bus terminal, the police officers prepared
themselves.104 The informant pointed at a woman crossing the street 105 and
identified her as "Aling Rosa."106 The police apprehended "Aling Rosa," and they
alleged that she allowed them to look inside her bag. 107 The bag contained
marijuana leaves.108

In Aruta, this court found that the search and seizure conducted was
illegal.109 There were no suspicious circumstances that preceded Aruta’s arrest and
the subsequent search and seizure.110 It was only the informant that prompted the
police to apprehend her.111 The evidence obtained was not admissible because of
the illegal search.112 Consequently, Aruta was acquitted.113
Arutais almost identical to this case, except that it was the jeepney driver, not the
police’s informant, who informed the police that Cogaed was "suspicious."

The facts in Arutaare also similar to the facts in People v. Aminnudin. 114 Here, the
National Bureau ofInvestigation (NBI) acted upon a tip, naming Aminnudin as
somebody possessing drugs.115 The NBI waited for the vessel to arrive and
accosted Aminnudin while he was disembarking from a boat. 116 Like in the case at
bar, the NBI inspected Aminnudin’s bag and found bundles of what turnedout to
be marijuana leaves.117 The court declared that the searchand seizure was
illegal.118 Aminnudin was acquitted.119

People v. Chua120 also presents almost the same circumstances. In this case, the
police had been receiving information that the accused was distributing drugs in
"different karaoke bars in Angeles City."121 One night, the police received
information that thisdrug dealer would be dealing drugs at the Thunder Inn Hotel
so they conducted a stakeout.122 A car "arrived and parked"123 at the hotel.124The
informant told the police that the man parked at the hotel was dealing drugs. 125 The
man alighted from his car.126 He was carrying a juice box.127 The police
immediately apprehended him and discovered live ammunition and drugs in his
person and in the juice box he was holding.128

Like in Aruta, this court did not find anything unusual or suspicious about Chua’s
situation when the police apprehended him and ruled that "[t]here was no
valid‘stop-and-frisk’."129

VI

None of the other exceptions to warrantless searches exist to allow the evidence to
be admissible.The facts of this case do not qualify as a search incidental to a lawful
arrest.

Rule 126, Section 13 of the Rules of Court allows for searches incidental to a
lawful arrest. For there to be a lawful arrest, there should be either a warrant of
arrest or a lawful warrantless arrest as enumerated in Rule 113, Section 5 of the
Rules of Court:

Section 5. Arrest without warrant; when lawful. – A peace officer or a private


person may, withouta warrant, arrest a person:

(a) When, in his presence, the person to be arrested has committed, is


actually committing, or is attempting to commit an offense;

(b) When an offense has just been committed and he has probable cause to
believe based on personal knowledge of facts or circumstances that the
person to be arrested has committed it; and
(c) When the person to be arrested is a prisoner who has escaped from a
penal establishment or place where he is serving final judgment or
temporarily confined while his case is pending, or has escaped while being
transferred from one confinement to another.

The apprehension of Cogaed was not effected with a warrant of arrest. None of the
instances enumerated in Rule 113, Section 5 of the Rules of Court were present
whenthe arrest was made. At the time of his apprehension, Cogaed has not
committed, was not committing, or was about to commit a crime. As in People v.
Chua, for a warrantless arrest of in flagrante delictoto be affected, "two elements
must concur: (1) the person to bearrested must execute anovert act indicating that
he has just committed, is actually committing, or is attempting to commit a crime;
and (2) such overt act is done inthe presence or within the view of the arresting
officer."130 Both elements were missing when Cogaed was arrested. 131 There were
no overt acts within plain view of the police officers that suggested that Cogaed
was in possession of drugs at that time.

Also, Cogaed was not an escapee prisoner that time; hence, he could not have
qualified for the last allowable warrantless arrest.

VII

There can be no valid waiver of Cogaed’s constitutional rights even if we assume


that he did not object when the police asked him to open his bags. As this court
previously stated:

Appellant’s silence should not be lightly taken as consent to such search. The
implied acquiescence to the search, if there was any, could not have been more
than mere passive conformity given under intimidating or coercive circumstances
and is thus considered no consent at all within the purview of the constitutional
guarantee.132 (Citations omitted) Cogaed’s silence or lack of aggressive objection
was a natural reaction to a coercive environment brought about by the police
officer’s excessive intrusion into his private space. The prosecution and the police
carry the burden of showing that the waiver of a constitutional right is one which is
knowing, intelligent, and free from any coercion. In all cases, such waivers are not
to be presumed.

The coercive atmosphere created by the presence of the police officer can be
discerned again from the testimony of SPO1 Taracatac during cross-examination:

ATTY. BINWAG:

Q Now, Mr. witness, you claimed that you only asked them what are the contents
of their bags, is it not?
WITNESS:

A Yes, ma’am.

Q And then without hesitation and voluntarily they just opened their bags, is it not?

A Yes, ma’am.

Q So that there was not any order from you for them to open the bags?

A None, ma’am.

Q Now, Mr. witness when you went near them and asked them what were the
contents ofthe bag, you have not seen any signs of hesitation or fright from them, is
it not?

A It seems they were frightened, ma’am.

Q But you actually [claimed] that there was not any hesitation from them in
opening the bags, is it not?

A Yes, ma’am but when I went near them it seems that they were
surprised.133 (Emphasis supplied)

The state of mind of Cogaed was further clarified with SPO1 Taracatac’s responses
to Judge Florendo’s questions:

COURT:

....

Q Did you have eye contact with Cogaed?

A When I [sic] was alighting from the jeepney, Your Honor I observed that he was
somewhat frightened.1âwphi1 He was a little apprehensive and when he was
already stepping down and he put down the bag I asked him, "what’s that," and he
answered, "I don’t know because Marvin only asked me to carry." 134

For a valid waiver by the accused of his or her constitutional right, it is not
sufficient that the police officerintroduce himself or herself, or be known as a
police officer.1âwphi1 The police officer must also inform the person to be
searched that any inaction on his orher part will amount to a waiver of any of his or
her objections that the circumstances do not amount to a reasonable search. The
police officer must communicate this clearly and in a language known to the
person who is about to waive his or her constitutional rights. There must be
anassurance given to the police officer that the accused fully understands his or her
rights. The fundamental nature of a person’s constitutional right to privacy requires
no less.

VIII
The Constitution provides:

Any evidence obtained in violation of [the right against unreasonable searches and
seizures] shall be inadmissible for any purpose in any proceeding. 135

Otherwise known as the exclusionary rule or the fruit of the poisonous tree
doctrine, this constitutional provision originated from Stonehill v. Diokno. 136 This
rule prohibits the issuance of general warrants that encourage law enforcers to go
on fishing expeditions. Evidence obtained through unlawful seizures should be
excluded as evidence because it is "the only practical means of enforcing the
constitutional injunction against unreasonable searches and seizures." 137 It ensures
that the fundamental rights to one’s person, houses, papers, and effects are not
lightly infringed upon and are upheld.

Considering that the prosecution and conviction of Cogaed were founded on the
search of his bags, a pronouncement of the illegality of that search means that there
is no evidence left to convict Cogaed.

Drugs and its illegal traffic are a scourgeto our society. In the fight to eradicate this
menace, law enforcers should be equipped with the resources to be able to perform
their duties better. However, we cannot, in any way, compromise our society’s
fundamental values enshrined in our Constitution. Otherwise, we will be seen as
slowlydismantling the very foundations of the society that we seek to protect.

WHEREFORE, the decisions of the Regional Trial Court, Branch 28, San
Fernando City, La Union and of the Court of Appeals in CA-G.R. CR-HC No.
03394 are hereby REVERSEDand SET ASIDE. For lack of evidence to establish
his guilt beyond reasonable doubt, accused-appellant VICTOR COGAED Y
ROMANA is hereby ACQUITTED and ordered RELEASED from confinement
unless he is being heldfor some other legal grounds. No costs.

SO ORDERED.

MARVIC MARIO VICTOR F. LEONEN

CASES:
People v. Figueroa, G.R No. 134056, July 6, 2000

Accused-appellant ROBERT FIGUEROA (hereafter OBET) appeals from the 18


May 1998 Decision1 of the Regional Trial Court of Parañaque City, Branch 259, in
Criminal Case No. 97-306, convicting him of violation of Section 14-A 2 , Article
III of R.A. No. 6425, otherwise known as the Dangerous Drugs Act of 1972, as
amended by RA. No. 7659. His co-accused Beatrice Valerio (hereafter Betty) was
acquitted.

OBET and Betty were indicted under an information, dated 2 April 1997, whose
accusatory portion reads as follows:

That on 16 February 1997 and for sometime prior thereto in Parañaque City and
within the jurisdiction of this Honorable Court, the above-named accused without
authority of law, conspiring, confederating and helping one another, did then and
there, wilfully, unlawfully and feloniously manufacture, produce, prepare or
process methamphetamine hydrochloride or shabu, a regulated drug amounting to a
2.4 liters, directly by means of chemical synthesis.

CONTRARY TO LAW.3

When arraigned OBET and Betty each entered a plea of not guilty. 4 Trial on the
merits then ensued.

The witnesses presented by the prosecution were NBI Forensic Chemist Mary Ann
T. Aranas, NBI Special Investigator III Pio M. Palencia (hereafter PALENCIA),
and NBI Intelligence Agent II Martin Soriano (hereafter SORIANO).

PALENCIA testified that on 15 February 1997, he was in the office of SORIANO


at Project 6, Quezon City, when they received a call from their informant, a
woman, who reported that a certain OBET was allegedly engaged in large-scale
drug trafficking in Makati City. PALENCIA and SORIANO forthwith instructed
their informant to establish contact with OBET for a buy-bust operation. After
several hours, the informant reported that OBET was already waiting for her at No.
1485 Soliman Street, Makati City, with instructions for her to come alone as soon
as she was ready with P150,000. PALENCIA then caused the dusting of
fluorescent powder over ten pieces of authentic P100 bills as buy-bust money and
gave them to the informant.5

On board a taxi, PALENCIA, SORIANO and their informant proceeded to the


rendezvous area. They arrived at half past twelve o'clock in the early morning of
16 February 1997. As the gate was already open, the informant entered the
premises, while PALENCIA and SORIANO discreetly crawled and positioned
themselves near the gate of the house. Strategically positioned, PALENCIA
overheard OBET ask the informant whether she had the money. PALENCIA then
saw the informant hand over the money to OBET. While counting the money,
OBET sensed the presence of other people in the area. OBET, who was in
possession of a .45 caliber pistol, fired it twice toward the direction of
PALENCIA, while hurrying towards the house. OBET then held hostage his
mistress, Estrella Brilliantes, and her two children for the next three hours until the
arrival of one Major Roberto Reyes to whom OBET surrendered. PALENCIA and
SORIANO brought OBET, his firearm and the recovered buy-bust money to the
WPD Headquarters for recording purposes and, thereafter, to the NBI
Headquarters.6

At the NBI Headquarters, PALENCIA and SORIANO methodically interrogated


OBET about the source of his shabu. OBET eventually volunteered that his source
was a certain Betty of 263 El Grande Street, B.F. Homes, Parañaque City.
PALENCIA and SORIANO took OBET to Betty's house as a follow-up operation.
They arrived at around 6:00 a.m. of the same day, 16 February 1997. As OBET
called Betty earlier to tell her that he was arriving, Betty already had the gate
opened for them. After parking, PALENCIA saw Betty waiting for them. Upon
seeing OBET in handcuffs, Betty asked what happened. OBET replied that he was
just caught in a buy-bust operation. PALENCIA and SORIANO then tried to
convince Betty to surrender the shabu that OBET insisted was hidden inside the
house. As Betty persistently denied the existence of the shabu, PALENCIA told
OBET to confer with Betty. After a while, OBET proceeded to the kitchen of the
guesthouse located outside the main house, followed by Betty. OBET then
promptly pointed to what he termed as liquid shabu inside a white pail along with
other drug paraphernalia, such as a beaker spray. PALENCIA and
SORIANO seized the items.7

Thereafter, PALENCIA requested a laboratory examination of all the seized items


and an ultraviolet light examination over the persons of OBET, Betty and a certain
Eva Baluyot.8 PALENCIA claimed that based on the certification issued by the
Forensic Chemistry Division of the NBI, all the items seized from Betty's residence
were positive for methamphetamine hydrochloride except specimen no.7; while
from among the persons subjected to ultraviolet light examination, only OBET was
found positive for fluorescent powder.9

On cross-examination, PALENCIA admitted that he and SORIANO conducted the


search without a search warrant, but with the consent of Betty. 10 He also admitted
that he did not actually see OBET or Betty in the act of manufacturing shabu. 11

NBI Intelligence Agent II SORIANO corroborated PALENCIA's testimony. He


likewise admitted that the custodial investigation of OBET, during which he
divulged Betty as the source of shabu, was conducted in the absence of any
counsel. SORIANO also confirmed PALENCIA's testimony that they were not
armed with a search warrant, but that they conducted the follow-up operation at
Betty's house under the hot pursuit theory.12 He further maintained that OBET, after
conferring with Betty, uttered, "Ako na nga, ako na nga"(I will do it, I will do it).
OBET then proceeded to the dirty kitchen, pointed to the refrigerator and had it
moved. Thereafter, SORIANO saw a plastic pail containing liquid with floating
brown substances.
SORIANO admitted that he and PALENCIA neither witnessed OBET and Betty
manufacture shabu in the manner described in Section 2(j) of the Dangerous Drugs
Act13 ; nor did they possess evidence, independent of the items they had seized, that
OBET and Betty were engaged in the labeling or manufacturing of shabu. 14

Forensic Chemist Mary Ann T. Aranas testified that on 16 February 1997, she
conducted a laboratory examination for the presence of any prohibited or regulated
drug on eleven different specimens (Exhibits "B"-"L"). 15 The result of the
examination disclosed that all the specimens except specimen no. 7 (Exhibit "H")
were positive for methamphetamine hydrochloride. 16 She further observed that
specimen no. 8 (Exhibit I- I-2), the brown liquid with floating solid flakes
contained in a plastic pail, was positive for epedrine, 17 a substance used in the
manufacture of methamphetamine hydrochloride. She opined that this crude form
of shabu would have to undergo chemical processes, like extraction,
crystallization, distillation, before it could be finally converted into shabu's
crystalline form. She also conducted a fluorescent powder examination over the
persons of OBET and Betty. Only OBET gave a positive result. 18

On the other hand, OBET testified that while he was watching television on the
night of 15 February 1997, he heard the doorbell rang. Upon seeing Eva Baluyot,
his childhood friend, he opened the door for her. Inside the house, Eva handed him
a bundle of money and stated that she was buying shabu from him. OBET
emphatically told Eva that he was not engaged in such illegal trade and returned
the money. OBET then accompanied Eva out of the house. At the garage, OBET
noticed someone peeping from the dark; so he told Eva to go back inside the house
with him. Eva ignored the request. OBET thus left Eva at the garage and got
his .45 caliber gun from his house. While he was locking the door, his handgun
accidentally fired off, as he forgot that it had already been cocked. This blast was
followed by shouts of people outside claiming that they were NBI men. Uncertain,
OBET did not go out of the house but instead told the alleged NBI men to call the
Makati Police, specifically Major Reyes. The NBI agents, however, persisted in
convincing OBET to go out of the house. He did get out of his house after three
hours when he heard the voice of Major Reyes. OBET gave to Major Reyes his
gun. The Makati Police and the NBI men thereafter conducted a joint search inside
OBET's house which, however, yielded nothing. OBET was then brought to the
Makati Police Headquarters where the incident was recorded. Thereafter,
PALENCIA, SORIANO and another NBI man brought OBET to the house of
Betty, his former live-in partner, at El Grande Street, B.F. Homes, Parañaque City,
upon the insistence and information of Eva Baluyot. 19

Upon entering B.F. Homes, SORIANO instructed OBET to call and tell Betty that
he was already near. The gate was already opened when they arrived, and the NBI
men freely parked their car at the garage. Then, PALENCIA and SORIANO
alighted from the car and entered Betty's house. OBET was left in the car under the
charge of the third NBI man; hence, he knew nothing of what happened inside
Betty's house.20

For her part, Betty admitted that she was romantically involved with OBET and
had a child by him. She recalled that on 16 February 1997, OBET called at around
6:00 a.m. and requested her to open the gate for him, as he was already near. She
ran down to the garage and opened the gate. Since her car was parked halfway
through the garage, she went to the main house to get her car keys to make way for
OBET's car. But as she came out of the main house, OBET's car was already
parked inside the garage. She noticed that OBET had two companions with long
firearms. The two, whom Betty later found out as NBI men PALENCIA and
SORIANO, informed her that they had just come from a buy-bust operation and
that OBET had led them to her house, as there were illegal chemicals kept in the
premises. Shocked andamazed, she then asked for a search warrant, but the NBI
men could not produce any.21

Betty further recalled that the NBI men claimed that they found contraband items
near the dirty kitchen at a small space behind the refrigerator where cases of
softdrinks were stored. Betty denied any knowledge that there were illegal
chemicals inside her house and that these were manufactured into shabu. She also
denied knowing Eva Baluyot.22

On cross-examination, Betty disclaimed her alleged consent to the search of her


house, for she specifically asked the NBI men for a search warrant. She asserted
that she did not see the NBI men find the shabu paraphernalia because she went up
to the second floor of her house. She only saw that the NBI men were bringing
several items out of her house.23

The trial court agreed with the prosecution's theory that the warrantless arrests of
OBET and Betty were conducted within the purview of valid warrantless arrests
enumerated in Section 5,24 Rule 113 of the Rules of Court. It then ruled as valid the
consented warrantless search conducted at the house of Betty. Consequently, it
found that the very items seized by the NBI agents at the kitchen of Betty's
guesthouse were admissible as the corpus delicti of the violation of Section 14-A
of the Dangerous Drugs Act. Thus, the trial court "believed" that the paraphernalia
seized were indispensable to the processing or manufacturing of shabu into
crystallized form. Although it conceded that the prosecution witnesses did not
actually see the crystallization processes, the trial court observed that the
Dangerous Drug Act does not require that there be actual manufacturing activities
at the time of the seizure.

The trial court, however, acquitted Betty for failure of the prosecution to adduce
evidence that she, in conspiracy with OBET, manufactured shabu without the
requisite authority. It did not arrive at a similar conclusion as far as OBET was
concerned, but declared that based on the evidence on record, OBET's guilt of the
crime charged was proved beyond reasonable doubt. Thus, in the decision of 18
May 1998 the trial court decreed as follows:

WHEREFORE, finding the evidence insufficient to warrant the conviction of


accused Beatrice Valerio y del Rosario for Violation of Sec. 14-a of Article III of
R.A. 6425 as amended by R.A. 7659, this court pronounces her NOT GUILTY and
considering that she is detained at the NBI the NBI is directed to immediately
release her from custody unless there be some reasons for her detention. Finding,
however, accused Robert Figueroa GUILTY as charged [of] the same offense in
the absence of any mitigating or aggravating circumstances, this Court hereby
sentences him to suffer the penalty of Reclusion Perpetua and to pay a fine of
P500,000.00 and to suffer the accessory penalties provided by law, specifically
Art. VI [sic] of the Revised Penal Code.

The Clerk of Court is directed to prepare the Mittimus for the immediate transfer of
Robert Figueroa to the Bureau of Corrections in Muntinlupa City.

SO ORDERED.

Unsatisfied with the verdict, OBET appealed the decision to us. He principally
premises his prayer for acquittal on the failure of the State to show by convincing
evidence that shortly prior to or during custodial investigation, he was apprised of
his constitutional rights to remain silent, to have a competent and independent
counsel preferably of his own choice, and to be informed of such rights. He asserts
that he did not waive those rights. Thus, whatever admissions were allegedly
extracted from him are inadmissible in evidence. Even assuming that his
extrajudicial statements were admissible, Betty's acquittal would work in his favor
because the indictment is based on conspiracy. In a conspiracy, the act of one is the
act of all. Therefore, the acts imputed to him were also the acts of Betty, and vice
versa. Since the trial court considered insufficient for conviction the acts of Betty,
then he, too, should be acquitted.

In the Appellee's Brief, the Office of the Solicitor General (OSG) maintains that
not all warrantless searches and seizures are illegal. For one, a warrantless search
and seizure is not unreasonable and offensive to the Constitution if consent is
shown. In this case, the prosecution convincingly proved that Betty consented to
the search of her house. With her consent, Betty validly waived her constitutional
right against unreasonable searches and seizure. Consequently, the items seized in
her house by virtue of the consented search are admissible in evidence against her
and OBET.

The OSG also contends that the acquittal of Betty does not per se work to absolve
OBET of the crime charged. Betty's believable disavowal of the location of the
paraphernalia and other circumstances on record reasonably indicative of her
innocence cannot redound in favor of OBET. The latter apparently knew the exact
location of the hidden paraphernalia. By such disclosure, it is not far-fetched to
conclude that OBET had been actually engaged in the manufacture of shabu.

We first resolve the question of whether Betty's acquittal would benefit OBET.

We disagree with the theory of OBET that in an indictment based on conspiracy,


the acquittal of a conspirator likewise absolves a co-conspirator from criminal
liability. Indeed, the rule is well-settled that once a conspiracy is established, the
act of one is the act of all, and each of the conspirators is liable for the crimes
committed by the other conspirators.25 It follows then that if the prosecution fails to
prove conspiracy, the alleged conspirators should be held individually responsible
for their own respective acts. Accordingly, OBET's criminal liability in this case
must be judged on the basis of his own acts as established by the quantum of proof
required in criminal cases.

We should then determine whether the prosecution was able to establish beyond
reasonable doubt OBET's guilt for unauthorized manufacture of shabu, a regulated
drug.

After a meticulous review of the records and of the evidence adduced by the parties
in this case, we find that what PALENCIA and SORIANO did left much to be
desired, thereby resulting in a bungled prosecution of the case. The evidence for
the prosecution miserably failed to prove OBET's guilt of the offense charged.

The buy-bust operation was a failure because no shabu or other regulated or


prohibited drug was found in OBET's person and residence. No evidence was
adduced to show that OBET handed shabu over to the informant. Yet, he was
placed in custody. For what offense he was held in custody does not, initially,
appear very clear on the record.

It was established that OBET fired two shots toward the direction of PALENCIA
and SORIANO and held hostage his mistress and her two children. Yet he was not
placed under custodial investigation for such crimes as grave threats, coercion,
illegal possession of firearms, or crimes other than that with which he was charged.

On the contrary, OBET was held in custody and investigated or interrogated about
the source of the shabu, none of which was found during the buy-bust operation. In
short he was held in custody as a consequence of the failed buy-bust operation and
as a follow-up to link him to the source and establish a conspiracy in the illegal
trade of shabu. Allegedly, he admitted that the source was Betty. On the basis of
that admission, PALENCIA and SORIANO, together with OBET, proceeded to the
residence of Betty. Needless to state, OBET cannot be investigated for anything in
relation to shabu while under custody without informing him of his rights to
remain silent and to have a competent and independent counsel preferably of his
own choice. Any waiver of such rights should be in writing and made in the
presence of a counsel pursuant to Section 12 (1) 26 , Article III of the Constitution. It
has been held that these rights attach from the moment the investigation
starts, i.e. when the investigating officers begin to ask questions to elicit
information and confessions or admissions from the suspect. 27

It is always incumbent upon the prosecution to prove at the trial that prior to in-
custody questioning, the confessant was informed of his constitutional rights. The
presumption of regularity of official acts does not prevail over the constitutional
presumption of innocence.28 Hence, in the absence of proof that the arresting
officers complied with these constitutional safeguards, extrajudicial statements,
whether inculpatory or exculpatory, made during custodial investigation are
inadmissible and cannot be considered in the adjudication of a case. 29 In other
words, confessions and admissions in violation of Section 12 (1), Article III of
the Constitution are inadmissible in evidence against the declarant and more so
against third persons.30 This is so even if such statements are gospel truth and
voluntarily given.31 Such statements are useless except as evidence against the very
police authorities who violated the suspect's rights. 32

SORIANO admitted that the custodial investigation of OBET was conducted


without the presence of a lawyer, and there is no proof that OBET waived said
right and the right to remain silent. No waiver in writing and in the presence of a
counsel was presented. Thus, pursuant to paragraph 3 of Section 12 of Article III of
the Constitution any admission obtained from OBET in the course of his custodial
investigation was inadmissible against him and cannot be used as a justification for
the search without a warrant.

The search conducted on Betty's house was allegedly consented to by Betty.


Indeed, a consented search is one of the exceptions to the requirement of a search
warrant. In People v. Chua Ho San @ Tsay Ho San,33 we pointed out that:

This interdiction against warrantless searches and seizures, however, is not


absolute and such warrantless searches and seizures have long been deemed
permissible by jurisprudence in instances of (1) search of moving vehicles, (2)
seizure in plain view, (3) customs searches, (4) waiver or consented searches, (5)
stop and frisk situations (Terry search), and (6) search incidental to a lawful arrest.
The last includes a valid warrantless search and seizure pursuant to an equally
valid warrantless arrest, for, while as a rule, an arrest is considered legitimate if
effected with a valid warrant of arrest, the Rules of Court recognize permissible
warrantless arrest, to wit: (1) arrest flagrante delicto, (2) arrest effected in hot
pursuit, and (3) arrest of escaped prisoners.

In case of consented searches or waiver of the constitutional guarantee, against


obtrusive searches, it is fundamental that to constitute, a waiver, it must first appear
that (1) the right exists; (2) that the person involved had knowledge, either actual
or constructive, of the existence of such right; and (3) the said person had an actual
intention to relinquish the right.34 The third condition does not exist in the instant
case. The fact is, Betty asked for a search warrant, thus:

Q And of course, these NBI Special Investigators informed you of their purpose is
that correct?

A Yes sir.

Q And of course believing that there was nothing in your house you acceded?

A No sir, I was asking for a search warrant.

Q And what was their reply?

A They did not have any but that Figueroa had led them to the property. 35

Neither can the search be appreciated as a search incidental to a valid warrantless


arrest of either Betty or OBET as intimated by the trial court. First, Betty's arrest
did not precede the search. Second, per the prosecution's evidence OBET was not
arrested for possession or sale of regulated or prohibited drugs as a consequence of
the buy-bust operation. He surrendered after taking hostage Estrella and her two
children, although he was thereafter held in custody for further questioning on
illegal drugs.

There is no showing that the house occupied by Betty and the articles confiscated
therefrom belong to OBET. That OBET pointed to PALENCIA and SORIANO the
places where the articles were found provides no sufficient basis for a conclusion
that they belonged to him. Even if the articles thus seized actually belonged to him,
they cannot be constitutionally and legally used against him to establish his
criminal liability therefor, since the seizure was the fruit of an invalid custodial
investigation.

WHEREFORE, in view of all the foregoing, the 18 May 1998 Decision of the
Regional Trial Court, Branch 259, Parañaque City, convicting herein accused-
appellant Robert Figueroa of violation of Section 14-A, Article III of the
Dangerous Drugs Act, as amended, is hereby REVERSED and SET ASIDE. He is
hereby ACQUITTED of the crime charged, and ORDERED immediately released
from confinement or detention unless his continued detention is warranted by
virtue of a valid legal cause. The Director of the Bureau of Corrections is directed
to submit within five (5) days from receipt of a copy of this decision a report on the
release of accused-appellant.

Costs de oficio.

SO ORDERED.
People v. Lacerna, G.R. No. 109250, September 5, 1997
The unrelenting and pervading war against illegal drugs has absorbed the attention
of all branches of government, both national and local, as well as media, parents,
educators, churches and the public at large. This case is one more intrepid battle in
such all-out war. Herein appellant seeks acquittal on the ground that his acts did
not constitute the crime of "giving away prohibited drugs" penalized by Section 4
of Republic Act No. 6425, as amended (The Dangerous Drugs Act). Nonetheless,
he cannot escape the law because the very same deeds, which appellant admits to
have performed, show his culpability for "illegal possession of prohibited drugs"
— penalized in Section 8 of R.A. 6425, as amended — which is necessarily
included in the crime charged in the information.chanrobles

Statement of the Case

This ruling is explained by the Court as it resolves this appeal from the Decision, 1
dated February 24, 1993, of the Regional Trial Court of Manila, Branch 16, 2
convicting Appellant Marlon Lacerna y Aranador "of violation of Section 4 of
Republic Act No. 6425, as amended . . ."cralaw virtua1aw library

Asst. City Prosecutor of Manila Juan O. Bermejo, Jr. charged appellant and Noriel
Lacerna in an Information, 3 dated September 16, 1992, which reads as follows: 4

"The undersigned accuses NORIEL LACERNA Y CORDERO and MARLON


LACERNA Y ARANADOR of a violation of Section 4 Art. II, in relation to
Section 21, Art. IV of Republic Act 6425, as amended by Presidential Decree No.
1675, . . .

"That on or about September 12, 1992, in the City of Manila, Philippines, the said
accused, not being authorized by law to sell, deliver or give away to another or
distribute any prohibited drug, did then and there wilfully, unlawfully and jointly
sell, deliver or give away to another the following, to wit:chanrob1es virtual 1aw
library

Eighteen (18) blocks of marijuana

flowering tops - weight — 18.235 kilograms

which is a prohibited drug."cralaw virtua1aw library

When the case was called for arraignment on October 7, 1992, appellant and his
co-accused appeared without counsel but they alleged that they had engaged the
services of a certain Atty. Kangleon. Thus, the trial court provisionally appointed
Atty. Rodolfo P. Libatique of the Public Attorney’s Office as counsel de oficio, in
case Atty. Kangleon did not appear for the arraignment on October 28, 1992. 5
Because the alleged counsel de parte failed to show up during the arraignment on
that date, Atty. Libatique assisted the accused who pleaded "not guilty." 6

After trial on the merits, the court a quo promulgated the assailed Decision, the
dispositive portion of which reads: 7

"WHEREFORE, PREMISES CONSIDERED, judgment is hereby


rendered:chanrob1es virtual 1aw library

I. The guilt of the accused Marlon Lacerna having been established beyond
reasonable doubt for the crime of violation of Section 4 of RA 6425, as amended,
he is found guilty of the same, sentencing him to life imprisonment and to pay a
fine of P20,000. With costs.

II. The guilt for the crime charged of accused Noriel Lacerna not having been
established beyond reasonable doubt he is hereby ACQUITTED. The warden of
the Manila City Jail is ordered to release his person, unless held on other charges.

The evidence seized in this case is to remain in the custody of the NBI Director as
Drugs Custodian of the Dangerous Drugs Board. (RA 425, Sec. 36; Supreme Court
Circular No. 9 dated July 18, 1973) to be properly disposed of after the final
disposition of this case."cralaw virtua1aw library

Hence, only Marlon Lacerna (his co-accused having been acquitted) interposed this
appeal direct to the Supreme Court in view of the life penalty imposed. 8

The Facts

Version of the Prosecution

The prosecution presented the following witnesses: PO3 Carlito P. Valenzuela,


Forensic Chemist Aida A. Pascual, and PO3 Rafael Melencio. Their testimonies
are summarized by the Solicitor General in the Appellee’s Brief as follows: 9

"On September 12, 1992, Police Officer 3 (PO3) Carlito P. Valenzuela, a member
of the Mobile Patrol Division of the Western Police District (WPD), was assigned
to man the checkpoint and patrol the area somewhere along the side streets of
Radial Road near Moriones Street. The assignment to monitor strategic places in
the city and barangays of Manila was a direct order from General Nazareno. Thus,
he and his companion PO3 Angelito Camero went about cruising the area in their
Mobile Patrol car, with PO3 Valenzuela at the helm. At about 2:00 p.m., appellant
and co-accused, who were aboard a taxicab, passed by PO3 Valenzuela’s place of
assignment, which was then heavy with traffic, looking suspicious (t.s.n., PO3
Valenzuela, Nov. 11, 1992, pp. 3-4; Nov. 20, 1992, pp. 2-7).

Appellant was seated beside the taxi driver while co-accused was seated at the left
back seat of the taxi. When PO3 Valenzuela looked at the occupants of said taxi,
the latter bowed their heads and slouched, refusing to look at him. Feeling that
something was amiss, PO3 Valenzuela and his companion stopped the vehicle,
signalling the driver to park by the side of the road (t.s.n., PO3 Valenzuela, Nov.
11, 1992, pp. 3-4).

PO3 Valenzuela and his co-police officer asked permission to search the vehicle.
As the occupants readily agreed, the police officers went about searching the
luggage in the vehicle which consisted of a knapsack and a dark blue plastic
grocery bag. They asked appellant what the contents of the plastic bag were. Co-
accused Noriel Lacerna immediately answered that the bag contained his vomit
(t.s.n., PO3 Valenzuela, Nov. 11, 1992, pp. 4-5).

Skeptical of what appellant and co-accused disclosed as regards the contents of the
plastic bag, PO3 Valenzuela made a hole in the bag and peeped inside. He found
several blocks wrapped in newspaper, with the distinct smell of marijuana
emanating from it. PO3 Valenzuela opened one of the boxes and saw dried
marijuana leaves. He told appellant and co-accused that the contents of the bag
were marijuana, which co-accused readily affirmed. According to both Lacernas,
the bag was a ‘padala’ of their uncle. Specifically, they claimed that the bag was
sent by their uncle, who happened to be in Baguio City, for shipment to Iloilo
(t.s.n., PO3 Valenzuela, Nov. 11, 1992, pp. 5-7; Nov. 20, 1992, pp. 8-10).

Appellant and co-accused, and the plastic bag containing blocks of marijuana were
brought by PO3 Valenzuela and PO3 Camero to the WPD Headquarters on UN
Avenue, Manila. 10 At about 9:00 p.m. of the same day, both appellant and co-
accused were turned over to PO3 Rafael Melencio for investigation while the
blocks were turned over to Lt. de Soto (tsn., PO3 Melencio, Dec. 11, 1992, pp. 3-5.
20).

Lt. de Soto counted the blocks of marijuana, numbering eighteen (18) in all. Each
block was wrapped in newspaper. After seeing what the contents of the blocks
were, the specimens (Exhs.’B’ to ‘B-19’) were brought to the National Bureau of
Investigation (NBI) for further examination. 11 On the other hand, PO3 Melencio
investigated appellant and co-accused, informing them of their constitutional rights
during a custodial investigation. Thereafter, he prepared the Affidavit of
Apprehension and the Booking Sheet and Arrest Report (Exhs.’A’, ‘G’, List of
Exhibits, pp. 1, 15; tsn., PO3 Melencio, Dec. 11, 1992, pp. 15-24).

NBI Forensic Chemist Aida A. Pascual examined the eighteen (18) confiscated
blocks which tested positive of containing marijuana (Exhs.’C’, ‘F’ to ‘F-9’. List
of Exhibits, pp. 2-14; tsn., A. Pascual, Dec. 2, 1992, pp. 2-5)."cralaw virtua1aw
library

Version of the Defense

Appellant sets up the defense of denial, alleging that the blue plastic bag was
owned by his uncle who requested him to bring it to Iloilo. He also denied
knowing that it contained marijuana. In his Brief prepared by the Public Attorney’s
Office, he narrated his version of the factual circumstances of this case, as follows:
12

"On September 12, 1992, at about 2:00 P.M., Accused Marlon and Noriel Lacerna
were riding in a taxicab on their way to (the) North Harbor to board a boat bound
for Iloilo City. While plying along Pier 15 their taxicab was flagged down by a
patrol mobile car. Accused Marlon Lacerna (appellant herein) was sitting in front
while accused Noriel Lacerna was at the back of the taxicab. The accused carried
two bags. One bag contained their personal belongings and the other bag contained
things which their uncle Edwin Lacerna asked them to bring along. When their
taxicab was stopped, the two policemen in the Mobile car requested them that they
and their baggage be searched. Confident that they have not done anything wrong,
they allowed to be searched. During the (search), the two accused were not allowed
to alight from the taxicab. The knapsack bag which contained their clothes was
first examined in front of them. The second bag was taken out from the taxi and
was checked at the back of the taxicab. The accused were not able to see the
checking when the policemen brought the plastic bag at the back of the taxi. After
checking, the policemen told them its ‘positive’. The accused were (asked) to
alight and go to the patrol car. They were brought to the WPD Headquarters at
United Nations. While there, they were brought inside a room. They asked what
wrong they have done but the policemen told them to wait for Major Rival. At
about 8:00 o’clock P.M., Major Rival talked to them and asked them where the
baggage came from and they answered that it was given to them by their uncle.
Then Major Rival asked them to hold the marijuana and pictures were taken. Later,
they were brought inside the cell where they were maltreated by the ‘Kabo’. The
‘Kabo’ forced them to admit ownership of the marijuana. Noriel was boxed on the
chest, blindfolded and a plastic (bag) was placed on his neck and was strangled.
The mauling took place for about 30 minutes inside the toilet. They refused to sign
the Booking and Arrest Report but they impressed their fingerprint on a white bond
paper. They were brought by Melencio to the Inquest Prosecutor at the City Hall.
On the way to the Inquest Prosecutor. Melencio told them to admit the charge
against them before the Inquest Fiscal, because if they will deny, something
(would happen) to them in the afternoon and Melencio even uttered to them ‘vulva
of your mother.’ Because they were apprehensive and afraid, they admitted the
charge before the Inquest Fiscal.
(Appellant) Marlon Lacerna first met his uncle Edwin Lacerna at Muñoz Market.
The second time was on September 11, 1992, when his uncle went to his brother’s
house in Caloocan City and requested him to bring his (uncle) personal belongings
upon learning that he (Marlon) is leaving for Iloilo City the next day, September
12, 1992. He told his uncle to bring his personal belongings either in the evening of
that day or the following day at the (Grand) Central (Station), Monumento because
he was going to buy a ticket for Noriel as he intend (sic) to bring the latter with
him in going home to the province. His uncle already gave a ticket for him. When
he and Noriel (arrived) at the Grand Central at about 10:00 o’clock A.M. on
September 12, 1992, their uncle was already there. The latter placed the plastic bag
besides their baggage. They no longer inspected the contents of the bag as the same
was twisted and knotted on top. After getting a ticket from the office of Don
Sulpicio Lines, Marlon told Noriel to hail a taxi and then they proceeded to the
pier.

(Appellant’s) purpose in going home to Iloilo was to get all the requirements
needed in his application to enter the Marines.

Accused Noriel just arrived in Manila three days before September 12, 1992 to
look for a job and was staying with (appellant) at Caloocan City. In the evening of
September 11, 1992, (appellant) requested him to come . . . with him to Iloilo and
assured him that he (would) be the one to pay for (Noriel’s) fare. (TSN., January 6,
1993, pp. 3-23; January 8, 1993, pp. 2-12; January 11, 1993, pp. 2-18; January 20,
1992, pp. 2-6; January 22, 1993, pp. 2-14)"

Ruling of the Trial Court

The court a quo observed that appellant could not be convicted of "delivering"
prohibited drugs because the Information did not allege that he knowingly
delivered marijuana. Neither could he be convicted of "transporting or dispatching
in transit" such prohibited drugs because these acts were not alleged in the
Information. The trial court mused further that appellant could not be convicted of
"selling" marijuana because the elements constituting this crime were not proven.
However, the Information charged appellant with "giving away to another"
prohibited drugs, a charge which was different from "delivery" defined under
Section 2 (f) 13 of R.A. 6245, as amended. Citing People v. Lo Ho Wing, 14 the
trial court ruled that "giving away" to another is akin to "transporting" prohibited
drugs, a malum prohibitum established by the mere commission of said act. Thus,
the court a quo convicted appellant of "giving away" marijuana to another on the
following premise: 15

"It is not denied by (appellant) that he did give to his co-accused cousin Noriel
Lacerna the bundled 18 blocks of marijuana who thereupon seated himself at the
rear of the taxi with the marijuana. His claim that he did not know the contents of
the blue plastic bag can hardly be believed because it is within judicial notice that
the marijuana contents readily emits a pungent odor so characteristic of marijuana
as what happened when the 18 blocks were displayed in open Court. But as stated,
guilty knowledge is not required by the phrase ‘GIVE AWAY TO ANOTHER’
(Sec. 4). It was clearly established that he gave the stuff to another, that is, to his
co-accused Noriel Lacerna. The law does not distinguish as to whether the word
‘another’ refers to a third person other than a co-accused or to a co-accused. The
information, as in the case at bar, need not allege guilty knowledge on the part of
Marlon Lacerna in ‘giving away’ to another the marijuana. (Appellant) should,
therefor be found culpable for violating Section 4 of RA 6425, as amended, as
charged for ‘giving away to another’ the marijuana."cralaw virtua1aw library

Accused Noriel Lacerna, on the other hand, was acquitted for insufficiency of
evidence. The court a quo reasoned that "it cannot be said that he did ‘give away to
another’ the marijuana for it was (appellant) who gave the marijuana to (Noriel)."
Besides, unlike appellant who was urbanized in mannerism and speech, Noriel
Lacerna manifested probinsyano traits and was, thus, unlikely to have dealt in
prohibited drugs.

The Issues

Appellant objects to the trial court’s Decision and assigns the following errors: 16

"I

The lower court erred in making a sweeping statement that the act of ‘giving away
to another(’) is not defined under R.A. 6425 specifically requiring knowledge what
intent one (sic) is passing is a dangerous drug, as contradistinguished from the term
‘deliver’; where knowledge is required.

II

The lower court erred in not giving credence to the assertion of accused-appellant
that he had no knowledge that what were inside the plastic bag given to him by his
uncle were marijuana leaves.

III

The trial court erred in convicting accused-appellant despite failure of the


prosecution to prove his guilt beyond reasonable doubt."cralaw virtua1aw library

The Court’s Ruling

After meticulously reviewing the records of the case and taking into account the
alleged errors cited above and the argument adduced in support thereof, the Court
believes that the issues can be restated as follows: (1) Was appellant’s right against
warrantless arrest and seizure violated? (2) Was the trial court correct in convicting
appellant for "giving away to another" 18 blocks of marijuana? and (3) May the
appellant be held guilty of "illegal possession" of prohibited drugs? The Court
answers the first two questions in the negative and the third in the
affirmative.chanrobles virtuallawlibrary

First Issue: Appellant’s Right Against

Warrantless Search and Seizure

The defense argues that the bricks of marijuana were inadmissible in evidence as
they were obtained through illegal search and seizure. Appellant alleges that at the
time of the search and seizure, he and his co-accused were not committing any
crime as they were merely riding a taxicab on the way to Pier 15, North Harbor in
Manila. Hence, the precipitate arrest and seizure violated their constitutional right
and the marijuana seized constituted "fruits of the poisonous tree."cralaw
virtua1aw library

The Solicitor General disagrees, contending that the search and seizure were
consistent with recent jurisprudential trend liberalizing warrantless search and
seizure where the culprits are riding moving vehicles, because a warrant cannot be
secured in time to apprehend the mobile target.

Both contentions are inaccurate. In the recent case of People v. Cuison, 17 this
Court reiterated the principles governing arrest, search and seizure. To summarize,
let us begin with Section 2, Article III of the 1987 Constitution which
provides:jgc:chanrobles.com.ph

"SEC. 2. The right of the people to be secure in their persons, houses, papers, and
effects against unreasonable searches and seizures of whatever nature and for any
purpose shall be inviolable, and no search warrant or warrant of arrest shall issue
except upon probable cause to be determined personally by the judge after
examination under oath or affirmation of the complainant and the witnesses he
may produce, and particularly describing the place to be searched and the persons
or things to be seized."cralaw virtua1aw library
The Constitution further decrees that any evidence obtained in violation of the
provision mentioned is inadmissible in evidence:jgc:chanrobles.com.ph

"SEC. 3. . . .

(2) Any evidence obtained in violation of . . . the preceding section shall be


inadmissible for any purpose in any proceeding."cralaw virtua1aw library

However, not being absolute, this right is subject to legal and judicial exceptions.
The Rules of Court, Section 12 of Rule 126, provides that a person lawfully
arrested may be searched for "dangerous weapons or anything which may be used
as proof of the commission of an offense, without a search warrant."cralaw
virtua1aw library

Five generally accepted exceptions to the rule against warrantless arrest have also
been judicially formulated as follows: (1) search incidental to a lawful arrest, (2)
search of moving vehicles, (3) seizure in plain view, (4) customs searches, and (5)
waiver by the accused themselves of their right against unreasonable search and
seizure. 18 Search and seizure relevant to moving vehicles are allowed in
recognition of the impracticability of securing a warrant under said circumstances.
In such cases, however, the search and seizure may be made only upon probable
cause, i.e., upon a belief, reasonably arising out of circumstances known to the
seizing officer, that an automobile or other vehicle contains an item, article or
object which by law is subject to seizure and destruction. 19 Military or police
checkpoints have also been declared to be not illegal per se as long as the vehicle is
neither searched nor its occupants subjected to body search, and the inspection of
the vehicle is merely visual. 20

In the case at bar, the taxicab occupied by appellant was validly stopped at the
police checkpoint by PO3 Valenzuela. It should be stressed as a caveat that the
search which is normally permissible in this instance is limited to routine checks
— visual inspection or flashing a light inside the car, without the occupants being
subjected to physical or body searches. A search of the luggage inside the vehicle
would require the existence of probable cause. 21

In applicable earlier Decisions, this Court held that there was probable cause in the
following instances: (a) where the distinctive odor of marijuana emanated from the
plastic bag carried by the accused; 22 (b) where an informer positively identified
the accused who was observed to have been acting suspiciously; 23 (c) where the
accused fled when accosted by policemen; 24 (d) where the accused who were
riding a jeepney were stopped and searched by policemen who had earlier received
confidential reports that said accused would transport a large quantity of
marijuana; 25 and (e) where the moving vehicle was stopped and searched on the
basis of intelligence information and clandestine reports by a deep penetration
agent or spy — one who participated in the drug smuggling activities of the
syndicate to which the accused belonged — that said accused were bringing
prohibited drugs into the country. 26

In the case at hand, however, probable cause is not evident. First, the radio
communication from General Nazareno, which the arresting officers received and
which they were implementing at that time, concerned possible cases of robbery
and holdups in their area. 27 Second, Noriel Lacerna’s suspicious reactions of
hiding his face and slouching in his seat when PO3 Valenzuela’s car passed
alongside the taxicab might have annoyed the latter, or any other law enforcer, and
might have caused him to suspect that something was amiss. But these bare acts do
not constitute probable cause to justify the search and seizure of appellant’s person
and baggage. Furthermore, the Claudio ruling cannot be applied to this case
because the marijuana was securely packed inside an airtight plastic bag and no
evidence, e.g., a distinctive marijuana odor, was offered by the prosecution.

Nonetheless, we hold that appellant and his baggage were validly searched, not
because he was caught in flagrante delicto, but because he freely consented to the
search. True, appellant and his companion were stopped by PO3 Valenzuela on
mere suspicion — not probable cause — that they were engaged in a felonious
enterprise. But Valenzuela expressly sought appellant’s permission for the search.
Only after appellant agreed to have his person and baggage checked did the actual
search commence. It was his consent which validated the search, waiver being a
generally recognized exception to the rule against warrantless search. 28

We are aware that this Court in Aniag, Jr. v. COMELEC outlawed a search based
on an implied acquiescence, because such acquiescence was not consent within the
purview of the constitutional guaranty, but was merely passive conformity to the
search given under intimidating and coercive circumstances. 29 In the case before
us, however, appellant himself who was "urbanized in mannerism and speech"
expressly said that he was consenting to the search as he allegedly had nothing to
hide and had done nothing wrong. 30 In his brief, appellant explicitly, even if
awkwardly, reiterated this: "Confident that they [the accused] have not done
anything wrong, they allowed to be searched." This declaration of appellant is a
confirmation of his intelligent and voluntary acquiescence to the search. The
marijuana bricks were, therefore, obtained legally through a valid search and
seizure. They were admissible in evidence; there was no poisonous tree to speak
of.

Second Issue: Did Appellant

"Give Away" the Prohibited Drug?

The trial court justified the conviction of appellant for "giving away to another" the
prohibited drugs, because he literally handed to Noriel the plastic bag containing
marijuana, manually transferring the plastic bag from the front seat to the backseat
of the taxicab. We hold, however, that this is not the act penalized by the
Dangerous Drugs Act of 1972.

Section 4 of R.A. 6425, as amended, the violation of which is charged in the


Information, penalizes "any person who, unless authorized by law, shall sell,
administer, deliver, give away to another, distribute, dispatch in transit or transport
any prohibited drug, or shall act as a broker in any of such transactions."cralaw
virtua1aw library

The phrase "give away" is commonly defined as "to make a present of; to donate,
or to make a sacrifice." 31 As used in a statute making it an offense to "sell, give
away, or otherwise dispose of" liquor without a license, this phrase was construed
as extending only to a disposition in ejusdem generis with a sale or a gift. 32 It is
synonymous with "to furnish," a broad term embracing the acts of selling and
giving away with the intent of transferring ownership. Selling by itself is one
distinct mode of committing the offense, and furnishing is intended only to include
other modes of affording something to others besides selling it. 33

As distinguished from "delivery," which is an incident of sale, "giving away" is a


disposition other than a sale. It is, therefore, an act short of a sale which involves
no consideration. The prohibited drug becomes an item or merchandise presented
as a gift or premium (giveaway), where ownership is transferred.

According to appellant, he gave the plastic bag and the knapsack to Noriel because
the latter got into the taxicab first and because there was more room in the backseat
than in the front. By handing the plastic bag to Noriel, appellant cannot be
punished for giving away marijuana as a gift or premium to another. In Cuison, 34
this Court acquitted an accused of carrying and transporting prohibited drugs
because the act per se of handing over a baggage at the airport cannot in any way
be considered criminal.

Further, adopting the trial court’s interpretation would lead to absurd conclusions.
Following the trial court’s line of reasoning, Noriel should have been held liable
for the same crime when he gave the plastic bag to PO3 Valenzuela for the latter’s
inspection. And yet, the trial court inexplicably acquitted him. Valenzuela would
similarly be criminally culpable as he testified that he turned over the plastic bag to
his superior, Lt. de Soto. It is a well-settled rule that statutes should receive a
sensible construction so as to give effect to the legislative intention and to avoid an
unjust or an absurd conclusion. 35

Third Issue:chanrob1es virtual 1aw library


May Appellant Be Convicted of Illegal Possession?

Appellant’s exoneration from giving away a prohibited drug to another under


Section 4 of the Dangerous Drugs Act does not, however, spell freedom from all
criminal liability. A conviction for illegal possession of prohibited drugs,
punishable under Section 8 of the same Act, is clearly evident.

In People v. Tabar, 36 the Court convicted appellant of illegal possession under


Section 8 of said Act, although he was charged with "selling" marijuana under
Section 4, Article II thereof. 37

The prevailing doctrine is that possession of marijuana is absorbed in the sale


thereof, except where the seller is further apprehended in possession of another
quantity of the prohibited drugs not covered by or included in the sale and which
are probably intended for some future dealings or use by the seller. 38

Possession is a necessary element in a prosecution for illegal sale of prohibited


drugs. It is indispensable that the prohibited drug subject of the sale be identified
and presented in court. 39 That the corpus delicti of illegal sale could not be
established without a showing that the accused possessed, sold and delivered a
prohibited drug clearly indicates that possession is an element of the former. The
same rule is applicable in cases of delivery of prohibited drugs and giving them
away to another.

In People v. Manzano, 40 the Court identified the elements of illegal sale of


prohibited drugs, as follows (1) the accused sold and delivered a prohibited drug to
another, and (2) he knew that what he had sold and delivered was a dangerous
drug. Although it did not expressly state it, the Court stressed delivery, which
implies prior possession of the prohibited drugs. Sale of a prohibited drug can
never be proven without seizure and identification of the prohibited drug, affirming
that possession is a condition sine qua non.

It being established that illegal possession is an element of and is necessarily


included in the illegal sale of prohibited drugs, the Court will thus determine
appellant’s culpability under Section 8.

From the penal provision under consideration and from the cases adjudicated, the
elements of illegal possession of prohibited drugs are as follows: (a) the accused is
in possession of an item or object which is identified to be a prohibited drug; (b)
such possession is not authorized by law; and (c) the accused freely and
consciously possessed the prohibited drug. 41

The evidence on record established beyond any doubt that appellant was in
possession of the plastic bag, containing prohibited drugs without the requisite
authority. The NBI forensic chemist’s identification of the marijuana or Indian
hemp was conclusive.

Appellant protests the trial court’s finding that he knew that the plastic bag
contained marijuana. The lower court ruled that appellant could not have possibly
missed the pervasive pungent smell emitted by marijuana which was duly noted
when the marijuana was exhibited in open court. This reasoning, however, is not
supported by the evidence; the plastic bag, at the time of the search and seizure,
was "twisted and tied at the top," and thus airtight. PO3 Valenzuela did not even
notice this pervasive characteristic smell until he poked a hole in the plastic bag
and unwrapped the newspaper covering one of the marijuana bricks.

It is well-settled that criminal intent need not be proved in the prosecution of acts
mala prohibita. On grounds of public policy and compelled by necessity, courts
have always recognized the power of the legislature, as "the greater master of
things," to forbid certain acts in a limited class of cases and to make their
commission criminal without regard to the intent of the doer. 42 Such legislative
enactments are based on the experience that repressive measures which depend for
their efficiency upon proof of the dealer’s knowledge or of his intent are of little
use and rarely accomplish their purposes; besides, the prohibited act is so injurious
to the public welfare that, regardless of the person’s intent, it is the crime itself. 43

This, however, does not lessen the prosecution’s burden because it is still required
to show that the prohibited act was intentional. 44 Intent to commit the crime and
intent to perpetrate the act must be distinguished. A person may not have
consciously intended to commit a crime; but if he did intend to commit an act, and
that act is, by the very nature of things, the crime itself, then he can be held liable
for the malum prohibitum. 45 Intent to commit the crime is not necessary, but
intent to perpetrate the act prohibited by the special law must be shown. In Bayona,
the Court declared: 46

". . . The law which the defendant violated is a statutory provision, and the intent
with which he violated it is immaterial. . . . The act prohibited by the Election Law
was complete. The intention to intimidate the voters or to interfere otherwise with
the election is not made an essential element of the offense. Unless such an
offender actually makes use of his revolver, it would be extremely difficult, if not
impossible, to prove that he intended to intimidate the voters.

The rule is that in acts mala in se there must be a criminal intent, but in those mala
prohibita it is sufficient if the prohibited act was intentionally done.’Care must be
exercised in distinguishing the difference between the intent to commit the crime
and the intent to perpetrate the act. . .’ (U.S. v. Go Chico, 14 Phil., 128)."cralaw
virtua1aw library
In illegal possession of prohibited drugs under Section 8 of the Dangerous Drugs
Act, the prosecution is not excused from proving that possession of the prohibited
act was done "freely and consciously," which is an essential element of the crime.

In the case at bar, appellant was found to have in his possession a plastic bag
containing 18 kg of marijuana formed into 18 bricks which were separately
wrapped. His possession thereof gives rise to a disputable presumption under
Section 3[j], Rule 131 of the Rules of Court, 47 that he is the owner of such bag
and its contents. His bare, unpersuasive, feeble and uncorroborated disavowal —
that the plastic bag was allegedly given to him by his uncle without his knowing
the contents — amounts to a denial which by itself is insufficient to overcome this
presumption. 48 Besides, this defense, unless substantiated by clear evidence, is
invariably viewed with disfavor by courts, for it can just as easily be concocted.
Verily, it is a common and standard defense ploy in most prosecutions involving
dangerous drugs. 49

Further, the trial court did not give credence to appellant’s denial. It is axiomatic
that appellate courts accord the highest respect to the assessment of witnesses’
credibility by the trial court, because the latter was in a better position to observe
their demeanor and deportment on the witness stand. 50 The defense failed to
present sufficient reasons showing that the trial court had overlooked or
misconstrued any evidence of substance that would justify the reversal of its
rejection of appellant’s defense of denial.

Appellant is, therefore, liable for illegal possession of prohibited drugs under
Section 8 of the Dangerous Drugs Act. 51chanrobles virtual lawlibrary

WHEREFORE, the assailed Decision is hereby MODIFIED. Appellant is


CONVICTED of illegal possession of prohibited drugs under Section 8 of R.A.
6425; SENTENCED, in accordance with the Indeterminate Sentence Law, to eight
(8) years as minimum to twelve (12) years as maximum; and ORDERED to pay a
fine of twelve thousand pesos (P12,000.00). Costs de oficio.

SO ORDERED.

Narvasa, C.J., Davide, Jr., Melo and Francisco, JJ., concur.


People v. Tabar, G.R. No. 101124, May 17, 1993

Carmelina Tabar y Carmilotes and her nephew, Rommel Arriesgado y Tabar, of


Tres de Abril, Punta Princesa, Cebu City, were charged with the violation of
Section 4, Article II of R.A.No. 6425, as amended, in an Information filed by the
Office of the City Fiscal of Cebu City with the Regional Trial Court of Cebu City
on 9 February 1989, the accusatory portion of which reads a follows:
That on or about the 8th day of February 1989, at about 3:00 PM. in
the City of Cebu, Philippines, and within the jurisdiction of this
Honorable Court, the said accused, conniving and confederating
together and mutually helping each other, with deliberate intent, did
then and there sell and deliver, without authority of law, Three (3)
sticks of marijuana cigarettes, a (sic) prohibited drugs, to a person
who posted himself as a buyer, in Viol. of Sec. 4, Art. 11, of RA
6425, as amended, otherwise known as the Dangerous Act of 1972. 1

The case was docketed as Criminal Case No. CBU-14863 and after it was raffled
off to Branch 15 of the said court, the accused were forthwith arraigned. Carmelina
entered a plea of not guilty while Rommel, then seventeen (17) years of age, with
the conformity of the prosecution, entered a plea of guilty to the lesser offense of
possession of marijuana under Section 8, Article II of R.A. No. 6425, as
amended.2 As a consequence of his plea, the trial court handed down on 24 April
1989 an Order which reads in part as follows:

Therefore this court being satisfied that the accused herein is the same
Ramil Tabar described in Annex 1 (Certificate of Birth) of the said
accused, and it appearing that he is still a minor (17 years), he is
entitled to a suspended sentence of the penalty for possession of
marijuana which is a jail term of six (6) years and one (1) day to
twelve (12) years and a fine of Six Thousand (P6,000.00) pesos (Sec.
8 RA 6425 as amended of B.P. 179, March 2, 1982).

WHEREFORE, the accused Rommel Tabar y Arriesgado is hereby


discharged on probation (Sec. 32 of RA 6425 as amended by B.P.
179) and committed to the custody of the Department of Social
Welfare and Development, Cebu Regional Office (No. 7) until he
reaches the age of majority, or otherwise finally discharged upon
orders of this court pursuant to P.D. 603 and B.P. 179, but to be
placed under the Supervision of the Dangerous Drugs Board, the
alleged crime being drug related, and for a period of one (1) year
from date hereof.

The Regional Director of the DSWD is hereby ordered to conduct and


submit a case study of the accused minor to this court, within sixty
days and to report on his conduct once every four months, to this
court.3

Thereafter, trial proceeded as against Carmelina alone. The prosecution presented


Pfc. Josephus Trangia and Myra P. Arreola, a forensic analyst of the PC Crime
Laboratory Service, as its witnesses. The testimony of Pfc. Raul Tumakay was
ordered stricken out since he could not be cross-examined. The defense had only
Carmelina as its witness.
On 22 December 1990, the trial court promulagated its decision, dated 17
December 1990,4 finding Carmelina "guilty, beyond reasonable doubt, for (sic)
violation of Section 4, Article II RA 6425, otherwise known as the Dangerous
Drug Act of 1972 as amended by PD 1675" and sentencing her to "Reclusion
Perpetua and to pay a fine of P20,000.00 for the act of selling and distributing
marijuana."5

The conviction is premised on the following findings of fact:

From the evidence which consists of the testimony of Pfc. Josephus


Trangia, the court gathered that at about 3:00 P.M. of February 8,
1989, he was with Pfc. Romeo Cortes and Gualberto Gabales on a
buy-bust operation for marijuana after receipt of information about
marijuana pushers in Punta Princesa, Cebu City and that they had
their informant go ahead of them after giving the P5.00 bill for him to
purchase marijuana.

He continued saying that their informant stood in front of a shanty


while they posted themselves at a distance of about 50 meters from
the place where their informant was standing. And that they saw a
young boy approached their informant and handed cigarettes to him
who in turn handed the marked money to the young boy. Then, their
informant gave them the pre-arranged signal of scratching his head
with his right hand; that after the signal, he and his companions
immediately approached the young boy and the informant introduced
them as police officers. This young boy was about 16-17 years old,
by the name of Rommel Arriesgado y Tabar. He had earlier pleaded
guilty to the lesser offense of mere possession of marijuana and was,
in fact, already convicted by this court. Upon being shown a P5.00
bill with the initials written thereon as: GDG-89 and bearing SL L F
637396, he identified the same bill as the one given to their informant
and marked as Exh. "A" for the prosecution. He explained the initials
GDG which stands for Gualberto G. Gabales, his team member. He
further declared that after the pre-arranged signal from their
informant, they immediately proceeded to the scene and were given
three sticks of marijuana by their informant after buying the same
from the boy, Rommel Arriesgado and that they proceeded to
confiscate the P5.00 bill from the boy. At this juncture, he claimed
that he observed that after the transaction, the boy went inside the
shanty and the moment he got out, he handed the three sticks of
marijuana to the informant. In fact, he claimed that after the
proceeding to the shanty, they met Carmelina Tabar, accused herein,
and that Carmelina Tabar was holding a white pants from where they
found other marijuana sticks in cigarette packs which they
confiscated. That they brought Carmelina Tabar to Fuente Police
Station for investigation. He claimed that there were 75 sticks of
marijuana in the Hope Cigarette pack; 22 sticks of marijuana
cigarettes in the Philip Morris pack and 99 sticks of marijuana in the
Mark cigarette pack. He said that they also confiscated the pants, but
only the marijuana sticks were submitted for testing to the PC Crime
Laboratory. That pursuant to this requested analysis, Lieut. Fortunato
Quijon of the Police issued a Certification of Field Test, Exh. "B"
which showed that three sticks of handrolled cigarettes marked
Rommel-89 were positive for marijuana. Shown the three packs of
cigarettes distincly marked as Hope, Mark and Philip Morris, he
identified the same as the ones confiscated from Carmelina Tabar. So
did he identify the cream-colored pants he said they confiscated from
Carmelina Tabar and which according to him was used to wrap
marijuana sticks inside the pack. He finally told the court that this
team was composed of Gualberto D. Gabales, Romeo Cortes, Pfc.
Tumakay and himself.

On cross-examination, this witness affirmed that when he asked the


boy, Rommel where he got the marijuana sticks, he was told he got it
from his aunt, the accused herein. He further told the court that they
arrested Carmelina Tabar later. When the young boy went inside,
they presumed the marijuana came from inside the shanty and that
when the accused Carmelina Tabar went out, suspicious-looking and
pale and afraid to face them, they told her to stop from going left
towards the houses and asked her to open the pants which revealed
the three cigarette packs contaning marijuana. The witness candidly
admitted they had no search warrant at the time they effected the
arrest and confiscation.

From the testimonies of Mrs. Myrna Areola, Police Lieutenant,


Forensic Analyst of the PC Crime Laboratory, it was established that
the specimens submitted to her were positive of marijuana. She then
identified Exh. "C", as her Chemistry Report C-038-89. She also
identified her signature, Exh. "C-4" thereon and her findings "All are
positive of Marijuana", Exh. "C-3"; the specimens submitted as Exh.
"C-2" and the name of the subject, Carmelina Tabar as Exh. "C-1".
She confirmed her findings on all handrolled cigarettes in all cigarette
packs; Hope, Exh. "E-1" to "E-75"; Philip Morris pack, Exh. "F-1" to
"F-22" and Mark cigarette pack, Exh. "G-1" to "G-99". She informed
the court that the specimens were submitted to her by Pfc Gabales on
February 11, 1989, at about 10:50 A.M., and that she examined the
sticks one by one and handrolled them again. She also claimed that
the police did not ask for a copy of her report and that this is the first
time it is presented in court. She informed the court that she placed
the specimens in her evidence store room, with keys, she herself
kept.6

The trial court discredited the bare denials of Carmelina and unfavorably
considered against her an admission that she had been arrested before by the
CANU for possession of marijuana, was charged for the violation of Section 8,
Article II of R.A. No. 6425 in Criminal Case No. CBU-8573, was convicted
therein, but is now on probation. 7 It further considered against her an allegedly
very damaging admission, thus:

She made a very damaging admission to the court when the Presiding
Judge asked her whether it is not true that she kept on crying because
she was caught again and she said, "Yes" and at which juncture she
admitted to the court that she was serving probation for the same
offense.8

It then concluded that:

[A]ccused actually employed her nephew, Rommel Arriesgado to sell


marijuana from her store and that she has been in that illicit business
for quite sometime now. The evidence notwithstanding, talks in the
community where the accused lives is rife with accusations (sic) that
she is indeed engaged with members of the family, in the sale and
distribution of prohibited drugs such as marijuana. Between the
positive testimony of the arresting officers, who appear to be more
credible than the accused's worthless and untrustworthy denials, the
court gives credence to the evidence of the prosecution. 9

Unable to accept the verdict, Carmelina filed her notice of appeal 10 manifesting


therein that she is appealing from the decision to the Court of Appeals. In its Order
of 27 February 1991,11 the trial court gave due course to the appeal and directed the
clerk of court "to submit all the records, evidences (sic) and trancripts of this
proceeding to the Hon. Court of Appeals, for proper disposition." Considering that
the penalty imposed is reclusion perpetua, the Court of Appeals transmitted to this
Court the records of the case on 12 August 1991. 12 In the Resolution of 11
September 1991, this Court accepted the appeal.

In her Appellant's Brief,13 Carmelina, hereinafter referred to as the appellant,


imputes upon the trial court the commission of the following errors in the appealed
decision:

I.

. . . IN CONVICTING ACCUSED-APPELLANTS (sic) OF


VIOLATION OF SECTION 4, ARTICLE II, REPUBLIC ACT 6425
AS AMENDED WHEN THE EVIDENCE DOES NOT WARRANT
IT.

II.

. . . IN ADMITTING EVIDENCE SEIZED WITHOUT ANY


SEARCH WARRANT.14

As to the first assigned error, the appellant claims that the prosecution presented no
evidence that she sold marijuana and since there exists no convincing, positive and
conclusive proof of conspiracy between her and her co-accused, Rommel
Arriesgado, she cannot be held liable for violation of Section 4, Article II of R.A.
No. 6425, as amended.

In support of the second assigned error, the appellant maintains that the marijuana
cigarettes seized from her are inadmissible in evidence because they were obtained
in violation of the constitutional guarantee against unreasonable search and seizure.

After a careful perusal of the records and evaluation of the evidence, this Court is
inclined to agree with the appellant that she should not be convicted under Section
4, Article II of R.A. No, 6425. We rule, however, that she is liable under Section 8,
Article II of the said Act. Her conviction by the trial court under Section 4 is
primarily based on its conclusion that the appellant "actually employed her nephew
Rommel Arriesgado to sell marijuana from her store and that she had been in that
illicit business for quite sometime now." This conclusion is based on the trial
court's sweeping statement that "talks in the community where the accused lives is
rife with accusations that she is indeed engaged with members of the family, in the
sale and distribution of prohibited drugs such as marijuana." We find no evidence
on record to sustain this charge. It may thus be said that such a conclusion is not
based on established facts but on "talks in the community." If indeed such was the
fact, it would not have been difficult for the prosecution to provide the court with
overwhelming evidence. Yet, it presented only Pat. Trangia who, rather
unfortunately, did not even testify or volunteer information that the main target of
the busy-bust operation was the appellant. He did not also disclose in his testimony
that the appellant was among the reported "pushers" in Punta Princesa, Cebu City.
It may be recalled that the buy-bust operation on 8 February 1989 was conducted
because, as he alleged:

A. Before that time we have already received information from the


community of Punta Princesa regarding marijuana pushers in that
place.15

Nevertheless, the prosecutor who conducted the direct-examination of


Trangia did not ask further as to identity of the pushers such that it was not
proven that the appellant was one of them. The Solicitor General, however,
maintains that there was conspiracy, established by circumstancial evidence,
between accused Rommel Arriesgado who was caught in flagrante selling
three (3) sticks of handrolled marijuana to the informant and accepting the
marked money. We are not persuaded since the evidence for the prosecution
does not show that (a) the appellant was in the mind of the members of the
team when they planned the buy-bust operation and when they carried out
such plan, (b) the three (3) sticks of handrolled marijuana came from the
appellant, and (c) the appellant used Rommel as her agent to sell the three
(3) sticks to the informant. Moreover, if indeed the prosecution truly
believed that such conspiracy existed, it should not have willingly given its
conformity to Rommel's plea to the lesser offense of illegal possession of
prohibited drugs under Section 8, Article II of R.A. No. 6425, as amended.
Having been caught in flagrante for selling marijuana, it was not difficult to
prove Rommel's culpability under Section 4, Article II of the Act. Yet it
readily consented to his offer to plead guilty to the said lesser offense.

It was, however, established beyond any shadow of doubt and, therefore, with
moral certainty, that the appellant kept in her possession handrolled sticks of
marijuana placed in empty Hope, Philip Morris and Mark cigarrette packs. 16 She
does not have any authority to possess them. She may have acquired them with the
intention to sell them for profit; but without proof of sale, she cannot be held liable
under Section 4, Article II of the Dangerous Drugs Act. For such possession, her
liability is covered by Section 8 of the said Article which penalizes possession or
use of prohibited drugs. The last paragraph thereof reads:

xxx xxx xxx

The penalty of the imprisonment ranging from six years and one day
to twelve years and a fine ranging from six thousand to twelve
thousand pesos shall be imposed upon any person who, unless
authorized by law, shall possess or use of Indian hemp.

Indian hemp is otherwise known as Marijuana.17

Appellant, therefore, may specifically be penalized under the aforesaid last


paragraph of Section 8, Article II of the Act. Applying the Indeterminate Sentence
Law,18 the penalty of eight (8) years as Minimum to twelve (12) years
as Maximum and a fine of P10,000.00 may then be imposed upon her.

The second assigned error is without merit. The evidence for the prosecution
discloses that the appellant placed the packs of marijuana sticks under the rolled
pair of pants which she was then carrying at the time she hurriedly left her shanty
after noticing the arrest of Rommel. When she was asked to spread it out, which
she voluntary did, the package containing the packs of marijuana sticks were thus
exposed in plain view to the member of the team. A crime was thus committed in
the presence of the policemen. Pursuant to Section 5, Rule 113 and Section 12
Rule 126 of the Revised Rules of Court, she could lawfully be arrested and
searched for anything which may be used as proof of the commission of an offense
without the corresponding arrest and search warrants. Her own counsel on cross-
examination of prosecution witness Josephus Trangia further obtained a
affirmation of these facts, thus:

Q. You mean to say that when you saw Carmelina Tabar


allegely went (sic) out of the shanty you only saw the
white long pants and not the cigarettes?

A. Only the pants.

Q. Did you say that in order to find out what was the
contents of the pants, you asked her to open the pants.
Isn't it?

A. Yes.

Q. Now, after she opened it, what did you see?

A. Three (3) packs of marijuana cigarettes.

Q. Who among you in your team approached Carmelina


Tabar?

A. It was PFC Raul Tumakay.19

Even assuming ex gratia argumenti that the seach and seizure were without
a warrant, the appellant had effectively waived her constitutional right
relative thereto by voluntarily submitting to the seach and seizure. In People
vs. Malasugui,20 this Court ruled:

When one voluntarily submits to a search and consent to have it made


of his person or premises, he is precluded from later complaining
thereof (Cooley, Constitutional Limitations, 8th ed., vol. I, pages
631). The right to be secure from unreasonable seach may, like every
right, be waived and such waiver may be made either expressly or
impliedly.

The exclusionary rule relied upon by the appellant does not provide her safe
refuge.

Before We close this case, a final observation for the guidance of trial judges must
be made.
For the violation of Section 4, Article II of R. A. No. 6425, as amended, the trial
court imposed the penalty of reclusion perpetua. The penalty provided for therein
is "life imprisonment to death and a fine ranging from twenty thousand to thirty
thousand pesos." In view of Section 19(1), Article III of the 1987 Constitution
which prohibits the imposition of the death penalty, the maximum penalty then
imposable thereunder would only be life imprisonment. Life imprisonment,
however, is not synonymous with reclusion perpetua. We have reiterated this time
and again21 and admonished judges to employ the proper legal terminology in the
imposition of imprisonment penalties because of their different accompanying
legal accessories and effects.22

IN THE LIGHT OF THE FOREGOING, judgment is hereby rendered modifying


the challenged Decision of Branch 15 of the Regional Trial Court of Cebu in
Criminal Case No. CBU-14863 dated 17 December 1990 and, as modified, finding
appellant CARMELINA TABAR y CARMILOTES guilty beyond reasonable
doubt of illegal possession of marijuana under Section 8, Article II of R. A. No.
6425, otherwise known as the Dangerous Drugs Act of 1972, as amended, and,
applying the Inderterminate Sentence Law, she is sentenced to suffer imprisonment
of eight (8) years as minimum to twelve (12) years as maximum and to pay a fine
of Ten Thousand Pesos (P10,000.00).

Costs against the appellant.

People v. Barros, G.R. No. 90640, March 29, 1994


Issue (1) WON the accused consented to the warrantless search and seizure
therefore waiving his rights to question its validity.

No, the accused did not consent.

Waiver of the non-admissibility of the "fruits" of an invalid warrantless arrest and


of a warrantless search and seizure is not casually to be presumed, if the
constitutional right against unlawful searches and seizures is to retain its vitality
for the protection of our people. 

In People v. Burgos, the Court ruled that the accused is not to be presumed to have
waived the unlawful search conducted on the occasion of his warrantless arrest
"simply because he failed to object".

". . . To constitute a waiver, it must appear first that the right exists;
secondly, that the person involved had knowledge, actual or
constructive, of the existence of such a right; and lastly, that said person
had an actual intention to relinquish the right (Pasion Vda. de Garcia vs.
Locsin, 65 Phil. 689). 
The Court apply the rule that: 'courts indulge every reasonable
presumption against waiver of fundamental constitutional rights and that
we do not presume acquiescence in the loss of fundamental rights .'
(Johnson vs. Zerbts, 304 U.S. 458)."

The accused's silence during the warrantless search should not be lightly taken as
consent to that search, but rather construed as explained by the Court in Burgos,
and as pointed out by Mr. Justice Laurel, a "demonstration of regard for the
supremacy of the law." 

It is possible that appellant Barros may in fact have been guilty of transporting the
four (4) kilos of marijuana. His guilt must, however, be established by
constitutional means. The non-admissibility of evidence secured through a
disregard of the constitutional right of the accused against unreasonable searches
and seizures is the sanction imposed by the Constitution for disregard of such right;
the sanction is a powerful one, for it renders inutile the work done by the police
officers, by the prosecutor and by the trial court.

Issue (2): WON the warrantless search and seizure of the carton box is invalid.

Yes, the warrantless search and seizure of the carton box is invalid.

The general rule is that a search and seizure must be carried out through or with a
judicial warrant; otherwise such search and seizure becomes "unreasonable" within
the meaning of Sections 2 and 3 [2], Article III of the 1987 Constitution. The
evidence secured thereby — i.e., the "fruits" of the search and seizure — will be
inadmissible in evidence "for any purpose in any proceeding."

The requirement that a judicial warrant must be obtained prior to the carrying out
of a search and seizure is, however, not absolute. 

There are certain exceptions recognized in our law, one of which relates to the
search of moving vehicles. Peace officers may lawfully conduct searches of
moving vehicles — automobiles, trucks, etc. — without need of a warrant, it not
being practicable to secure a judicial warrant before searching a vehicle, since such
vehicle can be quickly moved out of the locality or jurisdiction in which the
warrant may be sought. In carrying out warrantless searches of moving
vehicles, however, peace officers are limited to routine checks, that is, the
vehicles are neither really searched nor their occupants subjected to physical
or body searches, the examination of the vehicles being limited to visual
inspection.
When, however, a vehicle is stopped and subjected to an extensive search,
such a warrantless search would be constitutionally permissible only if the
officers conducting the search have reasonable or probable cause to believe,
before the search, that either the motorist is a law-offender or the contents or
cargo of the vehicle are or have been instruments or the subject matter or the
proceeds of some criminal offense.

In the case at bar, however, the Court have been unable to find in the record of this
case any circumstance which constituted or could have reasonably constituted
probable cause for the peace officers to search the carton box allegedly owned by
appellant Barros.

The carrying of such a box by appellant onto a passenger bus could not, by itself ,
have convinced M/Sgt. Francis Yag-as and S/Sgt. James Ayan either that the
appellant was a law violator or the contents of the box were instruments or the
subject matter or proceeds of some criminal offense. The carrying of carton boxes
is a common practice among our people, especially those coming from the rural
areas since such boxes constitute the most economical kind of luggage possible.

The peace officers here involved had not received any information or "tip-off"
from an informer; no such a "tip-off" was alleged by the police officers before or
during the trial. The police officers also did not contend that they had detected the
odor of dried marijuana, or appellant Barros had acted suspiciously in the course of
boarding the bus and taking a seat during the trip to Sabangan, nor in the course of
being asked whether he owned the carton box later ascertained
to contain four (4) kilos of marijuana.

The testimony of the law enforcement officers who had apprehended the accused
(M/Sgt. Francis Yag-as and S/Sgt. James Ayan), and who had searched the box in
his possession, (C2C Fernando Bongyao), simply did not suggest or indicate the
presence of any such probable
cause.

There was nothing to show that appellant Barros was then in the process of
"actually committing" or "attempting to commit" a crime. There was nothing on
the record that could have reasonably led the two (2) police officers to believe that
"an offense [had] in fact just been committed" when appellant Barros boarded the
bus at Chackchakan or when he was asked whether he owned the box here
involved at the checkpoint in Sabangan. The two (2) police officers, according to
the record, had no "personable knowledge of facts indicating that the person to be
arrested (appellant Barros) had committed it." 

There was no basis for a valid warrantless arrest. Accordingly, the search and
seizure of the carton box was equally non-permissible and invalid. The "fruits"
of the invalid search and seizure — i.e., the four (4) kilos of marijuana — should
therefore not have been admitted in evidence against appellant Barros.

Dispositive Portion:

The decision of the Regional Trial Court, Branch 35, Bontoc, Mountain Province,
in Criminal Case No. 687 is hereby REVERSED and SET ASIDE and appellant is
hereby ACQUITTED of the crime charged, the evidence lawfully before the trial
court not being sufficient to establish his guilt thereof beyond reasonable doubt.

People v. Aruta, G.R. No. 120915, April 3, 1998


Bonifacio Barros was charged with violating Section 4 of R.A. No. 6425, as
amended (known as the Dangerous Drugs Act of 1972), in an information which
read as follows:jgc:chanrobles.com.ph

"That on or about September 6, 1987, from Chackchakan, Bontoc, Mountain


Province, to Nacagang, Sabangan, Mountain Province, and within the jurisdiction
of this Honorable Court, the above-named accused while being a passenger in a
Dangwa Bus with Plate No. ABZ 242, destined for Baguio City, without lawful
authority did then and there willfully, unlawfully and feloniously carry with him as
part of his baggage and transport about four (4) kilos of dried marijuana which the
accused intended for distribution and sale at Baguio City, knowing fully well that
said marijuana is a prohibited drug or [a] source of [a] prohibited drug.chanrobles
law library

Contrary to law." 1

After trial, the trial court convicted Bonifacio Barros of violation of Section 4 of
R.A. No. 6425 as amended and sentenced him to suffer the penalty of reclusion
perpetua 2 and to pay a fine of P20,000.00.

Barros now appeals from the judgment of conviction and essentially asks this
Court to determine —

"Whether the [trial] court deprived [the] accused of his right to due process
by:chanrob1es virtual 1aw library

(1) ignoring manifest absence of the mandatory warrant in the arrest and search of
the accused;

(2) admitting confessions extracted from the accused after two hours of
interrogation conducted by four (4) soldiers one after the other under intimidating
circumstances; and
(3) misappreciation of facts." 3

The relevant facts as found by the trial court and as set forth in the court’s decision
are as follows:jgc:chanrobles.com.ph

"That on September 6, 1987, M/Sgt. Francis Yag-as and S/Sgt. James Ayan, both
members of the P.C. Mountain Province Command, rode the Dangwa Bus bearing
Plate No. ABZ-242 bound for Sabangan, Mountain Province. Upon reaching
Chackchakan, Bontoc, Mountain Province, the bus stopped and both M/Sgt. Yag-
as and S/Sgt. Ayan, who were seated at the back, saw accused carrying a carton,
board the bus and seated himself on seat No. 18 after putting the carton under his
seat. Thereafter, the bus continued and upon reaching Sabangan, M/Sgt. Yag-as
and S/Sgt. Ayan before they alighted, it being their station, called C2C [Fernando]
Bongyao to inspect the carton under seat No. 18. After C2C Bongyao inspected the
carton, he found out that it contained marijuana and he asked the passengers [who]
the owner of the carton [was] but nobody answered. Thereafter, C2C Bongyao
alighted with the carton and S/Sgt. Ayan and C2C Bongyao invited the herein
accused to the detachment for questioning as accused was the suspected owner of
the carton containing marijuana. As both P.C. officers Yag-as and Ayan saw
accused, Bonifacio Barros carrying that same carton when he boarded the bus at
Chackchakan. That upon entering the detachment the carton was opened in the
presence of accused and accused Bonifacio Barros was asked if he owned the
carton of marijuana and accused denied [this]. That when accused denied
ownership of the carton of marijuana, the P.C. officers called for the bus conductor
who pinpointed to Bonifacio Barros as the owner of the carton of marijuana. That
during the oral investigation of accused, he finally admitted ownership of the
carton (Exhibit ‘B’) containing [four] 4 paper-wrapped packages of dried
marijuana. (Exhibits ‘B-1, ‘B-2,’ ‘B-3’ and ‘B-4’).cralawnad

. . . [A]fter he was orally investigated, [the accused] was brought to the Abatan
General Hospital, Bauko, Mountain Province, for physical examination and a
Medico Legal Certificate was issued (Exhibits ‘F’ and ‘F-1’), indicating that
accused suffered no physical injuries and that accused was probably under the
influence of marijuana. That Dra. Danna Aleta inquired from accused Bonifacio
Barros if he smoked marijuana and accused admitted having smoked marijuana.
That after accused was medically examined, he was escorted by three members of
the P.C. to the P.C. detachment at Tadian, Mountain Province, where the carton of
marijuana (Exhibit ‘B’) was also brought. That at Tadian, a seizure receipt was
made together with a certification (Exhibit ‘C’) pointing out to the fact that
approximately 4 kilos of dried marijuana leaves were from accused Bonifacio
Barros and which certification was signed by the accused (Exhibit ‘C-1’) and
subscribed before Judge Romualdo P. Awisan (Exhibit ‘C-2’). That in connection
with the confiscation of the marijuana subject of the instant case and the
apprehension of accused Bonifacio Barros, the P.C. officers who figured in this
case namely M/Sgt. Yag-as and S/Sgt. Ayan and C2C Bongyao have
correspondingly executed their sworn statements (Exhibits ‘A’, ‘A-1, ‘A-2’, ‘D’,
‘D-1’ and ‘D-2’).

. . . [S]amples of the marijuana were taken from each of the four packages marked
Exhibits ‘B-1’, ‘B-2’, ‘B-3’, and ‘B-4’ and placed in four separate envelopes,
following an order of the court to that effect and were hand-carried by Police
Officer Jack Masilian to Camp Dangwa, La Trinidad, Benguet for laboratory test.
That Capt. Carlos Figueroa, the Forensic Expert conducted two kinds of test on the
four samples sent by the court and found them to be positive of marijuana as per
his report No. D-011-88. (Exhibits ‘I’ and ‘I-1’). 4

The defense of the accused on the facts consisted of a simple denial of the
ownership or possession of the carton box containing the four (4) kilos of
marijuana. The trial court summarized the story of the accused in the following
manner:jgc:chanrobles.com.ph

"That accused Bonifacio Barros since 1984 was employed at the Honeymoon
Disco Pad, Baguio City. That on September 5, 1987, Accused was sent by his
Manager, Engineer Arsenio Cuanguey to Bontoc, Mountain Province, to get their
records from one Billy Cuanguey at Chackchakan, Bontoc, Mountain Province.
That upon arriving at Chackchakan, Bontoc, Mountain Province, Accused looked
for the residence of Billy Cuanguey and he was pointed to a house where someone
was tending a store. That accused asked the man if Billy Cuanguey was there and
the man answered that he did not know where Billy went. So accused asked the
man if Billy left [in] his room the tapes and records and the man said he did not
know. Thereafter, Accused asked the man to stay over night in that house where
Billy was staying as it was the instruction of his manager. That the following day,
September 6, 1987, after taking breakfast, Accused, was going back to Baguio. On
that morning of September 6, 1987, Accused Bonifacio Barros boarded the
Dangwa Bus at Chackchakan, Bontoc, Mountain Province bound for Baguio City.
That when the Dangwa Bus reached the P.C. Checkpoint, soldiers went inside the
bus and checked the baggages. That a soldier fished out a carton under the seat of
[the] accused and shouted who owns the carton but nobody answered. Thereafter,
the soldier went down with the carton and moments later returned to the bus and
called accused Bonifacio Barros to alight from the bus. That Mr. Barros was
surprised why he was ordered to alight and accused took his baggage which
consisted of a pasiking and went down the bus. That accused was led by the
soldiers to a house where his pasiking was taken and his clothes removed and his
wallet taken. Accused was made to accept ownership of the carton of marijuana but
he refused.chanrobles virtual lawlibrary

. . . [A]t 11:00 o’clock that same day, September 6, 1987, three soldiers escorted
accused to the hospital and from the hospital, they proceeded to the Municipality
of Tadian, Mountain Province. That upon reaching Tadian, Accused was brought to
the P.C. Camp and there he saw someone typing. Later, the soldiers allegedly
presented to accused some papers which he was asked to sign but accused refused.
That accused was threatened and if he refused to sign the papers that something
will happen to him. That moments later, Accused was threatened [by] a soldier
[who] pointed a gun to him and told him to sign the paper and because of fear, he
had to sign the document marked Exhibit ‘C.’ Thereafter, the soldiers allegedly
threatened again accused and asked him to sign his name on the inside part of the
cover of the carton of marijuana. Exhibit ‘X’ for the court and Exhibit ‘B-5’ for the
prosecution. That after staying at Tadian for one night, Accused was brought back
to Sabangan and later transferred to the Bontoc Provincial Jail." 5

Turning to the legal defenses of the accused, we consider first his allegation that
the police authorities had impermissibly extracted confessions from him after two
(2) hours of interrogation, "under intimidating circumstances," by four (4) soldiers
one after the other. The accused complains that he was not informed of his rights to
remain silent and to counsel, that he had not waived his rights as an accused
person, and that he had signed a confession involuntarily and without the
assistance of counsel. He essentially contends that the confession is inadmissible as
evidence against him.

We find, however, that it is not necessary to pass upon the above contention of
appellant Barros. For the trial court in reaching its judgment of conviction had not
taken into consideration the statements which had been obtained from the appellant
during the interrogation conducted by the police officers. The trial court, so far as
can be determined from its decision, totally disregarded Exhibits "C", "E" and "B-
5," the alleged uncounselled confessions. The trial court made very clear the bases
of its conclusion that the accused was guilty beyond reasonable doubt of the
offense charged; those bases did not include the alleged
confessions:jgc:chanrobles.com.ph

"First — M/Sgt. Francis Yag-as and S/Sgt. James Ayan testified that they saw the
accused carrying the carton (Exhibit ‘B’) when he boarded the bus at
Chackchakan, Bontoc, Mountain Province. That the bus conductor pointed to
accused at the checkpoint of Sabangan, Mountain Province. That accused is the
owner of the carton (Exhibit ‘B’). That the carton (Exhibit ‘B’) which contained
four packages of dried marijuana leaves (Exhibits ‘B-1’, ‘B-2’, ‘B-3’ and ‘B-4’)
was fished out from under the seat of the accused which fact was admitted by the
accused himself.

Second — That per testimony of Dra. Danna Aleta, she examined accused
Bonifacio Barros and that he suffered no physical injuries that would show that the
accused was in anyway maltreated by the police authorities, and this fact was also
admitted by accused to the effect that he was never harmed by the police nor the
soldiers. Dra. Aleta also found that the accused was under the influence of drug[s]
and that the accused admitted [to] her that he, Accused, smoked marijuana. This is
clear evidence that accused is not only a pusher of marijuana but also a user of said
prohibited drugs. (See Exhibits ‘F’ and ‘F-1’ and TSN — Page 24 — Orpecio).

Third — The samples taken from Exhibits ‘B-1’, ‘B-2’, ‘B-3’ and ‘B-4’ sent by the
court for laboratory test at Camp Dangwa, La Trinidad, Benguet were all positive
of marijuana per Report No. D-011-88 (Exhibits ‘I’ and ‘I-1’) of Captain Carlos
Figueroa, forensical expert.

Lastly, Accused’s testimony in his own behalf does not impress the court at it lacks
the ring of truth. Besides, it is devoid of any corroboration. Our Supreme Court in
this respect said:chanrob1es virtual 1aw library

‘The weak and uncorroborated denial of the accused cannot prevail over the clear,
positive and straightforward testimony of prosecution witnesses [sic].’ (People v.
Acelajao, 148 SCRA 142).’" 6

We turn, therefore, to the second legal defense asserted by appellant Barros — i.e.,
that his constitutional right against unreasonable searches and seizures had been
violated by the police authorities. The relevant constitutional provisions are found
in Sections 2 and 3 [2], Article III of the 1987 Constitution which read as
follows:jgc:chanrobles.com.ph

"Sec. 2. The right of the people to be secure in their persons, houses, papers and
effects against unreasonable searches and seizures of whatever nature and for any
purpose shall be inviolable, and no search warrant or warrant of arrest shall issue
except upon probable cause to be determined personally by the judge after
examination under oath or affirmation of the complainant and the witness as he
may produce, and particularly describing the place to be searched and the persons
or things to be seized.

Sec. 3. . . .

(2) Any evidence obtained in violation of this or the preceding section shall be
inadmissible for any purpose in any proceeding."cralaw virtua1aw library

The general rule is that a search and seizure must be carried out through or with a
judicial warrant; otherwise such search and seizure becomes "unreasonable" within
the meaning of the above quoted constitutional provision. 7 The evidence secured
thereby — i.e., the "fruits" of the search and seizure — will be inadmissible in
evidence "for any purpose in any proceeding." 8

The requirement that a judicial warrant must be obtained prior to the carrying out
of a search and seizure is, however, not absolute. There are certain exceptions
recognized in our law, one of which relates to the search of moving vehicles. 9
Peace officers may lawfully conduct searches of moving vehicles — automobiles,
trucks, etc. — without need of a warrant, it not being practicable to secure a
judicial warrant before searching a vehicle, since such vehicle can be quickly
moved out of the locality or jurisdiction in which the warrant may be sought. 10 In
carrying out warrantless searches of moving vehicles, however, peace officers are
limited to routine checks, that is, the vehicles are neither really searched nor their
occupants subjected to physical or body searches, the examination of the vehicles
being limited to visual inspection. In Valmonte v. De Villa, 11 the Court
stated:jgc:chanrobles.com.ph

" [N]ot all searches and seizures are prohibited. Those which are reasonable are not
forbidden. A reasonable search is not to be determined by any fixed formula but is
to be resolved according to the facts of each case.

Where, for example, the officer merely draws aside the curtain of a vacant vehicle
which is parked on the public fair grounds, or simply looks into a vehicle, or
flashes a light therein, these do not constitute unreasonable search." (Citations
omitted)chanrobles law library : red

When, however, a vehicle is stopped and subjected to an extensive search, such a


warrantless search would be constitutionally permissible only if the officers
conducting the search have reasonable or probable cause to believe, before the
search, that either the motorist is a law-offender or the contents or cargo of the
vehicle are or have been instruments or the subject matter or the proceeds of some
criminal offense. 12

This Court has in the past found probable cause to conduct without a judicial
warrant an extensive search of moving vehicles in situations where (1) there had
emanated from a package the distinctive smell of marijuana; 13 (2) agents of the
Narcotics Command ("Narcom") of the Philippine National Police ("PNP") had
received a confidential report from informers that a sizeable volume of marijuana
would be transported along the route where the search was conducted; 14 (3)
Narcom agents were informed or "tipped off" by an undercover "deep penetration"
agent that prohibited drugs would be brought into the country on a particular
airline flight on a given date; 15 (4) Narcom agents had received information that a
Caucasian coming from Sagada, Mountain Province, had in his possession
prohibited drugs and when the Narcom agents confronted the accused Caucasian,
because of a conspicuous bulge in his waistline, he failed to present his passport
and other identification papers when requested to do so; 16 and (5) Narcom agents
had received confidential information that a woman having the same physical
appearance as that of the accused would be transporting marijuana. 17
In the case at bar, however, we have been unable to find in the record of this case
any circumstance which constituted or could have reasonably constituted probable
cause for the peace officers to search the carton box allegedly owned by appellant
Barros. The carrying of such a box by appellant onto a passenger bus could not, by
itself , have convinced M/Sgt. Francis Yag-as and S/Sgt. James Ayan either that
the appellant was a law violator or the contents of the box were instruments or the
subject matter or proceeds of some criminal offense. The carrying of carton boxes
is a common practice among our people, especially those coming from the rural
areas since such boxes constitute the most economical kind of luggage possible.
The peace officers here involved had not received any information or "tip-off"
from an informer; no such a "tip-off" was alleged by the police officers before or
during the trial. The police officers also did not contend that they had detected the
odor of dried marijuana, or appellant Barros had acted suspiciously in the course of
boarding the bus and taking a seat during the trip to Sabangan, nor in the course of
being asked whether he owned the carton box later ascertained to contain four (4)
kilos of marijuana. The testimony of the law enforcement officers who had
apprehended the accused (M/Sgt. Francis Yag-as and S/Sgt. James Ayan), and who
had searched the box in his possession, (C2C Fernando Bongyao), simply did not
suggest or indicate the presence of any such probable cause.

M/Sgt. Francis Yag-as testified as follows:jgc:chanrobles.com.ph

"Direct Examination by Fiscal Moises Ayochok:chanrob1es virtual 1aw library

x       x       x

Q: On September 6, 1987, do you recall if you reported for duty?

A: Yes, sir.

Q: And where did you go on the morning of September 6, 1987?

A: I went to Sabangan, sir.

Q: What transportation did you use?

A: Dangwa Bus with Plate No. ABZ-242.

Q: Where did you board the Dangwa Bus?

A: At the Dangwa Terminal at Bontoc.

Q: When you said you boarded the bus with Plate No. ABZ-242 which started for
Baguio City from Bontoc, Mountain Province, and while it stopped at
Chackchakan, Bontoc, Mountain Province, was there anything that happened?

x       x       x

A: When the bus stopped at Sitio Chackchakan, we saw a person carrying a


baggage or carton and boarded the bus then took his seat, seat No. 18.

Q: What was he carrying that time Mr. witness?chanrobles virtual lawlibrary

A: A carton.

Q: And where did he place that carton which he was carrying?

A: In front of seat No. 18 where he sat.

Q: You mean inside the bus?

A: Yes.

Q: And after this person boarded the bus at sitio Chackchakan and holding a carton
and placed it in front of seat No. 18, what happened to the bus afterwards?

A: It proceeded to Sabangan.

Q: And at Sabangan, Mountain Province, what happened, if any?

A: The bus stopped for the routinary checkpoint and inspection.

Q: When they [were at] the routinary checkpoint, what happened?

Atty. Sokoken:chanrob1es virtual 1aw library

He did not say routinary checkpoint. He said routinary inspection.

Fiscal Ayochok:chanrob1es virtual 1aw library

We substitute the words inspection with checkpoint to satisfy the objection of


counsel.

Q: What happened when you stopped for the routinary inspection?

A: We called C2C Bongyao a member of the detachment to inspect the baggage of


the suspect and when C2C . . .

Atty. Sokoken:chanrob1es virtual 1aw library

We request that [the] witness answers the question that he testifies [to] not in the
narrative way.

Fiscal Ayochok:chanrob1es virtual 1aw library

He is answering the question.

Court:chanrob1es virtual 1aw library

Let the witness finish.

A: When Bongyao inspected the baggage of the suspect and he found out that it
contained MJ.

Q: What do you mean MJ?

A: Marijuana.

x       x       x" 18

For his part, S/Sgt. James Ayan testified as follows:jgc:chanrobles.com.ph

"Direct Examination:chanrob1es virtual 1aw library

x       x       x

Q: And in the morning of September 6, 1987, do you recall where you were
particularly in the afternoon?

A: In the morning of September 6, 1987, we rode on a Dangwa bus [with Plate]


No. ABZ-242 going to Sabangan.

Q: You said we. Who was your companion that time?

A: Master Sgt. Yag-as, sir.

Q: And when this bus reached Chackchakan, Bontoc, Mountain Province, what did
you see?
A: We saw a civilian board the bus we were riding carrying a carton.

Q: And where did this civilian who boarded the bus which you were riding on
place that carton?chanrobles law library : red

A: He placed the carton under the seat of No. 18.

Q: Inside the bus, Mr. witness?

A: Inside the bus, sir.

Q: And what about the passenger who boarded the bus carrying the carton
baggage, where did he go?

A: He sat facing the seat No. 18.

Q: Between seat No. 18 and the seat seated by the civilian who brought the carton,
where was the carton exactly located?

A: As far as I know, sir, it was located just beneath seat No. 18.

Q: When this bus which you rode on which the passenger carrying the carton
luggage you saw reached Sabangan what happened there?

A: When the bus reached Sabangan that we were riding, it was stopped for
routinary inspection.

Q: What happened next?

A: We called C2C Bongyao to inspect the baggage that we have just seen at
Chackchakan.

Q: Did he inspect the baggage?

A: Yes, sir.

Q: And what was the contents of that baggage if there was any?

A: It turned out that the contents of the baggage was MJ sir.

Q: You mean marijuana?

A: Yes, sir.
x       x       x

Cross Examination:chanrob1es virtual 1aw library

x       x       x

Q: You stated that on September 6, 1987, a Dangwa bus stopped at Sabangan, Mt.
Province for purposes of military check-up, is that correct?

A: Routinary inspection, sir.

Q: But it was not you who entered the Dangwa bus for routinary check-up?

A: We were there riding in the bus, sir, and we called C2C Bongyao to come.

Q: So your purpose in riding inside the Dangwa bus was actually to see that person
carrying this carton which is marked Exhibit ‘B’?

A: No, sir, because I am a detachment commander at Sabangan and that is why I


called one of my men, sir.

Q: So that you have full knowledge that from Chackchakan, Bontoc, going to
Sabangan, there is already marijuana being carried inside that bus?

A: That is only our suspect [should be suspicion], sir.

Q: Would you please tell this Honorable Court why you have not inspected it when
you arrived at Alab? Why have you waited to reach Sabangan to inspect it?
chanrobles lawlibrary : rednad

A: Because it is the checkpoint, sir, at Nacagang, Sabangan.

Q: Are you now admitting that you do not have authority to inspect the baggage
here in Bontoc?

A: We just wanted it checked in Sabangan, sir.

Q: Could you give us a very special reason why you have to wait in Sabangan?

A: Because we are stationed in Sabangan and that is the checkpoint.

Fiscal Ayochok:chanrob1es virtual 1aw library


Why argue with the witness? It is up for them to check it at the proper checkpoint.

Court:chanrob1es virtual 1aw library

Sustained.

x       x       x" 19

The testimony of C2C Fernando Bongyao is much briefer, but equally


uninformative:jgc:chanrobles.com.ph

"Direct Examination:chanrob1es virtual 1aw library

Q: On September 6, 1987, at around 9:30 a.m., do you recall having reported for
duty at Nacagang, Sabangan, Mountain Province?

A: Yes, sir.

Q: And while you were on duty at Nacagang, Sabangan, was there anything
unusual that happened that time?

A: Yes, sir.

Q: What was that Mr. witness?

A: When we were on the checkpoint, the bus stopped bearing Plate No. ABZ-242.

Q: When the bus stopped, what did you do?

A: While on my way to check the bus, Master Sergeant Yag-as and Ayan called for
me, sir, and they told me that a carton was placed under seat No. 18, sir.

Q: And when you were told to inspect that carton under seat No. 18, did you
inspect that carton?

A: I inspected it, sir.chanrobles virtual lawlibrary

Q: You said you inspected that carton, what did you do in inspecting that carton?

A: I inserted my hand inside and when I removed my hand, it was a stuff of


marijuana, sir.

x       x       x 20
So far as the record itself is concerned, therefore, it would appear that there existed
no circumstance which might reasonably have excited the suspicion of the two (2)
police officers riding in the same bus as appellant Barros. They asked the police
officers at the checkpoint at Sabangan to inspect the box allegedly carried by
appellant Barros apparently on a mere guess that appellant Barros might be
carrying something in the nature of contraband goods. There was, in other words,
nothing to show that appellant Barros was then in the process of "actually
committing" or "attempting to commit" a crime. 21 There was, moreover, nothing
on the record that could have reasonably led the two (2) police officers to believe
that "an offense [had] in fact just been committed" when appellant Barros boarded
the bus at Chackchakan or when he was asked whether he owned the box here
involved at the checkpoint in Sabangan. The two (2) police officers, according to
the record, had no "personable knowledge of facts indicating that the person to be
arrested (appellant Barros) had committed it." There was, in brief, no basis for a
valid warrantless arrest. Accordingly, the search and seizure of the carton box was
equally non-permissible and invalid. 22 The "fruits" of the invalid search and
seizure — i.e., the four (4) kilos of marijuana — should therefore not have been
admitted in evidence against appellant Barros.

The Solicitor General, however, contends that appellant Barros had waived any
irregularities which may have attended his arrest. Presumably, the Solicitor
General also argues that appellant Barros has waived the non-admissibility of the
carton (Exhibit "B") which contained four (4) packages of dried marijuana leaves
(Exhibits "B-1", "B-2", "B-3" and "B-4." The Solicitor General
said:jgc:chanrobles.com.ph

". . . [E]ven assuming in gratia argumenti that irregularities attended the arrest of
appellant, still the same cannot be questioned at this late stage. Well-settled is the
doctrine laid down in the case of Callanta v. Villanueva (77 SCRA 377), and later
reiterated in the more recent case of Bagcal v. Villaraza (120 SCRA 525), that
‘posting of [a] bail bond constitutes waiver of any irregularity attending the arrest
of a person and estops him from questioning its validity.’ Here, appellant had in
fact posted the required bail to obtain his provisional liberty, albeit his application
was subsequently denied (see TSN, Feb. 10, 1988, p. 65). Consistent with
jurisprudence, therefore, he should be deemed to have waived any irregularity
attending his arrest, if any there be, and cannot now be heard to assail the same."
23

It might be supposed that the non-admissibility of evidence secured through an


invalid warrantless arrest or a warrantless search and seizure may be waived by an
accused person. The a priori argument is that the invalidity of an unjustified
warrantless arrest, or an arrest effected with a defective warrant of arrest may be
waived by applying for and posting of bail for provisional liberty, so as to estop as
accused from questioning the legality or constitutionality of his detention or the
failure to accord him a preliminary investigation. We do not believe, however, that
waiver of the latter (by, e.g., applying for and posting of bail) necessarily
constitutes, or carries with it, waiver of the former — an argument that the
Solicitor General appears to be making impliedly. Waiver of the non-admissibility
of the "fruits" of an invalid warrantless arrest and of a warrantless search and
seizure is not casually to be presumed, if the constitutional right against unlawful
searches and seizures is to retain its vitality for the protection of our people. In the
case at bar, defense counsel had expressly objected on constitutional grounds to the
admission of the carton box and the four (4) kilos of marijuana when these were
formally offered in evidence by the prosecution. 24 We consider that appellant’s
objection to the admission of such evidence was made clearly and seasonably and
that, under the circumstances, no intent to waive his rights under the premises can
be reasonably inferred from his conduct before or during the trial.cralawnad

In the dissenting opinion, my learned brother Melo, J. takes the view that appellant
Barros had waived his rights by his "stoic deportment" consisting of failure to
object to the search by the police authorities immediately after the opening of the
carton box:jgc:chanrobles.com.ph

". . . In point of fact, when the police authorities inspected the carton of marijuana
and asked accused-appellant who owned the box, Accused-appellant denied
ownership of the box or carton and failed to even mutter the least bit of protest (p.
3, Decision). His demeanor should therefore be construed as implicit acquiescence
to the search inasmuch as the objection thereto is vulnerable to express or implied
waiver (People v. Kaqui Malasuqui (63 Phil. 221 [1936]); 1 Bernas, Constitution
of the Republic of the Philippines, First ed., 1987, p. 108). . . ."25cralaw:red

It is submitted, with respect, that Kaqui Malasuqui is not applicable to the case at
bar; rather it is People v. Burgos, 26 promulgated fifty (50) years after Kaqui
Malasuqui, that is applicable. In Burgos, this Court ruled that the accused is not to
be presumed to have waived the unlawful search conducted on the occasion of his
warrantless arrest "simply because he failed to object" —

". . . To constitute a waiver, it must appear first that the right exists; secondly, that
the person involved had knowledge, actual or constructive, of the existence of such
a right; and lastly, that said person had an actual intention to relinquish the right
(Pasion Vda. de Garcia v. Locsin, 65 Phil. 689). The fact that the accused failed to
object to the entry into his house does not amount to a permission to make a search
therein (Magoncia v. Palacio, 80 Phil. 770). As pointed out by Justice Laurel in the
case of Pasion Vda. de Garcia v. Locsin (supra):chanrob1es virtual 1aw library

x       x       x
. . . As the constitutional quaranty is not dependent upon any affirmative act of the
citizen, the courts do not place the citizen in the position of either contesting an
officer’s authority by force, or waiving his constitutional rights; but instead they
hold that a peaceful submission to a search or seizure is not a consent or an
invitation thereto, but is merely a demonstration of regard for the supremacy of the
law.’ (Citation omitted).

We apply the rule that: ‘courts indulge every reasonable presumption against
waiver of fundamental constitutional rights and that we do not presume
acquiescence in the loss of fundamental rights.’ (Johnson v. Zerbts, 304 U.S.
458)." 27 (Emphasis supplied)

Kaqui Malasuqui is not applicable to the instant case, because there the Court
explicitly found that there was probable cause for the warrantless arrest of the
accused and therefore, the warrantless search effected immediately thereafter was
equally lawful. In Kaqui Malasuqui, a Chinese merchant was found lying on the
ground with several nasty wounds in the head; one resulted in skull fracture and
proved fatal. He died in the hospital to which he had been immediately brought by
a policeman. Mr. Malasuqui became a suspect because when the victim was found,
still alive, and upon being asked who had attacked him, laconically answered,
"Kagui." On the same day, the accused Kaqui Malasuqui was arrested and a search
of his person was conducted without objection from the accused. Before the body
search of the accused was carried out, the accused voluntarily surrendered to the
police authorities a couple of bracelets belonging to the deceased victim and when
asked if he had anything else to surrender, he, in a trembling voice, answered in the
negative. The police thereupon conducted a body search of the accused, without
any objection from him; the search resulted in the production of additional
personal effects belonging to the deceased victim. Under these circumstances, the
Court ruled that:jgc:chanrobles.com.ph

"When one voluntarily submits to a search or consents to have it made of his


person or premises, he is precluded from complaining later thereof. (Cooley,
Constitutional Limitations, 8th ed., [V]ol. I, p. 631.) The right to be secure from
unreasonable search may, like every right, be waived and such waiver may be
made either expressly or impliedly."cralaw virtua1aw library

A propos my distinguished brother Melo, J ‘s suggestion that the right against an


unlawful warrantless search or arrest is personal and may not be invoked by the
accused’s counsel during trial, it is relevant to note that the law (the Rules of
Court) specifies the proper time when objections to admission of evidence must be
raised and that in the case at bar, a timely objection was made by appellant Barros.
Finally, the accused’s silence during the warrantless search should not be lightly
taken as consent to that search, but rather construed as explained by the Court in
Burgos, 28 and as pointed out by Mr. Justice Laurel, a "demonstration of regard for
the supremacy of the law." chanrobles virtualawlibrary
chanrobles.com:chanrobles.com.ph

It is, of course, possible that appellant Barros may in fact have been guilty of
transporting the four (4) kilos of marijuana. His guilt must, however, be established
by constitutional means. The non-admissibility of evidence secured through a
disregard of the constitutional right of the accused against unreasonable searches
and seizures is the sanction imposed by the Constitution for disregard of such right;
the sanction is a powerful one, for it renders inutile the work done by the police
officers, by the prosecutor and by the trial court. It is a sanction which this Court
has no choice but to apply in the instant case.

WHEREFORE, for all the foregoing, the decision of the Regional Trial Court,
Branch 35, Bontoc, Mountain Province, in Criminal Case No. 687 is hereby
REVERSED and SET ASIDE and appellant is hereby ACQUITTED of the crime
charged, the evidence lawfully before the trial court not being sufficient to
establish his guilt thereof beyond reasonable doubt. No costs.

SO ORDERED.

CASES:
Papa v. Mago, G.R. No. L-27360, February 28, 1968

          This is an original action for prohibition and certiorari, with preliminary


injunction filed by Ricardo Papa, Chief of Police of Manila; Juan once Enrile,
Commissioner of Customs; Pedro Pacis, Collector of Customs of the Port of
Manila; and Martin Alagao, a patrolman of the Manila Police Department, against
Remedios Mago and Hon. Hilarion Jarencio, Presiding Judge of Branch 23 of the
Court of First Instance of Manila, praying for the annulment of the order issued by
respondent Judge in Civil Case No. 67496 of the Court of First Instance of Manila
under date of March 7, 1967, which authorized the release under bond of certain
goods which were seized and held by petitioners in connection with the
enforcement of the Tariff and Customs Code, but which were claimed by
respondent Remedios Mago, and to prohibit respondent Judge from further
proceeding in any manner whatsoever in said Civil Case No. 67496. Pending the
determination of this case this Court issued a writ of preliminary injunction
restraining the respondent Judge from executing, enforcing and/or implementing
the questioned order in Civil Case No. 67496 and from proceeding with said case.

          Petitioner Martin Alagao, head of the counter-intelligence unit of the Manila


Police Department, acting upon a reliable information received on November 3,
1966 to the effect that a certain shipment of personal effects, allegedly misdeclared
and undervalued, would be released the following day from the customs zone of
the port of Manila and loaded on two trucks, and upon orders of petitioner Ricardo
Papa, Chief of Police of Manila and a duly deputized agent of the Bureau of
Customs, conducted surveillance at gate No. 1 of the customs zone. When the
trucks left gate No. 1 at about 4:30 in the afternoon of November 4, 1966, elements
of the counter-intelligence unit went after the trucks and intercepted them at the
Agrifina Circle, Ermita, Manila. The load of the two trucks consisting of nine bales
of goods, and the two trucks, were seized on instructions of the Chief of Police.
Upon investigation, a person claimed ownership of the goods and showed to the
policemen a "Statement and Receipts of Duties Collected in Informal Entry No.
147-5501", issued by the Bureau of Customs in the name of a certain Bienvenido
Naguit.

          Claiming to have been prejudiced by the seizure and detention of the two
trucks and their cargo, Remedios Mago and Valentin B. Lanopa filed with the
Court of First Instance of Manila a petition "for mandamus with restraining order
or preliminary injunction, docketed as Civil Case No. 67496, alleging, among
others, that Remedios Mago was the owner of the goods seized, having purchased
them from the Sta. Monica Grocery in San Fernando, Pampanga; that she hired the
trucks owned by Valentin Lanopa to transport, the goods from said place to her
residence at 1657 Laon Laan St., Sampaloc, Manila; that the goods were seized by
members of the Manila Police Department without search warrant issued by a
competent court; that anila Chief of Police Ricardo Papa denied the request of
counsel for Remedios Mago that the bales be not opened and the goods contained
therein be not examined; that then Customs Commissioner Jacinto Gavino had
illegally assigned appraisers to examine the goods because the goods were no
longer under the control and supervision of the Commissioner of Customs; that the
goods, even assuming them to have been misdeclared and, undervalued, were not
subject to seizure under Section 2531 of the Tariff and Customs Code because
Remedios Mago had bought them from another person without knowledge that
they were imported illegally; that the bales had not yet been opened, although
Chief of Police Papa had arranged with the Commissioner of Customs regarding
the disposition of the goods, and that unless restrained their constitutional rights
would be violated and they would truly suffer irreparable injury. Hence, Remedios
Mago and Valentin Lanopa prayed for the issuance of a restraining order, ex parte,
enjoining the above-named police and customs authorities, or their agents, from
opening the bales and examining the goods, and a writ of mandamus for the return
of the goods and the trucks, as well as a judgment for actual, moral and exemplary
damages in their favor.

          On November 10, 1966, respondent Judge Hilarion Jarencio issued an


order ex parte restraining the respondents in Civil Case No. 67496 — now
petitioners in the instant case before this Court — from opening the nine bales in
question, and at the same time set the hearing of the petition for preliminary
injunction on November 16, 1966. However, when the restraining order was
received by herein petitioners, some bales had already been opened by the
examiners of the Bureau of Customs in the presence of officials of the Manila
Police Department, an assistant city fiscal and a representative of herein respondent
Remedios Mago.

          Under date of November 15, 1966, Remedios Mago filed an amended


petition in Civil Case No. 67496, including as party defendants Collector of
Customs Pedro Pacis of the Port of Manila and Lt. Martin Alagao of the Manila
Police Department. Herein petitioners (defendants below) filed, on November 24,
1966, their "Answer with Opposition to the Issuance of a Writ of Preliminary
Injunction", denying the alleged illegality of the seizure and detention of the goods
and the trucks and of their other actuations, and alleging special and affirmative
defenses, to wit: that the Court of First Instance of Manila had no jurisdiction to try
the case; that the case fell within the exclusive jurisdiction of the Court of Tax
Appeals; that, assuming that the court had jurisdiction over the case, the petition
stated no cause of action in view of the failure of Remedios Mago to exhaust the
administrative remedies provided for in the Tariff and Customs Code; that the
Bureau of Customs had not lost jurisdiction over the goods because the full duties
and charges thereon had not been paid; that the members of the Manila Police
Department had the power to make the seizure; that the seizure was not
unreasonable; and the persons deputized under Section 2203 (c) of the Tariff and
Customs Code could effect search, seizures and arrests in inland places in
connection with the enforcement of the said Code. In opposing the issuance of the
writ of preliminary injunction, herein petitioners averred in the court below that the
writ could not be granted for the reason that Remedios Mago was not entitled to
the main reliefs she prayed for; that the release of the goods, which were subject to
seizure proceedings under the Tariff and Customs Code, would deprive the Bureau
of Customs of the authority to forfeit them; and that Remedios Mago and Valentin
Lanopa would not suffer irreparable injury. Herein petitioners prayed the court
below for the lifting of the restraining order, for the denial of the issuance of the
writ of preliminary injunction, and for the dismissal of the case.

          At the hearing on December 9, 1966, the lower Court, with the conformity of
the parties, ordered that an inventory of the goods be made by its clerk of court in
the presence of the representatives of the claimant of the goods, the Bureau of
Customs, and the Anti-Smuggling Center of the Manila Police Department. On
December 13, 1966, the above-named persons filed a "Compliance" itemizing the
contents of the nine bales.

          Herein respondent Remedios Mago, on December 23, 1966, filed an ex


parte motion to release the goods, alleging that since the inventory of the goods
seized did not show any article of prohibited importation, the same should be
released as per agreement of the patties upon her posting of the appropriate bond
that may be determined by the court. Herein petitioners filed their opposition to the
motion, alleging that the court had no jurisdiction to order the release of the goods
in view of the fact that the court had no jurisdiction over the case, and that most of
the goods, as shown in the inventory, were not declared and were, therefore,
subject to forfeiture. A supplemental opposition was filed by herein petitioners on
January 19, 1967, alleging that on January 12, 1967 seizure proceedings against the
goods had been instituted by the Collector of Customs of the Port of Manila, and
the determination of all questions affecting the disposal of property proceeded
against in seizure and forfeiture proceedings should thereby be left to the Collector
of Customs. On January 30, 1967, herein petitioners filed a manifestation that the
estimated duties, taxes and other charges due on the goods amounted to
P95,772.00. On February 2, 1967, herein respondent Remedios Mago filed an
urgent manifestation and reiteration of the motion for the release under bond of the
goods.

          On March 7, 1967, the respondent Judge issued an order releasing the goods
to herein respondent Remedios Mago upon her filing of a bond in the amount of
P40,000.00, and on March 13, 1967, said respondent filed the corresponding bond.

          On March 13, 1967, herein petitioner Ricardo Papa, on his own behalf, filed
a motion for reconsideration of the order of the court releasing the goods under
bond, upon the ground that the Manila Police Department had been directed by the
Collector of Customs of the Port of Manila to hold the goods pending termination
of the seizure proceedings.

          Without waiting for the court's action on the motion for reconsideration, and
alleging that they had no plain, speedy and adequate remedy in the ordinary course
of law, herein petitioners filed the present action for prohibition and certiorari with
preliminary injunction before this Court. In their petition petitioners alleged,
among others, that the respondent Judge acted without jurisdiction in ordering the
release to respondent Remedios Mago of the disputed goods, for the following
reasons: (1) the Court of First Instance of Manila, presided by respondent Judge,
had no jurisdiction over the case; (2) respondent Remedios Mago had no cause of
action in Civil Case No. 67496 of the Court of First Instance of Manila due to her
failure to exhaust all administrative remedies before invoking judicial intervention;
(3) the Government was not estopped by the negligent and/or illegal acts of its
agent in not collecting the correct taxes; and (4) the bond fixed by respondent
Judge for the release of the goods was grossly insufficient.

          In due time, the respondents filed their answer to the petition for prohibition
and certiorari in this case. In their answer, respondents alleged, among others: (1)
that it was within the jurisdiction of the lower court presided by respondent Judge
to hear and decide Civil Case No. 67496 and to issue the questioned order of
March 7, 1967, because said Civil Case No. 67496 was instituted long before
seizure, and identification proceedings against the nine bales of goods in question
were instituted by the Collector of Customs; (2) that petitioners could no longer go
after the goods in question after the corresponding duties and taxes had been paid
and said goods had left the customs premises and were no longer within the control
of the Bureau of Customs; (3) that respondent Remedios Mago was purchaser in
good faith of the goods in question so that those goods can not be the subject of
seizure and forfeiture proceedings; (4) that the seizure of the goods was affected by
members of the Manila Police Department at a place outside control of jurisdiction
of the Bureau of Customs and affected without any search warrant or a warrant of
seizure and detention; (5) that the warrant of seizure and detention subsequently
issued by the Collector of Customs is illegal and unconstitutional, it not being
issued by a judge; (6) that the seizing officers have no authority to seize the goods
in question because they are not articles of prohibited importation; (7) that
petitioners are estopped to institute the present action because they had agreed
before the respondent Judge that they would not interpose any objection to the
release of the goods under bond to answer for whatever duties and taxes the said
goods may still be liable; and (8) that the bond for the release of the goods was
sufficient.

          The principal issue in the instant case is whether or not, the respondent
Judge had acted with jurisdiction in issuing the order of March 7, 1967 releasing
the goods in question.

          The Bureau of Customs has the duties, powers and jurisdiction, among
others, (1) to assess and collect all lawful revenues from imported articles, and all
other dues, fees, charges, fines and penalties, accruing under the tariff and customs
laws; (2) to prevent and suppress smuggling and other frauds upon the customs;
and (3) to enforce tariff and customs laws. 1 The goods in question were imported
from Hongkong, as shown in the "Statement and Receipts of Duties Collected on
Informal Entry". 2 As long as the importation has not been terminated the imported
goods remain under the jurisdiction of the Bureau of customs. Importation is
deemed terminated only upon the payment of the duties, taxes and other charges
upon the articles, or secured to be paid, at the port of entry and the legal permit for
withdrawal shall have been granted. 3 The payment of the duties, taxes, fees and
other charges must be in full. 4

          The record shows, by comparing the articles and duties stated in the
aforesaid "Statement and Receipts of Duties Collected on Informal Entry" with the
manifestation of the Office of the Solicitor General  5 wherein it is stated that the
estimated duties, taxes and other charges on the goods subject of this case
amounted to P95,772.00 as evidenced by the report of the appraiser of the Bureau
of Customs, that the duties, taxes and other charges had not been paid in full.
Furthermore, a comparison of the goods on which duties had been assessed, as
shown in the "Statement and Receipts of Duties Collected on Informal Entry" and
the "compliance" itemizing the articles found in the bales upon examination and
inventory, 6 shows that the quantity of the goods was underdeclared, presumably to
avoid the payment of duties thereon. For example, Annex B (the statement and
receipts of duties collected) states that there were 40 pieces of ladies' sweaters,
whereas Annex H (the inventory contained in the "compliance") states that in bale
No. 1 alone there were 42 dozens and 1 piece of ladies' sweaters of assorted colors;
in Annex B, only 100 pieces of watch bands were assessed, but in Annex H, there
were in bale No. 2, 209 dozens and 5 pieces of men's metal watch bands (white)
and 120 dozens of men's metal watch band (gold color), and in bale No. 7, 320
dozens of men's metal watch bands (gold color); in Annex B, 20 dozens only of
men's handkerchief were declared, but in Annex H it appears that there were 224
dozens of said goods in bale No. 2, 120 dozens in bale No. 6, 380 dozens in bale
No. 7, 220 dozens in bale No. 8, and another 200 dozens in bale No. 9. The articles
contained in the nine bales in question, were, therefore, subject to forfeiture under
Section 2530, pars. e and m, (1), (3), (4), and (5) of the Tariff and Customs Code.
And this Court has held that merchandise, the importation of which is effected
contrary to law, is subject to forfeiture, 7 and that goods released contrary to law are
subject to seizure and forfeiture. 8

          Even if it be granted, arguendo, that after the goods in question had been


brought out of the customs area the Bureau of Customs had lost jurisdiction over
the same, nevertheless, when said goods were intercepted at the Agrifina Circle on
November 4, 1966 by members of the Manila Police Department, acting under
directions and orders of their Chief, Ricardo C. Papa, who had been formally
deputized by the Commissioner of Customs, 9 the Bureau of Customs had regained
jurisdiction and custody of the goods. Section 1206 of the Tariff and Customs
Code imposes upon the Collector of Customs the duty to hold possession of all
imported articles upon which duties, taxes, and other charges have not been paid or
secured to be paid, and to dispose of the same according to law. The goods in
question, therefore, were under the custody and at the disposal of the Bureau of
Customs at the time the petition for mandamus, docketed as Civil Case No. 67496,
was filed in the Court of First Instance of Manila on November 9, 1966. The Court
of First Instance of Manila, therefore, could not exercise jurisdiction over said
goods even if the warrant of seizure and detention of the goods for the purposes of
the seizure and forfeiture proceedings had not yet been issued by the Collector of
Customs.

          The ruling in the case of "Alberto de Joya, et al. v. Hon. Gregorio Lantin, et
al.," G.R. No. L-24037, decided by this Court on April 27, 1967, is squarely
applicable to the instant case. In the De Joya case, it appears that Francindy
Commercial of Manila bought from Ernerose Commercial of Cebu City 90 bales of
assorted textiles and rags, valued at P117,731.00, which had been imported and
entered thru the port of Cebu. Ernerose Commercial shipped the goods to Manila
on board an inter-island vessel. When the goods where about to leave the customs
premises in Manila, on October 6, 1964, the customs authorities held them for
further verification, and upon examination the goods were found to be different
from the declaration in the cargo manifest of the carrying vessel. Francindy
Commercial subsequently demanded from the customs authorities the release of
the goods, asserting that it is a purchaser in good faith of those goods; that a local
purchaser was involved so the Bureau of Customs had no right to examine the
goods; and that the goods came from a coastwise port. On October 26, 1964,
Francindy Commercial filed in the Court of First Instance of Manila a petition
for mandamus against the Commissioner of Customs and the Collector of Customs
of the port of Manila to compel said customs authorities to release the goods.

          Francindy Commercial alleged in its petition for mandamus that the Bureau


of Customs had no jurisdiction over the goods because the same were not imported
to the port of Manila; that it was not liable for duties and taxes because the
transaction was not an original importation; that the goods were not in the hands of
the importer nor subject to importer's control, nor were the goods imported
contrary to law with its (Francindy Commercial's) knowledge; and that the
importation had been terminated. On November 12, 1964, the Collector of
Customs of Manila issued a warrant of seizure and identification against the goods.
On December 3, 1964, the Commissioner of Customs and the Collector of
Customs, as respondents in the mandamus case, filed a motion to dismiss the
petition on the grounds of lack of jurisdiction, lack of cause of action, and in view
of the pending seizure and forfeiture proceedings. The Court of First Instance held
resolution on the motion to dismiss in abeyance pending decision on the merits. On
December 14, 1964, the Court of First Instance of Manila issued a preventive and
mandatory injunction, on prayer by Francindy Commercial, upon a bond of
P20,000.00. The Commissioner of Customs and the Collector of Customs sought
the lifting of the preliminary and mandatory injunction, and the resolution of their
motion to dismiss. The Court of First Instance of Manila, however, on January 12,
1965, ordered them to comply with the preliminary and mandatory injunction,
upon the filing by Francindy Commercial of an additional bond of P50,000.00.
Said customs authorities thereupon filed with this Court, on January 14, 1965, a
petition for certiorari and prohibition with preliminary injunction. In resolving the
question raised in that case, this Court held:

          This petition raises two related issues: first, has the Customs bureau
jurisdiction to seize the goods and institute forfeiture proceedings against
them? and (2) has the Court of First Instance jurisdiction to entertain the
petition for mandamus to compel the Customs authorities to release the
goods?

          Francindy Commercial contends that since the petition in the Court of


first Instance was filed (on October 26, 1964) ahead of the issuance of the
Customs warrant of seizure and forfeiture (on November 12, 1964),the
Customs bureau should yield the jurisdiction of the said court.

          The record shows, however, that the goods in question were actually


seized on October 6, 1964, i.e., before Francindy Commercial sued in court.
The purpose of the seizure by the Customs bureau was to verify whether or
not Custom duties and taxes were paid for their importation. Hence, on
December 23, 1964, Customs released 22 bales thereof, for the same were
found to have been released regularly from the Cebu Port (Petition Annex
"L"). As to goods imported illegally or released irregularly from Customs
custody, these are subject to seizure under Section 2530 m. of the Tariff and
Customs Code (RA 1957).

          The Bureau of Customs has jurisdiction and power, among others to


collect revenues from imported articles, fines and penalties and suppress
smuggling and other frauds on customs; and to enforce tariff and customs
laws (Sec. 602, Republic Act 1957).

          The goods in question are imported articles entered at the Port of


Cebu. Should they be found to have been released irregularly from Customs
custody in Cebu City, they are subject to seizure and forfeiture, the
proceedings for which comes within the jurisdiction of the Bureau of
Customs pursuant to Republic Act 1937.

          Said proceeding should be followed; the owner of the goods may set
up defenses therein (Pacis v. Averia, L-22526, Nov. 20, 1966.) From the
decision of the Commissioner of Customs appeal lies to the Court of Tax
Appeals, as provided in Sec. 2402 of Republic Act 1937 and Sec. 11 of
Republic Act, 1125. To permit recourse to the Court of First Instance in
cases of seizure of imported goods would in effect render ineffective the
power of the Customs authorities under the Tariff and Customs Code and
deprive the Court of Tax Appeals of one of its exclusive appellate
jurisdictions. As this Court has ruled in Pacis v. Averia, supra, Republic
Acts 1937 and 1125 vest jurisdiction over seizure and forfeiture proceedings
exclusively upon the Bureau of Customs and the Court of Tax Appeals.
Such law being special in nature, while the Judiciary Act defining the
jurisdiction of Courts of First Instance is a general legislation, not to
mention that the former are later enactments, the Court of First Instance
should yield to the jurisdiction of the Customs authorities.

          It is the settled rule, therefore, that the Bureau of Customs acquires exclusive
jurisdiction over imported goods, for the purposes of enforcement of the customs
laws, from the moment the goods are actually in its possession or control, even if
no warrant of seizure or detention had previously been issued by the Collector of
Customs in connection with seizure and forfeiture proceedings. In the present case,
the Bureau of Customs actually seized the goods in question on November 4, 1966,
and so from that date the Bureau of Customs acquired jurisdiction over the goods
for the purposes of the enforcement of the tariff and customs laws, to the exclusion
of the regular courts. Much less then would the Court of First Instance of Manila
have jurisdiction over the goods in question after the Collector of Customs had
issued the warrant of seizure and detention on January 12, 1967. 10 And so, it
cannot be said, as respondents contend, that the issuance of said warrant was only
an attempt to divest the respondent Judge of jurisdiction over the subject matter of
the case. The court presided by respondent Judge did not acquire jurisdiction over
the goods in question when the petition for mandamus was filed before it, and so
there was no need of divesting it of jurisdiction. Not having acquired jurisdiction
over the goods, it follows that the Court of First Instance of Manila had no
jurisdiction to issue the questioned order of March 7, 1967 releasing said goods.

          Respondents also aver that petitioner Martin Alagao, an officer of the Manila
Police Department, could not seize the goods in question without a search warrant.
This contention cannot be sustained. The Chief of the Manila Police Department,
Ricardo G. Papa, having been deputized in writing by the Commissioner of
Customs, could, for the purposes of the enforcement of the customs and tariff laws,
effect searches, seizures, and arrests, 11 and it was his duty to make seizure, among
others, of any cargo, articles or other movable property when the same may be
subject to forfeiture or liable for any fine imposed under customs and tariff
laws. 12 He could lawfully open and examine any box, trunk, envelope or other
container wherever found when he had reasonable cause to suspect the presence
therein of dutiable articles introduced into the Philippines contrary to law; and
likewise to stop, search and examine any vehicle, beast or person reasonably
suspected of holding or conveying such article as aforesaid. 13 It cannot be doubted,
therefore, that petitioner Ricardo G. Papa, Chief of Police of Manila, could
lawfully effect the search and seizure of the goods in question. The Tariff and
Customs Code authorizes him to demand assistance of any police officer to effect
said search and seizure, and the latter has the legal duty to render said
assistance. 14 This was what happened precisely in the case of Lt. Martin Alagao
who, with his unit, made the search and seizure of the two trucks loaded with the
nine bales of goods in question at the Agrifina Circle. He was given authority by
the Chief of Police to make the interception of the cargo. 15

          Petitioner Martin Alagao and his companion policemen had authority to


effect the seizure without any search warrant issued by a competent court. The
Tariff and Customs Code does not require said warrant in the instant case. The
Code authorizes persons having police authority under Section 2203 of the Tariff
and Customs Code to enter, pass through or search any land, inclosure, warehouse,
store or building, not being a dwelling house; and also to inspect, search and
examine any vessel or aircraft and any trunk, package, or envelope or any person
on board, or to stop and search and examine any vehicle, beast or person suspected
of holding or conveying any dutiable or prohibited article introduced into the
Philippines contrary to law, without mentioning the need of a search warrant in
said cases. 16 But in the search of a dwelling house, the Code provides that said
"dwelling house may be entered and searched only upon warrant issued by a judge
or justice of the peace. . . ." 17 It is our considered view, therefor, that except in the
case of the search of a dwelling house, persons exercising police authority under
the customs law may effect search and seizure without a search warrant in the
enforcement of customs laws.
          Our conclusion finds support in the case of Carroll v. United States, 39
A.L.R., 790, 799, wherein the court, considering a legal provision similar to
Section 2211 of the Philippine Tariff and Customs Code, said as follows:

          Thus contemporaneously with the adoption of the 4th Amendment,


we find in the first Congress, and in the following second and fourth
Congresses, a difference made as to the necessity for a search warrant
between goods subject to forfeiture, when concealed in a dwelling house of
similar place, and like goods in course of transportation and concealed in a
movable vessel, where readily they could be put out of reach of a search
warrant. . . .

          Again, by the 2d section of the Act of March 3, 1815 (3 Stat. at L.231,


232, chap. 94), it was made lawful for customs officers not only to board
and search vessels within their own and adjoining districts, but also to stop,
search and examine any vehicle, beast or person on which or whom they
should suspect there was merchandise which was subject to duty, or had
been introduced into the United States in any manner contrary to law,
whether by the person in charge of the vehicle or beast or otherwise, and if
they should find any goods, wares, or merchandise thereon, which they had
probably cause to believe had been so unlawfully brought into the country,
to seize and secure the same, and the vehicle or beast as well, for trial and
forfeiture. This Act was renewed April 27, 1816 (3 Sta. at L. 315, chap.
100), for a year and expired. The Act of February 28, 1865, revived § 2 of
the Act of 1815, above described, chap. 67, 13 Stat. at L. 441. The substance
of this section was re-enacted in the 3d section of the Act of July 18, 1866,
chap. 201, 14 Stat. at L. 178, and was thereafter embodied in the Revised
Statutes as § 3061, Comp. Stat. § 5763, 2 Fed. Stat. Anno. 2d ed. p. 1161.
Neither § 3061 nor any of its earlier counterparts has ever been attacked as
unconstitutional. Indeed, that section was referred to and treated as operative
by this court in Von Cotzhausen v. Nazro, 107 U.S. 215, 219, 27 L. ed. 540,
541, 2 Sup. Ct. Rep. 503. . . .

          In the instant case, we note that petitioner Martin Alagao and his companion
policemen did not have to make any search before they seized the two trucks and
their cargo. In their original petition, and amended petition, in the court below
Remedios Mago and Valentin Lanopa did not even allege that there was a
search. 18 All that they complained of was,

          That while the trucks were on their way, they


were intercepted without any search warrant near the Agrifina Circle and
taken to the Manila Police Department, where they were detained.
          But even if there was a search, there is still authority to the effect that no
search warrant would be needed under the circumstances obtaining in the instant
case. Thus, it has been held that:

          The guaranty of freedom from unreasonable searches and seizures is


construed as recognizing a necessary difference between a search of a
dwelling house or other structure in respect of which a search warrant may
readily be obtained and a search of a ship, motorboat, wagon, or automobile
for contraband goods, where it is not practicable to secure a warrant because
the vehicle can be quickly moved out of the locality or jurisdiction in which
the warrant must be sought. (47 Am. Jur., pp. 513-514, citing Carroll v.
United States, 267 U.S. 132, 69 L. ed., 543, 45 S. Ct., 280, 39 A.L.R., 790;
People v. Case, 320 Mich., 379, 190 N.W., 389, 27 A.L.R., 686.)

          In the case of People v. Case (320 Mich., 379, 190 N.W., 389, 27 A.L.R.,
686), the question raised by defendant's counsel was whether an automobile truck
or an automobile could be searched without search warrant or other process and the
goods therein seized used afterwards as evidence in a trial for violation of the
prohibition laws of the State. Same counsel contended the negative, urging the
constitutional provision forbidding unreasonable searches and seizures. The Court
said:

          . . . Neither our state nor the Federal Constitution directly prohibits


search and seizure without a warrant, as is sometimes asserted. Only
"unreasonable" search and seizure is forbidden. . . .

          . . . The question whether a seizure or a search is unreasonable in the


language of the Constitution is a judicial and not a legislative question; but
in determining whether a seizure is or is not unreasonable, all of the
circumstances under which it is made must be looked to.

          The automobile is a swift and powerful vehicle of recent


development, which has multiplied by quantity production and taken
possession of our highways in battalions until the slower, animal-drawn
vehicles, with their easily noted individuality, are rare. Constructed as
covered vehicles to standard form in immense quantities, and with a
capacity for speed rivaling express trains, they furnish for successful
commission of crime a disguising means of silent approach and swift escape
unknown in the history of the world before their advent. The question of
their police control and reasonable search on highways or other public
places is a serious question far deeper and broader than their use in so-called
"bootleging" or "rum running," which is itself is no small matter. While a
possession in the sense of private ownership, they are but a vehicle
constructed for travel and transportation on highways. Their active use is not
in homes or on private premises, the privacy of which the law especially
guards from search and seizure without process. The baffling extent to
which they are successfully utilized to facilitate commission of crime of all
degrees, from those against morality, chastity, and decency, to robbery, rape,
burglary, and murder, is a matter of common knowledge. Upon that problem
a condition, and not a theory, confronts proper administration of our
criminal laws. Whether search of and seizure from an automobile upon a
highway or other public place without a search warrant is unreasonable is in
its final analysis to be determined as a judicial question in view of all the
circumstances under which it is made.

          Having declared that the seizure by the members of the Manila Police
Department of the goods in question was in accordance with law and by that
seizure the Bureau of Customs had acquired jurisdiction over the goods for the
purpose of the enforcement of the customs and tariff laws, to the exclusion of the
Court of First Instance of Manila, We have thus resolved the principal and decisive
issue in the present case. We do not consider it necessary, for the purposes of this
decision, to discuss the incidental issues raised by the parties in their pleadings.

          WHEREFORE, judgment is hereby rendered, as follows:

          (a) Granting the writ of certiorari and prohibition prayed for by petitioners;

          (b) Declaring null and void, for having been issued without jurisdiction, the
order of respondent Judge Hilarion U. Jarencio, dated March 7, 1967, in Civil
Code No. 67496 of the Court of First Instance of Manila;

          (c) Declaring permanent the preliminary injunction issued by this Court on


March 31, 1967 restraining respondent Judge from executing, enforcing and/or
implementing his order of March 7, 1967 in Civil Case No. 67496 of the Court of
First Instance of Manila, and from proceeding in any manner in said case;

          (d) Ordering the dismissal of Civil Case No. 67496 of the Court of First
Instance of Manila; and1äwphï1.ñët

          (e) Ordering the private respondent, Remedios Mago, to pay the costs.

          It is so ordered.

Concepcion, C.J., Reyes, J.B.L., Dizon, Makalintal, Bengzon, J.P., Sanchez,


Castro, Angeles and Fernando, JJ., concur.1äwphï1.ñët

CASES:
People v. Salanguit, supra
People v. Musa, G.R. No. 96177 January 27, 1993
Issue: W/N the seizure of the plastic bag and the marijuana inside it is
unreasonable, hence, inadmissible as evidence.

Ruling: Yes, the seizure of the plastic bag and the marijuana inside it is
unreasonable, hence, inadmissible as evidence.

Rule 126, Section 12 of the Rules of Court expressly authorizes a warrantless


search and seizure incident to a lawful arrest, thus:

Sec. 12. Search incident to lawful arrest. — A person lawfully arrested may be
searched for dangerous weapons or anything which may be used as proof of the
commission of an offense, without a search warrant.

The warrantless search and seizure, as an incident to a suspect's lawful arrest, may
extend beyond the person of the one arrested to include the premises or
surroundings under his immediate control. Objects in the "plain view" of an officer
who has the right to be in the position to have that view are subject to seizure and
may be presented as evidence.

In the case of Ker v. California, the Court held that it was not unreasonable for the
officer to walk to the doorway of the adjacent kitchen on seeing the defendant wife
emerge therefrom, that "the discovery of the brick of marijuana did not constitute a
search, since the officer merely saw what was placed before him in full view. The
U.S. Supreme Court ruled that the warrantless seizure of the marijuana was legal
on the basis of the "plain view" doctrine and upheld the admissibility of the
seized drugs as part of the prosecution's evidence. 
However, the “plain view” doctrine is not absolute. It cannot be used to launch
unbridled searches and indiscriminate seizures nor to extend a general exploratory
search made solely to find evidence of a defendant's guilt. The "plain view"
doctrine is usually applied where a police officer is not searching for evidence
against the accused, but nonetheless inadvertently comes across an incriminating
object.

What the "plain view" cases have in common is that the police officer in each of
them had a prior justification for an intrusion in the course of which he came
inadvertently across a piece of evidence incriminating the accused. The doctrine
serves to supplement the prior justification — whether it be a warrant for another
object, hot pursuit, search incident to lawful arrest, or some other legitimate
reason for being present unconnected with a search directed against the accused
— and permits the warrantless seizure. Of course, the extension of the original
justification is legitimate only where it is immediately apparent to the police that
they have evidence before them; the "plain view" doctrine may not be used to
extend a general exploratory search from one object to another until something
incriminating at last emerges.
In the instant case, the appellant was arrested and his person searched in the living
room. Failing to retrieve the marked money which they hoped to find, the
NARCOM agents searched the whole house and found the plastic bag in the
kitchen. The plastic bag was, therefore, not within their "plain view" when they
arrested the appellant as to justify its seizure. The NARCOM agents had to move
from one portion of the house to another before they sighted the plastic bag. Unlike
Ker vs. California, where the police officer had reason to walk to the doorway of
the adjacent kitchen and from which position he saw the marijuana, the NARCOM
agents in this case went from room to room with the obvious intention of fishing
for more evidence.
Moreover, when the NARCOM agents saw the plastic bag hanging in one corner
of the kitchen, they had no clue as to its contents. They had to ask the appellant
what the bag contained. When the appellant refused to respond, they opened it and
found the marijuana.
Even assuming then, that the NARCOM agents inadvertently came across the
plastic bag because it was within their "plain view," what may be said to be the
object in their "plain view" was just the plastic bag and not the marijuana. The
incriminating nature of the contents of the plastic bag was not immediately
apparent from the "plain view" of said object. It cannot be claimed that the plastic
bag clearly betrayed its contents, whether by its distinctive configuration, its
transparency, or otherwise, that its contents are obvious to an observer.
We, therefore, hold that under the circumstances of the case, the "plain view"
doctrine does not apply and the marijuana contained in the plastic bag was
seized illegally and cannot be presented in evidence pursuant to Article III,
Section 3(2) of the Constitution.
The exclusion of this particular evidence does not, however, diminish, in any way,
the damaging effect of the other pieces of evidence presented by the prosecution to
prove that the appellant sold marijuana, in violation of Article II, Section 4 of the
Dangerous Drugs Act of 1972. We hold that by virtue of the testimonies of Sgt.
Ani and T/Sgt. Belarga and the two wrappings of marijuana sold by the appellant
to Sgt. Ani, among other pieces of evidence, the guilt of the appellant of the crime
charged has been proved beyond reasonable doubt.

CASES:
People v. Exala, G.R. No. 76005, April 23, 1993 (Read also the dissent of J.
Cruz)

The admissibility of the evidence seized from the accused at a checkpoint after
being stopped for routine inspection is put to test in this appeal from the decision 1
of the Regional Trial Court of Cavite City finding inter alia accused-appellant
Restituto B. Bocalan guilty beyond reasonable doubt of violating Sec. 4, Art. II, of
R.A. 6425, as amended, otherwise known as "The Dangerous Drugs Act of 1972."

On 2 November 1982, at about 8:15 in the evening, a private jeep driven by


accused-appellant Restituto B. Bocalan was stopped at a police checkpoint in
Cavite City for routine inspection regarding unlicensed firearms and other
prohibited items. With Bocalan were his co-accused Jaime P. Fernandez and
Rodelio C. Exala. Pfc. Ricardo Galang, a member of the inspection team, went
near the jeep and asked the occupants if there were firearms inside. They answered
in the negative. Pfc. Galang then proceeded to inspect the vehicle by beaming a
flashlight inside. He noticed a black leather bag measuring about one (1) foot wide
and two (2) feet long with its sides bulging. He asked what it contained. There was
deadening silence. Nobody answered. Instead, the three (3) accused, Restituto B.
Bocalan, Jaime P. Fernandez and Rodelio C. Exala, suddenly became fidgety.
Suspicious, Pfc. Galang ordered the bag opened. He found what he excitedly
described as "marijuana, marijuana, napakaraming marijuana!" At this juncture, the
three (3) remained motionless in their seats and appeared petrified with fear. They
were brought to the police station that same night for further investigation. 2

After laboratory examination, the bag was verified to contain more than two (2)
kilos of Indian hemp otherwise known as marijuana. 3

Thereafter, Rodelio C. Exala, Restituto B. Bocalan and Jaime P. Fernandez were


accordingly charged for violation of Sec. 4, Art. II, of R.A. 6425, as amended.

After trial, Bocalan was held guilty as principal and sentenced to life
imprisonment. A fine of P25,000.00 was also imposed. 4 The other two (2) were
convicted as accomplices and received lighter penalties. Fernandez appealed to the
Court of Appeals. Exala did not.

Bocalan, whose punishment is reviewable only by this Court, is now before Us


assailing his conviction; hence, We deal only with him in this appeal.

Appellant Bocalan seeks exculpation by imputing ownership of the bag to Exala


alone. 5 Bocalan claims that while on the way to Cavite City, he and Fernandez
offered Exala a ride. Exala accepted the offer and requested Bocalan to make a
detour to Salitran, Dasmariñas, Cavite, where he was to pick up some clothes.
They agreed and Exala got the bag which he kept beside him all the time until their
apprehension at the checkpoint. 6

Bocalan further contends that the trial court erred in admitting the bag as evidence
against him since it was obtained through a warrantless search. 7

The protestations of Bocalan are devoid of merit. We agree with the trial court that
the conduct of Bocalan was not only unusual but also contrary to normal human
experience. 8 He alleged that he knew Exala only by face and had no personal
association with him; 9 yet, on that eventful day of 2 November 1982, he agreed to
detour to Salitran which was some fifteen (15) to twenty (20) kilometers out of his
way. Thus, his contention that it was Exala who owned the bag containing the
marijuana is hardly credible.

On the other hand, Exala declared that it was he who did not know the contents of
the bag as it was already in the jeep when he boarded it. Exala asserted that it was
either Bocalan or Fernandez who owned the bag. Exala swore that Bocalan and
Fernandez offered him P5,000.00, later raised to P10,000.00, to take the blame
alone, but he refused. 10

Proof of ownership is immaterial where the accused is charged with the unlawful
transportation of marijuana. 11 Section 4, Art. II, of R.A. 6425, as amended, does
not require that one be the owner of the prohibited drug before he can be
prosecuted for dispatching in transit or transporting a prohibited drug. The law
simply provides thus —

"Sec. 4. Sale, Administration, Delivery, Distribution and Transportation of


Prohibited Drugs. — The penalty of life imprisonment to death and a fine ranging
from twenty thousand to thirty thousand pesos shall be imposed upon any person
who, unless authorized by law, shall sell, administer, deliver, give away to another,
distribute, dispatch in transit or transport any prohibited drug, or shall act as a
broker in any of such transactions. If the victim of the offense is a minor, or should
a prohibited drug involved in any offense under this Section be the proximate
cause of the death of a victim thereof, the maximum penalty herein provided shall
be imposed."

Nonetheless, there is substantial evidence to prove that Bocalan was directly


involved in the unlawful dispatch in transit or transport of marijuana. The evidence
of the prosecution, particularly the testimonies of Pfc. Ricardo Galang and Pat.
Rosauro de Guzman, belies the defense of Bocalan and establishes beyond cavil
that he was caught in flagrante delicto of transporting the prohibited drug; that he
was the driver of the jeep owned by his father that carried the stuff; and, that he
was in fact the owner of the bag. The trial court noted that Bocalan picked up
Fernandez and Exala one after the other to accompany him to the place where the
bag of marijuana was taken and to help him bring the marijuana to Cavite City. 12
Regardless of the degree of participation of Fernandez and Exala, Bocalan is
correctly punished for his direct involvement in the crime.

Such factual conclusions by the trial court relative to the credibility of witnesses
are entitled to great respect and are generally sustained by the appellate court
unless some material facts have been overlooked or misconstrued as to affect the
result. 13 There is none in this case on appeal.
We turn to the legal question on the admissibility of the marijuana as evidence in
the light of Bocalan's contention that it was seized without a valid search warrant.
Since the search was conducted prior to the arrest, Bocalan argues that it was not
incident to a lawful arrest.

This issue was never raised in the proceedings below. Bocalan never objected to
the admissibility of the evidence on the ground that the same was obtained in a
warrantless search. Consequently, he is deemed to have waived his objection on
the legality of the search and the admissibility of the evidence obtained in the
course thereof. 14 In view of such waiver, the court is bound to admit the evidence.
15 But even assuming arguendo that there was no waiver, still appellant's
contention deserves scant consideration.

There are indeed instances where search and seizure can be effected without
necessarily being preceded by an arrest. 16 An illustration would be the "stop-and-
search" without a warrant at military or police checkpoints, the constitutionality of
which has already been upheld by this Court. 17 Vehicles are generally allowed to
pass through these checkpoints after a routine inspection and answering a few
questions. If vehicles are stopped and extensively searched it is because of some
probable cause which justifies a reasonable belief of those manning the
checkpoints that either the motorist is a law-offender or the contents of the vehicle
are or have been instruments in the commission of an offense. 18 However, lest it
be misunderstood, this doctrine is not intended to do away with the general rule
that no person shall be subjected to search of his person, personal effects and
belongings, or his residence except of virtue of a search warrant or on the occasion
of a lawful arrest. 19 The case before Us is an incident to or an offshoot of a lawful
"stop-and-search" at a military or police checkpoint.

The checkpoint in the instant case was established in line with "Operational Bakal"
the main object of which was to search for unlicensed firearms and other
prohibited items in the possession of unauthorized persons passing through it. 20
When the jeep carrying the contraband passed through the checkpoint, it was
flagged down and the occupants were asked routine questions. In the course
thereof, Pfc. Galang noticed a black leather bag the sides of which were bulging.
He asked what the contents of the bag were. None of the accused answered. At that
moment, the demeanor of the accused changed; they became suspiciously quiet
and nervous as if they were concealing something from Pfc. Galang. The accused
clearly appeared to be in abject fear of being discovered. Such peculiar
apprehensiveness if not restrained reaction of the accused, which did not appear
normal, provided the probable cause justifying a more extensive search that led to
the opening of the bag and the discovery of the prohibited stuff. Significantly, there
was no sign of any protest or objection to the search. The accused remained silent
even after their arrest.
Their submissive stance after the discovery of the bag of marijuana, as well as the
absence of any protest on their part when arrested, not only casts serious doubts on
their professed innocence 21 but also confirms their acquiescence to the search. 22
Clearly then, there was waiver of the right against unreasonable search and seizure.
23 In one case 24 We held —

". . . When one voluntarily submits to a search or consents to have it made of his
person or premises, he is precluded from later complaining thereof (Cooley,
Constitutional Limitations, 8th Ed., Vol. I, p. 361). The right to be secure from
unreasonable search and seizure may, like every right, be waived and such waiver
may be made either expressly or impliedly" (emphasis supplied).

The arrest of the three (3) accused was lawful because it was made upon the
discovery of the prohibited drug in their possession. There was no need for a
warrant; the arrest was made while a crime was committed. This is one of the
situations envisioned by Sec. 5, par. (a), of Rule 113 of the 1985 Rules on Criminal
Procedure, as amended, when a warrantless arrest may be made. 25 The accused
were caught in the act of dispatching in transit or transporting marijuana, in
violation of Sec. 4, Art. II, of R.A. 6425, as amended.

The alleged contradiction between the sworn statements of Pfc. Galang and Pat. de
Guzman was explained in their separate testimonies and, in any event, has been
resolved by the trial court as a factual issue. We find no reason to reverse its
findings.

Anent the argument that the three (3) accused should not have been assigned
different levels of liability, suffice it to say that whether a principal, co-principal or
conspirator, accused-appellant would have been meted out the same penalty
imposed by the trial court.

WHEREFORE, there being no reversible error in the decision appealed from


finding accused-appellant RESTITUTO B. BOCALAN guilty beyond reasonable
doubt of the crime charged, the same is AFFIRMED, with costs against him.

SO ORDERED.

Griño-Aquino and Quiason, JJ ., concur.

Separate Opinions

CRUZ, J., dissenting:

I dissent, for the reasons expressed in my dissenting opinions in Valmonte v. de


Villa, 185 SCRA 665/178 SCRA 211, and People v. Malmstedt, 198 SCRA 401,
and the following additional observations.
I am opposed to checkpoints as regular police measures aimed at reducing
criminality in general. I do not agree that in the interest of peace and order, any or
every vehicle may be stopped at any time by the authorities and searched without
warrant on the chance that it may be carrying prohibited articles. That possibility is
not the probable cause envisioned in the Bill of Rights.

In the case of the ordinary checkpoint, there is not even any suspicion to justify the
search. The search is made as a matter of course, either of all vehicles or at
random. There is no showing that a crime is about to be committed, is actually
being committed, or has just been committed and the searching officer has personal
knowledge that the person being searched or arrested is the culprit.

I will concede that checkpoints may be established at borders of states or at


"constructive borders" near the boundary for the purpose of preventing violations
of immigration and customs laws. But in the interior of the territory, the
requirements of a valid search and seizure must be strictly observed. The only
permissible exemption is where a crime like a bank robbery has just been
committed or a jailbreak has just occurred, and the authorities have to seal off all
possible avenues of escape in the area. In all other cases, I submit that the
checkpoint should not be allowed.

I realize that this view would result in the inadmissibility of the of the seized
marijuana as evidence against the petitioner and in his inevitable acquittal. But as I
have always maintained, we cannot retroactively validate an illegal search on the
justification that, after all, the articles seized are illegal. That is putting the cart
before the horse. I would rather see some criminals go unpunished now and then
than agree to the Bill of Rights being systematically ignored in the oppressive
checkpoint. Respect for the Constitution is more important than securing a
conviction based on a violation of the rights of the accused.

People v. Lo Ho Wing, G.R. No. 88017, January 21, 1991


Issue: May a judge determine probable cause and issue a warrant of arrest solely
on the basis of the resolution of the prosecutor (in the instant case, the Office of the
Special Prosecutor of the Ombudsman) who conducted the preliminary
investigation, without having before him any of the evidence (such as
complainant’s affidavit, respondent’s counter-affidavit, exhibits, etc.) which may
have been submitted at the preliminary investigation?

Ruling: No. 

Art III Section 2, 1987 Constitution: The right of the people to be secure in their
persons, houses, papers, and effects against unreasonable searches and seizures of
whatever nature and for any purpose shall be inviolable, and no search warrant or
warrant of arrest shall issue except upon probable cause to be determined
personally by the judge after examination under oath or affirmation of the
complainant and the witnesses he may produce and particularly describing the
place to be searched and the persons or things to be seized.

The word “personally” does not appear in the corresponding provisions of our
previous Constitutions. This emphasis shows the present Constitution’s intent to
place a greater degree of responsibility upon trial judges than that imposed under
the previous Charters. 

Soliven vs. Makasiar: “In satisfying himself of the existence of probable cause for
the issuance of a warrant of arrest, the judge is not required to personally examine
the complainant and his witnesses. Following established doctrine and procedure,
he shall: 
(1) personally evaluate the report and the supporting documents submitted by
the fiscal regarding the existence of probable cause and, on the basis thereof,
issue a warrant of arrest; or 
(2) if on the basis thereof he finds no probable cause, he may disregard the
fiscal’s report and require the submission of supporting affidavits of
witnesses to aid him in arriving at a conclusion as to the existence of
probable cause.” 

People vs. Inting: There is a difference between the judge’s goal from that of the
prosecutor’s.
 First, the determination of probable cause is a function of the Judge. It is
not for the Provincial Fiscal or Prosecutor or for the Election Supervisor to
ascertain. 
 Second, the preliminary inquiry made by a Prosecutor does not bind the
Judge. It merely assists him to make the determination of probable cause. 
 Third, Judges and Prosecutors alike should distinguish the preliminary
inquiry which determines probable cause for the issuance of a warrant of
arrest from the preliminary investigation proper which ascertains whether
the offender should be held for trial or released. Even if the two inquiries are
conducted in the course of one and the same proceeding, there should be no
confusion about the objectives. 

The Court, in this case, reiterated and elaborated on the doctrine laid down in
People vs. Inting and ruled that:

 “First, as held in Inting, the determination of probable cause by the


prosecutor is for a purpose different from that which is to be made by the
judge. Whether there is reasonable ground to believe that the accused is
guilty of the offense charged and should be held for trial is what the
prosecutor passes upon. The judge, on the other hand, determines whether a
warrant of arrest should be issued against the accused, i.e. whether there is a
necessity for placing him under immediate custody in order not to frustrate
the ends of justice. Thus, even if both should base their findings on one and
the same proceeding or evidence, there should be no confusion as to their
distinct objectives.
 Second, since their objectives are different, the judge cannot rely solely on
the report of the prosecutor in finding probable cause to justify the issuance
of a warrant of arrest. Obviously and understandably, the contents of the
prosecutor’s report will support his own conclusion that there is reason to
charge the accused of an offense and hold him for trial. However, the judge
must decide independently. Hence, he must have supporting evidence, other
than the prosecutor’s bare report, upon which to legally sustain his own
findings on the existence (or nonexistence) of probable cause to issue an
arrest order. This responsibility of determining personally and independently
the existence or nonexistence of probable cause is lodged in him by no less
than the most basic law of the land. Parenthetically, the prosecutor could
ease the burden of the judge and speed up the litigation process by
forwarding to the latter not only the information and his bare resolution
finding probable cause, but also so much of the records and the evidence on
hand as to enable His Honor to make his personal and separate judicial
finding on whether to issue a warrant of arrest.
 Lastly, it is not required that the complete or entire records of the case
during the preliminary investigation be submitted to and examined by the
judge. We do not intend to unduly burden trial courts by obliging them to
examine the complete records of every case all the time simply for the
purpose of ordering the arrest of an accused. What is required, rather, is that
the judge must have sufficient supporting documents (such as the complaint,
affidavits, counter-affidavits, sworn statements of witnesses or transcripts of
stenographic notes, if any) upon which to make his independent judgment
or, at the very least, upon which to verify the findings of the prosecutor as to
the existence of probable cause. The point is: he cannot rely solely and
entirely on the prosecutor’s recommendation, as the Respondent Court did
in this case. Although the prosecutor enjoys the legal presumption of
regularity in the performance of his official duties and functions, which in
turn gives his report the presumption of accuracy, the Constitution, we
repeat, commands the judge to personally determine probable cause in the
issuance of warrants of arrest. This Court has consistently held that a judge
fails in his bounden duty if he relies merely on the certification or the report
of the investigating officer.” 

In the instant case, the public respondent relied fully and completely upon the
resolution of the graft investigation officer and the memorandum of the reviewing
prosecutor, attached to the information filed before it, and its conjecture that the
Ombudsman would not have approved their recommendation without supporting
evidence. It had no other documents from either the complainant (the Anti-Graft
League of the Philippines) or the People from which to sustain its own conclusion
that probable cause exists. Respondent Court palpably committed grave abuse of
discretion in ipso facto issuing the challenged warrant of arrest on the sole basis of
the prosecutor’s findings and recommendation, and without determining on its own
the issue of probable cause based on evidence other than such bare findings and
recommendation.

Dispositive Portion:
WHEREFORE, the petitions are GRANTED and the assailed Resolution is SET
ASIDE. The warrant issued by the Sandiganbayan (Second Division) on May 20,
1992 in Case No. 17674 for the arrest of Petitioners Doris Teresa Ho and Rolando
Narciso is hereby declared NULL AND VOID.

Valmonte v. De Villa, G.R. No. 83988, September 29, 1989 (Re checkpoints)
→(Read also dissents of JJ. Cruz and Sarmiento)

This is a petition for prohibition with preliminary injunction and/or temporary


restraining order, seeking the declaration of checkpoints in Valenzuela, Metro
Manila or elsewhere, as unconstitutional and the dismantling and banning of the
same or, in the alternative, to direct the respondents to formulate guidelines in the
implementation of checkpoints, for the protection of the people.

Petitioner Ricardo C. Valmonte sues in his capacity as citizen of the Republic,


taxpayer, member of the Integrated Bar of the Philippines (IBP), and resident of
Valenzuela, Metro Manila; while petitioner Union of Lawyers and Advocates for
People's Rights (ULAP) sues in its capacity as an association whose members are
all members of the IBP.

The factual background of the case is as follows:

On 20 January 1987, the National Capital Region District Command (NCRDC)


was activated pursuant to Letter of Instruction 02/87 of the Philippine General
Headquarters, AFP, with the mission of conducting security operations within its
area of responsibility and peripheral areas, for the purpose of establishing an
effective territorial defense, maintaining peace and order, and providing an
atmosphere conducive to the social, economic and political development of the
National Capital Region.1 As part of its duty to maintain peace and order, the
NCRDC installed checkpoints in various parts of Valenzuela, Metro Manila.
Petitioners aver that, because of the installation of said checkpoints, the residents
of Valenzuela are worried of being harassed and of their safety being placed at the
arbitrary, capricious and whimsical disposition of the military manning the
checkpoints, considering that their cars and vehicles are being subjected to regular
searches and check-ups, especially at night or at dawn, without the benefit of a
search warrant and/or court order. Their alleged fear for their safety increased
when, at dawn of 9 July 1988, Benjamin Parpon, a supply officer of the
Municipality of Valenzuela, Bulacan, was gunned down allegedly in cold blood by
the members of the NCRDC manning the checkpoint along McArthur Highway at
Malinta, Valenzuela, for ignoring and/or refusing to submit himself to the
checkpoint and for continuing to speed off inspire of warning shots fired in the air.
Petitioner Valmonte also claims that, on several occasions, he had gone thru these
checkpoints where he was stopped and his car subjected to search/check-up
without a court order or search warrant.

Petitioners further contend that the said checkpoints give the respondents a blanket
authority to make searches and/or seizures without search warrant or court order in
violation of the Constitution; 2 and, instances have occurred where a citizen, while
not killed, had been harassed.

Petitioners' concern for their safety and apprehension at being harassed by the
military manning the checkpoints are not sufficient grounds to declare the
checkpoints as per se illegal. No proof has been presented before the Court to show
that, in the course of their routine checks, the military indeed committed specific
violations of petitioners' right against unlawful search and seizure or other rights.

In a case filed by the same petitioner organization, Union of Lawyers and


Advocates for People's Right (ULAP) vs. Integrated National Police, 3 it was held
that individual petitioners who do not allege that any of their rights were violated
are not qualified to bring the action, as real parties in interest.

The constitutional right against unreasonable searches and seizures is a personal


right invocable only by those whose rights have been infringed, 4 or threatened to
be infringed. What constitutes a reasonable or unreasonable search and seizure in
any particular case is purely a judicial question, determinable from a consideration
of the circumstances involved. 5

Petitioner Valmonte's general allegation to the effect that he had been stopped and
searched without a search warrant by the military manning the checkpoints,
without more, i.e., without stating the details of the incidents which amount to a
violation of his right against unlawful search and seizure, is not sufficient to enable
the Court to determine whether there was a violation of Valmonte's right against
unlawful search and seizure. Not all searches and seizures are prohibited. Those
which are reasonable are not forbidden. A reasonable search is not to be
determined by any fixed formula but is to be resolved according to the facts of
each case. 6

Where, for example, the officer merely draws aside the curtain of a vacant vehicle
which is parked on the public fair grounds, 7 or simply looks into a vehicle, 8 or
flashes a light therein, 9 these do not constitute unreasonable search.

The setting up of the questioned checkpoints in Valenzuela (and probably in other


areas) may be considered as a security measure to enable the NCRDC to pursue its
mission of establishing effective territorial defense and maintaining peace and
order for the benefit of the public. Checkpoints may also be regarded as measures
to thwart plots to destabilize the government, in the interest of public security. In
this connection, the Court may take judicial notice of the shift to urban centers and
their suburbs of the insurgency movement, so clearly reflected in the increased
killings in cities of police and military men by NPA "sparrow units," not to
mention the abundance of unlicensed firearms and the alarming rise in lawlessness
and violence in such urban centers, not all of which are reported in media, most
likely brought about by deteriorating economic conditions — which all sum up to
what one can rightly consider, at the very least, as abnormal times. Between the
inherent right of the state to protect its existence and promote public welfare and an
individual's right against a warrantless search which is
however reasonably conducted, the former should prevail.

True, the manning of checkpoints by the military is susceptible of abuse by the


men in uniform, in the same manner that all governmental power is susceptible of
abuse. But, at the cost of occasional inconvenience, discomfort and even irritation
to the citizen, the checkpoints during these abnormal times, when conducted within
reasonable limits, are part of the price we pay for an orderly society and a peaceful
community.

Finally, on 17 July 1988, military and police checkpoints in Metro Manila were
temporarily lifted and a review and refinement of the rules in the conduct of the
police and military manning the checkpoints was ordered by the National Capital
Regional Command Chief and the Metropolitan Police Director. 10

WHEREFORE, the petition is DISMISSED.

SO ORDERED.

Fernan, C.J., Narvasa, Melencio-Herrera, Gutierrez, Jr., Paras, Feliciano,


Gancayco, Bidin, Cortes, Griño-Aquino, Medialdea and Regalado, JJ., concur.

 
Separate Opinions

CRUZ, J., dissenting:

I dissent. The sweeping statements in the majority opinion are as dangerous as the
checkpoints it would sustain and fraught with serious threats to individual liberty.
The bland declaration that individual rights must yield to the demands of national
security ignores the fact that the Bill of Rights was intended precisely to limit the
authority of the State even if asserted on the ground of national security. What is
worse is that the searches and seizures are peremptorily pronounced to be
reasonable even without proof of probable cause and much less the required
warrant. The improbable excuse is that they are aimed at 'establishing an effective
territorial defense, maintaining peace and order, and providing an atmosphere
conducive to the social, economic and political development of the National
Capital Region." For these purposes, every individual may be stopped and searched
at random and at any time simply because he excites the suspicion, caprice,
hostility or malice of the officers manning the checkpoints, on pain of arrest or
worse, even being shot to death, if he resists.

I have no quarrel with a policeman flashing a light inside a parked vehicle on a


dark street as a routine measure of security and curiosity. But the case at bar is
different. Military officers are systematically stationed at strategic checkpoint to
actively ferret out suspected criminals by detaining and searching any individual
who in their opinion might impair "the social, economic and political development
of the National Capital Region." It is incredible that we can sustain such a measure.
And we are not even under martial law.

Unless we are vigilant of our rights, we may find ourselves back to the dark era of
the truncheon and the barbed wire, with the Court itself a captive of its own
complaisance and sitting at the death-bed of liberty.

SARMIENTO, J., dissenting:

I join Justice Isagani Cruz in his dissent, delivered so staightforwardly and


eloquently. I am agreed that the existence alone of checkpoints makes search done
therein, unreasonable and hence, repugnant to the Constitution.

The Charter says that the people enjoy the right of security of person, home, and
effects. (CONST., art. III, sec. 2.) It is also the bedrock — the right of the people to
be left alone — on which the regime of law and constitutionalism rest. It is not, as
the majority would put it, a matter of "occasional inconveniences, discomfort and
even irritation." (Resolution, 4.) To say that it is, is — so I submit — to trivialize
the plain command of the Constitution.
Checkpoints, I further submit, are things of martial rule, and things of the past.
They first saw the light of day by virtue of General Order No. 66 (AUTHORIZING
THE CHIEF OF CONSTABULARY TO ESTABLISH CHECKPOINTS,
UPDATE LISTS OF WANTED PERSONS AND CONDUCT DRAGNET
OPERATIONS AND FOR OTHER PURPOSES), a martial law issuance, as
amended by General Order No. 67 (AMENDING AND AMPLIFYING
PARAGRAPH 7 OF GENERAL ORDER NO. 66 DATED SEPTEMBER 12,
1980), yet another martial law issuance. (See O.G. 4224-4226; 4226-4227 [Aug.,
1983].) They are, so I strongly submit, repressive measures, the same measures
against which we had fought so painstakingly in our quest for liberty, a quest that
ended at EDSA and a quest that terminated a dictatorship. How soon we forget.

While the right against unreasonable searches and seizures, as my brethren


advance, is a right personal to the aggrieved party, the petitioners, precisely, have
come to Court because they had been, or had felt, aggrieved. I submit that in that
event, the burden is the State's, to demonstrate the reasonableness of the search.
The petitioners, Ricardo Valmonte in particular, need not, therefore, have
illustrated the "details of the incident" (Resolution, supra, 4) in all their gore and
gruesomeness.

In any event, the absence alone of a search warrant, as I have averred, makes
checkpoint searches unreasonable, and by itself, subject to constitutional
challenges. (Supra.) As it is, "checkpoints", have become "search warrants" unto
themselves a roving one at that.

That "[n]ot all searches and seizures are prohibited," the majority points out, is
fine. And so is "a reasonable search is not to be determined by any fixed formula
but is to be resolved according to the facts of each case." (Supra) But the question,
exactly, is: Is (are) the search(es) in this case reasonable? I submit that it (they) is
(are) not, for one simple reason: No search warrant has been issued by a judge.

I likewise do not find this case to be a simple matter of an "officer merely


draw(ing) aside the curtain of a vacant vehicle ... or simply look(ing) (supra) there,
"or flash(ing) a light therein." (Supra) What we have here is Orwell's Big Brother
watching every step we take and every move we make.

As it also is, "checkpoints" are apparently, State policy. The American cases the
majority refers to involve routine checks compelled by "probable cause". What we
have here, however, is not simply a policeman on the beat but armed men, CAFGU
or Alsa Masa, who hold the power of life or death over the citizenry, who fire with
no provocation and without batting an eyelash. They likewise shoot you simply
because they do not like your face. I have witnessed actual incidents.

Washington said that militia can not be made to dictate the terms for the nation. He
can not be anymore correct here.
"Between the inherent right of the state to protect its existence ... and on
individual's right against a warrantless search, which is reasonably conducted, "so
my brethren go on, the former shall prevail. (Supra) First, this is the same lie that
the hated despot foisted on the Filipino people. It is a serious mistake to fall for it a
second time around. Second, the checkpoint searches herein are unreasonable:
There was no warrant.

A final word. After twenty years of tyranny, the dawn is upon us. The country is
once again the "showcase of democracy" in Asia. But if in many cases, it has been
"paper democracy", let this Court anyway bring to pass its stand, and make liberty
in the land, a living reality.

I vote then, to grant the petition.

Separate Opinions

CRUZ, J., dissenting:

I dissent. The sweeping statements in the majority opinion are as dangerous as the
checkpoints it would sustain and fraught with serious threats to individual liberty.
The bland declaration that individual rights must yield to the demands of national
security ignores the fact that the Bill of Rights was intended precisely to limit the
authority of the State even if asserted on the ground of national security. What is
worse is that the searches and seizures are peremptorily pronounced to be
reasonable even without proof of probable cause and much less the required
warrant. The improbable excuse is that they are aimed at 'establishing an effective
territorial defense, maintaining peace and order, and providing an atmosphere
conducive to the social, economic and political development of the National
Capital Region." For these purposes, every individual may be stopped and searched
at random and at any time simply because he excites the suspicion, caprice,
hostility or malice of the officers manning the checkpoints, on pain of arrest or
worse, even being shot to death, if he resists.

I have no quarrel with a policeman flashing a light inside a parked vehicle on a


dark street as a routine measure of security and curiosity. But the case at bar is
different. Military officers are systematically stationed at strategic checkpoint to
actively ferret out suspected criminals by detaining and searching any individual
who in their opinion might impair "the social, economic and political development
of the National Capital Region." It is incredible that we can sustain such a measure.
And we are not even under martial law.
Unless we are vigilant of our rights, we may find ourselves back to the dark era of
the truncheon and the barbed wire, with the Court itself a captive of its own
complaisance and sitting at the death-bed of liberty.

SARMIENTO, J., dissenting:

I join Justice Isagani Cruz in his dissent, delivered so staightforwardly and


eloquently. I am agreed that the existence alone of checkpoints makes search done
therein, unreasonable and hence, repugnant to the Constitution.

The Charter says that the people enjoy the right of security of person, home, and
effects. (CONST., art. III, sec. 2.) It is also the bedrock — the right of the people to
be left alone — on which the regime of law and constitutionalism rest. It is not, as
the majority would put it, a matter of "occasional inconveniences, discomfort and
even irritation." (Resolution, 4.) To say that it is, is — so I submit — to trivialize
the plain command of the Constitution.

Checkpoints, I further submit, are things of martial rule, and things of the past.
They first saw the light of day by virtue of General Order No. 66 (AUTHORIZING
THE CHIEF OF CONSTABULARY TO ESTABLISH CHECKPOINTS,
UPDATE LISTS OF WANTED PERSONS AND CONDUCT DRAGNET
OPERATIONS AND FOR OTHER PURPOSES), a martial law issuance, as
amended by General Order No. 67 (AMENDING AND AMPLIFYING
PARAGRAPH 7 OF GENERAL ORDER NO. 66 DATED SEPTEMBER 12,
1980), yet another martial law issuance. (See O.G. 4224-4226; 4226-4227 [Aug.,
1983].) They are, so I strongly submit, repressive measures, the same measures
against which we had fought so painstakingly in our quest for liberty, a quest that
ended at EDSA and a quest that terminated a dictatorship. How soon we forget.

While the right against unreasonable searches and seizures, as my brethren


advance, is a right personal to the aggrieved party, the petitioners, precisely, have
come to Court because they had been, or had felt, aggrieved. I submit that in that
event, the burden is the State's, to demonstrate the reasonableness of the search.
The petitioners, Ricardo Valmonte in particular, need not, therefore, have
illustrated the "details of the incident" (Resolution, supra, 4) in all their gore and
gruesomeness.

In any event, the absence alone of a search warrant, as I have averred, makes
checkpoint searches unreasonable, and by itself, subject to constitutional
challenges. (Supra.) As it is, "checkpoints", have become "search warrants" unto
themselves a roving one at that.

That "[n]ot all searches and seizures are prohibited," the majority points out, is
fine. And so is "a reasonable search is not to be determined by any fixed formula
but is to be resolved according to the facts of each case." (Supra) But the question,
exactly, is: Is (are) the search(es) in this case reasonable? I submit that it (they) is
(are) not, for one simple reason: No search warrant has been issued by a judge.

I likewise do not find this case to be a simple matter of an "officer merely


draw(ing) aside the curtain of a vacant vehicle ... or simply look(ing) (supra) there,
"or flash(ing) a light therein." (Supra) What we have here is Orwell's Big Brother
watching every step we take and every move we make.

As it also is, "checkpoints" are apparently, State policy. The American cases the
majority refers to involve routine checks compelled by "probable cause". What we
have here, however, is not simply a policeman on the beat but armed men, CAFGU
or Alsa Masa, who hold the power of life or death over the citizenry, who fire with
no provocation and without batting an eyelash. They likewise shoot you simply
because they do not like your face. I have witnessed actual incidents.

Washington said that militia can not be made to dictate the terms for the nation. He
can not be anymore correct here.

"Between the inherent right of the state to protect its existence ... and on
individual's right against a warrantless search, which is reasonably conducted, "so
my brethren go on, the former shall prevail. (Supra) First, this is the same lie that
the hated despot foisted on the Filipino people. It is a serious mistake to fall for it a
second time around. Second, the checkpoint searches herein are unreasonable:
There was no warrant.

A final word. After twenty years of tyranny, the dawn is upon us. The country is
once again the "showcase of democracy" in Asia. But if in many cases, it has been
"paper democracy", let this Court anyway bring to pass its stand, and make liberty
in the land, a living reality.

I vote then, to grant the petition.

People v. Maspil, supra


8. Warrantless search during “on-the-spot apprehensions” with probable cause

CASE: People v. De Gracia, G. R. Nos. 102009-10 July 6, 1994 (Cf. Alih v.


Castro: “precarious state of lawlessness” cannot excuse the non-observance of the
constitutional guaranty, more so if no state of hostilities in the area to justify,
assuming it could, the unreasonable search and seizure.)
CASES:
People v. Tangliben, supra
People v. Aminnudin, supra
People v. Malmstedt, G.R. No. 91107, June 19, 1991 (Read also the dissents of CJ
Narvasa & Justice Cruz)

In an information dated 15 June 1989, accused-appellant Mikael Malmstedt


(hereinafter referred to as the accused) was charged before the Regional Trial
Court (RTC) of La Trinidad, Benguet, Branch 10, in Criminal Case No. 89-CR-
0663, for violation of Section 4, Art. II of Republic Act 6425, as amended,
otherwise known as the Dangerous Drugs Act of 1972, as amended. The factual
background of the case is as follows:

Accused Mikael Malmstedt, a Swedish national, entered the Philippines for the
third time in December 1988 as a tourist. He had visited the country sometime in
1982 and 1985.

In the evening of 7 May 1989, accused left for Baguio City. Upon his arrival
thereat in the morning of the following day, he took a bus to Sagada and stayed in
that place for two (2) days.

At around 7:00 o'clock in the morning of 11 May 1989, accused went to the
Nangonogan bus stop in Sagada to catch the first available trip to Baguio City.
From Baguio City, accused planned to take a late afternoon trip to Angeles City,
then proceed to Manila to catch his flight out of the country, scheduled on 13 May
1989. From Sagada, accused took a Skyline bus with body number 8005 and Plate
number AVC 902.1

At about 8: 00 o'clock in the morning of that same day (11 May 1989), Captain
Alen Vasco, the Commanding Officer of the First Regional Command
(NARCOM) stationed at Camp Dangwa, ordered his men to set up a temporary
checkpoint at Kilometer 14, Acop, Tublay, Mountain Province, for the purpose of
checking all vehicles coming from the Cordillera Region. The order to establish a
checkpoint in the said area was prompted by persistent reports that vehicles
coming from Sagada were transporting marijuana and other prohibited drugs.
Moreover, information was received by the Commanding Officer of NARCOM,
that same morning, that a Caucasian coming from Sagada had in his possession
prohibited drugs.2

The group composed of seven (7) NARCOM officers, in coordination with Tublay
Police Station, set up a checkpoint at the designated area at about 10:00 o'clock in
the morning and inspected all vehicles coming from the Cordillera Region.

At about 1:30 o'clock in the afternoon, the bus where accused was riding was
stopped. Sgt. Fider and CIC Galutan boarded the bus and announced that they were
members of the NARCOM and that they would conduct an inspection. The two (2)
NARCOM officers started their inspection from the front going towards the rear of
the bus. Accused who was the sole foreigner riding the bus was seated at the rear
thereof.

During the inspection, CIC Galutan noticed a bulge on accused's waist. Suspecting
the bulge on accused's waist to be a gun, the officer asked for accused's passport
and other identification papers. When accused failed to comply, the officer
required him to bring out whatever it was that was bulging on his waist. The
bulging object turned out to be a pouch bag and when accused opened the same
bag, as ordered, the officer noticed four (4) suspicious-looking objects wrapped in
brown packing tape, prompting the officer to open one of the wrapped objects. The
wrapped objects turned out to contain hashish, a derivative of marijuana.

Thereafter, accused was invited outside the bus for questioning. But before he
alighted from the bus, accused stopped to get two (2) travelling bags from the
luggage carrier.

Upon stepping out of the bus, the officers got the bags and opened them. A teddy
bear was found in each bag. Feeling the teddy bears, the officer noticed that there
were bulges inside the same which did not feel like foam stuffing. It was only after
the officers had opened the bags that accused finally presented his passport.

Accused was then brought to the headquarters of the NARCOM at Camp Dangwa,
La Trinidad, Benguet for further investigation. At the investigation room, the
officers opened the teddy bears and they were found to also contain hashish.
Representative samples were taken from the hashish found among the personal
effects of accused and the same were brought to the PC Crime Laboratory for
chemical analysis.

In the chemistry report, it was established that the objects examined were hashish.
a prohibited drug which is a derivative of marijuana. Thus, an information was
filed against accused for violation of the Dangerous Drugs Act.

During the arraignment, accused entered a plea of "not guilty." For his defense, he
raised the issue of illegal search of his personal effects. He also claimed that the
hashish was planted by the NARCOM officers in his pouch bag and that the two
(2) travelling bags were not owned by him, but were merely entrusted to him by an
Australian couple whom he met in Sagada. He further claimed that the Australian
couple intended to take the same bus with him but because there were no more
seats available in said bus, they decided to take the next ride and asked accused to
take charge of the bags, and that they would meet each other at the Dangwa
Station.

Likewise, accused alleged that when the NARCOM officers demanded for his
passport and other Identification papers, he handed to one of the officers his pouch
bag which was hanging on his neck containing, among others, his passport, return
ticket to Sweden and other papers. The officer in turn handed it to his companion
who brought the bag outside the bus. When said officer came back, he charged the
accused that there was hashish in the bag. He was told to get off the bus and his
picture was taken with the pouch bag placed around his neck. The trial court did
not give credence to accused's defense.

The claim of the accused that the hashish was planted by the NARCOM officers,
was belied by his failure to raise such defense at the earliest opportunity. When
accused was investigated at the Provincial Fiscal's Office, he did not inform the
Fiscal or his lawyer that the hashish was planted by the NARCOM officers in his
bag. It was only two (2) months after said investigation when he told his lawyer
about said claim, denying ownership of the two (2) travelling bags as well as
having hashish in his pouch bag.

In a decision dated 12 October 1989, the trial court found accused guilty beyond
reasonable doubt for violation of the Dangerous Drugs Act, specifically Section 4,
Art. II of RA 6425, as amended.3 The dispositive portion of the decision reads as
follows:

WHEREFORE, finding the guilt of the accused Mikael Malmstedt established


beyond reasonable doubt, this Court finds him GUILTY of violation of Section 4,
Article 11 of Republic Act 6425, as amended, and hereby sentences him to suffer
the penalty of life imprisonment and to pay a fine of Twenty Thousand Pesos
(P20,000.00), with subsidiary imprisonment in case of insolvency and to pay the
costs.

Let the hashish subject of this case be turned over to the First Narcotics Regional
Unit at Camp Bado; Dangwa, La Trinidad Benguet for proper disposition under
Section 20, Article IV of Republic Act 6425, as amended.

SO ORDERED.4

Seeking the reversal of the decision of the trial court finding him guilty of the
crime charged, accused argues that the search of his personal effects was illegal
because it was made without a search warrant and, therefore, the prohibited drugs
which were discovered during the illegal search are not admissible as evidence
against him.

The Constitution guarantees the right of the people to be secure in their persons,
houses, papers and effects against unreasonable searches and seizures. 5 However,
where the search is made pursuant to a lawful arrest, there is no need to obtain a
search warrant. A lawful arrest without a warrant may be made by a peace officer
or a private person under the following circumstances. 6

Sec. 5 Arrest without warrant; when lawful. –– A peace officer or a private person


may, without a warrant, arrest a person:

(a) When, in his presence, the person to be arrested has committed is actually
committing, or is attempting to commit an offense;

(b) When an offense has in fact just been committed, and he has personal
knowledge of facts indicating that the person to be arrested has committed it; and

(c) When the person to be arrested is a prisoner who has escaped from a penal
establishment or place where he is serving final judgment or temporarily confined
while his case is pending, or has escaped while being transferred from one
confinement to another.

In cases falling under paragraphs (a) and (b) hereof, the person arrested without a
warrant shall be forthwith delivered to the nearest police station or jail, and he shall
be proceeded against in accordance with Rule 112, Section 7. (6a 17a).

Accused was searched and arrested while transporting prohibited drugs (hashish).
A crime was actually being committed by the accused and he was caught
in flagrante delicto. Thus, the search made upon his personal effects falls squarely
under paragraph (1) of the foregoing provisions of law, which allow a warrantless
search incident to a lawful arrest.7

While it is true that the NARCOM officers were not armed with a search warrant
when the search was made over the personal effects of accused, however, under the
circumstances of the case, there was sufficient probable cause for said officers to
believe that accused was then and there committing a crime.

Probable cause has been defined as such facts and circumstances which could lead
a reasonable, discreet and prudent man to believe that an offense has been
committed, and that the objects sought in connection with the offense are in the
place sought to be searched.8 The required probable cause that will justify a
warrantless search and seizure is not determined by any fixed formula but is
resolved according to the facts of each case. 9
Warrantless search of the personal effects of an accused has been declared by this
Court as valid, because of existence of probable cause, where the smell of
marijuana emanated from a plastic bag owned by the accused, 10 or where the
accused was acting suspiciously,11 and attempted to flee.12

Aside from the persistent reports received by the NARCOM that vehicles coming
from Sagada were transporting marijuana and other prohibited drugs, their
Commanding Officer also received information that a Caucasian coming from
Sagada on that particular day had prohibited drugs in his possession. Said
information was received by the Commanding Officer of NARCOM the very same
morning that accused came down by bus from Sagada on his way to Baguio City.

When NARCOM received the information, a few hours before the apprehension of
herein accused, that a Caucasian travelling from Sagada to Baguio City was
carrying with him prohibited drugs, there was no time to obtain a search warrant.
In the Tangliben case,13 the police authorities conducted a surveillance at the
Victory Liner Terminal located at Bgy. San Nicolas, San Fernando Pampanga,
against persons engaged in the traffic of dangerous drugs, based on information
supplied by some informers. Accused Tangliben who was acting suspiciously and
pointed out by an informer was apprehended and searched by the police
authorities. It was held that when faced with on-the-spot information, the police
officers had to act quickly and there was no time to secure a search warrant.

It must be observed that, at first, the NARCOM officers merely conducted a


routine check of the bus (where accused was riding) and the passengers therein,
and no extensive search was initially made. It was only when one of the officers
noticed a bulge on the waist of accused, during the course of the inspection, that
accused was required to present his passport. The failure of accused to present his
identification papers, when ordered to do so, only managed to arouse the suspicion
of the officer that accused was trying to hide his identity. For is it not a regular
norm for an innocent man, who has nothing to hide from the authorities, to readily
present his identification papers when required to do so?

The receipt of information by NARCOM that a Caucasian coming from Sagada


had prohibited drugs in his possession, plus the suspicious failure of the accused to
produce his passport, taken together as a whole, led the NARCOM officers to
reasonably believe that the accused was trying to hide something illegal from the
authorities. From these circumstances arose a probable cause which justified the
warrantless search that was made on the personal effects of the accused. In other
words, the acts of the NARCOM officers in requiring the accused to open his
pouch bag and in opening one of the wrapped objects found inside said bag (which
was discovered to contain hashish) as well as the two (2) travelling bags containing
two (2) teddy bears with hashish stuffed inside them, were prompted by accused's
own attempt to hide his identity by refusing to present his passport, and by the
information received by the NARCOM that a Caucasian coming from Sagada had
prohibited drugs in his possession. To deprive the NARCOM agents of the ability
and facility to act accordingly, including, to search even without warrant, in the
light of such circumstances, would be to sanction impotence and ineffectiveness in
law enforcement, to the detriment of society.

WHEREFORE, premises considered, the appealed judgment of conviction by the


trial court is hereby AFFIRMED. Costs against the accused-appellant.

SO ORDERED.

Melencio-Herrera, Paras, Feliciano, Bidin, Griño-Aquino, Medialdea, Regalado


and Davide, Jr., JJ., concur.
Sarmiento, J., is on leave.

Separate Opinions

NARVASA, J., concurring and dissenting:

The ancient tradition that a man's home is his castle, safe from intrusion even by
the king, has not only found its niche in all our charters, from 1935 to the present;
it has also received unvarying recognition and acceptance in our case law. 1 The
present Constitution2 declares that —

The right of the people to be secure in their persons, houses, papers, and effects
against unreasonable searches and seizures of whatever nature and for any purpose,
shall be inviolable, and no search warrant or warrant of arrest shall issue except
upon probable cause to be determined personally by the judge after examination
under oath or affirmation of the complainant and the witnesses he may produce,
and particularly describing the place to be searched, and the persons or things to be
seized.

It further ordains that any evidence obtained in violation of said right, among
others, "shall be inadmissible for any purpose in any proceeding." 3

The rule is that no person may be subjected by the police or other government
authority to a search of his body, or his personal effects or belongings, or his
residence except by virtue of a search warrant or on the occasion of a legitimate
arrest.4

An arrest is legitimate, of course, if effected by virtue of a warrant of arrest. Even


without a warrant, an arrest may also be lawfully made by a peace officer or a
private person:5
(a) when, in his presence, the person to be arrested has committed is actually
committing, or is attempting to commit an offense;

(b) When an offense has in fact just been committed, and he has personal
knowledge of facts indicating that the person to be arrested has committed it; and

(c) When the person to be arrested is a prisoner who has escaped from a penal
establishment or place where he is serving final judgment or temporarily confined
while his case is pending, or has escaped while being transferred from one
confinement to another.

In cases falling under paragraphs (a) and (b) hereof, the person arrested without a
warrant shall be forthwith delivered to the nearest police station or jail, and he shall
be proceeded against in accordance with Rule 112, Section 7.

In any of these instances of a lawful arrest, the person arrested "may be searched
for dangerous weapons or anything which may be used as proof of the commission
of an offense, without a search warrant."6 And it has been held that the search may
extend to the area "within his immediate control," i.e., the area from which said
person arrested might gain possession of a weapon or destructible evidence. 7

Apart from "search incidental to an arrest," a warrantless search has also been held
to be proper in cases of "search of a moving vehicle, 8 and "seizure of evidence in
plain view."9 This was the pronouncement in Manipon, Jr. v. Sandiganbayan, 143
SCRA 267, 276, which drew attention to Moreno v. Ago Chi;10 Alvero v.
Dizon,11 Papa v. Mago,12 and an American precedent, Harris v. U.S.13

If, on the other, a person is searched without a warrant, or under circumstances


other than those justifying an arrest without warrant in accordance with law,
supra, merely on suspicion that he is engaged in some felonious enterprise, and in
order to discover if he has indeed committed a crime, it is not only the arrest which
is illegal but also, the search on the occasion thereof, as being "the fruit of the
poisonous tree.14 In that event, any evidence taken, even if confirmatory of the
initial suspicion, is inadmissible "for any purpose in any proceeding." 15 But the
right against an unreasonable search and seizure may be waived by the person
arrested, provided he knew of such right and knowingly decided not to invoke it. 16

There is unanimity among the members of the Court upon the continuing validity
of these established principles. However, the Court is divided as regards the
ultimate conclusions which may properly be derived from the proven facts and
consequently, the manner in which the principles just cited should apply thereto.

The proofs of the prosecution and those of the defense are diametrically at odds.
What is certain, however, is that the soldiers had no warrant of arrest when they
conducted a search of Malmstedt's person and the things in his possession at the
time. Indeed, the Court a quo acknowledged that the soldiers could "not be
expected to be armed with a warrant or arrest nor a search warrant everytime they
establish a temporary checkpoint . . . (and) no judge would issue them one
considering that searching questions have to be asked before a warrant could be
issued." Equally plain is that prior to the search, a warrantless arrest of Malmstedt
could not validly have been in accordance with the norms of the law. For
Malmstedt had not committed, nor was he actually committing or attempting to
commit a crime, in the soldiers' presence, nor did said soldiers have personal and
competent knowledge that Malmstedt had in fact just committed a crime. All they
had was a suspicion that Malmstedt might have some prohibited drug on him or in
his bags; all they had was, in the words of the Trial Court, "the hope of intercepting
any dangerous drug being transported," or, as the Office of the Solicitor General
asserts, "information that most of the buses coming . . . (from the Cordillera) were
transporting marijuana and other prohibited drugs."

This case, is remarkably similar to Peo. v. Aminnudin, decided on July 6, 1988 also
by the First Division.17 There, Aminnudin was arrested without a warrant by PC
officers as he was disembarking from an inter-island vessel. The officers were
waiting for him because he was, according to an informer's report, then
transporting marijuana. The search of Aminnudin's bag confirmed the informer's
report; the bag indeed contained marijuana. The Court nevertheless held that since
the PC officers had failed to procure a search warrant although they had sufficient
time (two days) to do so and therefore, the case presented no such urgency as to
justify a warrantless search, the search of Aminnudin's person and bag, the seizure
of the marijuana and his subsequent arrest were illegal; and the marijuana was
inadmissible in evidence in the criminal action subsequently instituted against
Aminnudin for violating the Dangerous Drugs Act.

There are, on the other hand, other cases adjudicated by this Court in which
apparently different conclusions were reached. It is needful to devote a few words
to them so that the relevant constitutional and legal propositions are not
misunderstood.

In People v. Claudio (decision promulgated on April 15, 1988),18 the accused


boarded a "Victory Liner" passenger bus going to Olongapo from Baguio City. She
placed the plastic bag she was carrying at the back of the seat then occupied by
Obiña, an INP member "on Detached Service with the Anti-Narcotics Unit." This
avowedly aroused Obiña's suspicion, and at the first opportunity, and without
Claudio's knowledge, he surreptitiously looked into the plastic bag and noted that it
contained camote tops as well as a package, and that there emanated from the
package the smell of marijuana with which he had become familiar on account of
his work. So when the bus stopped at Sta. Rita, and Claudio alighted, Obiña
accosted her, showed her his ID, identified himself as a policeman, and announced
his intention to search her bag which he said contained marijuana because of the
distinctive odor detected by him. Ignoring her plea — "Please go with me, let us
settle this at home" — he brought her to the police headquarters., where
examination of the package in Claudio's bag confirmed his suspicion that it indeed
contained marijuana. The Court held the warrantless arrest under the circumstances
to be lawful, the search justified, and the evidence thus discovered admissible in
evidence against the accused.

In People v. Tangliben (decision promulgated on April 6, 1990),19 two police


officers and a barangay tanod were conducting a "surveillance mission" at the
Victory Liner Terminal at San Nicolas, San Fernando, Pampanga, "aimed not only
against persons who may commit misdemeanors . . . (there) but also on persons
who may be engaging in the traffic of dangerous drugs based on information
supplied by informers; . . . they noticed a person carrying a red travelling bag . .
who was acting suspiciously;" they asked him to open the bag; the person did so
only after they identified themselves as peace officers; found in the bag were
marijuana leaves wrapped in plastic weighing one kilogram, more or less; the
person was then taken to the police headquarters at San Fernando, Pampanga,
where he was investigated; and an information was thereafter filed against that
person, Tangliben, charging him with a violation of the Dangerous Drugs Act of
1972 (RA 6425), as amended. Upon these facts it was ruled, citing Claudio, supra,
that there was a valid warrantless arrest and a proper warrantless search incident
thereto.

The facts in Tangliben were pronounced to be different from those in People v.


Aminnudin, supra. "In contrast" to Aminnudin where the Court perceived no
urgency as to preclude the application for and obtention of a search warrant, it was
declared that the Tangliben case —

. . . presented urgency. . . (The evidence revealed) that there was an informer who
pointed to the accused-appellant as carrying marijuana . . . Faced with such on-the-
spot information, the police officers had to act quickly. There was not enough time
to secure a search warrant . . . To require search warrants during on-the-spot
apprehensions of drug pushers, illegal possessors of firearms, jueteng collectors,
smugglers of contraband goods, robber, etc. would make it extremely difficult, if
not impossible to contain the crimes with which these persons are associated.

In Tangliben, therefore, there was in the Court's view sufficient evidence on hand


to enable the PC officers to secure a search warrant, had there been time. But
because there was actually no time to get the warrant, and there were "on-the-spot"
indications that Tangliben was then actually committing a crime, the search of his
person and his effects was considered valid.

Two other decisions presented substantially similar circumstance


instances: Posadas v. C.A., et al., decided on August 2, 1990,20 and People v.
Moises Maspil, Jr., et al., decided on August 20, 1990.21
In the first case, Posadas was seen to be acting suspiciously by two members of the
INP, Davao Metrodiscom, and when he was accosted by the two, who identified
themselves as police officers, he suddenly fled. He was pursued, overtaken and,
notwithstanding his resistance, placed in custody. The buri bag Posadas was then
carrying was found to contain a revolver, for which he could produce no license or
authority to possess, four rounds of live ammunition, and a tear gas grenade. He
was prosecuted for illegal possession of firearms and ammunition and convicted
after trial. This Court affirmed Posadas' conviction, holding that there was, in the
premises, probable cause for a search without warrant, i.e., the appellant was acting
suspiciously and attempted to flee with the buri bag he had with him at the time.
The Court cited with approval the ruling of the U.S. Federal Supreme Court
in John W. Terry v. State of Ohio,22 a 1968 case, which the Solicitor General had
invoked to justify the search.

In the case of Maspil, et al., a checkpoint was set up by elements of the First
Narcotics Regional Unit of the Narcotics Command at Sayangan, Atok, Benguet,
to monitor, inspect and scrutinize vehicles on the highway going towards Baguio
City. This was done because of a confidential report by informers that Maspil and
another person, Bagking, would be transporting a large quantity of marijuana to
Baguio City. In fact, the informers were with the policemen manning the
checkpoint. As expected, at about 2 o'clock in the early morning of November 1,
1986, a jeepney approached the checkpoint, driven by Maspil, with Bagking as
passenger. The officers stopped the vehicle and saw that on it were loaded 2 plastic
sacks, a jute sack, and 3 big round tin cans. When opened, the sacks and cans were
seen to contain what appeared to be marijuana leaves. The policemen thereupon
placed Maspil and Bagking under arrest, and confiscated the leaves which, upon
scientific examination, were verified to be marijuana leaves. The Court upheld the
validity of the search thus conducted, as being incidental to a lawful warrantless
arrest,23 and declared that, as in Tangliben, supra, Maspil and Bagking had been
caught in flagrante delicto transporting prohibited drugs at the time of their arrest.
Again, the Court took occasion to distinguish the case from Aminnudin24 in which,
as aforestated, it appeared that the police officers were aware of Aminnudin's
identity, his projected criminal enterprise and the vessel on which he would be
arriving, and, equally as importantly, had sufficient time and opportunity to obtain
a search warrant. In the case of Maspil and Bagking, the Court found that the
officers concerned had no exact description of the vehicle the former would be
using to transport marijuana, and no inkling of the definite time of the suspects'
arrival, and pointed out that a jeepney on the road is not the same as a passenger
boat on the high seas whose route and time of arrival are more or less certain, and
which ordinarily cannot deviate from or otherwise alter its course, or select another
destination.25

The most recent decision treating of warrantless search and seizure appears to
be People v. Lo Ho Wing; et al., G.R. No. 88017, decided on January 21, 1991 (per
Gancayco, J.). In that case, an undercover or "deep penetration" agent, Tia,
managed somehow to gain acceptance into a group of suspected drug smugglers,
which included Peter Lo and Lim Ching Huat. Tia accompanied Peter Lo to
Guangzhou, China, where he saw him and other person empty the contents of six
(6) tins of tea and replace them with white powder. On their return to Manila with
the cans of substituted "tea," they were met at the airport by Lim. As they were
leaving the airport in separate vehicles, they were intercepted by officers and
operatives of the Narcotics Command (NARCOM), who had earlier been tipped
off by Tia, and placed under arrest. As search of the luggage brought in by Tia and
Peter Lo, loaded on the group's vehicles, quickly disclosed the six (6) tin cans
containing fifty-six (56) bags of white crystalline powder which, upon analysis,
was identified as metamphetamine. Tia, Lo and Lim were indicted for violation of
the Dangerous Drugs Act of 1972. Tia was discharged as state witness. Lo and Lim
were subsequently convicted and sentenced to life imprisonment. One of the
questions raised by them in this Court on appeal was whether the warrantless
search of their vehicles and personal effects was legal. The Court, citing Manipon,
Jr. v. Sandiganbayan, 143 SCRA 267 (1986),26 held legal the search of the
appellants' moving vehicles and the seizure therefrom of the dangerous drug,
considering that there was intelligence information, including clandestine reports
by a planted spy actually participating in the activity, that the appellants were
bringing prohibited drugs into the country; that the requirement of obtaining a
search warrant "borders on the impossible in the case of smuggling effected by the
use of a moving vehicle that can transport contraband from one place to another
with impunity," and "it is not practicable to secure a warrant because the vehicle
can be quickly moved out of the locality or jurisdiction in which the warrant must
be sought.27

In all five cases, Claudio, Tangliben, Posadas, Maspil, and Lo Ho Wing, facts


existed which were found by the Court as justifying warantless arrests.
In Claudio, the arresting officer had secretly ascertained that the woman he was
arresting was in fact in possession of marijuana; he had personally seen that her
bag contained not only vegetables but also a package emitting the odor of
marijuana. In Tangliben, the person arrested and searched was acting suspiciously,
and had been positively pointed to as carrying marijuana. And in both cases, the
accused were about to board passenger buses, making it urgent for the police
officers concerned to take quick and decisive action. In Posadas, the person
arrested and searched was acting suspiciously, too, and when accosted had
attempted to flee from the police officers. And in Maspil and Lo Ho Wing, there
was definite information of the precise identity of the persons engaged in
transporting prohibited drugs at a particular time and place.

Now, as regards the precise issue at hand, whether or not the facts in the case at bar
make out a legitimate instance of a warrantless search and seizure, there is, as
earlier pointed out, a regrettable divergence of views among the members of the
Court.
Contrary to the conclusion reached by the majority, I believe that the appellant
should be absolved on reasonable doubt. There was in this case no confidential
report from, or positive identification by an informer; no attempt to flee; no bag or
package emitting tell-tale odors; no other reasonably persuasive indications that
Malmstedt was at the time in process of perpetrating the offense for which he was
subsequently prosecuted. Hence, when the soldiers searched Malmstedt's pouch
and the bags in his possession, they were simply "fishing" for evidence. It matters
not that the search disclosed that the bags contained prohibited substances,
confirming their initial information and suspicion. The search was not made by
virtue of a warrant or as an incident of a lawful warrantless arrest, i.e., under
circumstances sufficient to engender a reasonable belief that some crime was being
or about to be committed, or adjust been committed. There was no intelligent and
intentional waiver of the right against unreasonable searches and seizure. The
search was therefore illegal, since the law requires that there first be a lawful arrest
of an individual before a search of his body and his belongings may licitly be
made. The process cannot be reversed, i.e., a search be first undertaken, and then
an arrest effected, on the strength of the evidence yielded by the search. An arrest
made in that case would be unlawful, and the search undertaken as an incident of
such an unlawful arrest, also unlawful.

The fact that when investigated at the headquarters of the Narcotic Command at
Camp Dangwa, La Trinidad, Malmstedt had, it is said, willingly admitted that there
were was hashish inside the "teddy bears" in the luggage found in his possession
— an admission subsequently confirmed by laboratory examination — does not
help the cause of the prosecution one bit. Nothing in the record even remotely
suggests that Malmstedt was accorded the rights guaranteed by the Constitution to
all persons under custodial investigation.28 He was not informed, prior to being
interrogated, that he had the "right to remain silent and to have competent and
independent counsel preferably of his own choice," and that if he could not afford
the services of counsel, he would be provided with one; not does it appear at all
that he waived those rights "in writing and in the presence of counsel." The
soldiers and the police officers simply went ahead with the investigation of
Malmstedt, without counsel. The admissions elicited from Malmstedt under these
circumstances, as the Constitution clearly states, are "inadmissible in evidence
against him.29

The prohibited drugs supposedly discovered in Malmstedt's bags, having been


taken in violation of the constitutional right against unreasonable searches and
seizures, are inadmissible against him "for any purpose in any proceeding." Also
pronounced as incompetent evidence against him are the admissions supposedly
made by him without his first being accorded the constitutional rights of persons
under custodial investigation. Without such object evidence and admissions,
nothing remains of the case against Malmstedt.
It may be conceded that, as the Trial Court points out, the evidence presented by
Malmstedt in his defense is feeble, unworthy of credence. This is beside the point;
for conformably to the familiar axiom, the State must rely on the strength of its
evidence and not on the weakness of the defense. The unfortunate fact is that
although the existence of the hashish is an objective physical reality that cannot but
be conceded, there is in law no evidence to demonstrate with any degree of
persuasion, much less beyond reasonable doubt, that Malmstedt was engaged in a
criminal activity. This is the paradox created by the disregard of the applicable
constitutional safeguards. The tangible benefit is that the hashish in question has
been correctly confiscated and thus effectively withdrawn from private use.

What is here said should not by any means be taken as a disapproval or a


disparagement of the efforts of the police and military authorities to deter and
detect offenses, whether they be possession of and traffic in prohibited drugs, or
some other. Those efforts obviously merit the support and commendation of the
Courts and indeed of every responsible citizen. But those efforts must take account
of the basic rights granted by the Constitution and the law to persons who may fall
under suspicion of engaging in criminal acts. Disregard of those rights may not be
justified by the objective of ferreting out and punishing crime, no matter how
eminently desirable attainment of that objective might be. Disregard of those
rights, as this Court has earlier stressed, may result in the escape of the guilty, and
all because the "constable has blundered," rendering the evidence inadmissible
even if truthful or otherwise credible.30

I therefore vote to reverse the Trial Court's judgment of October 12, 1989 and to
acquit the appellant on reasonable doubt.

CRUZ, J., dissenting:

I join Mr. Justice Andres R. Narvasa in his dissent, which I believe represents the
correct application to the facts of this case of the provisions of the Bill of Rights
and the Rules of Court on searches and seizures. It is consistent with
my ponencia in People v. Aminnudin, 163 SCRA 402, and also with Alih v.
Castro, 151 SCRA 279, the latter being a unanimous decision of the Court en banc,
and my dissents in Umil v. Ramos (on warrantless arrests, 187 SCRA 311,
Valmonte v. De Villa (on checkpoints), 178, SCRA 211, 185 SCRA 665, and
Guazon v. De Villa (on "zonas"), 181 SCRA 623.

I write this separate opinion merely to remark on an observation made during the
deliberation on this case that some members of the Court seem to be coddling
criminals instead of extending its protection to society, which deserves our higher
concern. The inference is that because of our wrong priorities, criminals are being
imprudently let free, to violate our laws again; and it is all our fault.

Believing myself to be among those alluded to, I will say without apology that I do
not consider a person a criminal, until he is convicted by final judgment after a fair
trial by a competent and impartial court. Until then, the Constitution bids us to
presume him innocent. He may seem boorish or speak crudely or sport tattoos or
dress weirdly or otherwise fall short of our own standards of propriety and
decorum. None of these makes him a criminal although he may look like a
criminal.

It is so easy to condemn a person on the basis of his appearance but it is also so


wrong.

On the question before us, it seems to be the inclination of some judges to wink at
an illegal search and seizure as long as the suspect has been actually found in
possession of a prohibited article That fact will retroactively validate the violation
of the Bill of Rights for after all, as they would rationalize, the suspect is a
criminal. What matters to them is the fact of illegal possession, not the fact of
illegal search and seizure.

This kind of thinking takes us back to the intolerant days of Moncado v. People's
Court, 80 Phil. 1, which was discredited in Stonehill v. Diokno, 20 SCRA 383,
even before it was definitely rejected by an express provision in the 1973
Constitution. That provision, which has been retained in the present Constitution,
again explicitly declares that any evidence illegally obtained "shall be inadmissible
for any purpose in any proceeding."

The fruit of the poisonous tree should not be allowed to poison our system of
criminal justice.1âwphi1 In the case at bar, the search was made at a checkpoint
established for the preposterous reason that the route was being used by marijuana
dealers and on an individual who had something bulging at his waist that excited
the soldier's suspicion. Was that probable cause? The ponencia notes that the
military had advance information that a Caucasian was coming from the Sagada
with prohibited drugs in his possession. This is what the military says
now, after the fact, to justify the warrantless search. It is so easy to make such a
claim, and I am surprised that the majority should readily accept it.

The conclusion that there was probable cause may have been influenced by the
subsequent discovery that the accused was carrying a prohibited drug. This is
supposed to justify the soldier's suspicion. In other words, it was the fact of illegal
possession that retroactively established the probable cause that validated the
illegal search and seizure. It was the fruit of the poisonous tree that washed clean
the tree itself.
In Olmstead v. U.S., 277 U.S. 438, Justice Holmes said sixty-four years ago:

. . . It is desirable that criminals should be detected, and to that end that all
available evidence should be used.1avvphi1 It is also desirable that the government
should not itself foster and pay for other crimes, when they are the means by which
the evidence is to be obtained. If it pays its officers for having got evidence by
crime, I do not see why it may not as well pay them for getting it in the same way,
and I can attach no importance to protestations of disapproval if it knowingly
accepts and pays and announces that in the future it will pay for the fruits. We have
to choose, and for my part I think it a less evil that some criminals should escape
than that the government should play an ignoble part.

If by deterring the government from playing "an ignoble part," I am "coddling


criminals," I welcome the accusation and take pride in it. I would rather err in favor
of the accused who is impaled with outlawed evidence than exalt order at the price
of liberty.

2. Examination of complainant & witness not required for WOA

CASES:
Webb v. De Leon, G.R. No. 121234, August 23, 1995

Before the Court are petitions for the issuance of the extraordinary writs
of certiorari, prohibition and mandamus with application for temporary restraining
order and preliminary injunction to: (1) annul and set aside the Warrants of Arrest
issued against petitioners by respondent Judges Raul E. de Leon and Amelita
Tolentino in Criminal Case No. 95-404; (2) enjoin the respondents from
conducting any proceeding in the aforementioned criminal case; and (3) dismiss
said criminal case or include Jessica Alfaro as one of the accused therein. 1

From the records of the case, it appears that on June 19, 1994, the National Bureau
of Investigation (NBI) filed with the Department of Justice a letter-complaint
charging petitioners Hubert Webb, Michael Gatchalian, Antonio J. Lejano and six
(6) other persons,2 with the crime of Rape with Homicide. Forthwith, the
Department of Justice formed a panel of prosecutors headed by Assistant Chief
State Prosecutor Jovencio R. Zuño to conduct the preliminary investigation 3 of
those charged with the rape and killing on June 30, 1991 of Carmela N.
Vizconde;4 her mother Estrellita Nicolas-Vizconde, 5 and her sister Anne Marie
Jennifer6 in their home at Number 80 W. Vinzons, St., BF Homes, Parañaque,
Metro Manila.
During the preliminary investigation, the NBI presented the following: (1) the
sworn statement dated May 22, 1995 of their principal witness, Maria Jessica
M. Alfaro who allegedly saw the commission of the crime; 7 (2) the sworn
statements of two (2) of the former housemaids of the Webb family in the persons
of Nerissa E. Rosales and Mila S. Gaviola;8 (3) the sworn-statement of Carlos
J. Cristobal who alleged that on March 9, 1991 he was a passenger of United
Airlines Flight No. 808 bound for New York and who expressed doubt on whether
petitioner Webb was his co-passenger in the trip; (4) the sworn statement of Lolita
Birrer, a former live-in partner of Gerardo Biong, who narrated the manner of how
Biong investigated and tried to cover up the crime at bar; 9 (5) the sworn statements
of Belen Dometita and Teofilo Minoza, two of the Vizconde maids, and the sworn
statements of Normal White, a security guard and Manciano Gatmaitan, an
engineer. The autopsy reports of the victims were also submitted and they showed
that Carmela had nine (9) stab wounds, Estrellita twelve (12) and Jennifer nineteen
(19).10 The genital examination of Carmela confirmed the presence of
spermatozoa.11

Before submitting his counter-affidavit, petitioner Webb filed with the DOJ Panel
a Motion for Production And Examination of Evidence and Documents for the
NBI to produce the following:

(a) Certification issued by the U.S. Federal Bureau of Investigation on the


admission to and stay of Hubert Webb in the United States from March 9,
1991 to October 22, 1992;

(b) Laboratory Report No. SN-91-17 of the Medico Legal Officer, Dr.
Prospero A. Cabanayan, M.D.;

(c) Sworn Statements of Gerardo C. Biong (other than his Sworn Statement
dated October 7, 1991);

(d) Photographs of fingerprints lifted from the Vizconde residence taken


during the investigation;

(e) Investigation records of NBI on Engr. Danilo Aguas, et al.;

(f) List of names of 135 suspects/persons investigated by the NBI per


Progress Report dated September 2, 1991 submitted by Atty. Arlis Vela,
Supervising Agent;

(g) Records of arrest, interview, investigation and other written statements


of Jessica Alfaro (other than the May 22, 1995 Sworn Statement) conducted
by the NBI and other police agencies;
(h) transmittal letter to the NBI, including the report of the investigation
conducted by Superintendent Rodolfo C. Sison, Regional Deputy Director,
NCRC;

(i) The names of NBI officials/agents composing the Task Force Jecares,
including their respective positions and duties;

(j) Statements made by other persons in connection with the crime charged.

The motion was granted by the DOJ Panel and the NBI submitted photocopies of
the documents. It alleged it lost the original of the April 28, 1995 sworn statement
of Alfaro. This compelled petitioner Webb to file Civil Case No. 951099 in the
Regional Trial Court (RTC) of Makati, Br. 63, for the purpose, among others, of
obtaining the original of said sworn statement. He succeeded, for in the course of
its proceedings, Atty. Arturo L. Mercader, Jr., produced a copy of said original in
compliance with a subpoena duces tecum. The original was then submitted by
petitioner Webb to the DOJ Panel together with his other evidence. It appears,
however, that petitioner Webb failed to obtain from the NBI the copy of the
Federal Bureau of Investigation (FBI) Report despite his request for its production.

Petitioner Webb claimed during the preliminary investigation that he did not
commit the crime at bar as he went to the United States on March 1, 1991 and
returned to the Philippines on October 27, 1992. 12 His alibi was corroborated by
Honesto Aragon, Lecinia Edrosolano, Sylvia Climaco, Gina Roque, Sonia
Rodriguez, Edgardo Venture and Pamela Francisco. 13 To further support his
defense, he submitted documentary evidence that he bought a bicycle and a 1986
Toyota car while in the United States on said dates 14 and that he was issued by the
State of California Driver's License No. A8818707 on June 14, 1991. 15 Petitioner
Webb likewise submitted the letter dated July 25, 1995 of Mr. Robert Heafner,
Legal Attache of the US Embassy, citing certain records tending to confirm,
among others, his arrival at San Francisco, California on March 9, 1991 as a
passenger in United Airlines Flight No. 808.

The other respondents — Hospicio "Pyke" Fernandez, Michael Gatchalian,


Antonio "Tony Boy" Lejano, Peter Estrada, Miguel Rodriguez and Gerardo Biong
— submitted sworn statements, responses, and a motion to dismiss denying their
complicity in the rape-killing of the Vizcondes. 16 Only the respondents Joey Filart
and Artemio "Dong" Ventura failed to file their counter-affidavits though they
were served with subpoena in their last known address. 17 In his sworn statement,
petitioner Gatchalian alleged that from 11 o'clock in the evening of June 29, 1991
until 3 o'clock in the morning of the following day, he was at the residence of his
friends, Carlos and Andrew Syyap, at New Alabang Village, Muntinlupa watching
video tapes. He claimed that his co-petitioner Lejano was with him.
On August 8, 1995, the DOJ Panel issued a 26-page Resolution "finding probable
cause to hold respondents for trial" and recommending that an Information for rape
with homicide be filed against petitioners and their co-respondents, 18 On the same
date, it filed the corresponding Information 19 against petitioners and their co-
accused with the Regional Trial Court of Parañaque. The case was docketed as
Criminal Case No. 95-404 and raffled to Branch 258 presided by respondent judge
Zosimo V. Escano. It was, however, the respondent judge Raul de Leon, pairing
judge of Judge Escano, who issued the warrants of arrest against the petitioners.
On August 11, 1995, Judge Escano voluntarily inhibited himself from the case to
avoid any suspicion about his impartiality considering his employment with the
NBI before his appointment to the bench. The case was re-raffled to Branch 274,
presided by Judge Amelita Tolentino who issued new warrants of arrest against the
petitioners and their co-accused. On August 11, 1995, petitioner Webb voluntarily
surrendered to the police authorities at Camp Ricardo Papa Sr., in Bicutan, Taguig.
Petitioners Gatchalian and Lejano likewise gave themselves up to the authorities
after filing their petitions before us.

In their petitions at bar, petitioners contend: (1) respondent Judges de Leon and
Tolentino gravely abused their discretion when they failed to conduct a preliminary
examination before issuing warrants of arrest against them: (2) the DOJ Panel
likewise gravely abused its discretion in holding that there is probable cause to
charge them with the crime of rape with homicide; (3) the DOJ Panel denied them
their constitutional right to due process during their preliminary investigation; and
(4) the DOJ Panel unlawfully intruded into judicial prerogative when it failed to
charge Jessica Alfaro in the Information as an accused.

We find the petitions bereft of merit.

Petitioners fault the DOJ Panel for its finding of probable cause. They insist
that the May 22, 1995 sworn statement of Jessica Alfaro is inherently weak
and uncorroborated. They hammer on alleged material inconsistencies
between her April 28, 1995 and May 22, 1995 sworn statements. They assail
her credibility for her misdescription of petitioner Webb's hair as semi-
blonde. They also criticize the procedure followed by the DOJ Panel when it
did not examine witnesses to clarify the alleged incredulities and
inconsistencies in the sworn statements of the witnesses for the NBI.

We start with a restatement of the purpose of a preliminary investigation.


Section 1 of Rule 112 provides that a preliminary investigation should
determine " . . . whether there is a sufficient ground to engender a well-
grounded belief that a crime cognizable by the Regional Trial Court has
been committed and that the respondent is probably guilty thereof, and
should be held for trial." Section 3 of the same Rule outlines the procedure
in conducting a preliminary investigation, thus:

Sec. 3. Procedure. — Except as provided for in Section 7 hereof, no


complaint or information for an offense cognizable by the Regional
Trial Court shall be filed without a preliminary investigation having
been first conducted in the following manner:

(a) The complaint shall state the known address of the respondent and
be accompanied by affidavits of the complainant and his witnesses as
well as other supporting documents, in such number of copies as
there are respondents, plus two (2) copies for the official file. The
said affidavits shall be sworn to before any fiscal, state prosecutor or
government official authorized to administer oath, or, in their absence
or unavailability, a notary public, who must certify that he personally
examined the affiants and that he is satisfied that they voluntarily
executed and understood their affidavits.

(b) Within ten (10) days after the filing of the complaint, the
investigating officer shall either dismiss the same if he finds no
ground to continue with the inquiry, or issue a subpoena to the
respondent, attaching thereto a copy of the complaint, affidavits and
other supporting documents. Within ten (10) days from receipt
thereof, the respondent shall submit counter-affidavits and other
supporting documents. He shall have the right to examine all other
evidence submitted by the complainant.

(c) Such counter-affidavits and other supporting evidence submitted


by the respondent shall also be sworn to and certified as prescribed in
paragraph (a) hereof and copies thereof shall be furnished by him to
the complainant.

(d) If the respondent cannot be subpoenaed, or if subpoenaed, does


not submit counter-affidavits within the ten (10) day period, the
investigating officer shall base his resolution on the evidence
presented by the complainant.

(e) If the investigating officer believes that there are matters to be


clarified, he may set a hearing to propound clarificatory questions to
the parties or their witnesses, during which the parties shall be
afforded an opportunity to be present but without the right to examine
or cross-examine. If the parties so desire, they may submit questions
to the investigating officer which the latter may propound to the
parties or witnesses concerned.
(f) Thereafter, the investigation shall be deemed concluded, and the
investigating officer shall resolve the case within ten (10) days
therefrom. Upon the evidence thus adduced, the investigating officer
shall determine whether or not there is sufficient ground to hold the
respondent for trial.

Section 4 of Rule 112 then directs that "if the investigating fiscal finds cause
to hold the respondent for trial, he shall prepare the resolution and
corresponding information. He shall certify under oath that he, or as shown
by the record, an authorized officer, has personally examined the
complainant and his witnesses, that there is reasonable ground to believe
that a crime has been committed and that the accused is probably guilty
thereof . . ."

The need to find probable cause is dictated by the Bill of Rights which protects
"the right of the people to be secure in their persons . . . against unreasonable
searches and seizures of whatever nature . . ." 20 An arrest without a probable cause
is an unreasonable seizure of a person, and violates the privacy of persons which
ought not to be intruded by the State.21 Probable cause to warrant arrest is not an
opaque concept in our jurisdiction. Continuing accretions of case law reiterate that
they are facts and circumstances which would lead a reasonably discreet and
prudent man to believe that an offense has been committed by the person sought to
be arrested.22 Other jurisdictions utilize the term man of reasonable caution 23 or
the term ordinarily prudent and cautious man.24 The terms are legally synonymous
and their reference is not to a person with training in the law such as a prosecutor
or a judge but to the average man on the street.25 It ought to be emphasized that in
determining probable cause, the average man weighs facts and circumstances
without resorting to the calibrations of our technical rules of evidence of which his
knowledge is nil. Rather, he relies on the calculus of common sense of which all
reasonable men have an abundance.

Applying these basic norms, we are not prepared to rule that the DOJ Panel
gravely abused its discretion when it found probable cause against the
petitioners. Petitioners belittle the truthfulness of Alfaro on two (2) grounds:
(a) she allegedly erroneously described petitioner Webb's hair as semi-blond
and (b) she committed material inconsistencies in her two (2) sworn
statement, thus:26

xxx xxx xxx

To illustrate, the following are some examples of inconsistencies in


the two sworn statements of Alfaro:

On whether Alfaro knew Carmela before the incident in question


First Affidavit: She had NOT met Carmela before June
29, 1991.

Second Affidavit: "I met her in a party sometime in


February, 1991."

On whether Alfaro saw the dead bodies

First Affidavit: She did not see the three dead persons
on that night. She just said "on the following day I read
in the newspaper that there were three persons who were
killed . . ."

Second Affidavit: "I peeped through the first door on the


left. I saw two bodies on top of the bed, bloodied, and in
the floor, I saw Hubert on top of Carmela."

On the alleged rape of Carmela Vizconde

First Affidavit: She did not see the act of rape.

Second Affidavit: She saw Hubert Webb "with bare


buttocks, on top of Carmela and pumping, her mouth
gagged and she was moaning and I saw tears on her
eyes."

On how Webb, Lejano, and Ventura entered the Vizconde house

First Affidavit: "by jumping over the fence, which was


only a little more than a meter high."

Second Affidavit: They "entered the gate which was


already open."

On whether Alfaro entered the Vizconde house

First Affidavit: She never entered the house.

Second Affidavit: "I proceeded to the iron grill gate


leading to the dirty kitchen."

In its Resolution, the DOJ Panel ruled that these alleged misdescription and
inconsistencies did not erode the credibility of Alfaro. We quote the
pertinent ruling, viz.:27

xxx xxx xxx


As regards the admissibility of Alfaro's statements, granting for
purposes of argument merely that she is a co-conspirator, it is well to
note that confessions of a co-conspirator may be taken as evidence to
show the probability of the co-conspirator's participation in the
commission of the crime (see People vs. Lumahang, 94 Phil. 1084).

Furthermore, it is a well-established doctrine that conspiracy need not


be proved by direct evidence of prior agreement to commit the crime.
Indeed, "only rarely would such a prior agreement be demonstrable
since, in the nature of things, criminal undertakings are only rarely
documented by agreements in writing. Thus, conspiracy may be
inferred from the conduct of the accused before, during and after the
commission of the crime, showing that the several accused had acted
in concert or in unison with each other, evincing a common purpose
or design." (Angelo vs. Court of Appeals, 210 SCRA 402 [1992],
citations omitted; People vs. Molleda, 86 SCRA 699).

Neither can we discredit Alfaro merely because of the inconsistencies


in her two sworn statements. In Angelo, the Court refused to discredit
the testimony of a witness accusing therein petitioner for the slaying
of one Gaviano Samaniego even though said witness failed to name
Angelo in his affidavit which was executed five (5) months earlier.
Granting, the Court continued, that a part of the witness' testimony is
untrue, such circumstance is not sufficient to discredit the entire
testimony of the witness.

On August 7, 1995, another counsel for respondent Webb submitted


his memorandum suggesting that the instant complaint "should not be
decided within the month to give time to the NBI to coordinate with
the FBI on the latter's inquiry into the whereabouts of Hubert Webb . .
. and to check on our U.S.-based witnesses."

In said memorandum, counsel for respondent Webb calls for the


application of the maxim falsus in uno, falsus in omnibus arising
from the inconsistencies of Alfaro's statements, among others. This is
untenable. As held in Angelo:

There is no rule of law which prohibits a court from


crediting part of the testimony of a witness as worthy of
belief and from simultaneously rejecting other parts
which the court may find incredible or dubious. The
maxim falsus in uno, falsus in omnibus is not a rule of
law, let alone a general rule of law which is universally
applicable. It is not a legal presumption either. It is
merely a latinism describing the conclusion reached by a
court in a particular case after ascribing to the evidence
such weight or lack of weight that the court deemed
proper.

In the case before us, complainant reasoned out that Alfaro was then
having reservations when she first executed the first statement and
held back vital information due to her natural reaction of mistrust.
This being so, the panel believes that the inconsistencies in Alfaro's
two sworn statements have been sufficiently explained especially
specially so where there is no showing that the inconsistencies were
deliberately made to distort the truth. Consequently, the probative
value of Alfaro's testimony deserves full faith and credit. As it has
been often noted, ex parte statements are generally incomplete
because they are usually executed when the affiant's state of mind
does not give her sufficient and fair opportunity to comprehend the
import of her statement and to narrate in full the incidents which
transpired (People vs. Sarellana, 233 SCRA 31 [1994]; Angelo vs.
Court of Appeals, supra). In the case at bar, there is no dispute that a
crime has been committed and what is clear before us is that the
totality of the evidence submitted by the complainant indicate
a prima facie case that respondents conspired in the perpetration of
the imputed offense.

We note that the May 22, 1995 sworn statement of Alfaro was given with the
assistance of counsel28 and consists of six (6) pages, in single space reciting in rich
details how the crime was planned and then executed by the petitioners. In
addition, the DOJ Panel evaluated the supporting sworn statements of Nerissa
Rosales and Mila Gaviola, former housemaids of the Webbs, Carlos J. Cristobal, a
passenger in United Airlines Flight No. 808 and Lolita Birrer, a paramour of
Gerardo Biong. The Panel assayed their statements as follows: 29

xxx xxx xxx

According to Nerissa E. Rosales, a former housemaid of the Webb


family, on June 29, 1991, between 7:00 o'clock and 8:00 o'clock in
the evening, Hubert was at home inside his room with two male
visitors. She knew it because she and her co-housemaid, Loany, were
instructed by Hubert to bring them three glasses of juice. It was the
last time she saw Hubert and was later told by then Congressman
Webb that Hubert was in the United States.

While Mila S. Gaviola, another former housemaid of the Webb


family and who served as a laundry woman, claims, aside from
corroborating the statement of Nerissa Rosales, that on June 30, 1991,
she woke up at around 4:00 in the morning and as what she used to
do, she entered the rooms of the Webbs to get their clothes to be
washed. As a matter of fact, in that early morning, she entered
Hubert's room and saw Hubert, who was only wearing his pants,
already awake and smoking while he was sitting on his bed. She
picked up Hubert's scattered clothes and brought them together with
the clothes of the other members of the family to the laundry area.
After taking her breakfast, she began washing the clothes of the
Webbs. As she was washing the clothes of Hubert Webb, she noticed
fresh bloodstains in his shirt. After she finished the laundry, she went
to the servant's quarters. But feeling uneasy, she decided to go up to
the stockroom near Hubert's room to see what he was doing. In the
said stockroom, there is a small door going to Hubert's room and in
that door there is a small opening where she used to see Hubert and
his friends sniffing on something. She observed Hubert was quite
irritated, uneasy, and walked to and from inside his room.

On that day, she noticed Hubert left the house at around 1:00 in the
afternoon and came back at around 4:00 in the same afternoon and
went inside his room using the secret door of the house. It was the last
time that she saw Hubert until she left the Webb family.

On the other hand, Carlos J. Cristobal alleged that on March 9, 1991,


at about 10:00 in the morning, he was at the Ninoy Aquino
International Airport as he was then scheduled to take the United
Airlines Flight No. 808 at 2:00 in the afternoon for New York. At the
airport's lobby, he saw then Congressman Freddie Webb with a male
companion. He greeted him and Webb answered: "Mabuti naman, at
ito, ihahatid ko ang anak ko papuntang Florida." He knew Freddie
Webb because he often watched him then in a television show
"Chicks to Chicks." He observed that the man whom Freddie Webb
referred to as his son, was of the same height as Freddie. The son
referred to has fair complexion with no distinguishing marks on his
face. He (son of Webb) was then wearing a striped white jacket.
When he and his children were already inside the plane, he did not
see Freddie anymore, but he noticed his son was seated at the front
portion of the economy class. He never noticed Freddie Webb's son
upon their arrival in San Francisco. He claims that, while watching
the television program "DONG PUNO LIVE" lately, he saw the wife
of Freddie Webb with her lawyer being interviewed, and when she
described Hubert as "moreno" and small built, with a height of five
feet and seven inches tall, and who was the one who left for United
States on March 9, 1991, he nurtured doubts because such description
does not fit the physical traits of the son of Freddie, who left with him
for United States on the same flight and date.
Lolita Birrer, alleged that she know Gerardo Biong because she had
an affair with him for almost three (3) years and in fact, she had a
child with him who is now four (4) years old. Their relationship
started in February, 1991 until she broke up with him in September
1993. She recalls that on June 29, 1991, at around 6:00 p.m., Biong
invited her to play mahjong at the canteen of a certain Aling Glo
located at the back of the Parañaque Municipal Hall.

At about 2:30, in the early morning of January 30, 1991, the radio
operator of the Parañaque police told Biong that he has a phone call.
Before Biong went to the radio room, she was instructed to take him
over and after somebody won the game, she followed Biong at the
radio room where she overheard him uttering, "Ano?, Saan? Mahirap
yan, Paano, o sige, aantayin kita, O ano?, dilaw na taxi, o sige."
When he put the phone down, Biong told her, "Mayroon lang akong
rerespondehan, ikaw muna ang maupo" and then, he went outside the
canteen apparently waiting for somebody. Twenty minutes later, a
taxi, colored yellow, arrived with a male passenger sitting at the
backseat and parked near the canteen. After it made some signals by
blinking its headlight, Biong rode thereat at the front seat beside the
driver and then, they left. She was not able to recognize the male
passenger because the window of the taxi was tinted. Biong came
back at around 7:00 of the same morning and when he arrived, he
immediately washed his hands and face, and took his handkerchief
from his pocket which he threw at the trash can. She asked him why
he threw his handkerchief and he answered, "Hmp . . . amoy tae." She
inquired what happened in BF Homes and he replied, "Putang inang
mga batang iyon, pinahirapan nila ako."

Biong later invited her for breakfast, but they first went to his office
where she observed him doing something in his steel cabinet while he
appeared to be uneasy. Moments later, Galvan, another policeman of
Parañaque, arrived and said, "Oy Biong, may tatlong patay sa BF,
imbestigahan mo" to which Biong answered, "Oo susunod na ako."
Biong went to the office of Capt. Don Bartolome who offered to
accompany him and with whom she asked permission to go with
them. Before they proceeded to the place where the killings
happened, she asked Biong if he knew the exact address and the latter
immediately responded, "Alam ko na yon." She was surprised because
Galvan never told him the place of the incident.

As soon as they arrived at the Vizconde's residence, Biong instructed


the housemaids to contact the victim's relatives, while the security
guard fetched the barangay chairman and the president of the
Homeowners Association. When all these persons were already in the
house, Biong started recording the wounds of the victim. Inside the
master's bedroom, she saw Biong took a watch from the jewelry box.
Because she could not tolerate the foul odor, she and Capt. Bartolome
went out of the room and proceeded to the dining area. On top of the
dining table, she saw the scattered contents of a shoulder bag.
Moments later, Biong came out from the room and proceeded to the
front door to remove the chain lock; asked the keys from the
housemaid and it was only then that the main door was opened. Biong
noticed a stone in front of the broken glass of the door and requested
Capt. Bartolome to go inside the servant's quarters as he doubted the
housemaids' claim that they heard nothing unusual. Using the handle
of his gun, Biong broke the remaining glass of the door panel.
Bartolome then came out of the room and told Biong that he can hear
the sound of the glass being broken. At the garage, Biong also noticed
same marks on the hood of the car.

On the following day, at around 12:00 noon, Biong arrived in her


house together with the Vizconde housemaids. When Biong was
preparing to take a bath, she saw him remove from his pocket the
things she also saw from Vizconde's residence, to wit: calling cards,
driver's license, ATM card, a crossed check worth P80,000.00,
earrings, a ring, bracelet, necklace, and the watch he took from the
jewelry box inside the room of the Vizcondes. These jewelry items
were later pawned by Biong for P20,000.00 at a pawnshop in front of
Chow-Chow restaurant in Santos Avenue, Parañaque. The next day,
she saw Biong took from his locker at the Parañaque Police Station
an imported brown leather jacket, which the latter claimed to have
been given to him by the person who called him up in the early
morning of June 30, 1991.

Since then, Biong has been wearing said jacket until they broke up
sometime in 1993. She observed that Biong seemed not interested in
pursuing the investigation of the Vizconde case. In fact, when Biong
and this group picked up Mike Gatchalian and brought him to the
Parañaque Police Station, she was surprised that Biong halted the
investigation when Gatchalian was profusely sweating while being
interrogated. After the father of Gatchalian talked to Colonel Pureza,
the latter called up and instructed Biong to bring Gatchalian to him
(Colonel Pureza) and that was the last thing she remembered
regarding this case.

The DOJ Panel then weighed these inculpatory evidence against the exculpatory
evidence of petitioners. It ruled: 30

xxx xxx xxx


The voluminous number of exhibits submitted by respondent Webb to
support his defense of denial and alibi notwithstanding, the panel,
after a careful and thorough evaluation of the records, believes that
they cannot outweigh the evidence submitted by the complainant.
Alibi cannot prevail over the positive identification made by a
prosecution witness. Verily, alibi deserves scant consideration in the
face of positive identification especially so where the claim of alibi is
supported mainly by friends and relatives (People vs. Apolonia, 235
SCRA 124 [1994]; People vs. Lucas, 181 SCRA 316 and a long line
of cases).

Similarly, denial is a self-serving negative which cannot be given


greater evidentiary weight than the declaration of a credible witness
who testified on affirmative matters (People vs. Carizo, 233 SCRA
687 [1994]). Indeed, denial, like alibi, is weak and becomes even
more weaker when arrayed against the positive identification by the
witness for the prosecution (People vs. Onpaid, 233 SCRA 62
[1994]).

Surprisingly, Gatchalian's defense of alibi was not corroborated by


Lejano, whom he claimed was with him watching video tapes at the
Syyap residence. Other than claiming that he "was not and could not
have been at or near the area of the Vizconde residence at the time of
the alleged commission of the crime," respondent Lejano proffered no
evidence to substantiate his claim of alibi.

xxx xxx xxx

On the other hand, respondent Webb seeks to enhance the


acceptability of his alibi in the form of documents tending to show
that he was thousands of miles away when the incident occurred. We
have carefully deliberated and argued on the evidence submitted by
respondent Webb in support of his absence from the country since
March 9, 1991 to October 26, 1992 and found the same wanting to
exonerate him of the offense charged. The material dates in this case
are June 29 and 30, 1991. While respondent Webb may have
submitted proof tending to show that he was issued a California
driver's license on June 14, 1991, there is no showing that he could
not have been in the country on the dates above mentioned. Neither
do we find merit in the allegation that respondent Webb personally
bought a bicycle on June 30, 1991 in California in view of his
positive identification by Alfaro and the two (2) househelps of the
Webb family who testified that he was here in the country on said
dates. Additionally, the issuance of receipt evidencing the purchase of
a bicycle in California is no conclusive proof that the name appearing
thereon was the actual buyer of the merchandise.

Given these conflicting pieces of evidence of the NBI and the petitioners,
we hold that the DOJ Panel did not gravely abuse its discretion when it
found probable cause against the petitioners. A finding of probable cause
needs only to rest on evidence showing that more likely than not a crime has
been committed and was committed by the suspects. Probable cause need
not be based on clear and convincing evidence of guilt, neither on evidence
establishing guilt beyond reasonable doubt and definitely, not on evidence
establishing absolute certainty of guilt. As well put in Brinegar v. United
States,31 while probable cause demands more than "bare suspicion," it
requires "less than evidence which would justify . . . conviction." A finding
of probable cause merely binds over the suspect to stand trial. It is not a
pronouncement of guilt.

Considering the low quantum and quality of evidence needed to support a


finding of probable cause, we also hold that the DOJ Panel did not, gravely
abuse its discretion in refusing to call the NBI witnesses for clarificatory
questions. The decision to call witnesses for clarificatory questions is
addressed to the sound discretion of the investigator and the investigator
alone. If the evidence on hand already yields a probable cause, the
investigator need not hold a clarificatory hearing. To repeat, probable cause
merely implies probability of guilt and should be determined in a summary
manner. Preliminary investigation is not a part of trial and it is only in a trial
where an accused can demand the full exercise of his rights, such as the
right to confront and cross-examine his accusers to establish his innocence.
In the case at bar, the DOJ Panel correctly adjudged that enough evidence
had been adduced to establish probable cause and clarificatory hearing was
unnecessary.

II

We now come to the charge of petitioners that respondent Judge Raul de


Leon and, later, respondent Judge Amelita Tolentino issued warrants of
arrest against them without conducting the required preliminary
examination. Petitioners support their stance by highlighting the following
facts: (1) the issuance of warrants of arrest in a matter of few hours; (2) the
failure of said judges to issue orders of arrest; (3) the records submitted to
the trial court were incomplete and insufficient from which to base a finding
of probable cause; and (4) that even Gerardo Biong who was included in the
Information as a mere accessory had a "NO BAIL" recommendation by the
DOJ Panel. Petitioners postulate that it was impossible to conduct a
"searching examination of witnesses and evaluation of the documents" on
the part of said judges.
The issuance of a warrant of arrest interferes with individual liberty and is
regulated by no less than the fundamental law of the land. Section 2 of
Article III of the Constitution provides:

Sec. 2. The right of the people to be secure in their persons, houses,


papers, and effects against unreasonable searches and seizures of
whatever nature and for any purpose shall be inviolable, and no
search warrant or warrant of arrest shall issue except upon probable
cause to be determined personally by the judge after examination
under oath or affirmation of the complainant and the witnesses he
may produce and particularly describing the place to be searched and
the persons or things to be seized.

The aforequoted provision deals with the requirements of probable cause


both with respect to issuance of warrants of arrest or search warrants. The
similarities and differences of their requirements ought to be educational.
Some of them are pointed out by Professors LaFave and Israel, thus: 32 "It is
generally assumed that the same quantum of evidence is required whether
one is concerned with probable cause to arrest or probable cause to search.
But each requires a showing of probabilities as to somewhat different facts
and circumstances, and thus one can exist without the other. In search cases,
two conclusions must be supported by substantial evidence: that the items
sought are in fact seizable by virtue of being connected with criminal
activity, and that the items will be found in the place to be searched. It is not
also necessary that a particular person be implicated. By comparison, in
arrest cases there must be probable cause that a crime has been committed
and that the person to be arrested committed it, which of course can exist
without any showing that evidence of the crime will be found at premises
under that person's control." Worthy to note, our Rules of Court do not
provide for a similar procedure to be followed in the issuance of warrants of
arrest and search warrants. With respect to warrants of arrest, section 6 of
Rule 112 simply provides that "upon filing of an information, the Regional
Trial Court may issue a warrant for the arrest of the accused." In contrast,
the procedure to be followed in issuing search warrants is more defined.
Thus, Sections 3, 4 and 5 of Rule 126 provide:

xxx xxx xxx

Sec. 3. Requisites for issuing search warrant. — A search warrant


shall not issue but upon probable cause in connection with one
specific offense to be determined personally by the judge after
examination under oath or affirmation of the complainant and the
witnesses he may produce, and particularly describing the place to be
searched and the things to be seized.
Sec. 4. Examination of complainant; record. — The judge must,
before issuing the warrant, personally examine in the form of
searching questions and answers, in writing and under oath the
complainant and any witnesses he may produce on facts personally
known to them and attach to the record their sworn statements
together with any affidavits submitted.

Sec. 5. Issuance and form of search warrant. — If the judge is


thereupon satisfied of the facts upon which the application is based,
or that there is probable cause to believe that they exist, he must issue
the warrant, which must be substantially in the form prescribed by
these Rules.

We discussed the difference in the Procedure of issuing warrants of arrest


and search warrants in Soliven vs. Makasiar,33 thus:

xxx xxx xxx

The second issue, raised by Beltran, calls for an interpretation of the


constitutional provision on the issuance of warrants of arrest. The
pertinent provision reads:

Art. III, Sec. 2. The right of the people to be secure in


their persons, houses, papers and effects against
unreasonable searches and seizures of whatever nature
and for any purpose shall be inviolable, and no search
warrant or warrant of arrest shall issue except upon
probable cause to be determined personally by the judge
after examination under oath or affirmation of the
complainant and the witnesses he may produce, and
particularly describing the place to be searched and the
persons or things to be seized.

The addition of the word "personally" after the word "determined"


and the deletion of the grant of authority by the 1973 Constitution to
issue warrants to "other responsible officers as may be authorized by
law," has apparently convinced petitioner Beltran that the
Constitution now requires the judge to personally examine the
complainant and his witnesses in his determination of probable cause
for the issuance of warrants of arrest. This is not an accurate
interpretation.

What the Constitution underscores is the exclusive and personal


responsibility of the issuing judge to satisfy himself of the existence of
probable cause. In satisfying himself of the existence of probable
cause for the issuance of a warrant of arrest, the judge is not
required to personally examine the complainant and his witnesses.
Following established doctrine and procedure, he shall: (1) personally
evaluate the report and the documents submitted by the fiscal
regarding the existence of probable cause and, on the basis thereof,
issue a warrant; or (2) if on the basis thereof he finds no probable
cause, he may disregard the fiscal's report and require the submission
of supporting affidavits of witnesses to aid him in arriving at a
conclusions as to the existence of probable cause.

Sound policy dictates this procedure, otherwise judges would be


unduly laden with the preliminary examination and investigation of
criminal complaints instead of concentrating on hearing and deciding
cases filed before their courts.

Clearly then, the Constitution, the Rules of Court, and our case
law34 repudiate the submission of petitioners that respondent judges should
have conducted "searching examination of witnesses" before issuing
warrants of arrest against them. They also reject petitioners' contention that
a judge must first issue an order of arrest before issuing a warrant of arrest.
There is no law or rule requiring the issuance of an Order of Arrest prior to a
warrant of arrest.

In the case at bar, the DOJ Panel submitted to the trial court its 26-page
report, the two (2) sworn statements of Alfaro and the sworn statements of
Carlos Cristobal and Lolita Birrer35 as well as the counter-affidavits of the
petitioners. Apparently, the painstaking recital and analysis of the parties'
evidence made in the DOJ Panel Report satisfied both judges that there is
probable cause to issue warrants of arrest against petitioners. Again, we
stress that before issuing warrants of arrest, judges merely
determine personally the probability, not the certainty of guilt of an
accused. In doing so, judges do not conduct a de novo hearing to determine
the existence of probable cause. They just personally review the initial
determination of the prosecutor finding a probable cause to see if it is
supported by substantial evidence. The sufficiency of the review process
cannot be measured by merely counting minutes and hours. The fact that it
took the respondent judges a few hours to review and affirm the probable
cause determination of the DOJ Panel does not mean they made no personal
evaluation of the evidence attached to the records of the case. 36

Petitioners' reliance on the case of Allado vs. Diokno37 is misplaced.


Our Allado ruling is predicated on the utter failure of the evidence to show
the existence of probable cause. Not even the corpus delicti of the crime was
established by the evidence of the prosecution in that case. Given the clear
insufficiency of the evidence on record, we stressed the necessity for the
trial judge to make a further personal examination of the complainant and
his witnesses to reach a correct assessment of the existence or non-existence
of probable cause before issuing warrants of arrest against the accused. The
case at bar, however, rests on a different factual setting. As priorly
discussed, the various types of evidence extant in the records of the case
provide substantial basis for a finding of probable cause against the
petitioner. The corpus delicti of the crime is a given fact. There is an
eyewitness account of the imputed crime given by Alfaro. The alibi defense
of petitioner Webb is also disputed by sworn statements of their former
maids. It was therefore unnecessary for the respondent judges to take the
further step of examining ex parte the complainant and their witnesses with
searching questions.

III

Petitioners also complain about the denial of their constitutional right to due
process and violation of their right to an impartial investigation. They decry
their alleged hasty and malicious prosecution by the NBI and the DOJ Panel.
They also assail the prejudicial publicity that attended their preliminary
investigation.

We reject these contentions. The records will show that the DOJ Panel did
not conduct the preliminary investigation with indecent haste. Petitioners
were given fair opportunity to prove lack of probable cause against them.
The fairness of this opportunity is well stressed in the Consolidated
Comment of the Solicitor General, viz.:

Again, there is no merit in this contention. Petitioners were afforded


all the opportunities to be heard. Petitioner Webb actively
participated in the preliminary investigation by appearing in the initial
hearing held on June 30, 1995 and in the second hearing on July 14,
1995; and by filing a "Motion for Production and Examination of
Evidence and Documents" on June 27, 1995 (p. 4, Petition), a "Reply
to the compliance and Comment/Manifestation to the Motion for
Production and Examination of Evidence" on July 5, 1995 (p. 6,
Petition), a "Comment and Manifestation" on July 7, 1995 (p. 6,
Petition), his "Counter-Affidavit" on July 14, 1995 (pp. 6-7, Petition)
and a "Motion to Resolve" on August 1, 1995. Numerous letter-
requests were also sent by the petitioner Webb's counsel to the DOJ
Panel requesting the latter to furnish him a copy of the reports
prepared by the FBI concerning the petitioner's whereabouts during
the material period (Annexes "L", "L-1" and "L-2" of the
Supplemental Petition dated August 14, 1995). In fact, not satisfied
with the decision of the DOJ Panel not to issue subpoena duces
tecum to Atty. Arturo L. Mercader, Jr., petitioner Webb filed a
"Petition for Injunction, Certiorari, Prohibition and Mandamus" with
the Regional Trial Court, Branch 63 of Makati in order to compel said
Atty. Mercader, Jr. to produce the first sworn statement of Alfaro for
submission to the DOJ Panel. (p. 4, Petition) The said court dismissed
the petition after Mercader produced and submitted to the DOJ Panel
the first sworn statement of Alfaro, without ruling on the
admissibility and credence of the two (2) conflicting and inconsistent
sworn statements of the principal witness, Alfaro. (Attached hereto is
a copy of the order of Judge Ruben A. Mendiola, RTC-Makati,
Branch 63 dated July 28, 1995) marked as Annex "F."

It must also be pointed out that despite the declaration by the DOJ
Panel that the preliminary investigation was to be terminated after the
hearing held on July 14, 1995, the panel continued to conduct further
proceedings, e.g. comparison of the photo-copies of the submitted
documents with the originals on July 17, 1995. (p. 7, Petition) The
panel even entertained the "Response" submitted by accused Miguel
Rodriguez on July 18, 1995. (p. 17 Resolution) In addition to these,
the panel even announced that any party may submit additional
evidence before the resolution of the case. (p. 8, Petition) From the
time the panel declared the termination of the preliminary
investigation on July 14, 1995, twenty-seven (27) days elapsed before
the resolution was promulgated, and the information eventually filed
in the Regional Trial Court of Parañaque on August 10, 1995. This
notwithstanding the directive of Section 3(f) Rule 112 of the Revised
Rules of Court that the investigating officer shall resolve the
case within ten (10) days from the termination of the preliminary
investigation. The DOJ Panel precisely allowed the parties to adduce
more evidence in their behalf and for the panel to study the evidence
submitted more fully. This directly disputes the allegation of the
petitioners that the resolution was done with indecent haste in
violation of the rights of the petitioners. During the period of twenty-
seven (27) days, the petitioners were free to adduce and present
additional evidence before the DOJ Panel.

Verily, petitioners cannot now assert that they were denied due
process during the conduct of the preliminary investigation simply
because the DOJ Panel promulgated the adverse resolution and filed
the Information in court against them.

Petitioners cannot also assail as premature the filing of the Information in


court against them for rape with homicide on the ground that they still have
the right to appeal the adverse resolution of the DOJ Panel to the Secretary
of Justice. The filing of said Information is in accord with Department of
Justice Order No. 223, series of 1993, dated June 25, 1993. We quote its
pertinent sections, viz.:

Sec. 4. Non-Appealable Cases; Exceptions. — No appeal may be


taken from a resolution of the Chief State Prosecutor/Regional State
Prosecutor/Provincial or City Prosecutor finding probable
cause except upon showing of manifest error or grave abuse of
discretion. Notwithstanding the showing of manifest error or grave
abuse of discretion, no appeal shall be entertained where the
appellant had already been arraigned. If the appellant is arraigned
during the pendency of the appeal, said appeal shall be
dismissed motu propio by the Secretary of Justice.

An appeal/motion for reinvestigation from a resolution finding


probable cause, however, shall not hold the filing of the information
in court.

Sec. 2. When to appeal. — The appeal must be filed within a period


of fifteen (15) days from receipt of the questioned resolution by the
party or his counsel. The period shall be interrupted only by the filing
of a motion for reconsideration within ten (10) days from receipt of
the resolution and shall continue to run from the time the resolution
denying the motion shall have been received by the movant or his
counsel. (Emphasis supplied)

Without doubt then, the said DOJ Order No. 223 allows the filing of an
Information in court after the consummation of the preliminary investigation
even if the accused can still exercise the right to seek a review of the
prosecutor's recommendation with the Secretary of Justice.

Next, petitioners fault the DOJ Panel for not including Alfaro in the
Information considering her alleged conspiratorial participation in the crime
of rape with homicide. The non-inclusion of Alfaro is anchored on Republic
Act
No. 6981, entitled "An Act Providing For A Witness Protection, Security
And Benefit Program And For Other Purposes" enacted on April 24, 1991.
Alfaro qualified under its Section 10, which provides:

xxx xxx xxx

Sec. 10. State Witness. — Any person who has participated in the


commission of a crime and desires to a witness for the State, can
apply and, if qualified as determined in this Act and by the
Department, shall be admitted into the Program whenever the
following circumstances are present:
(a) the offense in which his testimony will be used is a grave felony
as defined under the R.P.C. or its equivalent under special laws;

(b) there is absolute necessity for his testimony;

(c) there is no other direct evidence available for the proper


prosecution of the offense committed;

(d) his testimony can be substantially corroborated on its material


points;

(e) he does not appear to be most guilty; and

(f) he has not at anytime been convicted of any crime involving moral
turpitude.

An accused discharged from an information or criminal complaint by


the court in order that he may be a State Witness pursuant to Sections
9 and 10 of Rule 119 of the Revised Rules of Court may upon his
petition be admitted to the Program if he complies with the other
requirements of this Act. Nothing in this Act shall prevent the
discharge of an accused so that he can be used as a Witness under
Rule 119 of the Revised Rules of Court.

Upon qualification of Alfaro to the program, Section 12 of the said law


mandates her non-inclusion in the criminal Complaint or Information, thus:

xxx xxx xxx

Sec. 12. Effect of Admission of a State Witness into the Program. —


The certification of admission into the Program by the Department
shall be given full faith and credit by the provincial or city prosecutor
who is required NOT TO INCLUDE THE WITNESS IN THE
CRIMINAL COMPLAINT OR INFORMATION and if included
therein, to petition the court for his discharge in order that he can be
utilized as a State Witness. The court shall order the discharge and
exclusion of the said accused from the information.

Admission into the Program shall entitle such State Witness to


immunity from criminal prosecution for the offense or offenses in
which his testimony will be given or used and all the rights and
benefits provided under Section 8 hereof.

The validity of these provisions is challenged by petitioner Webb. It is urged


that they constitute ". . . an intrusion into judicial prerogative for it is only
the court which has the power under the Rules on Criminal Procedure to
discharge an accused as a state witness." The argument is based on Section
9, Rule 11938 which gives the court the prerogative to approve the discharge
of an accused to be a state witness. Petitioner's argument lacks appeal for it
lies on the faulty assumption that the decision whom to prosecute is a
judicial function, the sole prerogative of courts and beyond executive and
legislative interference. In truth, the prosecution of crimes appertains to the
executive department of government whose principal power and
responsibility is to see that our laws are faithfully executed. A necessary
component of this power to execute our laws is the right to prosecute their
violators. The right to prosecute vests the prosecutor with a wide range of
discretion — the discretion of whether, what and whom to charge, the
exercise of which depends on a smorgasbord of factors which are best
appreciated by prosecutors. We thus hold that it is not constitutionally
impermissible for Congress to enact R.A. No. 6981 vesting in the
Department of Justice the power to determine who can qualify as a witness
in the program and who shall be granted immunity from
prosecution.39 Section 9 of Rule 119 does not support the proposition that
the power to choose who shall be a state witness is an inherent judicial
prerogative. Under this provision, the court, is given the power to discharge
a state witness only because it has already acquired jurisdiction over the
crime and the accused. The discharge of an accused is part of the exercise of
jurisdiction but is not a recognition of an inherent judicial function.
Moreover, the Rules of Court have never been interpreted to be beyond
change by legislation designed to improve the administration of our justice
system. R.A. No. 6981 is one of the much sought penal reform laws to help
government in its uphill fight against crime, one certain cause of which is
the reticence of witnesses to testify. The rationale for the law is well put by
the Department of Justice, viz.: "Witnesses, for fear of reprisal and economic
dislocation, usually refuse to appear and testify in the
investigation/prosecution of criminal complaints/cases. Because of such
refusal, criminal complaints/cases have been dismissed for insufficiency
and/or lack of evidence. For a more effective administration of criminal
justice, there was a necessity to pass a law protecting witnesses and granting
them certain rights and benefits to ensure their appearance in investigative
bodies/courts."40 Petitioner Webb's challenge to the validity of R.A. No.
6981 cannot therefore succeed.

Further, petitioners charge the NBI with violating their right to discovery
proceedings during their preliminary investigation by suppressing the April
28, 1995 original copy of the sworn statement of Alfaro and the FBI Report.
The argument is novel in this jurisdiction and as it urges an expansive
reading of the rights of persons under preliminary investigation it deserves
serious consideration. To start with, our Rules on Criminal Procedure do not
expressly provide for discovery proceedings during the preliminary
investigation stage of a criminal proceeding. 41 Sections 10 and 11 of Rule
117 do provide an accused the right to move for a bill of particulars and for
production or inspection of material evidence in possession of the
prosecution.42 But these provisions apply after the filing of the Complaint or
Information in court and the rights are accorded to the accused to assist them
to make an intelligent plea at arraignment and to prepare for trial. 43

This failure to provide discovery procedure during preliminary investigation


does not, however, negate its use by a person under investigation when
indispensable to protect his constitutional right to life, liberty and property.
Preliminary investigation is not too early a stage to guard against any
significant erosion of the constitutional right to due process of a potential
accused. As aforediscussed, the object of a preliminary investigation is to
determine the probability that the suspect committed a crime. We hold that
the finding of a probable cause by itself subjects the suspect's life, liberty
and property to real risk of loss or diminution. In the case at bar, the risk to
the liberty of petitioners cannot be understated for they are charged with the
crime of rape with homicide, a non-bailable offense when the evidence of
guilt is strong.

Attuned to the times, our Rules have discarded the pure inquisitorial system
of preliminary investigation. Instead, Rule 112 installed a quasi-judicial type
of preliminary investigation conducted by one whose high duty is to be fair
and impartial.44 As this Court emphasized in Rolito Go vs. Court of
Appeals,45 "the right to have a preliminary investigation conducted before
being bound over for trial for a criminal offense, and hence formally at risk
of incarceration or some other penalty, is not a mere formal or technical
right; it is a substantive right." A preliminary investigation should therefore
be scrupulously conducted so that the constitutional right to liberty of a
potential accused can be protected from any material damage. We uphold
the legal basis of the right of petitioners to demand from their prosecutor,
the NBI, the original copy of the April 28, 1995 sworn statement of Alfaro
and the FBI Report during their preliminary investigation considering their
exculpatory character, and hence, unquestionable materiality to the issue of
their probable guilt. The right is rooted on the constitutional protection of
due process which we rule to be operational even during the preliminary
investigation of a potential accused. It is also implicit in section (3) (a) of
Rule 112 which requires during the preliminary investigation the filing of a
sworn complaint, which shall ". . . state the known address of the respondent
and be accompanied by affidavits of the complainant and his witnesses as
well as other supporting documents . . ."

In laying down this rule, the Court is not without enlightened precedents
from other jurisdictions. In the 1963 watershed case of Brady
v. Maryland 46 the United States Supreme Court held that "suppression of
evidence favorable to an accused upon request violates due process where
the evidence is material to guilt or punishment, irrespective of the good faith
or bad faith of the prosecution." Its progeny is the 1935 case of Mooney
v. Holohan 47 which laid down the proposition that a prosecutor's
intentional use of perjured testimony to procure conviction violates due
process. Thus, evolved jurisprudence firming up the prosecutor's duty to
disclose to the defense exculpatory evidence in its possession. 48 The
rationale is well put by Justice Brennan in Brady49 — "society wins not only
when the guilty are convicted but when criminal trials are fair." Indeed,
prosecutors should not treat litigation like a game of poker where surprises
can be sprung and where gain by guile is not punished.

But given the right of petitioners to compel the NBI to disclose exculpatory
evidence in their favor, we are not prepared to rule that the initial non-
production of the original sworn statement of Alfaro dated April 28, 1995
could have resulted in the reasonable likelihood that the DOJ Panel would
not have found probable cause. To be sure, the NBI, on July 4, 1995, upon
request of petitioners, submitted a photocopy of Alfaro's April 28, 1995
sworn statement. It explained it cannot produce the original as it had been
lost. Fortunately, petitioners, on July 28, 1995, were able to obtain a copy of
the original from Atty. Arturo Mercader in the course of the proceedings in
Civil Case No. 951099.50 As petitioners admit, the DOJ Panel accepted the
original of Alfaro's April 28, 1995 sworn statement as a part of their
evidence.51 Petitioners thus had the fair chance to explain to the DOJ Panel
then still conducting their preliminary investigation the exculpatory aspects
of this sworn statement. Unfortunately for petitioners, the DOJ Panel still
found probable cause to charge them despite the alleged material
discrepancies between the first and second sworn statements of Alfaro. For
reasons we have expounded, this finding of probable cause cannot be struck
down as done with grave abuse of discretion. 52 On the other hand, the FBI
Report while corroborative of the alibi of petitioner Webb cannot by itself
reverse the probable cause finding of the DOJ Panel in light of the totality of
evidence presented by the NBI.

Finally, we come to the argument of petitioner that the DOJ Panel lost its
impartiality due to the prejudicial publicity waged in the press and broadcast
media by the NBI.

Again, petitioners raise the effect of prejudicial publicity on their right to


due process while undergoing preliminary investigation. We find no
procedural impediment to its early invocation considering the substantial
risk to their liberty while undergoing a preliminary investigation.
In floating this issue, petitioners touch on some of the most problematic
areas in constitutional law where the conflicting demands of freedom of
speech and of the press, the public's right to information, and an accused's
right to a fair and impartial trial collide and compete for prioritization. The
process of pinpointing where the balance should be struck has divided men
of learning as the balance keeps moving either on the side of liberty or on
the side of order as the tumult of the time and the welfare of the people
dictate. The dance of balance is a difficult act to follow.

In democratic settings, media coverage of trials of sensational cases cannot


be avoided and oftentimes, its excessiveness has been aggravated by kinetic
developments in the telecommunications industry. For sure, few cases can
match the high volume and high velocity of publicity that attended the
preliminary investigation of the case at bar. Our daily diet of facts and
fiction about the case continues unabated even today. Commentators still
bombard the public with views not too many of which are sober and
sublime. Indeed, even the principal actors in the case — the NBI, the
respondents, their lawyers and their sympathizers — have participated in
this media blitz. The possibility of media abuses and their threat to a fair
trial notwithstanding, criminal trials cannot be completely closed to the
press and the public. In the seminal case of Richmond Newspapers,
Inc. v. Virginia,53 it was wisely held:

xxx xxx xxx

(a) The historical evidence of the evolution of the criminal trial in


Anglo-American justice demonstrates conclusively that at the time
this Nation's organic laws were adopted, criminal trials both here and
in England had long been presumptively open, thus giving assurance
that the proceedings were conducted fairly to all concerned and
discouraging perjury, the misconduct of participants, or decisions
based on secret bias or partiality. In addition, the significant
community therapeutic value of public trials was recognized: when a
shocking crime occurs, a community reaction of outrage and public
protest often follows, and thereafter the open processes of justice
serve an important prophylactic purpose, providing an outlet for
community concern, hostility, and emotion. To work effectively, it is
important that society's criminal process "satisfy the appearance of
justice," Offutt v. United States, 348 US 11, 14, 99 L Ed 11, 75 S Ct
11, which can best be provided by allowing people to observe such
process. From this unbroken, uncontradicted history, supported by
reasons as valid today as in centuries past, it must be concluded that a
presumption of openness inheres in the very nature of a criminal trial
under this Nation's system of justice, Cf., e.g., Levine v. United
States, 362 US 610, 4 L Ed 2d 989, 80 S Ct 1038.
(b) The freedoms of speech, press, and assembly, expressly
guaranteed by the First Amendment, share a common core purpose of
assuring freedom of communication on matters relating to the
functioning of government. In guaranteeing freedoms such as those of
speech and press, the First Amendment can be read as protecting the
right of everyone to attend trials so as to give meaning to those
explicit guarantees; the First Amendment right to receive information
and ideas means, in the context of trials, that the guarantees of speech
and press, standing alone, prohibit government from summarily
closing courtroom doors which had long been open to the public at
the time the First Amendment was adopted. Moreover, the right of
assembly is also relevant, having been regarded not only as an
independent right but also as a catalyst to augment the free exercise
of the other First Amendment rights with which it was deliberately
linked by
the draftsmen. A trial courtroom is a public place where the people
generally — and representatives of the media — have a right to be
present, and where their presence historically has been thought to
enhance the integrity and quality of what takes place.

(c) Even though the Constitution contains no provision which by its


terms guarantees to the public the right to attend criminal trials,
various fundamental rights, not expressly guaranteed, have been
recognized as indispensable to the enjoyment of enumerated rights.
The right to attend criminal trials is implicit in the guarantees of the
First Amendment; without the freedom to attend such trials, which
people have exercised for centuries, important aspects of freedom of
speech and of the press could be eviscerated.

Be that as it may, we recognize that pervasive and prejudicial publicity


under certain circumstances can deprive an accused of his due process right
to fair trial. Thus, in Martelino, et al. vs. Alejandro, et al.,54 we held that to
warrant a finding of prejudicial publicity there must be allegation and
proof that the judges have been unduly influenced, not simply that they
might be, by the barrage of publicity. In the case at bar, we find nothing in
the records that will prove that the tone and content, of the publicity that
attended the investigation of petitioners fatally infected the fairness and
impartiality of the DOJ Panel. Petitioners cannot just rely on the subliminal
effects of publicity on the sense of fairness of the DOJ Panel, for these are
basically unbeknown and beyond knowing. To be sure, the DOJ Panel is
composed of an Assistant Chief State Prosecutor and Senior State
Prosecutors. Their long experience in criminal investigation is a factor to
consider in determining whether they can easily be blinded by the klieg
lights of publicity. Indeed, their 26-page Resolution carries no indubitable
indicia of bias for it does not appear that they considered any extra-record
evidence except evidence properly adduced by the parties. The length of
time the investigation was conducted despite its summary nature and the
generosity with which they accommodated the discovery motions of
petitioners speak well of their fairness. At no instance, we note, did
petitioners seek the disqualification of any member of the DOJ Panel on the
ground of bias resulting from their bombardment of prejudicial publicity.

It all remains to state that the Vizconde case will move to a more critical
stage as petitioners will now have to undergo trial on the merits. We stress
that probable cause is not synonymous with guilt and while the light of
publicity may be a good disinfectant of unfairness, too much of its heat can
bring to flame an accused's right to fair trial. Without imposing on the trial
judge the difficult task of supervising every specie of speech relating to the
case at bar, it behooves her to be reminded of the duty of a trial judge in
high profile criminal cases to control publicity prejudicial to the fair
administration of justice.55 The Court reminds judges that our ability to
dispense impartial justice is an issue in every trial and in every criminal
prosecution, the judiciary always stands as a silent accused. More than
convicting the guilty and acquitting the innocent, the business of the
judiciary is to assure fulfillment of the promise that justice shall be done and
is done — and that is the only way for the judiciary to get an acquittal from
the bar of public opinion.

IN VIEW WHEREOF, the petitions are dismissed for lack of showing of


grave abuse of discretion on the part of the respondents. Costs against
petitioners.

SO ORDERED.

Regalado, J., concurs.

Mendoza, J., concurs in the result.

Narvasa, C.J., is on leave.

Separate Opinion

FRANCISCO, J., concurring:
The thrust of petitioners' arguments involve the validity and exercise of the
prosecutory powers of the State. Maintaining their innocence, petitioners
assert that the filing of an information and the issuance of warrants of arrest
against them were without probable cause. Petitioners, in my considered
view, failed to make a case to warrant the Court's interference.

Preliminary investigation, unlike trial, is summary in nature, the purpose of


which is merely to determine whether a crime has been committed and
whether there is probable cause to believe that the accused is guilty thereof
(Paderanga v. Drilon, 196 SCRA 86, 92 [1991]). It is not intended to find
guilt beyond reasonable doubt. Courts should give deference, in the absence
of a clear showing of arbitrariness, as in this case, to the finding and
determination of probable cause by prosecutors in preliminary
investigations. If not, the functions of the courts will be unduly hampered by
innumerable petitions compelling the review of the exercise of discretion on
the part of fiscals or prosecuting attorneys if each time they decide to file an
information in court their finding can be immediately brushed aside at the
instance of those charged (Ocampo IV v. Ombudsman, 225 SCRA 725, 730
[1993]). The Court, therefore, must look askance at unmeritorious moves
that could give a dent in the efficient and effective administration of justice.

Petitioners characterize the evidence against them to be inherently weak and


uncorroborated vis-a-vis their defenses. The weight or sufficiency of
evidence, to my mind, is best assayed in the trial proper. In the search for
truth, a trial has distinct merits over a preliminary investigation. We have
had occasion to stress that trial is to be preferred to ferret out the truth
(Abugotal v. Tiro, 66 SCRA 196, 201 [1975]). The validity and merits of a
party's defense or accusation as well as the admissibility or inadmissibility
of testimonies and evidence are better ventilated during the trial stage than
in the preliminary investigation level. The ineluctable media attention
notwithstanding, truth as to their innocence or guilt is still best determined at
the trial.

With respect to petitioners' contention that public respondent judge failed to


personally examine and determine the existence of probable cause for the
issuance of a warrant, suffice it to say that the judge does not have to
personally examine the complainant and his witnesses in order to issue a
warrant of arrest as he can rely on the certification of the prosecutor/s
(Circular No. 12 — Guidelines on Issuance of Warrants of Arrests [June 30,
1987]; Soliven v. Makasiar, 167 SCRA 393, 398 [1988]). There is ample
evidence and sufficient basis on record that support the trial court's issuance
of the warrant as petitioners themselves do not contend that the prosecutors'
certification was unaccompanied by the records of the preliminary
investigation to take their case outside the ambit of the rule. Moreover,
contrary to what the petitioners imply, the Court may not determine how
cursory or exhaustive the judge's examination of the certification, report and
findings of the preliminary investigation and its annexes should be as this
depends not only upon the sound exercise of the judge's discretion in
personally determining the existence of probable cause, but also from the
circumstances of each case (Lim, Sr. v. Felix, 194 SCRA 292, 306 [1991]).
Besides, respondent judge, being a public officer, enjoys the presumption of
regularity in the performance of his duties (Rule 131, Sec. 3 [m], Rules of
Court). The issuance of the warrants of arrest against petitioners thus can not
be said to be whimsical or arbitrary.

Lastly, the law in this jurisdiction is lopsided in favor of the accused. The
1987 Constitution and the Rules of Court enumerate an array of rights upon
which an accused can seek protection and solace. To mention a few: he has
the right to be presumed innocent until the contrary is proved, the right
against self-incrimination, the right to remain silent, to confront and cross-
examine the witnesses against him, to have a speedy, impartial and public
trial, to be heard by himself and counsel, to have competent and independent
counsel preferably of his own choice. These rights are afforded to the
accused and not to the complainant. Therefore, petitioners need not be
distressed if they henceforth go to trial.

I vote to dismiss the petitions.

Mendoza, J., concurs.

Allado v. Diokno, G.R. No. 113630, May 5, 1994

On balance at the fulcrum once again are the intrinsic right of the State to
prosecute perceived transgressors of the law, which can be regulated, and the
innate value of human liberty, which can hardly be weighed.

Some twelve years ago we were confronted with a similar problem when former
Senator Jovito R. Salonga invoked before this Court his "right to life and liberty
guaranteed by the due process clause, alleging that no prima facie case has been
established to warrant the filing of an information for subversion against him." 1 We
resolved the issue then and sustained him. He is now back before us, this time as
counsel pleading the cause of petitioners herein who, he claims, are in a situation
far worse than his predicament twelve (12) years ago. He postulates that no
probable cause likewise exists in this case, and what is worse is that no bail is
recommended.
This petition gives us an opportunity to revisit the concept and implication of
probable cause, the existence of which is necessary for the prosecutor to have an
accused held for trial and for a trial judge to issue a warrant for his arrest. It is
mandatory therefore that there be probable cause before an information is filed and
a warrant of arrest issued. Unfortunately, however, at times a criminal case is filed,
a warrant of arrest issued and a person consequently incarcerated on
unsubstantiated allegations that only feign probable cause.

Petitioners Diosdado Jose Allado and Roberto L. Mendoza, alumni of the College
of Law, University of the Philippines, are partners of the Law Firm of Salonga,
Hernandez and Allado. In the practice of their profession, and on the basis of an
alleged extrajudicial confession of a security guard, they have been accused of the
heinous crime of kidnapping with murder by the Presidential Anti-Crime
Commission (PACC) and ordered arrested without bail by respondent judge.

The focal source of the information against petitioners is the sworn statement dated
16 September 1993 of Security Guard Escolastico Umbal, a discharge of the
Philippine Constabulary, implicating them as the brains behind the alleged
kidnapping and slaying of one Eugen Alexander Van Twest, a German
national. 2 In that extrajudicial confession, Umbal claimed that he and his
companions were met by petitioners at Silahis Hotel and in exchange for P2.5M
the former undertook to apprehend Van Twest who allegedly had an international
warrant of arrest against him. Thus, on 16 June 1992, after placing him under
surveillance for nearly a month, Umbal, Ex-policeman Rolando Gamatero,
AFPCIG Agent Roberto Santiago and SPO2 Sergio Antonino abducted Van Twest.
They blocked his blue Nissan Pathfinder under the Alabang overpass and forced
him into their car. They brought him to a "safe house" just behind the New Bilibid
Prisons. Umbal was tasked to watch over their quarry. After four (4) days,
Gamatero, Santiago and Antonino returned to the "safe house" together with
petitioners and SPO2 Roger Bato, known to Umbal also as "Batok." SPO2 Bato
faked the interrogation of Van Twest, pretending it was official, and then made
him sign certain documents. The following day, Gamatero shot Van Twest in the
chest with a baby armalite, after which Antonino stabbed him repeatedly, cut off
his private part, and later burned his cadaver into fine ashes using gasoline and
rubber tires. Umbal could not recall the exact date when the incident happened, but
he was certain it was about a year ago.

A day after Umbal executed his extrajudicial confession, the operatives of the
PACC, armed with a search warrant issued by Judge Roberto A. Barrios of the
Regional Trial Court of Manila, Br. 11, 3 separately raided the two (2) dwellings of
Santiago, one located at No. 7 Sangley Street, and the other, along Amalingan
Street, both in Green Heights Subdivision, Parañaque. The raiders recovered a blue
Nissan Pathfinder and assorted firearms and ammunition and placed Santiago and
his trusted aide, Efren Madolid, under arrest. Also arrested later that day were
Antonio and Bato who were found to have in their possession several firearms and
ammunition and Van Twest's Cartier sunglasses.

After evaluating the pieces of evidence gathered by PACC operatives, Sr., Supt.
Panfilo Lacson, Chief of PACC Task Force Habagat, referred the case to the
Department of Justice for the institution of criminal proceedings against AFPCIG
Agent Roberto Santiago, SPO1 Sergio Antonino, SPO2 Roger Bato, Ex-policeman
Rolando Gamatero, Efren Madolid, and petitioners herein, Atty. Diosdado Jose
Allado and Atty. Roberto L. Mendoza, for illegal possession of firearms and
ammunition, carnapping, kidnapping for ransom with murder, and usurpation of
authority. 4 In his letter to the State Prosecutor dated 17 September 1993, Sr. Supt.
Lacson charged that —

Atty. Roberto L. Mendoza and Atty. Allado of Salonga, Hernandez


and Allado Law Offices . . . planned and conspired with other
suspects to abduct and kill the German national Alexander Van Twest
in order to eliminate him after forcing the victim to sign several
documents transferring ownership of several properties amounting to
several million pesos and caused the withdrawal of P5M deposit from
the victim's bank account.

Thereafter, Senior State Prosecutor Ferdinand prosecutor Ferdinand R. Abesamis


issued a subpoena to petitioners informing them that a complaint
was filed against them by PACC TF-Habagat, directing them to appear on
30 September 1993 at the Multi-Purpose Hall of the Department of Justice and to
submit their counter-affidavits. Attached to the subpoena were copies of the
affidavits executed by Umbal and members of the team who raided the two (2)
dwellings of Santiago. 5

Not satisfied merely with the affidavits attached to the subpoena, petitioner
Mendoza moved for the production of other documents for examination and
copying to enable him to fully prepare for his defense and to submit an intelligible
counter-affidavit. 6 Specifically, petitioner Mendoza was interested in (a) the
"several documents transferring ownership of several properties amounting to
several million pesos and the withdrawal of P5M deposits from the victim's bank
account," as stated in the complaint; (b) the complete records of the PACC's
investigation, including investigations on other suspects and their disposition,
PACC's Order of Battle for 1992 and early 1993; and, (c) such other written
statements issued in the above-entitled case, and all other documents intended to be
used in this case. 7 Petitioners likewise sought the inhibition of the members of the
panel of prosecutors, which was created to conduct the preliminary investigation,
on the ground that they were members of the legal staff assigned to PACC and thus
could not act with impartiality.
In its Order of 11 October 1993,8 the new panel of prosecutors composed of Senior
State Prosecutor Bernelito R. Fernandez as Chairman, with Rogelio F. Vista and
Purita M. Deynata as Members, confirmed that the motion for inhibition of the
members of the old panel as well as the appeal to the Secretary of Justice was
resolved on 8 October 1993 resulting in the creation of a new panel. Thereafter, the
new panel granted the prayer of petitioner Mendoza for the production of
additional documents used or intended to be used against him. Meanwhile, Task
Force Habagat, in compliance with the order, submitted only copies of the request
for verification of the firearms seized from the accused, the result of the request for
verification, and a Philippine Times Journal article on the case with a marginal
note of President Fidel V. Ramos addressed to the Chief of the Philippine National
Police directing the submission of a report and summary of actions taken thereon.

Not having been provided with the requested documents, petitioners nevertheless
submitted their respective counter-affidavits denying the accusations against them. 9

After a preliminary hearing where clarificatory questions were additionally


propounded, the case was deemed submitted for resolution. But before the new
panel could resolve the case, SPO2 Bato filed a manifestation stating that he was
reconsidering the earlier waiver of his right to file counter- affidavit, 10 and "in the
greater interest of truth, justice and fair play" moved for the admissions of his
counter-affidavit 11 confessing participation in the abduction and slaying of Van
Twest and implicating petitioners Allado and Mendoza. Sometime in January
1994, however, before petitioners could refute Bato's counter-affidavit, he moved
to suppress it on the ground that it was extracted through intimidation and duress.

On 3 February 1994, with the new penal failing to act on the twin motions of SPO2
Bato, petitioners heard over the radio that the panel had issued a resolution finding
a prima facie case against them and that an information had already been filed in
court. Upon verification with the Department of Justice, however, petitioners were
informed that the resolution was not yet ready for release, but later that afternoon
they were able to secure a copy of the information for kidnapping with murder
against them 12 and the 15-page undated resolution under the letterhead of PACC,
signed by the panel of prosecutors, with the Head of the PACC Task Force
recommending approval thereof. 13 That same day, the information was filed before
the Regional Trial Court of Makati and raffled off to Branch 62 presided by
respondent Judge Roberto C. Diokno.

On 4 February 1994, respondent judge, in response to petitioners' request, gave


them until 8 February 1994 to submit their opposition to the issuance of a warrant
of arrest against all the accused. 14 On 7 February 1994, petitioners complied with
the order of respondent judge. 15 The following day,
8 February 1994, petitioner Allado filed an appeal with the Secretary of Justice
seeking review and reversal of the undated resolution of the panel
of prosecutors, 16 which appeal was adopted by petitioner Mendoza. 17 On
11 February 1994, petitioner Allado moved to defer the proceedings before the trial
court pending resolution of his appeal before the Secretary of Justice. 18 However,
on even date, respondent judge issued the assailed warrant of arrest against
petitioners. 19 Hence, on 15 February 1994, petitioners filed with us the instant
petition for certiorari and prohibition with prayer for a temporary restraining
order.

On 16 February 1994, we required respondents to comment on the petition and set


the case for hearing on 28 February 1994. After the hearing, we issued a temporary
restraining order enjoining PACC from enforcing the warrant of arrest and
respondent judge from conducting further proceedings on the case and, instead, to
elevate the records to us. Meanwhile, on 27 February 1994, petitioners voluntarily
surrendered at the Headquarters of the Capital Command (CAPCOM), Philippine
National Police (PNP), Camp Bagong Diwa, Bicutan, Metro Manila, and on 29
February 1994, they were released on the basis of our temporary restraining order.

Petitioners, in their 335-page petition, inclusive of annexes, principally contend


that respondent judge acted with grave abuse of discretion and in excess of
jurisdiction in "whimsically holding that there is probable cause against petitioners
without determining the admissibility of the evidence against petitioners and
without even stating the basis of his findings," 20 and in "relying on the Resolution
of the Panel and their certification that probable cause exists when the certification
is flawed." 21 Petitioners maintain that the records of the preliminary investigation
which respondent judge solely relied upon failed to establish probable cause
against them to justify the issuance of the warrant of arrest. Petitioners likewise
assail the prosecutors' "clear sign of bias and impartiality (sic)." 22

On the other hand, the Office of the Solicitor General argues that the determination
of probable cause is a function of the judge who is merely required to personally
appreciate certain facts to convince him that the accused probably committed the
crime charged.

Section 2, Art. III, of the 1987 Constitution, lays down the requirements for the
issuance of a warrant of arrest, i.e., a warrant of arrest shall issue only upon
probable cause to be determined personally by the judge after examination under
oath or affirmation of the complainant and the witnesses he may produce.

As early as 1915, in Buchanan v. Viuda de Esteban, 23 this Court speaking through


Associate Justice Sherman Moreland defined probable cause as "the existence of
such facts and circumstances as would excite the belief, in a reasonable mind,
acting on the facts within the knowledge of the prosecutor, that the person charged
was guilty of the crime for which he was prosecuted." This definition is still
relevant today as we continue to cite it in recent cases. 24 Hence, probable cause for
an arrest or for the issuance of a warrant of arrest has been defined as such facts
and circumstances which would lead a reasonable discreet and prudent man to
believe that an offense has been committed by the person sought to be
arrested. 25 And as a protection against false prosecution and arrest, it is the
knowledge of facts, actual or apparent, strong enough to justify a reasonable man
in the belief that he was lawful grounds for arresting the accused. 26

Pilapil v. Sandiganbayan 27 sets a standard for determining the existence of


probable cause. While it appears in that case that we have granted the prosecutor
and the trial judge seemingly unlimited latitude in determining the existence of
absence of probable cause by affirming the long-standing procedure that they can
base their findings merely on their personal opinion and reasonable belief, yet, this
permissiveness should not be interpreted as giving them arbitrary powers and
letting them loose in the determination of the existence of probable cause, a
delicate legal question which can result in the harassment and deprivation of liberty
of the person sought to be charged or arrested. There we said —

Probable cause is a reasonable ground of presumption that a matter is,


or may be, well founded, such a state of facts in the mind of the
prosecutor as would lead a person of ordinary caution and prudence
to believe, or entertain an honest or strong suspicion, that a thing is
so. The term does not mean "actual and positive cause" nor does it
import absolute certainty. It is merely based on opinion and
reasonable belief. Thus, a finding of probable cause does not require
an inquiry into whether there is sufficient evidence to procure a
conviction. It is enough that it is it believed that the act or omission
complained of constitutes the offense charged. Precisely, there is a
trial for the reception of evidence of the prosecution in support of the
charge.

Whether an act was done causing undue injury to the government and
whether the same was done with manifest partiality or evident bad
faith can only be made out by proper and sufficient testimony.
Necessarily, a conclusion can be arrived at when the case has already
proceeded on sufficient proof. 28

Accordingly, before issuing a warrant of arrest, the judge must satisfy himself that
based on the evidence submitted there is sufficient proof that a crime has been
committed and that the person to be arrested is probably guilty thereof. In the
Order of respondent judge dated 11 February 1994, it is expressly stated that "[t]his
court after careful evaluation of the evidence on record, believes and rules that
probable cause exists; and therefore, a warrant of arrest should be issued."
However, we are unable to see how respondent judge arrived at such ruling. We
have painstakingly examined the records and we cannot find any support for his
conclusion. On the contrary, we discern a number of reasons why we consider the
evidence submitted to be insufficient for a finding of probable cause against
petitioners.
The Presidential Anti-Crime Commission relies heavily on the sworn statement of
Security Guard Umbal who supposedly confessed his participation in the alleged
kidnapping and murder of Van Twest. For one, there is serious doubt on Van
Twest's reported death since the corpus delicti has not been established, nor have
his remains been recovered. Umbal claims that Van Twest was completely burned
into ashes with the use of gasoline and rubber tires from around ten o'clock in the
evening to six o'clock the next morning. 29 This is highly improbable, if not
ridiculous. A human body cannot be pulverized into ashes by simply burning it
with the use of gasoline and rubber tires in an open field. Even crematoria use
entirely closed incinerators where the corpse is subjected to intense
heat. 30 Thereafter, the remains undergo a process where the bones are completely
ground to dust.

In the case of Van Twest, there is not even any insinuation that earnest efforts were
exerted to recover traces of his remains from the scene of the alleged
cremation. 31 Could it be that the government investigators did to the place of
cremation but could not find any? Or could it be that they did not go at all because
they knew that there would not be any as no burning ever took place? To allege
then that the body of Van Twest was completely burned to ashes in an open field
with the use merely of tires and gasoline is a tale too tall to gulp.

Strangely, if not awkwardly, after Van Twest's reported abduction on


16 June 1992 which culminated in his decimation by cremation, his counsel
continued to represent him before judicial and quasi-judicial proceedings. Thus on
31 July 1992, his counsel filed in his behalf a petition for review before this Court,
docketed as G.R. Nos. 106253, and on 18 March 1993, a memorandum before the
Securities and Exchange Commission in SEC Case No. 3896. On
26 November 1993, during the preliminary investigation conducted by the panel of
prosecutors, counsel again manifested that "even then and even as of this time, I
stated in my counter-affidavit that until the matter of death is to be established in
the proper proceedings, I shall continue to pursue my duties and responsibilities as
counsel for Mr. Van Twest." 32 Hence, even Asst. Solicitor General Estoesta
believes that counsel of Van Twest doubted the latter's
death. 33 Obviously, counsel himself does not believe that his client is in fact
already dead otherwise his obligation to his client would have ceased except to
comply with his duty "to inform the court promptly of such death . . . and to give
the name and residence of his executor, administrator, guardian or other legal
representative," 34 which he did not.

Under the circumstances, we cannot discount petitioners' theory that the supposed
death of Van Twest who is reportedly an international fugitive from justice, a fact
substantiated by petitioners and never refuted by PACC, is a likely story to stop the
international manhunt for his arrest. In this regard, we are reminded of the leading
case of U.S. v. Samarin  35 decided ninety-two years ago where this Court ruled that
when the supposed victim is wholly unknown, his body not found, and there is but
one witness who testifies to the killing, the corpus delicti is not sufficiently
proved.

Then, the extrajudicial statement of Umbal suffers from material inconsistencies.


In his sworn statement, he said that he together with his cohorts was met by
petitioners in Silahis Hotel where they hatched the plan to abduct Van
Twest. 36 However, during the preliminary investigation, he stated that he was not
part of the actual meeting as he only waited outside in the car for his companions
who supposedly discussed the plan inside Silahis Hotel. 37

Umbal also said that petitioners arrived with Bato and conducted a mock
interrogation of Van Twest who thereafter signed various documents upon being
compelled to do so. 38 During the clarificatory questioning, however, Umbal
changed his story and said that he was asked to go outside of the "safe house" at
the time Van Twest was interrogated and thus did not see if Van Twest indeed
signed certain documents. Why Umbal had to be sent out of the "safe house,"
no explanation was offered. Did these documents really exist? Or could the
non-existence of these documents be the reason why PACC was not able to comply
with the order of the prosecutors to produce them during the preliminary
investigation? And then, what happened to the P2.5M that was supposedly offered
by petitioners in exchange for the abduction of Van Twest? These and more remain
unanswered.

Most perplexing however is that while the whole investigation was supposedly
triggered off by Umbal's confession of 16 September 1993, the application of the
PACC operatives for a search warrant to be served in the
two (2) dwellings of Santiago was filed and granted by the Regional Trial Court of
Manila on 15 September 1993, a day before Umbal executed his sworn statement.
In support of the application, the PACC agents claimed that Umbal had been in
their custody since 10 September 1993. Significantly, although he was said to be
already under their custody, Umbal claims he was never interrogated until 16
September 1993 and only at the security barracks of Valle Verde V, Pasig, where
he was a security guard. 39

The alleged counter-affidavit of SPO2 Bato, which the panel of prosecutors also
considered in filing the charges against petitioners, can hardly be credited as its
probative value has tremendously waned. The records show that the alleged
counter-affidavit, which is self-incriminating, was filed after the panel had
considered the case submitted for resolution. And before petitioners could refute
this counter-affidavit, Bato moved to suppress the same on the ground that it was
extracted through duress and intimidation.

For sure, the credibility of Umbal is badly battered. Certainly, his bare allegations,
even if the State invokes its inherent right to prosecute, are insufficient to justify
sending two lawyers to jail, or anybody for that matter. More importantly, the
PACC operatives who applied for a warrant to search the dwellings of Santiago
never implicated petitioners. In fact they claimed that according to Umbal, it was
Santiago, and not petitioners, who masterminded the whole affair. 40 While there
may be bits of evidence against petitioners'
co-accused, i.e., referring to those seized from the dwellings of Santiago, these do
not in the least prove petitioners' complicity in the crime charged. Based on the
evidence thus far submitted there is nothing indeed, much less is there probable
cause, to incriminate petitioners. For them to stand trial and be deprived in the
meantime of their liberty, however brief, the law appropriately exacts much more
to sustain a warrant for their arrest — facts and circumstances strong enough in
themselves to support the belief that they are guilty of a crime that in fact
happened. Quite obviously, this has not been met.

Verily, respondent judge committed grave abuse of discretion in issuing the


warrant for the arrest of petitioners it appearing that he did not personally examine
the evidence nor did he call for the complainant and his witnesses in the face of
their incredible accounts. Instead, he merely relied on the certification of the
prosecutors that probable cause existed. For, otherwise, he would have found out
that the evidence thus far presented was utterly insufficient to warrant the arrest of
petitioners. In this regard, we restate the procedure we outlined in various cases we
have already decided.

In Soliven v. Makasiar, 41 we said that the judge (a) shall personally evaluate the
report and the supporting documents submitted by the fiscal regarding the
existence of probable cause and, on the basis thereof, issue a warrant of arrest; or,
(b) if on the basis thereof he finds no probable cause, may disregard the fiscal's
report and require the submission of supporting affidavits of witnesses to aid him
in arriving at a conclusion on the existence of probable cause.

In People v. Inting, 42 we emphasized the important features of the constitutional


mandate: (a) The determination of probable cause is a function of the judge; it is
not for the provincial fiscal or prosecutor to ascertain. Only the judge and the judge
alone makes this determination; (b) The preliminary inquiry made by a prosecutor
does not bind the judge. It merely assists him in making the determination of
probable cause. The judge does not have to follow what the prosecutor presents to
him. By itself, the prosecutor's certification of probable cause is ineffectual. It is
the report, the affidavits, the transcript of stenographic notes (if any), and all other
supporting documents behind the prosecutor's certification which are material in
assisting the judge in his determination of probable cause; and, (c) Judges and
prosecutors alike should distinguish the preliminary inquiry which determines
probable cause for the issuance of a warrant of arrest from the preliminary
investigation proper which ascertains whether the offender should be held for trial
or released. Even if the two inquiries be conducted in the course of one and the
same proceeding, there should be no confusion about their objectives. The
determination of probable cause for the warrant is made by the judge. The
preliminary investigation
proper — whether or not there is reasonable ground to believe that the accused is
guilty of the offense charged and therefore, whether or not he should be subjected
to the expense, rigors and embarrassment of trial — is a function of the prosecutor.

In Lim v. Felix, 43 where we reiterated Soliven v. Makasiar and People v. Inting,


we said —

[T]he Judge does not have to personally examine the complainant and
his witnesses. The Prosecutor can perform the same functions as a
commissioner for the taking of the evidence. However, there should
be a report and necessary documents supporting the Fiscal's bare
certification. All these should be before the Judge.

The extent of the Judge's personal examination of the report and its
annexes depends on the circumstances of each case. We cannot
determine beforehand how cursory or exhaustive the Judge's
examination should be. The Judge has to exercise sound discretion
for, after all, the personal determination is vested in the Judge by the
Constitution. It can be as brief or as detailed as the circumstances of
each case require. To be sure, the judge must go beyond the
Prosecutor's certification and investigation report whenever
necessary. He should call for the complainant and witnesses
themselves to answer the court's probing questions when the
circumstances of the case so require.

Clearly, probable cause may not be established simply by showing that a trial
judge subjectively believes that he has good grounds for his action. Good faith is
not enough. If subjective good faith alone were the test, the constitutional
protection would be demeaned and the people would be "secure in their persons,
houses, papers and effects" only in the fallible discretion of the judge. 44 On the
contrary, the probable cause test is an objective one, for in order that there be
probable cause the facts and circumstances must be such as would warrant a belief
by a reasonably discreet and prudent man that the accused is guilty of the crime
which has just been committed. 45 This, as we said, is the standard. Hence, if upon
the filing of the information in court the trial judge, after reviewing the information
and the documents attached thereto, finds that no probable cause exists must either
call for the complainant and the witnesses themselves or simply dismiss the case.
There is no reason to hold the accused for trial and further expose him to an open
and public accusation of the crime when no probable cause exists.

But then, it appears in the instant case that the prosecutors have similarly
misappropriated, if not abused, their discretion. If they really believed that
petitioners were probably guilty, they should have armed themselves with facts and
circumstances in support of that belief; for mere belief is not enough. They should
have presented sufficient and credible evidence to demonstrate the existence of
probable cause. For the prosecuting officer "is the representative not of an ordinary
party to a controversy, but of a sovereignty whose obligation to govern impartially
is as compelling as its obligation to govern all; and whose interest, therefore, in a
criminal prosecution is not that it shall win a case, but that justice shall be done. As
such, he is in a peculiar and very definite sense the servant of the law, the twofold
aim of which is that guilt shall not escape or innocence suffer. He may prosecute
with earnestness and vigor — indeed, he should do so. But, while he may strike
hard blows, he is not at liberty to strike foul ones. It is as much his duty to refrain
from improper methods calculated to produce a wrongful conviction as it is to use
every legitimate means to bring about a just one" 46

In the case at bench, the undue haste in the filing of the information and the
inordinate interest of the government cannot be ignored. From the gathering of
evidence until the termination of the preliminary investigation, it appears that the
state prosecutors were overly eager to file the case and secure a warrant for the
arrest of the accused without bail and their consequent detention. Umbal's sworn
statement is laden with inconsistencies and improbabilities. Bato's counter-affidavit
was considered without giving petitioners the opportunity to refute the same. The
PACC which gathered the evidence appears to have had a hand in the
determination of probable cause in the preliminary inquiry as the undated
resolution of the panel not only bears the letterhead of PACC but was also
recommended for approval by the head of the PACC Task Force. Then petitioners
were given the runaround in securing a copy of the resolution and the information
against them.

Indeed, the task of ridding society of criminals and misfits and sending them to jail
in the hope that they will in the future reform and be productive members of the
community rests both on the judiciousness of judges and the prudence of
prosecutors. And, whether it is a preliminary investigation by the prosecutor,
which ascertains if the respondent should be held for trial, or a preliminary inquiry
by the trial judge which determines if an arrest warrant should issue, the
bottomline is that there is a standard in the determination of the existence of
probable cause, i.e., there should be facts and circumstances sufficiently strong in
themselves to warrant a prudent and cautious man to believe that the accused is
guilty of the crime with which he is charged. Judges and prosecutors are not off on
a frolic of their own, but rather engaged in a delicate legal duty defined by law and
jurisprudence.

In this instance, Salonga v. Paño  47 finds application —

The purpose of a preliminary investigation is to secure the innocent


against hasty, malicious and oppressive prosecution, and to protect
him from an open and public accusation of crime, from the trouble,
expense and anxiety of a public trial, and also to protect the state
from useless and expensive trial (Trocio v. Manta, 118 SCRA
241, citing Hashim v. Boncan, 71 Phil. 216). The right to a
preliminary investigation is a statutory grant, and to withhold it would
be to transgress constitutional due process (People v. Oandasa, 25
SCRA 277). However, in order to satisfy the due process clause it is
not enough that the preliminary investigation is conducted in the
sense of making sure that the transgressor shall not escape with
impunity. A preliminary investigation serves not only for the purposes
of the State. More importantly, it is a part of the guarantees of
freedom and fair play which are birthrights of all who live in the
country. It is therefore imperative upon the fiscal or the judge as the
case may be, to relieve the accused from the pain of going thru a trial
once it is ascertained that the evidence is insufficient to sustain a
prima facie case or that no probable cause exists to form a sufficient
belief as to the guilt of the accused (emphasis supplied).

The facts of this case are fatefully distressing as they showcase the seeming
immensity of government power which when unchecked becomes tyrannical and
oppressive. Hence the Constitution, particularly the Bill of Rights, defines the
limits beyond which lie unsanctioned state actions. But on occasion, for one reason
or another, the State transcends this parameter. In consequence, individual liberty
unnecessarily suffers. The case before us, if uncurbed, can be illustrative of a
dismal trend. Needless injury of the sort inflicted by government agents is not
reflective of responsible government. Judges and law enforcers are not, by reason
of their high and prestigious office, relieved of the common obligation to avoid
deliberately inflicting unnecessary injury.

The sovereign power has the inherent right to protect itself and its people from
vicious acts which endanger the proper administration of justice; hence, the State
has every right to prosecute and punish violators of the law. This is essential for its
self- preservation, nay, its very existence. But this does not confer a license for
pointless assaults on its citizens. The right of the State to prosecute is not a carte
blanche for government agents to defy and disregard the rights of its citizens under
the Constitution. Confinement, regardless of duration, is too high a price to pay for
reckless and impulsive prosecution. Hence, even if we apply in this case the
"multifactor balancing test" which requires the officer to weigh the manner and
intensity of the interference on the right of the people, the gravity of the crime
committed and the circumstances attending the incident, still we cannot see
probable cause to order the detention of petitioners. 48

The purpose of the Bill of Rights is to protect the people against arbitrary and
discriminatory use of political power. This bundle of rights guarantees the
preservation of our natural rights which include personal liberty and security
against invasion by the government or any of its branches or instrumentalities.
Certainly, in the hierarchy of rights, the Bill of Rights takes precedence over the
right of the State to prosecute, and when weighed against each other, the scales of
justice tilt towards the former. Thus, relief may be availed of to stop the purported
enforcement of criminal law where it is necessary to provide for an orderly
administration of justice, to prevent the use of the strong arm of the law in an
oppressive and vindictive manner, and to afford adequate protection to
constitutional rights. 49

Perhaps, this case would not have reached this Court if petitioners were ordinary
people submissive to the dictates of government. They would have been illegally
arrested and detained without bail. Then we would not have the opportunity to
rectify the injustice. Fortunately, the victims of injustice are lawyers who are
vigilant of their rights, who fight for their liberty and freedom not otherwise
available to those who cower in fear and subjection.

Let this then be a constant reminder to judges, prosecutors and other government
agents tasked with the enforcement of the law that in the performance of their
duties they must act with circumspection, lest their thoughtless ways, methods and
practices cause a disservice to their office and maim their countrymen they are
sworn to serve and protect. We thus caution government agents, particularly the
law enforcers, to be more prudent in the prosecution of cases and not to be
oblivious of human rights protected by the fundamental law. While we greatly
applaud their determined efforts to weed society of felons, let not their impetuous
eagerness violate constitutional precepts which circumscribe the structure of a
civilized community.

WHEREFORE, the petition for certiorari and prohibition is GRANTED. The


temporary restraining order we issued on 28 February 1994 in favor of petitioners,
Atty. Diosdado Jose Allado and Atty. Roberto L. Mendoza, is made permanent.
The warrant of arrest issued against them is SET ASIDE and respondent Judge
Roberto C. Diokno is ENJOINED from proceeding any further against herein
petitioners in Crim. Case No. 94-1757 of the Regional Trial Court of Makati.

SO ORDERED

CASES:
Ponsica v. Ignalaga, G.R. No. 72301, July 31, 1987
The chief issue raised by the petitioners in this case is whether or not Section 143
of the Local Government Code 1 granting power to the municipal mayor to
conduct preliminary investigations and order the arrest of the accused, was
repealed by the 1985 Rules on Criminal Procedure promulgated by this Court; and
is, in addition, unconstitutional as vesting the power to conduct preliminary
investigations in an official who cannot be deemed a "neutral and detached
magistrate" within the contemplation of Section 3, Article IV of the 1973
Constitution. The issue is hereby resolved adversely to the petitioners, with the
stressed qualification that the mayor’s power to order arrest ceased to exist as of
February 2, 1987 when the new Constitution was ratified by the Filipino people,
and that, in any event, the investigation actually conducted by respondent mayor in
the case at bar was fatally defective.

Shortly after noon on September 20, 1985, an attempt was made by firemen and
soldiers to disperse a crowd of demonstrators massed in front of the Municipal
Building of Escalante, Negros Occidental, with the use first, of water spewed from
fire hoses, and later, tear gas. Eventually there was gunfire. Within moments,
rallyists lay dead on and by the National Road. The fatalities numbered fifteen
(15), according to the military officers; twenty-nine (29), according to the
demonstrators.chanrobles.com.ph : virtual law library

In the afternoon of that day, Escalante Town Mayor Braulio Lumayno, in view of
the absence of the Municipal Circuit Court Judge (Emilio Ignalaga), took
cognizance of a complaint filed by the Military Station Commander charging some
of the rallyists with the felony of inciting to sedition, and after avowedly
conducting an investigation of the witnesses presented by the complainant, issued
an order for the arrest of certain of the demonstrators. His order reads as
follows:jgc:chanrobles.com.ph

"It appearing that the Presiding Municipal Circuit Court Judge, Hon. Emilio M.
Ignalaga, is on official leave of absence and, in the interest of justice, the
undersigned has to urgently act on the complaint filed by the Station Commander,
against the above-named accused for `INCITING TO SEDITION’ and, on the
basis of the evidence submitted after a searching question and answer were
conducted and, being satisfied that said crime has been committed, in order not to
frustrate the ends of justice, it is necessary that the above-named accused be placed
under custody. Let therefore, a warrant of arrest be issued for said above-named
accused.

"Bail recommended: P12,000.00." 2

In the record of the Court a quo appear the following inter alia:chanrob1es virtual
1aw library

1) Complaint for "Inciting to Sedition" (RPC 142, as and by PD 183 and PD 1974),
signed by a Capt. Jugan, and sworn to before Mayor Lumayno on Sept. 20, 1985,
bearing the stamped notation of filing with the MTC: "9/24/85, 4:00 PM:" 3

2) 2-page Affidavit of Capt. Sanson, dated, and sworn to before Actg. City Fiscal
Abros (Cadiz City) on Sept. 20, 1985; 4

3) 3-page document, "Searching Questions and Answers" signed by Capt. Sanson,


dated, and sworn to before Mayor Lumayno on, September 20, 1985; 5

4) 3-page sworn statement of Godofredo Hoyo-A y Jayme, General Manager of the


Balintawak-Escalante Water District; 6

5) Affidavit of Leopoldo Villalon; 7

6) Affidavit of Elpidio Carbajosa; 8 and

7) Affidavit of Eduardo Flores. 9

The gist of the testimony of Capt. Sanson and the other affiants is that on
September 20, 1985, the demonstrators, numbering "about 1,000," had blockaded
the main highway in front of the Escalante Municipal Building, by massing
themselves on the road as well as by piling stones, coconut trunks and pieces of
wood in the middle of the highway. They were also "shouting invectives, seditious
and scurrilous words against the government." Negotiations with Ponsica,
Chairman of the Escalante Chapter of "BAYAN" (Bagong Alyansang Makabayan),
to have the road cleared having been unavailing, firemen on firetrucks began to
train a "torrent of water" from their fire hoses on the demonstrators. The rallyists
retaliated by hurling stones at the firemen. One of them "approached the security of
the firetruck and stabbed him." Others climbed aboard the trucks and tried to grab
the firehoses and firearms of the officers. At this point, on Capt. Sanson’s orders,
his "back-up teams" of soldiers commenced to throw tear gas at the crowd. One of
the demonstrators picked up a tear gas canister and hurled it back at the soldiers. At
the same time gunfire from "different assorted firearms" emanated from the
rallyists; and some of the shots hit the blinker of a firetruck and the headlight of
another. The soldiers shot back. This exchange of gunfire resulted in "fourteen (14)
demonstrators killed on the spot." Recovered at the scene were a rifle; a U.S. 45
cal. pistol; 2 "homemade" pistols; 14 steel arrows and 4 assorted slings; 30 assorted
knives; a "pogakhang" with 2 live cartridges; 2 grenades; and several empty shells
of different caliber. Hours later, another corpse, identified as that of a
demonstrator, too, was brought to the PC Headquarters.chanroblesvirtualawlibrary

The petitioners however give a different version of the facts. What happened,
according to them, was that at 9 o’clock in the morning on that day, a group of
demonstrators, "composed mostly of laid-off sugar field workers," gathered in
front of Escalante Municipal Hall "in the exercise of their constitutionally
guaranteed right to freedom of expression and to assemble peacefully to petition
the government for redress of grievances." 10 About an hour afterwards, fire trucks
arrived one after another, as well as jeeploads of soldiers and CHDF members, in
full combat gear, Shortly after noon, after "going thru the motions of negotiating
with the demonstrators," the military officers ordered the crowd to disperse; but
without warning, fire hoses were trained on and sprayed water at the
demonstrators. When the rallyists did not budge, tear gas canisters were thrown at
them. A demonstrator picked up a canister and threw it at an "empty space in the
plaza" The soldier and CHDF members thereupon fired indiscriminately at the
crowd, killing 29 and injuring at least 30 persons. 11

After the Mayor had referred the case to Municipal Trial Court Judge Ignalaga on
September 24, 1985, 12 an "URGENT MOTION TO QUASH WARRANT OF
ARREST" was filed on September 26, 1985 by petitioners’ counsel on the ground
that a mayor no longer has authority to conduct preliminary investigations or issue
warrants of arrest that authority having been "withheld in the 1985 New Rules on
Criminal Procedure." 13 This was opposed by the Station Commander 14 who
invoked Section 143 of the Local Government Code providing that" (i)n case of
temporary absence of the Judge assigned to the municipality, the mayor may
conduct the preliminary examination in criminal cases when, in his opinion the
investigation cannot be delayed without prejudice to the interest of justice." A
reply was filed by the petitioners after their receipt of the opposition "only last
October 8, 1985." 15 They contended that the "power of the municipal mayor to
conduct preliminary investigation and issue a warrant of arrest under the 1964
Revised Rules of Court . . . (had been) impliedly repealed by the 1985 New Rules
on Criminal Procedure;" that "the 1985 New Rules on Criminal Procedure being a
special law, controls over provisions of the Local Government Code (BP
337,1983), which is a general law;" and in any case, "subject warrants of arrest
should be reviewed and revoked as done without observance of legal
requisites."cralaw virtua1aw library

By Resolution dated Oct. 11, 1985, 16 the Judge confirmed the mayor’s arrest
order. He opined that in the absence of the judge, the mayor still has authority to
conduct preliminary investigations and issue arrest warrants, since Rule 112, Sec. 2
(d), of the 1985 Rules, includes as among those authorized to conduct preliminary
investigations, "Such other officers as may be authorized by law;" and the Local
Government Code of 1983, Section 143, grants a town mayor authority to conduct
preliminary examinations in case of the temporary absence of the judge when such
investigation cannot be delayed without prejudice to the interest of justice. The
Judge declared that in the case at bar, the mayor had conducted the examination
personally, and having in the exercise of his discretion found probable cause,
issued the warrants of arrest in question; and conceding arguendo irregularity in
that the preliminary examination was conducted without according the parties the
assistance of lawyers (contrary to par. 2, Sec. 143, Chap. 3, Title 2, Book II, Local
Government Code), this does not render the proceedings void because at any rate,
the mayor had duly observed the uniform procedure under PD 91 (citing: People v.
Paran, 52 Phil. 712; Hashim v. Boncan, 71 Phil. 216; Lino v. Fugoso, 77 Phil.
933).chanrobles law library

It is Mayor Lumayno’s order for the petitioners’ arrest of September 20, 1985, and
Jude Ignalaga’s Order of October 11, 1985 "validating that order of arrest" that the
petitioners would have this Court nullify and perpetually enjoin. 17

In their petition for certiorari and prohibition filed on October 15, 1985, and their
Memorandum of January 21, 1986, 18 they assert that:chanrob1es virtual 1aw
library

1) while Section 3, Rule 112 of the 1964 Rules of Court, empowers the municipal
mayor, "in case of temporary absence of both the municipal and the auxiliary
municipal judges from the municipality, town or place wherein they exercise their
jurisdiction, to make the preliminary examination in criminal cases when such
examination cannot be delayed without prejudice to the interest of justice," that
power has been removed from him by the 1985 New Rules on Criminal Procedure
which "do not mention the Municipal Mayor as among the officers authorized to
conduct preliminary investigation (Section 2, Rule 112), much less to issue an
order of arrest (Section 6, Ibid.)" 19

2) Section 143 of BP Blg. 337 (Local Government Code) — under which the order
of arrest in question is presumably made to rest — cannot withstand the
"constitutional test" of Section 3, Article IV, 20 which safeguards the right against
unreasonable searches and seizures, and requires the interposition of a "judge, or
such other responsible officer as may be authorized by law," meaning "a neutral
and detached magistrate competent to determine probable cause (Shadwick v. City
of Tampa, 40 LW 4758; Castillo v. Jias, 62 SCRA 124; Ang Tibay v. CIR, 69 Phil.
635; Zambales Chromite, etc. v. C.A., 934 SCRA 2617);" and since a mayor is
obviously not such a magistrate (U.S. v. Chadwick, 433 U.S. 197), the orders of
arrest at bar are constitutionally infirm; 21 moreover, said Section 143 of the Local
Government Code is "merely a rule of procedure . . . (and is thus) deemed to have
been superseded by the New Rules of Criminal Procedure;" 22

3) the Mayor’s examination "falls short of the requirements of ‘searching questions


and answers;" ‘ the statements of the witnesses supposedly interrogated by the
mayor, are either merely conclusions of law or sterile as regards seditious
utterances, hence, probable cause was non-existent; 23 and

4) Article 142 of the Revised Penal Code, as amended — under which the
petitioners are charged — is based on the US Sedition Act of 1978, which has been
declared by the US Supreme Court as "repugnant to the constitutional guarantee of
freedom of speech and expression (New York Times Co. v. Sullivan, 376 U.S.
254);" hence, said Article 142 is also fatally flawed and therefore, the warrant of
arrest in question was in effect issued for a "crime which in the context of the
constitutionally guaranteed freedom of speech and expression does not exist." 24

In his comment filed with this Court, 25 Judge Ignalaga argues that —
1) The validity of the preliminary examination on the basis of which the arrest
warrants were issued, may not be raised for the first time in the Supreme Court, but
should first be ventilated before the RTC having cognizance of the crime. 26 In
any case, upon the considerations set out in his resolution of October 11, 1985, the
preliminary examination in question is valid. 27

2) The petitioners raise factual issues which are best left for determination by the
RTC, the Supreme Court not being a trier of facts. 28

3) The Mayor in fact followed the procedure in the constitution prior to issuing the
arrest warrants. 29

4) The constitutionality of PD 1974 should be assailed by separate petition. 30

The Solicitor General at the time, Estelito Mendoza, also filed a comment on
December 6, 1985. 31 The comment addressed itself only to the issue of "the
validity of the law authorizing municipal mayors to issue warrants of arrest and the
law punishing inciting to sedition," in view of the fact that the respondents had
presented their own separate comments on the petition. 32 The comment points out
that:chanrob1es virtual 1aw library

1) Section 3, Article IV, of the 1973 Constitution, which mentions a "judge, or


such other responsible officer as may be authorized by law" as vested with
competence to conduct preliminary investigations, is an innovation. In
implementation thereof, BP Blg. 337 was enacted on February 10, 1983,
empowering mayors to conduct preliminary investigations (Sec. 143). This is a
recognition of the truism that the determination of probable cause is but a quasi
judicial function (Ocampo v. US, 58 L ED 1231). 33 Petitioners’ American
authorities are not applicable: the mayor is the highest official in the municipality;
he exercises only general supervision over the police but is not directly involved in
police work; the old rules precisely expressed a recognition of the capability of
mayors to determine probable cause, and the omission of mayors in the 1985 Rules
simply means that the determination of officers who may be authorized to conduct
preliminary investigations was deemed best left to legislation. 34

2. The Philippine sedition law is not akin to the US Sedition Law; in any event our
own sedition law has passed the test of constitutionality (Espuelas v. People, 90
Phil. 524). 35

Solicitor General Sedfrey A. Ordonez subsequently declared that he "stands by the


constitutionality of the statutes the petitioners question and, therefore, sustains the
position taken by his predecessor in office;" 36 this, in response to this Court’s
resolution dated May 15, 1986, requiring the parties to state whether supervening
events had transpired materially affecting the case. 37

It is clear from the outset that the issue before this Court does not involved the
adjudgment of the guilt or innocence of the soldiers in the tragic and regrettable
killings in front of the Escalante Town Hall in the early afternoon of that day, the
twentieth of September, 1986, an event that caused a great outcry of lamentation
and condemnation throughout the land. This is a matter that should be and is in fact
now subject of a separate criminal proceeding. Neither is the propriety of the
victims’ exercise of their constitutional rights of free speech and free assembly for
redress of grievances in the premises at issue here. The basic question before the
Court is divorced of the drama and the passion of those issues; it deals mainly with
the dry, unexciting, but nonetheless important matter of whether or not the
municipal mayor has the power to conduct preliminary investigations in the light
of the 1985 amendments of the rules governing criminal procedure in the Rules of
Court. The answer to the question entails a re examination and analysis of the
relevant legal revisions.chanrobles law library

The 1964 Rules of Court explicitly gave the mayor authority to conduct
preliminary investigations.

"SECTION 3. Preliminary examination by the municipal mayor. — In case of


temporary absence of both the municipal and the auxiliary municipal judges from
the municipality, town, or place wherein they exercise their jurisdiction, the
municipal mayor shall make the preliminary examination in criminal cases when
such examination cannot be delayed without prejudice to the interest of justice. He
shall make a report of any preliminary examination so made to the municipal or to
the auxiliary municipal judge immediately upon the return of one or the other. He
shall have authority in such cases to order the arrest of the defendant and to grant
him bail in the manner and cases provided for in Rule 114." 38

The Rules of 1940 contained an identical provision, in Section 3, Rule 108.

The 1985 Rules on Criminal Procedure did not reproduce this provision, and did
not include the mayor in the enumeration of the officers authorized to conduct
preliminary investigations, those listed being judges of municipal trial courts and
municipal circuit trial courts; city or provincial fiscals and their assistants; national
and regional state prosecutors; and "such other officers as may be authorized by
law." 39

"SECTION 6. When warrant of arrest may issue. —

"(a) By the Regional Trial Court. — Upon the filing of an information, the
Regional Trial Court may issue a warrant for the arrest of the accused.
"(b) By the Municipal Trial Court. — If the municipal trial judge conducting the
preliminary investigation is satisfied after an examination in writing and under oath
of the complainant and his witnesses in the form of searching questions and
answers, that a probable cause exists and that there is a necessity of placing the
respondent under immediate custody in order not to frustrate the ends of justice, he
shall issue a warrant of arrest. (n)." 40

It is of course the deletion in the 1985 rules of reference to the municipal mayor as
among those authorized to conduct preliminary investigations and order arrests
upon which the petitioners chiefly rely as basis for their impugnation of Mayor
Lumayno’s preliminary investigation and order of arrest based thereon.

The matter is however treated of not solely by the Rules of Court but also by the
1973 Constitution, Section 3, Article IV whereof reads:jgc:chanrobles.com.ph

"SECTION 3. The right of the people to be secure in their persons, houses, papers,
and effects against unreasonable searches and seizures of whatever nature and for
any purpose shall not be violated, and no search warrant or warrant of arrest shall
issue except upon probable cause to be determined by the judge, or such other
responsible officer as may be authorized by law, after examination under oath or
affirmation of the complainant and the witnesses he may produce and particularly
describing the place to be searched, and the persons or things to be seized." 41

Parenthetically, it may be noted that Section 3, Rule III of the 1935 Constitution
mentioned only "the judge" as having power to determine probable cause and issue
search and arrest warrants.

The matter is further dealt with in Section 143 of Batas Pambansa Bilang 337,
otherwise known as the Local Government Code, which took effect on February
10, 1983. This section evidently deems the mayor a "responsible officer" in
contemplation of the cited constitutional provision, and explicitly authorizes him to
conduct preliminary examination in criminal cases and order the arrest of the
accused upon probable cause.

"SECTION 143. Authority of the Mayor to Conduct Preliminary Examination.

"(1) In case of temporary absence of the judge assigned to the municipality, the
mayor may conduct the preliminary examination in criminal cases when in his
opinion the investigation cannot be delayed without prejudice to the interest of
justice.

"(2) No examination shall be conducted unless the parties are assisted by lawyers.

"(3) In cases where he may conduct preliminary examination the mayor shall, upon
probable cause after examination of witnesses, have authority to order the arrest of
the accused and to grant him bail in the manner and cases provided in the Revised
Rules of Court and order his provisional release.

"(4) The mayor shall make a report of any preliminary examination so made
immediately after the return of the judge assigned in the area, or upon the
designation of his replacement." 42

The 1973 Constitution plainly and unmistakably grants to the legislature the power
to determine which "responsible officers," aside from judges, may issue warrants
of arrest after examination under oath or affirmation of the complainant and the
witnesses he may produce. And as plainly and unmistakably, the legislature, the
Batas Pambansa, has in the Local Government Code made the determination that
the mayor is such a "responsible officer" and has in consequence authorized him to
conduct preliminary investigations in criminal cases and order the arrest of the
accused upon a finding of probable cause.chanrobles virtual lawlibrary

The first issue raised by the petitioners — that in September, 1985 the mayor no
longer had power to conduct preliminary investigations and issue arrest warrants
43 — must therefore be resolved against them. The argument that Section 143 of
the Local Government Code is just "a rule of procedure merely having
incorporated Rule 112, Section 2 of the old rules of Criminal Procedure," and
should therefore be deemed "superseded by the New Rules of Criminal Procedure
pursuant to the power of this . . . Court to promulgate rules of procedure (Article
X, Section 5 [5], Constitution . . .)" 44 cannot be sustained. However superior the
Supreme Court may be to the other branches of government in the realm of
adjudication, its power to ordain rules of court was at the time inferior to the
lawmaking power of the legislature. 45 It is true that the 1935 Constitution
repealed all procedural laws then in force "as statutes" and declared them to be
"rules of court:" but this was only so that they could be subject to repeal or
modification by the Supreme Court, which was given the power to promulgate
rules of procedure, 46 and has since been exercising such power by promulgating
the Rules of Court of 1940 and of 1964, and the 1985 Rules on Criminal
Procedure, etc. But the 1935 and 1973 Constitutions explicitly conferred on the
legislature the power to repeal, alter or supplement those rules of court, although it
would appear that power is no longer granted to it by the 1987 Charter. 47

Now, the conditions under which the mayor could conduct preliminary
investigations are also clearly indicated by the law, to wit:chanrob1es virtual 1aw
library

1) in case of the temporary absence of the judge assigned to the municipality; and

2) in his (the mayor’s) opinion, the investigation cannot be delayed without


prejudice to the interests of justice.

On these occasions, the mayor may order the arrest of the accused upon a finding
of probable cause after searching examination of the complainant and the witnesses
the latter may present.

In the case at bar, there is no showing that these conditions have not been met.
Indeed, quite apart from the presumption that official duty has been regularly
performed. 48 and the affirmative statements of respondent official that on that
fateful 20th day of September, 1985, Judge Ignalaga was in Manila 49 "on official
leave or absence," 50 the petitioners have not put the fulfillment of those
conditions at issue at all, or the matter of whether or not the accused were "assisted
by lawyers" in the course of the investigation. 51

The petitioners also challenge the constitutionality of Section 143 of the Local
Government Code, it being claimed that the mayor has been thereby authorized to
conduct preliminary investigations and issue warrants of arrest, although he can
not be deemed a "neutral and detached magistrate" within the contemplation of
Section 3, Article IV of the 1973 Constitution. 52

The competence of the Batasang Pambansa to decide and declare by statute which
"responsible officers," aside from judges, should be entrusted with the authority to
conduct preliminary investigations and issue arrest warrants cannot be doubted in
view of the clear language of the Constitution. 53 And the determination by the
Batasan that a municipal mayor is one such "responsible officer" who may
properly be entrusted with the function of conducting preliminary investigations
and ordering arrests of suspects upon probable cause, can not be subject of judicial
review, absent any indication that the legislative proceedings leading to that
statutory determination are void on account of some grave cause. Certainly, the
wisdom of the statute, or the validity of the reasons underlying it, or the adequacy
of the statistics, facts and circumstances considered by the legislature in its
enactment, are beyond the sphere of inquiry of the courts 54 The Batasan was
apparently of the belief that a municipal mayor could be sufficiently objective and
impartial as to be relied upon to conduct preliminary investigations and issue
orders of arrest in the exceptional situation when the judge assigned in the
municipality was absent. The petitioners disagree. They contend that the mayor
cannot in the very nature of things be "neutral and detached." The disagreement,
and the fact that plausible reasons may be adduced by one side or the other on the
proposition does not make the question a justiciable one. The theory advocated by
the petitioners that the mayor’s "deep involvement in law enforcement functions is
likely to color his judgment as a trier of probable cause," 55 does not induce
persuasion. In the first place the premise cannot be conceded. While it is true that
the mayors do "exercise general supervision over units and elements of the INP
stationed or assigned in their respective jurisdictions," they are not themselves
directly involved in police work and cannot in any sense be described, as the
petitioners do, as being deeply involved in law enforcement functions. And even if
that "deep involvement" be conceded, it does not follow that this would necessarily
preclude their assuming "the cold neutrality of an impartial judge" in conducting
preliminary investigations of persons suspected of
crimes.chanroblesvirtualawlibrary

But it must be emphasized here and now that what has just been described is the
state of the law as it was in September, 1985. The law has since been alter. No
longer does the mayor have at this time the power to conduct preliminary
investigations, much less issue orders of arrest. Section 143 of the Local
Government Code, conferring this power on the mayor has been abrogated,
rendered functus officio by the 1987 Constitution which took effect on February 2,
1987, the date of its ratification by the Filipino people. Section 2, Article III of the
1987 Constitution pertinently provides that "no search warrant or warrant of arrest
shall issue except upon probable cause to be determined personally by the judge
after examination under oath or affirmation of the complainant and the witnesses
he may produce, and particularly describing the place to be searched and the
person or things to be seized." The constitutional proscription has thereby been
manifested that thenceforth, the function of determining probable cause and
issuing, on the basis thereof, warrants of arrest or search warrants, may be validly
exercised only by judges, this being evidenced by the elimination in the present
Constitution of the phrase, "such other responsible officer as may be authorized by
law" found in the counterpart provision of said 1973 Constitution, 56 — who, aside
from judges, might conduct preliminary investigations and issue warrants of arrest
or search warrants.

As the law now stands, the mayor may no longer conduct preliminary
investigation, the authority to do so being limited under Section 2, Rule 112 of the
Rules of Court to (1) provincial or city fiscals and their assistants; (2) judges of the
Municipal Trial Courts and Municipal Circuit Trial Courts; (3) national and
regional state prosecutors; and (d) such other officers as may be authorized by law.
57 But only "the judge" may issue search and arrest warrants after due
determination of probable cause. 58

The petitioners’ assault, on the other hand, upon the constitutionality of Article 141
of the Revised Penal Code, defining and penalizing the felony of inciting to
sedition, upon the claim that it was "borrowed" from the U.S. Sedition Act of 1798
which in turn has been struck down as inconsistent with the First Amendment of
the American Constitution, 59 is repelled by the Solicitor General’s arguments. 60

"In the first place, . . . our law on inciting to sedition is not akin to the US Sedition
Act of 1798 which was imposed on the American colonies by their British ruler.
With the success of the American revolution, the 1798 Sedition Act naturally
ceased to have effect as it would be utterly incongrous to punish those who sought
the overthrow of the British government in America.

"x       x       x

"To annul our law on sedition is to give license to those who seek the application
of lawless methods in the advancement of their political views. Our constitution
surely does not contemplate this."cralaw virtua1aw library

Finally, the petitioners postulate that in the determination of the existence of


probable cause from the constitutional aspect, it is required that:" (1) The judge
(or) officer must examine the . . . witnesses personally; (2) The examination must
be under oath; and (3) The examination must be reduced to writing in the form of
searching questions and answers. (Marinas v. Siochio, 104 SCRA 403)."cralaw
virtua1aw library

In the light of these principles they contend: firstly, that the mayor’s questioning of
the witnesses was not "searching" enough; and secondly, that the witnesses’
testimony does not establish prima facie the commission of the felony of inciting to
sedition.chanrobles virtualawlibrary chanrobles.com:chanrobles.com.ph

The fact is that is shown by the record, questions about the material events were in
truth propounded by Mayor Lumayno to the chief witness. Capt. Sanson; 61 and no
proof to the contrary has been submitted. The circumstance that the answers given
by Capt. Sanson to the mayor’s questions are closely reflective of the contends of
his affidavit should not come as a surprise and cannot, without more, be taken as
debilitating or nullifying the interrogation.

The petitioner’s second point is much more substantial and is decisive of the
controversy. The petitioners are correct in their claim that Capt. Sanson’s
testimony 62 does not in truth contain any facts demonstrating the actuality of the
crime of inciting to sedition, which is the crime for which arrest warrants were
issued by Mayor Lumayno. The facts recited by Capt. Sanson may perhaps warrant
a charge of traffic obstruction, or grave coercion, or malicious mischief, or illegal
possession of firearms or deadly weapons, or maybe, attempted murder or
homicide. But it is barren of facts to support an accusation of inciting to sedition.

The other evidence on record exhibits the same barreness.

Two witnesses, Godofredo Hoyo-A and Elpidio Carbajosa, advert in their


affidavits 63 to the shouting of "seditious words," etc. Hoyo-A is quoted as
deposing that "the demonstrators were already unruly . . . (and kept) on shouting
anti-government slogans, . . . invectives and seditious words against the
government;" 64 that certain of the petitioners were identified as being "against the
government, . . . and using the (Mt. Carmel) School in doctrining (sic) the students
against the government;" 65 that petitioner Rogelio Arnaiz had delivered a speech
in which he had said, "Rumpagon ang dictatorial na regimen ni Pres. Marcos,
pamatukan ang NSL, ibagsak ang military kagwalaon ang CHDF , suklan nato ang
gobyerno," a woman speaker had said the same thing, and a third speaker,
petitioner Daniel Gempesala, also stated: Lumpagon ang dictatorial na regimen ni
Marcos, kag ibagsak ang Gobyerno;" 66 and they had elicited a response from the
people there who had raised their right hands and repeated the shouted "slogan." 67
Carbajosa in his turn declared that the demonstrators . . . were shouting for trouble
and ready to make revolution with the use of arms." 68

Another affiant, Eduardo Flores, stated that the demonstrators were unruly,
"tumultuously shouting seditious words against the government and shouting for
revolution." 69

The sworn statement of the only other witness, Leopoldo Villalon, 70 is totally
innocuous as far as proving the elements of inciting to sedition is
concerned.chanrobles.com : virtual law library

The evidence can not justify the action taken by the respondent Mayor and Judge.
The Court thus declares as sorely inadequate and mortally defective the avowed
evidentiary foundation for Mayor Lumayno’s finding of probable cause respecting
the commission by the petitioners of the crime of inciting to sedition. The affiants’
declarations in their sworn statements which might otherwise be pertinent to the
offense, are generalities, mere conclusions of their, not positive averments of
particular facts within their personal knowledge. They do not identify the specific
persons supposed to have perpetrated the crime charged, except two. But even the
identification of these two is of no moment. For except as regards Capt. Sanson,
whose testimony, to repeat, is in any case ineffectual to prove the precise offense
ascribed to the petitioners, there had been no searching interrogation by Mayor
Lumayno of the witnesses as required by the Constitution. Hence, whatever credit
could possibly have been accorded to the affidavit of Hoyo-A — which ventures to
quote the exact words allegedly shouted by petitioners Arnaiz and Gempesela, and
an unidentified woman — was thereby effectively foreclosed.

WHEREFORE, the writs of certiorari and prohibition are granted. The order of


respondent Mayor Lumayno issued on September 20, 1985 and the resolution
promulgated by respondent Judge Ignalaga on October 11, 1985 are annulled and
set aside, and the respondents are perpetually forbidden to enforce or in any way
implement the orders for the arrest of any of the petitioners. No costs.

Teehankee, C.J., Yap, Fernan, Melencio-Herrera, Gutierrez, Jr., Cruz, Paras,


Feliciano, Gancayco, Padilla, Bidin and Cortes, JJ., concur.
Sarmiento, J., took no part.
Morano v. Vivo, G.R. No. L-22196, June 30, 1967

Chan Sau Wah, a Chinese citizen born in Fukien, China on January 6, 1932,
arrived in the Philippines on November 23, 1961 to visit her cousin, Samuel Lee
Malaps. She left in mainland China two of her children by a first marriage: Fu Tse
Haw and Fu Yan Kai With her was Fu Yan Fun, her minor son also by the first
marriage, born in Hongkong on September 11, 1957.

Chan Sau Wah and her minor son Fu Yan Fun were permitted only into the
Philippines under a temporary visitor's visa for two (2) months and after they
posted a cash bond of P4,000.00.

On January 24, 1962, Chan Sau Wah married Esteban Morano, a native-born
Filipino citizen. Born to this union on September 16, 1962 was Esteban Morano,
Jr.

To prolong their stay in the Philippines, Chan Sau Wah and Fu Yan Fun obtained
several extensions. The last extension expired on September 10, 1962.1äwphï1.ñët

In a letter dated August 31, 1962, the Commissioner of Immigration ordered Chan
Sau Wah and her son, Fu Yan Fun, to leave the country on or before September 10,
1962 with a warning that upon failure so to do, he will issue a warrant for their
arrest and will cause the confiscation of their bond.

Instead of leaving the country, on September 10, 1962, Chan Sau Wah (with her
husband Esteban Morano) and Fu Yan Fun petitioned the Court of First Instance of
Manila for mandamus to compel the Commissioner of Immigration to cancel
petitioners' Alien Certificates of Registration; prohibition to stop the Commissioner
from issuing a warrant for their arrest, and preliminary injunction to restrain the
Commissioner from confiscating their cash bond and from issuing warrants of
arrest pending resolution of this case.1 The trial court, on November 3, 1962, issued
the writ of preliminary injunction prayed for, upon a P2,000-bond. After trial and
the stipulations of facts filed by the parties, the Court of First Instance rendered
judgment, viz:

IN VIEW OF ALL THE FOREGOING, judgment is hereby rendered as


follows:

(a) Granting this petition for Mandamus and Prohibition with respect to


petitioner CHAN SAU WAH, who is hereby declared a citizen of the
Philippines; ordering the respondent to cancel her Alien Certificate of
Registration and other immigration papers, upon the payment of proper
dues; and declaring the preliminary injunction with respect to her
permanent, prohibiting the respondent, his representatives or subordinates
from arresting and/or deporting said petitioner;

(b) Dismissing this petition with respect to petitioner FU YAN FUN, and
dissolving the writ of preliminary injunction issued herein, restraining the
respondent, his representatives or subordinates from arresting and/or
deporting said petitioner;

(c) Authorizing respondent Commissioner to forfeit the bond filed by herein


petitioners CHAN SAU WAH and FU YAN FUN in the amount of
P4,000.00; and

(d) Denying, for lack of merit, the prayer to declare Sec. 37 (a) of the
Philippine Immigration Act of 1940 unconstitutional;

Without pronouncement, as to costs.

Petitioners and respondent Commissioner both appealed.

We will deal with the claims of both appellants in their proper sequence.

1. The Solicitor General's brief assails the trial court's declaration that Chan Sau
Wah is a citizen of the Philippines. The court a quo took the position that "Chan
Sau Wah became, by virtue of, and upon, her marriage to Esteban Morano, a
natural-born Filipino, a Filipino citizen.2

Placed to the fore is paragraph 1, Section 15 of Commonwealth Act 473 [Revised


Naturalization Act], which reads:

Sec. 15. Effect of the naturalization on wife children. — Any woman who is


now or may hereafter be married to a citizen of the Philippines, and who
might herself be lawfully naturalized shall be deemed a citizen of the
Philippines.

To apply this provision, two requisites must concur: (a) valid marriage of an alien
woman to a citizen of the Philippines and (b) the alien woman herself might be
lawfully naturalized.

We may concede that the first requisite has been properly met. The validity of the
marriage is presumed.

But can the same be said of the second requisite? This question by all means is not
new. In a series of cases, this Court has declared that the marriage of an alien
woman to a Filipino citizen does not ipso facto make her a Filipino citizen. She
must satisfactorily show that she has all the qualifications and none of the
disqualifications required by the Naturalization Law. 3 Ly Giok Ha alias Wy Giok
Ha et al. vs. Emilio Galang, L-21332, March 18, 1966,* clearly writes down the
philosophy behind the rule in the following expressive language, viz:

Reflection will reveal why this must be so. The qualifications prescribed
under section 2 of the Naturalization Act, and the disqualifications
enumerated in its section 4, are not mutually exclusive; and if all that were
to be required is that the wife of a Filipino be not disqualified under section
4, the result might well be that citizenship would be conferred upon persons
in violation of the policy of the statute. For example, section 4 disqualifies
only —

"(c) Polygamists or believers in the practice of polygamy; and

(b) Persons convicted of crimes involving moral turpitude,"

so that a blackmailer, or a maintainer of gambling or bawdy houses, not


previously convicted by a competent court, would not be thereby
disqualified; still it is certain that the law did not intend such a person to, be
admitted as a citizen in view of the requirement of section 2 that an
applicant for citizenship "must be of good moral character."

Similarly, the citizen's wife might be a convinced believer in racial


supremacy, in government by certain selected classes, in the right to vote
exclusively by certain "herrenvolk," and thus disbelieve in the principles
underlying the Philippine Constitution; yet she would not be disqualified
under section 4, as long as she is not "opposed to organized government,"
nor affiliated to groups "upholding or teaching doctrines opposing all
organized governments," nor "defending or teaching the necessity or
propriety of violence, personal assault or assassination for the success or
predominance of their ideas." Et sic de caeteris.

Upon the principle of selective citizenship, we cannot afford to depart from the
wise precept affirmed and reaffirmed in the cases heretofore noted.

In the additional stipulation of facts of July 3, 1963, petitioners admit that Chan
Sau Wah is not possessed of all the qualifications required by the Naturalization
Law.

Because of all these we are left under no doubt that petitioner Chan Sau Wah did
not become a Filipino citizen.

2. Squarely put in issue by petitioners is the constitutionality of Section 37 (a) of


the Immigration Act of 1940, which reads:
Sec. 37. (a) The following aliens shall be arrested upon the warrant of the
Commissioner of Immigration or of any other officer designated by him for
the purpose and deported upon the warrant of the Commissioner of
Immigration after a determination by the Board of Commissioners of the
existence of the ground for deportation as charged against the alien:

xxx     xxx     xxx

(7) Any alien who remains in the Philippines in violation of any limitation
or condition under which he was admitted as a nonimmigrant.

Petitioners argue that the legal precept just quoted trenches upon the constitutional
mandate in Section 1 (3), Article III [Bill of Rights] of the Constitution, to wit:

(3) The right of the people to be secure in their persons, houses, papers, and
effects against unreasonable searches and seizures shall not be violated, and
no warrants shall issue but upon probable cause, to be determined by the
judge after examination under oath or affirmation of the complainant and the
witnesses he may produce, and particularly describing the place to be
searched, and the persons or things to be seized.

They say that the Constitution limits to judges the authority to issue warrants of
arrest and that the legislative delegation of such power to the Commissioner of
Immigration is thus violative of the Bill of Rights.

Section 1 (3), Article III of the Constitution, we perceive, does not require judicial
intervention in the execution of a final order of deportation issued in accordance
with law. The constitutional limitation contemplates an order of arrest in the
exercise of judicial power4 as a step preliminary or incidental to prosecution or
proceedings for a given offense or administrative action, not as a measure
indispensable to carry out a valid decision by a competent official, such as a legal
order of deportation, issued by the Commissioner of Immigration, in pursuance of
a valid legislation.

The following from American Jurisprudence,5 is illuminating:

It is thoroughly established that Congress has power to order the deportation


of aliens whose presence in the country it deems hurtful. Owing to the
nature of the proceeding, the deportation of an alien who is found in this
country in violation of law is not a deprivation of liberty without due
process of law. This is so, although the inquiry devolves upon executive
officers, and their findings of fact, after a fair though summary hearing, are
made conclusive.

xxx     xxx     xxx
The determination of the propriety of deportation is not a prosecution for, or
a conviction of, crime; nor is the deportation a punishment, even though the
facts underlying the decision may constitute a crime under local law. The
proceeding is in effect simply a refusal by the government to harbor persons
whom it does not want. The coincidence of local penal law with the policy
of Congress is purely accidental, and, though supported by the same facts, a
criminal prosecution and a proceeding for deportation are separate and
independent.

In consequence, the constitutional guarantee set forth in Section 1 (3), Article III of
the Constitution aforesaid, requiring that the issue of probable cause be determined
by a judge, does not extend to deportation proceedings. 6

The view we here express finds support in the discussions during the constitutional
convention. The convention recognized, as sanctioned by due process, possibilities
and cases of deprivation of liberty, other than by order of a competent court. 7

Indeed, the power to deport or expel aliens is an attribute of sovereignty. Such


power is planted on the "accepted maxim of international law, that every sovereign
nation has the power, as inherent in sovereignty, and essential to self-preservation,
to forbid the entrance of foreigners within its dominions." 8 So it is, that this Court
once aptly remarked that there can be no controversy on the fact that where aliens
are admitted as temporary visitors, "the law is to the effect that temporary visitors
who do not depart upon the expiration of the period of stay granted them are
subject to deportation by the Commissioner of Immigration, for having violated the
limitation or condition under which they were admitted as non-immigrants
(Immigration Law, Sec. 37 (a), subsection (7); C.A. 613, as amended)." 9

And, in a case directly in point, where the power of the Commissioner to issue
warrants of arrest was challenged as unconstitutional, because "such power is only
vested in a judge by Section 1, paragraph 3, Article III of our Constitution," this
Court declared —

This argument overlooks the fact that the stay of appellant Ng Hua To as
temporary visitor is subject to certain contractual stipulations as contained in
the cash bond put up by him, among them, that in case of breach the
Commissioner may require the recommitment of the person in whose favor
the bond has been filed. The Commissioner did nothing but to enforce such
condition. Such a step is necessary to enable the Commissioner to prepare
the ground for his deportation under section 37 (a) of Commonwealth Act
613. A contrary interpretation would render such power nugatory to the
detriment of the State.10

It is in this context that we rule that Section 37 (a) of the Immigration Act of 1940
is not constitutionally proscribed.
3. A sequel to the questions just discussed is the second error set forth in the
government's brief. The Solicitor General balks at the lower court's ruling that
petitioner Chan Sau Wah is entitled to permanent residence in the Philippines
without first complying with the requirements of Sections 9 and 13 of the
Immigration Act of 1940, as amended by Republic Act 503.

We first go to the law, viz:

SEC. 9 [last paragraph]

An alien who is admitted as a nonimmigrant cannot remain in the


Philippines permanently. To obtain permanent admission, a nonimmigrant
alien must depart voluntarily to some foreign country and procure from the
appropriate Philippine consul the proper visa and thereafter undergo
examination by the officers of the Bureau of Immigration at a Philippine
port of entry for determination of his admissibility in accordance with the
requirements of this Act.

SEC. 13. Under the conditions set forth in this Act there may be admitted
into the Philippines immigrants, termed "quota immigrants" not in excess of
fifty (50) of any one nationality or without nationality for any one calendar
year, except that the following immigrants, termed "nonquota immigrants,"
maybe admitted without regard to such numerical limitations.

The corresponding Philippine Consular representative abroad shall


investigate and certify the eligibility of a quota immigrant previous to his
admission into the Philippines. Qualified and desirable aliens who are in the
Philippines under temporary stay may be admitted within the quota, subject
to the provisions of the last paragraph of section 9 of this Act.

(a) The wife or the husband or the unmarried child under twenty-one years
of age of a Philippine citizen, if accompanying or following to join such
citizen;

(b) A child of alien parents born during the temporary visit abroad of the
mother, the mother having been previously lawfully admitted into the
Philippine for permanent residence, if the child is accompanying or coming
to join a parent and applies for admission within five years from the date of
its birth;

Concededly, Chan Sau Wah entered the Philippines on a tourist-


temporary visitor's visa. She is a non-immigrant. Under Section 13 just quoted, she
may therefore be admitted if she were a qualified and desirable alien and subject to
the provisions of the last paragraph of Section 9. Therefore, first, she must depart
voluntarily to some foreign country; second, she must procure from the appropriate
consul the proper visa; and third, she must thereafter undergo examination by the
officials of the Bureau of Immigration at the port of entry for determination of her
admissibility in accordance with the requirements of the immigration Act.

This Court in a number of cases has ruled, and consistently too, that an alien
admitted as a temporary visitor cannot change his or her status without first
departing from the country and complying with the requirements of Section 9 of
the Immigration Act. 11

The gravamen of petitioners' argument is that Chan Sau Wah has, since her entry,
married in Manila a native-born Filipino, Esteban Morano. It will not particularly
help analysis for petitioners to appeal to family solidarity in an effort to thwart her
deportation. Chan Sau Wah, seemingly is not one who has a high regard for such
solidarity. Proof: She left two of her children by the first marriage, both minors, in
the care of neighbors in Fukien, China.

Then, the wording of the statute heretofore adverted to is a forbidding obstacle


which will prevent this Court from writing into the law an additional provision that
marriage of a temporary alien visitor to a Filipino would ipso facto make her a
permanent resident in his country. This is a field closed to judicial action. No
breadth of discretion is allowed us. We cannot insulate her from the State's power
of deportation.

Really, it would be an easy matter for an alien woman to enter the Philippines as a
temporary visitor, go through a mock marriage, but actually live with another man
as husband and wife, and thereby skirt the provisions of our immigration law.
Also, a woman of undesirable character may enter this country, ply a pernicious
trade, marry a Filipino, and again throw overboard Sections 9 and 13 of the Act.
Such a flanking movement, we are confident, is impermissible.

Recently we confirmed the rule that an alien wife of a Filipino may not stay
permanently without first departing from the Philippines. Reason: Discourage
entry under false pretenses. 12

The ruling of the trial court on this score should be reversed.

4. It is petitioners' turn to point as error the dismissal of the petition


for mandamus and prohibition with respect to petitioner Fu Yan Fun.

Petitioners' line of thought is this: Fu Yan Fun follows the citizenship of his
mother. They cite Section 15, paragraph 3, Commonwealth Act 473, which says
that:
A foreign-born minor child, if dwelling in the Philippines at the time of the
naturalization of the parent, shall automatically become a Philippine
citizen. . . .

Petitioners' position is based on the assumption that Chan Sau Wah, the mother, is
a Filipino citizen. We have held that she is not. At best, Fu Yan Fun is a step-son
of Esteban Morano, husband of Chan Sau Wah. A step-son is not a foreign-born
child of the step-father. The word child, we are certain, means legitimate child, not
a step-child. We are not wanting in precedents. Thus, when the Constitution
provides that "[t]hose whose fathers are citizens of the Philippines" are citizens
thereof, 13 the fundamental charter intends "those" to apply to legitimate
children. 14 In another case, the term "minor children" or "minor child" in Section
15 of the Revised Naturalization Law refers only to legitimate children of Filipino
citizens. This Court, thru Mr. Chief Justice Roberto Concepcion, there said: 15

It is claimed that the phrases "minor children" and "minor child," used in
these provisions, include adopted children. The argument is predicated upon
the theory that an adopted child is, for all intents and purposes, a legitimate
child. Whenever, the word "children" or "child" is used in statutes, it is
generally understood, however, to refer to legitimate children, unless the
context of the law and its spirit indicate clearly the contrary. Thus, for
instance, when the Constitution provides that "those whose fathers are
citizens of the Philippines," and "those whose mothers are citizens of the
Philippines" who shall elect Philippine citizenship upon reaching the age of
majority, are citizens of the Philippines (Article IV, Section 1, subdivisions
[3] and [4]), our fundamental law clearly refers to legitimate children
(Chiongbian vs. De Leon, 46 Off. Gaz., 3652-3654; Serra v. Republic, L-
4223, May 12, 1952).

At any rate, Fu Yan Fun entered the Philippines as a temporary visitor. The status
of a temporary visitor cannot be converted into, that of a permanent resident, as we
have heretofore held, without first complying with Section 9 of the Immigration
Law.

5. Petitioners finally aver that the lower court erred in authorizing respondent
Commissioner to forfeit the bond filed by petitioners Chan Sau Wah and Fu Yan
Fun in the amount of P4,000.00.

Here is petitioners' posture. They enjoyed their stay in the Philippines upon a bond.
Now they come to court and say that as the prescribed form of this bond was not
expressly approved by the Secretary of Justice in accordance with Section 3 of
Commonwealth Act 613, which reads —

SEC. 3. . . . He [Commissioner of Immigration] shall issue, subject to the


approval of the Department Head, such rules and regulations and prescribes
such forms of bond, reports, and other papers, and shall issue from time to
time such instruction, not inconsistent with law, as he shall deem best
calculated to carry out the provisions of the immigration laws. . . .

that bond is void.

Reasons there are which prevent us from giving our imprimatur to this argument.

The provision requiring official approval of a bond is merely directory.


"Irregularity or entire failure in this respect does not affect the validity of the
bond. 16 The reason for the rule, is found in 9 C.J., p. 26 (footnote), which reads:

(a) Reason for rule. — "Statutes requiring bonds to be approved by certain


officials are not for the purpose of protecting the obligors in the bond, but are
aimed to protect the public, to insure their solvency, and to create evidence of an
unimpeachable character of the fact of their execution. When they are executed for
a legal purpose, before a proper tribunal, and are in fact accepted and approved by
the officer or body, whose duty it was to approve them, it could serve no useful
purpose of the law to hold them invalid, to release all the obligors thereon, and to
defeat every purpose of its execution, simply because the fact of approval was not
indorsed precisely as had been directed by the Legislature." American Book Co.
vs. Wells, 83 SW 622, 627, 26 Ky L-1159. (emphasis supplied)

And another. This bond was accepted by the government. It had been there. The
form of the bond here used is of long continued usage. If the government did not
question the form of the bond at all, then we must assume that it counted with the
Secretary's approval. For the presumption is that official duty has been legally
performed.

Surely enough, equitable considerations will stop petitioners from pleading


invalidity of the bond. They offered that bond to enable them to enter and stay in
this country. They enjoyed benefits therefrom. They cannot, "in law, and good
conscience, be allowed to reap the fruits" of that bond, and then jettison the same.
They are "precluded from attacking the validity" of such bond. 17

Actually, to petitioners the bond was good while they sought entry into the
Philippines; they offered it as security for the undertaking; that they "will actually
depart from the Philippines" when their term of stay expires. Now that the bond is
being confiscated because they overstayed, they make an about-face and say that
such bond is null and void. They shall not profit from this inconsistent position.
Their bond should be confiscated.

Conformably to the foregoing, the judgment under review is hereby modified as


follows:
(1) The portion thereof which reads:

(a) Granting their petition for Mandamus and Prohibition with respect to


petitioner CHAN SAU WAH, who is hereby declared a citizen of the
Philippines; ordering the respondent to cancel her Alien Certificate of
Registration and other immigration papers, upon the payment of proper
dues; and declaring preliminary injunction with respect to her permanent,
prohibiting the respondent, his representatives or subordinates from
arresting and/or deporting said petitioner;

is hereby reversed: and, in consequence —

The petition for mandamus and prohibition with respect to petitioner Chan Sau


Wah is hereby denied; and the judgment declaring her a citizen of the Philippines,
directing respondent to cancel her Alien Certificate of Registration and other
immigration papers, and declaring the preliminary injunction with respect to her
permanent, are all hereby set aside; and

(2) In all other respects, the decision appealed from is hereby affirmed.

No costs. So ordered.

        
Harvey v. Santiago, G.R. No. 82544, June 28, 1988

Petitioners Andrew Harvey and John Sherman, 52 and 72 years, respectively, are
both American nationals residing at Pagsanjan, Laguna, while Adriaan Van
Elshout, 58 years old, is a Dutch citizen also residing at Pagsanjan, Laguna.

The case stems from the apprehension of petitioners on 27 February 1988 from
their respective residences by agents of the Commission on Immigration and
Deportation (CID) by virtue of Mission Orders issued by respondent
Commissioner Miriam Defensor Santiago of the CID. Petitioners are presently
detained at the CID Detention Center.

Petitioners were among the twenty-two (22) suspected alien pedophiles who were
apprehended after three months of close surveillance by CID agents in Pagsanjan,
Laguna. Two (2) days after apprehension, or on 29 February 1988, seventeen (17)
of the twenty-two (22) arrested aliens opted for self-deportation and have left the
country. One was released for lack of evidence; another was charged not for being
a pedophile but for working without a valid working visa. Thus, of the original
twenty two (22), only the three petitioners have chosen to face deportation.

Seized during petitioners apprehension were rolls of photo negatives and photos of
the suspected child prostitutes shown in salacious poses as well as boys and girls
engaged in the sex act. There were also posters and other literature advertising the
child prostitutes.

The "Operation Report," on Andrew Harvey and Richard Sherman dated 29


February 1988 stated:

xxx xxx xxx

ANDREW MARK HARVEY was found together with two young


boys.

RICHARD SHERMAN was found with two naked boys inside his
room.

In respect of Van Den Elshout the "After Mission Report," dated 27 February 1988
read in part:

Noted:

There were two (2) children ages 14 & 16 which subject


readily accepted having been in his care and live-in for
quite sometime.

On 4 March 1988, deportation proceedings were instituted against petitioners for


being undesirable aliens under Section 69 of the Revised Administrative Code
(Deportation Case No. 88-13). The "Charge Sheet" read inter alia:

Wherefore, this Office charges the respondents for deportation, as


undesirable aliens, in that: they, being pedophiles, are inimical to
public morals, public health and public safety as provided in Section
69 of the Revised Administrative Code.

On 7 March 1988, Warrants of Arrest were issued by respondent against


petitioners for violation of Sections 37, 45 and 46 of the Immigration Act and
Section 69 of the Revised Administrative Code On the same date, the Board of
Special Inquiry III commenced trial against petitioners.

On 14 March 1988, petitioners filed an Urgent Petition for Release Under Bond
alleging that their health was being seriously affected by their continuous
detention. Upon recommendation of the Board of Commissioners for their
provisional release, respondent ordered the CID doctor to examine petitioners, who
certified that petitioners were healthy.

On 22 March 1988, petitioners filed a Petition for Bail which, however, respondent
denied considering the certification by the CID physician that petitioners were
healthy. To avoid congestion, respondent ordered petitioners' transfer to the CID
detention cell at Fort Bonifacio, but the transfer was deferred pending trial due to
the difficulty of transporting them to and from the CID where trial was on-going.

On 4 April 1988 petitioner Andrew Harvey filed a Manifestation/Motion stating


that he had "finally agreed to a self-deportation" and praying that he be
"provisionally released for at least 15 days and placed under the custody of Atty.
Asinas before he voluntarily departs the country." On 7 April 1988, the Board of
Special Inquiry — III allowed provisional release of five (5) days only under
certain conditions. However, it appears that on the same date that the aforesaid
Manifestation/ Motion was filed, Harvey and his co-petitioners had already filed
the present petition.

On 4 April 1988, as heretofore stated, petitioners availed of this Petition for a Writ
of Habeas Corpus. A Return of the Writ was filed by the Solicitor General and the
Court heard the case on oral argument on 20 April 1988. A Traverse to the Writ
was presented by petitioners to which a Reply was filed by the Solicitor General.

Petitioners question the validity of their detention on the following grounds:

1) There is no provision in the Philippine Immigration Act of 1940 nor under


Section 69 of the Revised Administrative Code, which legally clothes the
Commissioner with any authority to arrest and detain petitioners pending
determination of the existence of a probable cause leading to an administrative
investigation.

2) Respondent violated Section 2, Article III of the 1987 Constitution prohibiting


unreasonable searches and seizures since the CID agents were not clothed with
valid Warrants of arrest, search and seizure as required by the said provision.

3) Mere confidential information made to the CID agents and their suspicion of the
activities of petitioners that they are pedophiles, coupled with their association
with other suspected pedophiles, are not valid legal grounds for their arrest and
detention unless they are caught in the act. They further allege that being a
pedophile is not punishable by any Philippine Law nor is it a crime to be a
pedophile.

We reject petitioners' contentions and uphold respondent's official acts ably


defended by the Solicitor General.

There can be no question that the right against unreasonable searches and seizures
guaranteed by Article III, Section 2 of the 1987 Constitution, is available to all
persons, including aliens, whether accused of crime or not (Moncado vs. People's
Court, 80 Phil. 1 [1948]. One of the constitutional requirements of a valid search
warrant or warrant of arrest is that it must be based upon probable cause. Probable
cause has been defined as referring to "such facts and circumstances antecedent to
the issuance of the warrant that in themselves are sufficient to induce a cautious
man to rely on them and act in pursuance thereof." (People vs. Syjuco 64 Phil. 667
[1937]; Alverez vs. CFI, 64 Phil. 33 [1937]).

The 1985 Rules on Criminal Procedure also provide that an arrest wit a warrant
may be effected by a peace officer or even a private person (1) when such person
has committed, actually committing, or is attempting to commit an offense in his
presence; and (2) when an offense has, in fact, been committed and he has personal
knowledge of facts indicating that the person to be arrested has committed it (Rule
113, Section 5).

In this case, the arrest of petitioners was based on probable cause determined after
close surveillance for three (3) months during which period their activities were
monitored. The existence of probable cause justified the arrest and the seizure of
the photo negatives, photographs and posters without warrant (See Papa vs. Mago,
L-27360, February 28, 1968,22 SCRA 857; People vs. Court of First Instance of
Rizal, L-41686, November 17, 1980, 101 SCRA 86, cited in CRUZ, Constitutional
Law, 1987 ed., p. 143). Those articles were seized as an incident to a lawful arrest
and, are therefore, admissible in evidence (Section 12, Rule 126,1985 Rules on
criminal Procedure).

But even assuming arguendo that the arrest of petitioners was not valid at its
inception, the records show that formal deportation charges have been filed against
them, as undesirable aliens, on 4 March 1988. Warrants of arrest were issued
against them on 7 March 1988 "for violation of Section 37, 45 and 46 of the
Immigration Act and Section 69 of the Administrative Code." A hearing is
presently being conducted by a Board of Special Inquiry. The restraint against their
persons, therefore, has become legal. The Writ has served its purpose. The process
of the law is being followed (Cruz vs. Montoya, L-39823, February 25, 1975, 62
SCRA 543). "were a person's detention was later made by virtue of a judicial order
in relation to criminal cases subsequently filed against the detainee, his petition
for hebeas corpus becomes moot and academic" (Beltran vs. Garcia, L-49014,
April 30, 1979, 89 SCRA 717). "It is a fumdamental rule that a writ of habeas
corpus will not be granted when the confinement is or has become legal, although
such confinement was illegal at the beginning" (Matsura vs. Director of Prisons, 77
Phil. 1050 [1947]).

That petitioners were not "caught in the act" does not make their arrest illegal.
Petitioners were found with young boys in their respective rooms, the ones with
John Sherman being naked. Under those circumstances the CID agents had
reasonable grounds to believe that petitioners had committed "pedophilia" defined
as "psychosexual perversion involving children" (Kraft-Ebbing Psychopatia
Sexualis p. 555; Paraphilia (or unusual sexual activity) in which children are the
preferred sexual object" (Webster's Third New International Dictionary, 1971 ed.,
p. 1665) [Solicitor General's Return of the Writ, on p. 101. While not a crime under
the Revised Penal Code, it is behavior offensive to public morals and violative of
the declared policy of the State to promote and protect the physical, moral,
spiritual, and social well-being of our youth (Article II, Section 13, 1987
Constitution).

At any rate, the filing by petitioners of a petition to be released on bail should be


considered as a waiver of any irregularity attending their arrest and estops them
from questioning its validity (Callanta v. Villanueva, L-24646 & L-24674, June 20,
1977, 77 SCRA 377; Bagcal vs. Villaraza, L-61770, January 31, 1983, 120 SCRA
525).

The deportation charges instituted by respondent Commissioner are in accordance


with Section 37(a) of the Philippine Immigration Act of 1940, in relation to
Section 69 of the Revised Administrative Code. Section 37(a) provides in part:

(a) The following aliens shall be arrested upon the warrant of the
Commissioner of Immigration and Deportation or any other officer
designated by him for the purpose and deported upon the warrant of
the Commissioner of Immigration and Deportation after a
determination by the Board of Commissioners of the existence of the
ground for deportation as charged against the alien;

xxx xxx xxx

The foregoing provision should be construed in its entirety in view of the summary
and indivisible nature of a deportation proceeding, otherwise, the very purpose of
deportation proceeding would be defeated.

Section 37(a) is not constitutionally proscribed (Morano vs. Vivo, L-22196, June
30, 1967, 20 SCRA 562). The specific constraints in both the 1935 1 and
1987 2 Constitutions, which are substantially Identical, contemplate prosecutions
essentially criminal in nature. Deportation proceedings, on the other hand, are
administrative in character. An order of deportation is never construed as a
punishment. It is preventive, not a penal process. It need not be conducted strictly
in accordance with ordinary Court proceedings.

It is of course well-settled that deportation proceedings do not


constitute a criminal action. The order of deportation is not a
punishment, (Maliler vs. Eby, 264 U.S., 32), it being merely the
return to his country of an alien who has broken the conditions upon
which he could continue to reside within our borders (U.S. vs. De los
Santos, 33 Phil., 397). The deportation proceedings are administrative
in character, (Kessler vs. Stracker 307 U.S., 22) summary in nature,
and need not be conducted strictly in accordance with the ordinary
court proceedings (Murdock vs. Clark, 53 F. [2d], 155). It is essential,
however, that the warrant of arrest shall give the alien sufficient
information about the charges against him, relating the facts relied
upon. (U.S. vs. Uhl 211 F., 628.) It is also essential that he be given a
fair hearing with the assistance of counsel, if he so desires, before
unprejudiced investigators (Strench vs. Pedaris, 55 F. [2d], 597; Ex
parte Jew You On, 16 F. [2d], 153). However, all the strict rules of
evidence governing judicial controversies do not need to be observed;
only such as are fumdamental and essential like the right of cross-
examination. (U.S. vs. Hughes, 104 F. [2d], 14; Murdock vs. Clark,
53 F. [2d], 155.) Hearsay evidence may even be admitted, provided
the alien is given the opportunity to explain or rebut it (Morrell vs.
Baker, 270 F., 577; Sercerchi vs. Ward, 27 F. Supp., 437). (Lao Tang
Bun vs. Fabre 81 Phil. 682 [1948]).

The ruling in Vivo vs. Montesa (G. R. No. 24576, July 29, 1968, 24 SCRA 155)
that "the issuance of warrants of arrest by the Commissioner of Immigration, solely
for purposes of investigation and before a final order of deportation is issued,
conflicts with paragraph 3, Section I of Article III of the Constitution" (referring to
the 1935 Constitution) 3 is not invocable herein. Respondent Commissioner's
Warrant of Arrest issued on 7 March 1988 did not order petitioners to appear and
show cause why they should not be deported. They were issued specifically "for
violation of Sections 37, 45 and 46 of the Immigration Act and Section 69 of the
Revised Administrative Code." Before that, deportation proceedings had been
commenced against them as undesirable aliens on 4 March 1988 and the arrest was
a step preliminary to their possible deportation.

Section 37 of the Immigration Law, which empowers the


Commissioner of Immigration to issue warrants for the arrest of
overstaying aliens is constitutional. The arrest is a stop preliminary to
the deportation of the aliens who had violated the condition of their
stay in this country. (Morano vs. Vivo, L-22196, June 30, 1967, 20
SCRA 562).

To rule otherwise would be to render the authority given the Commissioner


nugatory to the detriment of the State.

The pertinent provision of Commonwealth Act No. 613, as amended,


which gives authority to the Commissioner of Immigration to order
the arrest of an alien temporary visitor preparatory to his deportation
for failure to put up new bonds required for the stay, is not
unconstitutional.

xxx xxx xxx


... Such a step is necessary to enable the Commissioner to prepare the
ground for his deportation under Section 37[al of Commonwealth Act
613. A contrary interpretation would render such power nugatory to
the detriment of the State. (Ng Hua To vs. Galang, G. R. No. 10145,
February 29, 1964, 10 SCRA 411).

"The requirement of probable cause, to be determined by a Judge, does not extend


to deportation proceedings." (Morano vs. Vivo, supra, citing Tiu Chun Hai vs.
Commissioner, infra). There need be no "truncated" recourse to both judicial and
administrative warrants in a single deportation proceedings.

The foregoing does not deviate from the ruling in Qua Chee Gan vs. Deportation
Board (G. R. No. 10280, September 30, 1963, 9 SCRA 27 [1963]) reiterated
in Vivo vs. Montesa, supra, that "under the express terms of our Constitution (the
1935 Constitution), it is therefore even doubtful whether the arrest of an individual
may be ordered by any authority other than a judge if the purpose is merely to
determine the existence of a probable cause, leading to an administrative
investigation." For, as heretofore stated, probable cause had already been shown to
exist before the warrants of arrest were issued.

What is essential is that there should be a specific charge against the alien intended
to be arrested and deported, that a fair hearing be conducted (Section 37[c]) with
the assistance of counsel, if desired, and that the charge be substantiated by
competent evidence. Thus, Section 69 of the Revised Administrative Code
explicitly provides:

Sec. 69. Deportation of subject of foreign power. A subject of a


foreign power residing in the Philippines shall not be deported,
expelled, or excluded from said Islands or repatriated to his own
country by the President of the Philippines except upon prior
investigation, conducted by said Executive or his authorized agent, of
the ground upon which such action is contemplated. In such a case
the person concerned shall be informed of the charge or charges
against him and he shall be allowed not less than 3 days for the
preparation of his defense. He shall also have the right to be heard by
himself or counsel, to produce witnesses in his own behalf, and to
cross-examine the opposing witnesses.

The denial by respondent Commissioner of petitioners' release on bail, also


challenged by them, was in order because in deportation proceedings, the right to
bail is not a matter of right but a matter of discretion on the part of the
Commissioner of Immigration and Deportation. Thus, Section 37(e) of the
Philippine Immigration Act of 1940 provides that "any alien under arrest in a
deportation proceeding may be released under bond or under such other conditions
as may be imposed by the Commissioner of Immigration." The use of the word
"may" in said provision indicates that the grant of bail is merely permissive and not
mandatory on the part of the Commissioner. The exercise of the power is wholly
discretionary (Ong Hee Sang vs. Commissioner of Immigration, L-9700, February
28,1962, 4 SCRA 442). "Neither the Constitution nor Section 69 of the Revised
Administrative Code guarantees the right of aliens facing deportation to
provisional liberty on bail." (Tiu Chun Hai et al vs. Deportation Board, 104 Phil.
949 [1958]). As deportation proceedings do not partake of the nature of a criminal
action, the constitutional guarantee to bail may not be invoked by aliens in said
proceedings (Ong Hee Sang vs. Commissioner of Immigration, supra).

Every sovereign power has the inherent power to exclude aliens from its territory
upon such grounds as it may deem proper for its self-preservation or public interest
(Lao Tan Bun vs. Fabre 81 Phil. 682 [1948]). The power to deport aliens is an act
of State, an act done by or under the authority of the sovereign power (In re
McCulloch Dick, 38 Phil. 41 [1918]). It is a police measure against undesirable
aliens whose continued presence in the country is found to be injurious to the
public good and the domestic tranquility of the people (Forbes vs. Chuoco Tiaco et
al., 16 Phil. 534 [1910]). Particularly so in this case where the State has expressly
committed itself to defend the tight of children to assistance and special protection
from all forms of neglect, abuse, cruelty, exploitation, and other conditions
prejudicial to their development (Article XV, Section 3[2]). Respondent
Commissioner of Immigration and Deportation, in instituting deportation
proceedings against petitioners, acted in the interests of the State.

WHEREFORE, the Petition is dismissed and the Writ of Habeas Corpus is hereby
denied.

SO ORDERED.

Ho v. People, G.R. No. 106632, October 9, 1997

May a judge issue a warrant of arrest solely on the basis of the report and
recommendation of the investigating prosecutor, without personally determining
probable cause by independently examining sufficient evidence submitted by the
parties during the preliminary investigation?

The Case

This is the main question raised in these two consolidated petitions


for certiorari under Rule 65 of the Rules of Court challenging the Sandiganbayan's
August 25, 1992 Resolution 1 which answered the said query in the affirmative.

The Facts
Both petitions have the same factual backdrop. On August 8, 1991, the Anti-Graft
League of the Philippines, represented by its chief prosecutor and investigator,
Atty. Reynaldo L. Bagatsing, filed with the Office of the Ombudsman a
complaint 2 against Doris Teresa Ho, Rolando S. Narciso (petitioners in G.R. Nos.
106632 and 106678, respectively), Anthony Marden, Arsenio Benjamin Santos and
Leonardo Odoño. The complaint was for alleged violation of Section 3 (g) of
Republic Act 3019 3 prohibiting a public officer from entering into any contract or
transaction on behalf of the government if it is manifestly and grossly
disadvantageous to the latter, whether or not the public officer profited or will
profit thereby. After due notice, all respondents therein filed their respective
counter-affidavits with supporting documents. On January 8, 1992, Graft
Investigation Officer Titus P. Labrador (hereafter, "GIO Labrador") submitted his
resolution 4 with the following recommendations:

WHEREFORE, all premises considered, it is respectfully


recommended that an information for violation of Section 3 (g) of
R.A. 3019 as amended be filed against respondent Rolando S.
Narciso before the Sandiganbayan.

It is likewise recommending that the case against the other


respondents be DISMISSED for insufficiency of evidence.

However, after a review of the above resolution, Special Prosecution Officer


Leonardo P. Tamayo (hereafter "SPO Tamayo") recommended that
both Rolando Narciso and Doris Teresa Ho be charged with violation of Section 3
(e) of R.A. 3019. The resolution of GIO Labrador, as modified by the
memorandum 5 of SPO Tamayo, was approved by Ombudsman Conrado M.
Vasquez on May 5, 1992. Thus, herein petitioners were charged accordingly before
the Sandiganbayan in an information 6 filed on May 18, 1992. Attached to the
information were the resolution of GIO Labrador and the memorandum of SPO
Tamayo. The said information reads:

The undersigned Special Prosecution Officer III, Office of the Special


Prosecutor, hereby accuses ROLANDO NARCISO and DORIS
TERESA HO, President of National Marine Corporation, of violation
of Section 3(e) of RA 3019, as amended, committed as follows:

That on or about April 4, 1989, and for sometime prior


and/or subsequent thereto, in the City of Manila,
Philippines and within the jurisdiction of this Honorable
Court, the above-named accused ROLANDO
NARCISO, a public officer, being then the Vice-
President of the National Steel Corporation (NSC), a
government-owned or controlled corporation organized
and operating under the Philippine laws, and DORIS
TERESA HO, a private individual and then the
President of National Marine Corporation (NMC), a
private corporation organized and operating under our
Corporation law, conspiring and confederating with one
another, did then and there wilfully, unlawfully and
criminally, with evident bad faith and through manifest
partiality, cause undue injury to the National Steel
Corporation (NSC), by entering without legal
justification into a negotiated contract of affreightment
disadvantageous to the NSC for the haulage of its
products at the rate of P129.50/MT, from Iligan City to
Manila, despite their full knowledge that the rate they
have agreed upon was much higher than those offered
by the Loadstar Shipping Company, Inc. (LSCI) and
Premier Shipping Lines, Inc. (PSLI), in the amounts of
P109.56 and P123.00 per Metric Ton, respectively, in
the public bidding held on June 30, 1988, thereby giving
unwarranted benefits to the National Marine
Corporation, in the total sum of One Million One
Hundred Sixteen Thousand Fifty Two Pesos and
Seventy Five Centavos (P1,116,052.75), Philippine
Currency, to the pecuniary damage and prejudice of the
NSC in the aforestated sum. The said offense was
committed by Rolando S. Narciso in the performance of
his official functions as Vice-President of the National
Steel Corporation.

CONTRARY TO LAW.

Acting on the foregoing information, the Sandiganbayan issued the now questioned
warrant of arrest against Petitioners Ho and Narciso. Petitioner Ho initially
questioned the issuance thereof in an "Urgent Motion to Recall Warrant of
Arrest/Motion for Reconsideration" which was adopted by Petitioner Narciso.
They alleged that the Sandiganbayan, in determining probable cause for the
issuance of the warrant for their arrest, merely relied on the information and the
resolution attached thereto, filed by the Ombudsman without other supporting
evidence, in violation of the requirements of Section 2, Article III of the
Constitution, and settled jurisprudence. Respondent Sandiganbayan denied said
motion in the challenged Resolution. It ratiocinated in this wise.

Considering, therefore, that this Court did not rely solely on


the certification appearing in the information in this case in the
determination of whether probable cause exists to justify the issuance
of the warrant of arrest but also on the basis predominantly shown by
the facts and evidence appearing in the resolution/memorandum of
responsible investigators/prosecutors, then the recall of the warrant of
arrest, or the reconsideration sought for, cannot be granted. More so,
when the information, as filed, clearly shows that it is sufficient in
form and substance based on the facts and evidence adduced by both
parties during preliminary investigation. To require this Court to have
the entire record of the preliminary investigation to be produced
before it, including the evidence submitted by the complainant and
the accused-respondents, would appear to be an exercise in futility.

Thus, these petitions.

The Issue

Petitioner Ho raises this sole issue:

May a judge determine probable cause and issue [a] warrant of


arrest solely on the basis of the resolution of the prosecutor (in the
instant case, the Office of the Special Prosecutor of the Ombudsman)
who conducted the preliminary investigation, without having before
him any of the evidence (such as complainant's affidavit, respondent's
counter-affidavit, exhibits, etc.) which may have been submitted at
the preliminary investigation? 7

In his separate petition, Rolando S. Narciso adopts the foregoing and raised no
other distinct issue.

Petitioners Ho and Narciso similarly contend that a judge, in personally


determining the existence of probable cause, must have before him
sufficient evidence submitted by the parties, other than the information filed by the
investigating prosecutor, to support his conclusion and justify the issuance of an
arrest warrant. Such evidence should not be "merely described in a prosecutor's
resolution." Citing People vs. Inting, 8 petitioners insist that the judge "must have
before him 'the report, the affidavits, the transcripts of stenographic notes (if any),
and all other supporting documents which are material in assisting the judge to
make his determination.'"

The Court's Ruling

The petitions are meritorious.

The pertinent provision of the Constitution reads:

Sec. 2 [Article III]. The right of the people to be secure in their


persons, houses, papers, and effects against unreasonable searches
and seizures of whatever nature and for any purpose shall be
inviolable, and no search warrant or warrant of arrest shall issue
except upon probable cause to be determined personally by the judge
after examination under oath or affirmation of the complainant and
the witnesses he may produce and particularly describing the place to
be searched and the persons or things to be seized. (Emphasis
supplied.)

In explaining the object and import of the aforequoted constitutional mandate,


particularly the power and the authority of judges to issue warrants of arrest, the
Court elucidated in Soliven vs. Makasiar  9:

What the Constitution underscores is the exclusive and personal


responsibility of the issuing judge to satisfy himself of the existence
of probable cause. In satisfying himself of the existence of probable
cause for the issuance of a warrant of arrest, the judge is not required
to personally examined the complainant and his witnesses. Following
established doctrine and procedure, he shall: (1) personally evaluate
the report and the supporting documents submitted by the fiscal
regarding the existence of probable cause and, on the basis thereof,
issue a warrant of arrest; or (2) if on the basis thereof he finds no
probable cause, he may disregard the fiscal's report and require the
submission of supporting affidavits of witnesses to aid him in arriving
at a conclusion as to the existence of probable cause. 10 [emphasis
supplied]

We should stress that the 1987 Constitution requires the judge to determine
probable cause "personally." The word "personally" does not appear in the
corresponding provisions of our previous Constitutions. This emphasis shows the
present Constitution's intent to place a greater degree of responsibility upon trial
judges than that imposed under the previous Charters.

While affirming Soliven, People vs. Inting  11 elaborated on what "determination of


probable cause" entails, differentiating the judge's object or goal from that of the
prosecutor's.

First, the determination of probable cause is a function of the Judge.


It is not for the Provincial Fiscal or Prosecutor nor for the Election
Supervisor to ascertain. Only the Judge and the Judge alone makes
this determination.

Second, the preliminary inquiry made by a Prosecutor does not bind


the Judge. It merely assists him to make the determination of
probable cause. The Judge does not have to follow what the
Prosecutor presents to him. By itself, the Prosecutor's certification of
probable cause is ineffectual. It is the report, the affidavits the
transcripts of stenographic notes (if any), and all other supporting
documents behind the Prosecutor's certification which are material in
assisting the Judge to make his determination.

And third, Judges and Prosecutors alike should distinguish the


preliminary inquiry which determines probable cause for the issuance
of a warrant of arrest from the preliminary investigation proper which
ascertains whether the offender should be held for trial or released.
Even if the two inquiries are conducted in the course of one and the
same proceeding, there should be no confusion about the objectives.
The determination of probable cause for the warrant of arrest is made
by the Judge. The preliminary investigation proper — whether or not
there is reasonable ground to believe that the accused is guilty of the
offense charged and, therefore, whether or not he should be subjected
to the expense, rigors and embarrassment of trial — is the function of
the Prosecutor. 12

And clarifying the statement in People vs. Delgado 13 — that the "trial court may
rely on the resolution of the COMELEC to file the information, by the same token
that it may rely on the certification made by the prosecutor who conducted the
preliminary investigation, in the issuance of the warrant of arrest" — this Court
underscored in Lim Sr. vs. Felix  14 that "[r]eliance on the COMELEC resolution or
the Prosecutor's certification presupposes that the records of either the COMELEC
or the Prosecutor have been submitted to the Judge and he relies on the
certification or resolution because the records of the investigation sustain the
recommendation." We added, "The warrant issues not on the strength of the
certification standing along but because of the records which sustain it." Summing
up, the Court said:

We reiterate the ruling in Soliven vs. Makasiar that the Judge does not


have to personally examine the complainant and his witnesses. The
Prosecutor can perform the same functions as a commissioner for the
taking of the evidence. However, there should be a
report and necessary documents supporting the Fiscal's bare
certification. All of these should be before the Judge.

The extent of the Judge's personal examination of the report and its
annexes depends on the circumstances of each case. We cannot
determine beforehand how cursory or exhaustive the Judge's
examination should be. The Judge has to exercise sound discretion
for, after all, the personal determination is vested in the Judge by the
Constitution. It can be as brief or as detailed as the circumstances of
each case require. To be sure, the Judge must go beyond the
Prosecutor's certification and investigation report whenever
necessary. He should call for [the] complainant and [the] witnesses
themselves to answer the court's probing questions when the
circumstances of the case so require. 15 [emphasis supplied]

The above rulings in Soliven, Inting and Lim Sr. were iterated in Allado


vs. Diokno 16 where we explained again what probable cause means. Probable
cause for the issuance of a warrant of arrest is the existence of such facts and
circumstances that would lead a reasonably discreet and prudent person to believe
that an offense has been committed by the person sought to be arrested. 17 Hence,
the judge, before issuing a warrant of arrest, "must satisfy himself that based on the
evidence submitted there is sufficient proof that a crime has been committed and
that the person to be arrested is probably guilty thereof." 18 At this stage of the
criminal proceeding, the judge is not yet tasked to review in detail the evidence
submitted during the preliminary investigation. It is sufficient that he personally
evaluates such evidence in determining probable cause. 19 In Webb vs. De
Leon, 20 we stressed that the judge merely determines the probability, not the
certainty, of guilt of the accused and, in doing so, he need not conduct a de
novo hearing. He simply personally reviews the prosecutor's initial determination
finding probable cause to see if it is supported by substantial evidence.

In the recent case of Roberts Jr. vs. Court of Appeals, 21 this Court's application of


the dictum laid down in Soliven — affirmed and fortified in Inting, Lim
Sr., Allado and Webb — should lay to rest the issue raised in the instant petitions.
In Robets Jr., this Court, through Mr. Justice Hilario G. Davide, Jr., set aside the
order of the respondent judge directing inter alia the issuance of warrants of arrest
against the accused, reasoning that said judge did not personally determine the
existence of probable cause, since he had "only the information, amended
information, and Joint Resolution as bases thereof. He did not have the records or
evidence supporting the prosecutor's finding of probable cause."

In like manner, herein Respondent Sandiganbayan had only the information filed
by the Office of the Ombudsman, the thirteen-page resolution of the investigating
officer and the three-page memorandum of the prosecution officer, when it issued
the warrant of arrest against the petitioners. The latter two documents/reports even
had dissimilar recommendations — the first indicting only Petitioner Narciso, the
second including Petitioner Ho. This alone should have prompted the public
respondent to verify, in the records and other documents submitted by the parties
during the preliminary investigation, whether there was sufficient evidence to
sustain the Ombudsman's action charging both petitioners with violation of Sec.
3(e) of Anti-Graft law. But in its initial justification of the issuance of the warrant,
the Sandiganbayan simply said:

JUSTICE ESCAREAL:

xxx xxx xxx


But in this particular case we believe there is prima facie case based
on our examination of the resolution because we believe, we think the
Ombudsman will not approve a resolution just like that, without
evidence to back it up. 22

In attempting to further justify its challenged action, the public respondent


explained in its assailed Resolution.

In the instant case, there are attached to the information, two (2)
Memorandum/Resolution (sic) emanating from the Offices of the
Ombudsman and the Special Prosecutor (Pp. 4-6, 7-19, respectively,
Record) which clearly and indubitably established, firstly, the
conduct of a due and proper preliminary investigation, secondly, the
approval by proper officials clothed with statutory authority;
and thirdly, the determination and ascertainment of probable caused
based on the documentary evidence submitted by the complainant
(Anti-Graft League of the Philippines), foremost among which is the
Contract of Affreightment entered into between National Steel
Corporation (NSC), and National Marine Corporation (NMC) and the
COA-NSC audit report, together with the counter-affidavits of
accused Rolando Narciso and NMC officials, among whom is
accused-movant. Outlined in detail in the aforesaid Resolution of
Titus P. Labrador, Graft Investigation Officer II, which was reviewed
by Attys. Romeo I. Tan and Arturo Mojica, Director, Community
Coordination Bureau and Assistant Ombudsman, PACPO,
[respectively,] are the facts leading to the questioned transaction
between NSC and NMC, together with an evaluation of the propriety
and legality of the bidding process involved therein and which
revealed that there were supposed non-compliance with proper
bidding procedures. GIO Labrador's findings and recommendations,
extensively set out in his thirteen-page resolution, is complemented
by the three-page Memorandum of Special Prosecution Officer II
Leonardo P. Tamayo, both of which meticulously delved into the
merits and demerits of the evidence presented by the complainant and
accused-respondents and which resulted in their respective
recommendation which led the Honorable Conrado M. Vasquez to
approve the recommendations of Deputy Special

Prosecutor Jose de G. Ferrer and Special Prosecutor Aniano A.


Desierto for the filling of the information in the case at bar.

xxx xxx xxx

Considering, therefore, that this Court did not rely solely on


the certification appearing in the information in this case in the
determination of whether probable cause exists to justify the issuance
of the warrant of arrest but also on the basis predominantly shown by
the facts and evidence appearing in the resolution/memorandum of
responsible investigators/prosecutors, then the recall of the warrant of
arrest, or the reconsideration sought for, cannot be granted. More so,
when the information, as filed, clearly shows that it is sufficient in
form and substance based on the facts and evidence adduced by both
parties during the preliminary investigation. To require this Court to
have the entire record of the preliminary investigation to be produced
before it, including the evidence submitted by the complainant and
the accused-respondents, would appear to be an exercise in futility. 23

In light of the aforecited decisions of this Court, such justification cannot be


upheld. Lest we be too repetitive, we only wish to emphasize three vital matters
once more: First, as held in Inting, the determination of probable cause by the
prosecutor is for a purpose different from that which is to be made by the judge.
Whether there is reasonable ground to believe that the accused is guilty of the
offense charged and should be held for trial is what the prosecutor passes upon.
The judge, on the other hand, determines whether a warrant of arrest should be
issued against the accused, i.e. whether there is a necessity for placing him under
immediate custody in order not to frustrate the ends of justice. 24 Thus, even if both
should base their findings on one and the same proceeding or evidence, there
should be no confusion as to their distinct objectives.

Second, since their objectives are different, the judge cannot rely solely on the
report of the prosecutor in finding probable cause to justify the issuance of a
warrant of arrest. Obviously and understandably, the contents of the prosecutor's
report will support his own conclusion that there is reason to charge the accused of
an offense and hold him for trial. However, the judge must decide independently.
Hence, he must have supporting evidence, other than the prosecutor's bare report,
upon which to legally sustain his own findings on the existence (or nonexistence)
of probable cause to issue an arrest order. This responsibility of determining
personally and independently the existence or nonexistence of probable cause is
lodged in him by no less than the most basic law of the land. Parenthetically, the
prosecutor could ease the burden of the judge and speed up the litigation process
by forwarding to the latter not only the information and his bare resolution finding
probable cause, but also so much of the records and the evidence on hand as to
enable His Honor to make his personal and separate judicial finding on whether to
issue a warrant of arrest. 25

Lastly, it is not required that the complete or entire records of the


case during the preliminary investigation be submitted to and examined by the
judge. 26 We do not intend to unduly burden trial courts by obliging them to
examine the complete records of every case all the time simply for the purpose of
ordering the arrest of an accused. What is required, rather, is that the judge must
have sufficient supporting documents (such as the complaint, affidavits, counter-
affidavits, sworn statements of witnesses or transcripts of stenographic notes, if
any) upon which to make his independent judgment or, at the very least, upon
which to verify the findings of the prosecutor as to the existence of probable cause.
The point is: he cannot rely solely and entirely on the prosecutor's
recommendation, as Respondent Court did in this case. Although the prosecutor
enjoys the legal presumption of regularity in the performance of his official duties
and functions, which in turn gives his report the presumption of accuracy, the
Constitution we repeat, commands the judge to personally determine probable
cause in the issuance of warrants of arrest. This Court has consistently held that a
judge fails in his bounden duty if he relies merely on the certification or the report
of the investigating officer.

True, in Webb vs. De Leon, we found that "the painstaking recital and analysis of
the parties' evidence made in the DOJ Panel Report satisfied both judges that there
[was] probable cause to issue warrants of arrest against petitioners." This statement
may have been wrongly construed by the public respondent to mean that the
narration or description of portions of the evidence in the prosecutor's report may
serve as sufficient basis to make its own independent judgment. What it should
bear in mind, however, is that, aside from the 26-page report of the DOJ panel, the
sworn statements of three witnesses and counter-affidavits of the petitioners
in Webb were also submitted to the trial court, and the latter is presumed to have
reviewed these documents as well, prior to its issuance of the warrants of arrest.

In the instant case, the public respondent relied fully and completely upon the
resolution of the graft investigation officer and the memorandum of the reviewing
prosecutor, attached to the information filed before it, and its conjecture that the
Ombudsman would not have approved their recommendation without supporting
evidence. It had no other documents from either the complainant (the Anti-Graft
League of the Philippines) or the People from which to sustain its own conclusion
that probable cause exists. Clearly and ineluctably, Respondent Court's findings of
"the conduct of a due and proper preliminary investigation" and "the approval by
the proper officials clothed with statutory authority" are not equivalent to
the independent and personal responsibility required by the Constitution and
settled jurisprudence. At least some of the documentary evidence mentioned
(Contract of Affreightment between National Steel Corporation and National
Marine Corporation, the COA-NSC audit report, and counter-affidavits of Rolando
Narciso and NMC officials), upon which the investigating officials of the
Ombudsman reportedly ascertained the existence of probable cause, should have
been physically present before the public respondent for its examination, to enable
it to determine on its own whether there is substantial evidence to support the
finding of probable cause. But is stubbornly stood pat on its position that it had
essentially complied with its responsibility. Indisputably, however, the procedure it
undertook contravenes the Constitution and settled jurisprudence. Respondent
Court palpably committed grave abuse of discretion in ipso facto issuing the
challenged warrant of arrest on the sole basis of the prosecutor's findings and
recommendation, and without determining on its own the issue of probable cause
based on evidence other than such bare findings and recommendation.

WHEREFORE, the petitions are GRANTED and the assailed Resolution is SET
ASIDE. The warrant issued by the Sandiganbayan (Second Division) on May 20,
1992 in Case No. 17674 for the arrest of Petitioners Doris Teresa Ho and Rolando
Narciso is hereby declared NULL AND VOID.

SO ORDERED.

Roberts v. CA, G.R. No. 113930, March 5, 1996 (Read also the dissent of J. Puno)

G.R. No. 113930 March 5, 1996

PAUL G. ROBERTS, JR., RODOLFO C. SALAZAR, LUIS LORENZO, SR.,


LUIS LORENZO, JR., AMAURY R. GUTIERREZ, BAYANI N. FABIC,
JOSE YULO, JR., ESTEBAN B. PALANNUAYAN, and WONG FONG
FUI, petitioners,
vs.
THE COURT OF APPEALS, THE HON. MAXIMIANO ASUNCION, in his
capacity as the Presiding Judge of the Regional Trial Court, Quezon City,
Branch 104, HON. APOLINARIO G. EXEVEA, HON. HENRICK F.
GINGOYON, and HON. PHILIP A. AGUINALDO, in their capacities as
Members of the Department of Justice "349" Committee, and the CITY
PROSECUTOR OF QUEZON CITY, respondents.

J. ROBERT DELGADO, petitioner-Intervenor.

DAVIDE, JR., J.:p

We are urged in this petition to set aside (a) the decision of the Court of Appeals of
28 September 1993 in CA-G.R. SP No. 31226,1 which dismissed the petition
therein on the ground that it has been "mooted with the release by the Department
of Justice of its decision . . . dismissing petitioners' petition for review"; (b) the
resolution of the said court of 9 February 19942 denying the petitioners' motion to
reconsider the decision; (c) the order of 17 May 1993 3 of respondent Judge
Maximiano C. Asuncion of Branch 104 of the Regional Trial Court (RTC) of
Quezon City in Criminal Case No. Q-93-43198 denying petitioners' motion to
suspend proceedings and to hold in abeyance the issuance of the warrants of arrest
and the public prosecutor's motion to defer arraignment; and (d) the resolutions of
23 July 1993 and 3 February 19944 of the Department of Justice (DOJ) dismissing
petitioners' petition for the review of the Joint Resolution of the Assistant City
Prosecutor of Quezon City and denying the motion to reconsider the dismissal,
respectively.

The petitioners rely on the following grounds for the grant of the reliefs prayed for
in this petition:

Respondent Judge acted with grave abuse of discretion when he


ordered the arrest of the petitioners without examining the record of
the preliminary investigation and in determining for himself on the
basis thereof the existence of probable cause.

II

The Department of Justice "349" Committee acted with grave abuse


of discretion when it refused to review the City Prosecutor's Joint
Resolution and dismissed petitioner's appeal therefrom.

III

The Court of Appeals acted with grave abuse of discretion when it


upheld the subject order directing the issuance of the warrants of
arrest without assessing for itself whether based on such records there
is probable cause against petitioners.

IV

The facts on record do not establish prima facie probable cause and
Criminal Case No. Q-93-43198 should have been dismissed. 5

The antecedents of this petition are not disputed.

Several thousand holders6 of "349" Pepsi crowns in connection with the Pepsi Cola
Products Phils., Inc.'s (PEPSI's) Number Fever Promotion7 filed with the Office of
the City Prosecutor of Quezon City complaints against the petitioner's in their
respective capacities as Presidents or Chief Executive Officers, Chairman of the
Board, Vice-Chairman of the Board, and Directors of PEPSI, and also against other
officials of PEPSI. The complaints respectively accuse the petitioners and the other
PEPSI officials of the following crimes: (a) estafa; (b) violation of R.A. No. 7394,
otherwise known as the Consumer Act of the Philippines; (c) violation of E.O. No.
913;8 and (d) violation of Act No. 2333, entitled "An Act Relative to Untrue,
Deceptive and Misleading Advertisements," as amended by Act No. 3740.9
After appropriate proceedings, the investigating prosecutor, Ramon M. Gerona,
released on 23 March 1993 a Joint Resolution 10 where he recommended the filing
of an information against the petitioners and others for the violation of Article 318
of the Revised Penal Code and the dismissal of the complaints for the violation of
Article 315, 2(d) of the Revised Penal Code; R.A. No. 7394; Act No. 2333, as
amended by Act No. 3740; and E.O. No. 913. The dispositive portion thereof reads
as follows:

In view of all the foregoing, it is recommended that:

1. The attached information be filed against respondents Paul G.


Roberts, Jr., Rodolfo C. Salazar, Rosemarie R. Vera, Luis F. Lorenzo,
Sr., Luis P. Lorenzo, Jr., J. Roberto Delgado, Amaury R. Gutierrez,
Bayani N. Fabic, Jose Yulo, Jr., Esteban B. Pacannuayan, Jr., Wong
Fong Fui, Quintin J. Gomez, Jr. and Chito V. Gutierrez for estafa
under Article 318, Revised Penal Code, while the complaint for
violation of Article 315, 2(d), Revised Penal Code against same
respondents Juanito R. Ignacio, R. Sobong, R.O. Sinsuan, M.P.
Zarsadias, L.G. Dabao, Jr., R.L. Domingo, N.N. Bacsal, Jesus M.
Manalastas, Janette P. Pio de Roda, Joaquin W. Sampaico, Winefreda
O. Madarang, Jack Gravey, Les G. Ham, Corazon Pineda, Edward S.
Serapio, Alex O. Caballes, Sandy Sytangco, Jorge W. Drysdale,
Richard Blossom, Pablo de Borja, Edmundo L. Tan, Joseph T.
Cohen, Delfin Dator, Zosimo B. San Juan, Joaquin Franco, Primitivo
S. Javier, Jr., Luisito Guevarra, Asif H. Adil, Eugenio Muniosguren,
James Ditkoff and Timothy Lane be dismissed;

2. The complaints against all respondents for violation of R.A. 7394


otherwise known as the Consumer Act of the Philippines and
violation of Act 2333 as amended by Act 3740 and E.O. 913 be also
dismissed for insufficiency of evidence, and

3. I.S. Nos. 92-7833; 92-8710 and 92-P-1065 involving Crowns Nos.


173; 401; and 117, 425, 703 and 373, respectively, alleged to be
likewise winning ones be further investigated to afford respondents a
chance to submit their counter-evidence. 11

On 6 April 1993, City Prosecutor Candido V. Rivera approved the


recommendation with the modification that Rosemarie Vera, Quintin Gomez, Jr.,
and Chito Gonzales be excluded from the charge on the ground of insufficiency of
evidence.12

The information for estafa attached to the Joint Resolution was approved (on 7
April 1993) by Ismael P. Casabar, Chief of the Prosecution Division, upon
authority of the City Prosecutor of Quezon City, and was filed with the RTC of
Quezon City on 12 April 1993. It was docketed as Criminal Case No. Q-93-
43198.13 The information reads as follows:

The undersigned 1st Assistant City Prosecutor accuses PAUL G.


ROBERTS, JR. RODOLFO C. SALAZAR, LUIS F. LORENZO,
SR., LUIS P. LORENZO, JR., J. ROBERTO DELGADO, AMAURY
R. GUTIERREZ, BAYANI N. FABIC, JOSE YULO, JR.,
ESTEBAN B. PACANNUAYAN, JR. and WONG FONG FUI, of
the crime of ESTAFA, committed as follows:

That in the month of February, 1992, in Quezon City, Philippines and


for sometime prior and subsequent thereto, the above-named
accused —

Paul G. Roberts, Jr. ) being then the Presidents

Rodolfo G. Salazar ) and Executive Officers

Luis F. Lorenzo, Sr. ) being then the Chairman

of the Board of Directors

Luis P. Lorenzo, Jr. ) being then the Vice

Chairman of the Board

J. Roberto Delgado )

Amaury R. Gutierrez ) being then Members of

Bayani N. Fabic ) the Board

Jose Yulo, Jr. )

Esteban B. Pacannuayan, )

Jr. and

Wong Fong Fui )

OF THE PEPSI COLA PRODUCTS PHILIPPINES, INC.,


CONSPIRING with one another, with intent of gain, by means of
deceit, fraudulent acts or false pretenses, executed prior to or
simultaneously with the commission of the fraud, did then and there
willfully, unlawfully and feloniously defraud the private
complainants whose names with their prizes claimed appear in the
attached lists marked as Annexes "A" to "A-46"; "B" to "-33"; "C" to
"C-281"; "D" to "D-238"; "E" to "E-30" and "F" to "F-244" in the
following manner: on the date and in the place aforementioned, said
accused pursuant to their conspiracy, launched the Pepsi Cola
Products Philippines, Inc. "Number Fever Promotion" from February
17 to May 8, 1992 later extended to May 11-June 12, 1992 and
announced and advertised in the media that "all holders of crowns
and/or caps of Pepsi, Mirinda, Mountain Dew and Seven-up bearing
the winning 3-digit number will win the full amount of the prize
printed on the crowns/caps which are marked with a seven-digit
security code as a measure against tampering or faking of crowns and
each and every number has its own unique matching security code",
enticing the public to buy Pepsi softdrinks with aforestated alluring
and attractive advertisements to become millionaires, and by virtue of
such representations made by the accused, the said complainants
bought Pepsi softdrinks, but, the said accused after their TV
announcement on May 25, 1992 that the winning number for the next
day was "349", in violation of their aforecited mechanics, refused as
they still refuse to redeem/pay the said Pepsi crowns and/or caps
presented to them by the complainants, who, among others, were able
to buy Pepsi softdrinks with crowns/caps bearing number "349" with
security codes L-2560-FQ and L-3560-FQ, despite repeated demands
made by the complainants, to their damage and prejudice to the extent
of the amount of the prizes respectively due them from their winning
"349" crowns/caps, together with such amounts they spent in going to
and from the Office of Pepsi to claim their prizes and such other
amounts used in buying Pepsi softdrinks which the complainants
normally would not have done were it not for the false, fraudulent and
deceitful posters of Pepsi Cola Products Philippines, Inc.

CONTRARY TO LAW.

On 14 April 1993, the petitioners filed with the Office of the City Prosecutor a
motion for the reconsideration of the Joint Resolution 14 alleging therein that (a)
there was neither fraud in the Number Fever Promotion nor deviation from or
modification of the promotional rules approved by the Department of Trade and
Industry (DTI), for from the start of the promotion, it had always been clearly
explained to the public that for one to be entitled to the cash prize his crown must
bear both the winning number and the correct security code as they appear in the
DTI list; (b) the complainants failed to allege, much less prove with prima
facie evidence, the specific overt criminal acts or omissions purportedly committed
by each of the petitioners; (c) the compromise agreement entered into by PEPSI is
not an admission of guilt; and (d) the evidence establishes that the promo was
carried out with utmost good faith and without malicious intent.
On 15 April 1993, the petitioners filed with the DOJ a Petition for
Review15 wherein, for the same grounds adduced in the aforementioned motion for
reconsideration, they prayed that the Joint Resolution be reversed and the
complaints dismissed. They further stated that the approval of the Joint Resolution
by the City Prosecutor was not the result of a careful scrutiny and independent
evaluation of the relevant facts and the applicable law but of the grave threats,
intimidation, and actual violence which the complainants had inflicted on him and
his assistant prosecutors.

On that same date, the petitioners filed in Criminal Case No. Q-93-43198 Motions
to Suspend Proceedings and to Hold in Abeyance Issuance of Warrants of Arrest
on the ground that they had filed the aforesaid Petition for Review. 16

On 21 April 1993, acting on the Petition for Review, Chief State Prosecutor Zenon
L. de Guia issued a 1st Indorsement,17 directing the City Prosecutor of Quezon
City to inform the DOJ whether the petitioners have already been arraigned, and if
not, to move in court for the deferment of further proceedings in the case and to
elevate to the DOJ the entire records of the case, for the case is being treated as an
exception pursuant to Section 4 of Department Circular No. 7 dated 25 January
1990.

On 22 April 1993, Criminal Case No. Q-93-41398 was raffle to Branch 104 of the
RTC of Quezon City.18

In the morning of 27 April 1993, private prosecutor Julio Contreras filed an Ex-
Parte Motion for Issuance of Warrants of Arrest. 19

In the afternoon of that same day, petitioner Paul Roberts, Jr., filed a Supplemental
Urgent Motion to Hold in Abeyance Issuance of Warrant of Arrest and to Suspend
Proceedings.20 He stressed that the DOJ had taken cognizance of the Petition for
Review by directing the City Prosecutor to elevate the records of I.S. No. P-4401
and its related cases and asserted that the petition for review was an essential part
of the petitioners' right to a preliminary investigation.

The next day, respondent Judge Asuncion, Presiding Judge of Branch 104 of the
RTC of Quezon City, issued an order advising the parties that his court would "be
guided by the doctrine laid down by the Supreme Court in the case of Crespo
vs. Mogul, 151 SCRA 462 and not by the resolution of the Department of Justice
on the petition for review undertaken by the accused." 21

On 30 April 1993, Assistant City Prosecutor Tirso M. Gavero filed with the trial
court a Motion to Defer Arraignment wherein he also prayed that "further
proceedings be held in abeyance pending final disposition by the Department of
Justice."22
On 4 May 1993, Gavero filed an Amended Information, 23 accompanied by a
corresponding motion 24 to admit it. The amendments merely consist in the
statement that the complainants therein were only "among others" who were
defrauded by the accused and that the damage or prejudice caused amounted "to
several billions of pesos, representing the amounts due them from their winning
'349' crowns/caps." The trial court admitted the amended information on the same
date.25

Later, the attorneys for the different private complainants filed, respectively, an
Opposition to Motion to Defer Arraignment,26 and Objection and Opposition to
Motion to Suspend Proceedings and to Hold in Abeyance the Issuance of Warrants
of Arrest.27

On 14 May 1993, the petitioners filed a Memorandum in Support of their Motion


to Suspend Proceedings and to Hold in Abeyance the Issuance of the Warrants of
Arrest.28

On 17 May 1993, respondent Judge Asuncion issued the challenged order (1)
denying the petitioners' Motion to Suspend Proceedings and to Hold in Abeyance
Issuance of Warrants of Arrest and the public prosecutor's Motion to Defer
Arraignment and (2) directing the issuance of the warrants of arrest "after June
1993" and setting the arraignment on 28 June 1993.29 Pertinent portions of the
order read as follows:

In the Motion filed by the accused, it is alleged that on April 15,


1993, they filed a petition for review seeking the reversal of the
resolution of City Prosecutor of Quezon City approving the filing of
the case against the accused, claiming that:

1. The resolution constituting [sic] force and duress;

2. There was no fraud or deceit therefore there can be no


estafa;

3. No criminal overt acts by respondents were proved;

4. Pepsi nor the accused herein made no admission of


guilt before the Department of Trade and Industry;

5. The evidence presented clearly showed no malicious


intent on the part of the accused.

Trial Prosecutor Tirso M. Gavero in his Motion to Defer Arraignment


averred that there is a pending petition for review with the
Department of Justice filed by the accused and the Office of the City
Prosecutor was directed, among other things, to cause for the
deferment of further proceedings pending final disposition of said
Petition by the Department of Justice.

The motions filed by the accused and the Trial Prosecutor are hereby
DENIED.

This case is already pending in this Court for trial. To follow


whatever opinion the Secretary of Justice may have on the matter
would undermine the independence and integrity of this Court. This
Court is still capable of administering justice.

The Supreme Court in the case of Crespo vs. Mogul (SCRA 151, pp.


471-472) stated as follows:

In order therefor to avoid such a situation whereby the


opinion of the Secretary of Justice who reviewed the
action of the fiscal may be disregarded by the trial court,
the Secretary of Justice should, as far as practicable,
refrain from entertaining a petition for review or appeal
from the action of the fiscal, when the complaint or
information has already been filed in Court. The matter
should be left entirely for the determination of the
Court.

WHEREFORE, let warrant of arrest be issued after June 21, 1993,


and arraignment be set on June 28, 1993, at 9:30 in the morning.

On 7 June 1993, the petitioners filed with the Court of Appeals a special civil
action for certiorari and prohibition with application for a temporary restraining
order,30 which was docketed as CA-G.R. SP No. 31226. They contended therein
that respondent Judge Asuncion had acted without or in excess of jurisdiction or
with grave abuse of discretion in issuing the aforementioned order of 17 May 1993
because

I. RESPONDENT JUDGE FAILED TO EXAMINE THE RECORD


OF PRELIMINARY INVESTIGATION BEFORE ORDERING THE
ARREST OF PETITIONERS.

II. THERE IS NO PROBABLE CAUSE TO HOLD PETITIONERS


CRIMINALLY LIABLE FOR ESTAFA, OTHER DECEITS, OR
ANY OTHER OFFENSE.
III. THE PROCEEDINGS BELOW SHOULD HAVE BEEN
SUSPENDED TO AWAIT THE SECRETARY OF JUSTICE'S
RESOLUTION OF PETITIONERS' APPEAL, AND

IV. THERE IS NO OTHER PLAIN, SPEEDY AND ADEQUATE


REMEDY IN THE ORDINARY COURSE OF LAW.

On 15 June 1993, the Court of Appeals issued a temporary restraining order to


maintain the status quo.31 In view thereof; respondent Judge Asuncion issued an
order on 28 June 199332 postponing indefinitely the arraignment of the petitioners
which was earlier scheduled on that date.

On 28 June 1993, the Court of Appeals heard the petitioners' application for a writ
of preliminary injunction, granted the motion for leave to intervene filed by J.
Roberto Delgado, and directed the Branch Clerk of Court of the RTC of Quezon
City to elevate the original records of Criminal Case No. 4-93-43198. 33

Upon receipt of the original records of the criminal case, the Court of Appeals
found that a copy of the Joint Resolution had in fact been forwarded to, and
received by, the trial court on 22 April 1993, which fact belied the petitioners'
claim that the respondent Judge had not the slightest basis at all for determining
probable cause when he ordered the issuance of warrants of arrest. It ruled that the
Joint Resolution "was sufficient in itself to have been relied upon by respondent
Judge in convincing himself that probable cause indeed exists for the purpose of
issuing the corresponding warrants of arrest"; and that the "mere silence of the
records or the absence of any express declaration" in the questioned order as to the
basis of such finding does not give rise to an adverse inference, for the respondent
Judge enjoys in his favor the presumption of regularity in the performance of his
official duty. The Court of Appeals then issued a resolution 34 denying the
application for a writ of preliminary injunction.

On 8 June 1993, the petitioners filed a motion to reconsider 35 the aforesaid


resolution. The Court of Appeals required the respondents therein to comment on
the said motion.36

On 3 August 1993, the counsel for the private complainants filed in CA-G.R. SP
No. 31226 a Manifestation3 7 informing the court that the petitioners' petition for
review filed with the DOJ was dismissed in a resolution dated 23 July 1993. A
copy 38 of the resolution was attached to the Manifestation.

On 21 September 1993, the public respondents filed in CA-G.R. SP No. 31226 a


motion to dismiss the petition39 on the ground that it has become moot and
academic in view of the dismissal by the DOJ of the petitioners' petition to review
the Joint Resolution. The dismissal by the DOJ is founded on the following
exposition:
You questioned the said order of the RTC before the Court of
Appeals and prayed for the issuance of a writ of preliminary
injunction to restrain the Trial Judge from issuing any warrant of
arrest and from proceeding with the arraignment of the accused. The
appellate court in a resolution dated July 1, 1993, denied your
petition.

In view of the said developments, it would be an exercise in futility to


continue reviewing the instant cases for any further action on the part
of the Department would depend on the sound discretion of the Trial
Court. The denial by the said court of the motion to defer arraignment
filed at our instance was clearly an exercise of its discretion. With the
issuance of the order dated May 17, 1993, Trial Court was in effect
sending a signal to this Department that "the determination of the case
is within its exclusive jurisdiction and competence." The rule is that ".
. . once a complaint or information is filed in Court, any disposition
of the case as to dismissal or the conviction or acquittal of the
accused rests in the sound discretion of the Court. Although the fiscal
retains the direction and control of the prosecution of criminal cases
even while the case is already in Court, he cannot impose his opinion
on the trial court. The court is the best and sole judge on what to do
with the case before it. . . ." (Crespo vs. Mogul, 151 SCRA 462). 40

On 28 September 1993, the Court of Appeals promulgated a decision 41 dismissing


the petition because it had been "mooted with the release by the Department of
Justice of its decision . . . dismissing petitioners' petition for review by inerrantly
upholding the criminal court's exclusive and unsupplantable authority to control
the entire course of the case brought against petitioners, reiterating with approval
the dictum laid down in the 'Crespo' case."

The petitioners filed a motion to reconsider the DOJ's dismissal of the petition
citing therein its resolutions in other similar cases which were favorable to the
petitioners and adverse to other "349" Pepsi crowns holders.

In its resolution of 3 February 1994, the DOJ, through its "349" Committee, denied
the motion and stated: "The instant petition is different from the other petitions
resolved by this Department in similar cases from
the provinces. In the latter petitions, the complaints against herein respondents
[sic]42 were dismissed inasmuch as the informations have not yet been filed or even
if already filed in court, the proceedings have been suspended by the courts to
await the outcome of the appeal pending with this Department." 43

The petitioners likewise filed a motion to reconsider 44 the aforesaid Court of


Appeals' decision, which the said court denied in its resolution 45 of 9 February
1994. Hence, the instant petition.
The First Division of this Court denied due course to this petition in its resolution
of 19 September 1994.46

On 7 October 1994, the petitioners filed a motion for the


reconsideration 4 7 of the aforesaid resolution. Acting thereon, the First Division
required the respondents to comment thereon.

Later, the petitioners filed a supplemental motion for reconsideration 48 and a


motion to refer this case to the Court en banc. 49 In its resolution of 14 November
1994,50 the First Division granted the latter motion and required the respondents to
comment on the supplemental motion for reconsideration.

In the resolution of 24 November 1994, the Court en banc accepted the referral.

On 10 October 1995, after deliberating on the motion for reconsideration and the
subsequent pleadings in relation thereto, the Court en banc granted the motion for
reconsideration; reconsidered and set aside the resolution of 19 September 1994;
and reinstated the petition. It then considered the case submitted for decision,
"since the parties have exhaustively discussed the issues in their pleadings, the
original records of Criminal Case No. Q-93-43198 and of CA-G.R. SP No. 31226
had been elevated to this Court, and both the petitioners and the Office of the
Solicitor General pray, in effect, that this Court resolve the issue of probable cause
on the basis thereof."

The pleadings of the parties suggest for this Court's resolution the following key
issues:

1. Whether public respondent Judge Asuncion committed grave abuse


of discretion in denying, on the basis of Crespo vs. Mogul, the
motions to suspend proceedings and hold in abeyance the issuance of
warrants of arrest and to defer arraignment until after the petition for
review filed with the DOJ shall have been resolved.

2. Whether public respondent Judge Asuncion committed grave abuse


of discretion in ordering the issuance of warrants of arrest without
examining the records of the preliminary investigation.

3. Whether the DOJ, through its "349" Committee, gravely abused its
discretion in dismissing the petition for review on the following
bases: (a) the resolution of public respondent Court of Appeals
denying the application for a writ of preliminary injunction and (b) of
public respondent Asuncion's denial of the abovementioned motions.

4. Whether public respondent Court of Appeals committed grave


abuse of discretion (a) in denying the motion for a writ of preliminary
injunction solely on the ground that public respondent Asuncion had
already before him the Joint Resolution of the investigating
prosecutor when he ordered the issuance of the warrants of arrest, and
(b) in ultimately dismissing the petition on the ground of mootness
since the DOJ had dismissed the petition for review.

5. Whether this Court may determine in this proceedings the


existence of probable cause either for the issuance of warrants of
arrest against the petitioners or for their prosecution for the crime of
estafa.

We resolve the first four issues in the affirmative and the fifth, in the negative.

I.

There is nothing in Crespo vs. Mogul51 which bars the DOJ from taking cognizance
of an appeal, by way 'of a petition for review, by an accused in a criminal case
from an unfavorable ruling of the investigating prosecutor. It merely advised the
DOJ to, "as far as practicable, refrain from entertaining a petition for review or
appeal from the action of the fiscal, when the complaint or information has already
been filed in Court." More specifically, it stated:

In order therefore to avoid such a situation whereby the opinion of the


Secretary of Justice who reviewed the action of the fiscal may be
disregarded by the trial court, the Secretary of Justice should, as far as
practicable, refrain from entertaining a petition for review or appeal
from the action of the fiscal, when the complaint or information has
already been filed in Court. The matter should, be left entirely for the
determination of the Court.52

In Marcelo vs. Court of Appeals,53 this Court explicitly declared:

Nothing in the said ruling forecloses the power or authority of the


Secretary of Justice to review resolutions of his subordinates in
criminal cases. The Secretary of Justice is only enjoined to refrain as
far as practicable from entertaining a petition for review or appeal
from the action of the prosecutor once a complaint or information is
filed in court. In any case, the grant of a motion to dismiss, which the
prosecution may file after the Secretary of Justice reverses an
appealed resolution, is subject to the discretion of the court.

Crespo could not have intended otherwise without doing violence to, or repealing,
the last paragraph of Section 4, Rule 112 of the Rules of Court 54 which recognizes
the authority of the Secretary of Justice to reverse the resolution of the provincial
or city prosecutor or chief state prosecutor upon petition by a proper party.
Pursuant to the said provision, the Secretary of Justice had promulgated the rules
on appeals from resolutions in preliminary investigation. At the time the petitioners
filed their petition for the review of the Joint Resolution of the investigating
prosecutor, the governing rule was Circular No. 7, dated 25 January 1990. Section
2 thereof provided that only resolutions dismissing a criminal complaint may be
appealed to the Secretary of Justice. Its Section 4, 55 however, provided an
exception, thus allowing, upon a showing of manifest error or grave abuse of
discretion, appeals from resolutions finding probable cause, provided that the
accused has not been arraigned.

The DOJ gave due course to the petitioners' petition for review as an exception
pursuant to Section 4 of Circular No. 7.

Meanwhile, the DOJ promulgated on 30 June 1993 Department Order No.


22356 which superseded Circular No. 7. This Order, however, retained the
provisions of Section 1 of the Circular on appealable cases and Section 4 on the
non-appealable cases and the exceptions thereto.

There is nothing in Department Order No. 223 which would warrant a recall of the
previous action of the DOJ giving due course to the petitioners' petition for review.
But whether the DOJ would affirm or reverse the challenged Joint Resolution is
still a matter of guesswork. Accordingly, it was premature for respondent Judge
Asuncion to deny the motions to suspend proceedings and to defer arraignment on
the following grounds:

This case is already pending in this Court for trial. To follow


whatever opinion the Secretary of Justice may have on the matter
would undermine the independence and integrity of this Court. This
Court is still capable of administering justice.

The real and ultimate test of the independence and integrity of his court is not the
filing of the aforementioned motions at that stage of the proceedings but the filing
of a motion to dismiss or to withdraw the information on the basis of a resolution
of the petition for review reversing the Joint Resolution of the investigating
prosecutor. Before that time, the following pronouncement in Crespo did not yet
truly become relevant or applicable:

The rule therefore in this jurisdiction is that once a complaint or


information is filed in Court any disposition of the case as its
dismissal or the conviction or acquittal of the accused rests in the
sound discretion of the court. Although the fiscal retains the direction
and control of the prosecution of criminal cases even while the case is
already in court he cannot impose his opinion on the trial court. The
court is the best and sole judge on what to do with the case before it.
The determination of the case is within its exclusive jurisdiction and
competence. A motion to dismiss the case filed by the fiscal should
be addressed to the Court who has the option to grant or deny the
same. It does not matter if this is done before or after the arraignment
of the accused or that the motion was filed after a reinvestigation or
upon instructions of the Secretary of Justice who reviewed the
records of the investigation.57

However, once a motion to dismiss or withdraw the information is filed the


trial judge may grant or deny it, not out of subservience to the Secretary of
Justice, but in faithful exercise of judicial prerogative. This Court
pertinently stated so in Martinez vs. Court of Appeals:58

Whether to approve or disapprove the stand taken by the prosecution


is not the exercise of discretion required in cases like this. The trial
judge must himself be convinced that there was indeed no sufficient
evidence against the accused, and this conclusion can be arrived at
only after an assessment of the evidence in the possession of the
prosecution. What was imperatively required was the trial judge's
own assessment of such evidence, it not being sufficient for the valid
and proper exercise of judicial discretion merely to accept the
prosecution's word for its supposed insufficiency.

As aptly observed the Office of the Solicitor General, in failing to


make an independent finding of the merits of the case and merely
anchoring the dismissal on the revised position of the prosecution, the
trial judge relinquished the discretion he was duty bound to exercise.
In effect, it was the prosecution, through the Department of Justice
which decided what to do and not the court which was reduced to a
mere rubber stamp in violation of the ruling in Crespo vs. Mogul.

II.

Section 2, Article III of the present Constitution provides that no search warrant or
warrant of arrest shall issue except upon probable cause to be determined
personally by the judge after examination under oath or affirmation of the
complainant and the witnesses he may produce.

Under existing laws, warrants of arrest may be issued (1) by the Metropolitan Trial
Courts (MeTCs) except those in the National Capital Region, Municipal Trial
Courts (MTCs), and Municipal Circuit Trial Courts (MCTCs) in cases falling
within their exclusive original jurisdiction; 59 in cases covered by the rule on
summary procedure where the accused fails to appear when required; 60 and in cases
filed with them which are cognizable by the Regional Trial Courts (RTCs); 61 and
(2) by the Metropolitan Trial Courts in the National Capital Region (MeTCs-NCR)
and the RTCs in cases filed with them after appropriate preliminary investigations
conducted by officers authorized to do so other than judges of MeTCs, MTCs and
MCTCs.62

As to the first, a warrant can issue only if the judge is satisfied after an examination
in writing and under oath of the complainant and the witnesses, in the form of
searching questions and answers, that a probable cause exists and that there is a
necessity of placing the respondent under immediate custody in order not to
frustrate the ends of justice.

As to the second, this Court held in Soliven vs. Makasiar 63 that the judge is not
required to personally examine the complainant and the witnesses, but

[f]ollowing established doctrine and procedure, he shall: (1)


personally evaluate the report and supporting documents submitted
by the fiscal regarding the existence of probable cause and, on the
basis thereof; issue a warrant of arrest; or (2) if on the basis thereof he
finds no probable cause, he may disregard the fiscal's report and
require the submission of supporting affidavits of witnesses to aid
him in arriving at a conclusion as to the existence of probable cause. 64

Sound policy supports this procedure, "otherwise judges would be unduly


laden with the preliminary examination and investigation of criminal
complaints instead of concentrating on hearing and deciding cases filed
before their courts." It must be emphasized that judges must not rely solely
on the report or resolution of the fiscal (now prosecutor); they must evaluate
the report and the supporting document. In this sense, the aforementioned
requirement has modified paragraph 4(a) of Circular No. 12 issued by this
Court on 30 June 1987 prescribing the Guidelines on Issuance of Warrants
of Arrest under Section 2, Article III of the 1987 Constitution, which
provided in part as follows:

4. In satisfying himself of the existence of a probable cause for the


issuance of a warrant of arrest, the judge, following established
doctrine and procedure, may either:

(a) Rely upon the fiscal's certification of the existence of


probable cause whether or not the case is cognizable
only by the Regional Trial Court and on the basis
thereof, issue a warrant of arrest. . . .

This requirement of evaluation not only of the report or certification of the fiscal
but also of the supporting documents was further explained
in People vs. Inting,65 where this Court specified what the documents may consist
of, viz., "the affidavits, the transcripts of stenographic notes (if any), and all other
supporting documents behind the Prosecutor's certification which are material in
assisting the Judge to make his determination" of probable cause. Thus:

We emphasize the important features of the constitutional mandate


that ". . . no search warrant or warrant of arrest shall issue except
upon probable cause to be determined personally by the judge . . ."
(Article III, Section 2, Constitution).

First, the determination of probable cause is a function of the Judge.


It is not for the Provincial Fiscal or Prosecutor nor the Election
Supervisor to ascertain. Only the Judge and the Judge alone makes
this determination.

Second, the preliminary inquiry made by a Prosecutor does not bind


the Judge. It merely assists him to make the determination of
probable cause. The Judge does not have to follow what the
Prosecutor presents to him. By itself, the Prosecutor's certification of
probable cause is ineffectual. It is the report, the affidavits, the
transcripts of stenographic notes (if any), and all other supporting
documents behind the Prosecutor's certification which are material in
assisting the Judge to make his determination.

In adverting to a statement in People vs. Delgado66 that the judge may rely on the


resolution of the Commission on Elections (COMELEC) to file the information by
the same token that it may rely on the certification made by the prosecutor who
conducted the preliminary investigation in the issuance of the warrant of arrest, this
Court stressed in Lim vs. Felix67 that

Reliance on the COMELEC resolution or the Prosecutor's


certification presupposes that the records of either the COMELEC or
the Prosecutor have been submitted to the Judge and he relies on the
certification or resolution because the records of the investigation
sustain the recommendation. The warrant issues not on the strength of
the certification standing alone but because of the records which
sustain it.

And noting that judges still suffer from the inertia of decisions and practice
under the 1935 and 1973 Constitutions, this Court found it necessary to
restate the rule "in greater detail and hopefully clearer terms." It then
proceeded to do so, thus:

We reiterate the ruling in Soliven vs. Makasiar that the Judge does


not have to personally examine the complainant and his witnesses.
The Prosecutor can perform the same functions as a commissioner for
the taking of the evidence. However, there should be a report and
necessary documents supporting the Fiscal's bare certification. All of
these should be before the Judge.

The extent of the Judge's personal examination of the report and its
annexes depends on the circumstances of each case. We cannot
determine beforehand how cursory or exhaustive the Judge's
examination should be. The Judge has to exercise sound discretion
for, after all, the personal determination is vested in the Judge by the
Constitution. It can be as brief as or detailed as the circumstances of
each case require. To be sure, the Judge must go beyond the
Prosecutor's certification and investigation report whenever
necessary. He should call for the complainant and witnesses
themselves to answer the court's probing questions when the
circumstances of the case so require.

This Court then set aside for being null and void the challenged order of
respondent Judge Felix directing the issuance of the warrants of arrest
against petitioners Lim, et al., solely on the basis of the prosecutor's
certification in the informations that there existed probable cause "without
having before him any other basis for his personal determination of the
existence of a probable cause."

In Allado vs. Diokno,68 this Court also ruled that "before issuing a warrant of


arrest, the judge must satisfy himself that based on the evidence submitted
there is sufficient proof that a crime has been committed and that the person
to be arrested is probably guilty thereof."

In the recent case of Webb vs. De Leon,69 this Court rejected the thesis of the
petitioners of absence of probable cause and sustained the investigating panel's and
the respondent Judge's findings of probable cause. After quoting extensively
from Soliven vs. Makasiar,70 this Court explicitly pointed out:

Clearly then, the Constitution, the Rules of Court, and our case law
repudiate the submission of petitioners that respondent judges should
have conducted "searching examination of witnesses" before issuing
warrants of arrest against them. They also reject petitioners'
contention that a judge must first issue an order of arrest before
issuing a warrant of arrest. There is no law or rule requiring the
issuance of an Order of Arrest prior to a warrant of arrest.

In the case at bar, the DOJ Panel submitted to the trial court its 26-
page report, the two (2) sworn statements of Alfaro and the sworn
statements of Carlos Cristobal and Lolita Birrer as well as the
counter-affidavits of the petitioners. Apparently, the painstaking
recital and analysis of the parties' evidence made in the DOJ Panel
Report satisfied both judges that there is probable cause to issue
warrants of arrest against petitioners. Again, we stress that before
issuing warrants of arrest, judges merely determine personally the
probability, not the certainty of the guilt of an accused. In doing so,
judges do not conduct a de novo hearing to determine the existence of
probable cause. They just personally review the initial determination
of the prosecutor finding a probable cause to see if it is supported by
substantial evidence. The sufficiency of the review process cannot be
measured by merely counting minutes and hours. The fact that it took
the respondent judges a few hours to review and affirm the probable
cause determination of the DOJ Panel does not mean they made no
personal evaluation of the evidence attached to the records of the
case. (emphasis supplied)

The teachings then of Soliven, Inting, Lim, Allado, and Webb reject the proposition


that the investigating prosecutor's certification in an information or his resolution
which is made the basis for the filing of the information, or both, would suffice in
the judicial determination of probable cause for the issuance of a warrant of arrest.
In Webb, this Court assumed that since the respondent Judges had before them not
only the 26-page resolution of the investigating panel but also the affidavits of the
prosecution witnesses and even the counter-affidavits of the respondents, they
(judges) made personal evaluation of the evidence attached to the records of the
case.

Unfortunately, in Criminal Case No. Q-93-43198, nothing accompanied the


information upon its filing on 12 April 1993 with the trial court. As found by the
Court of Appeals in its resolution of 1 July 1993, a copy of the Joint Resolution
was forwarded to, and received by, the trial court only on 22 April 1993. And as
revealed by the certification71 of Branch Clerk of Court Gibson Araula, Jr., no
affidavits of the witnesses, transcripts of stenographic notes of the proceedings
during the preliminary investigation, or other documents submitted in the course
thereof were found in the records of Criminal Case No. Q-93-43198 as of 19 May
1993. Clearly, when respondent Judge Asuncion issued the assailed order of 17
May 1993 directing, among other things, the issuance of warrants of arrest, he had
only the information, amended information, and Joint Resolution as bases thereof.
He did not have the records or evidence supporting the prosecutor's finding of
probable cause. And strangely enough, he made no specific finding of probable
cause; he merely directed the issuance of warrants of arrest "after June 21, 1993."
It may, however, be argued that the directive presupposes a finding of probable
cause. But then compliance with a constitutional requirement for the protection of
individual liberty cannot be left to presupposition, conjecture, or even convincing
logic.

III.
As earlier stated, per its 1st Indorsement of 21 April 1993, the DOJ gave due
course to the petitioners' petition for review pursuant to the exception provided for
in Section 4 of Circular No. 7, and directed the Office of the City Prosecutor of
Quezon City to forward to the Department the records of the cases and to file in
court a motion for the deferment of the proceedings. At the time it issued the
indorsement, the DOJ already knew that the information had been filed in court,
for which reason it directed the City Prosecutor to inform the Department whether
the accused have already been arraigned and if not yet arraigned, to move to defer
further proceedings. It must have been fully aware that, pursuant to Crespo
vs. Mogul, a motion to dismiss a case filed by the prosecution either as a
consequence of a reinvestigation or upon instructions of the Secretary of Justice
after a review of the records of the investigation is addressed to the trial court,
which has the option to grant or to deny it. Also, it must have been still fresh in its
mind that a few months back it had dismissed for lack of probable cause other
similar complaints of holders of "349" Pepsi crowns. 72 Thus, its decision to give
due course to the petition must have been prompted by nothing less than an honest
conviction that a review of the Joint Resolution was necessary in the highest
interest of justice in the light of the special circumstances of the case. That decision
was permissible within the "as far as practicable" criterion in Crespo.

Hence, the DOJ committed grave abuse of discretion when it executed on 23 July
1993 a unilateral volte-face, which was even unprovoked by a formal pleading to
accomplish the same end, by dismissing the petition for review. It dismissed the
petition simply because it thought that a review of the Joint Resolution would be
an exercise in futility in that any further action on the part of the Department would
depend on the sound discretion of the trial court, and that the latter's denial of the
motion to defer arraignment filed at the instance of the DOJ was clearly an exercise
of that discretion or was, in effect, a signal to the Department that the
determination of the case is within the court's exclusive jurisdiction and
competence. This infirmity becomes more pronounced because the reason adduced
by the respondent Judge for his denial of the motions to suspend proceedings and
hold in abeyance issuance of warrants of arrest and to defer arraignment finds, as
yet, no support in Crespo.

IV.

If the only issue before the Court of Appeals were the denial of the petitioners'
Motion to Suspend Proceedings and to Hold in Abeyance Issuance of Warrants of
Arrest and the public prosecutor's Motion to Defer Arraignment, which were both
based on the pendency before the DOJ of the petition for the review of the Joint
Resolution, the dismissal of CA-G.R. SP No. 31226 on the basis of the dismissal
by the DOJ of the petition for review might have been correct. However, the
petition likewise involved the issue of whether respondent Judge Asuncion gravely
abused his discretion in ordering the issuance of warrants of arrest despite want of
basis. The DOJ's dismissal of the petition for review did not render moot and
academic the latter issue.

In denying in its resolution of 1 July 1993 the petitioners' application for a writ of
preliminary injunction to restrain respondent Judge Asuncion from issuing
warrants of arrest, the Court of Appeals justified its action in this wise:

The Joint Resolution was sufficient in itself to have been relied upon
by respondent judge in convincing himself that probable cause indeed
exists for the purpose of issuing the corresponding warrants of arrest.
The mere silence of the records or the absence of any express
declaration in the questioned Order of May 17, 1993 as to where the
respondent Judge based his finding of probable cause does not give
rise to any adverse inference on his part. The fact remains that the
Joint Resolution was at respondent Judge's disposal at the time he
issued the Order for the issuance of the warrants of arrest. After all,
respondent Judge enjoys in his favor the presumption of regularity in
the performance of official actuations. And this presumption prevails
until it is overcome by clear and convincing evidence to the contrary.
Every reasonable intendment will be made in support of the
presumption, and in case of doubt as to an officer's act being lawful
or unlawful it should be construed to be lawful. (31 C.J.S., 808-
810. See also Mahilum, et al. vs. Court of Appeals, 17 SCRA 482;
People vs. Cortez, 21 SCRA 1228; Government of the P.I. vs.
Galarosa, 36 Phil. 338).

We are unable to agree with this disquisition, for it merely assumes at least two
things: (1) that respondent Judge Asuncion had read and relied on the Joint
Resolution and (2) he was convinced that probable cause exists for the issuance of
the warrants of arrest against the petitioners. Nothing in the records provides
reasonable basis for these assumptions. In his assailed order, the respondent Judge
made no mention of the Joint Resolution, which was attached to the records of
Criminal Case No. Q-93-43198 on 22 April 1993. Neither did he state that he
found probable cause for the issuance of warrants of arrest. And, for an
undivinable reason, he directed the issuance of warrants of arrest only "after June
21, 1993." If he did read the Joint Resolution and, in so reading, found probable
cause, there was absolutely no reason at all to delay for more than one month the
issuance of warrants of arrest. The most probable explanation for such delay could
be that the respondent Judge had actually wanted to wait for a little while for the
DOJ to resolve the petition for review.

It is, nevertheless, contended in the dissenting opinion of Mr. Justice Reynato S.


Puno that whatever doubts may have lingered on the issue of probable cause was
dissolved when no less than the Court of Appeals sustained the finding of probable
cause made by the respondent Judge after an evaluation of the Joint Resolution.
We are not persuaded with that opinion. It is anchored on erroneous premises. In
its 1 July 1993 resolution, the Court of Appeals does not at all state that it either
sustained respondent Judge Asuncion's finding of probable cause, or found by
itself probable cause. As discussed above, it merely presumed that Judge Asuncion
might have read the Joint Resolution and found probable cause from a reading
thereof. Then too, that statement in the dissenting opinion erroneously assumes that
the Joint Resolution can validly serve as sufficient basis for determining probable
cause. As stated above, it is not.

V.

In criminal prosecutions, the determination of probable cause may either be an


executive or a judicial prerogative. In People vs. Inting,73 this Court aptly stated:

And third, Judges and Prosecutors alike should distinguish the


preliminary inquiry which determines probable cause for the issuance
of a warrant of arrest from a preliminary investigation proper which
ascertains whether the offender should be held for trial or released.
Even if the two inquiries are conducted in the course of one and the
same proceeding, there should be no confusion about the objectives.
The determination of probable cause for the warrant of arrest is made
by the Judge. The preliminary investigation proper — whether or not
there is reasonable ground to believe that the accused is guilty of the
offense charged and, therefore, whether or not he should be subjected
to the expense, rigors and embarrassment of
trial — is the function of the Prosecutor.

....

We reiterate that preliminary investigation should be distinguished as


to whether it is an investigation for the determination of a sufficient
ground for the filing of the information or it is an investigation for the
determination of a probable cause for the issuance of a warrant of
arrest. The first kind of preliminary investigation is executive in
nature. It is part of the prosecution's job. The second kind of
preliminary investigation which is more properly called preliminary
examination is judicial in nature and is lodged with the judge. . . .

Ordinarily, the determination of probable cause is not lodged with this Court. Its
duty in an appropriate case is confined to the issue of whether the executive or
judicial determination, as the case may be, of probable cause was done without or
in excess of jurisdiction or with grave abuse of discretion amounting to want of
jurisdiction. This is consistent with the general rule that criminal prosecutions may
not be restrained or stayed by injunction, preliminary or final. There are, however,
exceptions to this rule. Among the exceptions are enumerated in Brocka
vs. Enrile74 as follows:

a. To afford adequate protection to the constitutional rights of the


accused (Hernandez vs. Albano, et al., L-19272, January 25, 1967, 19
SCRA 95);

b. When necessary for the orderly administration of justice or to


avoid oppression or multiplicity of actions (Dimayuga, et al. vs.
Fernandez, 43 Phil. 304; Hernandez vs. Albano, supra; Fortun vs.
Labang, et al., L-38383, May 27, 1981, 104 SCRA 607);

c. When there is a pre-judicial question which is sub judice (De Leon


vs. Mabanag, 70 Phil. 202);

d. When the acts of the officer are without or in excess of authority


(Planas vs. Oil, 67 Phil. 62);

e. Where the prosecution is under an invalid law, ordinance or


regulation (Young vs. Rafferty, 33 Phil. 556; Yu Cong Eng vs.
Trinidad, 47 Phil. 385, 389);

f. When double jeopardy is clearly apparent (Sangalang vs. People


and Avendia, 109 Phil. 1140);

g. Where the court has no jurisdiction over the offense (Lopez vs.
City Judge, L-25795, October 29, 1966, 18 SCRA 616);

h. Where it is a case of persecution rather than prosecution (Rustia vs.


Ocampo, CA-G.R. No. 4760, March 25, 1960);

i. Where the charges are manifestly false and motivated by the lust for
vengeance (Recto vs. Castelo, 18 L.J. [1953], cited in Rañoa vs.
Alvendia, CA-G.R. No. 30720-R, October 8, 1962; Cf. Guingona, et
al. vs. City Fiscal, L-60033, April 4, 1984, 128 SCRA 577); and

j. When there is clearly no prima facie case against the accused and a


motion to quash on that ground has been denied (Salonga vs. Paño, et
al., L- 59524, February 18, 1985, 134 SCRA 438).

7. Preliminary injunction has been issued by the Supreme Court to


prevent to threatened unlawful arrest of petitioners (Rodriguez vs.
Castelo, L- 6374, August 1, 1953). (cited in Regalado, Remedial Law
Compendium, p. 188, 1988 Ed.)
In these exceptional cases, this Court may ultimately resolve the existence
or non-existence of probable cause by examining the records of the
preliminary investigation, as it did in Salonga vs. Paño,75 Allado, and Webb.

There can be no doubt that, in light of the several thousand private complainants in
Criminal Case No. Q-93-43198 and several thousands more in different parts of the
country who are similarly situated as the former for being holders of "349" Pepsi
crowns, any affirmative holding of probable cause in the said case may cause or
provoke, as justly feared by the petitioners, the filing of several thousand cases in
various courts throughout the country. Inevitably, the petitioners would be exposed
to the harassments of warrants of arrest issued by such courts and to huge
expenditures for premiums on bailbonds and for travels from one court to another
throughout the length and breadth of the archipelago for their arraignments and
trials in such cases. Worse, the filing of these staggering number of cases would
necessarily affect the trial calendar of our overburdened judges and take much of
their attention, time, and energy, which they could devote to other equally, if not
more, important cases. Such a frightful scenario would seriously affect the orderly
administration of justice, or cause oppression or multiplicity of actions — a
situation already long conceded by this Court to be an exception to the general rule
that criminal prosecutions may not be restrained or stayed by injunction. 76

We shall not, however, reevaluate the evidence to determine if indeed there is


probable cause for the issuance of warrants of arrest in Criminal Case No. Q-93-
43298. For, as earlier stated, the respondent Judge did not, in fact, find that
probable cause exists, and if he did he did not have the basis therefor as mandated
by Soliven, Inting, Lim, Allado, and even Webb. Moreover, the records of the
preliminary investigation in Criminal Case No. Q-93-43198 are not with this
Court. They were forwarded by the Office of the City Prosecutor of Quezon City to
the DOJ in compliance with the latter's 1st Indorsement of 21 April 1993. The trial
court and the DOJ must be required to perform their duty.

WHEREFORE, the instant petition is GRANTED and the following are hereby
SET ASIDE:

(a) Decision of 28 September 1993 and Resolution of 9 February


1994 of respondent Court of Appeals in CA-G.R. SP No. 31226;

(b) The Resolutions of the "349" Committee of the Department of


Justice of 23 July 1993 dismissing the petitioners' petition for review
and of 3 February 1994 denying the motion to reconsider the
dismissal; and

(c) The Order of respondent Judge Maximiano C. Asuncion of 17


May 1993 in Criminal Case No. Q-93-43198.
The Department of Justice is DIRECTED to resolve on the merits, within sixty
(60) days from notice of this decision, the petitioners' petition for the review of the
Joint Resolution of Investigating Prosecutor Ramon Gerona and thereafter to file
the appropriate motion or pleading in Criminal Case No. Q-93-43198, which
respondent Judge Asuncion shall then resolve in light of Crespo vs. Mogul, Soliven
vs. Makasiar, People vs. Inting, Lim vs. Felix, Allado vs. Diokno, and Webb vs. De
Leon.

In the meantime, respondent Judge Asuncion is DIRECTED to cease and desist


from further proceeding with Criminal Case No. Q-93-43198 and to defer the
issuances of warrants of arrest against the petitioners.

No pronouncement as to costs.

PUNO, J., dissenting:

The constitutional policy of speedy adjudication of cases demand that we now


affirm or reverse the judicial finding of probable cause to hold petitioners for trial
on the charge of estafa. Pepsi's Number Fever Promotion, the root cause of the case
at bar, was held way back in 1992. Since 1993, City Prosecutor Candido Rivera of
Quezon City, RTC Judge Maximiano Asuncion and the Court of Appeals have
uniformly found the existence of probable cause against petitioners. It is now 1996
and petitioners have yet to be tried in court. Three (3) long years of expensive
litigation on the part of private respondents, mostly belonging to the powerless of
our people, will go to naught by remanding the case to the Department of Justice
for another executive determination of the issue of probable cause.

To be sure, the case at bar is deeply impressed with public interest. On one hand
are some 12,000 people holding "349" Pepsi crowns and who have long been
clamoring for payment of their prize money. Their collective claim runs to billions
of pesos. On the other hand is petitioners' business integrity which needs a shield
from false and malicious charges. We should decide this dispute with dispatch and
with little resort to procedural technicalities, otherwise, our people's search for
justice will be too wearisome a toil.

II

Pursuant to this precis, I will skip capillary issues and immediately go to the heart
of the case — i.e., determine whether the respondent Court of Appeals committed
reversible error in affirming the respondent trial judge who found probable cause to
hold petitioners for trial on the charge of estafa. The concept of probable cause is
not a high level legal abstraction to be the subject of warring thoughts. It is well
established that "a finding of probable cause needs only to rest on evidence
showing that more likely than not a crime has been committed and was committed
by the suspects. Probable cause need not be based on clear and convincing
evidence of guilt, neither on evidence establishing guilt beyond reasonable doubt,
and definitely not on evidence establishing absolute certainty of guilt." 1

On the basis of the evidence presented by the parties in a long and exhaustive
preliminary investigation, Quezon City Prosecutor Rivera determined that there is
a sufficient ground to engender a well founded belief that petitioners committed
estafa. City Prosecutor Rivera approved the findings of First Assistant City
Prosecutor Ramon M. Gerona contained in a 17-page Joint Resolution. I quote
in extenso the factual findings relied upon by the prosecutors in finding probable
cause, viz.:

xxx xxx xxx

The complaints-affidavits and replies by complainant and counter-


affidavits and rejoinder by respondents as well as arguments and
counter- arguments from both sides may be summed up to three
simple but comprehensive issues, to wit:

1. Was there fraud or deceit committed by Pepsi through


respondents prior to or simultaneously with their
deliberate act of refusal to pay complainants the prizes
indicated in their crown/caps?

2. Did Pepsi officials, herein respondents, comply with


the rules and regulations imposed by the DTI especially
on the mechanics of the promotion, or deviation,
modification, addition or deletion of aforenamed
mechanics?

3. Was there a way respondents could have avoided the


fraud?

Relative to the first and second issues, respondents insist that they
had complied with all the requirements or conditions imposed by the
DTI particularly with respect to the prior approval of the latter of the
mechanics of the promotion. Respondent likewise contend that the
deviation of the duly approved mechanics of the promotion was also
approved by the DTI. In this regard, Section 10.1 of the Ministry
Order No. 33 reads as follows:

10.1 All advertisements, brochures or any printed


material indicating or describing the mechanics of the
promotion shall conform with the mechanics approved
by this Bureau. Any deviation, modification, addition or
deletion shall first be submitted to this Bureau for
approval.

Parenthetically, the contention by respondents that the mechanics of


the promotion was approved by the DTI is not in question, but, the
additional contention that the deviation thereof was likewise
approved by the DTI is not supported by or does not jibe with the
facts. The report of Task Force DTI, page 14 thereof, says and we
quote:

It appears that after the "349" controversy which came


about during the extension period of Pepsi "Number
Fever" promo, the significance of the security code as a
measure against tampering and faking of the crowns or
caps has been modified. For after May 26, 1995 the
"349" number surfaced to have both winning and non-
winning security codes." (emphasis supplied)

Page 15 of the same Task Force Report reads:

The DTI-NRC records show that the


modification/deviation on the use of security code as
explained in the trade posters and other joint
advertisements was never submitted for approval in
violation of the specific requirements of 10.1 of
Memorandum Order No. 33.

As to why only number "349" has both a winning and


non-winning security code, Mr. Q.J. Gomez, Jr. could
not amplify the same except by testifying that the
supplier from Mexico gives them the list of winning
numbers and security codes together with the master list
of the non-winning number which were done through a
computer program.

Respondents admit that only "349" was given two kinds of security
codes, winning and non-winning. This condition was added by
respondents while the promo was going on and after "349" had been
announced as winner. The modification sans approval by the DTI as
shown in the preceding DTI findings to the extent that the holders of
the '"349" crowns are prejudiced or damaged after said number had
been drawn and announced as winner constitutes deceit, commencing
from the date of the launching of the promotion sometime in February
1992 up to the present with Pepsi's refusal to honor complainants'
demand for payment.

The alteration was found to be factual by the DTI in the last portion
of the Task Force Report which says with specifity:

xxx xxx xxx

The TF (Task Force) however noted it was only for No. "349" that a


deviation in the use of security code from what was originally
approved by the DTI-NCR was made. In all the other winning
numbers PPCPI and PCI complied with the approved mechanics.
(Emphasis supplied)

Indeed, the mechanics mentioned the use of "a 3 digit security code as
a measure against tampering or faking the crowns" and that "each and
every number has its own unique, matching security code." (counter-
affidavit, Rosemarie Vera, p. 13).

It is worth reproducing complainants' discussion of these two points


in their Memorandum.

Let us analyze these two rules:

4.2 The first rule defines the purposes of the security


code, which is to provide the basis for detecting whether
or not a crown containing a winning number is fake,
spurious or tampered with. By the wording of this rule, a
genuine, true and real Pepsi, Mirinda, 7-up or Mountain
Dew crown bearing a winning number, as drawn and
announced, could not possibly lose in the promo. The
genuineness of the crown will be assured by the security
code; and the drawn winning number it bears will make
it win.

In other words, the certainty about the genuineness of


the crown that is, not fake or tampered with is the
objective of the security code, not the crown's number
being a winning number. Stated otherwise, the rule, as
published makes the security code the determinant of
the genuineness of the crown, not the winning quality of
the number it bears.

Deliberately, however, Pepsi is now applying this rule


— nay, bending it — (see par. 4.6.1. Counter-Affidavit)
to make the security code determinant of which, among
the crowns bearing the winning number "349," is really
a winner! By giving the rule unwarranted and on-second
thought application, Pepsi has effectively defrauded
complainants of their prizes. Is this not deceit?

4.3 The second rule above-stated must be tackled in


conjunction with par. 4.6 of the Counter-Affidavit which
shows the meaning of the term "number" as used in this
rule. It means "A 3-digit number ranging from 001 to
999" found under the specially-maked crowns of
softdrinks manufactured and sold by Pepsi.

The rule uses the term "unique" which the dictionary


defines as "Being the only one of its kind" (Funk and
Gagnalls Standard) and "without another of the same
kind" (Webster's). A contextual and syntactical
appreciation of the rule would tell us that there is only
one security code of each number under the crown for
insuring the genuineness of the crown.

It is thus clear under the rule in question that "349" has


its own unique 7-digit security code to insure that the
crown bearing it is not fake or tampered with, do all the
other winning numbers have or should have. But what
did Pepsi do after "349" was drawn as a winner on May
25, 1992? Pepsi announced that "349" did not have only
one unique security code, but that it had both "winning"
and "non-winning" security codes. The security code of
"349" was not the one unique, but "349" itself became
unique because it became a winning and non-winning
number at the same time. Was this unique "uniqueness"
of "349" announced at the start of the promo? No! When
was the revelation made? Only after "349" was drawn as
a winner and numerous-thousands of winning crown
holders had stormed the Pepsi plants all over the
country, specially along Aurora Boulevard, Quezon
City, claiming their prizes.

The actuations of Pepsi vis-a-vis the above-stated two


rules are indubitable cases of "changing the rules as the
game is being played" to defraud the winners of the
prizes. If DECEIT has many faces, this is one of the
ugliest among them.
We also concur with the argument of complainants that additional
deceit was committed by respondents when they attempted to
substitute number "123" for number "349" as the winning number
announced and drawn on May 24, 1992 and the closure of Pepsi Plant
along Aurora Boulevard previously announced as redemption center
for winning crowns. The acts of respondents were described by
complainants as a continuation of their adamant refusal to pay and
even hear the claims of complainants who thereby sustained damage
not for their expenses for transportation but for the amounts of prizes
absolutely denied them, let alone their expense in buying Pepsi
softdrinks in quantities beyond their normal needs. There is merit in
the description.

The third issue is could Pepsi have remedied the fraud? Definitely, by
taking reasonable steps in paying the "349" holders. Pepsi could not
have succeeded in requesting approval by DTI of the deviation from
and/or modification of the mechanics previously approved as an
alternative remedy since sanctioning such deviation or modification
could have placed DTI in equal footing with respondents, making
them co-conspirators to the fraud.

The pertinent provision of the Revised Penal Code reads as follows:

Art. 318. Other Deceits. The penalty of Arresto


Mayor and a fine of not less than the amount of the
damage caused and not more than twice such amount
shall be imposed upon any person who shall defraud or
damage another by any other deceit not mentioned in the
preceding Articles of this Chapter.

As aptly contended by complainants any other kind of conceivable


deceit may fall under this Article. As in other cases of estafa, damage
to the offended party is required (Reyes, Revised Penal Code, p. 775,
Book 2, 11th Ed. 1977).

Fraudulently obtaining a loan on the promise that realty would be


mortgaged as security for said loan which promise was not fulfilled
because the borrower sold the property would constitute estafa under
Article 318 . . .

Complainants have, to our mind, succeeded in proving deceit and


fraud by respondents to avoid payment of prizes complainants are
claiming in the "Number Fever Promotion" for the "349" winning
number to hold respondents, whose names we will hereinafter
enumerate, liable for estafa (Art. 318, RPC).
The prosecutors' finding of probable cause rests on two (2) critical facts
established by substantial evidence: one, that petitioners deviated from the
Department of Trade and Industry (DTI) rules when they required that only
"349" crowns with security codes can win, and two, that petitioners
attempted to substitute "134" for "349" as the winning number. These acts
were interpreted by the prosecutors as prima facie deceitful and fraudulent. I
do not see how the resolution of the prosecutors finding sufficient ground to
charge petitioners with estafa can be successfully assailed as grave abuse of
discretion.

III

To be sure, respondent judge Asuncion affirmed the prosecutors' finding when


petitioners challenged its validity. He found probable cause against the petitioners
and ordered their arrest. The majority opinion faults the procedure followed by
Judge Asuncion in issuing the warrants of arrest against petitioners. It cites two (2)
reasons, viz.: (1) that Judge Asuncion issued the warrants merely on the basis of
the Information, Amended Information and Joint Resolution of the City
Prosecutors of Quezon City; he did not check and consult the complete records of
the case which include the affidavits of the witnesses, transcripts of stenographic
notes and other documents submitted in the preliminary investigation; and (2)
Judge Asuncion did not expressly make any finding of probable cause.

The procedure to be followed by a judge in reviewing the finding of probable


cause by a prosecutor has long been a quiescent area. In Soliven vs. Makasiar,2 we
laid down the following procedure, viz.:

xxx xxx xxx

The second issue, raised by Beltran, calls for an interpretation of the


constitutional provision on the issuance of warrants of arrest. The
pertinent provision reads:

Art. III, Sec. 2. The right of the people to be secure in


their persons, houses, papers and effects against
unreasonable searches and seizures of whatever nature
and for any purpose shall be inviolable, and no search
warrant or warrant of arrest shall issue except upon
probable cause to be determined personally by the judge
after examination under oath or affirmation of the
complainant and the witnesses he may produce, and
particularly describing the place to be searched and the
persons or things to be seized.
The addition of the word "personally" after the word "determined"
and the deletion of the grant of authority by the 1973 Constitution to
issue warrants to "other responsible officers as may be authorized by
law" has apparently convinced petitioner Beltran that the Constitution
now requires the judge to personally examine the complainant and his
witnesses in his determination of probable cause for the issuance of
warrants of arrest. This is not an accurate interpretation.

What the Constitution underscores is the exclusive and personal


responsibility of the issuing judge to satisfy himself of the existence
of probable cause. In satisfying himself of the existence of probable
cause for the issuance of a warrant of arrest, the judge is not required
to personally examine the complainant and his witnesses. Following
established doctrine and procedure, he shall: (1) personally evaluate
the report and the supporting documents submitted by the fiscal
regarding the existence of probable cause and, on the basis thereof,
issue a warrant of arrest; or (2) if on the basis thereof he finds no
probable cause, he may disregard the fiscal's report and require the
submission of supporting affidavits of witnesses to aid him in arriving
at a conclusion as to the existence of probable cause.

Sound policy dictates this procedure, otherwise judges would be


unduly laden with the preliminary examination and investigation of
criminal complaints instead of concentrating on hearing and deciding
cases filed before their courts.

Soliven and other related3 cases did not establish the absolute rule that
unless a judge has the complete records of the preliminary investigation
before him, he cannot lawfully determine probable cause and issue a warrant
of arrest. Soliven only held that it is the personal responsibility of the judge
to determine probable cause on the basis of the report and supporting
documents submitted by the fiscal; that he must independently evaluate the
report and supporting documents submitted by the fiscal; and, if he finds no
probable cause on the basis thereof, he can require submission of additional
supporting affidavits of witnesses. There is nothing in Soliven that requires
prosecutors to submit to the judge the complete records of the preliminary
investigation especially if they are voluminous. Nor is there anything
in Soliven that holds that the omission to physically submit the complete
records of the case would constitutionally infirm a finding of probable cause
by a judge even if it was made on the basis of an exhaustive prosecutor's
report or resolution. Indeed, in Webb vs. de Leon,4 we sustained the finding
of probable cause made by the trial judge even if the complete records of the
preliminary investigation were not elevated to the said judge.
A revisit of our case law will reveal that what we condemned in the past as
constitutionally impermissible was the practice of judges of totally relying on pro
forma certifications of fiscals that they conducted a preliminary investigation and
found probable cause that the accused committed the crime charged in the
Information. These pro forma certifications usually consisted of a short sentence.
They did not relate the relevant proceedings in the preliminary investigation nor
did they calibrate the weight of diverse and dueling evidence submitted by the
parties. These bare certifications carried no findings of fact and made no legal
analysis which could be used by judges as a rational basis for a determination of
probable cause. Thus, we laid down the jurisprudence that a judge who determines
probable cause by relying on such meaningless certifications violates the
constitutional provision prohibiting issuance of warrants of arrest ". . . except upon
probable cause to be determined personally by the judge . . .

The case at bar does not involve these outlawed certifications. The respondent
Court of Appeals found that the 17-page Joint Resolution of the prosecutors
provided the trial judge with sufficient factual basis to find probable cause and to
issue warrants of arrest against the petitioners. To repeat, the finding of probable
cause against petitioners rests on two (2) critical facts established by evidence: one,
that petitioners deviated from the Department of Trade and Industry rules when
they required that only "349" crowns with security codes could win, and two, that
petitioners attempted to substitute "134" for "349" as the winning number. The
finding of deviation is based on the Task Force Report of the DTI, the relevant
portion of which was liberally quoted in the prosecutors' Joint Resolution. The
finding of attempt at substitution was taken from the affidavits of witnesses of the
private respondents. Petitioners do not charge that the Task Force Report of the
DTI and the affidavits of witnesses of the private respondents were incorrectly
quoted by the prosecutors in their joint Resolution. Thus, respondent judge need
not be burdened by the duty of ordering the elevation of the complete records of
the preliminary investigation to check the accuracy of the critical evidence as
stated in the Joint Resolution.

The majority opinion also flays Judge Asuncion allegedly because


". . . he made no finding of probable cause . . ." I am not disposed to make this
serious charge. When Judge Asuncion issued the warrants of arrest against
petitioners, I assume as did the respondent Court of Appeals, that he had studied
the Information and 17-page Resolution of the prosecutors and that he agreed with
the prosecutors' finding of probable cause. It is unnecessary for him to issue an
Order just to reiterate the findings of the prosecutors. It ought to be likewise
underscored that before Judge Asuncion issued the warrants of arrest, the matter of
probable cause was the subject of exhaustive pleadings before him. Thus, the
parties submitted the following for the respondent judge's consideration: (1)
Motions to Suspend Proceedings and to Hold in Abeyance Issuance of Warrants of
Arrest; (2) Motion for Issuance of Warrants of Arrest; (3) Supplemental Urgent
Motion to Hold in Abeyance Issuance of Warrants of Arrest and to Suspend
Proceedings; (4) Opposition to Motion to Defer Arraignment; (5) Objection and
Opposition to Motion to Suspend Proceedings and to Hold in Abeyance the
Issuance of Warrants of Arrest; and (6) Memorandum in Support of the Motion to
Suspend Proceedings and to Hold in Abeyance the Issuance of the Warrants of
Arrest. In these pleadings, the parties, especially the petitioners, discussed in length
and in depth the findings of the prosecutors as contained in their 17-page Joint
Resolution. It is, thus, erroneous to assume that the respondent judge had nothing
before him when he ruled that there is probable cause to charge petitioners with
estafa.

With due respect to the majority, the ruling that a judge should always order the
elevation of the complete records of a preliminary investigation before proceeding
with the task of reviewing the finding of probable cause made by prosecutors will
exacerbate the mischief of delays in the disposition of criminal cases. This will not
sit well with our people who are complaining that their continuing calls for speedy
justice are only receiving dial tones from courts. The transcription of stenographic
notes and the transfer of physical and documentary evidence, especially when
voluminous, will consume time, result in loss of valuable evidence and aggravate
the burden of litigants. It is my humble submission that the forwarding of complete
records is not necessary when the prosecutor's report is exhaustive and accurate as
in the case at bar.

IV

The majority has deviated from the general rule when it set aside the finding of
probable cause made by the respondent Court of Appeals and the respondent trial
judge. To be sure, this Court can restrain the prosecution of criminal prosecutions
in exceptional cases. These exceptional cases are: 5

a. To afford adequate protection to the constitutional rights of the


accused (Hernandez vs. Albano, et al., L-19272, January 25, 1967, 19
SCRA 95);

b. When necessary for the orderly administration of justice or to


avoid oppression or multiplicity of actions (Dimayuga, et al. vs.
Fernandez, 43 Phil. 304; Hernandez vs. Albano, supra; Fortun vs.
Labang, et al., L-38383, May 27, 1981, 104 SCRA 607);

c. When there is a prejudicial question which is sub judice (De Leon


vs. Mabanag, 70 Phil. 202);

d. When the acts of the officer are without or in excess of authority


(Planas vs. Gil, 67 Phil. 62);
e. Where the prosecution is under an invalid law, ordinance or
regulation (Young vs. Rafferty, 33 Phil. 556; Yu Cong Eng vs.
Trinidad, 47 Phil. 385, 389);

f. When double jeopardy is clearly apparent (Sangalang vs. People


and Avendia, 109 Phil. 1140);

g. Where the court has no jurisdiction over the offense (Lopez vs.
City Judge, L-25795, October 29, 1966, 18 SCRA 616)

h. Where it is a case of persecution rather than prosecution (Rustia vs.


Ocampo, CA-G.R. 4760, March 25, 1960);

i. Where the charges are manifestly false and motivated by the lust for
vengeance (Recto vs. Castelo, 18 L.J. [1953], cited in Ranoa vs.
Alvendia, CA-G.R. No. 30720-R, October 8, 1962, cf. Guingona, et
al. vs. City Fiscal, L-60033, April 4, 1984, 128 SCRA 577); and

j. Where there is clearly no prima facie case against the accused and a


motion to quash on that ground has been denied (Salonga vs. Pano, et
al., L-59524, February 19, 1985, 134 SCRA 438).

7. Preliminary injunction has been issued by the Supreme Court to


prevent the threatened unlawful arrest of petitioners (Rodriguez vs.
Castelo, L-6374, August 1, 1953). (cited in Regalado, Remedial Law
Compendium, p. 288, 1988 Ed.)

It must be stressed, however, that in these exceptional cases, the Court took
the extraordinary step of annulling findings of probable cause either to
prevent the misuse of the strong arm of the law or to protect the orderly
administration of justice. The constitutional duty of this Court in criminal
litigations is not only to acquit the innocent after trial but to insulate, from
the start, the innocent from unfounded charges. For the Court is aware of the
strains of a criminal accusation and the stresses of litigation which should
not be suffered by the clearly innocent. The filing of an unfounded criminal
information in court exposes the innocent to severe distress especially when
the crime is not bailable. Even an acquittal of the innocent will not fully
bleach the dark and deep stains left by a baseless accusation for reputation
once tarnished remains tarnished for a long length of time. The expense to
establish innocence may also be prohibitive and can be more punishing
especially to the poor and the powerless. Innocence ought to be enough and
the business of this Court is to shield the innocent from senseless suits right
from the start.
I respectfully submit, however, that the peculiar facts obtaining in the case at bar
do not warrant us to take the exceptional step of setting aside the finding of
probable cause made by the respondent appellate court and the trial court. Their
finding is supported by substantial evidence and the issuance of warrants of arrest
against the petitioners to hold them for trial for estafa does not constitute misuse of
prosecutorial powers. To be sure, petitioners will be exposed to the inconvenience
of facing numerous similar criminal suits but so long as the inconvenience is no
more than what is necessary to dispense justice, they have no cause to gripe for
justice equally belongs to the private respondents.

It is also respectfully submitted that the Department of Justice did not act with
grave abuse of discretion when it refused to review the City Prosecutor's Joint
Resolution and dismissed petitioners' appeal. The applicable case law is Crespo
vs. Mogul, et al.,6 where we held:

xxx xxx xxx

The rule therefore in this jurisdiction is that once a complaint or


information is filed in Court any disposition of the case as its
dismissal or the conviction or acquittal of the accused rests in the
sound discretion of the Court. Although the fiscal retains the direction
and control of the prosecution of criminal cases even while the case is
already in Court he cannot impose his opinion on the trial court. The
Court is the best and sole judge on what to do with the case before it.
The determination of the case is within its exclusive jurisdiction and
competence. A motion to dismiss the case filed by the fiscal should
be addressed to the Court who has the option to grant or deny the
same. It does not matter if this is done before or after the arraignment
of the accused or that the motion was filed after a reinvestigation or
upon instructions of the Secretary of Justice who reviewed the
records of the investigation.

In order therefore to avoid such a situation whereby the opinion of the


Secretary of Justice who reviewed the action of the fiscal may be
disregarded by the trial court, the Secretary of Justice should, as far as
practicable, refrain from entertaining a petition for review or appeal
from the action of the fiscal, when the complaint or information has
already been filed in Court. The matter should be left entirely for the
determination of the Court.

I concede that respondent judge Asuncion misread Crespo when he denied


the prosecution's Motion to Defer Further Proceedings on the ground that ". .
. to follow whatever opinion the Secretary of Justice may have on the matter
would undermine the independence and integrity of this Court." I agree
that Crespo did not prohibit the Department of Justice from reviewing
resolutions of its prosecutors even if the proper informations have already
been filed with the courts. Crespo merely counselled the Secretary of Justice
to refrain from exercising said power of review "as far as practicable" taking
into account the broader interest for a more orderly administration of justice.
In exceptional instances where it is practicable for the Secretary of Justice to
exercise the power of review, courts should not be heard to complain that
their independence will be undermined. The dispensation of justice is not
the monopoly of courts. It is as much the responsibility of the two other
great branches of our government, the Executive and the Legislative.

Nevertheless, the refusal of the respondent Judge Asuncion to defer proceedings


based on a misperception of Crespo is now of deminimis importance. The initial
decision of the DOJ to review petitioners' case was due to its impression that the
finding of probable cause made by the prosecutors of Quezon City was, at that
time, open to honest contentions. This doubt, however, dissolved when no less than
the respondent Court of Appeals sustained the finding of probable cause made by
the respondent judge after an evaluation of the Joint Resolution of the Quezon City
prosecutors. With the imprimatur of the respondent Court of Appeals on the
existence of probable cause and following Crespo, it is no longer "practicable" for
the DOJ to further review petitioners' case. Contrary to the impression of the
majority, the appellate court affirmed the ruling of respondent judge on probable
cause only after a long and deliberate study of the issue. The issue of probable
cause was the subject of oral arguments and extensive pleadings before the
appellate court which even directed the elevation of the original records of
Criminal Case No. Q-93-43198. The probability that the DOJ will reach a finding
different from the appellate court is nil considering that it will be reviewing the
same set of evidence.

Finally, petitioners justify the need for DOJ to review their case in view of the
latter's alleged contradictory rulings on cases brought by different parties involving
the same controversy. The DOJ has denied the charge that it has issued
contradictory rulings. But if these contradictory rulings were truly rendered by
DOJ, there is more reason for DOJ to let the issue be resolved by the courts. As
ultimate arbiters of rights in conflict, only the courts can write finis to the
controversy between petitioners and private respondents.

I vote to dismiss the petition.

CASE: Enrile v. Salazar, G.R. No. 92163, June 5, 1990


ISSUE:
Whether or not the warrant of arrest is valid
RULING:
Yes. This Court has already ruled, however, that it is not the unavoidable duty of
the judge to make such a personal examination, it being sufficient that he follows
established procedure by personally evaluating the report and the supporting
documents submitted by the prosecutor." 

Moreover, alleged absence of sufficient time to personally go over the voluminous


records of the preliminary investigation not a valid reason to assume that judge had
not complied with his duty.

It is sufficient that Judge Salazar followed the established procedure by personally


evaluating the report and the supporting documents submitted by the prosecutor.
Merely because said respondent had what some might consider only a relatively
brief period within which to comply with that duty, gives no reason to assume that
he had not, or could not have, so complied; nor does that single circumstance
suffice to overcome the legal presumption that official duty has been regularly
performed.

H. Warrantless arrests (Sec. 5, Rule 113, The Revised Rules of Criminal


Procedure) 1. In flagrante arrest (person to be arrested has committed, is actually
committing, or is attempting to commit an offense)

2. Hot pursuit arrest (an offense has just been committed and arresting officer has
probable cause to believe based on personal knowledge of facts or circumstances
that the person to be arrested has committed the offense)

3. Arrest of prisoner who has escaped

1. Who may raise

2. When to raise

CASES:
Umil v. Ramos, G.R. No. 81567, July 9, 1990 (decision); October 3, 1991
(resolution) (read also the concurring and dissenting opinion of J. Feliciano)
The are eight (8) petitioners for habeas corpus filed before the Court, which have
been consolidated because of the similarity of issues raised, praying for the
issuance of the writ of habeas corpus, ordering the respective respondents to
produce the bodies of the persons named therein and to explain why they should
not be set at liberty without further delay.

In their respective Returns, the respondents uniformly assert that the privilege of
the writ of habeas corpus is not available to the petitioners as they have
been legally arrested and are detained by virtue of valid informations filed in court
against them.

The petitioners counter that their detention is unlawful as their arrests were
made without warrant and, that no preliminary investigation was first conducted,
so that the informations filed against them are null and void.

The Court has carefully reviewed the contentions of the parties in their respective
pleadings, and it finds that the persons detained have not been illegally arrested nor
arbitrarily deprived of their constitutional right to liberty, and that the
circumstances attending these cases do not warrant their release on habeas corpus.

The arrest of a person without a warrant of arrest or previous complaint is


recognized in law. The occasions or instances when such an arrest may be effected
are clearly spelled out in Section 5, Rule 113 of the Rules of Court, as amended,
which provides:

Sec. 5. Arrest without warrant; when lawful. — A peace officer or a


private person may, without a warrant, arrest a person:

(a) When, in his presence, the person to be arrested has committed, is


actually committing, or is attempting to commit an offense;

(b) When an offense has in fact just been committed, and he has
personal knowledge of facts indicating that the person to be arrested
has committed it; and

(c) When the person to be arrested is a prisoner who has escaped from
a penal establishment or place where he is serving final judgment or
temporarily confined while his case is pending, or has escaped while
being transferred from one confinement to another.

In cases falling under paragraphs (a) and (b) hereof, the person
arrested without a warrant shall be forthwith delivered to the nearest
police station or jail, and he shall be proceeded against in accordance
with Rule 112, Section 7.

An arrest without a warrant of arrest, under Section 5 paragraphs (a) and (b) of
Rule 113 of the Rules of Court, as amended, is justified when the person arrested is
caught in flagranti delicto, viz., in the act of committing an offense; or when an
offense has just been committed and the person making the arrest has personal
knowledge of the facts indicating that the person arrested has committed it. The
rationale behind lawful arrests, without warrant, was stated by this Court in the
case of People vs. Kagui Malasugui 1 thus:

To hold that no criminal can, in any case, be arrested and searched for
the evidence and tokens of his crime without a warrant, would be to
leave society, to a large extent, at the mercy of the shrewdest, the
most expert, and the most depraved of criminals, facilitating their
escape in many instances.

The record of the instant cases would show that the persons in whose behalf these
petitions for habeas corpus have been filed, had freshly committed or were
actually committing an offense, when apprehended, so that their arrests without a
warrant were clearly justified, and that they are, further, detained by virtue of valid
informations filed against them in court.

A brief narration of the facts and events surrounding each of the eight (8) petitions
is in order.

In G.R. No. 81567 (Umil vs. Ramos), the record shows that, on 1 February 1988,
the Regional Intelligence Operations Unit of the Capital Command (RIOU-
CAPCOM) received confidential information about a member of the NPA Sparrow
Unit (liquidation squad) being treated for a gunshot wound at the St. Agnes
Hospital in Roosevelt Avenue, Quezon City. Upon verification, it was found that
the wounded person, who was listed in the hospital records as Ronnie Javelon, is
actually Rolando Dural, a member of the NPA liquidation squad, responsible for
the killing of two (2) CAPCOM soldiers the day before, or on 31 January 1988, in
Macanining Street, Bagong Barrio, Caloocan City. In view of this verification,
Rolando Dural was transferred to the Regional Medical Services of the CAPCOM,
for security reasons. While confined thereat, or on 4 February 1988, Rolando Dural
was positively identified by eyewitnesses as the gunman who went on top of the
hood of the CAPCOM mobile patrol car, and fired at the two (2) CAPCOM
soldiers seated inside the car identified as T/Sgt. Carlos Pabon and CIC Renato
Manligot.
As a consequence of this positive identification, Rolando Dural was referred to the
Caloocan City Fiscal who conducted an inquest and thereafter filed with the
Regional Trial Court of Caloocan City an information charging Rolando
Dural alias Ronnie Javelon with the crime of "Double Murder with Assault Upon
Agents of Persons in Authority." The case was docketed therein as Criminal Case
No. C-30112 and no bail was recommended. On 15 February 1988, the information
was amended to include, as defendant, Bernardo Itucal, Jr. who, at the filing of the
original information, was still unidentified.

Meanwhile, on 6 February 1988, a petition for habeas corpus was filed with this


Court on behalf of Roberto Umil, Rolando Dural, and Renato Villanueva. The
Court issued the writ of habeas corpus on 9 February 1988 and the respondents
filed a Return of the Writ on 12 February 1988. Thereafter, the parties were heard
on 15 February 1988.

On 26 February 1988, however, Roberto Umil and Renato Villanueva posted bail


before the Regional Trial Court of Pasay City where charges for violation of the
Anti-Subversion Act had been filed against them, and they were accordingly
released. The petition for habeas corpus, insofar as Umil and Villanueva are
concerned, is now moot and academic and is accordingly dismissed, since the writ
of habeas corpus does not lie in favor of an accused in a criminal case who has
been released on bail. 2

As to Rolando Dural, it clearly appears that he was not arrested while in the act of
shooting the two (2) CAPCOM soldiers aforementioned. Nor was he arrested just
after the commission of the said offense for his arrest came a day after the said
shooting incident. Seemingly, his arrest without warrant is unjustified.

However, Rolando Dural was arrested for being a member of the New Peoples
Army (NPA), an outlawed subversive organization. Subversion being a continuing
offense, the arrest of Rolando Dural without warrant is justified as it can be said
that he was committing an offense when arrested. The crimes of rebellion,
subversion, conspiracy or proposal to commit such crimes, and crimes or offenses
committed in furtherance thereof or in connection therewith constitute direct
assaults against the State and are in the nature of continuing crimes. As stated by
the Court in an earlier case:

From the facts as above-narrated, the claim of the petitioners that they
were initially arrested illegally is, therefore, without basis in law and
in fact. The crimes of insurrection or rebellion, subversion,
conspiracy or proposal to commit such crimes, and other crimes and
offenses committed in the furtherance, on the occasion thereof, or
incident thereto, or in connection therewith under Presidential
Proclamation No. 2045, are all in the nature of continuing offenses
which set them apart from the common offenses, aside from their
essentially involving a massive conspiracy of nationwide magnitude.
Clearly then, the arrest of the herein detainees was well within the
bounds of the law and existing jurisprudence in our jurisdiction.

2. The arrest of persons involved in the rebellion whether as its


fighting armed elements, or for committing non-violent acts but in
furtherance of the rebellion, is more an act of capturing them in the
course of an armed conflict, to quell the rebellion, than for the
purpose of immediately prosecuting them in court for a statutory
offense. The arrest, therefore, need not follow the usual procedure in
the prosecution of offenses which requires the determination by a
judge of the existence of probable cause before the issuance of a
judicial warrant of arrest and the granting of bail if the offense is
bailable. Obviously, the absence of a judicial warrant is no legal
impediment to arresting or capturing persons committing overt acts of
violence against government forces, or any other milder acts but
equally in pursuance of the rebellious movement. The arrest or
capture is thus impelled by the exigencies of the situation that
involves the very survival of society and its government and duly
constituted authorities. If killing and other acts of violence against the
rebels find justification in the exigencies of armed hostilities which is
of the essence of waging a rebellion or insurrection, most assuredly
so in case of invasion, merely seizing their persons and detaining
them while any of these contingencies continues cannot be less
justified. . . . 3

The record, moreover, shows that the criminal case filed against Rolando
Dural and Bernardo Itucal, Jr. for "Double Murder, etc." was tried in the court
below and at the conclusion thereof, or on 17 August 1988, Rolando Dural and
Bernardo Itucal, Jr. were found guilty of the charge and sentenced accordingly.
Rolando Dural is now serving the sentence imposed upon him by the trial court.
Thus, the writ of habeas corpus is no longer available to him. For, as held in the
early case of U.S. vs. Wilson: 4

In this case, whatever may be said about the manner of his arrest, the
fact remains that the defendant was actually in court in the custody of
the law on March 29, when a complaint sufficient in form and
substance was read to him. To this he pleaded not guilty. The trial
followed, in which, and in the judgment of guilty pronounced by the
court, we find no error. Whether, if there were irregularities in
bringing him personally before the court, he could have been released
on a writ of habeas corpus or now has a civil action for damages
against the person who arrested him we need not inquire. It is enough
to say that such irregularities are not sufficient to set aside a valid
judgment rendered upon a sufficient complaint and after a trial free
from error.

II

In G.R. Nos. 84581-82 (Roque vs. De Villa), the arrest of Amelia


Roque and Wilfredo Buenaobra, without warrant, is also justified. When
apprehended at the house of Renato Constantino in Marikina Heights, Marikina,
Metro Manila, Wilfredo Buenaobra admitted that he was an NPA courier and he
had with him letters to Renato Constantino and other members of the rebel group.
Amelia Roque, upon the other hand, was a member of the National United Front
Commission, in charge of finance, and admitted ownership of subversive
documents found in the house of her sister in Caloocan City. She was also in
possession of ammunition and a fragmentation grenade for which she had no
permit or authority to possess.

The record of these two (2) cases shows that on 27 June 1988, one Rogelio Ramos
y Ibanes, a member of the NPA, who had surrendered to the military authorities,
told military agents about the operations of the Communist Party of the Philippines
(CPP) and the New Peoples Army (NPA) in Metro Manila. He identified some of
his former comrades as "Ka Mong", a staff member of the Communications and
Transportation Bureau; "Ka Nelia", a staff member in charge of finance; "Ka
Miller", an NPA courier from Sorsogon and Lopez, Quezon; "Ka Ted", and "Ka
Totoy". He also pointed to a certain house occupied by Renato Constantino located
in the Villaluz Compound, Molave St., Marikina Heights, Marikina, Metro Manila,
which is used as a safehouse of the National United Front Commission (NUFC) of
the CPP-NPA.

In view of these revelations, the Constantino house was placed under military
surveillance and on 12 August 1988, pursuant to a search warrant issued by Judge
Eutropio Migrino of the Regional Trial Court of Pasig, a search of the house was
conducted at about 5:00 o'clock in the afternoon, by a combined team of the
Criminal Investigation Service, National Capital District (CIS-NCD) and the
Constabulary Security Group (CSG). In the course of the search, the following
articles were found and taken under proper receipt:

a) One (1) Colt M16A1 long rifle with defaced serial number;

b) One (1) Cal. .380 ACT/9mm Model PPK/8 SN: 260577 & 2605778;

c) Two (2) fragmentation hand grenades;

d) Fifty-six (56) live ammunition for Cal. 5.56 mm;

e) Five (5) live ammunition for Cal. .380;


f) One (1) ICOM VHF FM Radio Transciever SN: 14903

g) One (1) Regulated power supply 220V AC;

h) One (1) Antennae (adjustable);

i) One (1) Speaker with cord ALEXAR;

j) Voluminous Subversive documents.

When confronted, Renato Constatino could not produce any permit or authority to
possess the firearms, ammunition, radio and other communications equipment.
Hence, he was brought to the CIS Headquarters for investigation. When
questioned, he refused to give a written statement, although he admitted that he
was a staff member of the executive committee of the NUFC and a ranking
member of the International Department of the Communist Party of the Philippines
(CPP).

At about 8:00 o'clock in the evening of the same day (12 August 1988), Wilfredo
Buenaobra arrived at the house of Renato Constantino in the Villaluz Compound.
When accosted, he readily admitted to the military agents that he is a regular
member of the CPP/NPA and that he went to the place to deliver letters to "Ka
Mong", referring to Renato Constatino, and other members of the rebel group. On
further questioning, he also admitted that he is known as "Ka Miller" and that he
was from Barangay San Pedro, Lopez, Quezon. Among the items taken from him
were the following:

(1) Handwritten letter addressed to "Ka Bing & Co. from A & Co."
dated August 11, 1988;

(2) Handwritten letter addressed to "ROD from VIC (Schell datre)"


dated August 11, 1988;

(3) Handwritten letter addressed to "Suzie" from "Vic", dated August


11, 1988.

Also found Buenaobra's possession was a piece of paper containing a written but
jumbled telephone number of Florida M. Roque, sister of Amelia Roque alias "Ka
Nelia", at 69 Geronimo St., Caloocan City. Acting on the lead provided as to the
whereabouts of Amelia Roque, the military agents went to the given address the
next day (13 August 1988). They arrived at the place at about 11:00 o'clock in the
morning. After identifying themselves as military agents and after seeking
permission to search the place, which was granted, the military agents conducted a
search in the presence of the occupants of the house and the barangay captain of
the place, one Jesus D. Olba.
The military agents found the place to be another safehouse of the NUFC/CPP.
They found ledgers, journals, vouchers, bank deposit books, folders, computer
diskettes, and subversive documents as well as live ammunition for a .38 SPL
Winchester, 11 rounds of live ammunition for a cal. .45, 19 rounds of live
ammunition for an M16 Rifle, and a fragmentation grenade. As a result, Amelia
Roque and the other occupants of the house were brought to the PC-CIS
Headquarters at Camp Crame, Quezon City, for investigation. Amelia
Roque admitted to the investigators that the voluminous documents belonged to
her and that the other occupants of the house had no knowledge of them. As a
result, the said other occupants of the house were released from custody.

On 15 August 1988, Amelia Roque was brought to the Caloocan City Fiscal for
inquest after which an information charging her with violation of PD 1866 was
filed with the Regional Trial Court of Caloocan City. The case is docketed therein
as Criminal Case No. C-1196. Another information for violation of the Anti-
Subversion Act was filed against Amelia Roque before the Metropolitan Trial
Court of Caloocan City, which is docketed therein as Criminal Case No. C-150458.

An information for violation of the Anti-Subversion Act was filed against Wilfredo


Buenaobra before the Metropolitan Trial Court of Marikina, Metro Manila. The
case is docketed therein as Criminal Case No. 23715. Bail was set at P4,000.00.

On 24 August 1988, a petition for habeas corpus was filed before this Court on


behalf of Amelia Roque and Wilfredo Buenaobra. At the hearing of the case,
however, Wilfredo Buenaobra manifested his desire to stay in the PC-INP
Stockade at Camp Crame, Quezon City. According, the petition for habeas
corpus filed on his behalf is now moot and academic. Only the petition of Amelia
Roque remains for resolution.

The contention of respondents that petitioners Roque and Buenaobra are officers
and/or members of the National United Front Commission (NUFC) of the CPP was
not controverted or traversed by said petitioners. The contention must be deemed
admitted. 5 As officers and/or members of the NUFC-CPP, their arrest, without
warrant, was justified for the same reasons earlier stated vis-a-vis Rolando Dural.
The arrest without warrant of Roque was additionally justified as she was, at the
time of apprehension, in possession of ammunitions without license to possess
them.

III

In G.R. Nos. 84583-84 (Anonuevo vs. Ramos), the arrest of Domingo


Anonuevo and Ramon Casiple, without warrant, is also justified under the rules.
Both are admittedly members of the standing committee of the NUFC and, when
apprehended in the house of Renato Constatino, they had a bag containing
subversive materials, and both carried firearms and ammunition for which they had
no license to possess or carry.

The record of these two (2) cases shows that at about 7:30 o'clock in the evening of
13 August 1988, Domingo T. Anonuevo and Ramon Casiple arrived at the house
of Renato Constatino at Marikina Heights, Marikina, which was still under
surveillance by military agents. The military agents noticed bulging objects on
their waist lines. When frisked, the agents found them to be loaded guns.
Anonuevo and Casiple were asked to show their permit or license to possess or
carry firearms and ammunition, but they could not produce any. Hence, they were
brought to PC Headquarters for investigation. Found in their possession were the
following articles:

a) Voluminous subversive documents

b) One (1) Cal. 7.65 MOD 83 2C Pistol SN: 001412 with one (1)
magazine for Cal. 7.65 containing ten (10) live ammunition of same
caliber;

c) One (1) Cal. 7.65 Pietro Barreta SN; A18868 last digit tampered
with one (1) magazine containing five (5) live ammunition of same
caliber.

At the PC Stockade, Domingo Anonuevo was identified as "Ka Ted", and Ramon
Casiple as "Ka Totoy" of the CPP, by their comrades who had previously
surrendered to the military.

On 15 August 1988, the record of the investigation and other documentary


evidence were forwarded to the Provincial Fiscal at Pasig, Metro Manila, who
conducted an inquest, after which Domingo Anonuevo and Ramon Casiple were
charged with violation of Presidential Decree No. 1866 before the Regional Trial
Court of Pasig, Metro Manila. The cases are docketed therein as Criminal Cases
Nos. 74386 ad 74387, respectively. No bail was recommended.

On 24 August 1988, a petition for habeas corpus was filed with this Court on


behalf of Domingo Anonuevo and Ramon Casiple, alleging that the said Anonuevo
and Casiple were unlawfully arrested without a warrant and that the informations
filed against them are null and void for having been filed without prior hearing and
preliminary investigation. On 30 August 1988, the Court issued the writ of habeas
corpus, and after the respondents had filed a Return of the Writ, the parties were
heard.

The petitioners' (Anonuevo and Casiple) claim that they were unlawfully arrested
because there was no previous warrant of arrest, is without merit The record shows
that Domingo Anonuevo and Ramon Casiple were carrying unlicensed firearms
and ammunition in their person when they were apprehended.

There is also no merit in the contention that the informations filed against them are
null and void for want of a preliminary investigation. The filing of an information,
without a preliminary investigation having been first conducted, is sanctioned by
the Rules. Sec. 7, Rule 112 of the Rules of Court, as amended, reads:

Sec. 7. When accused lawfully arrested without a warrant. — When a


person is lawfully arrested without a warrant for an offense
cognizable by the Regional Trial Court the complaint or information
may be filed by the offended party, peace officer or fiscal without a
preliminary investigation having been first conducted, on the basis of
the affidavit of the offended party or arresting officer or person.

However, before the filing of such complaint or information, the


person arrested may ask for a preliminary investigation by a proper
officer in accordance with this Rule, but he must sign a waiver of the
provisions of Article 125 of the Revised Penal Code, as amended,
with the assistance of a lawyer and in case of non-availability of a
lawyer, a responsible person of his choice. Notwithstanding such
waiver, he may apply for bail as provided in the corresponding rule
and the investigation must be terminated within fifteen (15) days
from its inception.

If the case has been filed in court without a preliminary investigation


having been first conducted, the accused may within five (5) days
from the time he learns of the filing of the information, ask for a
preliminary investigation with the same right to adduced evidence in
his favor in the manner prescribed in this Rule.

The petitioners Domingo Anonuevo and Ramon Casiple, however, refused to sign
a waiver of the provisions of Article 125 of the Revised Penal Code, as amended.
In the informations filed against them, the prosecutor made identical certifications,
as follows:

This is to certify that the accused has been charged in accordance


with Sec. 7, Rule 112 of the 1985 Rules on Criminal Procedure, that
no preliminary investigation was conducted because the accused has
not made and signed a waiver of the provisions of Art. 125 of the
Revised Penal Code, as amended; that based on the evidence
presented, there is reasonable ground to believe that the crime has
been committed, and that the accused is probably guilty thereof.
Nor did petitioners ask for a preliminary investigation after the informations had
been filed against them in court. Petitioners cannot now claim that they have been
deprived of their constitutional right to due process.

IV

In G.R. No. 83162 (Ocaya vs. Aguirre), the arrest without warrant, of Vicky
Ocaya is justified under the Rules, since she had with her unlicensed ammunition
when she was arrested. The record of this case shows that on 12 May 1988, agents
of the PC Intelligence and Investigation of the Rizal PC-INP Command, armed
with a search warrant issued by Judge Eutropio Migrino of the Regional Trial
Court of Pasig, Metro Manila, conducted a search of a house located at Block 19,
Phase II, Marikina Green Heights, Marikina, Metro Manila, believed to be
occupied by Benito Tiamson, head of the CPP-NPA. In the course of the search,
Vicky Ocaya arrived in a car driven by Danny Rivera. Subversive documents and
several rounds of ammunition for a .45 cal. pistol were found in the car of Vicky
Ocaya. As a result, Vicky Ocaya and Danny Rivera were brought to the PC
Headquarters for investigation. When Vicky Ocaya could not produce any permit
or authorization to possess the ammunition, an information charging her with
violation of PD 1866 was filed with the Regional Trial Court of Pasig, Metro
Manila. The case is docketed therein as Criminal Case No. 73447. Danny Rivera,
on the other hand, was released from custody.

On 17 May 1988, a petition for habeas corpus was filed, with this Court on behalf
of Vicky Ocaya and Danny Rivera. It was alleged therein that Vicky Ocaya was
illegally arrested and detained, and denied the right to a preliminary investigation.

It would appear, however, that Vicky Ocaya was arrested in flagranti delicto so
that her arrest without a warrant is justified. No preliminary investigation was
conducted because she was arrested without a warrant and she refused to waive the
provisions of Article 125 of the Revised Penal Code, pursuant to Sec. 7, Rule 112
of the Rule of Court, as amended.

The petitioners Vicky Ocaya, Domingo Anonuevo, Ramon Casiple, and Amelia
Roque claim that the firearms, ammunition and subversive documents alleged to
have been found in their possession when they were arrested, did not belong to
them, but were "planted" by the military agents to justify their illegal arrest.

The petitioners, however, have not introduced any evidence to support their
aforesaid claim. On the other hand, no evil motive or ill-will on the part of the
arresting officers that would cause the said arresting officers in these cases to
accuse the petitioners falsely, has been shown. Besides, the arresting officers in
these cases do not appear to be seekers of glory and bounty hunters for, as counsel
for the petitioners Anonuevo and Casiple say, "there is absolutely nothing in the
evidence submitted during the inquest that petitioners are on the 'AFP Order of
Battle with a reward of P150,000.00 each on their heads.'" 6 On the other hand, as
pointed out by the Solicitor General, the arrest of the petitioners is not a product of
a witch hunt or a fishing expedition, but the result of an in-depth surveillance of
NPA safehouses pointed to by no less than former comrades of the petitioners in
the rebel movement.

The Solicitor General, in his Consolidated Memorandum, aptly observes:

. . . . To reiterate, the focal point in the case of petitioners Roque,


Buenaobra, Anonuevo and Casiple, was the lawful search and seizure
conducted by the military at the residence of Renato Constantino at
Villaluz Compound, Molave St., Marikina Heights, Marikina, Metro
Manila. The raid at Constantino's residence, was not a witch hunting
or fishing expedition on the part of the military. It was a result of an
in-depth military surveillance coupled with the leads provided by
former members of the underground subversive organizations. That
raid produced positive results. to date, nobody has disputed the fact
that the residence of Constantino when raided yielded communication
equipment, firearms and ammunitions, as well as subversive
documents.

The military agents working on the information provided by


Constantino that other members of his group were coming to his
place, reasonably conducted a "stake-out" operation whereby some
members of the raiding team were left behind the place. True enough,
barely two hours after the raid and Constantino's arrest, petitioner
Buenaobra arrived at Constantino's residence. He acted suspiciously
and when frisked and searched by the military authorities, found in
his person were letters. They are no ordinary letters, as even a cursory
reading would show. Not only that, Buenaobra admitted that he is a
NPA courier and was there to deliver the letters to Constantino.

Subsequently, less than twenty four hours after the arrest of


Constantino and Buenaobra, petitioners Anonuevo and Casiple
arrived at Constantino's place. Would it be unreasonable for the
military agents to believe that petitioners Anonuevo and Casiple are
among those expected to visit Constantino's residence considering
that Constatino's information was true, in that Buenaobra did come to
that place? Was it unreasonable under the circumstances, on the part
of the military agents, not to frisk and search anyone who should visit
the residence of Constantino, such as petitioners Anonuevo and
Casiple? Must this Honorable Court yield to Anonuevo and Casiple's
flimsy and bare assertion that they went to visit Constantino, who was
to leave for Saudi Arabia on the day they were arrested thereat?

As to petitioner Roque, was it unreasonable for the military


authorities to effect her arrest without warrant considering that it was
Buenaobra who provided the leads on her identity? It cannot be
denied that Buenaobra had connection with Roque. Because the
former has the phone number of the latter. Why the necessity of
jumbling Roque's telephone number as written on a piece of paper
taken from Buenaobra's possession? Petitioners Roque and
Buenaobra have not offered any plausible reason so far.

In all the above incidents, respondents maintain that they acted


reasonably, under the time, place and circumstances of the events in
question, especially considering that at the time of petitioner's arrest,
incriminatory evidence, i.e, firearms, ammunitions and/or subversive
documents were found in their possession.

Petitioners, when arrested, were neither taking their snacks nor


innocently visiting a camp, but were arrested in such time, place and
circumstances, from which one can reasonably conclude tat they were
up to a sinister plot, involving utmost secrecy and comprehensive
conspiracy.

IV

In. G.R. No. 85727 (Espiritu vs. Lim), the release on habeas corpus of the
petitioner Deogracias Espiritu, who is detained by virtue of an Information for
Violation of Article 142 of the Revised Penal Code (Inciting to Sedition) filed with
the Regional Trial Court of Manila, is similarly not warranted.

The record of the case shows that the said petitioner is the General Secretary of the
Pinagkaisahang Samahan ng Tsuper at Operators Nationwide (PISTON), an
association of drivers and operators of public service vehicles in the Philippines,
organized for their mutual aid and protection.

Petitioner claims that at about 5:00 o'clock in the morning of 23 November 1988,
while he was sleeping in his home located at 363 Valencia St., Sta. Mesa, Manila,
he was awakened by his sister Maria Paz Lalic who told him that a group of
persons wanted to hire his jeepney. When he went down to talk to them, he was
immediately put under arrest. When he asked for the warrant of arrest, the men,
headed by Col. Ricardo Reyes, bodily lifted him and placed him in their owner-
type jeepney. He demanded that his sister, Maria Paz Lalic, be allowed to
accompany him, but the men did not accede to his request and hurriedly sped
away.
He was brought to Police Station No. 8 of the Western Police District at
Blumentritt, Manila where he was interrogated and detained. Then, at about 9:00
o'clock of the same morning, he was brought before the respondent Lim and, there
and then, the said respondent ordered his arrest and detention. He was thereafter
brought to the General Assignment Section, Investigation Division of the Western
Police District under Police Capt. Cresenciano A. Cabasal where he was detained,
restrained and deprived of his liberty. 7

The respondents claim however, that the detention of the petitioner is justified in
view of the Information filed against him before the Regional Trial Court of
Manila, docketed therein as Criminal Case No. 88-683-85, charging him with
violation of Art. 142 of the Revised Penal Code (Inciting to Sedition).

The respondents also claim that the petitioner was lawfully arrested without a
judicial warrant of arrest since petitioner when arrested had in fact just committed
an offense in that in the afternoon of 22 November 1988, during a press conference
at the National Press Club.

Deogracias Espiritu through tri-media was heard urging all drivers


and operators to go on nationwide strike on November 23, 1988, to
force the government to give into their demands to lower the prices of
spare parts, commodities, water and the immediate release from
detention of the president of the PISTON (Pinag-isang Samahan ng
Tsuper Operators Nationwide). Further, we heard Deogracias Espiritu
taking the place of PISTON president Medardo Roda and also
announced the formation of the Alliance Drivers Association to go on
nationwide strike on November 23, 1988. 8

Policemen waited for petitioner outside the National Pres Club in order to
investigate him, but he gave the lawmen the slip. 9 He was next seen at about 5:00
o'clock that afternoon at a gathering of drivers and symphatizers at the corner of
Magsaysay Blvd. and Valencia Street, Sta. Mesa, Manila where he was heard to
say:

Bukas tuloy ang welga natin, sumagot na ang Cebu at Bicol na kasali
sila, at hindi tayo titigil hanggang hindi binibigay ng gobyerno ni
Cory ang gusto nating pagbaba ng halaga ng spare parts, bilihin at
and pagpapalaya sa ating pinuno na si Ka Roda hanggang sa
magkagulo na. 10 (emphasis supplied)

The police finally caught up with the petitioner on 23 November 1988. He was
invited for questioning and brought to police headquarters after which an
Information for violation of Art. 142 of the Revised Penal Code was filed against
him before the Regional Trial Court of Manila. 11
Since the arrest of the petitioner without a warrant was in accordance with the
provisions of Rule 113, Sec. 5(b) of the Rules of Court and that the petitioner is
detained by virtue of a valid information filed with the competent court, he may
not be released on habeas corpus. He may, however be released upon posting bail
as recommended. However, we find the amount of the recommended bail
(P60,000.00) excessive and we reduce it to P10,000.00 only.

VII

In G.R. No. 86332 (Nazareno vs. Station Commander), we also find no merit in the
submission of Narciso Nazareno that he was illegally arrested and is unlawfully
detained. The record of this case shows that at about 8:30 o'clock in the morning of
14 December 1988, one Romulo Bunye II was killed by a group of men near the
corner of T. Molina and Mendiola Streets in Alabang, Muntinglupa, Metro Manila.
One of the suspects in the killing was Ramil Regal who was arrested by the police
on 28 December 1988. Upon questioning, Regal pointed to Narciso Nazareno as on
of his companions in the killing of the said Romulo Bunye II. In view thereof, the
police officers, without warrant, picked up Narciso Nazareno and brought him to
the police headquarters for questioning. Obviously, the evidence of petitioner's
guilt is strong because on 3 January 1989, an information charging Narciso
Nazareno, Ramil Regala, and two (2) others, with the killing of Romulo Bunye II
was filed with the Regional Trial Court of Makati, Metro Manila. The case is
docketed therein as Criminal Case No. 731.

On 7 January 1989, Narciso Nazareno filed a motion to post bail, but the motion
was denied by the trial court in an order dated 10 January 1989, even as the motion
to post bail, earlier filed by his co-accused, Manuel Laureaga, was granted by the
same trial court.

On 13 January 1989, a petition for habeas corpus was filed with this Court on


behalf of Narciso Nazareno and on 13 January 1989, the Court issued the writ
of habeas corpus, returnable to the Presiding Judge of the Regional Trial Court of
Biñan, Laguna, Branch 24, ordering said court to hear the case on 30 January 1989
and thereafter resolve the petition.

At the conclusion of the hearing, or on 1 February 1989, the Presiding Judge of the
Regional Trial Court of Biñan, Laguna issued a resolution denying the petition
for habeas corpus, it appearing that the said Narciso Nazareno is in the custody of
the respondents by reason of an information filed against him with the Regional
Trial Court of Makati, Metro Manila which had taken cognizance of said case and
had, in fact, denied the motion for bail filed by said Narciso Nazareno (presumably
because of the strength of the evidence against him).

The findings of the Presiding Judge of the Regional Trial Court of Biñan, Laguna
are based upon the facts and the law. Consequently, we will not disturb the same.
Evidently, the arrest of Nazareno was effected by the police without warrant
pursuant to Sec. 5(b), Rule 113, Rules of Court after he was positively implicated
by his co-accused Ramil Regala in the killing of Romulo Bunye
II; and after investigation by the police authorities. As held in People
vs. Ancheta: 12

The obligation of an agent of authority to make an arrest by reason of


a crime, does not presuppose as a necessary requisite for the
fulfillment thereof, the indubitable existence of a crime. For the
detention to be perfectly legal, it is sufficient that the agent or person
in authority making the arrest has reasonably sufficient grounds to
believe the existence of an act having the characteristics of a crime
and that the same grounds exist to believe that the person sought to be
detained participated therein.

VIII

It is to be noted that, in all the petitions here considered, criminal charges have
been filed in the proper courts against the petitioners. The rule is, that if a person
alleged to be restrained of his liberty is in the custody of an officer under process
issued by a court judge, and that the court or judge had jurisdiction to issue the
process or make the order, of if such person is charged before any court, the writ
of habeas corpus will not be allowed. Section 4, Rule 102, Rules of Court, as
amended is quite explicit in providing that:

Sec. 4. When writ is allowed or discharge authorized. — If it appears


that the person alleged to be restrained of his liberty is in the custody
of an officer under process issued by a court or judge or by virtue of a
judgment or order of a court of record, and that the court or judge had
jurisdiction to issue the process, render the judgment, or make the
order, the writ shall not be allowed; or if the jurisdiction appears after
the writ is allowed, the person shall not be discharged by reason of
any informality or defect in the process, judgment, or order. Nor shall
anything in this rule be held to authorize the discharge of a person
charged with a convicted of an offense in the Philippines or of a
person suffering imprisonment under lawful judgment. (emphasis
supplied)

At this point, we refer to petitioner's plea for the Court of re-examine and,
thereafter, abandon its pronouncement in Ilagan vs. Enrile, 13 that a writ of habeas
corpus is no longer available after an information is filed against the person
detained and a warrant of arrest or an order of commitment, is issued by the court
where said information has been filed. 14 The petitioners claim that the said ruling,
which was handed down during the past dictatorial regime to enforce and
strengthen said regime, has no place under the present democratic dispensation and
collides with the basic, fundamental, and constitutional rights of the people.
Petitioners point out that the said doctrine makes possible the arrest and detention
of innocent persons despite lack of evidence against them, and, most often, it is
only after a petition for habeas corpus is filed before the court that the military
authorities file the criminal information in the courts of law to be able to hide
behind the protective mantle of the said doctrine. This, petitioners assert, stands as
an obstacle to the freedom and liberty of the people and permits lawless and
arbitrary State action.

We find, however, no compelling reason to abandon the said doctrine. It is based


upon express provision of the Rules of Court and the exigencies served by the law.
The fears expressed by the petitioners are not really unremediable. As the Court
sees it, re-examination or reappraisal, with a view to its abandonment, of the Ilagan
case doctrine is not the answer. The answer and the better practice would be, not to
limit the function of the habeas corpus to a mere inquiry as to whether or not the
court which issued the process, judgment or order of commitment or before whom
the detained person is charged, had jurisdiction or not to issue the process,
judgment or order or to take cognizance of the case, but rather, as the Court itself
states in Morales, Jr. vs. Enrile, 15 "in all petitions for habeas corpus the court
must inquire into every phase and aspect of petitioner's detention-from the moment
petition was taken into custody up to the moment the court passes upon the merits
of the petition;" and "only after such a scrutiny can the court satisfy itself that the
due process clause of our Constitution has in fact been satisfied." This is exactly
what the Court has done in the petitions at bar. This is what should henceforth be
done in all future cases of habeas corpus. In Short, all cases involving deprivation
of individual liberty should be promptly brought to the courts for their immediate
scrutiny and disposition.

WHEREFORE, the petitions are hereby DISMISSED, except that in


G.R. No. 85727 (Espiritu vs. Lim), the bail bond for petitioner's provisional liberty
is hereby ordered reduced from P60,000.00 to P10,000.00. No costs.

SO ORDERED.

Fernan C.J., Narvasa, C.J., Melencio-Herrera, Gutierrez, Jr., Paras, Gancayco,


Padilla, Bidin, Griño-Aquino, Medialdea and Regalado, JJ., concur.

Separate Opinions
 

CRUZ, J., dissenting and concurring:

I dissent insofar as the ponencia affirms the ruling in Garcia-Padilla v. Enrile that


subversion is a continuing offense, to justify the arrest without warrant of any
person at any time as long as the authorities say he has been placed under
surveillance on suspicion of the offense. That is a dangerous doctrine. A person
may be arrested when he is doing the most innocent acts, as when he is only
washing his hands, or taking his supper, or even when he is sleeping, on the ground
that he is committing the "continuing" offense of subversion. Libertarians were
appalled when that doctrine was imposed during the Marcos regime. I am alarmed
that even now this new Court is willing to sustain it. I strongly urge my colleagues
to discard it altogether as one of the disgraceful vestiges of the past dictatorship
and uphold the rule guaranteeing the right of the people against unreasonable
searches and seizures. We can do no less if we are really to reject the past
oppression and commit ourselves to the true freedom. Even if it be argued that the
military should be given every support in our fight against subversion, I maintain
that that fight must be waged honorably, in accordance with the Bill of Rights. I do
not believe that in fighting the enemy we must adopt the ways of the enemy, which
are precisely what we are fighting against. I submit that our more important
motivation should be what are we fighting for.

Except for this reservation and appeal, I concur with the decision.

FELICIANO, J., concurring:

I concur in the result reached in each of the eight (8) consolidated Petitions
for Habeas Corpus. At the same time, I have some reservations concerning certain
statements made by the Court in G.R. No. 81567 (Umil, et al. v. Ramos) (Part I of
the Decision) and in G.R. No. 85727 (Espiritu v. Lim) (Part VI of the Decision).

In G.R. No. 81567 (Umil, et al. v. Ramos), the per curiam opinion states


categorically that: "the crimes of rebellion, subversion, conspiracy or proposal to
commit such crimes, and crimes or offenses committed in furtherance thereof or in
connection therewith constitute direct assaults against the State and are in the
nature of continuing crimes." The majority here relies upon Garcia-Padilla
v. Enrile (121 SCRA 472 [1983]). The majority there made the same equally broad
statement but without any visible effort to examine the basis, scope and meaning of
such a sweeping statement. Garcia-Padilla did not even identify the specific
offenses which it regarded as "in the nature of continuing offenses which set them
apart from the common offenses" (121 SCRA at 489). It appears to me that in G.R.
No. 85727 (Espiritu v. Lim) (Part VI of the Decision), the per curiam opinion has
in effect included the offense of "inciting to sedition" penalized under Article 142
of the Revised Penal Code as a "continuing offense" under the capacious blanket of
the majority opinion in Garcia-Padilla, at least for purposes of determining the
legality of the arrest without a warrant of petitioner Deogracias Espiritu.

I would respectfully recall to my learned colleagues in the Court that "inciting to


sedition" is defined in Article 142 of the Revised Penal Code in terms of
speech 1 and that consequently it is important constantly do distinguish between
speech which is protected by the constitutional guaranty of freedom of speech and
of the press and speech which may constitutionally be regarded as violative of
Article 142 of the Revised Penal Code. Precisely because speech which the police
authorities might regard as seditious or as criminal inciting to sedition may well
turn out to be only an exercise of a constitutionally guaranteed freedom, I would
submit that we must apply the concept of "continuing offense" narrowly for
purposes of application of Section 5(b), Rule 113 of the Revised Rules of Court.

In my view, the very broad statement made about "continuing crimes" in G.R. No.
81567 (Umil, et al v. Ramos) constitutes dictum, considering that Rolando Dural
and Bernardo Itucal, Jr. had already been tried in the court below for "double
murder, etc." and found guilty of the offense charged, sentenced accordingly, and
at least in the case of Rolando Dural, service of the sentence imposed upon him by
the trial court had already begun.

Similarly, in G.R. No. 85727 (Espiritu v. Lim) the statement that the arrest of
petitioner Espiritu without a warrant was in accordance with the provisions of
Section 5(b), Rule 113 of the Revised Rules of Court does not appear strictly
necessary, considering that the petitioner had already been charged in a valid
information filed with the competent court, which court had presumably issued an
order for his commitment, and considering further that he is entitled to bail.

There is thus no obstacle, to my mind, to a careful examination of the doctrine of


"continuing crimes" as applied to such offenses as subversion and inciting to
sedition and possibly other offenses, in some future case where that issue is raised
squarely and is unavoidable.

Cortes, J., concurs.

SARMIENTO, J., dissenting:

I beg to differ from my brethren. I submit that habeas corpus lies in all eight cases.

G.R. No. 81567


The majority says that Rolando Dural's arrest without a warrant is lawful under the
Rules of Court, which reads:

Sec. 5. Arrest without warrant; when lawful. — A peace officer or a


private person may, without a warrant, arrest a person:

(a) When, in his presence, the person to be arrested has committed, is


actually committing, or is attempting to commit an offense;

(b) When an offense has in fact just been committed, and he has
personal knowledge of facts indicating that the person to be arrested
has committed it; and

(c) When the person to be arrested is a prisoner who has escaped from
a penal establishment or place where he is serving final judgment or
temporarily confined while his case is pending, or has escaped while
being transferred from one confinement to another.

In cases falling under paragraphs (a) and (b) hereof, the person arrested without a
warrant shall be forthwith delivered to the nearest police station or jail, and he shall
be proceeded against in accordance with Rule 112, Section 7. 1

"Rolando Dural," so states the majority, "was arrested for being a member of the
New People's Army (NPA), an outlawed subversive organization," 2 and that
"[s]ubversion being a continuing offense, the arrest of Rolando Dural without a
warrant is justified as it can be said that he was committing an offense when
arrested." 3

As I said, I beg to differ.

First, Rolando Dural was charged with "Double Murder with Assault upon Agents
of Authority." 4 If he had been guilty of subversion — the offense for which he was
supposedly arrested via a warrantless arrest — subversion was the logical crime
with which he should have been charged.

The authorities could not have rightly arrested him for subversion on account of
the slay of the two CAPCOM soldiers, a possible basis for violation of the Anti-
Subversion Act, because as the majority points out, "he was not arrested while in
the act of shooting [them] . . . [n]or was he arrested just after the commission of the
said offense for his arrest came a day after the said shooting incident." 5

Second, I do not believe that a warrantless (or citizen's) arrest is possible in case of
subversion — in the absence of any overt act that would justify the authorities to
act. "Subversion," as the term is known in law, means "knowingly, wilfully and by
overt acts affiliat[ing] [oneself] with, becom[ing] or remain[ing] a member of the
Communist Party of the Philippines and/or its successor or of any subversion
association as defined in sections two and three hereof. . . . " 6 Logically, the
military could not have known that Dural, at the time he was taken, was a member
of the New People's Army because he was not performing any over act that he was
truly, a rebel. Indeed, it had to take a "verification" 6 before he could be identified
as allegedly a member of the underground army. Under these circumstances, I am
hard put to say that he was committing subversion when he was arrested, assuming
that he was guilty of subversion, for purposes of a warrantless arrest.

"Overt act" is made up of "[e]very act, movement, deed and word of


the [accused]," 7 indicating intent to accomplish a criminal objective.
Dural, at the time he was arrested, was lying in a hospital bed. This is
not the overt act contemplated by law.

Under the Rule above-quoted, the person must have either been apprehended in
flagranti (first paragraph) or after the act, provided that the peace officer has
"personal knowledge" that he, the suspect, is guilty. (second paragraph.) As I
stated, Dural was not caught in the act. Moreover, what the Regional Intelligence
Operations Unit of the Capital Command (RIOU-CAPCOM) had in its hands was
a mere "confidential information." I do not think that this is the personal
knowledge referred to by the second paragraph. 8 Plainly and simply, it is hearsay.

The rule, furthermore, on warrantless arrest is an exceptional one. By its language,


it may be exercised only in the most urgent cases and when the guilt of an offender
is plain and evident. What I think we have here is purely and simply, the military
taking the law in its hands.

By stamping validity to Rolando Dural's warrantless arrest, I am afraid that the


majority has set a very dangerous precedent. With all due respect, my brethren has
accorded the military a blanket authority to pick up any Juan, Pedro, and Maria
without a warrant for the simple reason that subversion is supposed to be a
continuing offense.

That Rolando Dural was arrested for being a member of the New People's
Army" 9 is furthermore to me, a hasty statement. It has yet to be established that
Dural is indeed a member of the Communist Party's military arm. And unless
proven guilty, he is presumed, and must be presumed most of all by this Court, to
be innocent.

The majority also says that habeas corpus is moot and academic because Dural has
been convicted and is serving sentence. I likewise take exception. It has been held
that: "The writ may be granted upon a judgment already final." 10

The writ of liberty is a high prerogative writ. 11 Vindication of due process is its


historic office. 12
G.R. Nos. 84581-82

In the case of Wilfredo Buenaobra, the majority avers that he had "manifested his
desire to stay in the PC-INP stockade," 13 for which habeas corpus has supposedly
become moot and academic. I am not convinced that that is reason enough to
dismiss habeas corpus as moot and academic. It is the duty of this Court, in my
opinion, to make sure that Buenaobra has made his choice freely and voluntarily.
Personally, I find it indeed strange why he should prefer to stay in jail than go scot-
free.

There is further no doubt that Buenaobra's petition is one impressed with a public
interest. In one case 14 we denied a motion to withdraw a petition for habeas
corpus in view of its far-reaching importance to the motion, I do not see how we
should act differently, perhaps even insouciantly, here, especially since it involves
persons who think and believe differently from the rest of us.

Both Buenaobra and Amelia Roque supposedly admitted that they were ranking
officers of the Communist Party of the Philippines. According to the majority,
Buenaobra and Roque are bound by their admissions. 15

That both parties had admitted to be members of the Communist Party of the
Philippines (the National United Front Commission) is a naked contention of the
military. The fact that it has not been controverted, in my view, does not justify the
couple's arrest without warrant. Worse, by relying on the bare word of the military,
this very Court has, to all intents and purposes, condemned the duo for a crime
(subversion and/or illegal possession of firearms) the bone of contention, precisely,
below.

G.R. Nos. 84583-84

I also find the warrantless arrests of Domingo Añonuevo and Ramon Casiple to be
contrary to law. That they are "admittedly members of the standing committee of
the NUFC" 16 and that "subversive materials" 17 and unlicensed firearms were
found in their possession, are, like Buenaobra's and Roque's cases, barren claims of
the military. I also fear that by the majority's strong language (that Añonuevo and
Casiple are admitted NUCF officers) the majority has pronounced the petitioners
guilty, when the lower courts have yet to sit in judgment. I think we should be the
last to preempt the decision of the trial courts. We would have set to naught the
presumption of innocence accused persons enjoy.

G.R. No. 83162

With respect to the case of Vicky Ocaya, I am afraid that I am inclined towards the
same conclusion. There was basis — at the outset — to say that Ocaya was
probably guilty of illegal possession of firearms. As I have observed, a warrantless
arrest must be predicated upon the existence of a crime being actually committed
or having been committed. What I find here, rather, is nothing less than a
successful fishing expedition conducted by the military upon an unwary citizen. I
am quite distressed to note that this is still possible under a supposed democracy.

G.R. No. 85727

Deogracias Espiritu was fast asleep in his house when he was placed under arrest.
For the life of me, I can not figure out how one can be picked upon in one's own
home and held moments later without a warrant of arrest.

Espiritu was allegedly guilty of inciting to sedition as a result of a speech delivered


in a press conference at the National Press Club on November 21, 1988. He was,
however, arrested the day after, November 22, 1988. Under these circumstances, it
eludes me how an arrest without a warrant could be justified, either under
paragraph (a) or paragraph (b) of the Rule on warrantless arrests.

The majority avers that since an information had been filed with the court,
Espiritu's detention, is allegedly justifiable. The question is whether or not an
information is an authority to hold a person in custody. Under the Rules, an
information means "an accusation in writing charging a person with an offense
subscribed by the fiscal and filed with the court." 18 It is not, however, an order to
keep one under detention.

G.R. No. 86332

The offense for which Narciso Nazareno is being held — the fatal shooting of
Romulo Bunye II — was committed on December 14, 1988. It was, however, only
on December 28, 1988 that the police collared a suspect, Ramil Regala, who
subsequently pointed to Nazareno as his accomplice. It also escapes me how
Nazareno, under these circumstances, could have been validly put under arrest
without a warrant or the existence of the circumstance described under either
paragraph (a) or (b) of the Rule above-quoted: The crime had long been committed
prior to the arrest.

G.R. Nos. 81567; 84581-82; 84583-84; 83162;


85727 & 86332; Postscripts

The majority has disposed of these cases on the bedrock of what I view as
doctrines that have lost their luster:

1. The teaching of Garcia-Padilla v. Enrile, 19 which held that subversion is a


continuing offense;

2. The ruling in Ilagan v. Enrile. 20


I also find, for reasons to be set forth hereinafter, a glossing over of the
fundamental rights of the petitioners under the Constitution in the authorities'
handling of the petitioners' cases.

I hold that Garcia-Padilla is no longer good law under the present Constitution.


Two reasons persuade me. First, it is repugnant to due process of law. ("The arrest,
therefore, need not follow the usual procedure in the prosecution of offenses which
require the determination by a judge of the existence of probable cause before the
issuance of a judicial warrant of arrest and the granting of bail if the offense is
bailable." 21 Under the 1987 Constitution, not even "[a] state of martial law
suspend[s] the operation of [the Charter]. . ." 22 Second, it leaves the liberty of
citizens to the whim of one man ("On these occasions [the existence of a state of
emergency], the President takes absolute command, for the very life of the Nation
and its government, which, incidentally, includes the courts, is in grave peril. In so
doing, the President is answerable only to his conscience, the people and to God.
For their part, in giving him the supreme mandate as their President, the people can
only trust and pray that, giving him their own loyalty and without patriotism, the
President will not fail them." 23 ) Under the Charter now prevailing, the Chief
Executive shares, to a certain extent, the exercise of emergency powers, with
Congress. 24

As a law advocate under the regime of Marcos, I had challenged the soundness
of Garcia-Padilla. I doubted whether it could stand up under the aegis of the 1973
Constitution. I still doubt whether it can withstand scrutiny under the 1987
Constitution.

The majority also fails to point out that six days after Garcia-Padilla was handed
down, the Court promulgated Morales, Jr. v. Enrile, 25 a case that in my view has
significantly whittled down Garcia-Padilla's very esse. In that case, Mr. Justice
Hermogenes Concepcion, Jr. wrote for the majority:

xxx xxx xxx

16. After a person is arrested . . . without a warrant . . . the proper


complaint or information against him must be filed with the courts of
justice within the time prescribed by law. . .

17. Failure of the public officer to do so without any valid reason


would constitute a violation of Art. 125, Revised Penal Code, as
amended. And the person detained would be entitled to be released on
a writ of habeas corpus, unless he is detained under subsisting
process issued by a competent court. 26

I also gather from the records that none of the petitioners had been: (1) informed of
their right to remain silent; and (2) to have competent and independent counsel. 27
As I said, the majority is denying habeas corpus on self-serving claims of the
military that the petitioners (Dural, Buenaobra, Roque, Añonuevo, and Casiple) are
members of the Communist Party of the Philippines — and that they have
supposedly confessed to be in fact members of the outlawed organization. The
question that has not been answered is whether or not these supposed confessions
are admissible, for purposes of a warrantless arrest, as evidence of guilt, in the
absence of any showing that they were apprised of their constitutional rights. I am
perturbed by the silence of the majority. I am distressed because as we held in one
case, violation of the Constitution divests the court of jurisdiction and entitles the
accused to habeas corpus. 28

According to the majority, a "re-examination or re-appraisal . . . of


the Ilagan doctrine is not the answer." 29 In my considered opinion, Ilagan
v. Enrile 30 does not rightfully belong in the volumes of Philippine jurisprudence.
In that case, the petitioners, three Davao-based lawyers, were held by virtue of a
simple information ("the petition herein has been rendered moot and academic by
virtue of the filing of an Information against them for Rebellion . . . and the
issuance of a Warrant of Arrest against them" 31 ) without any preliminary
investigation (examination) having been previously conducted (to justify the
issuance of a warrant).i•t•c-aüsl As I have stated, an information is not a warrant
of arrest. The fact that an information exists does not mean that a warrant will be
issued.

Accused persons have the right of preliminary investigation (examination). 32 It


forms part and parcel of due process of law .33

I find the majority's reliance on U.S. v. Wilson, 34 an ancient (1905) decision, inapt
and untenable. In that case, the accused had been served with a warrant and
thereafter taken into custody. The question that faced the Court was whether or not
the warrant was valid, amid the accused's charges that the judge who issued it did
not examine the complainant under oath. We held that the query was academic,
because the accused had already pleaded, and the case had entered the trial stage.

The cases at bar are not on all fours. Here, no warrant has been issued. I submit
that in that event, the petitioners are entitled to freedom by way of the writ of
liberty.

x x x           x x x          x x x

The apprehensions in question chronicle in my mind the increasing pattern of


arrests and detention in the country without the sanction of a judicial decree. Four
years ago at "EDSA", and many years before it, although with much fewer of us,
we valiantly challenged a dictator and all the evils his regime had stood for:
repression of civil liberties and trampling on of human rights. We set up a popular
government, restored its honored institutions, and crafted a democratic constitution
that rests on the guideposts of peace and freedom. I feel that with this Court's
ruling, we have frittered away, by a stroke of the pen, what we had so
painstakingly built in four years of democracy, and almost twenty years of struggle
against tyranny.

It also occurs to me that I am interposing what looms as a quixotic outlook of


Philippine law on warrantless arrests and its implications on liberty. It is an
impression that does not surprise me. Quixotic as they may seem, and modesty
aside, my views reflect a strong bias on my part — forged by years of experience
and sharpened by a painful and lonely struggle for freedom and justice — toward
men and women who challenge settled beliefs. If this dissent can not gain any
adherent for now, let it nevertheless go on record as a plea to posterity and an
appeal for tolerance of opinions with which we not only disagree, but opinions we
loathe.

I feel it is my duty to articulate this dissent.

Separate Opinions

CRUZ, J., dissenting and concurring:

I dissent insofar as the ponencia affirms the ruling in Garcia-Padilla v. Enrile that


subversion is a continuing offense, to justify the arrest without warrant of any
person at any time as long as the authorities say he has been placed under
surveillance on suspicion of the offense. That is a dangerous doctrine. A person
may be arrested when he is doing the most innocent acts, as when he is only
washing his hands, or taking his supper, or even when he is sleeping, on the ground
that he is committing the "continuing" offense of subversion. Libertarians were
appalled when that doctrine was imposed during the Marcos regime. I am alarmed
that even now this new Court is willing to sustain it. I strongly urge my colleagues
to discard it altogether as one of the disgraceful vestiges of the past dictatorship
and uphold the rule guaranteeing the right of the people against unreasonable
searches and seizures. We can do no less if we are really to reject the past
oppression and commit ourselves to the true freedom. Even if it be argued that the
military should be given every support in our fight against subversion, I maintain
that that fight must be waged honorably, in accordance with the Bill of Rights. I do
not believe that in fighting the enemy we must adopt the ways of the enemy, which
are precisely what we are fighting against. I submit that our more important
motivation should be what are we fighting for.

Except for this reservation and appeal, I concur with the decision.

 
FELICIANO, J., concurring:

I concur in the result reached in each of the eight (8) consolidated Petitions
for Habeas Corpus. At the same time, I have some reservations concerning certain
statements made by the Court in G.R. No. 81567 (Umil, et al. v. Ramos) (Part I of
the Decision) and in G.R. No. 85727 (Espiritu v. Lim) (Part VI of the Decision).

In G.R. No. 81567 (Umil, et al. v. Ramos), the per curiam opinion states


categorically that: "the crimes of rebellion, subversion, conspiracy or proposal to
commit such crimes, and crimes or offenses committed in furtherance thereof or in
connection therewith constitute direct assaults against the State and are in the
nature of continuing crimes." The majority here relies upon Garcia-Padilla
v. Enrile (121 SCRA 472 [1983]). The majority there made the same equally broad
statement but without any visible effort to examine the basis, scope and meaning of
such a sweeping statement. Garcia-Padilla did not even identify the specific
offenses which it regarded as "in the nature of continuing offenses which set them
apart from the common offenses" (121 SCRA at 489). It appears to me that in G.R.
No. 85727 (Espiritu v. Lim) (Part VI of the Decision), the per curiam opinion has
in effect included the offense of "inciting to sedition" penalized under Article 142
of the Revised Penal Code as a "continuing offense" under the capacious blanket of
the majority opinion in Garcia-Padilla, at least for purposes of determining the
legality of the arrest without a warrant of petitioner Deogracias Espiritu.

I would respectfully recall to my learned colleagues in the Court that "inciting to


sedition" is defined in Article 142 of the Revised Penal Code in terms of
speech 1 and that consequently it is important constantly do distinguish between
speech which is protected by the constitutional guaranty of freedom of speech and
of the press and speech which may constitutionally be regarded as violative of
Article 142 of the Revised Penal Code. Precisely because speech which the police
authorities might regard as seditious or as criminal inciting to sedition may well
turn out to be only an exercise of a constitutionally guaranteed freedom, I would
submit that we must apply the concept of "continuing offense" narrowly for
purposes of application of Section 5(b), Rule 113 of the Revised Rules of Court.

In my view, the very broad statement made about "continuing crimes" in G.R. No.
81567 (Umil, et al v. Ramos) constitutes dictum, considering that Rolando Dural
and Bernardo Itucal, Jr. had already been tried in the court below for "double
murder, etc." and found guilty of the offense charged, sentenced accordingly, and
at least in the case of Rolando Dural, service of the sentence imposed upon him by
the trial court had already begun.

Similarly, in G.R. No. 85727 (Espiritu v. Lim) the statement that the arrest of
petitioner Espiritu without a warrant was in accordance with the provisions of
Section 5(b), Rule 113 of the Revised Rules of Court does not appear strictly
necessary, considering that the petitioner had already been charged in a valid
information filed with the competent court, which court had presumably issued an
order for his commitment, and considering further that he is entitled to bail.

There is thus no obstacle, to my mind, to a careful examination of the doctrine of


"continuing crimes" as applied to such offenses as subversion and inciting to
sedition and possibly other offenses, in some future case where that issue is raised
squarely and is unavoidable.

Cortes, J., concurs.

SARMIENTO, J., dissenting:

I beg to differ from my brethren. I submit that habeas corpus lies in all eight cases.

G.R. No. 81567

The majority says that Rolando Dural's arrest without a warrant is lawful under the
Rules of Court, which reads:

Sec. 5. Arrest without warrant; when lawful. — A peace officer or a


private person may, without a warrant, arrest a person:

(a) When, in his presence, the person to be arrested has committed, is


actually committing, or is attempting to commit an offense;

(b) When an offense has in fact just been committed, and he has
personal knowledge of facts indicating that the person to be arrested
has committed it; and

(c) When the person to be arrested is a prisoner who has escaped from
a penal establishment or place where he is serving final judgment or
temporarily confined while his case is pending, or has escaped while
being transferred from one confinement to another.

In cases falling under paragraphs (a) and (b) hereof, the person arrested without a
warrant shall be forthwith delivered to the nearest police station or jail, and he shall
be proceeded against in accordance with Rule 112, Section 7. 1

"Rolando Dural," so states the majority, "was arrested for being a member of the
New People's Army (NPA), an outlawed subversive organization," 2 and that
"[s]ubversion being a continuing offense, the arrest of Rolando Dural without a
warrant is justified as it can be said that he was committing an offense when
arrested." 3
As I said, I beg to differ.

First, Rolando Dural was charged with "Double Murder with Assault upon Agents
of Authority." 4 If he had been guilty of subversion — the offense for which he was
supposedly arrested via a warrantless arrest — subversion was the logical crime
with which he should have been charged.

The authorities could not have rightly arrested him for subversion on account of
the slay of the two CAPCOM soldiers, a possible basis for violation of the Anti-
Subversion Act, because as the majority points out, "he was not arrested while in
the act of shooting [them] . . . [n]or was he arrested just after the commission of the
said offense for his arrest came a day after the said shooting incident." 5

Second, I do not believe that a warrantless (or citizen's) arrest is possible in case of
subversion — in the absence of any overt act that would justify the authorities to
act. "Subversion," as the term is known in law, means "knowingly, wilfully and by
overt acts affiliat[ing] [oneself] with, becom[ing] or remain[ing] a member of the
Communist Party of the Philippines and/or its successor or of any subversion
association as defined in sections two and three hereof. . . . " 6 Logically, the
military could not have known that Dural, at the time he was taken, was a member
of the New People's Army because he was not performing any over act that he was
truly, a rebel. Indeed, it had to take a "verification" 6 before he could be identified
as allegedly a member of the underground army. Under these circumstances, I am
hard put to say that he was committing subversion when he was arrested, assuming
that he was guilty of subversion, for purposes of a warrantless arrest.

"Overt act" is made up of "[e]very act, movement, deed and word of


the [accused]," 7 indicating intent to accomplish a criminal objective.
Dural, at the time he was arrested, was lying in a hospital bed. This is
not the overt act contemplated by law.

Under the Rule above-quoted, the person must have either been apprehended in
flagranti (first paragraph) or after the act, provided that the peace officer has
"personal knowledge" that he, the suspect, is guilty. (second paragraph.) As I
stated, Dural was not caught in the act. Moreover, what the Regional Intelligence
Operations Unit of the Capital Command (RIOU-CAPCOM) had in its hands was
a mere "confidential information." I do not think that this is the personal
knowledge referred to by the second paragraph. 8 Plainly and simply, it is hearsay.

The rule, furthermore, on warrantless arrest is an exceptional one. By its language,


it may be exercised only in the most urgent cases and when the guilt of an offender
is plain and evident. What I think we have here is purely and simply, the military
taking the law in its hands.
By stamping validity to Rolando Dural's warrantless arrest, I am afraid that the
majority has set a very dangerous precedent. With all due respect, my brethren has
accorded the military a blanket authority to pick up any Juan, Pedro, and Maria
without a warrant for the simple reason that subversion is supposed to be a
continuing offense.

That Rolando Dural was arrested for being a member of the New People's
Army" 9 is furthermore to me, a hasty statement. It has yet to be established that
Dural is indeed a member of the Communist Party's military arm. And unless
proven guilty, he is presumed, and must be presumed most of all by this Court, to
be innocent.

The majority also says that habeas corpus is moot and academic because Dural has
been convicted and is serving sentence. I likewise take exception. It has been held
that: "The writ may be granted upon a judgment already final." 10

The writ of liberty is a high prerogative writ. 11 Vindication of due process is its


historic office. 12

G.R. Nos. 84581-82

In the case of Wilfredo Buenaobra, the majority avers that he had "manifested his
desire to stay in the PC-INP stockade," 13 for which habeas corpus has supposedly
become moot and academic. I am not convinced that that is reason enough to
dismiss habeas corpus as moot and academic. It is the duty of this Court, in my
opinion, to make sure that Buenaobra has made his choice freely and voluntarily.
Personally, I find it indeed strange why he should prefer to stay in jail than go scot-
free.

There is further no doubt that Buenaobra's petition is one impressed with a public
interest. In one case 14 we denied a motion to withdraw a petition for habeas
corpus in view of its far-reaching importance to the motion, I do not see how we
should act differently, perhaps even insouciantly, here, especially since it involves
persons who think and believe differently from the rest of us.

Both Buenaobra and Amelia Roque supposedly admitted that they were ranking
officers of the Communist Party of the Philippines. According to the majority,
Buenaobra and Roque are bound by their admissions. 15

That both parties had admitted to be members of the Communist Party of the
Philippines (the National United Front Commission) is a naked contention of the
military. The fact that it has not been controverted, in my view, does not justify the
couple's arrest without warrant. Worse, by relying on the bare word of the military,
this very Court has, to all intents and purposes, condemned the duo for a crime
(subversion and/or illegal possession of firearms) the bone of contention, precisely,
below.

G.R. Nos. 84583-84

I also find the warrantless arrests of Domingo Añonuevo and Ramon Casiple to be
contrary to law. That they are "admittedly members of the standing committee of
the NUFC" 16 and that "subversive materials" 17 and unlicensed firearms were
found in their possession, are, like Buenaobra's and Roque's cases, barren claims of
the military. I also fear that by the majority's strong language (that Añonuevo and
Casiple are admitted NUCF officers) the majority has pronounced the petitioners
guilty, when the lower courts have yet to sit in judgment. I think we should be the
last to preempt the decision of the trial courts. We would have set to naught the
presumption of innocence accused persons enjoy.

G.R. No. 83162

With respect to the case of Vicky Ocaya, I am afraid that I am inclined towards the
same conclusion. There was basis — at the outset — to say that Ocaya was
probably guilty of illegal possession of firearms. As I have observed, a warrantless
arrest must be predicated upon the existence of a crime being actually committed
or having been committed. What I find here, rather, is nothing less than a
successful fishing expedition conducted by the military upon an unwary citizen. I
am quite distressed to note that this is still possible under a supposed democracy.

G.R. No. 85727

Deogracias Espiritu was fast asleep in his house when he was placed under arrest.
For the life of me, I can not figure out how one can be picked upon in one's own
home and held moments later without a warrant of arrest.

Espiritu was allegedly guilty of inciting to sedition as a result of a speech delivered


in a press conference at the National Press Club on November 21, 1988. He was,
however, arrested the day after, November 22, 1988. Under these circumstances, it
eludes me how an arrest without a warrant could be justified, either under
paragraph (a) or paragraph (b) of the Rule on warrantless arrests.

The majority avers that since an information had been filed with the court,
Espiritu's detention, is allegedly justifiable. The question is whether or not an
information is an authority to hold a person in custody. Under the Rules, an
information means "an accusation in writing charging a person with an offense
subscribed by the fiscal and filed with the court." 18 It is not, however, an order to
keep one under detention.

G.R. No. 86332


The offense for which Narciso Nazareno is being held — the fatal shooting of
Romulo Bunye II — was committed on December 14, 1988. It was, however, only
on December 28, 1988 that the police collared a suspect, Ramil Regala, who
subsequently pointed to Nazareno as his accomplice. It also escapes me how
Nazareno, under these circumstances, could have been validly put under arrest
without a warrant or the existence of the circumstance described under either
paragraph (a) or (b) of the Rule above-quoted: The crime had long been committed
prior to the arrest.

G.R. Nos. 81567; 84581-82; 84583-84; 83162;


85727 & 86332; Postscripts

The majority has disposed of these cases on the bedrock of what I view as
doctrines that have lost their luster:

1. The teaching of Garcia-Padilla v. Enrile, 19 which held that subversion is a


continuing offense;

2. The ruling in Ilagan v. Enrile. 20

I also find, for reasons to be set forth hereinafter, a glossing over of the
fundamental rights of the petitioners under the Constitution in the authorities'
handling of the petitioners' cases.

I hold that Garcia-Padilla is no longer good law under the present Constitution.


Two reasons persuade me. First, it is repugnant to due process of law. ("The arrest,
therefore, need not follow the usual procedure in the prosecution of offenses which
require the determination by a judge of the existence of probable cause before the
issuance of a judicial warrant of arrest and the granting of bail if the offense is
bailable." 21 Under the 1987 Constitution, not even "[a] state of martial law
suspend[s] the operation of [the Charter]. . ." 22 Second, it leaves the liberty of
citizens to the whim of one man ("On these occasions [the existence of a state of
emergency], the President takes absolute command, for the very life of the Nation
and its government, which, incidentally, includes the courts, is in grave peril. In so
doing, the President is answerable only to his conscience, the people and to God.
For their part, in giving him the supreme mandate as their President, the people can
only trust and pray that, giving him their own loyalty and without patriotism, the
President will not fail them." 23 ) Under the Charter now prevailing, the Chief
Executive shares, to a certain extent, the exercise of emergency powers, with
Congress. 24

As a law advocate under the regime of Marcos, I had challenged the soundness
of Garcia-Padilla. I doubted whether it could stand up under the aegis of the 1973
Constitution. I still doubt whether it can withstand scrutiny under the 1987
Constitution.
The majority also fails to point out that six days after Garcia-Padilla was handed
down, the Court promulgated Morales, Jr. v. Enrile, 25 a case that in my view has
significantly whittled down Garcia-Padilla's very esse. In that case, Mr. Justice
Hermogenes Concepcion, Jr. wrote for the majority:

xxx xxx xxx

16. After a person is arrested . . . without a warrant . . . the proper


complaint or information against him must be filed with the courts of
justice within the time prescribed by law. . .

17. Failure of the public officer to do so without any valid reason


would constitute a violation of Art. 125, Revised Penal Code, as
amended. And the person detained would be entitled to be released on
a writ of habeas corpus, unless he is detained under subsisting
process issued by a competent court. 26

I also gather from the records that none of the petitioners had been: (1) informed of
their right to remain silent; and (2) to have competent and independent counsel. 27

As I said, the majority is denying habeas corpus on self-serving claims of the


military that the petitioners (Dural, Buenaobra, Roque, Añonuevo, and Casiple) are
members of the Communist Party of the Philippines — and that they have
supposedly confessed to be in fact members of the outlawed organization. The
question that has not been answered is whether or not these supposed confessions
are admissible, for purposes of a warrantless arrest, as evidence of guilt, in the
absence of any showing that they were apprised of their constitutional rights. I am
perturbed by the silence of the majority. I am distressed because as we held in one
case, violation of the Constitution divests the court of jurisdiction and entitles the
accused to habeas corpus. 28

According to the majority, a "re-examination or re-appraisal . . . of


the Ilagan doctrine is not the answer." 29 In my considered opinion, Ilagan
v. Enrile 30 does not rightfully belong in the volumes of Philippine jurisprudence.
In that case, the petitioners, three Davao-based lawyers, were held by virtue of a
simple information ("the petition herein has been rendered moot and academic by
virtue of the filing of an Information against them for Rebellion . . . and the
issuance of a Warrant of Arrest against them" 31 ) without any preliminary
investigation (examination) having been previously conducted (to justify the
issuance of a warrant).i•t•c-aüsl As I have stated, an information is not a warrant
of arrest. The fact that an information exists does not mean that a warrant will be
issued.

Accused persons have the right of preliminary investigation (examination). 32 It


forms part and parcel of due process of law .33
I find the majority's reliance on U.S. v. Wilson, 34 an ancient (1905) decision, inapt
and untenable. In that case, the accused had been served with a warrant and
thereafter taken into custody. The question that faced the Court was whether or not
the warrant was valid, amid the accused's charges that the judge who issued it did
not examine the complainant under oath. We held that the query was academic,
because the accused had already pleaded, and the case had entered the trial stage.

The cases at bar are not on all fours. Here, no warrant has been issued. I submit
that in that event, the petitioners are entitled to freedom by way of the writ of
liberty.

x x x           x x x          x x x

The apprehensions in question chronicle in my mind the increasing pattern of


arrests and detention in the country without the sanction of a judicial decree. Four
years ago at "EDSA", and many years before it, although with much fewer of us,
we valiantly challenged a dictator and all the evils his regime had stood for:
repression of civil liberties and trampling on of human rights. We set up a popular
government, restored its honored institutions, and crafted a democratic constitution
that rests on the guideposts of peace and freedom. I feel that with this Court's
ruling, we have frittered away, by a stroke of the pen, what we had so
painstakingly built in four years of democracy, and almost twenty years of struggle
against tyranny.

It also occurs to me that I am interposing what looms as a quixotic outlook of


Philippine law on warrantless arrests and its implications on liberty. It is an
impression that does not surprise me. Quixotic as they may seem, and modesty
aside, my views reflect a strong bias on my part — forged by years of experience
and sharpened by a painful and lonely struggle for freedom and justice — toward
men and women who challenge settled beliefs. If this dissent can not gain any
adherent for now, let it nevertheless go on record as a plea to posterity and an
appeal for tolerance of opinions with which we not only disagree, but opinions we
loathe.

I feel it is my duty to articulate this dissent.

David v. Arroyo, G.R. No. 171396, May 3, 2006


CASE: Umil v. Ramos, supra (concurring and dissenting opinion of J. Feliciano
and separate opinion of J. Regalado to the per curiam resolution)
I. Issue of legality of search and seizure or arrest

CASE: Stonehill v. Diokno, supra


CASE: People v. Salvatierra, G.R. No. 104663, July 24, 1997
For the fatal stabbing of Charlie Fernandez y De Guzman, the Regional Trial Court
of Manila, Branch XLIX,1 meted accused David Salvatierra y Eguia the penalty
of reclusion perpetua and the payment of the amounts of P30,183.25 as actual
damages and P50,000.00 as indemnity to the heirs of the victim, plus the costs of
suit (Criminal Case No. 90-88985).

At around 4:30 in the afternoon of August 17, 1990, Charlie Fernandez, a vendor
of "palamig" was walking along M. de la Fuente Street, 2 going towards the
direction of Quiapo at the opposite side of the street. 3 Suddenly, three (3) persons
met him. One of them was appellant David Salvatierra, who lunged a pointed
instrument at Charlie. The latter was able to parry the thrust but appellant swung
the instrument anew hitting Charlie at the left breast. Thereafter, all three persons
scampered away.4 Charlie still managed to walk home to tell his father about the
incident but suddenly collapsed.5 He was taken immediately to the hospital where
he was operated on.

The assault was witnessed by Milagros Martinez, an ambulant vendor of fish and
salted eggs who stopped by to rest at the right side of J. Fajardo St. in Sampaloc,
Manila near the Trabajo Market. Milagros told the incident only to her
daughter. 6 She did not immediately report the incident to the police authorities
because she was afraid.

Charlie's father, Marciano Fernandez, reported the crime to the police at Station
No. 4 at about 5:40 that afternoon. Since the victim could not be interviewed as he
was then undergoing operation, the police and Marciano Fernandez proceeded to
the crime scene to get information about the incident but their effort was fruitless
as no one in the area would volunteer to identify the culprits. 7 A relative of the
victim informed the police that appellant was one of the suspects in the crime. 8

The next day or on August 18, 1990, 20-year-old Charlie expired. His death was
caused by hemorrhage secondary to the stab wound on the anterior chest wall. 9 The
medical report prepared by Dr. Sergio Alteza, Jr., medico-legal officer of the
U.S.T. Hospital, showed that Charlie sustained, aside from the stab wound on the
chest, another stab wound on the left forearm and an incised wound on the left
wrist. 10

Marciano Fernandez went back to Police Station No. 4 to inform the authorities
that his son had died. He was advised to report the matter to the Homicide Section
of the Western Police District (WPD) 11 where an "advance information" was
prepared indicating that four (4) unidentified persons perpetrated the crime. 12

On November 15, 1990 at about 4:35 in the afternoon, Police Station No. 4
received a complaint that appellant was creating a commotion along Miguelin
Street, Sampaloc, Manila. He was thereby taken in custody by Pat. Celso Tan and
two other policemen who later found out that appellant was a suspect in the killing
of Charlie Fernandez. 13 Later that day, appellant was turned over to the WPD.

Milagros Martinez learned about the apprehension of appellant from her children.
Later, she was approached by Marciano Fernandez who persuaded her to testify on
what she witnessed on August 17, 1990. Upon being informed that appellant was
transferred to the WPD, the two proceeded to said station where Milagros executed
a sworn statement implicating appellant to the crime. 14 In a police line-up,
Milagros pinpointed appellant as the person who stabbed Charlie. Thereafter, Pat.
Amores prepared a booking sheet and arrest order which appellant signed. 15

On November 19, 1990, appellant was charged with murder in an information


which reads as follows:

That on or about August 17, 1990 in the City of Manila, Philippines,


the said accused, conspiring and confederating with three others
whose true names, identities and present whereabouts are still
unknown and helping one another, did then and there wilfully,
unlawfully and feloniously with intent to kill and with treachery and
evident premeditation, attack, assault and use personal violence upon
one CHARLIE FERNANDEZ Y DE GUZMAN, by then and there
stabbing the latter twice with a bladed weapon on the chest, thereby
inflicting upon him mortal stab wounds which were direct and
immediate cause of his death thereafter.

Contrary to law. 16

At his arraignment, appellant pleaded not guilty to the crime charged. 17

Appellant put up the defense of alibi alleging that at 4:30 in the afternoon of
August 17, 1990, he was having merienda with his wife and children at their home
in 459 Miguelin Street, Sampaloc, Manila and could not possibly be near the
Trabajo Market. 18

Appellant further testified that in the afternoon of November 15, 1990, he had an
altercation with a woman in their neighborhood who caused his arrest for the crime
of malicious mischief. He was detained for a few hours at Police Station No. 4.
Later, police from WPD arrived and picked him up and brought him to the
Homicide Section where he was investigated, interrogated and detained for the
stabbing of one Charlie Fernandez on August 17, 1990. 19 After two (2) days, he
was brought out of his cell where a man and two (2) women were made "to view"
him. One of the women was the mother of the victim while the other one was
someone he was not acquainted with. The latter was the witness against him who
pointed to him as the killer of Charlie in the police line-up. 20 Two days later, he
was made to sign a document the contents of which he was not allowed to read.
When he insisted on reading the document, his head was hit with a key and he was
forced to sign it. The document was the booking and information sheet. 21

In this appeal, he makes the following assignment of errors:

I. — THE COURT A QUO GRAVELY ERRED IN NOT FINDING


THAT THE ARREST, INVESTIGATION AND DETENTION OF
THE ACCUSED-APPELLANT FOR THE OFFENSE CHARGED
IN THE INSTANT CASE WAS VIOLATIVE OF HIS
CONSTITUTIONAL RIGHTS. 22

II. — THE COURT A QUO ERRED IN FINDING THAT


TREACHERY ATTENDED THE KILLING OF THE DECEASED
CHARLIE FERNANDEZ. 23

III. — THE COURT A QUO GRAVELY ERRED IN ACCEPTING


AT FACE VALUE THE VAGUE AND AMBIGUOUS
TESTIMONY OF MILAGROS MARTINEZ AND UTILIZING
SUCH INCONCLUSIVE TESTIMONY AS THE BASIS FOR
CONVICTING THE ACCUSED-APPELLANT OF THE CRIME OF
MURDER. 24

Anent the first error, appellant claims that his constitutional right against
warrantless arrests was violated because "(t)here is nothing on record to show that
(his) arrest . . . for the minor offense of malicious mischief was effected by virtue
of a warrant." 25 Indeed, appellant's arrest on suspicion that he was involved in the
killing of Charlie Fernandez was made almost three (3) months after the
commission of the crime on August 17, 1990 and only after he had been taken in
police custody for a minor offense. As such, because no warrant had been obtained
during the 3-month intervening period between the commission of the crime and
his apprehension, his arrest would have ordinarily been rendered unconstitutional
and illegal inasmuch as even warrantless arrests made within shorter periods like
ten (10) days 26 are illegal. The element of immediacy between the time of the
commission of the offense and the time of the arrest had not been complied with. It
should be stressed that Section 5(b) of Rule 113 of the Rules of Court has excluded
situations under the old rule which allowed a warrantless arrest provided that the
offense "has in fact been committed." 27

While these arguments may be valid, appellant's claim that the case against him
should be dismissed for violation of his constitutional rights, must fail. Appellant is
estopped from questioning the legality of his arrest considering that he never raised
this before entering his plea. Any objection involving a warrant of arrest or the
procedure in the acquisition of jurisdiction over the person of an accused must be
made before he enters his plea, otherwise, the objection is deemed waived. 28 This
is the first time that appellant is raising this issue as he did not even move for the
quashal of the information before the trial court on the ground of illegal
arrest. 29 Consequently, any irregularity attendant to his arrest, if any, had been
cured by his voluntary submission to the jurisdiction of the trial court when he
entered his plea and participated during the trial. 30 Verily, the illegal arrest of
appellant is not a sufficient cause for setting aside a valid judgment rendered upon
a sufficient complaint and where the trial was free from error. 31

Neither may appellant successfully assert that the case should be dismissed
because during custodial investigation and the police line-up he was deprived of
his constitutional right to counsel. To bolster his assertion, appellant
quotes People v. Campos 32 and People v. Vasquez, 33 where the Court in effect
held that during custodial investigation, an accused should be assisted by counsel.
Those cases, however, should be distinguished from the case at bar because in the
former, the extrajudicial confessions of the accused during custodial investigation
were the only bases for conviction, unlike in this case where there are other pieces
of evidence by the which the culpability of the appellant may be founded.

Moreover, in People v. Lamsing, 34 the Court categorically stated as follows:

Finally, although it is not assigned as error, accused-appellant


complains that he was made to join a police lineup where he was
identified by three persons, including Elizabeth De los Santos,
without the assistance of counsel. It was settled in Gamboa
v. Cruz (G.R. No. 56291, June 27, 1988, 162 SCRA 642, 651),
however, that the right to counsel guaranteed in Art. III, Section
12(1) of the Constitution does not extend to police lineups because
they are not part of custodial investigations. The reason for this is
that at that point, the process has not yet shifted from the
investigatory to the accusatory. The accused's right to counsel
attaches only from the time that adversary judicial proceedings are
taken against him. 35 (Emphasis supplied.)

In the same vein, appellant may not validly claim that dismissal of the case against
him should be a matter of course because he signed the booking and information
sheet without the assistance of counsel. Granting that affixing the signature of an
accused is covered by the constitutional mandate requiring assistance of counsel to
an accused during custodial investigation, this piece of evidence may be
disregarded without the least diluting the prosecution's case against appellant. The
booking and information sheet is not the only incriminatory evidence against
appellant.

What needs scrutiny is the testimony of eyewitness Milagros Martinez. In assailing


the testimony of Milagros, appellant points out that it is flawed by inconsistencies
on material matters such as while she testified that she did not know the identity of
the other assailants, she could delineate appellant's features — his curly hair,
mustache and piercing ("nanlilisik") eyes. 36

On the very material point of identification of appellant as the perpetrator of the


crime, she testified as follows:

FISCAL PERALTA:

Did you come to know the name of that person whom


you said (was) the victim of that stabbing?

WITNESS:

A certain Charlie, sir.

FISCAL PERALTA:

What about that person whom you identified a while ago


as the one who stabbed the victim Charlie? When did
you come to know the name David Salvatierra?

WITNESS:

When I went to the Homicide Section and there was a


police lineup made by the police officers there
consisting of eight (8) persons and I was made to point
to that person who stabbed the victim and I pointed to
that person, sir.

FISCAL PERALTA:

Could you recall, Madam Witness, when was that police


lineup conducted?

WITNESS:

November 17, 1990, sir.

FISCAL PERALTA:

Now, the incident happened on August 17, 1990. Why is


it that it was only November 17, 1990 that you identified
the assailant David Salvatierra?

WITNESS:
Because he was not yet arrested and I was also afraid,
sir.

x x x           x x x          x x x

FISCAL PERALTA:

Now, that person whom you said you saw stabbed the
victim together with two (2) other companions and the
person whom you pointed to in the police lineup
conducted by the police on November 17, 1990, if you
see him again, can you still identify him?

WITNESS:

Yes, sir.

FISCAL PERALTA:

Will you please look around the courtroom and point to


him?

COURT:

Fiscal, what are you asking this witness?

FISCAL PERALTA:

The one who was pointed to by the witness in the police


station during the police lineup because she already
identified the accused as the one who stabbed, Your
Honor.

FISCAL PERALTA:

(to the Witness)

Will you please point to him?

WITNESS:

That person, sir.

INTERPRETER:
Witness pointed to a persons (sic) who, when asked,
stated his name as David Salvatierra. 37

Even on cross-examination, she remained steadfast on the issue of identification of


appellant. Thus:

ATTY. CORTES:

What pecularities (sic) did these companions of


Salvatierra have that could possibly help you identify
them when you see them again?

WITNESS:

When those persons will be showed (sic) to me, I could


remember their faces, sir. I just do not know how to
describe their appearances but whenever they will be
showed (sic) to me, I could possibly identify them,
because of my low educational attainment.

ATTY. CORTES:

Not even their noses, you could not describe?

WITNESS:

I could not tell you, sir, but I fully remember him.

INTERPRETER:

Witness is pointing to the Accused.

WITNESS: (continuing)

Because he has a moustache and he has a curly hair and


at that time, sir, his eyes were fiercing (sic)
"nanlilisik." 38

From this testimony, it is clear that although Milagros did not know appellant's
name, she remembered his features and recognized him as the perpetrator of the
crime. Indeed, she could not have failed to identify him because she was only eight
(8) meters away when the assault occurred. Visibility was not a problem because
while there were some pedestrians in the area, traffic was light and could not have
obstructed Milagros' view. Furthermore, it was 4:30 in the afternoon when the day
was still bright.
Aside from the said "inconsistency" regarding the identity of the perpetrators of the
crime, appellant points to other "inconsistencies" in her testimony such as: (a) she
first testified that the three persons "met" the victim in the sense that, as appellant
puts it in his brief, "the three (3) persons and the victim were walking towards each
other until they met," but later she said that they were only standing on that
occasion; (b) she claimed that she "never told anyone" about the incident and yet
Marciano Fernandez was able to locate her; (c) although no one knew the identity
of the assailant, Milagros went to Police Station No. 4 upon appellant's
apprehension, to identify him; (d) Marciano Fernandez asked her to testify on
November 16, 1990 and they went to the police the following day; while on the
other hand, the police, through Pat. Amores, testified that Milagros and Fernandez
went to the police station as early as November 15, 1990; and (e) it was impossible
for Milagros to have remembered the features of appellant considering the lapse of
time between the commission of the crime and appellant's arrest because, contrary
to the trial court's finding, she was not so startled by the incident that she continued
vending after its occurrence. 39

Obviously geared at derailing the eyewitness' credibility,


such "inconsistencies," however, are collateral matters which are too trivial and
minor to affect the credibility of Milagros and the evidentiary value of her
testimony. 40 Minor discrepancies in the testimony of a witness even enhances her
credibility, as these minor discrepancies could also indicate that the response given
by the witness was honest and unrehearsed. 41 In fact, when an unlettered person
like Milagros testifies, inconsistencies in her testimony may be disregarded without
impairing her credibility.

The evidence actually shows that Milagros Martinez, an ambulant fish vendor who
finished only Grade 3, was witness to the crime. She never told anybody about
what she saw except to her daughter.

How she was tracked down by Marciano Fernandez to testify intrigues appellant
indeed. But this was a marketplace where people were at least familiar to each
other since they were selling their wares regularly in said place and where word got
around easily. It was, therefore, not highly improbable that word spread around
pointing to Martinez as a possible witness.

Martinez did not report the crime to the police immediately because she was afraid.
Appellant was a known tough guy in the area. He was a member of the "Bahala Na
Gang" and said to belong to a family of killers. 42

The trial court correctly observed that witnesses to a horrendous crime do not
involve themselves by reporting the commission of such crimes because of the
attendant and consequent peril to their lives and those of their loved ones. Unless
the victims are relatives or close friends of such witnesses, the latter ordinarily
keep mum about such incidents and attend to their usual business 43 just as what
Milagros exactly did in this case.

In crimes such as this, the police, as part of their investigatory work certainly had
leads and knew more or less who the suspects were. All what was necessary was a
credible witness to confirm their suspicions. Thus, when appellant was arrested,
Milagros Martinez was persuaded by Marciano Fernandez to confirm if appellant
was indeed the killer.

Only sufficient proof of a sinister motive could have discredited Milagros as a


credible eyewitness. This the defense failed to provide. Its insinuations that
Marciano Fernandez colluded with Milagros to pin down appellant as the killer of
Charlie is bereft of factual foundation and, therefore, they serve no purpose.
Having failed to prove such ill motive certainly demolished appellant's
protestations on the credibility of the prosecution's sole eyewitness. Her testimony
is thus entitled to full faith and credit 44 more so because Milagros was even
presented by the defense as a hostile witness to prove that she "was not around
during the incident. 45 Unfortunately, aside from a repetition of her story for the
prosecution, the defense elicited no more than the added information that she did
not inform the parents of the victim on what she saw because she and her children
were afraid of the accused who belonged to a "family of killers." 46

Contrary to appellant's claim, treachery attended the killing of Charlie Fernandez.


Appellant's claim that there was no treachery because two of the three assailants
"did nothing" and that "the stabbing of the victim could probably be attributed to a
whim or impulse and not a planned and deliberate act" 47 is too preposterous for
comfort. Treachery is present when the offender adopts means, methods or forms
in the execution of a felony, which insure its commission without risk to himself
arising from the defense which the offended party might make. 48

The prosecution proved beyond reasonable doubt that all these elements were
present in the case at bar. The were sufficiently proven by the testimony of
Milagros Martinez whose credibility the defense failed to destroy. Appellants and
his two (2) companions suddenly appeared, surrounded the victim and appellant
stabbed him at least two times. The victim was unarmed. He did not provoke nor
attack the assailants. He was alone walking on a street with people around. He had
no inkling whatsoever that an assailant and his cohorts were lurking and about to
assault him.

The fact that the victim and the malefactors were facing each other during the
assault does not negate the presence of treachery. Even a frontal attack can be
treacherous when it is sudden and unexpected and the victim is unarmed. 49 Neither
may the presence of "defense wounds" on the body of the victim rule out treachery,
Charlie's act of parrying with his bare hands the first thrust inflicted by appellant
was an instinctive reaction to an attack. After all, the law recognizes man's natural
instinct to protect himself from impending danger. 50

The trial court correctly disregarded appellant's alibi. It is elementary that for alibi
to prosper, credible and tangible proof of physical impossibility for the accused to
be at the scene of the crime is indispensable. 51 Even if the testimony of appellant's
wife that the distance between the crime scene and their house was about twenty
minutes' walk away, still, it was not impossible for appellant to be in the crime
scene and return home for merienda not only by walking but by means of
transportation like pedicabs and jeepneys which abounded in the
area. 52

The killing of Charlie Fernandez, being qualified by treachery, constituted murder


as defined and penalized by Article 248 of the Revised Penal Code punishable
by reclusion perpetua in the absence of any aggravating or mitigating
circumstances. 53 The proper imposable penalty being an indivisible one, appellant
shall not benefit from the provisions of the Indeterminate Sentence Law. 54

The trial court also correctly imposed actual damages of P30,183.25, which
amount was duly proven 55 and not contested by the defense, apart from the civil
indemnity of P50,000.00.

WHEREFORE, the decision of the trial court convicting appellant David


Salvatierra of the crime of murder for the killing of Charlie Fernandez is hereby
AFFIRMED in toto. Cost de oficio.

SO ORDERED.

J. The exclusionary rule & the “fruit of the poisonous tree” doctrine

CASES:
Burgos v. Chief of Staff, supra
People v. Cogaed, supra
K. Disposal of seized items
L. Instances of non-availability of right against unreasonable search & seizure

1. Search at the instance of private entities

2. In disbarment proceedings filed by private persons against a lawyer

CASE: People v. Marti, G.R. No. 81561, January 18, 1991


This is an appeal from a decision * rendered by the Special Criminal Court of
Manila (Regional Trial Court, Branch XLIX) convicting accused-appellant of
violation of Section 21 (b), Article IV in relation to Section 4, Article 11 and
Section 2 (e) (i), Article 1 of Republic Act 6425, as amended, otherwise known as
the Dangerous Drugs Act.

The facts as summarized in the brief of the prosecution are as follows:

On August 14, 1987, between 10:00 and 11:00 a.m., the appellant and his
common-law wife, Shirley Reyes, went to the booth of the "Manila Packing
and Export Forwarders" in the Pistang Pilipino Complex, Ermita, Manila,
carrying with them four (4) gift wrapped packages. Anita Reyes (the
proprietress and no relation to Shirley Reyes) attended to them. The
appellant informed Anita Reyes that he was sending the packages to a friend
in Zurich, Switzerland. Appellant filled up the contract necessary for the
transaction, writing therein his name, passport number, the date of shipment
and the name and address of the consignee, namely, "WALTER FIERZ,
Mattacketr II, 8052 Zurich, Switzerland" (Decision, p. 6)

Anita Reyes then asked the appellant if she could examine and inspect the
packages. Appellant, however, refused, assuring her that the packages
simply contained books, cigars, and gloves and were gifts to his friend in
Zurich. In view of appellant's representation, Anita Reyes no longer insisted
on inspecting the packages. The four (4) packages were then placed inside a
brown corrugated box one by two feet in size (1' x 2'). Styro-foam was
placed at the bottom and on top of the packages before the box was sealed
with masking tape, thus making the box ready for shipment (Decision, p. 8).

Before delivery of appellant's box to the Bureau of Customs and/or Bureau


of Posts, Mr. Job Reyes (proprietor) and husband of Anita (Reyes),
following standard operating procedure, opened the boxes for final
inspection. When he opened appellant's box, a peculiar odor emitted
therefrom. His curiousity aroused, he squeezed one of the bundles allegedly
containing gloves and felt dried leaves inside. Opening one of the bundles,
he pulled out a cellophane wrapper protruding from the opening of one of
the gloves. He made an opening on one of the cellophane wrappers and took
several grams of the contents thereof (tsn, pp. 29-30, October 6, 1987;
Emphasis supplied).

Job Reyes forthwith prepared a letter reporting the shipment to the NBI and
requesting a laboratory examination of the samples he extracted from the
cellophane wrapper (tsn, pp. 5-6, October 6, 1987).

He brought the letter and a sample of appellant's shipment to the Narcotics


Section of the National Bureau of Investigation (NBI), at about 1:30 o'clock
in the afternoon of that date, i.e., August 14, 1987. He was interviewed by
the Chief of Narcotics Section. Job Reyes informed the NBI that the rest of
the shipment was still in his office. Therefore, Job Reyes and three (3) NBI
agents, and a photographer, went to the Reyes' office at Ermita, Manila (tsn,
p. 30, October 6, 1987).

Job Reyes brought out the box in which appellant's packages were placed
and, in the presence of the NBI agents, opened the top flaps, removed the
styro-foam and took out the cellophane wrappers from inside the
gloves. Dried marijuana leaves were found to have been contained inside the
cellophane wrappers (tsn, p. 38, October 6, 1987; Emphasis supplied).

The package which allegedly contained books was likewise opened by Job
Reyes. He discovered that the package contained bricks or cake-like dried
marijuana leaves. The package which allegedly contained tabacalera cigars
was also opened. It turned out that dried marijuana leaves were neatly
stocked underneath the cigars (tsn, p. 39, October 6, 1987).

The NBI agents made an inventory and took charge of the box and of the
contents thereof, after signing a "Receipt" acknowledging custody of the
said effects (tsn, pp. 2-3, October 7, 1987).

Thereupon, the NBI agents tried to locate appellant but to no avail. Appellant's
stated address in his passport being the Manila Central Post Office, the agents
requested assistance from the latter's Chief Security. On August 27, 1987,
appellant, while claiming his mail at the Central Post Office, was invited by the
NBI to shed light on the attempted shipment of the seized dried leaves. On the
same day the Narcotics Section of the NBI submitted the dried leaves to the
Forensic Chemistry Section for laboratory examination. It turned out that the dried
leaves were marijuana flowering tops as certified by the forensic chemist.
(Appellee's Brief, pp. 9-11, Rollo, pp. 132-134).

Thereafter, an Information was filed against appellant for violation of RA 6425,


otherwise known as the Dangerous Drugs Act.

After trial, the court a quo rendered the assailed decision.

In this appeal, accused/appellant assigns the following errors, to wit:

THE LOWER COURT ERRED IN ADMITTING IN EVIDENCE THE


ILLEGALLY SEARCHED AND SEIZED OBJECTS CONTAINED IN
THE FOUR PARCELS.

THE LOWER COURT ERRED IN CONVICTING APPELLANT


DESPITE THE UNDISPUTED FACT THAT HIS RIGHTS UNDER THE
CONSTITUTION WHILE UNDER CUSTODIAL PROCEEDINGS WERE
NOT OBSERVED.

THE LOWER COURT ERRED IN NOT GIVING CREDENCE TO THE


EXPLANATION OF THE APPELLANT ON HOW THE FOUR
PARCELS CAME INTO HIS POSSESSION (Appellant's Brief, p. 1; Rollo,
p. 55)

1. Appellant contends that the evidence subject of the imputed offense had been
obtained in violation of his constitutional rights against unreasonable search and
seizure and privacy of communication (Sec. 2 and 3, Art. III, Constitution) and
therefore argues that the same should be held inadmissible in evidence (Sec. 3 (2),
Art. III).

Sections 2 and 3, Article III of the Constitution provide:

Sec. 2. The right of the people to be secure in their persons, houses, papers
and effects against unreasonable searches and seizures of whatever nature
and for any purpose shall be inviolable, and no search warrant or warrant of
arrest shall issue except upon probable cause to be determined personally by
the judge after examination under oath or affirmation of the complainant and
the witnesses he may produce, and particularly describing the place to be
searched and the persons or things to be seized.

Sec. 3. (1) The privacy of communication and correspondence shall be


inviolable except upon lawful order of the court, or when public safety or
order requires otherwise as prescribed by law.

(2) Any evidence obtained in violation of this or the preceding section shall
be inadmissible for any purpose in any proceeding.

Our present constitutional provision on the guarantee against unreasonable search


and seizure had its origin in the 1935 Charter which, worded as follows:

The right of the people to be secure in their persons, houses, papers and
effects against unreasonable searches and seizures shall not be violated, and
no warrants shall issue but upon probable cause, to be determined by the
judge after examination under oath or affirmation of the complainant and the
witnesses he may produce, and particularly describing the place to be
searched, and the persons or things to be seized. (Sec. 1 [3], Article III)

was in turn derived almost verbatim from the Fourth Amendment ** to the United
States Constitution. As such, the Court may turn to the pronouncements of the
United States Federal Supreme Court and State Appellate Courts which are
considered doctrinal in this jurisdiction.
Thus, following the exclusionary rule laid down in Mapp v. Ohio by the US
Federal Supreme Court (367 US 643, 81 S.Ct. 1684, 6 L.Ed. 1081 [1961]), this
Court, in Stonehill v. Diokno (20 SCRA 383 [1967]), declared as inadmissible any
evidence obtained by virtue of a defective search and seizure warrant, abandoning
in the process the ruling earlier adopted in Moncado v. People's Court (80 Phil. 1
[1948]) wherein the admissibility of evidence was not affected by the illegality of
its seizure. The 1973 Charter (Sec. 4 [2], Art. IV) constitutionalized
the Stonehill ruling and is carried over up to the present with the advent of the
1987 Constitution.

In a number of cases, the Court strictly adhered to the exclusionary rule and has
struck down the admissibility of evidence obtained in violation of the
constitutional safeguard against unreasonable searches and seizures. (Bache & Co.,
(Phil.), Inc., v. Ruiz, 37 SCRA 823 [1971]; Lim v. Ponce de Leon, 66 SCRA 299
[1975]; People v. Burgos, 144 SCRA 1 [1986]; Roan v. Gonzales, 145 SCRA 687
[1987]; See also Salazar v. Hon. Achacoso, et al., GR No. 81510, March 14, 1990).

It must be noted, however, that in all those cases adverted to, the evidence so
obtained were invariably procured by the State acting through the medium of its
law enforcers or other authorized government agencies.

On the other hand, the case at bar assumes a peculiar character since the evidence
sought to be excluded was primarily discovered and obtained by a private person,
acting in a private capacity and without the intervention and participation of State
authorities. Under the circumstances, can accused/appellant validly claim that his
constitutional right against unreasonable searches and seizure has been violated?
Stated otherwise, may an act of a private individual, allegedly in violation of
appellant's constitutional rights, be invoked against the State?

We hold in the negative. In the absence of governmental interference, the liberties


guaranteed by the Constitution cannot be invoked against the State.

As this Court held in Villanueva v. Querubin (48 SCRA 345 [1972]:

1. This constitutional right (against unreasonable search and seizure) refers


to the immunity of one's person, whether citizen or alien, from interference
by government, included in which is his residence, his papers, and other
possessions. . . .

. . . There the state, however powerful, does not as such have the access
except under the circumstances above noted, for in the traditional
formulation, his house, however humble, is his castle. Thus is outlawed any
unwarranted intrusion by government, which is called upon to refrain from
any invasion of his dwelling and to respect the privacies of his life. . . . (Cf.
Schermerber v. California, 384 US 757 [1966] and Boyd v. United States,
116 US 616 [1886]; Emphasis supplied).

In Burdeau v. McDowell (256 US 465 (1921), 41 S Ct. 547; 65 L.Ed. 1048), the


Court there in construing the right against unreasonable searches and seizures
declared that:

(t)he Fourth Amendment gives protection against unlawful searches and


seizures, and as shown in previous cases, its protection applies to
governmental action. Its origin and history clearly show that it was intended
as a restraint upon the activities of sovereign authority, and was not intended
to be a limitation upon other than governmental agencies; as against such
authority it was the purpose of the Fourth Amendment to secure the citizen
in the right of unmolested occupation of his dwelling and the possession of
his property, subject to the right of seizure by process duly served.

The above ruling was reiterated in State v. Bryan (457 P.2d 661 [1968]) where a
parking attendant who searched the automobile to ascertain the owner thereof
found marijuana instead, without the knowledge and participation of police
authorities, was declared admissible in prosecution for illegal possession of
narcotics.

And again in the 1969 case of Walker v. State (429 S.W.2d 121), it was held that
the search and seizure clauses are restraints upon the government and its agents,
not upon private individuals (citing People v. Potter, 240 Cal. App.2d 621, 49 Cap.
Rptr, 892 (1966); State v. Brown, Mo., 391 S.W.2d 903 (1965); State v. Olsen, Or.,
317 P.2d 938 (1957).

Likewise appropos is the case of Bernas v. US (373 F.2d 517 (1967). The Court
there said:

The search of which appellant complains, however, was made by a private


citizen — the owner of a motel in which appellant stayed overnight and in
which he left behind a travel case containing the evidence*** complained
of. The search was made on the motel owner's own initiative. Because of it,
he became suspicious, called the local police, informed them of the bag's
contents, and made it available to the authorities.

The fourth amendment and the case law applying it do not require exclusion
of evidence obtained through a search by a private citizen. Rather, the
amendment only proscribes governmental action."

The contraband in the case at bar having come into possession of the Government
without the latter transgressing appellant's rights against unreasonable search and
seizure, the Court sees no cogent reason why the same should not be admitted
against him in the prosecution of the offense charged.

Appellant, however, would like this court to believe that NBI agents made an
illegal search and seizure of the evidence later on used in prosecuting the case
which resulted in his conviction.

The postulate advanced by accused/appellant needs to be clarified in two days. In


both instances, the argument stands to fall on its own weight, or the lack of it.

First, the factual considerations of the case at bar readily foreclose the proposition
that NBI agents conducted an illegal search and seizure of the prohibited
merchandise. Records of the case clearly indicate that it was Mr. Job Reyes, the
proprietor of the forwarding agency, who made search/inspection of the packages.
Said inspection was reasonable and a standard operating procedure on the part of
Mr. Reyes as a precautionary measure before delivery of packages to the Bureau of
Customs or the Bureau of Posts (TSN, October 6 & 7, 1987, pp. 15-18; pp. 7-8;
Original Records, pp. 119-122; 167-168).

It will be recalled that after Reyes opened the box containing the illicit cargo, he
took samples of the same to the NBI and later summoned the agents to his place of
business. Thereafter, he opened the parcel containing the rest of the shipment and
entrusted the care and custody thereof to the NBI agents. Clearly, the NBI agents
made no search and seizure, much less an illegal one, contrary to the postulate of
accused/appellant.

Second, the mere presence of the NBI agents did not convert the reasonable search
effected by Reyes into a warrantless search and seizure proscribed by the
Constitution. Merely to observe and look at that which is in plain sight is not a
search. Having observed that which is open, where no trespass has been committed
in aid thereof, is not search (Chadwick v. State, 429 SW2d 135). Where the
contraband articles are identified without a trespass on the part of the arresting
officer, there is not the search that is prohibited by the constitution (US v. Lee 274
US 559, 71 L.Ed. 1202 [1927]; Ker v. State of California 374 US 23, 10 L.Ed.2d.
726 [1963]; Moore v. State, 429 SW2d 122 [1968]).

In Gandy v. Watkins (237 F. Supp. 266 [1964]), it was likewise held that where the
property was taken into custody of the police at the specific request of the manager
and where the search was initially made by the owner there is no unreasonable
search and seizure within the constitutional meaning of the term.

That the Bill of Rights embodied in the Constitution is not meant to be invoked
against acts of private individuals finds support in the deliberations of the
Constitutional Commission. True, the liberties guaranteed by the fundamental law
of the land must always be subject to protection. But protection against whom?
Commissioner Bernas in his sponsorship speech in the Bill of Rights answers the
query which he himself posed, as follows:

First, the general reflections. The protection of fundamental liberties in the


essence of constitutional democracy. Protection against whom? Protection
against the state. The Bill of Rights governs the relationship between the
individual and the state. Its concern is not the relation between individuals,
between a private individual and other individuals. What the Bill of Rights
does is to declare some forbidden zones in the private sphere inaccessible to
any power holder. (Sponsorship Speech of Commissioner Bernas , Record
of the Constitutional Commission, Vol. 1, p. 674; July 17, 1986; Emphasis
supplied)

The constitutional proscription against unlawful searches and seizures therefore


applies as a restraint directed only against the government and its agencies tasked
with the enforcement of the law. Thus, it could only be invoked against the State to
whom the restraint against arbitrary and unreasonable exercise of power is
imposed.

If the search is made upon the request of law enforcers, a warrant must generally
be first secured if it is to pass the test of constitutionality. However, if the search is
made at the behest or initiative of the proprietor of a private establishment for its
own and private purposes, as in the case at bar, and without the intervention of
police authorities, the right against unreasonable search and seizure cannot be
invoked for only the act of private individual, not the law enforcers, is involved. In
sum, the protection against unreasonable searches and seizures cannot be extended
to acts committed by private individuals so as to bring it within the ambit of
alleged unlawful intrusion by the government.

Appellant argues, however, that since the provisions of the 1935 Constitution has
been modified by the present phraseology found in the 1987 Charter, expressly
declaring as inadmissible any evidence obtained in violation of the constitutional
prohibition against illegal search and seizure, it matters not whether the evidence
was procured by police authorities or private individuals (Appellant's Brief, p.
8, Rollo, p. 62).

The argument is untenable. For one thing, the constitution, in laying down the
principles of the government and fundamental liberties of the people, does not
govern relationships between individuals. Moreover, it must be emphasized that
the modifications introduced in the 1987 Constitution (re: Sec. 2, Art. III) relate to
the issuance of either a search warrant or warrant of arrest vis-a-vis the
responsibility of the judge in the issuance thereof (See Soliven v. Makasiar, 167
SCRA 393 [1988]; Circular No. 13 [October 1, 1985] and Circular No. 12 [June
30, 1987]. The modifications introduced deviate in no manner as to whom the
restriction or inhibition against unreasonable search and seizure is directed against.
The restraint stayed with the State and did not shift to anyone else.

Corolarilly, alleged violations against unreasonable search and seizure may only be
invoked against the State by an individual unjustly traduced by the exercise of
sovereign authority. To agree with appellant that an act of a private individual in
violation of the Bill of Rights should also be construed as an act of the State would
result in serious legal complications and an absurd interpretation of the
constitution.

Similarly, the admissibility of the evidence procured by an individual effected


through private seizure equally applies, in pari passu, to the alleged violation, non-
governmental as it is, of appellant's constitutional rights to privacy and
communication.

2. In his second assignment of error, appellant contends that the lower court erred
in convicting him despite the undisputed fact that his rights under the constitution
while under custodial investigation were not observed.

Again, the contention is without merit, We have carefully examined the records of
the case and found nothing to indicate, as an "undisputed fact", that appellant was
not informed of his constitutional rights or that he gave statements without the
assistance of counsel. The law enforcers testified that accused/appellant was
informed of his constitutional rights. It is presumed that they have regularly
performed their duties (See. 5(m), Rule 131) and their testimonies should be given
full faith and credence, there being no evidence to the contrary. What is clear from
the records, on the other hand, is that appellant refused to give any written
statement while under investigation as testified by Atty. Lastimoso of the NBI,
Thus:

Fiscal Formoso:

You said that you investigated Mr. and Mrs. Job Reyes. What about the
accused here, did you investigate the accused together with the girl?

WITNESS:

Yes, we have interviewed the accused together with the girl but the accused
availed of his constitutional right not to give any written statement, sir.
(TSN, October 8, 1987, p. 62; Original Records, p. 240)

The above testimony of the witness for the prosecution was not contradicted by the
defense on cross-examination. As borne out by the records, neither was there any
proof by the defense that appellant gave uncounselled confession while being
investigated. What is more, we have examined the assailed judgment of the trial
court and nowhere is there any reference made to the testimony of appellant while
under custodial investigation which was utilized in the finding of conviction.
Appellant's second assignment of error is therefore misplaced.

3. Coming now to appellant's third assignment of error, appellant would like us to


believe that he was not the owner of the packages which contained prohibited
drugs but rather a certain Michael, a German national, whom appellant met in a
pub along Ermita, Manila: that in the course of their 30-minute conversation,
Michael requested him to ship the packages and gave him P2,000.00 for the cost of
the shipment since the German national was about to leave the country the next
day (October 15, 1987, TSN, pp. 2-10).

Rather than give the appearance of veracity, we find appellant's disclaimer as


incredulous, self-serving and contrary to human experience. It can easily be
fabricated. An acquaintance with a complete stranger struck in half an hour could
not have pushed a man to entrust the shipment of four (4) parcels and shell out
P2,000.00 for the purpose and for appellant to readily accede to comply with the
undertaking without first ascertaining its contents. As stated by the trial court, "(a)
person would not simply entrust contraband and of considerable value at that as the
marijuana flowering tops, and the cash amount of P2,000.00 to a complete stranger
like the Accused. The Accused, on the other hand, would not simply accept such
undertaking to take custody of the packages and ship the same from a complete
stranger on his mere say-so" (Decision, p. 19, Rollo, p. 91). As to why he readily
agreed to do the errand, appellant failed to explain. Denials, if unsubstantiated by
clear and convincing evidence, are negative self-serving evidence which deserve
no weight in law and cannot be given greater evidentiary weight than the testimony
of credible witnesses who testify on affirmative matters (People v. Esquillo, 171
SCRA 571 [1989]; People vs. Sariol, 174 SCRA 237 [1989]).

Appellant's bare denial is even made more suspect considering that, as per records
of the Interpol, he was previously convicted of possession of hashish by the Kleve
Court in the Federal Republic of Germany on January 1, 1982 and that the
consignee of the frustrated shipment, Walter Fierz, also a Swiss national, was
likewise convicted for drug abuse and is just about an hour's drive from appellant's
residence in Zurich, Switzerland (TSN, October 8, 1987, p. 66; Original Records,
p. 244; Decision, p. 21; Rollo, p. 93).

Evidence to be believed, must not only proceed from the mouth of a credible
witness, but it must be credible in itself such as the common experience and
observation of mankind can approve as probable under the circumstances (People
v. Alto, 26 SCRA 342 [1968], citing Daggers v. Van Dyke, 37 N.J. Eg. 130; see
also People v. Sarda, 172 SCRA 651 [1989]; People v. Sunga, 123 SCRA 327
[1983]); Castañares v. CA, 92 SCRA 567 [1979]). As records further show,
appellant did not even bother to ask Michael's full name, his complete address or
passport number. Furthermore, if indeed, the German national was the owner of the
merchandise, appellant should have so indicated in the contract of shipment (Exh.
"B", Original Records, p. 40). On the contrary, appellant signed the contract as the
owner and shipper thereof giving more weight to the presumption that things
which a person possesses, or exercises acts of ownership over, are owned by him
(Sec. 5 [j], Rule 131). At this point, appellant is therefore estopped to claim
otherwise.

Premises considered, we see no error committed by the trial court in rendering the
assailed judgment.

WHEREFORE, the judgment of conviction finding appellant guilty beyond


reasonable doubt of the crime charged is hereby AFFIRMED. No costs.

SO ORDERED.

Fernan, C.J., Gutierrez, Jr. and Feliciano, JJ., concur.

Issue: WON the appellant can invoke his constitutional rights against unreasonable
search and seizure and privacy of communication.

Ruling: No, the appellant cannot invoke his constitutional rights against
unreasonable search and seizure and privacy of communication.

The evidence sought to be excluded was primarily discovered and obtained by


a private person, acting in a private capacity and without the intervention and
participation of State authorities.

The Court held that in the absence of governmental interference, the liberties
guaranteed by the Constitution cannot be invoked against the State.

In the 1969 case of Walker v. State (429 S.W.2d 121), it was held that the search
and seizure clauses are restraints upon the government and its agents, not upon
private individuals, (citing People v. Potter, 240 Cal. App.2d 621, 49 Cap. Rptr,
892 (1966); State v. Brown, Mo., 391 S.W.2d 903 (1965); State v. Olsen, Or., 317
P.2d 938 (1957).

Likewise appropos is the case of Bernas v. US (373 F.2d 517 (1967). The Court
there said: "The fourth amendment and the case law applying it do not require
exclusion of evidence obtained through a search by a private citizen. Rather, the
amendment only proscribes governmental action."

The contraband in the case at bar having come into possession of the Government
without the latter transgressing appellant's rights against unreasonable search and
seizure, the Court sees no cogent reason why the same should not be admitted
against him in the prosecution of the offense charged.

Records of the case clearly indicate that it was Mr. Job Reyes, the proprietor
of the forwarding agency, who made search/inspection of the packages.
Said inspection was reasonable and a standard operating procedure on
the part of Mr. Reyes as a precautionary measure before delivery of
packages to the Bureau of Customs or the Bureau of Posts. 

After Reyes opened the box containing the illicit cargo, he took samples of
the same to the NBI and later summoned the agents to his place of business.
Thereafter, he opened the parcels containing the rest of the shipment and
entrusted the care and custody thereof to the NBI agents. Clearly, the NBI
agents made no search and seizure, much less an illegal one, contrary to
the postulate of accused/appellant.

Also, the Bill of Rights embodied in the Constitution is not meant to be invoked
against acts of private individuals finds support in the deliberations of the
Constitutional Commission. 

Hence, the constitutional proscription against unlawful searches and seizures


therefore applies as a restraint directed only against the government and its
agencies tasked with the enforcement of the law. Thus, it could only be invoked
against the State to whom the restraint against arbitrary and unreasonable exercise
of power is imposed. 

If the search is made at the behest or initiative of the proprietor of a private


establishment for its own and private purposes, as in the case at bar, and without
the intervention of police authorities, the right against unreasonable search and
seizure cannot be invoked for only the act of private individual, not the law
enforcers, is involved. In sum, the protection against unreasonable searches
and seizures cannot be extended to acts committed by private individuals so as
to bring it within the ambit of alleged unlawful intrusion by the government.

Dispositive Portion:
WHEREFORE, the judgment of conviction finding appellant guilty beyond
reasonable doubt of the crime charged is hereby AFFIRMED.

CASE: Tolentino v. Atty. Mendoza, A.C. No. 5151, October 19, 2004

ADM. CASE NO. 5151             October 19, 2004

PEDRO G. TOLENTINO, ROMEO M. LAYGO, SOLOMON M.


LUMALANG, SR., MELITON D. EVANGELISTA, SR., complainants,
vs.
ATTY. NORBERTO M. MENDOZA, respondent.

RESOLUTION

AUSTRIA-MARTINEZ, J.:

Before us is a complaint filed by Pedro G. Tolentino, Romeo M. Laygo, Solomon


M. Lumalang, Sr., Meliton D. Evangelista, Sr., and Nelson B. Melgar against Atty.
Norberto M. Mendoza for Grossly Immoral Conduct and Gross Misconduct.

Complainants allege in their Affidavit-Complaint that respondent, a former


Municipal Trial Court Judge, abandoned his legal wife, Felicitas V. Valderia in
favor of his paramour, Marilyn dela Fuente, who is, in turn, married to one Ramon
G. Marcos; respondent and Marilyn dela Fuente have been cohabiting openly and
publicly as husband and wife in Brgy. Estrella, Naujan, Oriental Mindoro;
respondent had fathered two children by his paramour Marilyn dela Fuente;
respondent and Marilyn dela Fuente declared in the birth certificates of their two
daughters that they were married on May 12, 1986, making it appear that their two
children are legitimate, while in respondent’s Certificate of Candidacy filed with
the COMELEC during the 1995 elections, respondent declared that his wife is
Felicitas V. Valderia; in respondent’s certificate of candidacy for the 1998
elections, he declared his civil status as separated; such declarations in the birth
certificates of his children and in his certificate of candidacy are acts constituting
falsification of public documents; and respondent’s acts betray his lack of good
moral character and constitute grounds for his removal as a member of the bar.

Respondent filed his Comment wherein he states that complainants, who are his
political opponents in Naujan, Oriental Mindoro, are merely filing this case to
exact revenge on him for his filing of criminal charges against them; complainants
illegally procured copies of the birth certificates of Mara Khrisna Charmina dela
Fuente Mendoza and Myrra Khrisna Normina dela Fuente Mendoza, in violation of
Rule 24, Administrative Order No. 1, series of 1993, thus, such documents are
inadmissible in evidence; respondent did not participate in the preparation and
submission with the local civil registry of subject birth certificates; respondent
never declared that he had two wives, as he has always declared that he is
separated in fact from his wife, Felicitas V. Valderia; and complainants have used
this issue against him during elections and yet, the people of Naujan, Oriental
Mindoro still elected him as Mayor, hence, respondent has not offended the
public’s sense of morality.

The administrative case was referred to the Integrated Bar of the Philippines
(hereinafter IBP) for investigation, report and recommendation. Thereafter, the
Commission on Bar Discipline of the IBP conducted hearings.
Witnesses for complainants, Nelson B. Melgar and Romeo M. Laygo, submitted
their affidavits as their direct testimony and were subjected to cross-examination
by respondent’s counsel.

Witness Nelson B. Melgar declares in his affidavit as follows: He knows


respondent for they both reside in Naujan, Oriental Mindoro. Respondent is known
as a practicing lawyer and a former Municipal Trial Court Judge. Respondent has
been cohabiting openly and publicly with Marilyn dela Fuente, representing
themselves to be husband and wife, and from their cohabitation, they produced two
children, namely, Mara Khrisna Charmina dela Fuente Mendoza and Myrra
Khrisna Normina dela Fuente Mendoza. Sometime in 1995, he (witness Melgar)
received a letter from a concerned citizen, informing him that respondent was
married to Felicitas Valderia of San Rafael, Bulacan, on January 16, 1980, but
respondent abandoned his wife to cohabit with Marilyn dela Fuente. Attached to
the letter was a photocopy of a Certification issued by the Civil Register attesting
to the marriage between respondent and Felicitas Valderia. He also received
information from concerned citizens that Marilyn dela Fuente is also legally
married to one Ramon G. Marcos, as evidenced by a Certification from the Office
of the Civil Register. Respondent stated in his Certificate of Candidacy filed with
the COMELEC in 1995 that he is still legally married to Felicitas Valderia. In
respondent’s Certificate of Candidacy filed with the COMELEC in 1998, he
declared his civil status as separated. Respondent has represented to all that he is
married to Marilyn dela Fuente. In the Naujanews, a local newspaper where
respondent holds the position of Chairman of the Board of the Editorial Staff,
respondent was reported by said newspaper as husband to Marilyn dela Fuente and
the father of Mara Khrisna Charmina and Myrra Khrisna Normina.

On cross-examination, witness Melgar testified as follows: He was the former


mayor of Naujan and he and respondent belong to warring political parties. It was
not respondent who told him about the alleged immoral conduct subject of the
present case. Although he received the letter of a concerned citizen regarding the
immoral conduct of respondent as far back as 1995, he did not immediately file a
case for disbarment against respondent. It was only after respondent filed a
criminal case for falsification against him that he decided to file an administrative
case against respondent.1

On re-direct examination, witness Melgar testified that there were people who were
against the open relationship between respondent and Marilyn dela Fuente as
respondent had been publicly introducing the latter as his wife despite the fact that
they are both still legally married to other persons, and so someone unknown to
him just handed to their maid copies of the birth certificates of Mara Khrisna
Charmina and Myrra Khrisna Normina.2

The affidavit of Mr. Romeo M. Laygo, which was adopted as his direct testimony,
is practically identical to that of witness Melgar. On cross-examination, witness
Laygo testified that he was not the one who procured the certified true copies of
the birth certificates of Mara Khrisna Charmina dela Fuente Mendoza and Myrra
Khrisna Normina dela Fuente Mendoza, as somebody just gave said documents to
Nelson Melgar. He was a municipal councilor in 1995 when the letter of a
concerned citizen regarding respondent’s immorality was sent to Melgar, but he
did not take any action against respondent at that time. 3

Complainants then formally offered documentary evidence consisting of


photocopies which were admitted by respondent’s counsel to be faithful
reproductions of the originals or certified true copies thereof, to wit: a letter of one
Luis Bermudez informing Nelson Melgar of respondent’s immoral acts, 4 the
Certification of the Local Civil Registrar of San Rafael, Bulacan, attesting to the
celebration of the marriage between respondent and one Felicitas Valderia, 5 the
Birth Certificate of Mara Khrisna Charmina dela Fuente Mendoza, 6 the Birth
Certificate of Myrra Khrisna Normina dela Fuente Mendoza, 7 the Certificate of
Candidacy of respondent dated March 9, 1995,8 the Certificate of Candidacy of
respondent dated March 25, 1998,9 Certification issued by the Civil Registrar of
Naujan, Oriental Mindoro dated October 27, 1998, attesting to the marriage
celebrated between Marilyn dela Fuente and Ramon Marcos, 10 and the editorial
page of the Naujanews (February-March 1999 issue),11 wherein it was stated that
respondent has two daughters with his wife, Marilyn dela Fuente.

Respondent, on the other hand, opted not to present any evidence and merely
submitted a memorandum expounding on his arguments that the testimonies of
complainants’ witnesses are mere hearsay, thus, said testimonies and their
documentary evidence have no probative weight.

On February 27, 2004, the Board of Governors of the IBP passed Resolution No.
XVI-2004-123, reading as follows:

RESOLVED to ADOPT and APPROVE, as it is
hereby ADOPTED and APPROVED, the Report and Recommendation of
the Investigating Commissioner of the above-entitled case, herein made part
of this Resolution as Annex "A"; and, finding the recommendation fully
supported by the evidence on record and the applicable laws and rules, and
considering respondent’s violation of Rule 1.01 of the Code of Professional
Responsibility, Atty. Norberto M. Mendoza is hereby SUSPENDED
INDEFINITELY from the practice of law until he submits satisfactory
proof that he is no longer cohabiting with a woman who is not his wife and
has abandoned such immoral course of conduct.

Portions of the report and recommendation of the IBP Commission on Bar


Discipline, upon which the above-quoted Resolution was based, read as follows:

FINDINGS:
The evidence of complainants to support their charge of immorality consists
in a) the testimonies of Nelson Melgar and Romeo Laygo given by way of
affidavits executed under oath and affirmed before the Commission and b)
their documentary evidence consisting of their Exhibits "A" to "H".

Respondent filed his comment through counsel and did not formally present
or offer any evidence. Respondent opted not to present his evidence
anymore because according to him "there is none to rebut vis-à-vis the
evidence presented by the private complainants." Respondent instead
submitted a memorandum through counsel to argue his position. As can be
seen from the comment and memorandum submitted, respondent’s counsel
argues that the complaint is politically motivated since complainants are
political rivals of respondent and that the birth certificates Exhibits "D" and
"D-1" which were offered to show that respondent sired the children namely
Mara Khrisna Charmina dela Fuente Mendoza and Myrra Khrisna Normina
dela Fuente Mendoza out of his cohabitation with Marilyn dela Fuente are
inadmissible because they were allegedly secured in violation of
Administrative Order No. 1, Series of 1993. The rest of the exhibits are
either hearsay or self-serving according to respondent.

The witnesses who are also two of the complainants herein, on the other hand,
categorically state in their affidavits [Exhibits "A" and "B"] particularly in
paragraph 2 that "Respondent has been cohabiting openly and publicly with
Marilyn de la Fuente, representing themselves to be husband and wife." In
paragraph 10 of said affidavits the witnesses also categorically state that
"respondent has even represented to all and sundry that Marilyn de la Fuente is his
wife." These categorical statements made under oath by complainants are not
hearsay and remain un-rebutted. Respondent chose not to rebut them.

Exhibit "E," the Certificate of Candidacy executed by respondent shows that


respondent is married to one, Felicitas V. Valderia. As shown by Exhibit "H", a
marriage certificate, Marilyn de la Fuente is married to one, Ramon G. Marcos.
Duly certified true copies of said exhibits have been presented by complainants.

With respect to Exhibits "D" and "D-1", we believe that they are competent and
relevant evidence and admissible in this proceedings. The exclusionary rule which
bars admission of illegally obtained evidence applies more appropriately to
evidence obtained as a result of illegal searches and seizures. The instant case
cannot be analogous to an illegal search or seizure. A person who violates Rule 24
of Administrative Order No. 1 Series of 1993 as cited by respondent risks the
penalty of imprisonment or payment of a fine but it does not make the document so
issued inadmissible as evidence specially in proceedings like the present case.
Exhibits "D" and "D-1" which are duly certified birth certificates are therefore
competent evidence to show paternity of said children by respondent in the
absence of any evidence to the contrary.
By and large the evidence of complainants consisting of the testimonies of
witnesses Nelson Melgar and Romeo Laygo, and corroborated by the documentary
exhibits will show that indeed respondent has been cohabiting publicly with a
certain Marilyn de la Fuente who is not his wife and that out of said cohabitation
respondent sired two children. These facts we repeat have not been denied by
respondent under oath since he chose to just argue on the basis of the improper
motivations and the inadmissibility, hearsay and self-serving nature of the
documents presented. Complainants have presented evidence sufficient enough to
convince us that indeed respondent has been cohabiting publicly with a person who
is not his wife. The evidence taken together will support the fact that respondent is
not of good moral character. That respondent chose not to deny under oath the
grave and serious allegations made against him is to our mind his undoing and his
silence has not helped his position before the Commission. As between the
documents and positive statements of complainants, made under oath and the
arguments and comments of respondent submitted through his lawyers, which were
not verified under oath by respondent himself, we are inclined and so give weight
to the evidence of complainants. The direct and forthright testimonies and
statements of Nelson Melgar and Romeo Laygo that respondent was openly
cohabiting with Marilyn de la Fuente is not hearsay. The witnesses may have
admitted that respondent Mendoza did not tell them that a certain Marilyn de la
Fuente was his paramour (for why would respondent admit that to complainants)
but the witnesses did state clearly in their affidavits under oath that respondent was
cohabiting with Marilyn de la Fuente who is not respondent’s wife. Again their
categorical statements taken together with the other documents, are enough to
convince us and conclude that respondent is not of good moral character.

Members of the Bar have been repeatedly reminded that possession of good moral
character is a continuing condition for membership in the Bar in good standing.
The continued possession of good moral character is a requisite condition for
remaining in the practice of law [Mortel vs. Aspiras 100 Phil. 586 (1956); Cordova
vs. Cordova 179 SCRA 680 (1989); People vs. Tuanda 181 SCRA 682 (1990)].
The moral delinquency that affects the fitness of a member of the bar to continue
as such includes conduct that outrages the generally accepted moral standards of
the community, conduct for instance, which makes "mockery of the inviolable
social institution of marriage" [Mijares vs. Villaluz 274 SCRA 1 (1997)].

In the instant case respondent has disregarded and made a mockery of the
fundamental institution of marriage. Respondent in fact even so stated in Exhibit
"F" that he is separated from his wife. This fact and statement without any further
explanation from respondent only contributes to the blot in his moral character
which good moral character we repeat is a continuing condition for a member to
remain in good standing. Under Rule 1.01 of the Code of Professional
Responsibility, a lawyer shall not engage in unlawful, dishonest, immoral or
deceitful conduct. Respondent has violated this rule against engaging in immoral
conduct.
We agree, as cited by the respondent, with the pronouncement made in Santos vs.
Dischoso, 84 SCRA 622 (1978) that courts should not be used by private persons
particularly disgruntled opponents to vent their rancor on members of the Bar
through unjust and unfounded accusations. However, in the instant case the
charges can hardly be considered as unfounded or unjust based on the evidence
presented. The evidence presented shows that respondent no longer possess (sic)
that good moral character necessary as a condition for him to remain a member of
the Bar in good standing. He is therefore not entitled to continue to engage in the
practice of law.

We find such report and recommendation of the IBP to be fully supported by the
pleadings and evidence on record, and, hence, approve and adopt the same.

The evidence presented by complainants reach that quantum of evidence required


in administrative proceedings which is only substantial evidence, or that amount of
relevant evidence that a reasonable mind might accept as adequate to support a
conviction.12

Witness Melgar’s testimony that respondent had been publicly introducing Marilyn
dela Fuente as his wife is corroborated by the contents of an article in
the Naujanews, introducing respondent as one of Naujan’s public servants, and
stating therein that respondent has been blessed with two beautiful children with
his wife, Marilyn dela Fuente.13 It should be noted that said publication is under the
control of respondent, he being the Chairman of the Board thereof. Thus, it could
be reasonably concluded that if he contested the truth of the contents of subject
article in the Naujanews, or if he did not wish to publicly present Marilyn dela
Fuente as his wife, he could have easily ordered that the damning portions of said
article to be edited out.

With regard to respondent’s argument that the credibility of witnesses for the
complainants is tainted by the fact that they are motivated by revenge for
respondent’s filing of criminal cases against them, we opine that even if witnesses
Melgar and Laygo are so motivated, the credibility of their testimonies cannot be
discounted as they are fully supported and corroborated by documentary evidence
which speak for themselves. The birth certificates of Mara Khrisna Charmina dela
Fuente Mendoza and Myrra Khrisna Normina dela Fuente Mendoza born on June
16, 1988 and May 22, 1990, respectively, to Norberto M. Mendoza and Marilyn
Dela Fuente; and the Certification from the Office of the Local Civil Registrar of
Bulacan attesting to the existence in its records of an entry of a marriage between
respondent and one Felicitas Valderia celebrated on January 16, 1980, are public
documents and are prima facie evidence of the facts contained therein, as provided
for under Article 41014 of the Civil Code of the Philippines.

Respondent mistakenly argues that the birth certificates of Mara Khrisna Charmina
dela Fuente Mendoza and Myrra Khrisna Normina dela Fuente Mendoza born on
June 16, 1988 and May 22, 1990, respectively, to Norberto M. Mendoza and
Marilyn Dela Fuente, are inadmissible in evidence for having been obtained in
violation of Rule 24, Administrative Order No. 1, series of 1993, which provides as
follows:

Rule 24. Non-Disclosure of Birth Records. –

(1) The records of a person’s birth shall be kept strictly confidential


and no information relating thereto shall be issued except on the
request of any of the following:

a. the concerned person himself, or any person authorized by


him;

b. the court or proper public official whenever absolutely


necessary in administrative, judicial or other official
proceedings to determine the identity of the child’s parents or
other circumstances surrounding his birth; and

c. in case of the person’s death, the nearest of kin.

(2) Any person violating the prohibition shall suffer the penalty of
imprisonment of at least two months or a fine in an amount not
exceeding five hundred pesos, or both in the discretion of the court.
(Article 7, P.D. 603)

Section 3, Rule 128 of the Revised Rules on Evidence provides that "evidence is
admissible when it is relevant to the issue and is not excluded by the law or these
rules." There could be no dispute that the subject birth certificates are relevant to
the issue. The only question, therefore, is whether the law or the rules provide for
the inadmissibility of said birth certificates allegedly for having been obtained in
violation of Rule 24, Administrative Order No. 1, series of 1993.

Note that Rule 24, Administrative Order No. 1, series of 1993 only provides for
sanctions against persons violating the rule on confidentiality of birth records, but
nowhere does it state that procurement of birth records in violation of said rule
would render said records inadmissible in evidence. On the other hand, the Revised
Rules of Evidence only provides for the exclusion of evidence if it is obtained as a
result of illegal searches and seizures. It should be emphasized, however, that said
rule against unreasonable searches and seizures is meant only to protect a person
from interference by the government or the state. 15 In People vs. Hipol,16 we
explained that:

The Constitutional proscription enshrined in the Bill of Rights does not


concern itself with the relation between a private individual and another
individual. It governs the relationship between the individual and the State
and its agents. The Bill of Rights only tempers governmental power and
protects the individual against any aggression and unwarranted interference
by any department of government and its agencies. Accordingly, it cannot
be extended to the acts complained of in this case. The alleged "warrantless
search" made by Roque, a co-employee of appellant at the treasurer’s office,
can hardly fall within the ambit of the constitutional proscription on
unwarranted searches and seizures.

Consequently, in this case where complainants, as private individuals, obtained the


subject birth records as evidence against respondent, the protection against
unreasonable searches and seizures does not apply.

Since both Rule 24, Administrative Order No. 1, series of 1993 and the Revised
Rules on Evidence do not provide for the exclusion from evidence of the birth
certificates in question, said public documents are, therefore, admissible and
should be properly taken into consideration in the resolution of this administrative
case against respondent.

Verily, the facts stated in the birth certificates of Mara Khrisna Charmina dela
Fuente Mendoza and Myrra Khrisna Normina dela Fuente Mendoza and
respondent’s Certificate of Candidacy dated March 9, 1995 wherein respondent
himself declared he was married to Felicitas Valderia, were never denied nor
rebutted by respondent. Hence, said public documents sufficiently prove that he
fathered two children by Marilyn dela Fuente despite the fact that he was still
legally married to Felicitas Valderia at that time.

In Bar Matter No. 1154,17 good moral character was defined thus:

. . . good moral character is what a person really is, as distinguished from


good reputation or from the opinion generally entertained of him, the
estimate in which he is held by the public in the place where he is known.
Moral character is not a subjective term but one which corresponds to
objective reality. The standard of personal and professional integrity is not
satisfied by such conduct as it merely enables a person to escape the penalty
of criminal law.

In Zaguirre vs. Castillo,18 we reiterated the definition of immoral conduct, to wit:

. . . that conduct which is so willful, flagrant, or shameless as to show


indifference to the opinion of good and respectable members of the
community. Furthermore, such conduct must not only be immoral, but
grossly immoral. That is, it must be so corrupt as to constitute a criminal act
or so unprincipled as to be reprehensible to a high degree or committed
under such scandalous or revolting circumstances as to shock the common
sense of decency.

In the above-quoted case, we pointed out that a member of the Bar and officer of
the court is not only required to refrain from adulterous relationships or the
keeping of mistresses but must also behave himself as to avoid scandalizing the
public by creating the belief that he is flouting those moral standards and, thus,
ruled that siring a child with a woman other than his wife is a conduct way below
the standards of morality required of every lawyer. 19

We must rule in the same wise in this case before us. The fact that respondent
continues to publicly and openly cohabit with a woman who is not his legal wife,
thus, siring children by her, shows his lack of good moral character. Respondent
should keep in mind that the requirement of good moral character is not only a
condition precedent to admission to the Philippine Bar but is also a continuing
requirement to maintain one’s good standing in the legal profession. 20 In Aldovino
vs. Pujalte, Jr.,21 we emphasized that:

This Court has been exacting in its demand for integrity and good moral
character of members of the Bar. They are expected at all times to uphold
the integrity and dignity of the legal profession and refrain from any act or
omission which might lessen the trust and confidence reposed by the public
in the fidelity, honesty, and integrity of the legal profession. Membership in
the legal profession is a privilege. And whenever it is made to appear that an
attorney is no longer worthy of the trust and confidence of the public, it
becomes not only the right but also the duty of this Court, which made him
one of its officers and gave him the privilege of ministering within its Bar,
to withdraw the privilege.

WHEREFORE, respondent Atty. Norberto M. Mendoza is hereby


found GUILTY of immorality, in violation of Rule 1.01 of the Code of
Professional Responsibility. He is SUSPENDED INDEFINITELY from the
practice of law until he submits satisfactory proof that he has abandoned his
immoral course of conduct.

Let a copy of this resolution be served personally on respondent at his last known
address and entered in his record as attorney. Let the IBP, the Bar Confidant, and
the Court Administrator be furnished also a copy of this resolution for their
information and guidance as well as for circularization to all courts in the country.

SO ORDERED.

You might also like